Location via proxy:   [ UP ]  
[Report a bug]   [Manage cookies]                

Final Internalmed2

Download as pdf or txt
Download as pdf or txt
You are on page 1of 1788

Hyperglycaemia:-

Obesity , dry mouth , thirst , frequent urination , weight loss , skin dry , apical impulse shift left down
, heart boundary expand left , accent of 2nd sounds l over aorta , blood pressure slightly increased...

Cholesterol increased

GFR decreased

Hypoglycemia:-
Feeling of hunger , dizziness , trembling of extremities , skin wet , muscle tone increased,

Pulse rapid or slow both ,

Glucosuria, no acetone

Acute Hyperglycaemia:-
Same as Hyperglycaemia but...

Polyuria , epigastric pain , dyspeptic symptoms , smell of acetone in exhaled air , eye ball soft

Hyperthyroidism :-
Palpitations , difficulty in swallowing , sweating , weight loss , irritability , glazed eyes , frequent
stools

Graefe , kocher , mobius , pretibial myxedema

Skin moist

TSH decreased

T3 T4 increased, sinus tachycardia , atrial fibrillation

If "leukocytes" then acute inflammation of thyroid gland...

If "lymphocytes" answer will be sub-acute inflammation

Acute inflammation's pathogens - microbial

Sub-acute inflammation's pathogens – viral

cushingoid or Cushing syndrome in question....then answer will be hypercorticism


Internal Medicine (Endocrine System)

1. Factors predisposing to the development of hyperglycemia syndrome:


A. obesity, physical inactivity
B. excess in the diet of coarse fiber
C. excessive physical activity
E. environmental influences
D. hypоthyroidism
2. A 45-year-old patient was admitted to the clinic with complaints of dry mouth,
thirst, frequent urination, itching of the genitals. The above complaints began to
bother about 3 years ago, had not been treated anywhere before, there is a child,
weight 4.8 kg. Objective data: weight 88 kg, height 156 cm, dry skin. Vesicular
breathing above the lungs, rhythmic heart sounds, with an emphasis on 2 sounds
above the aorta. The abdomen is soft, painless. Laboratory data: fasting blood
glucose level of 15.1 mmol / l, 2 hours after eating - 19.3 mmol / l, cholesterol - 7.4
mmol / l. Your preliminary diagnosis?
A. hyperglycemia
B. hypercortisolism
C. hyperthyroidism
D. hyperparathyroidism
E. hyperglobinemia

3. A 16-year-old patient complains of intense thirst (the volume of liquid drunk up


to 6 liters per day), dry mouth, increased urine output, weight loss by 6 kg in 2
months, severe weakness. From the anamnesis: these complaints began to bother
after suffering a severe flu. Diabetes mellitus heredity is not burdened. Objective
data: height 167 cm, weight 50 kg, dry skin, facial rubeosis. Above the lungs,
vesicular breathing, heart sounds are clean, rhythmic, heart rate-89 beats 1 min,
blood pressure 120/75 mm Hg. The abdomen is soft, painless, the liver is not
enlarged. What analysis should be done first of all in these cases?
A. total cholesterol
B. liver test
C. kidney test
D. glucose level
E. urea

4. Causes of hypoglycemia
A. fatty food
B. excessive exercise
C. stress
D. excess in the diet of coarse fiber

5. An 18-year-old patient was admitted to the hospital unconscious. It is known


from the anamnesis that he has been suffering from diabetes mellitus for 5 years
and receives insulin. According to relatives, in the morning the patient
independently increased the dose of this insulin, explaining this by the fact that it
was an increased thirst and dry mouth recently. In the morning he felt good, and
after 3 hours he was found unconscious by his relatives.
Objective data: there is no consciousness, the skin is moist, the muscle tone is
increased. Vesicular breathing, rhythmic heart sounds, blood pressure 118/75 mm
Hg, pulse 80 beats per minute. The abdomen is painless on palpation. What help is
needed in this case?
A. in / in the jet introduction of 40% glucose
B. iv administration of glucagon + insulin
C. s / c administration of short-acting insulin
D. s / c injection of ultrashort action insulin

6 Rare blinking in hyperthyroidism syndrome, this is a symptom:


A. Kocher
B. Shtelvaga
C. Moebius
D. Krause

7. Patient K., 23 years old, complains of irritability, sharp weight loss, rapid mood
swings, tearfulness, palpitations throughout the year.
Objectively: height 170 cm, body weight 59 kg. the skin is clean, moist, hot to the
touch. The thyroid gland is enlarged. Pulse 118 in I min. BP - 160/60 mm Hg. Art.,
Heart sounds are quite loud, arrhythmic. Tests: cholesterol - 2.8 mmol / l.
Laboratory: TSH is reduced, T3, T4 is increased. Diagnosed with hyperfunction of
the thyroid gland, taking into account tachycardia, which examination method
should be recommended in this case?

A. CT scan
B. Ultrasound
C. ECG
D. X-ray
E. MRI

8. Patient K., 30 years old, complains of severe weakness, headache, chilliness,


apathy, loss of appetite, constipation.Objectively: height 180 cm, body weight 89
kg. Adynamic, the skin is pale, cold. The thyroid gland is not palpable. Pulse 59 in
I min. BP- 100/60 mm Hg. heart sounds are weakened. The rest of the data are
without deviations from the norm.Tests: cholesterol 8.8 mmol / l. TSH increased,
T3, T4 decreased. What is your preliminary diagnosis?

A. hypoglycemia
B. hypothyroidism
C. hypoparathyroidism
D. hypopituitarism
E. Hypocorticism

9. Patient A., 46 years old. Complaints: the appearance of edema on the face and
lower extremities, lethargy, weakness, increased fatigue, hearing loss, baldness.
The patient lives in an endemic area. Since childhood, an enlarged thyroid gland. A
year ago, the size of the thyroid gland increased, weakness began to progress, and
edema appeared. Objectively: the patient is lethargic, answers questions slowly,
speech is recognized. The skin of the hands is thick, rough, cold, pigmented, does
not collect folds. Respiratory organs without pathology. The heart is enlarged on
the left. Heart sounds are muffled, rhythmic, pulse 54 beats per minute, blood
pressure 100/60 mm Hg. The tongue is thickened with dental facets. What analysis
is necessary in this case, given the patient's complaints?

A. TTG, T3, T4
B. cortisol
C. ACTH
D. prolactin
E. FSH, LH

10. Patient K.A., 50 years old, complained of weight gain, constipation,


drowsiness, swelling of the face and body, dry skin, hair loss. From the anamnesis:
these complaints have been disturbing for 1 year. Was not examined, treatment
was not treated. According to the patient, she previously suffered from
autoimmune thyroiditis.On physical examination: the food is moderate, the skin is
pale, dry to the touch, there is continuous swelling on the lower limbs. Rhythmic
heart sounds, weakened, pulse 60 beats per minute, blood pressure 120/80 mm Hg.
The tongue is thickened, with imprints of teeth. On palpation, the thyroid gland is
not enlarged, the consistency is dense, painless. Laboratory: TSH increased, T3, T4
decreased. The patient is diagnosed with hypofunction of the thyroid gland, taking
into account the symptoms, which analysis should also be performed

A. CT scan
B. Ultrasound
C. Аngiography
D. X-ray
E. MRI.

11. A 40-year-old patient fell ill acutely: chills, weakness, difficulty swallowing,
pain in the neck with irradiation to the ears, head, aggravated when turning,
temperature 38 ° C. The patient is worried about sweating. On palpation, the
thyroid gland is enlarged, painful, not soldered to the surrounding tissues,
compacted, there are no fluctuating areas. Submandibular, cervical lymph nodes
are not enlarged. In the analysis of blood: leukocytosis, ESR up to 21 mm / h.
What is your preliminary syndrome?

A. syndrome of acute inflammation of the thyroid gland

B. syndrome of chronic inflammation of the thyroid gland

C. syndrome of hypothyroidism

D. thyroid nodule syndrome

E. syndrome of hypofunction of the parathyroid glands

12. Patient A. 40-year-old applied to a polyclinic with complaints of an increase in


body temperature up to 39˚C, pain in the thyroid gland, aggravated by swallowing,
coughing, chills, tachycardia, weakness. Anamnesis: frequent tonsillitis.
Objectively: On palpation, the thyroid gland is sharply painful and dense.
Laboratory: Complete blood count: leukocytosis with a shift to the left, increased
ESR. Choose the most informative thyroid examination method in this situation:

A. determination of T3, T4 in blood

B. thyroid scan with 131 I

C. ultrasound with biopsy

D. lateral radiography of the neck.

E. auscultation of the thyroid gland


13. The main symptoms of thyroid inflammation syndrome:

A. increased body temperature


B. Thyroid gland is painless
C. Lowering blood pressure
D. lymph nodes are not enlarged
E. hoarseness, constipation
14. Subacute thyroiditis is a consequence of:
A. viral infection
B. Bacterial infection
C. Fungal infection
D. traumatic injury
E. radiation injury
15. Patient A., 40 years old, went to the clinic with complaints of pain in the bones,
especially in the feet, long-term healing fractures, decreased appetite, nausea,
vomiting, not associated with food intake, diarrhea.
A biochemical blood test revealed: hypercalcemia, hypophosphatemia. X-ray
examination revealed osteoporosis of the bones. What is your preliminary
syndrome?
A. thyroid inflammation syndrome

B. hyperthyroidism syndrome

C. hypercortisol syndrome

D. hyperparathyroidism syndrome

E. hyperglycemic syndrome

16. Which symptom is characterized by, if the patients lower limb is held at the
knee joint and flexed strongly at the hip joint there will soon be an extensor spasm
at the knee joint with extreme supination of the foot.
A. Trousseau̕ s sign
B. Chvostek ̕ s sign
C. Schlesinger̕ s sign
D. Hoffmann̕ s sign
E. Weiss sign

17. Patient A., 32 years old, after a traumatic injury to the neck, notes the periodic
appearance of paresthesias (pathological discomfort, for example, "creeping") with
the subsequent development of seizures. Before the development of seizures, he
has nausea, vomiting. Convulsions are tonic in nature, beginning in the upper
limbs and face. Consciousness during this period is preserved. Positive symptoms
of Trousseau and Chvostek. Which laboratory changes are typical for this
syndrome?
A. hypophtoremia

B. hypocalcemia

C. hypophosphatemia

D. hypokalemia

E. hyponatremia

18. Causes of secondary hyperparathyroidism:


A. hyperplasia of the parathyroid glands
B. chronic renal failure
C. Lack of calcium in food
D. removal of the thyroid gland
E. pituitary adenoma

19. Patient V., 65 years old, consulted a doctor at the place of residence with
complaints of increased body weight, pain in the lumbar region, increased blood
pressure up to 200/100 mm Hg. Objectively: height - 165 cm, body weight - 121
kg. Obesity with redistribution of subcutaneous fat according to the "cushingoid"
type, redness of the cheeks, striae of red-violet color in the hips and abdomen,
multiple acne on the face and back, hypotrophy of the limbs. Fasting blood glucose
- 15 mmol / l, urine sugar 6%. What hormonal research method will help in making
a diagnosis?
A. cortisol
B. insulin
C. thyroxine
D. vasopressin
E. testosterone

20. Patient,67 years old. At the endocrinologist's appointment for the first time.
Complaints of increased fatigue, overweight. From the anamnesis: the patient's
mother suffered from type 2 diabetes mellitus, obesity. Objectively: height 173 cm,
body weight 102 kg. The type of distribution of adipose tissue is abdominal
obesity. The skin is normal in color, dry to the touch. Blood pressure -130/80 mm
Hg, heart rate - 82 per minute. Laboratory results: fasting glycemia 11.5 mmol / l.
Calculate body mass index (kg / m2).
A. 24,6
B. 34.1
C. 38.7
D. 42.0
E. 44.3

21.Patient E., 42 years old, at a doctor's appointment complains of muscle


weakness, rapid physical fatigue, lack of appetite, addiction to salty foods,
menstrual irregularities. She had a history of pulmonary tuberculosis 3 years ago.
On examination, reduced nutrition, weight - 50 kg, height - 165 cm. The skin has a
bronze color, dark spots on the mucous membrane of the oral cavity, subcutaneous
fatty tissue is thinned. BP - 80/50 mm Hg. Laboratory data: blood glucose - 3.2
mmol / l, blood cortisol - 28.9 nmol / l. What the main physical symptom of this
syndrome has patient ?
A. hyperpigmentation
B. hirsutism
C. prognatisn
D. striae
E. xanthelasma
Pathophysiology

Module "Endocrine system" Option 2

1. With an increase in ACTH production, synthesis is primarily increased:


A. cortisol
B. aldosterone
S. estrogen
D. androgen
2. Which of the following factors leads to the development of acromegaly:
A. increased formation of reactivity of IGF 1
B. STH producing tumors outside the pituitary origin
C. hypothalamic tumors producing GH
D. all of the above
3. Pan-hypopituitarism does not affect the function of:
A. Thyroid
B. adrenal cortex
C. gonads
D. parathyroid glands
4. Specify the statements typical for hypoproduction of vasopressin (ADH):
A. characterized by hypernatremic hyperhydration
AT. . characterized by hyponatremic hyperhydration
S. is characterized by the development of dehydration
D. characterized by an increase in urine density (specific gravity)
5. In the pathogenesis of an increase in the size of the thyroid gland with endemic goiter,
it matters:
A. increase TSH production by feedback mechanism
B. decrease TTG production by feedback mechanism
C. increase production of T3, T4
D. increase IGF 1 (insulin-like growth factor)
6. Autoimmune damage to the thyroid gland leads to the development of:
A. hypothyroidism
B. hyperthyroidism
C. endemic goiter
D. diffuse toxic goiter
7. The pathogenesis of secondary hypothyroidism is associated with:
A. decreased secretion of thyroliberin
B. Thyroid Immunity to TSH
C. increased secretion of TSH
D. Decreased TSH secretion
8. Indicate the main reason for the development of diabetes in the disease or Cushing's
syndrome:
A. stimulation of lipolysis
B. activation of gluconeogenesis
C. activation of glycogenolysis
D. activation of proteolysis
9. For idiopathic primary hyperaldosteronism, all statements are true, except:
A. Basal aldosterone levels are elevated, reninaplasma activity is seen
B. Hyperplasia of the glomerular zone of the adrenal cortex
C. Hyperplasia of the adrenal cortex
D. With orthostatic load (march test), an increase in aldosterone levels is noted
10. Pheochromocytoma is a tumor of chromafin tissue producing:
A. Androgens
B. Aldosterone
C. Glucocorticoids
D. Adrenaline
11. Indicate the hallmark of the manifestation of Cushing's syndrome due to
corticosteroma:
A. Unilateral adrenal enlargement
B. Bilateral adrenal enlargement
C. Basophilic pituitary adenoma
D. Increased ACTH

12. hypothyroidism (myxedema) is characterized by the following changes:


A. Increased sensitivity to high temperatures
B. Decrease in sensitivity to high temperatures
C. Increase in glycosaminoglycans in tissues
D. Decrease in glycosaminoglycans in tissues
13. Indicate the role of antibodies (Ig G) in the mechanism of development of diffuse toxic
trunk (bazedova disease):
A. Stimulate thyroid receptors
B. Inhibit thyroid receptors
C. Block thyroid receptors
D. Destroy thyroid receptors

14. Select characteristic changes in the cardiovascular system with hyperthyroidism


(diffuse toxic goiter):
A. Increased diastolic pressure
B. Decrease in systolic pressure
C. Hypertrophy of the heart muscle
D. Atrial fibrillation

15. Diabetes insipidus develops in defeat:


A. Anterior pituitary gland
B. The posterior pituitary gland
C. Pancreas
D. Adrenal glands
Module "Endocrine system" Option 1

1. The development of panhypopituitarism in adults Does not include:


A. Ischemic damage to the pituitary gland
B. hypophysectomy
C. severe physical exertion
D. Radiation therapy of the pituitary gland
2. Indicate the characteristic metabolic changes with excess STH:
A. negative nitrogen balance
B. positive nitrogen balance
C. development of hypoglycomia
D. decrease in basal metabolism
3. Basophilic pituitary adenoma causes the development of:
A. gigantism
B. hypercortisolism
C. hyperthyroidism
D. acromegaly
4. The development of panhypopituitarism in the experiment is associated with the
removal of:
A. Thyroid
B. adrenal glands
C. pituitary gland
D. pancreas
5. The excessive action of thyroid hormones in the body causes:
A. Strengthening the processes of protein synthesis and inhibition of catabolism
B. increased catabolism
C. enhanced TTG production
D. increases insulin activity
6. Thyrotoxicosis is characterized by:
A. increased metabolism
B. increased heat production
C. increased catecholamine activity
D. All of the above is true.
7. Choose the mechanisms of development of endocrine ophthalmopathy:
A. autoimmune inflammation
B. Infectious edema of retrobulbar fiber
C. hypertrophy of retrobulbar fiber
D. increase in intraocular pressure
8. Hypothyroidism is characterized by all of the following clinical and laboratory
manifestations, with the exception of:
A. swelling of the face and limbs
B. Bradycardia and hypotension
C. weight gain
D. lower cholesterol
9. For hyperthyroidism, all of these symptoms are characteristic except for:
A. tachycardia
B. weight gain
C. hand tremor
D. exophthalmos
10. Choose the least typical symptom characteristic of primary adrenal insufficiency:
A. hyperpigmentation of the mucous membranes and skin
B. asthenic syndrome
C. hyperglycemia
D. arterial
11. Indicate the most likely cause of acute adrenal insufficiency:
A. Adrenal gland adenoma
B. autoimmune destruction of the adrenal cortex
C. withdrawal of glucocorticoid therapy after prolonged use
D. Pituitary hemorrhage
12. With pituitary nanism, the following changes are observed:
A. proportional reduction in the size of all parts of the body
B. Inhibition of mental development
C. Decrease limb size with normal body size
D. all answers are incorrect
13. Primary hypocorticism occurs in associations:
A. Type 1 diabetes
B. autoimmune thyroid disease
C. hypogonadism
D. hypoparathyroidism
14. Hyperplasia of the adrenal cortex leads to:
A. virilization
B. hyperglycemia
C. arterial hypertension
D. all answers are correct
15. What is the sign of primary hypocorticism will not occur with secondary:
A. body weight reduction
B. hyperpigmentation of the skin and mucous membranes
C. General weakness of asthenia
D. hyponatremia
Pediatric

The Training Tests Module “Endocrine System”


For students of 3d course ЛДАЯО

1. Select, from the listed which is the main thyroid hormone?


A. thriiodthyronine
B. thyroxine
C. thyrocalcitonin
D. thyroglobulin
E. TSH

2. What is the main effect of the thyroid gland on the growing body of the child?
A. immunoglobulin content
B. development of the muscles
C. mental development
D. Fformation of immunity
E. motor function

3. Indicate the blood glucose level in healthy children (mmol/l


A. 1,55-2,33
B. 3,33-5,55
C. 6,65-7,55
D. 7,99-8,88
E. 9,22-9,99

4. Describe the stool for hypothyroidism in early childhood:


A. form as a pea puree
B. persistant constipation
C. frequent with blood
D. form of rice water
E. frequent, frothy with mucus

5. Explain the cause of overweight in hypothyroidism:


A. hypoproteinemia
B. mixedema edema
C. violation of oxydative processes
D. hypernatriemia
E. Hyperlipidemia

6. What disorders lead to the development of hyperglycemia syndrome?


A. liver function
B. intestinal digestion
C. deficiency of insulin secretion
D. assimilation of fats
E. absorption of fats
7. Explain, why accelerated growth and ossification occurs with hyperthyroidism
syndrome?
A. high levels of thyrocalcitonin
B. hypercalcemia
C. development of bone epiphyses
D. high growth hormone levels
E. high thyroglobulin levels

8. Baby, 4 months with mother turned to the family doctor with complaints on
constipations, overweight, drowsiness, difficulty swallowing. Anamnesis: 2-nd
pregnancy with toxicosis, 2nd delivery, urgent, sucked sluggishly. The umbilical
residue disappeared on the 14th days. Jaundice was noted up to 1,5 months.
Objectively: weight-8000,0. The face is edematous, the tongue is large, sticking out.
The skin is pale with yellowness, dry. Heart tones very weak, bradycardia. Pulse rate
is 100 in 1 minute. What is the first diagnostic test to use?
A. bilirubin, cholesterol
B. T4, TSH
C. chest X-ray
D. calcium, phosphorus
E. common analysis of blood

9. At what level of glucose hypoglycemia syndrome develops (below…mmol/l)?


A. 7,9
B. 6,9
C. 5,9
D. 4,9
E. 3,9
Radiology
Exam questions for students of the Faculty of Foreign Citizens according to the module
"Endocrine System"

1.When conducting a test with triiodothyronine, toxic goiter is characterized by:

A) the absence of inhibition of the absorption of radioactive iodine by the thyroid gland

B) increasing the absorption of radioactive iodine by the thyroid gland by 30%

C) a decrease in the absorption of radioactive iodine by the thyroid gland (by 50%)

D) increasing the absorption of radioactive iodine by the thyroid gland by 50% and>

E) inhibition of the absorption of radioactive iodine by the thyroid gland (by 60% and>)

2.The mechanism of action of radioactive iodine in diffuse toxic goiter is due to:

A) exposure to cells of the follicular epithelium with their replacement with connective tissue

B) impact on the autoimmune process in the thyroid gland

C) blocking the flow of iodine into the thyroid gland

D) inhibition of the conversion of thyroxine into triiodothyronine

E) blocking TSH

3.Preparation of patients with thyrotoxicosis for radioiodine therapy includes:

A) reaching the euthyroid state before the appointment of radioactive iodine

B) treatment against the background of thyrotoxicosis

C) treatment against the background of reaching hypothyroidism

D) treatment against the background of beta-blockers

E) treatment with radioactive iodine for any function of the thyroid gland

4. A 38-year-old patient came to the clinic for thyroid gland removal. The removal operation
was successful, but then there were convulsions. Identify what went wrong, what was damaged?

A. vagus nerve

B. lower laryngeal nerve


C. sympathetic trunk

D. parathyroid gland

E. inferior laryngeal artery

5. Name which diagnostic method is more applicable for the diagnosis of the hypothalamic-
pituitary system:

A. Magnetic resonance imaging

B. Positron Emission Tomography

C. Craniography

D. Computed tomography

E. Endoscopy

6. On computed tomography, the pancreas is located centrally, the contours are even, clear, the
structure is homogeneous, focal changes are not observed. Determine the condition of the
pancreas:

A. endothelial cancer

B. metastatic tumor

C. normal pancreas

D. chronic pancreatitis

E. cystadenoma in the stage of resorption

7. young woman M., 28 years old, smoking 8 years old, found in the right

the mammary gland is a nodular formation up to 2 cm in diameter. She turned to a


mammologist, who confirmed her education.

Predict which studies a patient should be referred to in order to obtain reliable information?

A. multispiral computed tomography


B. mammography

C. magnetic resonance imaging

D. chest x-ray

E. echocardiography

8.Scintigraphy for autoimmune thyroiditis is characterized by:

A) "cold" node

B) "hot" node

C) no absorption of the radiopharmaceutical

D) uneven absorption of the radiopharmaceutical

E) increased absorption of the radiopharmaceutical

9.In a pregnant woman with an increased level of total T3 and total T4, first of all, it is necessary
to carry out:

A) definition of TSH

B) determination of free T3 and T4

C) ultrasound of the thyroid gland

D) radioisotope scintigraphy of the thyroid gland

E) study of intrathyroid iodine

10. In a 36-year-old woman, an ultrasound of the right adrenal gland revealed a homogeneous
hypoechoic structure, a well-limited solid formation of a rounded shape, 1.9 cm in size.

Complaints about weight gain with the deposition of adipose tissue in the neck, chest, abdomen;
moon face; also complains of general weakness, memory loss, depression, bone pain. What
research method can be prescribed additionally to clarify the diagnosis:

A. radiography

B. endoscopy
C. computed tomography with contrasting

D. Magnetic resonance imaging

E. scintigraphy
Previous year Internal Medicine

"Endocrine System"

1) What is the level of blood glucose corresponds to the hypoglycemia


syndrome in venous plasma:

A. 2.8 mmol / L
B. 4.0 mmol / L
C. 4.7 mmol / L
D. 5.6 mmol / L
E. 7.0 mmol \ L

2) Which of the following corresponds to impaired glucose tolerance (venous


plasma):

A. Fasting glucose - 7.4 mmol / l, 2 hours after OGTT - 12.2 mmol / l


B. Fasting glucose - 6.8 mmol / l, 2 hours after OGTT - 10.9 mmol / l
C. Fasting glucose- 6.2 mmol / l, 2 hours after OGTT - 7.7 mmol / l
D. Fasting glucose- 5.6 mmol / l, 2 hours after OGTT - 7.4 mmol / l
E. Fasting glucose- 4.0 mmol / l, 2 hours after OGTT - 6.8 mmol / l

3) List the triad of symptoms in hyperglycemia syndrome:

A. Polydipsia, polyphagy, polyuria


B. Hypertension, hyperpigmentation, obesity
C. Hypotension, anorexia, anuria
D. Exophthalmos, hypertension, hyperthermia
E. Alopecia, bradycardia, hypercholesterolemia

4) Complaints of patients with hypoglycemia syndrome may be:


A. Epigastric pain
B. Slow wound healing
C. Burning in the lower extremities
D. Pustular skin lesion
E. Tremor in the body and limbs

5) List the main risk factors for the development of hyperglycemia syndrome:
A. Heredity, obesity, lack of physical activity
B. Long-term stress, puberty, smoking
C. Age, Gender, Ethnicity
D. viral infections, radiation, injuries
E. Regular nutrition, pregnancy, alcoholism

6) Which of the following is the cause of hypoglycemic syndrome:


A. Hypercorticism syndrome
B. Overweight
C. Hypocorticism syndrome
D. Thyroid inflammation syndrome
E. Hyperthyroidism syndrome

7) Which of the following is the cause of hyperglycemic syndrome:


A. Hypercorticism syndrome
B. Hypothyroidism syndrome
C. Hypocorticism syndrome
D. Hypoparathyroidism syndrome
E. Anorexia Syndrome

8) The glucose level in the plasma of venous blood is normal:


A. Fasting 3.3-5.5 mmol / l, 2 hours after meals> 7.8 mmol / l
B. Fasting 5.6-6.1 mmol / l, 2 hours after meals 6.1-7.0 mmol / l
C. Fasting 5.6-6.1 mmol / l, 2 hours after meals<6.7 mmol / l
D. Fasting 4-6.1 mmol / l, 2 hours after a meal <7.8 mmol / l
E. Fasting 4.6-5.1 mmol / l, 2 hours after meals <8.5 mmol / l

9) The appearance of edema in hypercorticism syndrome is associated with:


A. Hypermagnesemia
B. Hypernatremia
C. Hypokalemia
D. Hypomagnesemia
E. Hyponatremia

10) An oral glucose tolerance test is performed:


A. On the background of acute illness
B. When glycogemoglobin is more than 6.5%.
C. In case of doubtful glycemia for clarification
the diagnosis
D. When blood glucose is greater than 11.0 mmol / L.
E. When fasting blood glucose is less than 3.3 mmol / L.

11) When viewed in the oral cavity with hyperglycemia syndrome can be
identified:

A. Loosening and early tееth loss


B. Geographic language
C. Tongue is covered with yellow bloom
D. Black spots on tongue and gums
E. Varnished language

12) Rare blinking in hyperthyroidism syndrome, this is a symptom:


A. Kocher
B. Shtelvag
C. Moebius
D. Krause
E. Elinek

13) The appearance of myxedema on the limbs of a patient with


hypothyroidism syndrome is due to a violation of:
A. Basic exchange
B. Carbohydrate Exchange
C. Lipid metabolism
D. Protein metabolism
E. Water-salt metabolism
14) A man, 36 years old, went to the clinic to the local doctor with complaints
of pain and swelling in the chest area from 2 sides. On examination: increased
nutrition, uneven distribution of the subcutaneous fat layer with a
predominant deposition in the upper body and on the face, on the front
surface of the abdomen and on the inner thighs of the strip of red-purple
color, palpation in the chest reveals dense painful formations with a diameter
of up to 5, blood pressure - 160/100 mm Hg
Highlight the main syndrome:
A. Thyroid hyperfunctions
B. Hypofunction of the thyroid gland
C. Hyperfunction of the adrenal cortex
D. Hypofunction of the adrenal cortex
E. Hyperfunctions of the parathyroid glands

15) On examination of the skin in a patient with hyperglycemia syndrome, it is


determined:

A. Hyperpigmentation
B. Humidity
C. Rubeosis of the cheeks
D. Cyanotic
E. Facial hair growth in women

16) For hyperfunction of the thyroid gland is characteristic:

A. Sleepiness
B. Bradycardia
C. Puffiness of the face and narrowing of the palpebral fissures
D. Hyperexcitability
E. Obesity

17) With acute hypoglycemia syndrome:


A. Skin is wet, motor agitation
B. Breathing is noisy and rare
C. Skin is dry
D. Reduced turgor
E. Thirst

18) Gynecomastia in patients with hypercorticism syndrome occurs due to


metabolic changes:
A. Glucocorticoids
B. Thyroid hormones
C. Sex hormones
D. Mineralocorticoids
E. Pituitary hormones

19) Symptoms of hypothyroidism include:

A. Diarrhea
B. Sweating
C. Constipation
D. Cyanosis
E. Heartbeat

20) Laboratory signs of hypothyroidism syndrome:

A. Decrease in T3 and T4 in the blood


B. Increase in T3 and T4 in the blood
C. Elevation of TSH and T4 in the blood
D. Decrease in the level of TSH in the blood
E. Increased T3 and T4, decreased TSH

21) Hypocorticism syndrome is characterized by:

A. Hyperpigmentation of the skin


B. Weight gain
C. Increased blood pressure
D. Tachycardia
E. Polyphagia

22) The volume of the waist in men normally should not exceed?
A. Not more than 94 cm
B. Not more than 106 cm
C. Not more than 86 cm
D. Less than 60 cm
E. Morethan 102 cm

23) How to calculate BMI (body mass index)?


A. Height / weight2
B. Weight / Height2
C. Height2/ weight
D. Weight2 / Height
E. Weight / Height

24) In which units is body mass index measured?


A. m2 / kg
B. m / kg2
C. kg / m2
D. kg2 / m
E. kg / m

25) Waist size in women is normal?

A. Not more than 65 cm


B. Not more than 70 cm
C. Not more than 80 cm
D. Less than 60 cm
E. More than 88 cm

26) The patient came to the endocrinologist. When calculating the body mass
index was 29.8 kg / m2. Determine the degree of impaired fat metabolism.

A. Underweight
B. Overweight
C. Obesity I degree
D. Obesity II degree
E. Obesity III degree

27) With the introduction of 1 mg of dexamethasone at 12 o'clock at night, a


decrease in cortisol by 50% or more the next morning indicates:

A. Hypocorticism
B. About the normal function of the adrenal cortex
C. About hypercorticism
D. About secondary hypocorticism
E. On increased secretion of ACTH

28) The following changes on an ECG are characteristic of a syndrome of a


hypofunction of a thyroid gland:
A. reduction of teeth voltage, bradycardia
B. tachycardia, ST segment reduction
C. atrial fibrillation
D. Pathological Q wave
E. AV blockade

29) Goiter is:

A. well palpable thyroid


B. thyroid disease, occurring in violation of its function
C. thyroid disease, complicated by tracheal compression
D. thyroid enlargement more than 18 ml in women and more than 25 ml in
men
E. An increase in volume of more than 16 ml in women and more than 18 ml in
men.

30) Laboratory criteria for hypoparathyroidism syndrome:

A. increased blood potassium levels


B. increased blood levels of parathyroid hormone
C. lower blood calcium levels
D. lower blood sodium levels
E. Increased blood glucose

31) With an enlargement of the thyroid gland of I degree (WHO):

A. The thyroid gland is invisible, palpable


B. The thyroid gland is not visible to the eye, is not palpable.
C. Thyroid visible on the eye, palpable
D. The thyroid gland is visible to the eye, not palpable.
E. Thyroid lobe smaller than the distal phalanx of the thumb studied

32) At the reception, patient Z., 36 years old. Height - 180 cm, weight - 95 kg.
Give an opinion on the violation of fat metabolism:

A. Deficiency of body weight


B. Overweight
C. Obesity I degree
D. Obesity II degree
E. Obesity III degree

33) A young man of 18 years old after a cold has thirst, polyuria, weakness.
Blood sugar level is 16mmol / l. What is the patient's syndrome?

A. hypoglycemia syndrome
B. hyperglycemia syndrome
C. thyroid hypofunction syndrome
D. Hypercortisolism syndrome
E. Thyroidhyperfunctionsyndrome

34) Palpation in adults in the normal thyroid gland:

A. soft, agile
B. Dense, welded to underlying tissues.
C. moderate soft, not mobile
D. soft, motionless
E. tight, painful

35) WHO classification of goiter:

A. 0,1,2,3,4 degrees
B. 0,1,2,3 degrees
C. 0,1,2 degrees
D. 1,2,3 degrees
E. 1,2,3,4,5 degrees

36) What changes in the content of thyroid-stimulating hormone,


triiodothyronine and thyroxin are characteristic of hypothyroidism
syndrome?

A. A decrease in serum thyroid-stimulating hormone (TSH) and an increase in


thyroxin (T4) and triiodothyronine (T3) levels
B. Normal levels of TSH and increased serum T3 and T4 levels
C. Elevated levels of TSH and a decrease in serum T3 and T4
D. Normal serum TSH, T3 and T4
E. Elevated serum TSH, T3 and T4
37) Indicate the characteristic electrolyte disturbances in hypocorticism
syndrome:

A. Hypochloremia, hypermagnesemia
B. Hypomagnesaemia, hyperchloremia
C. Hypochloremia, hypomagnesemia
D. Hypernatremia, hypokalemia
E. Hyperkalemia, hyponatremia

38) Atrial fibrillation can occur when:

A. hyperglycemia syndrome
B. hypoglycemia syndrome
C. hyperthyroidism syndrome
D. hypothyroidism syndrome
E. hypocorticism syndrome

39) Features of edema in thyroid hypofunction syndrome:

A. dense, do not leave a fossa


B. leaves a hole for palpation
C. pronounced in the evening on the lower limbs
D. expressed under the morning under the eyes
E. mild, painful

40) General weakness, fatigue, severe muscle weakness, darkening of the skin,
weight loss, loss of appetite, hypotension - these complaints are characteristic
of the syndrome:

A. Hypothyroidism
B. Hyperthyroidism
C. Hypocorticoidism
D. Hypercortisolism
E. Hypopituitarism

41) What syndrome is characterized by "matronism", "bull neck", "marble


skin pattern":
A. hyperglycemia
B. hyperthyroidism
C. Hypocorticoidism
D. hypercortisolism
E. hypothyroidism

42) Pigmentation of the eyelids (upper) with thyroid hyperfunction syndrome


is a symptom:
A. Kocher
B. Moebius
C. Krause
D. Elinek
E. Raceman

43) A white sclera line between the iris and the upper eyelid when looking
down with thyroid hyperfunction syndrome is a symptom:

A. Kocher
B. Shtelvag
C. Grefe
D. Krause
E. Elinek

44) A white sclera line between the iris and upper eyelid when looking up with
thyroid hyperfunction syndrome is a symptom:

A. Kocher
B. Shtelvag
C. Grefe
D. Elinek
E. Raceman

45) The wide opening of the eye slits for thyroid hyperfunction syndrome is a
symptom:

A. Kocher
B. Delrymple
C. Grefe
D. Elinek
E. Shtelvag

46) Enhanced pulsation of the carotid arteries, ascending apical impulse, pulse
110 beats. per minute, arrhythmic or rapid pulse are characteristic of:

A. Hyperglycemia syndrome
B. Hypoglycemia syndrome
C. Thyroid hyperfunction syndrome
D. Thyroid hypofunction syndrome
E. Hypopituitarism syndrome

47) In hyperthyroidism syndrome, the pulse:

A. Voltage, low speed


B. Tense, fast
C. Rare
D. Soft, slow
E. Large, rare
48) What is the amount of anhydrous glucose necessary for the oral glucose
tolerance test in adults?

A. 35 g
B. 50 g
C. 75 g
D. 50 mg
E. 100 mg
49) Indicate laboratory signs of primary hypocorticism syndrome:

A. Increased adrenocorticotropic hormone, decreased cortisol


B. Increased adrenocorticotropic hormone, increased cortisol
C. Decrease in adrenocorticotropic hormone, increased cortisol
D. Lowering adrenocorticotropic hormone, lowering cortisol
E. Adrenocorticotropic hormone is normal, lowering cortisol
50) The patient has hyperglycemia syndrome, after exercise there was a
tremor, palpitations, severe hunger, weakness, anxiety. Your tactics:

A. In/v the jet introduction of 40% glucose


B. I/v administration of glucagon + insulin
C. S/c administration of short-acting insulin
D. S/c administration of insulin ultrashort action
E. Urgent feeding of the patient, give sweet tea
51) It is typical for hyperglycemia syndrome:

A. Polyuria
B. Hypertension
C. Anuria
D. Shortness of breath
E. Oliguria
52) Male 30 years old, height 170 cm, body weight 105 kg, fat deposition
predominates in the trunk, on the skin of the inner surface of the shoulders
and thighs of purple streak, on the face and back multiple acne vulgaris. BP
steadily increased at the level of 150/90 - 165/110 mm Hg. Art. Daily excretion
of cortisol in the urine is higher than normal. Fasting glycemia 6.0 mmol / l,
sugar in urine - neg., The reaction of urine to acetone is negative. Highlight
the main syndrome.

A. Hyperglycemia syndrome
B. Hypoglycemia syndrome
C. Syndrome of chronic insufficiency of the adrenal cortex
D. Hypercortisolism syndrome
E. Overweight syndrome
53) Craving for salty foods is most characteristic of the syndrome:

A. Hypoparathyroidism
B. Hyperparathyroidism
C. Hypercorticism
D. Hypocorticism
E. Hypothyroidism
54) Patient L., 34 years old. Complains of overweight, fatigue. Body weight
increased significantly 5 years ago after childbirth. Menstruation from the age
of 13, regular. Loves flour products, sweets. Father and mother are obese 1-2
tbsp. The younger brother has obesity 1 tbsp. Objectively. Height - 168 cm,
body weight - 96 kg. BMI - 34kg / 2, waist circumference - 95 cm. Hip
circumference - 106 cm. Deposition of subcutaneous fat in the thighs.
Determine the types and degrees of obesity.

A. Android obesity type I


B. Guinea type II obesity
C. Cushingoid type of obesity grade II
D. Android type II obesity
E. Ginoid type of obesity I degree
55) What syndrome is manifested by an increase in the size of the thyroid
gland, bug-eyed, weight loss, tachycardia, increased blood pressure:

A. Hypothyroidism
B. Hyperthyroidism
C. Hypocorticoidism
D. Hypercortisolism
E. Hypopituitarism

56) For what syndrome are dry, wrinkled, cold, thickened skin, anemia,
narrowing of the palpebral fissures, thickening of the lips, tongue,
constipation, bradycardia, hypothermia?

A. Hypothyroidism
B. Hyperthyroidism
C. Pituitary Hyperfunction
D. Hypocorticoidism
E. Hypercortisolism
57) Hypoaldosteronism syndrome is a manifestation of the following
syndrome:

A. Hypercorticism
B. Hypoglycemia
C. Hypocorticism
D. Hyperthyroidism
E. Hypoparathyroidism
58) Under what pathology skin is smooth, warm, tender to the touch with
hyperhidrosis:

A. Hypothyroidism
B. Hyperthyroidism
C. Pituitary Hyperfunction
D. Pituitary hypofunction
E. Hyperglycemia

59) The manifestation of which pathology is mental excitement, imbalance,


quickness of mood changes, constant anxiety:

A. Hypothyroidism
B. Hyperthyroidism
C. Pituitary Hyperfunction
D. Pituitary hypofunction
E. Obesity
60) Patient B., 48 years old. Complaints of swelling of the face, extremities,
chilliness, weakness, memory loss, hoarseness, constipation. Objectively:
height - 170 cm, body weight - 89 years. The skin is dry, pale, flaky. The voice
is low. Pulse - 50 per min., Blood pressure - 110/80 mm Hg. What syndrome
are we talking about?

A. Hyperthyroidism
B. Obesity
C. Hypoglycemia
D. Hypothyroidism
E. Hyperglycemia
61) A young man of 17 years old after severe stress appeared thirst, polyuria.
General weakness, blood sugar level - 20 mmol / l, in urine - 5%, acetone in
urine is positive. Highlight the main syndrome.

A. Hyperglycemia
B. Hyperthyroidism
C. Pituitary Hyperfunction
D. Pituitary hypofunction
E. Chronic adrenal insufficiency

62) Patient 56 years old, height 160 cm, weight 105 kg. No complaints. Fat
distribution is even, especially on the limbs, abdomen, thighs. Borders of the
heart are increased to the left, the deaf tones, systolic murmur at the apex.
Highlight the main syndrome.

A. Hyperglycemia
B. Hypoglycemia
C. Pituitary Hyperfunction
D. pituitary hypofunction
E. Obesity syndrome
63) A patient came to see an endocrinologist. When calculating the BMI was
30.8 kg / m2. Determine the degree of violation of fat metabolism?

A. Overweight
B. Obesity grade II
C. Obesity grade I
D. Grade III obesity
E. Normal body weight
64) Abdominal obesity in women is considered to be a waist circumference
greater than:
A. 76 cm
B. 80 cm
C. 84 cm
D. 88 cm
E. 92 cm
65) Normally, the volume of the thyroid gland (according to ultrasound) in
women is:

A. Up to 16 ml
B. Up to 18 ml
C. More than 20 ml
D. Up to 25 ml
E. Up to 30 ml
66) Normally, the volume of the thyroid gland (according to ultrasound) in
men is:

A. Up to 16 ml
B. Up to 18 ml
C. To 20 ml
D. Up to 25 ml
E. More than 30 ml
67) The most common cause of adrenal insufficiency is:

A. Infections
B. Autoimmune adrenal disease
C. Tuberculosis
D. Syphilis
E. Nephroangiosclerosis
68) What is the characteristic sign of hypercorticismsyndrome:

A. Exophthalmos
B. Matronism
C. Hypogonadism
D. Hypoglycemia
E. Alopecia
69) Patient C., 46 years old, complains of severe weakness, lack of appetite,
weight loss in 6 months per 10 kg, nausea, occasionally loose stools, and at
times decrease in blood pressure. From the anamnesis: ill for six months, the
above symptoms gradually appeared and progressed. Highlight the main
syndrome.
A. Hyperglycemia syndrome
B. Hypoglycemia syndrome
C. Pituitary gland syndrome
D. Syndrome of chronic insufficiency of the adrenal cortex
E. Thyroid hyperfunction syndrome
70) Patient D., 32 years old. Complaints of significant muscle weakness,
dizziness, pain in the heart, emaciation, loss of appetite. History: ill for about
6 months. The disease does not bind to anything. At the age of 26 he suffered
pulmonary tuberculosis. Objectively: height - 176 cm, weight - 60 kg, the skin
on the open parts of the body is pigmented. What syndrome has a patient
developed?

A. hypoglycemia syndrome
B. syndrome of hypofunction of the thyroid gland
C. Hypercortisolism syndrome
D. syndrome hypopituitrism
E. syndrome of chronic insufficiency of the adrenal cortex
71) Patient D., 50 years old, housewife. She complains about weight gain,
flashing of “flies” before her eyes, headaches, weakness. From the anamnesis:
the above complaints began with a change in appearance: the face was
rounded, the belly increased in volume. Objectively: the patient looks much
older than her years. Height-156 cm, weight - 90 kg, on the thighs - wide
purple strii. Determine the pre-syndrome:

A. Hyperglycemia
B. Hypoglycemia
C. Pituitary Hyperfunction
D. Pituitary hypofunction
E. Hypercoticism

72) For what syndrome are typical "moonlike" face, increased hair growth
over the upper lip, in the chin area and on the lateral surfaces of the face. The
skin is dry, with a marble pattern. Hyperpigmentation of the neck. In axillary
areas, on the thighs and lateral surfaces of the abdomen, broad purple-red
bands:

A. Hypothyroidism
B. Hyperthyroidism
C. Pituitary Hyperfunction
D. Hypocorticoidism
E. Hypercortisolism
73) Patient I., 65 years old, complains of chilliness and swelling. Anamnesis is
difficult to collect because of the pronounced loss of memory and inhibition of
the patient. Survey results: T4 free - 0.1 pmol / l; TTG - 69 mMe / l;
Antibodies to TPO - 1000 IU / ml. Highlight the main syndrome.

A. Hypothyroidism
B. Hyperthyroidism
C. Obesity I degree.
D. Hypocorticoidism
E. Hypercortisolism
74) Chilliness, swelling, change in voice, impaired speech, dry skin,
constipation - these complaints are characteristic of the syndrome:

A. Hyperthyroidism
B. Hypocorticoidism
C. Hypercortisolism
D. Hypopituitarism
E. Hypothyroidism
75) What syndrome is manifested by an increase in the size of the thyroid
gland, a puffy, emaciation, tachycardia, an increase in blood pressure?

A. Hypothyroidism
B. Hyperthyroidism
C. Hypocorticoidism
D. Hypercortisolism
E. Hypopituitism
76) Patient Sh., 47 years old, for four years addressed various specialists with
complaints of weakness in the limbs, persistent pain in the calf muscles and
back. Osteoporosis, cysts and pathological fractures were found on
radiographs of the bones. Specify the syndrome.

A. Hypoparathyroidism
B. Thyrotoxicosis
C. Hyperparathyroidism
D. Hypoglycemia
E. Hypothyroidism
77) The patient after strumectomy had convulsions and a positive symptom of
Chvostek. Highlight the main syndrome.
A. Obesity syndrome
B. Thyrotoxicosis syndrome
C. Anorexia syndrome
D. Hypoparathyroidism syndrome
E. Hyperparathyroidism syndrome
78) Thin arms and legs, excessive deposition of fat on the face and torso,
“moon-shaped” hyperemic face occur with the syndrome:

A. Hypocorticoidism
B. Hypercortisolism
C. Hypopituitarism
D. Hypothyroidism
E. Hyperthyroidism
79) A 40-year-old patient complains of chilliness, drowsiness, constipation, dry
skin. An objective study found an enlarged thyroid gland. To confirm this
syndrome the most informative:

A. Determination of the level of Ca2 + in the blood


B. Determination of the level of TSH, sv. T4
C. Ultrasound of the thyroid gland
D. Determination of blood cholesterol
E. Definition of T3
80) Patient Z., turned to the local doctor with complaints of a change in skin
color and severe muscle weakness. From the anamnesis it is known that 6
years ago the patient suffered a tuberculous lesion of the lungs. Objectively:
low nutrition, depigmentation of open areas of the body.

What research is needed?

A. Free thyroxine, thyroid stimulating hormone


B. Cortisol, adrenocorticotropic hormone
C. Fasting plasma glucose, glycohemoglobin
D. Parathyroid hormone, ionized calcium
E. Progesterone, luteinizing hormone
81) Ultrasound of the thyroid gland: right lobe: length - 6.8 cm, width - 3.0
cm, thickness - 1.8 cm; left lobe: length - 6.0 cm, width - 2.0 cm, thickness - 1.9
cm. Determine the volume of the thyroid gland.

A. 14.0 ml
B. 18.0 ml
C. 18.4 ml
D. 28.4 ml
E. 40.0 ml
Previous year Pediatric
Endocrine System
1. Important complain in congenital hypothyroidism in newborn:
A. moist skin
B. constipation
C. bug - eyed
D. loss of weight
E. diarrhea

2. In congenital hypothyroidism skin is:


A. dry
B. moist
C. hyperemia
D. hat
E. hyperhidrasis

3. Clinical sign of hypothyroidism:


A. prolonged jaundice
B. exphthalmas
C. growth acceleration
D. hypertension
E. oss of weight

4. In hyperthyroidism skin is:


A. dry
B. moist
C. pink
D. cyanotic
E. jaundice

5. Hormones in congenital hypothyroidism are:


A. T3, T4 high
B. TSH decrease
C. T3, T4 decrease
D. not change
E. TSH high
6. Laboratory sign of syndrome hyperthyroidism:
A. T3, T4 high
B. TSH increase
C. T3, T4 decrease
D. hormones not change
E. cholesterol decrease
7.Main hormone of thyroid gland is:
A. TSH
B. thyroglobulin
C. thyroxin
D. triiodthyronine
E. thyrocalcitonin
8.The main function of the regulation of thyroid gland in children is:
A. physical and mental development
B. fat metabolism
C. glucore metabolism
D. mineral metabolism
E. water – salt metabolism
9. Main signs in diabetes mellitus syndrome:
A. high temperature
B. obesity
C. diarrhea
D. palydipsia and polyuria
E. nervous irritability
10.Blood glucose norm in children (mmol/l):
A. 2,0-2,5
B. 3,33-5,55
C. 1,5-2,0
D. 7-8
E. 9-10
1
1)Typical symptoms of hypoglycemia syndrome:

A. hunger, increased sweating

B. fatigue, thirst

C. nausea, vomiting, abdominal pain

D. pain in the heart, increased blood pressure

E. dry skin

2) First aid of hypoglycemic syndrome, if the patient is conscious:

A. urgent feeding the patient, give sweet tea

B. i / v administration of glucagon

C. s / c administration of short-acting insulin

D. s / c administration of insulin ultrashort action

E. i / v jet injection of 40% glucose - 100.0

3) The triad of symptoms in hyperglycemia syndrome:

A. polyuria, polydipsia, polyphagia

B. reduced vision, hearing, memory

C. nausea, vomiting, abdominal pain

D. Headache, dizziness, tinnitus

E. diarrhea, abdominal pain, nausea

4) Causes of hypoglycemia:

A. obesity

B. excessive exercise
C. stress

D. hypodynamia

E. pregnancy

5) Predisposing factors for the development of hyperglycemia syndrome:

A. obesity, hypodynamia, stress

B. excess in the diet of coarse fiber

C. excessive exercise

D. Underweight

E. Inadequate nutrition

6) Which of the following is the cause of the hypoglycemic syndrome:

A. syndrome hypercortisolism

B. overweight

C. hypocorticism syndrome

D. thyroid inflammation syndrome

E. hyperthyroidism syndrome

7) Which of the following causes hyperglycemic syndrome:

A. syndrome hypercortisolism

B. Hypothyroidism syndrome

C. hypocorticism syndrome

D. Hypoparathyroidism syndrome

E. anorexia
8) The glucose level in the plasma of venous blood is normal:

A. on an empty stomach 3.3–5.5 mmol / l, 2 hours after a meal> 7.8 mmol / l

B. fasting 5.6-6.1 mmol / l, 2 hours after eating 6.1-7.0 mmol / l

C. fasting 5.6-6.1 mmol / l, 2 hours after eating <6.7 mmol / l

D. fasting 4-6.1 mmol / l, 2 hours after eating <7.8 mmol / l

E. fasting 4.6-5.1 mmol / l, 2 hours after eating <8.5 mmol / l

9) Conditions for conducting an oral glucose tolerance test:

A. The test is conducted in the evening.

B. Pre-test analgesics

C. reception on the eve of at least 300 g of carbohydrates per day

D. The patient must not eat at least 4 hours before the test

E. OGTT should be carried out in the morning on the background of not less
than 3-day unlimited power.

10) The oral glucose tolerance test is performed:

A. amid acute illness

B. When glycohemoglobin is more than 6.5%.

C. in case of doubtful glycemia values to clarify diagnosis

D. When blood glucose is more than 11.0 mmol / L.

E. When fasting blood glucose is less than 3.3 mmol / L.

11) When viewed in the oral cavity with hyperglycemia syndrome can be identified:

A. loosening and early tooth loss


B. geographic language

C. tongue is covered with yellow bloom

D. black spots on tongue and gums

E. varnished language

12) Rare blinking in hyperthyroidism syndrome, this is a symptom:

A. Kocher

B. Shtelvag

C. Moebius

D. Krause

E. Elinek

13) Heart rate in 1 minute with hyperthyroidism syndrome:

A. less than 50 beats / min

B. 50-60 beats / min

C. 60-70 beats / min

D. 70-80 beats / min

E. more than 80 beats / min

14) Palpation of the thyroid gland reveals:

A. size, consistency, pain

B. total thyroid volume

C. retinal goiter

D. Thyroid function
E. iodine deficiency

15) On examination of the skin in a patient with hyperglycemia syndrome, it is


determined:

A. hyperpigmentation

B. humidity

C. rubeosis of the cheeks

D. cyanotic

E. facial hair growth in women

16) For hyperfunction of the thyroid gland is characteristic:

A. sleepiness

B. Bradycardia

C. puffiness of the face and narrowing of the palpebral fissures

D. hyperexcitability

E. obesity

17) With acute hypoglycemia syndrome:

A. skin is wet, motor agitation

B. breathing is noisy and rare

C. skin is dry

D. reduced turgor

E. thirst

18) It is typical for hyperglycemia syndrome:


A. polyuria

B. cyanosis

C. melasma

D. aglucosuria

E. hand tremor

19) Symptoms of hypothyroidism include:

A. diarrhea

B. sweating

C. constipation

D. cyanosis

E. heartbeat

20) Laboratory signs of hypothyroidism syndrome:

A. decrease in T3 and T4 in the blood

B. increase in T3 and T4 in the blood

C. elevation of TSH and T4 in the blood

D. decrease in the level of TSH in the blood

E. increased T3 and T4, decreased TSH

21) Hypocorticism syndrome is characterized by:

A. hyperpigmentation of the skin

B. weight gain

C. increased blood pressure


D. tachycardia

E. polyphagia

22) The volume of the waist in men normally should not exceed?

A. not more than 94 cm

B. not more than 106 cm

C. not more than 86 cm

D. less than 60 cm

E. morethan 102 cm

23) How to calculate BMI (body mass index)?

A. height / weight2

B. Weight / Height2

C. height2/ weight

D. Weight2 / Height

E. Weight / Height

24) In which units is body mass index measured?

A. m2 / kg

B. m / kg2

C. kg / m2

D. kg2 / m

E. kg / m
25) Waist size in women is normal?

A. not more than 65 cm

B. not more than 70 cm

C. not more than 80 cm

D. less than 60 cm

E. more than 88 cm

26) A patient came to see an endocrinologist. When calculating the body mass
index was 29.8 kg / m2. Determine the degree of violation of fat metabolism?

A. overweight

B. obesity grade II

C. obesity grade I

D. No fat metabolism abnormalities - normal body weight

E. Guinoid obesity

27) With the introduction of 1 mg of dexamethasone at 12 o'clock at night, a


decrease in cortisol by 50% or more the next morning indicates:

A. Hypocorticism

B. About the normal function of the adrenal cortex

C. about hypercorticism

D. About secondary hypocorticism

E. on increased secretion of ACTH

28) The following changes on an ECG are characteristic of a syndrome of a


hypofunction of a thyroid gland:

A. reduction of teeth voltage, bradycardia

B. tachycardia, ST segment reduction


C. atrial fibrillation

D. Pathological Q wave

E. AV blockade

29) Goiter is:

A. well palpable thyroid

B. thyroid disease, occurring in violation of its function

C. thyroid disease, complicated by tracheal compression

D. thyroid enlargement more than 18 ml in women and more than 25 ml in men

E. An increase in volume of more than 16 ml in women and more than 18 ml in


men.

30) Laboratory criteria for hypoparathyroidism syndrome:

A. increased blood potassium levels

B. increased blood levels of parathyroid hormone

C. lower blood calcium levels

D. lower blood sodium levels

E. Increased blood glucose

31) With an enlargement of the thyroid gland of I degree (WHO):

A. The thyroid gland is invisible, palpable

B. The thyroid gland is not visible to the eye, is not palpable.

C. thyroid visible on the eye, palpable

D. The thyroid gland is visible to the eye, not palpable.

E. Thyroid lobe smaller than the distal phalanx of the thumb studied
32) In hyperglycemia syndrome, whole capillary blood sugar on an empty
stomach:

A. 3.5-4.4 mmol / l

B. 5.2-5.9 mmol / l

C. more than 6.1 mmol / l

D. 3.3-5.5 mmol / l

E. 3.3 - 5.6 mmol / l

33) A young man of 18 years old after a cold has thirst, polyuria, weakness. Blood
sugar level is 16mmol / l. What is the patient's syndrome?

A. hypoglycemia syndrome

B. hyperglycemia syndrome

C. thyroid hypofunction syndrome

D. Hypercortisolism syndrome

E. Thyroidhyperfunctionsyndrome

34) Palpation in adults in the normal thyroid gland:

A. soft, agile

B. Dense, welded to underlying tissues.

C. moderate soft, not mobile

D. soft, motionless

E. tight, painful

35) WHO classification of goiter:

A. 0,1,2,3,4 degrees
B. 0,1,2,3 degrees

C. 0,1,2 degrees

D. 1,2,3 degrees

E. 1,2,3,4,5 degrees

36) What changes in the content of thyroid-stimulating hormone, triiodothyronine


and thyroxin are characteristic of hypothyroidism syndrome?

A. a decrease in serum thyroid-stimulating hormone (TSH) and an increase in


thyroxin (T4) and triiodothyronine (T3) levels

B. Normal levels of TSH and increased serum T3 and T4 levels

C. elevated levels of TSH and a decrease in serum T3 and T4

D. Normal serum TSH, T3 and T4

E. elevated serum TSH, T3 and T4

37) What research should be carried out in the first place with hyperparathyroidism
syndrome:

A. complete blood count

B. urinalysis

C. blood glucose

D. level of calcium and blood phosphorus

E. level of potassium and sodium in the blood

38) Atrial fibrillation can occur when:

A. hyperglycemia syndrome

B. hypoglycemia syndrome

C. hyperthyroidism syndrome

D. hypothyroidism syndrome
E. hypocorticism syndrome

39) Features of edema in thyroid hypofunction syndrome:

A. dense, do not leave a fossa

B. leaves a hole for palpation

C. pronounced in the evening on the lower limbs

D. expressed under the morning under the eyes

E. mild, painful

40) General weakness, fatigue, severe muscle weakness, darkening of the skin,
weight loss, loss of appetite, hypotension - these complaints are characteristic of
the syndrome:

A. hypothyroidism

B. hyperthyroidism

C. Hypocorticoidism

D. hypercortisolism

E. hypopituitarism

41) What syndrome is characterized by "matronism", "bull neck", "marble skin


pattern":

A. hyperglycemia

B. hyperthyroidism

C. Hypocorticoidism

D. hypercortisolism

E. hypothyroidism
42) Pigmentation of the eyelids (upper) with thyroid hyperfunction syndrome is a
symptom:

A. Kocher

B. Moebius

C. Krause

D. Elinek

E. Raceman

43) A white sclera line between the iris and the upper eyelid when looking down
with thyroid hyperfunction syndrome is a symptom:

A. Kocher

B. Shtelvag

C. Grefe

D. Krause

E. Elinek

44) A white sclera line between the iris and upper eyelid when looking up with
thyroid hyperfunction syndrome is a symptom:

A. Kocher

B. Shtelvag

C. Grefe

D. Elinek

E. Raceman

45) The wide opening of the eye slits for thyroid hyperfunction syndrome is a
symptom:

A. Kocher
B. Delrymple

C. Grefe

D. Elinek

E. Shtelvag

46) Enhanced pulsation of the carotid arteries, ascending apical impulse, pulse 110
beats. per minute, arrhythmic or rapid pulse are characteristic of:

A. hyperglycemia syndrome

B. hypoglycemia syndrome

C. thyroid hyperfunction syndrome

D. thyroid hypofunction syndrome

E. hypopituitarism syndrome

47) In hyperthyroidism syndrome, the pulse:

A. low voltage, low speed

B. tense, fast

C. rare

D. soft, slow

E. large, rare

48) What is the amount of anhydrous glucose necessary for the oral glucose
tolerance test in adults?

A. 35 g

B. 50 g

C. 75 g

D. 50 mg
E. 100 mg

49) What is characteristic of hypoparathyroidismsyndrome:

A. Frequent pathological bone fractures

B. epigastric pain, dyspepsia

C. the appearance of kidney stones

D. severe thirst

E. Increased neuromuscular excitability with tetany attacks

50) The patient has hyperglycemia syndrome, after exercise there was a tremor,
palpitations, severe hunger, weakness, anxiety. Your tactics:

A. In / v the jet introduction of 40% glucose

B. i / v administration of glucagon + insulin

C. s / c administration of short-acting insulin

D. s / c administration of insulin ultrashort action

E. urgent feeding of the patient, give sweet tea

51) It is typical for hyperglycemia syndrome:

A. polyuria

B. hypertension

C. anuria

D. shortness of breath

E. oliguria

52) Male 30 years old, height 170 cm, body weight 105 kg, fat deposition
predominates in the trunk, on the skin of the inner surface of the shoulders and
thighs of purple streak, on the face and back multiple acne vulgaris. BP steadily
increased at the level of 150/90 - 165/110 mm Hg. Art. Daily excretion of cortisol
in the urine is higher than normal. Fasting glycemia 6.0 mmol / l, sugar in urine -
neg., The reaction of urine to acetone is negative. Highlight the main syndrome.

A. hyperglycemia syndrome

B. Hypoglycemia syndrome

C. syndrome of chronic insufficiency of the adrenal cortex

D. Hypercortisolism syndrome

E. Overweight syndrome

53) For carrying out a large dexamethasone test for hypercorticism, use:

A. 1 mg of dexamethasone

B. 8 mg dexamethasone

C. 16 mg dexamethasone

D. 2 mg dexamethasone

E. 4 mg dexamethasone

54) Patient L., 34 years old. Complains of overweight, fatigue. Body weight
increased significantly 5 years ago after childbirth. Menstruation from the age of
13, regular. Loves flour products, sweets. Father and mother are obese 1-2 tbsp.
The younger brother has obesity 1 tbsp. Objectively. Height - 168 cm, body weight
- 96 kg. BMI - 34kg / 2, waist circumference - 95 cm. Hip circumference - 106 cm.
Deposition of subcutaneous fat in the thighs. Determine the types and degrees of
obesity.

A. android obesity type I

B. Guinea type II obesity

C. Cushingoid type of obesity grade II

D. android type II obesity

E. ginoid type of obesity I degree


55) What syndrome is manifested by an increase in the size of the thyroid gland,
bug-eyed, weight loss, tachycardia, increased blood pressure:

A. hypothyroidism
B. hyperthyroidism
C. Hypocorticoidism
D. hypercortisolism
E. hypopituitarism

56) For what syndrome are dry, wrinkled, cold, thickened skin, anemia, narrowing
of the palpebral fissures, thickening of the lips, tongue, constipation, bradycardia,
hypothermia?

A. hypothyroidism

B. hyperthyroidism

C. Pituitary Hyperfunction

D. Hypocorticoidism

E. hypercortisolism

57) At what syndrome are observed loss of eyelashes, eyebrows, mustaches, hair
on the head, massive, dense edema:

A. hypothyroidism

B. hyperthyroidism

C. Pituitary Hyperfunction

D. pituitary hypofunction

E. Chronic adrenal insufficiency

58) Under what pathology skin is smooth, warm, tender to the touch with
hyperhidrosis:

A. hypothyroidism
B. hyperthyroidism

C. Pituitary Hyperfunction

D. pituitary hypofunction

E. hyperglycemia

59) The manifestation of which pathology is mental excitement, imbalance,


quickness of mood changes, constant anxiety:

A. hypothyroidism

B. hyperthyroidism

C. Pituitary Hyperfunction

D. pituitary hypofunction

E. obesity

60) Patient B., 48 years old. Complaints of swelling of the face, extremities,
chilliness, weakness, memory loss, hoarseness, constipation. Objectively: height -
170 cm, body weight - 89 years. The skin is dry, pale, flaky. The voice is low.
Pulse - 50 per min., Blood pressure - 110/80 mm Hg. What syndrome are we
talking about?

A. hyperthyroidism

B. obesity

C. hypoglycemia

D. hypothyroidism

E. hyperglycemia

61) A young man of 17 years old after severe stress appeared thirst, polyuria.
General weakness, blood sugar level - 20 mmol / l, in urine - 5%, acetone in urine
is positive. Highlight the main syndrome.
A. hyperglycemia

B. hyperthyroidism

C. Pituitary Hyperfunction

D. pituitary hypofunction

E. Chronic adrenal insufficiency

62) Patient 56 years old, height 160 cm, weight 105 kg. No complaints. Fat
distribution is even, especially on the limbs, abdomen, thighs. Borders of the heart
are increased to the left, the deaf tones, systolic murmur at the apex. Highlight the
main syndrome.

A. hyperglycemia

B. hypoglycemia

C. Pituitary Hyperfunction

D. pituitary hypofunction

E. obesity syndrome

63) A patient came to see an endocrinologist. When calculating the BMI was 30.8
kg / m2. Determine the degree of violation of fat metabolism?

A. overweight

B. obesity grade II

C. obesity grade I

D. Grade III obesity

E. normal body weight

64) Hypopituitarism is a decrease in function:

A. thyroid gland
B. hypothalamus

C. adrenal glands

D. pituitary gland

E. pineal gland

65) Normally, the volume of the thyroid gland (according to ultrasound) in women
is:

A. up to 16 ml

B. up to 18 ml

C. more than 20 ml

D. up to 25 ml

E. up to 30 ml

66) Normally, the volume of the thyroid gland (according to ultrasound) in men is:

A. up to 16 ml

B. up to 18 ml

C. to 20 ml

D. up to 25 ml

E. more than 30 ml

67) The most common cause of adrenal insufficiency is:

A. infections

B. Autoimmune adrenal disease

C. tuberculosis

D. syphilis

E. nephroangiosclerosis
68) When examining a patient with hypercortisol syndrome, it is revealed:

A. uniform obesity

B. facial hair growth in women

C. muscle wasting

D. thick, rough skin

E. weight loss

69) Patient C., 46 years old, complains of severe weakness, lack of appetite, weight
loss in 6 months per 10 kg, nausea, occasionally loose stools, and at times decrease
in blood pressure. From the anamnesis: ill for six months, the above symptoms
gradually appeared and progressed. Highlight the main syndrome.

A. hyperglycemia syndrome

B. Hypoglycemia syndrome

C. pituitary gland syndrome

D. syndrome of chronic insufficiency of the adrenal cortex

E. Thyroid hyperfunction syndrome

70) Patient D., 32 years old. Complaints of significant muscle weakness, dizziness,
pain in the heart, emaciation, loss of appetite. History: ill for about 6 months. The
disease does not bind to anything. At the age of 26 he suffered pulmonary
tuberculosis. Objectively: height - 176 cm, weight - 60 kg, the skin on the open
parts of the body is pigmented. What syndrome has a patient developed?

A. hypoglycemia syndrome

B. syndrome of hypofunction of the thyroid gland

C. Hypercortisolism syndrome

D. syndrome hypopituitrism
E. syndrome of chronic insufficiency of the adrenal cortex

71) Patient D., 50 years old, housewife. She complains about weight gain, flashing
of “flies” before her eyes, headaches, weakness. From the anamnesis: the above
complaints began with a change in appearance: the face was rounded, the belly
increased in volume. Objectively: the patient looks much older than her years.
Height-156 cm, weight - 90 kg, on the thighs - wide purple strii. Determine the
pre-syndrome:

A. hyperglycemia

B. hypoglycemia

C. Pituitary Hyperfunction

D. pituitary hypofunction

E. hypercoticism

72) For what syndrome are typical "moonlike" face, increased hair growth over the
upper lip, in the chin area and on the lateral surfaces of the face. The skin is dry,
with a marble pattern. Hyperpigmentation of the neck. In axillary areas, on the
thighs and lateral surfaces of the abdomen, broad purple-red bands:

A. hypothyroidism

B. hyperthyroidism

C. Pituitary Hyperfunction

D. Hypocorticoidism

E. hypercortisolism

73) Patient I., 65 years old, complains of chilliness and swelling. Anamnesis is
difficult to collect because of the pronounced loss of memory and inhibition of the
patient. Survey results: T4 free - 0.1 pmol / l; TTG - 69 mMe / l; Antibodies to
TPO - 1000 IU / ml. Highlight the main syndrome.

A. hypothyroidism
B. hyperthyroidism

C. obesity Art. I

D. Hypocorticoidism

E. hypercortisolism

74) Chilliness, swelling, change in voice, impaired speech, dry skin, constipation -
these complaints are characteristic of the syndrome:

A. hyperthyroidism

B. Hypocorticoidism

C. hypercortisolism

D. hypopituitarism

E. hypothyroidism

75) What syndrome is manifested by an increase in the size of the thyroid gland, a
puffy, emaciation, tachycardia, an increase in blood pressure?

A. hypothyroidism

B. hyperthyroidism

C. Hypocorticoidism

D. hypercortisolism

E. hypopituitism

76) Patient Sh., 47 years old, for four years addressed various specialists with
complaints of weakness in the limbs, persistent pain in the calf muscles and back.
Osteoporosis, cysts and pathological fractures were found on radiographs of the
bones. Specify the syndrome.

A. hypoparathyroidism
B. thyrotoxicosis

C. hyperparathyroidism

D. hypoglycemia

E. hypothyroidism

77) The patient after strumectomy had convulsions and a positive symptom of
Chvostek. Highlight the main syndrome.

A. obesity syndrome

B. thyrotoxicosis syndrome

C. anorexia syndrome

D. Hypoparathyroidism syndrome

E. hyperparathyroidism syndrome

78) Thin arms and legs, excessive deposition of fat on the face and torso, “moon-
shaped” hyperemic face occur with the syndrome:

A. Hypocorticoidism

B. hypercortisolism

C. Hypopituitarism

D. hypothyroidism

E. hyperthyroidism

79) A 40-year-old patient complains of chilliness, drowsiness, constipation, dry


skin. An objective study found an enlarged thyroid gland. To confirm this
syndrome the most informative:

A. determination of the level of Ca2 + in the blood

B. determination of the level of TSH, sv. T4

C. Ultrasound of the thyroid gland


D. determination of blood cholesterol

E. Definition of T3

80) Woman A., 33 years old, six months after strumectomy, had complaints of
weakness, apathy, drowsiness, constipation, and her menstrual cycle was disrupted.
Objectively: height - 167 cm, body weight - 78 years kg. The skin is dry, hair is
dry, brittle, hair loss of the outer parts of the eyebrows. Pulse - 56 per minute,
rhythmic. BP - 120/80 mm Hg. Art. The boundaries of the relative dullness of the
heart is normal, the tones are weakened. Highlight the syndrome.

A. Hypothyroidism syndrome

B. Hyperthyroidism syndrome

C. obesity syndrome

D. Hypoglycemia syndrome

E. hyperglycemia syndrome

81) Ultrasound of the thyroid gland: right lobe: length - 6.8 cm, width - 3.0 cm,
thickness - 1.8 cm; left lobe: length - 6.0 cm, width - 2.0 cm, thickness - 1.9 cm.
Determine the volume of the thyroid gland.

A. 14.0 ml

B. 18.0 ml

C. 18.4 ml

D. 28.4 ml

E. 40.0 ml
2
1. Diffuse alopecia is:

A. Increased hair breakage

B. Dryness and section of hair

C. Uniform hair loss

D. Local alopecia

E. Excessive hair growth

2. Lupus cheilitis is:

A. Bubble eruptions on the mucosa of lips

B. Painless ulcers on the gum mucosa

C. Dense dry grayish scales, with inflammation of the red border of the
lips

D. "Bits" at the corners of the mouth

E. Aphthous stomatitis on the background of DBST

3. "Dermatomyositis glasses" are:

A. Periorbital edema with purplish-purple erythema

B. Cyanotic eyelid skin staining

C. Dark circles under the eyes

D. Hyperpigmentation of the upper eyelids

E. Eye constriction with erythema of eyelids

4. Sjogren syndrome is:

A. "dry" syndrome due to dehydration

B. "dry" syndrome due to lesions of the endocrine glands

C. "dry" syndrome due to lesions of the exocrine glands


D. “dry” syndrome associated with dry skin eczema

E. “dry” syndrome due to photosensitivity

5. Raynaud's syndrome is:

A. Episodes of limb arteriole constriction, manifested by whitening and


cyanosis of the fingers

B. Numbness of fingers

C. Redness of the fingertips after changing the temperature

D. The appearance of ulcers in the periungual bed

E. The appearance of areas of skin atrophy at the tips of the fingers of the
hands and feet

6. DIC syndrome stands for:

A. Degenerative intraarticular syndrome

B. Disseminated intravascular coagulation syndrome

C. Diffuse vegetative-vascular syndrome

D. Diffuse intravascular sclerosis

E. Demyelinating intracranial sclerosing syndrome

7. DIC may be a complication of:

A. Syndrome of diffuse lesions of connective tissue

B. Syndrome of degenerative lesions of the joints

C. Syndrome of soft tissue lesions around the articular tissues

D. Osteopenic syndrome

E. Sjogren's syndrome

8. Antiphospholipid syndrome occurs in:

A. Degenerative lesions of the joints

B. Inflammatory lesions of the joints


C. Diffuse lesions of connective tissue syndrome

D. Syndrome of soft tissue lesions around the articular tissues

E. Osteopenic syndrome

9. Viscerites are:

A. Pathognomonic symptomacomplexes, characteristic of lesions within the


same system

B. Concurrent lesions of internal organs

C. Complications of drug therapy for visceral organs

D. Affection of the peritoneum in the syndrome of diffuse lesion of the


connective tissue

E. Syndromes caused by damage to internal organs

10. Viscerite includes:

A. Lymphadenitis

B. Onychodystrophy

C. Cardit

D. Alopecia

E. Arteritis

11. Of the extra-articular manifestations in the syndrome of diffuse lesions of


the connective tissue, the most important is:

A. Lesions of the lymphatic vessels

B. Kidney damage

C. Lesion of the spleen

D. Liver damage

E. Damage of exocrine glands

12. Livedo reticularis is:


A. Varicose saphenous veins

B. Damage to blood vessels with spasm of the capillary bed

C. Resistant bluish color, uneven skin color in the form of tree spots

D. The appearance of bruises of various sizes on the skin of the limbs

E. Palpable nodules along the arteries of medium caliber

13. Osteolysis is:

A. Formation of bone defects in spongy bones

B. Decrease in bone mass per unit volume

C. Bone resorption, often of nail phalanges

D. Thinning of the bones

E. Cystic bone remodeling

14. Aseptic bone necrosis is:

A. Cellular (holes) pattern of bone tissue

B. Complete necrosis of spongy bone tissue and bone marrow

C. Bone resorption, often of nail phalanges

D. Decrease in bone mass per unit volume

E. Formation of bone defects in tubular bones

15. Patient M., 36 years old. There is hyperemia, hyperthermia, sharp pain in
1 metatarsophalangeal joint of the foot on the left. Uric acid 455mmol / l. ESR
20 mm / hour. Specify the main syndrome:

A. Syndrome of degenerative-dystrophic lesions of the joints

B. Syndrome of inflammatory lesions of the joints

C. Syndrome of diffuse lesions of the connective tissue

D. Osteopenic syndrome

E. Syndrome of soft tissue damage near the articular tissues

16. 16-years old girl complaints of periodic whitening and cyanosis of the
fingers. Weight loss up to 6 kg for 2 months. Hardening of the skin on the hands
and face. Diffuse alopecia. Antibodies to topoisomerase -1 in blood serum
analysis. Specify the main syndrome:

A. Anemic syndrome

B. Syndrome of diffuse connective tissue damage

C. Astheno-vegetative syndrome

D. Sjogren's syndrome

E. Syndrome of soft tissue damage near the articular tissues

17. The clinical features of autoimmune diseases are:

A. Predominant involvement of males;

B. Full recovery with continuous treatment;

C. Propensity for progression and recurrence;

D. There are no changes in blood tests;

E. Lack of complete remission;

18. The primary role in the development of diffuse diseases of the connective
tissue is:

A. Congenital anomalies of the internal organs;

B. Children's age;

C. A combination of environmental problems with nutritional patterns;

D. A combination of genetic predisposition and immune disorders;

E. Bad habits (smoking);

19. Female 35-years old. The complaints of marked myasthenia (muscle


weakness) and fever. Objectively: periorbital edema with a purple-lilac
erythema around the eyes, thickening of the muscles of the shoulders and hips
with a painful reaction to palpation. In the blood, an increase in CRP, ESR.
What analysis needed to send the patient to clarify the diagnosis?

A. Creatine phosphokinase;

B. Rheumatoid factor;

C. Antistreptolysin O;
D. Antibodies to DNA strands

E. Creatinine blood;

20. The patient has complaints of pain and swelling in the right knee joint. The
ultrasound in the joint cavity shows about 50 ml of fluid. It corresponds to:

A. A pronounced degree of joint defiguration;

B. Weak degree of joint defiguration;

C. No defiguration;

D. Moderate joint defiguration;

E. Permissible fluid volume;

21. In a patient with pain and limitation of range of motion in the hip joint, an
X-ray examination can determine:

A. Thickness of the synovial membrane;

B. Atrophy of muscles;

C. Viscosity of synovial fluid;

D. The presence of destructive changes in the joint;

E. Volume of synovial fluid;

22. Boutonniere deformation is:

A. Deformation with flexion of the proximal and over-flexion of the distal


interphalangeal joints

B. Deformation, accompanied by subluxation of the distal phalanges of the hands

C. Deviation of the wrist to the radial side

D. Thickening of the distal interphalangeal joints resembling a flower bud

E. Deviation of the hand to the elbow side

23. Ulnar deviation is:

A. Lateral over-extension of the hand


B. Deformation, accompanied by the deviation of the wrist in the radial direction

C. Deformity of the joints of the hands, resembling a snail

D. Deformation, accompanied by a deviation of the hand in the elbow side

E. Deformation, accompanied by subluxations of the distal phalanges of the hands

24. Deformation in the form of mutilating arthritis is characterized by:

A. Abnormal development of the hands and feet due to various genetic mutations

B. Multidirectional curvature and shortening of the phalanges

C. Disruption of the axis of the joints of the hands and feet

D. The fusion of the phalanges between the fingers

E. Deviation of the hand to the radial side

25. Bone erosion detected by X-ray examination is:

A. Marginal bone tissue defects in the form of uzura

B. Bone cysts

C. Subluxations of joints

D. Sharp narrowing of the joint space

E. "punched-out" lesion

26. Monoarticular lesions are:

A. Damage to the joints on one side

B. Damage to one joint

C. A single, non-recurrent joint disorder

D. Monotonic, chronic lesions of the joints

E. Damage of one tendon

27. Visual analog scale (VAS) is:

A. Visual inspection of the joints

B. Evaluation of the function of similar joints compared with the symmetrical side
C. Assessment of the condition of the joint using visualizing methods of instrumental
diagnostics

D. Method of objectification of pain syndrome, assessed by the patient

E. Evaluation of muscle function using visualization methods of instrumental


diagnostics

28. Morning stiffness is:

A. morning stiffness in the whole body

B. Feeling of stiffness in the joints in the morning after waking up

C. Joint pain in the morning

D. Tight swelling of the joints in the morning

E. General weakness in the morning due to intoxication

29. Tophi are:

A. Subcutaneous accumulations of sodium monourate salts

B. Marginal bone formation in the distal phalanges of the fingers

C. Subcutaneous inflammatory masses with central fibrinoid necrosis

D. Subcutaneous calcium deposits in periarticular tissue

E. Subcutaneous nodules with purulent contents

30. Rheumatoid nodules are most characteristic of:

A. Syndrome of degenerative lesions of the joints

B. Syndrome of inflammatory lesions of the joints

C. Syndrome of diffuse lesions of the connective tissue

D. CREST syndrome

E. Syndrome of soft tissue lesions of the periarticular tissues

31. In inflammatory lesions of the joint, the ultrasonography can identify the
following characteristic features:

A. Cluster of synovial exudate, proliferation of the synovial membrane


B. The presence of microorganisms that cause joint disease

C. Bone UZURATIONS

D. "punched-out" symptom

E. Regional osteosclerosis

32. Rheumatoid factor is:

A. Risk factor for developing rheumatoid arthritis

B. Factor contributing to the development of deformities in rheumatoid arthritis

C. Serologic marker for rheumatoid arthritis

D. Mandatory criteria for rheumatoid arthritis

E. Factor showing reactive arthritis

33. Myasthenia is:

A. Pain in the muscles of the limbs

B. Muscle hypotrophy

C. Muscle weakness

D. Muscle hyperkinesis

E. Muscle atrophy

34. What is the term for increased sensitivity to ultraviolet irradiation of open
skin surfaces?

A. Photosensitization

B. Dermafotosis

C. Photosynthesis

D. Ringworm

E. Photophobia

35. Patient R., 36 years old, was admitted to the clinic with complaints of pain
in the metacarpophalangeal, wrist, knee joints, swelling of these joints,
restriction of movements in them. In the morning, notes stiffness in the affected
joints until 12 o'clock in the afternoon. Specify the main syndrome:
A. Syndrome of diffuse inflammation of the connective tissue

B. Osteopenic syndrome

C. Syndrome of inflammatory changes of the joints

D. Muscle inflammatory syndrome

E. Syndrome of inflammation of the soft periarticular tissues

36. The most typical sign of deformity of the hand in the syndrome of
degenerative joint damage is:

A. Heberden's nodules

B. Ulnar deviation

C. "swan neck" deformity

D. "buttonhole" deformity

E. "m-shaped deformation"

37. Which joints are often affected in degenerative-dystrophic lesions of the


osteo-articular system:

A. elbow joint

B. knee joints

C. first metacarpophalangeal joint

D. Thoracic spine joints

E. shoulder joints

38. Bouchard nodes appear:

A. in the syndrome of damage to the soft periarticular tissues

B. in the syndrome of inflammatory changes in the joints

C. in the syndrome of degenerative joint damage

D. in diffuse connective tissue syndrome

E. in the syndrome of inflammatory muscle changes

39. The characteristic symptom of degenerative joint disease is:


A. Fever

B. morning stiffness for more than 1 hour

C. myalgia

D. hemorrhagic rash

E. “starting” character of pain

40. Deformation of the joint is due to:

A. Lack of joint pathology

B. Exudative- proliferative phenomena in the joint and periarticular tissues

C. bone ingrowths (fusion)

D. destruction of muscles

E. tight formation, rising above the skin

41. Ankylosis is:

A. lack of mobility in the joint due to bone adhesions.

B. due to proliferative changes in the joint or soft tissues

C. loss of joint native form

D. destruction of articular surfaces

E. restriction of movement in the joint

42. The measurement of the circumference of the joint is performed using:

A. Visually

B. centimeter tape

C. protractor

D. special pattern

E. Stopmeter

43. Which of the following is used to determine the range of motion in the joint:

A. Arthroscope
B. Osteometer

C. Goniometer

D. Caliper

E. Densitometry

44. Crepitation is:

A. limiting the range of motion in a joint in a certain direction

B. The presence of “joint muscle” inside the joint

C. palpation of cracking sound in the joint when moving

D. abnormal movement in the joint

E. fluid accumulation inside the joint

45. The symptom of "blockade" of the joint is:

A. Periodic acute pain in the joint with movements, fragments of cracking


cartilage getting into the joint space

B. inability to move in the joint due to fusion of the articular surfaces

C. increased pain in the joint after intra-articular blockade of the joint

D. morning joint pain upon waking

E. Periodic "stumbling" of the joint while walking

46. The starting nature of the pain is:

A. joint pain in an athlete during the start

B. pain in the joint at the beginning of the movement

C. morning pain in the joint after waking up

D. pain, aggravated at rest and relieved by movement

E. Arthralgia more disturbing in the evening and in the first half of the night.

47. One of the features of pain in a degenerative lesion of the hip joint is:

A. increased pain in the morning

B. irradiation of the pain to the groin area


C. Pain of "lumbago" type

D. burning character of the pain

E. pain worsening at rest

48. In a laboratory study in the case of degenerative lesions of the joints reveal:

A. high ESR

B. increasing level of antibodies to microorganisms

C. leukocytosis

D. Normal KLA scores

E. positive RF

49. Patient A., 67-years old. Complaints of pain in the knee, hip joints, when
moving and when going down the stairs. From the anamnesis: pain in the joints
for 5-6 years. The nodes of Heberden and Bouchard are noted on the hands.
CRP - neg. RF - neg. X-ray of the knee joints: narrowing of the interarticular
crevices, subchondral osteosclerosis, osteophytes. What syndrome are we
talking about?

A. diffuse lesion syndrome of connective tissue

B. muscle inflammatory syndrome

C. syndrome of degenerative joint damage

D. syndrome of lesions of the soft periarticular tissues

E. syndrome of inflammatory changes in the joints

53. Patient D., 54 years old, complaints of pain in the knee joints, which are
worse in the evening. Locally: In the area of the interphalangeal joints of the
hands, the Heberden nodules are defined, the knee joints are de-configured and
varus deformity. There is a crunch in the knee joints. UAC, OAM: no change.
What is your intended syndrome?

A. muscle inflammatory syndrome

B. syndrome of degenerative joint damage

C. syndrome of lesions of the soft periarticular tissues

D. syndrome of inflammatory changes in the joints


E. diffuse connective tissue syndrome

54. Osteopenic syndrome is:

A. Vascular pathology characterized by transient ischemia of the fingers, tip of the


nose and / or ears, caused by external factors (for example, cold).

B. It is the presence of an inflammatory process (edema, increased local temperature,


redness, soreness and impairment of function) in the joints, which is determined by
an objective examination.

C. Intermittent muscle weakness in patients with obliterating arteriosclerosis without


marked trophic disturbances.

D. The decrease in bone mineral density and the disruption of its


microarchitecture with the subsequent transition to osteoporosis

E. Aseptic necrosis of metacarpal bones with secondary osteosclerosis and arthrosis.

55. Which bone cells are responsible for resorption during bone remodeling?

A. Osteoblasts

B. Osteocyte

C. Osteoclasts

D. Osteoid

E. Chondrocytes

56. The most informative instrumental method of diagnosis of osteopenic


syndrome is:

A. X-ray densitometry

B. Conventional radiography

C. Computed tomography

D. Ultrasound of the joints

E. Arthroscopy

57. Densitometry determines:

A. Bone mineral density

B. The ratio of trace elements in bone tissue


C. The presence of synovial fluid

D. Swelling of periarticular soft tissue

E. Cartilage structure

58. The most frequent localization of osteopenic fractures:

A. Neck of the femur, vertebral body, fracture of the radius

B. Ribs, clavicle, shoulder blade,

C. Upper and lower jaw

D. Tibia and ankles

E. Bones of fingers and facial skull

59. Bone Formation Marker

A. Pyridinoline (PD, NTX) and deoxypyridinoline (U-DPD - free)

B. Bone alkaline phosphatase (bone ALP)

C. Hydroxproline (OP)

D. Galactosyloxylysin (GOP)

E. Tartrate-resistant acid phosphatase (t ARF)

60. Age-related changes in bone tissue are related:

A. with increasing bone sensitivity to parathyroid hormone

B. with increasing calcium excretion

C. with calcitonin deficiency

D. with inhibition of the synthesis of vitamin D3

E. with increasing testosterone levels

61. The defiguration is due to:

A. lack of mobility in the joint

B. Exudative- proliferative phenomena in the synovial membrane


C. Loss of joint native form

D. Destruction of articular surfaces

E. Restricted joint mobility

62. Nodules are:

A. A sign of active rheumatic fever, a pale pink, narrow thread that creates a lace
pattern on the skin, quickly appears and disappears

B. Skin infiltrates of red-purple color, clearly elevating above the skin surface, are
located mainly on the front surface of the tibia

C. on the nose and cheeks, such as erythema

D. Multiple aphthous and ulcerative lesions of the mucous membranes

E. dense formation, rising above the skin level, located in the deep layers of the
dermis and subcutaneous tissue

63. A LE cell is:

A. Eosinophils modified

B. Small Platelet Count

C. Discoid erythrocytes

D. Mature neutrophils phagocytizing altered nuclear inclusions

E. Ragocyte

64. When the joint function is changed, it changes:

A. Joint circumference

B. Gait

C. Patient Weight

D. Analysis of synovial fluid

E. Patient growth

65. The method of research that reveals flat feet is:

A. Plantography

B. Futoscopy
C. Stopometry

D. Sledoscopy

E. Tonometry

66. Modifiable risk factors for osteopenia syndrome:

A. Female gender, age over 65

B. Caucasian race, family history

C. Anterior fractures, hypogonadism

D. Insufficient intake of calcium from food and vitamin D deficiency

E. Early menopause in women

67. Measuring the circumference of a joint helps to identify:

A. Swelling of the joint

B. Joint deformity

C. Impaired joint function

D. Hypermobility of the joint

E. Crunch and Crepitations

68. Compensatory fractures of the spine are manifested by:

A. There are no manifestations

B. Swelling of the soft tissue of the back

C. Redness of the skin in the paravertebral zone

D. Acute pain in the lower back arising after exercise

E. Constant pain, aggravated at night

69. A synovial biopsy can be used to identify:

A. Tumors, granulomatous diseases

B. Defibration and necrosis of cartilage

C. Quantities of synovial fluid


D. A large number of ragocytes

E. Viscosity reduction

70. In the analysis of synovial fluid take into account:

A. Cytosis, protein content, the presence and nature of salt crystals

B. pronounced nuclear cell pathology

C. a large number of ragocytes

D. white blood cell count

E. number of uniform elements

71. Tendonitis is:

A. Isolated tendon lesion

B. Inflammation of the joint capsule

C. sprain

D. inflammatory lesion of periarticular tissue

E. lesion located in the tendon-muscular ligament area

72. Ligamentitis is:

A. inflammatory lesion of the joint capsule

B. Inflammatory lesion of extra-articular ligaments, most often the ligamentous


canal.

C. sprain

D. Isolated tendon lesion

E. Inflammatory lesion of fascia and aponeurosis

73. Myotendinitis is:

A. lesion located in the region of the tendon-muscular ligament

B. Inflammatory lesion of extra-articular ligaments, most often the ligamentous


canal through which the tendon passes

C. lesion of the joint capsule


D. Isolated lesion of the tendon of the corresponding muscle

E. Inflammatory lesion of periarticular lymph nodes

74. Fibrositis is:

A. Isolated lesion of the tendon of the corresponding muscle

B. lesion of the joint capsule

C. Inflammation of the fascia and aponeuroses

D. inflammatory lesion of extra-articular ligaments, most often the ligamentous


canal through which the tendon passes

E. Sprain

75. Paniculitis is:

A. inflammatory foci in subcutaneous perivascular tissue with the development


of necrosis and sclerosis in them

B. Isolated tendon lesions

C. inflammatory lesion of extra-articular ligaments, most often the ligamentous


canal through which the tendon passes

D. Periarticular lymph node inflammation

E. Sprain

76. The syndrome of diffuse inflammation of the connective tissue is


characterized by:

A. Systemic autoimmune inflammatory lesions of the connective tissue

B. Degenerative lesions of the joints and internal organs

C. Inflammation of the skin and subcutaneous tissue

D. Polyarticular inflammatory lesion type

E. Infectious-inflammatory lesions of the connective tissue

77. The butterfly skin symptom is:

A. Erythematous-squamous foci on the back of the nose, in the brow area and
cheeks
B. allergic skin rash

C. disturbed skin pigmentation in the form of a butterfly

D. violet erythema of the paraorbital region

E. Discoid eruptions

78. A typical manifestation of the outer epicondylitis of the shoulder is

A. Deformity of the elbow joint

B. presence of effusion in the synovial bag of the process of the ulna

C. pain when straightening fingers

D. Pain when bending fingers

E. pain with shoulder rotation

79. The formation of the "snapping" finger due to:

A. Severe osteoarthritis of the hand joints

B. mismatch between the diameter of the fibrous ring and the thickness of the flexor
tendon of the finger

C. nodular tendovaginitis of the flexor of the finger

D. arthritis of the interphalangeal joint of the hand

E. arthritis of the shoulder joint

80. Patient S., 18 years old, complaints of weakness and pain in muscles, fever,
shortness of breath, weight loss of 6 kg. From the anamnesis: after a viral
infection pains in the joints and muscles appeared. Objectively: The condition
is serious. He can not raise his head, stand up, eat on their own. Periorbital
edema, purple spots on the skin of the thighs. Which examination of the above
is most appropriate for verification of the diagnosis?

A. Determination of creatine phosphokinase

B. Determination of rheumatoid factor

C. Determination of antinuclear factor

D. Determination of antibodies to double-stranded DNA


E. Detection of antibodies to topoisomerase (SCL-70)

81. Patient J. 38 years. She has admitted with the complaints of face and arm
skin tightening, weight loss of 12 kg, pain in the joints, difficulty swallowing dry
food, shortness of breath. From the anamnesis: Sick for 5 years, there were
thick edema on the hands and face, arthralgia, and swallowing disorders.
Objective: Low body nutrition. Teleangiectasia on the neck, chest. Masked face.
Restricted mouth. Dummy fingers. The skin in the fold is not taken by
grabbing. What examination is most appropriate for verification of the
diagnosis?

A. Rheumatoid factor

B. Antinuclear antibodies

C. Antibodies to double-stranded DNA

D. Antibodies to topoisomerase (SCL-70)

E. Antibodies to cyclic-citrulinated peptide

82. Patient V., 23 years old. Complaints: pain in the joints, cough, shortness of
breath, rashes on the skin, appearing after exposure to the sun, general
weakness, alopecia, irregular menstruation. From the anamnesis: is ill for the
past six months, started after abortion. Worried about joint pain and
subfebrile. Objectively: General condition of moderate severity. In the area of
the cheekbones and the back of the nose erythema. On the lips - scaly crusts.
What is your presumed syndrome?

A. Syndrome of diffuse lesions of the connective tissue

B. Syndrome of degenerative lesions of the joints

C. Syndrome of soft tissue lesions of the periarticular tissues

D. Osteopenic syndrome

E. Syndrome of inflammatory changes of the joints

83. Joint pain is:

A. Ossalgia

B. Myalgia

C. Arthralgia
D. Tendalgia

E. Odontalgia

84. Dorsalgia is

A. Pain in the joints of the lower limbs

B. Pain in the sacrum

C. Back pain

D. Pain in tailbone

E. Pain in the pelvic region

85. Ossalgia is:

A. Bone pain

B. Muscle pain

C. Pain in the joints

D. Pain in the hands

E. Pain in the whole body

86. The rheumatoid factor is:

A. Subtype of antinuclear antibodies.

B. Antibodies to the Fc fragment of IgG.

C. Antibodies to the components of the synovial membrane of the joints.

D. Immune complex containing IgM class antibodies and IgG Fc fragment.

E. A set of cytokines that play a key role in the pathogenesis of RA.

87. Varus deformity is:

A. O-shaped deformity of the knee joint

B. X-shaped deformity of the knee joint

C. Knee over-bending

D. Z-shaped deformity of the knee joint


E. M-shaped deformity of the knee

88. Valgus deformity is:

A. O-shaped deformity of the knee joint

B. X-shaped deformity of the knee joint

C. Knee over-bending

D. Z-shaped deformity of the knee joint

E. M-shaped deformity of the knee

89. Arthrocentesis is

A. Puncture of the joint

B. Joint prosthetics

C. Measuring the range of motion of the joint

D. Measurement of the circumference of the joint

E. Muscle puncture

90. The manifestation of degenerative changes in which Bouchard's nodules are


found?

A. proximal interphalangeal joints of the hand;

B. Distal interphalangeal joints of the hand;

C. knee joints;

D. first metatarsophalangeal joint;

E. elbow joint.

91. What method is used to evaluate the functions of the wrist?

A. palpation method

B. physical examination

C. Duplex compression method

D. active squeezing fingers

E. uplift
92. Pain of the “mechanical” type, intermittent “blockage” of the joints, slow
progression of the disease, predominant lesion of the stressed joints are
characteristics if which syndrome:

A. Syndrome of inflammatory lesions of the joints

B. Syndrome of diffuse inflammation of the connective tissue

C. Osteopenia syndrome

D. Syndrome of degenerative-dystrophic changes of the joints

E. Reiter's syndrome

93. Increased radiolucency of the bone tissue, underlined contours of the


vertebral bodies, thinning of the cortical layer of the diaphysis of the tubular
bones, what syndrome are we talking about:

A. Syndrome of inflammatory lesions of the joints

B. Syndrome of diffuse inflammation of the connective tissue

C. Osteopenia syndrome

D. Syndrome of degenerative-dystrophic changes of the joints

E. Reiter's syndrome

94. Kushelevsky symptom is characteristic of damage to:

A. the joints of the lumbar spine;

B. pubic symphysis;

C. hip joints;

D. ileosacral articulations;

E. costal sternum

95. What determines Forestier's symptom?

A. mobility of the cervical spine

B. mobility of the lumbar spine

C. mobility of the thoracic spine

D. severity of cervical kyphosis


E. severity of lumbar kyphosis

96. Talalgia is:

A. Pain in the palm

B. Pain in the chest

C. Pain in the scapula

D. Pain in the heel

E. Pain in the patella

97. A patient is 57-years old, had an episode of pain in the area of the hip joint
a month ago when walking. Which of the following testifies in favor of above
symptoms:

A. palpable tenderness in the trochanter area

B. limiting the hip rotation on the affected side

C. shortening the right leg by 4 cm

D. Lassega positive symptom on the right

E. positive Kushelevsky symptom

98. In osteopenia, according to the results of densitometry, the T-criterion is


equal to:

A. "-1" and above

B. "-1" - "-2.5"

C. "-2.5" and below

D. “+1” and above

E. "+2.5" and below

99. The rheumatoid factor is:

A. Macrophage phagocytizing antinuclear antibodies

B. Immunoglobulin M

C. Immunoglobulin E

D. Immunoglobulin A
E. antinuclear factor

100. Rheumatoid factor is determined in the reaction of:

A. Bordet-Zhangu

B. Wasserman

C. Coombs

D. Vaaler-Rose

E. Hedelson Reaction

3
Examination of patients
1. Clinical manifestations of the facies refritica are:
A. pallor and swelling of the face
B. acrocyanosis
C. hemorrhagic rash on face
D. swelling of the neck veins
E. Xanthelasma on face

2. Which of listed scientists made greatest contribution to the development of nephrology?


A. KOH
B. Pasteur
C. Mechnikov
D. Brightz
E. Zakharyin

3. What causes chronic pyelonephritis:


A. gram positive flora
B. Gram-negative floras
C. viruse
D. rickettsia
E. protozoa

4. Under the action of the renin occurs


A. arteriole glomerular contraction
B. expansion of glomerular afferent arterioles
C. Reducing the pore size of the basement membrane
D. hypertrophy of the vascular loops of the glomerulus
E. Reduced protein reabsorption

5. When urinary tract infection syndrome in the sample according to Nechyporenko observe:
A. 1. leukocytes - 1500, E - 800, cyl - 450
B. 2. leukocytes - 2000, E - 1000, cyl - 500
C. 3. leukocytes - 5000, E - 1000, cyl - 500
D. 4. leukocytes - 2000, E - 800, cyl - 250
E. 5. leukocytes - 200, E - 7300, cyl - 450

6. To establish the fact hematuria is used:


A. urine sediment microscopy
B. prednisolone test
C. Zimnitsky's test
D. excretory urogram
E. urine dilution test

7. Effect of antidiuretic hormone is carried out in


A. proximal tubule
B. loop of Henle
C. distal tubules
D. collecting tubule
E. along the entire nephron

8. Bacteriuria is considered to be excretion with urine of opportunistic flora in an amount of


more than:
A. 100 microbial cell per ml
B. 1000 microbial cell per ml
C. 10,000 microbial cell per ml
D. 100,000 microbial cells per ml
E. 1,000,000 microbial cell per ml

9. Acute pain syndrome glomerulonephritis due to:


A. violation of urine outflow;
B. inflammatory edema of the ureter;
C. stretching the renal pelvis;
D. spastic contraction of the ureter;
E. stretching the kidney capsules .

10. When infection of urinary tract affects:


A. vascular and lymphatic system of the kidney
B. tubules, cup and pelvis apparatus, and interstices
C. a ball
D. bladder mucosa
E. RAAS

11. Low urine the specific gravities are indicated as


A. hypostenuria
B. hyperstenuria
C. isostenuria
D. ishuria
E. dysuria

12. the Urinary frequency is called:


A. pollakiuria;
B. nocturia ;
C. polyuria;
D. oliguria ;
E. stranguria .
13. In acute nephritis syndrome affects:
A. kidney interstitial tissue
B. glomerules
C. tubules, cup and pelvis apparatus, and interstitium
D. cortex and medulla of the kidney
E. bladder mucosa

14. What are signs characteristic of acute nephritic syndrome ?


A. swelling , hyperlipidemia
B. hematuria , itchy skin
C. arterial hypertension , pallor of the skin
D. hyperlipidemia , xanthomas , hypotension
E. edema, hematuria, the arterial hypertension

15. In the etiology of acute nephritic syndrome is of the great importance:


A. bacterial infection
B. heavy metal salts
C. unbalanced diet
D. alcohol abuse
E. birth defects of the renal vascular system

16. What syndrome is characterized by radiographic signs: blurred borders of renal pelvis,
deformity of the renal cups, narrowing and retraction of the necks?
A. acute-nephritic
B. urinary tract infections
C. nephrotic
D. hypertension
E. acute renal failure

17. What syndrome involves the detection of leukocyte cylinders in urine ?


A. acute-nephritic
B. urinary tract infections
C. nephrotic
D. hypertension
E. acute renal failure

18. Occurrence of urine leukocyturia and bacteriuria (more than 100,000 bacteria in 1 ml of
urine) suggests that patient has the syndrome:
A. uric
B. acute renal failure
C. chronic renal failure
D. acutely - nephritic
E. urinary tract infections

19. For orthostatic proteinuria, the following is true:


A. Most patients develop chronic renal failure.
B. proteinuria exceeds 2 g / day
C. proteinuria is absent in the horizontal position of the patient
D. proteinuria is always combined with edema
E. proteinuria occurs in patients in old age

20. What syndrome is characterized by the sudden onset of increasing edema, hematuria and
proteinuria, arterial hypertension combined with signs of impaired renal function (decreased GFR,
azotemia):
A. nephrotic
B. acutely - nephritic
C. acute renal failure
D. eclampsia
E. chronic r9enal failure

21. Nephrotic syndrome is talked about in the presence of:


A. hyperproteinemia
B. hypocholesterolemia
C. hypokalemia
D. hypertension
E. proteinuria> 3.5 g / day

22. Note the reasons for the formation of o- oligo anuria and acute renal failure:
A. Increase effective filtration pressure.
B. Reducing effective filtration pressure.
C. Reducing the number of ADH.
D. Increasing the number of ADH.
E. Angiospasm and blockage of tubules with cylinders, blood clots .

23. What syndrome is characterized by a combination of the following symptoms: edema, high
proteinuria, hypoproteinemia , dysproteinemia , hyperlipidemia ?
A. Nephrotic
B. uric
C. hypertensive
D. infectious complications syndrome
E. chronic renal failure

24. Edema in nephrotic syndrome:


A. common
B. local
C. intensify in the evening
D. accompanied by hyperproteinemia
E. pass independently

25. What characterizes an arrester?


A. night diuresis above daytime
B. isostenuria in the absence of azotemia
C. azotemia with isohyposthenuria
D. polyuria with azotemia
E. azotemia with hypostenuria or oligoanuria

26. What pathological processes are the main cause of CRF?


A. hardening of the majority of nephrons
B. necrosis of glomeruli
C. renal artery vasculitis
D. glomerular inflammation
E. tubular dystrophy

27. What pathological processes are the main cause of AKI?


A. hardening of the majority of nephrons
B. tubular necrosis
C. renal artery vasculitis
D. glomerular inflammation
E. tubular dystrophy

28. In the clinical course of acute renal failure produce


A. 1 period
B. 2 periods
C. 3 periods
D. 4 periods
E. 5 periods

29. Indicate the causes of prerenal ARF.


A. Increase in cardiac output
B. Dehydration
C. Hypervolemya
D. urolithiasis disease
E. Hypertensive crisis

30. Indicate the most common causes of anemia in chronic renal failure:
A. metroragia
B. Lack of vit. B-12 and folic acid
C. Chronic hemolysis
D. Reduced erythropoietin synthesis
E. Chronic aluminum intoxication

31. When CRF is available


A. tubular damage
B. glomerular defeat
C. interstitial lesion
D. combined defeat of the glomeruli and tubules
E. defeat pelvis

32. What are the earliest symptoms in the development of CRF?


A. polyuria
B. dyspeptic disorders
C. arterial hypertension
D. swelling
E. anemia

33. What kind of syndrome is a sudden, rapid, but potentially reversible renal dysfunction,
sufficient to lead to the accumulation of nitrogenous slags in body fluids:
A. acute renal failure
B. chronic renal failure
C. nephrotic syndrome
D. eclampsia
E. dysuric syndrome
34. Acute renal failure may result from:
A. significant reduction in renal perfusion
B. History of prolonged arterial hypertension
C. Systemic diseases
D. Metabolic diseases
E. progressive and irreversible destruction of nephrons

35. A sample of Zimnitsky allows you to identify:


A. nocturia
B. leukocyturia
C. calcium ion
D. oxaluria
E. proteinuria

36. Ways of infection of the renal pelvis and interstitium :


A. hematogenous spread
B. lymphogenous spread
C. ascending path
D. contact
E. airborne

37. Units of glomerular filtration:


A. mmol / l
B. ml / s
C. international units
D. ml / min
E. ml / h

38 What is the value of tubular filtration in the norm:


A. 50-60 ml / min
B. 70-80 ml / min
C. 80-120 ml / min
D. 120-160 ml / min
E. more than 160 ml / min

39 Nechiporenko's test allows:


A. determine the value of daily proteinuria
B. determine glomerular filtration
C. determine urine creatinine concentration
D. to determine the amount of leukocyturia and erythrocyturia
E. Evaluate the degree of canalicular reabsorption

40 Kidneys are palpable at :


A. pyelonephritis;
B. hydronephrosis ;
C. glomerulonephritis ;
D. amyloidosis of the kidney;
E. diabetic nephropathy.

41 Determining which indicator is the most informative for detecting nephrotic syndrome:
A. daily proteinuria value
B. the concentration of serum creatinine
C. serum cholesterol concentration
D. Size of canalicular reabsorption
E. uric acid concentration

42 Microalbuminuria is:
A. Albumin daily excretion 1-2 g / day
B. Daily excretion of albumin 30-300 mg / day
C. proteinuria with albumin excretion of 2-3.5 g / day
D. Albumin daily excretion of less than 50 mg / day
E. Daily excretion of albumin more than 5 g / day

43 Brush rim in nephron is available in :


A. Proximal tubule
B. Loop of Henle
C. Distal convoluted tubule
D. Linking department
E. Collecting tube

44. Specify the early sign of chronic renal failure (CRF):


A. Metabolic acidosis
B. Hypercalcemia
C. Nocturia
D. Azotemia
E. Reduced kidney size

45. The leading pathogenetic mechanism in chronic renal failure is:


A. acute kidney ischemia
B. progressive nephron death
C. increase in the mass of active nephrons
D. necrosis of the tubular epithelium
E. ischemia of the cortical layer of the kidney

46. Indicate signs of renal edema.


A. blue, cold, dense
B. blue, warm, soft
C. pale, warm, soft
D. pale, cold, dense
E. red, warm, soft

47. The leading mechanism of edema in nephrotic syndrome is:


A. increased capillary permeability
B. reduction of oncotic pressure of blood plasma
C. Delay in blood and tissue of sodium ions
D. acute urinary excretion of urine
E. increase of hydrostatic pressure in capillaries

48. The leading mechanism of hypertension is:


A. reduced glomerular filtration of water and sodium
B. calcium accumulation in the vascular wall
C. activation of the renin- angiotensin - aldosterone system
D. catecholamine retention
E. decrease in depressor kidney function

49. Swelling of renal origin is most often accompanied by the following coloring of the skin:
A. cyanotic
B. red
C. pale
D. icteric
E. plain, pale pink

50. The main morphological and functional unit of the kidney is:
A. cup and pelvis system
B. vascular glomerulus
C. renal tubules
D. nephron
E. renal body

51. The permissible amount of protein in daily urine is:


A. 500 mg / day
B. 50 mg / day
C. 150 mg / day
D. Should not be
E. traces of protein

52. Urea, creatinine , glucose are filtered in the ball:


A. completely;
B. partially;
C. not filtered;
D. Only glucose is completely filtered.
E. only creatinine is completely filtered

53. The formation of angiotensin II occurs


A. only in the lungs
B. only in the heart
C. kidney only
D. Liver only
E. mainly in the lungs and kidneys

54. Protein reabsorption occurs.


A. in the proximal tubule
B. in the loops of genle
C. in distal convoluted tubule
D. in collecting tubules
E. in bellium duct

55 . What causes edema in nephrotic syndrome?


A. Ischemia of the kidney;
B. Oliguria ;
C. Decreased blood pressure;
D. Reduced oncotic pressure;
E. Increased hydrostatic pressure.

56 . In the study of blood in patients with nephrotic syndrome revealed:


A. Hyperproteinaemia ;
B. Hypoalbuminemia ;
C. Hyperalbuminemia ;
D. Hypolipidemia ;
E. Hyperglycemia.

57 . The signs of nephrotic syndrome include I tsya:


A. Hypertension;
B. Massive swelling;
C. hematuria;
D. Proteinuria> 1.5 g / day ;
E. Leukocyturia .

58. Signs of urinary syndrome:


A. glucosuria ;
B. Proteinuria;
C. Protein Bens- Jones;
D. Bacteriuria;
E. Hiluria.

59. What proteinuria is characteristic of acute-nephritic syndrome?


A. Glomerular ;
B. Tubular;
C. Orthostatic;
D. Proteinuria voltage;
E. Proteinuria overflow.

60. What test can detect a decrease in glomerular filtration?


A. Nechiporenko's test;
B. Test Zimnitsky ;
C. Sample Reberga-Tareeva ;
D. Addis – Kakovsky test ;
E. Pasternatsky test .

61. For nephrotic syndrome is characterized by:


A. The increase in serum VLDL and LDL;
B. Development of hypocoagulation disorders;
C. Hypercalcemia ;
D. Convulsive syndrome;
E. Arterial hypertension.
62. Characteristic hematological disorders in chronic renal failure:
A. Anemia;
B. Erythrocytosis;
C. Leukocytosis;
D. Thrombocytosis;
E. Pancytopenia .

63. What signs carry information about the functional abilities of the kidney?
A. urine protein content;
B. isostenuria ;
C. decrease in glomerular filtration;
D. increased plasma creatinine ;
E. Increased blood pressure.

64. What determines the dark and yellowish color of the skin in chronic renal failure?
A. from increasing direct bilirubin;
B. from increasing indirect bilirubin;
C. violation of urochromic excretion ;
D. from eating dyes;
E. From violation of the synthesis of bilirubin.

65. The nitrogenous function of the kidneys can be judged:


A. by plasma protein level;
B. urine protein level;
C. in terms of urea and plasma creatinine ;
D. on the relative density of urine;
E. according to blood lipids.

66. The predominance of nighttime diuresis over daytime is called:


A. oliguria ;
B. anuria;
C. nocturia ;
D. pollakiuria;
E. stranguria .

67. Criteria for determining chronic renal failure:


A. GFR <100 ml / min / 1.73 m 2 for> 3 months;
B. GFR <60 ml / min / 1.73 m 2 for> 3 months;
C. GFR <30 ml / min / 1.73 m 2 for> 3 months;
D. Renal impairment for> 3 months;
E. Kidney damage for <3 months.

68. For the appearance of a patient with nephrotic syndrome is characterized by:
A. skin color "coffee with milk";
B. bronze skin tone;
C. cyanosis of the lips, earlobes, fingers;
D. severe swelling of the face;
E. pronounced cyanotic swelling of the lower extremities.
69. In CKD C5 note :
A. lubochkov speed to the second filtration and less than 15 ml / min;
B. Poliuri Yu ;
C. Hypokalemia th;
D. Hypercalcemia th;
E. Increase the size of the kidneys on ultrasound.

70. Normal level of potassium in blood serum:


A. 1.5 - 2.5 mmol / l.
B. 2.5-3.5 mmol / l;
C. 3.5-5.5 mmol / l;
D. 5.5-6.5 mmol / l;
E. 6.5-8.5 mmol / l;

71. Normal serum creatinine level:


A. 25-44 µmol / l;
B. 44-105 μmol / l;
C. 130-140 μmol / l;
D. 145-265 μmol / l.
E. 265-618 µmol / L;

72. The leukocyte indices in the urine analysis according to Nechipo p enko are normal :
A. up to 6000 in 1 ml;
B. up to 2000 in 1 ml;
C. up to 4000 in 1 ml;
D. up to 1000 in 1 ml;
E. up to 5000 in 1 ml.

73 . Indicators of e p itycocytes in the analysis of urine according to Nechipopenko are normal :


A. 1000 in 1 ml;
B. 2000 in 1 ml;
C. 4000 in 1 ml;
D. 6000 in 1 ml;
E. 10,000 in 1 ml;

7 4. The most accurate nature of the morphological changes in the kidneys allows you to
determine:
A. intravenous urografiya ;
B. retro pyelography ;
C. ultrasound study;
D. kidney biopsy;
E. Doppler sonography of the kidneys.

75. To determine glomerular filtration, the following method is used:


A. daily diupeza of determination;
B. urine analysis according to Zimnitsky ;
C. definition klipensa of endogenous kpeatininu;
D. Nechiporenko test;
E. determination of daily proteuria .

76. Which of the following pathogens is most often the cause of the development of acute nephritic
syndrome?
A. E. coli;
B. Viruses;
C. B-hemolytic streptococcus group A;
D. Chlamydia and other intermediate forms of bacteria;
E. The simplest.

77. Which of the following forms of acute renal failure (ARF)


is hypercatabolic ?
A. Medicinal
B. Postrenal
C. Crash syndrome
D. Prerenal
E. Hepato-renal syndrome

78 What signs distinguish renal eclampsia from hypertensive crisis?


A. The presence of convulsions
B. Brain edema
C. Lack of convulsive syndrome
D. Severe headaches
E. Blurred vision

79. In 1 5-year-old patient 2 weeks after suffering a sore throat there were edemas, gross
hematuria, increased blood pressure. Most possible diagnosis:
A. Acute - Nephritic Syndrome
B. Urinary syndrome
C. Nephrotic syndrome
D. hypertension syndrome
E. acute renal failure syndrome

80. What methods can be used for the differential diagnosis of transient acute from
chronic irreversible renal failure?
A. Reberg Trial
B. Ultrasound examination of the kidneys
C. Zimnitsky's test
D. Prednisone challenge test
E. Determination of serum urea level

81. Urine color "meat slops" is typical for:


A. acute pyelonephritis;
B. acute glomerulonephritis ;
C. kidney amyloidosis;
D. nephrotic syndrome;
E. chronic pyelonephritis.

82. Aching, dull, bilateral pain in the lumbar region is typical for:
A. kidney infarction;
B. kidney prolapses;
C. urolithiasis;
D. bend of the ureter;
E. acute glomerulonephritis .

83. Acutely emerged headache, increased blood pressure, blurred vision, swelling of the face,
decrease in urine output and change its colors are characteristic:
A. acute pyelonephritis;
B. acute pyelitis;
C. acute glomerulonephritis ;
D. nephrotic syndrome;
E. terminal renal failure

4
PULMONOLOGY
1. Vesicular respiration results from….

A. Air swirls when passing through the glottis

B. Air swirls as it passes through the bronchi

C. Fluctuations of the alveolar walls in the inhalation and exhalation phases

D. Friction of pleural leaves in the inhalation and exhalation phases

E. Air swirls in cavities

2. Bronchial respiration results from….

A. Alveolar dissemination during inhalation

B. Air swirls when passing through the glottis

C. Fluctuations of the alveolar walls in the inhalation and exhalation phases

D. Friction of pleural leaves in the inhalation and exhalation phases

E. Air swirls in the nasal cavity

3. Identify the most characteristic changes in the chest with inflammatory compaction of the lung
lobe:

A. Reduction of half of the chest, its depression and lag in breathing


B. lag in breathing, an increase in the half of the chest and smoothing of the intercostal spaces

C. only lag in breathing of the chest

D. hypersthenic chest

E. an increase in anteroposterior and lateral dimensions of the chest, retraction of the intercostal
spaces in the lower lateral regions on both sides

4. What percussion sound appears when inflammatory compaction of the lung tissue?

A. blunt or dull

B. clear pulmonary

C. Tympanic

D. tympanic blunting

E. boxed

5. What is the purpose of using an additional technique for auscultation of the lungs - coughing?

A. to distinguish pleural friction noise from crepitus and wheezing

B. to detect hidden bronchial obstruction

C. to distinguish dry rales from wet rales

D. To distinguish wheezing from crepitus

E. for better listening for pathological bronchial respiration

6. What is the main respiratory noise most often heard with lobar inflammatory compaction?

A. weakened vesicular respiration

B. Amphora breath

C. bronchial respiration

D. hard breathing

E. mixed bronchovesicular respiration

7. What caused the appearance of crepitus?

A. the presence in the alveoli (near-wall) of a small amount of exudate or transudate


B. inflammation of the pleura (“dry” pleurisy)

C. Alveoli are completely filled with exudate or transudate.

D. viscous bronchial secretion in large bronchi

E. viscous bronchial secretion in the small bronchi and / or their spasm

8. Crepitus occurs in:

A. Alveoli

B. Small bronchi

C. The middle bronchi.

D. Large bronchi.

E. Bronchioles.

9. What sputum often occurs in patients with the syndrome of lobar compaction of the lung
tissue?

A. Vitreous

B. Muco-purulent

C. Three-layer

D. "Rusty."

E. Serous, frothy pink.

10. What is detected by X-ray examination of the lungs in lung tissue induction syndrome?

A. Fractional darkening of the lung tissue.

B. Increased pulmonary transparency

C. Lobular lumen tissue enlightenment.

D. The presence of a rounded focus of enlightenment with a fluid level.

E. Darkening of the course of the interlobar sulcus.

11. What is the difference between crepitus and finely bubbling moist rales?
A. occurs while exhaling;

B. occurs while inhaling;

C. occurs during inhalation and exhalation;

D. increases with forced breathing

E. increases after coughing

12. Select the symptom complex characteristic of the syndrome of inflammatory consolidation of
lung tissue:

A. chest pain radiating to the left arm

B. cough with a mouthful of sputum, mainly in the morning

C. fever, chills, cough with "rusty" phlegm

D. choking attacks with distant wheezing

E. swelling in the legs, liver enlargement

13. Identify the most characteristic changes in the chest with obstructive atelectasis:

A. Reduction of half of the chest, its depression and lag in breathing

B. lag in breathing, an increase in the half of the chest and smoothing of the intercostal spaces

C. only lag in breathing of the chest

D. Hyperstenotic chest

E. an increase in anteroposterior and lateral dimensions of the chest, retraction of the intercostal
spaces in the lower lateral regions on both sides

14. Under which syndrome is voice tremor enhanced?

A. Seals lung tissue

B. Enhancement of airiness of lung tissue

C. Accumulations of fluid in the pleural cavity

D. The presence in the lung cavity, not communicating with the bronchus

E. Accumulations of air in the pleural cavity


15. What is the main respiratory noise most often heard in the presence of a cavity that connects
to the bronchus?

A. weakened vesicular respiration

B. bronchial respiration

C. amphoric breathing

D. hard breathing

E. Mixed bronchovesicular respiration

16. Strengthening vocal tremor, tympanic sound, bronchial, sometimes amphoraic breathing, an
abundance of moist sonorous wheezing in a limited area, profuse purulent sputum observed in
the syndrome:

A. increased airiness of the lungs

B. sealed lung tissue

C. the presence of fluid in the pleural cavity

D. availability of air in the pleural cavity

E. cavity in the lung

17. Characteristics of sputum for cavity syndrome in the lung tissue:

A. Abundant, goes with full mouth, with an unpleasant smell mixed with blood, purulent,
double-layered.

B. Goes in a large amount in the mornings, mucopurulent character.

C. Scanty, viscous.

D. Rusty, mucopurulent, odorless.

E. Muco-purulent in a small amount without admixture of blood.

18. What caused the appearance of moist, large wheezing?

A. viscous bronchial secretion in large bronchi

B. viscous bronchial secretion in the small bronchi and / or their spasm


C. liquid bronchial secretion in the large bronchi or cavities communicating with the
bronchus

D. Liquid bronchial secretion in the small bronchi while maintaining the airiness of the
surrounding lung tissue

E. Liquid bronchial secretion in the small bronchi and inflammatory sealing of the surrounding
lung tissue

19. Which of the sputum elements reliably indicates the destruction of lung tissue?

A. Charcot-Leiden crystals;

B. leukocytes;

C. Kurshman spiral;

D. elastic fibers;

E. red blood cells.

20. Give the characteristic of wet wheezing:

A. are heard on the inhale and exhale, crackling or “bursting” bubbles, disappear with
imitation of respiratory movements

B. are heard on the inhale, the sound "f", over all the pulmonary fields, disappears when
imitating respiratory movements

C. are heard on the inhale and exhale over the larynx, the sound "f”, disappears when imitating
respiratory movements

D. are heard on the inhale and exhale above the lung fields, the sound "f", disappears when
imitating respiratory movements

E. listened only on inspiration, crackling or creaking of snow, disappears on inspiration

21. The most informative method for diagnosing the presence of cavity syndrome in the lung:

A. blood test

B. sputum analysis

C. spirometry

D. x-ray
E. bronchophony

22 The syndrome of bronchiectasis is characterized by the presence of:

A. caverns in the lung tissue

B. bronchial tumors

C. accumulations of purulent sputum in the enlarged areas of the bronchi

D. Fluid accumulation in the pleural cavity

E. thin-walled closed cavities in the lung

23. What is the change in respiration most characteristic of the spasm of the small bronchi?

A. wheezing breath

B. expiratory dyspnea

C. Kussmaul breathing

D. Cheyne-Stokes breathing

E. inspiratory dyspnea

24. What caused dry buzzing (bass) wheezing?

A. the presence in the alveoli (near-wall) of a small amount of exudate or transudate

B. inflammation of the pleura (“dry” pleurisy)

C. alveoli are completely filled with exudate or transudate.

D. viscous bronchial secretion in large bronchi

E. viscous bronchial secretion in the small bronchi and / or their spasm

25. What is the purpose of using an additional technique for auscultation of the lungs - forced
expiration?

A. to distinguish the pleural friction noise from crepitus and wheezing

B. to detect hidden bronchial obstruction

C. in order to distinguish dry rales from wet rales

D. to distinguish wheezing from crepitus or pleural friction


E. for better listening for pathological bronchial respiration

26. For bronchial obstruction syndrome, the most characteristic auscultation signs are:

A. bronchial respiration and crepitus

B. breathing with prolonged expiration, the appearance or strengthening of wheezing


during forced expiration

C. vesicular breathing and wheezing

D. weakened vesicular respiration

E. bronchovesicular respiration with prolonged inhalation

27. The functional symptom of bronchial obstruction is:

A. change in tidal volume

B. decrease Tiffno index

C. decrease VC

D. increase residual volume

E. increase in MSV

28. The mechanism of dry wheezing is associated with the following factors:

A. Accumulation of fluid secretion in the lumen of the bronchi

B. Accumulation of viscous secretion in the lumen of the bronchi

C. Fluid accumulation in the pleural cavity

D. The surface roughness of the pleura

E. Accumulation of fluid secretion in the lumen of the alveoli

29. What are the distinguishing features of wet rales:

A. Listened to both phases of respiration.

B. Listened to at the height of breath.

C. The character of the sound changes after changing the position of the body.

D. The character of the sound does not change after coughing.

E. Listened better with tighter stethoscope


30. Asphyxiation is:

A. subjective feeling of lack of air, accompanied by a change in the frequency, depth and rhythm
of breathing

B. arbitrary or involuntary reflex act of protecting and self-cleaning the respiratory tract from
foreign bodies, mucus, pus and sputum

C. excretion of blood in sputum

D. a sudden onset of lack of air

E. agonizing "barking" cough

31. The peculiarity of cough in bronchial obstruction syndrome:

A. mucus or purulent sputum in the morning

B. simultaneous discharge of purulent sputum in large quantities

C. agonizing cough with scanty sputum

D. cough with a "rusted" sputum

E. cough with frothy pink sputum

32. What syndrome is characterized by mucous vitreous sputum with the discovery of
eosinophils, Kurshman spirals and Charcot-Leiden crystals in it?

A. pleura inflammation syndrome

B. pulmonary tissue compaction syndrome

C. cavity in lung tissue syndrome

D. syndrome of air in the pleural cavity

E. bronchial obstruction syndrome

33. What percussion sound appears when emphysema?

A. blunt or dull

B. clear pulmonary
C. tympanic

D. tympanic blunting

E. boxed

34. Indicate the most characteristic changes in the chest during emphysema of the lungs:

A. reduction of half of the chest, its depression and lag in breathing

B. lag in breathing, an increase in the half of the chest and smoothing of the intercostal spaces

C. only lag in breathing half of the chest

D. hypersthenic chest

E. an increase in the anteroposterior and lateral dimensions of the chest, the involvement of
the intercostal spaces in the lower lateral regions on both sides during inhalation.

35. The leading complaint of patients with lung tissue airiness is:

A. Cough

B. Dyspnea

C. Chest Pain

D. Hemoptysis.

E. Increased body temperature

36. With changes in what structures in the lung is associated the development of emphysema?

A. By overstretching the elastic fibers in the interstitium of the lung

B. Change in muco-ciliary index

C. Blood overflow of the cavernous venous plexus of the lower

nasal passage

D. Spasm of the muscle plate of the mucous membrane of the small bronchi

E. Violation of the synthesis of surfactant

37. Dyspnea is:


A. Subjective feeling of lack of air, accompanied by a change in the frequency, depth and
rhythm of breathing

B. arbitrary or involuntary reflex act of protection and

self-cleaning of the respiratory tract from foreign bodies, mucus, pus and

sputum

C. shallow breathing

D. a sudden onset of lack of air

E. sudden loss of consciousness

38. When conducting comparative percussion of the lungs, the doctor determined a boxed sound
over both lungs, the lower border of the lungs is lowered by 3 cm, and the active mobility of the
pulmonary edges is limited. What syndrome are we talking about?

A. pleural inflammation syndrome

B. lung tissue induction syndrome

C. cavity in lung tissue syndrome

D. syndrome of air in the pleural cavity

E. syndrome of increased pulmonary airiness

39. Predisposition to the development of increased airiness of lung tissue causes:

A. α-1-antitrypsin deficiency

B. excess α-1-antitrypsin

C. excess collagen

D. lack of creatine phosphokinase

E. excess phospholipase

40. What caused the appearance of pleural friction noise?

A. the presence in the alveoli (near-wall) of a small amount of exudate or transudate


B. inflammation of the pleura (“dry” pleurisy)

C. alveoli are completely filled with exudate or transudate.

D. viscous sputum in large bronchi

E. viscous sputum in the small bronchi and / or their spasm

41. Indicate the most characteristic changes in the chest with a closed pneumothorax:

A. a decrease in half of the chest, its depression and lag in breathing

B. lag in breathing, an increase in half of the chest and intercostal space leveling

C. only lag in breathing half of the chest

D. hyperstenotic chest

E. an increase in the anterior-posterior and transverse dimensions of the chest, intercostal spacing
in the lower side of both parties

42. What is the main breathing noise most often heard in a closed pneumothorax?

A. weakened vesicular respiration

B. amphora breath

C. bronchial respiration

D. hard breathing

E. mixed bronchovesicular respiration

43. Indicate the most characteristic changes in the chest with unilateral hydrothorax:

A. a decrease in half of the chest, its depression and lag in breathing

B. lag in breathing, an increase in half of the chest and intercostal space leveling

C. only lag in breathing half of the chest

D. hyperstenotic chest

E. an increase in the anterior-posterior and transverse dimensions of the chest, intercostal spacing
in the lower side of both parties

44. What percussion sound appears when pneumothorax:


A. blunt or dull

B. clear pulmonary

C. tympanic

D. tympanic blunting

E. boxed

45. What is the main respiratory noise most often heard in hydrothorax:

A. weakened vesicular respiration

B. amphora breath

C. bronchial respiration

D. hard breathing

E. mixed bronchovesicular respiration

46. What is the difference between pleural friction noise and fine wheezing?

A. amplified in a horizontal position

B. is heard only on inhale;

C. heard only on exhale;

D. does not change with coughing;

E. changes with coughing.

47. Evaluate the result of the study of pleural contents: relative density - 1027, turbid liquid,
greenish-yellow, protein - 60 g / l, Rivalt sample ++++, sediment microscopy - neutrophils cover
all fields of view:

A. transudate;

B. purulent exudate;

C. hemorrhagic exudate;

D. serous exudate

E. chylous exudate
48. X-ray picture in the presence of fluid in the pleural cavity:

A. homogeneous darkening of the lower part of the lung with the Damozo line.

B. increased transparency of the lung fields.

C. light pulmonary field without pulmonary pattern.

D. spotty darkening of the pulmonary field.

E. darkening of the interlobar groove in the form of a spindle.

49. In the syndrome of fluid accumulation in the pleural cavity over the area of fluid
accumulation is noted:

A. tympanic sound.

B. sound box.

C. pulmonary sound.

D. metallic tone tympanic sound

E. blunt or dull

50. Radiographic signs of syndrome of the presence of air in the pleural cavity:

A. homogeneous darkening of the lower part of the lung.

B. Increased transparency of the lung fields.

C. light pulmonary field without pulmonary pattern.

D. spotty darkening of the pulmonary pattern.

E. darkening along the interlobar pleura in the form of a spindle.

51. Mediastinum in the syndrome of fluid presence in the pleural cavity:

A. shifted to a healthy side

B. shifts to the affected side

C. does not shift anywhere

D. shifts down

E. shifts up
52. Mediastinum in the syndrome of air presence in the pleural cavity:

A. shifts to the healthy side

B. shifts to the sick side

C. does not shift anywhere

D. shifts down

E. shifts up

53. Tympanic percussion sound over the chest can be detected in the syndrome ....

A. sealed lung tissue

B. fluid accumulations in the pleural cavity

C. air accumulations in the pleural cavity

D. increase the airiness of the lung tissue

E. formation in the lung cavity, not communicating with the bronchus

54. A characteristic change in the composition of the blood during respiratory failure is:

A. anemia

B. hyperazotemia

C. hypoxemia

D. hyperlipidemia

E. hypoproteinemia

55. Obstructive type of respiratory failure syndrome occurs when:

A. bronchospasm.

B. hydrothorax.

C. pneumosclerosis.

D. pneumothorax.

E. kyphoscoliosis
56. The compensatory mechanisms for respiratory failure include:

A. lymphocytosis

B. thrombocytosis

C. erythrocytosis

D. eosinophilia

E. leukopenia

57. When respiratory failure III degree, the level of SpO2 is:

A.> 90%

B. 90-94%

C. 75-89%

D. <75%

E. <60%

58. The main causes of the development of the hypoxemic form of respiratory failure:

A. Diseases directly affecting the pulmonary parenchyma

B. Mechanical defects of muscle and rib cage of the chest

C. obesity

D. Violation of the regulatory functions of the respiratory center

E. Bronchial obstruction

59. What sign revealed during the general examination indicates the presence of respiratory
failure:

A. change in the shape of the chest

B. pallor of the skin

C. diffuse cyanosis

D. facial flushing

E. erythematous rash
60. The first degree of chronic respiratory failure is characterized by:

A. lack of shortness of breath on exertion

B. shortness of breath with considerable physical exertion

C. shortness of breath with little exertion and inclusion

compensatory mechanisms alone

D. dyspnea and cyanosis alone

E. dyspnea and cyanosis in sleep

61. Specify the place of the best listening valve of the pulmonary artery:

A. second intercostal space on the left at the sternum

B. second intercostal space on the right at the sternum

C. in the fifth intercostal space on the right

D. at the base of the xiphoid process of the sternum

E. in the fifth intercostal space on the left

62. How will II of the heart tone change with an increase in pressure in the pulmonary artery and
severe right ventricular hypertrophy?

A. weakening of the II tone on the pulmonary artery

B. only accent II of the pulmonary artery.

C. accent and splitting II tone on the pulmonary artery

D. only splitting of the II tone on the pulmonary artery

E. amplification of I tone over the pulmonary artery

63. A complication of the cardiovascular system that develops as a result of prolonged chronic
respiratory failure is:

A. arterial hypertension;

B. right ventricular hypertrophy;

C. hypotension
D. left ventricular hypertrophy

E. dilatation of the left atrium

64. A characteristic radiological feature of the pulmonary heart is:

A. expansion of the heart to the right due to the pancreas

B. expansion of the heart to the left due to LV

C. mediastinal displacement to the right

D. mediastinal dislocation to the left

E. thickening of the pulmonary pattern

65. Which of the following instrumental methods of investigation can confirm pulmonary arterial
hypertension:

A. bronchography

B. fluorography

C. ECG

D. ECHO

E. spirometry

66. What level of systolic pressure in the pulmonary artery is considered normal:

A. 25-30mmHg

B. 50-60 mm Hg

C. 100-200mm water column

D. 25-30mm water column

E. 70-80 mm Hg

67. Coughing is:

A. subjective feeling of lack of air, accompanied by a change in the frequency, depth and rhythm
of breathing

B. arbitrary or involuntary reflex act of protection and self-cleaning of the respiratory


tract from foreign bodies, mucus, pus and sputum
C. excretion of blood in sputum

D. a sudden onset of lack of air

E. sputum mucus or purulent character during the day

68. Hemoptysis is:

A. subjective feeling of lack of air, accompanied by a change in the frequency, depth and rhythm
of breathing

B. arbitrary or involuntary reflex exhalation, act of protection and self-cleaning of the respiratory
tract from foreign bodies, mucus, pus and sputum

C. excretion of blood in sputum

D. a sudden onset of lack of air

E. agonizing "barking" cough

69. What respiratory sounds are heard in the lung cavity formation syndrome with the level of
fluid communicating with the bronchus?

A. vesicular breathing

B. crepitus

C. bronchial breathing

D. wheezing dry

E. wheezing moist sonorous, large and medium bubble

70. What is the distinguishing feature of transudate?


A. Fluid density above 1015.
B. turbid liquid.
C. Protein content is more than 3%.
D. Trial Rivalta negative.
E. Trial Rivalta positive.

71. Name the syndrome in which on both sides of the symmetrical parts of the chest are
determined: weakened vocal tremor, boxed sound, weakened vesicular breathing.
A. Accumulation of fluid in the pleural cavity.
B. Accumulation of air in the pleural cavity.
C. Increased airiness of the lung tissue.
D. Compaction of the lung tissue.
E. Presence in the lung cavity, communicating with the bronchus.

72. In a patient in the area of the projection of the lower lobe of the right lung, enhanced voice
trembling, dull percussion sound, bronchial breathing, enhanced bronchophony are determined.
What syndrome are we talking about?
A. Accumulation of fluid in the pleural cavity.
B. Accumulation of air in the pleural cavity.
C. Increased airiness of the lung tissue.
D. Compaction of the lung tissue.
E. Education in the lung cavity, communicating with the bronchus.

73. Name the syndrome in which over one half of the rib cage the vocal trembling, the tympanic
sound is sharply weakened, the respiratory sounds are not heard.
A. Compaction of the lung tissue.
B. Increased airiness of lung tissue.
C. Accumulation of fluid in the pleural cavity.
D. Accumulation of air in the pleural cavity.
E. Education in the lung cavity, communicating with the bronchus.

74. Name the syndrome in which in the area of the projection of the lobe of the lung are
determined: increased voice tremor, dull sound, weakened vesicular breathing, crepitus.
A. Increased airiness of the lung tissue.
B. Compaction of the lung tissue.
C. Accumulation of fluid in the pleural cavity.
D. Education in the lung cavity, communicating with the bronchus.
E. Accumulation of air in the pleural cavity.

75. Name the syndrome in which both halves of the chest reveal: weakened voice trembling,
boxed sound, weakened vesicular breathing and dry "wheezing" wheezing, mainly during the
extended expiratory phase.
A. Compaction of the lung tissue.
B. Education in the lung cavity, communicating with the bronchus.
C. Violation of bronchial patency (broncho-obstructive).
D. Accumulation of fluid in the pleural cavity
E. Accumulation of air in the pleural cavity

76. A patient in the area of the projection of the lower lobe on the right sharply weakened vocal
tremor, with percussion dull sound, breathing noises are not heard. Name the syndrome.
A. Syndrome seal lung tissue.
B. Syndrome of air accumulation in the pleural cavity.
C. Syndrome of fluid accumulation in the pleural cavity
D. Syndrome violation of the patency of the bronchi.
E. Syndrome of increasing the airiness of the lung tissue.
77. Name the syndrome in which vocal tremor, tympanic sound, amphoraic breathing and moist
large-bubbly sonorous rales are enhanced in a limited area in the projection of the upper lobe of
the lung.
A. Syndrome seal lung tissue.
B. Syndrome violation of the patency of the bronchi.
C. Syndrome of fluid accumulation in the pleural cavity.
D. Syndrome of education in the lung cavity, communicating with the bronchus.
E. Syndrome of increasing the airiness of the lung tissue.

78. Name the syndrome in which unilateral recession of the supra- and subclavian fossae is
detected, voice tremor is not performed in the projection of the upper lobe, a dull percussion
sound and breathing is not heard.
A. Syndrome of fluid accumulation in the pleural cavity.
B. Syndrome compression atelectasis.
C. Obstructive atelectasis syndrome.
D. Syndrome violation of bronchial patency.
E. Syndrome of forming cavity in the lung, communicating with the bronchus.

79. The vital capacity of the lungs is ....


A. The volume of air that a person is able to exhale during the deepest breath after the
maximum inhalation
B. Volume of inhaled and exhaled air with quiet breathing
C. Air volume, determined at the fastest forced expiration
D. Breathing volume multiplied by breathing rate
E. Volume of air exhaled in the first second

80. Pulse oximetry is ...


A. Invasive method for determining blood gas composition
B. Non-invasive method for determining blood gas composition
C. Method for the determination of blood oxygen saturation
D. Method for determination of blood saturation with carbon dioxide
E. Method for determining respiratory rate in one minute

81. A pronounced decrease in FVC and a decrease in FEV1 is observed when ...
A. Syndrome of forming cavity in the lung, communicating with the bronchus.
B. Syndrome of fluid accumulation in the pleural cavity
C. Syndrome lung tissue compaction
D. Syndrome violation of bronchial patency
E. Syndrome of air accumulation in the pleural cavity

82. The most sensitive parameter for the determination of bronchial obstruction during
spirometry is ...
A. FEV1
B. FVC
C. VC
D. MVV
E. LVV

83. Peak flow measurement allows you to measure ....


A. Maximum inspiratory speed
B. peak expiratory flow rate
C. Forced expiration volume in the first second
D. VC
E. MVV

84. In which syndrome the main complaint is a cough with a separation of purulent and
sometimes offensive sputum in a daily amount of more than 100ml:
A. When focal compaction of the lung tissue
B. With a fractional compaction of the lung tissue.
C. In the presence of fluid in the pleural cavity.
D. If there is a cavity in the lung after opening.
E. With bronchial obstruction syndrome.

85. What syndrome is characterized by attacks of expiratory dyspnea:


A. seals lung tissue
B. the presence of air in the pleural cavity.
C. the presence of fluid in the pleural cavity.
D. having a cavity in the lung.
E. bronchial obstruction.

86. The patient complains of shortness of breath. On examination, it looks like a pink puffer. The
rib cage bulges in the lower side regions, the intercostal spaces are wide, the course of the ribs
approaches the horizontal, and the supraclavicular fossae are erupted. With topographic
percussion, the lower boundaries of the lungs are shifted down one edge. What syndrome are we
talking about:
A. syndrome of increased airiness of the lung tissue
B. lung tissue induction syndrome
C. syndrome of air in the pleural cavity.
D. syndrome of fluid in the pleural cavity.
E. Lung cavity syndrome.

87. A patient has a complaint of cough with a separation of mucopurulent sputum, shortness of
breath on exertion. During auscultation over the entire surface of the lungs is determined hard
breathing, dry buzzing and whistling wheezing. What syndrome are we talking about:
A. compaction syndrome of the lung tissue
B. syndrome of air in the pleural cavity.
C. syndrome of fluid in the pleural cavity.
D. Lung cavity syndrome
E. bronchial obstruction syndrome

88. The patient complains of fever up to 39 ° C, chills, heaviness in the left half of the chest,
shortness of breath. With topographic percussion, the lower limit of the left lung passes through
the line of Demoiso. Below the line of the Demoiso on the affected side is determined the
absence of voice tremor. During auscultation in this zone, breathing is not heard. What syndrome
are we talking about:
A. syndrome of increased airiness of the lung tissue
B. lung tissue induction syndrome
C. syndrome of air in the pleural cavity.
D. syndrome of fluid in the pleural cavity.
E. Lung cavity syndrome.

89. A patient coughing out 300 ml of purulent sputum, which, upon standing, was divided into
three layers. What syndrome are we talking about?
A. syndrome of increased airiness of the lung tissue
B. lung tissue induction syndrome
C. syndrome of air in the pleural cavity.
D. syndrome of fluid in the pleural cavity.
E. Lung cavity syndrome.

90. The patient felt a sudden pain in the left side of the chest. On examination, the left half of the
chest bulges, lags behind in the act of breathing. Voice trembling on the left is amplified, with
percussion a tympanic sound is determined. What pathology are we talking about?
A. syndrome of increased airiness of the lung tissue
B. lung tissue induction syndrome
C. syndrome of air in the pleural cavity.
D. syndrome of fluid in the pleural cavity.
E. Lung cavity syndrome.

91. The patient complains of fever up to 39.6 ° C, chills, pain in the right half of the chest,
shortness of breath, mostly dry cough. On examination, there is a blush on the right cheek,
herpeslabialis, the right half of the chest lags behind in the act of breathing, voice tremor is
enhanced in the lower parts of the right lung. At auscultation in the same area crepitus is heard.
What syndrome are we talking about:
A. syndrome of increased airiness of the lung tissue
B. Syndrome compaction of the lung tissue
C. syndrome of air in the pleural cavity.
D. syndrome of the presence of bronchiectasis in the lung.
E. Lung cavity syndrome.

92. Sputum has a purulent character, three-layer, microscopic examination reveals a large
number of leukocytes, red blood cells, elastic fibers. Which of the following syndromes is
characterized by such sputum:
A. syndrome of increased airiness of the lung tissue
B. lung tissue induction syndrome
C. syndrome of air in the pleural cavity.
D. syndrome of fluid in the pleural cavity.
E. Lung cavity syndrome.
93. In the study of respiratory function is determined:
FVC - 70%; FEV1 - 45%; Tiffno index - 64%. In which syndrome observed such changes:
A. compaction syndrome of the lung tissue
B. syndrome of air in the pleural cavity.
C. bronchial obstruction syndrome
D. syndrome of fluid in the pleural cavity.
E. Lung cavity syndrome

94. When does pathological bronchial respiration occur?


A. in the syndrome of violation of bronchial patency
B. compaction syndrome of the lung tissue
C. In the syndrome of increased airiness of the lung tissue
D. In the syndrome of fluid accumulation in the pleural cavity
E. With gas accumulation syndrome in the pleural cavity

95. What method of diagnosis of bronchiectasis of the following is the leading one:
A. radiography
B. spirometry
C. bronchoscopy
D. bronchography
E. fluorography

96. The feature of pain syndrome in dry pleurisy:


A. short-term
B. long
C. is associated with coughing and breathing
D. stopped by nitroglycerin
E. is associated with physical activity

97. X-ray signs of the lung tissue compaction syndrome are:


A. the absence of pulmonary pattern, the presence of gas in the pleural cavity
B. The presence of a rounded shade with fluid level
C. the presence of inhomogeneous non-intensive shadow within a fraction without clear
boundaries
D. the presence of a homogeneous intense shadow with distinct boundaries
E. increased transparency of the lung fields

98. Examination and palpation of the heart area of a patient with a chronic pulmonary heart can
reveal the following abnormalities:
A. heart "hump";
B. precordial and epigastric pulsation;
C. spilled, shifted to the left apical impulse;
D. Systolic tremor along the left edge of the sternum
E. keeled chest
99. Which examination is decisive in addition to the clinical criteria for making a diagnosis of
lung tissue compaction syndrome:
A. respiratory function
B. bronchoscopy
C. X-ray of the chest
D. complete blood count
E. sputum analysis

100. With chronic pulmonary heart notes


A. Left and right ventricular hypertrophy
B. dilatation of the right ventricle and left atrial hypertrophy
C. Thickening of the interventricular septum and right atrium
D. hypertrophy and dilatation of the left heart
E. hypertrophy and dilatation of the right heart

101. The most reliable method for the diagnosis of bronchiectasis of the following is
A. fibrobronchoscopy
B. spirography
C. High Resolution Computed Tomography
D. open lung biopsy
E. X-ray

102. Indicate the objective characteristic characteristic of pulmonary hypertension:


A. Accent II tone over the pulmonary artery
B. systolic murmur at the base of the xiphoid process
C. severe shortness of breath
D. enlarged liver
E. peripheral edema

103. Pleural friction noise occurs when:


A. the presence in the alveoli (near-wall) of a small amount of exudate or transudate;
B. inflammation of the pleura (“dry” pleurisy);
C. Filling the alveoli with full exudate or transudate;
D. Cluster of viscous mucus in the large bronchi;
E. Cluster of viscous mucus in the small bronchi and / or their spasm.

104. What changes can be found on palpation of the chest above the fluid during exudative
pleurisy?
A. palpable sensation of pleural friction noise;
B. weakening of voice jitter;
C. weakening or disappearance of voice jitter;
D. a slight increase in voice jitter.
E. voice tremor does not change

105. What are the data auscultation with exudative pleurisy above the fluid?
A. bronchial respiration;
B. vesicular respiration;
C. vesicular respiration weakened or absent;
D. amphoric respiration;
E. pleural friction noise.

106. For which syndrome is hemoptysis most characteristic?


A. syndrome of increased airiness of the lung tissue
B. syndrome of fluid accumulation in the pleural cavity
C. syndrome of accumulation of fluid and gas in the pleural cavity
D. lung cavity syndrome
E. bronchial obstruction syndrome

107. What is the pathological type of the chest is characterized by a pronounced elongation of
the anteroposterior size due to the protruding sternum?
A. emphysematous
B. paralytic
C. rachitic
D. Funnel chest
E. navicular thorax

108. At what pathological type of the chest is marked swelling of the posterolateral regions,
expansion of the intercostal spaces, participation of the auxiliary muscles in the act of breathing?
A. emphysematous
B. paralytic
C. rachitic
D. Funnel chest
E. navicular thorax

109. What is the location of the lower border of the right lung in the mid-clavicular line in a
healthy person of normostenic body type:
A. YI rib
B. YII rib
C. YIII rib
D. IX rib
E. X rib

110. What is the location of the lower border of the lung in the mid-axillary line in a healthy
person of the normostenic body type:
A. YI rib
B. YII rib
C. YIII rib
D. IX rib
E. X rib

111. A 25-year-old patient turned to a family doctor with complaints of coughing with a
discharge of a difficult-to-separate sputum of mucopurulent character, fever up to 38 degrees C.
From the anamnesis: she became ill acutely after hypothermia. When auscultation listened finely
wheezing left in the subscapularis area. There is also determined enhanced bronchophony and
dullness of percussion sound. In general blood analysis is marked leukocytosis and accelerated
ESR. Determine the syndrome:
A. Syndrome of increased airiness of the lung tissue
B. Syndrome lung tissue seal
C. Syndrome of air in the pleural cavity.
D. The syndrome of the presence of bronchiectasis in the lung.
E. Syndrome of the cavity in the lung.

112. A patient of 50 years old, for 2 weeks, notes weakness, pain in the right-hand chest. Fever,
cough with purulent sputum. On the radiograph in the upper lobe on the right is determined by a
rounded formation of 3 cm in diameter, filled with liquid. The most likely diagnosis:
A. Syndrome of increased airiness of the lung tissue
B. Syndrome lung tissue seal
C. Syndrome of air in the pleural cavity.
D. Syndrome of fluid in the pleural cavity.
E. Syndrome of the cavity in the lung.

113. A characteristic radiological sign of emphysema is:


A. Strengthening of the vascular pattern in the lungs
B. depletion of vascular pattern in the lungs, the expansion of the intercostal spaces
C. raising the diaphragm, lowering the transparency of the lung tissue
D. reduction of intercostal spaces
E. compaction of the roots of the lungs

114. Pleural friction noise ...


A. listens more often on expiration
B. listened during inhalation and exhalation.
C. weakens after coughing
D. worse after coughing
E. Bugs only while inhaling.

115. A 22-year-old man turned to the emergency department complaining of the appearance of
severe shortness of breath after being kicked in the side. BH - 40 in 1 minute. Body temperature
is normal. With auscultation - easing breathing on the right. Estimated diagnosis:
A. Syndrome of increased airiness of the lung tissue
B. Syndrome lung tissue seal
C. Syndrome of air in the pleural cavity.
D. Syndrome of fluid in the pleural cavity.
E. Syndrome of the cavity in the lung.

116. Specify the main mechanism of the appearance of pathological bronchial respiration:
A. reduced lung tissue elasticity
B. conduction of laryngo-tracheal respiration to the surface of the chest (with a change in
its timbre) during compaction of the lung or the presence of a cavity in it connected to the
bronchus
C. bronchoconstriction (spasm, viscous sputum)
D. the presence of a small center of compaction of the lung tissue, surrounded by unchanged
alveoli
E. increased alveolar wall vibration during breathing

117. Specify the main mechanism of the appearance of hard breathing:


A. reduced lung tissue elasticity
B. Laryngotracheal breathing on the surface of the thorax (with a change in its timbre) lung
consolidations or the presence of a cavity in it connected to the bronchus
C. bronchoconstriction (spasm, viscous bronchial secretion)
D. the presence of a small center of compaction of the lung tissue, surrounded by unchanged
alveoli
E. increased alveolar wall vibration during breathing

118. Respiratory rate in a healthy person alone is:


A. 10-12 respiratory movements per minute
B. 16-20 respiratory movements per minute
C. 20-25 breaths per minute
D. 25-30 respiratory movements per minute
E. More than 30 respiratory movements per minute

119. Give the characteristic of breathing in inspiratory dyspnea:


A. constant amplitude
B. Superficial
C. difficult and prolonged breathing
D. difficult and prolonged exhalation
E. difficult to inhale and exhale

120. Give a characteristic breathing during expiratory dyspnea:


A. constant amplitude
B. Superficial
C. difficult and prolonged breathing
D. difficult and prolonged exhalation
E. difficult to inhale and exhale

121. The expansion of the affected side of the chest, smoothing of the intercostal space,
weakening or absence of voice tremor, tympanic percussion sound, weakened vesicular
breathing are characteristic of the syndrome;
A. increased airiness of the lungs;
B. seals lung tissue;
C. the presence of fluid in the pleural cavity;
D. the presence of air in the pleural cavity;
E. cavity in the lung.
122. The most effective method of radiation diagnosis for the determination of small quantities
of fluid in the pleural cavity is?
A. magnetic resonance imaging;
B. radionuclide diagnosis;
C. radiography of the chest;
D. Ultrasound examination of the pleural cavities
E. fluorography.

123. What is characterized by emphysema of the lungs:


A. a decrease in lung capacity
B. increased residual lung volume
C. a decrease in lung minute volume
D. development of respiratory acidosis
E. reduced residual lung volume

124. Sputum discharge "full mouth" is observed in case of


A. increased airiness of the lungs;
B. seals lung tissue;
C. the presence of fluid in the pleural cavity;
D. the presence of air in the pleural cavity;
E. cavity in the lung.

125. What is the main respiratory noise most often heard with lobar inflammatory compaction?
A. weakened or weakened vesicular respiration
B. Amphora breath
C. bronchial respiration
D. hard breathing
E. Mixed bronchovesicular respiration

126. Indicate the irreversible component of bronchial obstruction:


A. bronchospasm;
B. inflammatory edema of the bronchial mucosa;
C. local expansion of the bronchial wall;
D. stenosis and obliteration of the lumen of the bronchi, as well as their expiratory collapse.
E. hypersecretion

127. The most reliable auscultatory sign of bronchial obstruction is:


A. Wet fine wheezing
B. crepitus
C. prolonged exhalation, dry rales
D. bronchial respiration
E. weakened vesicular respiration

128. The low voluntary ventilation (LVV) is the amount of air that:
A. ventilated through the lungs for 1 minute;
B. enters the lungs for 1 respiratory cycle;
C. as much as possible exhale after a deep breath;
D. as much as you can exhale after a normal breath
E. is inhaled and exhaled with calm breathing.

129. What ensures a quiet exhalation after inhalation is normal?


A. contraction of the small bronchi
B. contraction of expiratory muscles
C. contraction of the diaphragm
D. relaxation of expiratory muscles
E. relaxation of the inspiratory muscles

130. What is the minimum amount of fluid in the pleural cavity determined by physical
examination?
A. more than 100 ml
B. more than 500 ml
C. more than 1 l
D. 1.5 liters or more
E. more than 300 ml

131. Characteristics of the cavity in the lung, connected with the bronchus:
A. blunting, impaired breathing and bronchophony, displacement of the mediastinum in the
opposite direction
B. The same, but the bias towards blunting
C. Dullness with tympanic sound, amphora breathing, large bubble rales
D. blunting, bronchial respiration, enhanced bronchophony
E. inspiratory dyspnea, lung volume reduction, crepitus

132. The clinical sign of right ventricular hypertrophy in patients with pulmonary heart
syndrome is
A. Enhanced 2nd tone on the pulmonary artery
B. ripple 3-4 intercostal space to the left of the sternum
C. reinforced spilled apical impulse
D. precordial and epigastric pulsations
E. Offset relative dullness of the heart to the left

133. What is the purpose of using an additional technique for auscultation of the lungs - pressing
the stethoscope on the chest?
A. to distinguish pleural friction noise from crepitus and wheezing
B. to detect hidden bronchial obstruction
C. in order to distinguish dry rales from wet rales
D. for better hearing of vesicular breathing
E. for better listening for pathological bronchial respiration

134. What value of blood oxygen saturation (SaO2) of the following can be considered normal?
A. ≥86%
B. ≥90%.
C. ≥95%
D. ≥76%
E. ≥69%

135. The respiratory volume is:


A. the maximum amount of air inhaled after the end of a normal breath;
B. the maximum amount of air exhaled after the end of a normal exhalation;
C. volume of inhaled or exhaled air;
D. the volume of air remaining in the lungs after maximum expiration;
E. Maximum expiratory volume for the first second.

136. Residual lung volume is:


A. the maximum amount of air inhaled after the end of a normal breath;
B. the maximum amount of air exhaled after the end of a normal exhalation;
C. volume of inhaled or exhaled air;
D. the volume of air remaining in the lungs after maximum expiration;
E. maximum expiratory volume in the first second;

137. What is the most likely reason for the limited mobility of the lower edges of the lungs?
A. bronchial obstruction
B. respiratory failure
C. atrophy of the respiratory muscles
D. Increased airiness of lung tissue
E. compaction of the lung tissue

138. A 15-year-old patient was admitted with complaints of cough with discharge of
mucopurulent sputum up to 200 ml per day, with an unpleasant smell, temperature rise up to 38.2
° С, shortness of breath. Since childhood, cough with purulent sputum often worries.
Auscultatory to the right below the angle of the scapula in a limited area, sonorous moist
various-sized wheezes are heard. The most likely diagnosis:
A. Syndrome of bronchial obstruction
B. Syndrome increased airiness of the lung tissue
C. Syndrome seal lung tissue
D. Syndrome of air in the pleural cavity.
E. Syndrome presence of bronchiectasis in the lung.

139. What changes on the ECG do you expect to receive in a patient with a chronic pulmonary
heart?
A. Left ventricular hypertrophy
B. Right ventricular hypertrophy
C. Left atrial hypertrophy
D. Reduced ECG voltage
E. Pathological Q wave

140. In laboratory studies of blood in hypoxemia is noted


A. an increase in hemoglobin and hematocrit;
B. decrease in hemoglobin and hematocrit;
C. leukopenia and thrombocytopenia;
D. leukocytosis and thrombocytosis;
E. lymphopenia.

5
DEPARTMENT OF PROPEDEUTICS OF INTERNAL DISEASES WITH COURSE OF
ENDOCRINOLOGY
Tests for the 3rd course
GASTROENTEROLOGY

1. In physiological conditions, for how much time food stays in the mouth:
A. 3-5 min
B. 15-20 seconds
C. 10-15 min
D. 5-10 min
E. over 15 minutes

2. An adult tongue normally is :


A. wet, clean and pink
B. dark, mild rash and raids on her
C. pale pink coloration, scurfy
D. papillae on the tongue mild
E. dry, crimson

3. The upper border of normal liver on midclavicular line is:


A. intercostal space in the V
B. at the VI rib
C. in the intercostal space VII
D. VIII in mezhrebere
E. in the IX intercostal space

4. The length of the esophagus is normal:


A. 16-17 cm
B. 40-60 cm
C. 25-30 cm
D. 10-20 cm
E. 60-70 cm
5. Percussion size of the liver by Kurlov in normal:
A. 9-8-7 cm
B. 3-4-5cm
C. 11-12-13cm
D. 12-13-14cm
E. 6-7-8 cm

6. Intraventricular pH-metry estimates:


A. the most accurate acid-producing function of the stomach
B. It determines the amount of gastric juice
C. enzyme secretion function of the stomach
D. pepsin-forming function
E. determines the amount of intestinal juice

7. The surface-estimated, methodical deep sliding palpation:


A. Obrazcov- Strazhesko
B. Botkin
C. Corvisart
D. Aunburgerru
E. Ortner

8. Dysphagia is:
A. violation of passage of food through the esophagus
B. chest pain
C. nausea
D. bitter taste in the mouth
E. excessive salivation

9. Painful swallowing is called:


A. dysphagia
B. odynophagia
C. dyspepsia
D. ataxia
E. melena

10. When duodenal sounding portion "A" - is the content of:


A. 12 duodenal ulcer
B. common bile duct
C. the neck of the gall bladder
D. stomach
E. gallbladder

11. When duodenal sounding portion of the "C" - is the content of:
A. 12 duodenal ulcer
B. stomach
C. common bile duct
D. intrahepatic bile ducts
E. gallbladder

12.Charactericstic of palpation of the pancreas:


A. It has the form of horizontally disposed, soft cylinder with a diameter of about 2-3 cm,
and the movable morbid
B. pancreas has the form of a vertically disposed, cylinder soft, d = 4 cm
C. pancreas has the form of transversely located, soft cylinder, d = 1,5-3 cm, fixed and
painless
D. pancreas looks directly located, soft cylinder, d = 5 cm, mobile and painless
E. has the form of obliquely positioned, dense cylinder, d = 7 cm, movable, painless

13. The normal pH of basal secretion:


A. pH 1.6-2.6
B. pH 1.0 - 2.0
C. pH 1.2 - 3.0
D. pH 1.6 - 2.0
E. pH 1.6 - 5.0

14. A doctor examining patient with bent fingers to produce jerky movements without
moving away from the abdominal wall. This technique is called:
A. Surface palpation
B. deep palpation
C. Determination of "splashing noise"
D. Definition of "symptom Mendel"
E. auskultation

15. Which lines define the size of the liver by Kurlov?


A. midclavicular line, the middle and the right costal arch
B. midclavicular line, the middle and the left costal arch
C. anterior axillary, median and right costal arch
D. midsternal line, medial, anterior axillary
E. posterior axillary, front axillary and median

16. In diseases of the liver and biliary tract pain irradiates to:
A. left shoulder
B. right scapula
C. hearts
D. waist
E. the left hand

17. Normal spleen length on X rib is:


A. 3-6 cm
B. 5-6 cm
C. 6-8 cm
D. 6-10 cm
E. 6-12 cm

18. Normal spleen diameter is equal to:


A. 4-6 cm
B. 4-8 cm
C. 4-10 cm
D. 6-8 cm
E. 8-10 cm

19. The most typical complain characteristic for the pathology of the esophagus is:
A. dysphagia
B. belching
C. epigastric pain
D. nausea
E. vomiting

20. GERD patients have complaints:


A. diarrhea
B. heartburn
C. nausea
D. burp air
E. abdominal distention

21. Diseases of the esophagus are characterized by:

A. dysphagia, hypersalivation
B. constipation
C. flatulence
D. pain around the navel
E. tenesmus

22. Name the symptom of painful swallowing or pain when passing food through the
esophagus?

A. heartburn
B. odynophagia
C. dysphagia
D. flatulence
E. malabsorption

23. Dyspeptic complaints include:


A. nausea, discomfort in the epigastric region
B. pain in the iliac region
C. dizziness
D. tarry stools
E. generalized weakness

24. Select the most informative method of investigation of patients with diseases of the
esophagus:
A. inspection
B. palpation
C. percussion
D. auscultation
E. esophagoscopy

25. The estimated surface palpation usually begin with:


A. epigastric
B. the left iliac region
C. left hypochondrium
D. the right iliac region
E. right hypochondrium

26. Parietal cells secrete:

A. hydrochloric acid
B. lactic acid
C. gastric mukoprotein
D. mucin
E. pepsin

27. What can be heard normally on auscultation of the stomach of an adult ?


A. noise peritoneum friction
B. splashing sound
C. a sharp increase in the peristalsis of bowel sounds
D. slight rumbling, transfusion fluid
E. the absence of sounds and noises

28. Pain that are characteristic of gastric ulcer:


A. early pain
B. later pain
C. night pain
D. hunger pains
E. stabbing pain

29. Helicobacter pylori infection is a major cause of development:


A. chronic pancreatitis
B. gastric ulcer and 12 duodenal ulcer
C. chronic hepatitis
D. inflammation of the gall bladder syndrome
E. syndrome, inflammation of the pancreas
30. Gastric bleeding is manifested by :
A. hemoptysis
B. hematuria
C. bloody vomit
D. the noise of splashing epigastric empty stomach
E. leukocyturia

31. Which method is the most informative to determine the cause and location of bleeding
from the upper gastrointestinal tract:
A. radiography
B. portomanometriya
C. laparoscopy
D. gastroduodenoscopy
E. tseliakografiya

32. The most informative and most common method in the syndrome of peptic ulcer:
A. irrigoscopy
B. cholangiography
C. esophagogastroduodenoscopy
D. duodenal intubation
E. total blood test

33. The most common ulcers in the stomach are located:


A. in subcardiac part
B. in the cardiac part
C. on a small curvature in the antrum
D. on the greater curvature
E. gastric body

34. In healthy individuals the volume of gastric juice secretion in basal (in ml) is:
A. up to 50
B. 50-100
C. 100-140
D. 140-180
E. 100-200

35. Patient S. 37y.o., passed esophagogastroduodenoscopy. On endoscopy: esophagus is


passable, mucous membrane is not changed,cardiac pulp has conventional shapes and
sizes. Stomach normal in shape and size. Hyperemic mucosa in the antrum has deep
ulcerative defect measuring 1.5 -2,0sm folds normal shape and size. Bulb of duodenum has
normal form and size, the mucous is not changed. To determine the cause of stomach
ulcers, which additional tests necessary?
A. X-ray inspection
B. elektrogastroenterografy
C. Study of gastric secretory function
D. study of Helicobacter pylori
E. total protein and its fractions

36. For accurate diagnosis of the syndrome of inflammation of the gastric mucosa is used:
A. gastric analysis
B. fluoroscopy
C. gastroscopy, pH-metry
D. Endoscopy with biopsy
E. irrigoscopy

37. The largest size of gastric ulcers:


A. to 0.3 cm
B. to 0.5 cm
C. 0.3-0.4 cm
D. 2-3 cm and above
E. 0.2sm less

38. Choose localization of ulcers that are characteristic for hunger pains after 1.5 hours
after a meal:
A. pyloric ulcer of the stomach, duodenum ulcer 12
B. ulcers subcardial department
C. ulcers, middle and lower third of the stomach.
D. body of the stomach ulcer
E. ulcers of the greater curvature of the stomach

39. Patient M., aged 35, complained of aching, hungry pain at night in the epigastric
region. Pains go away after a meal. From the history of the disease: for many years
observed with a diagnosis of chronic gastritis. Eats irregularly, often fast foods. On
examination: belly is symmetrical, involved in the act of breathing. On palpation:
Shchetkin-Blumberg symptom is negative, Mendel's positive symptom. What kind of
syndrome is to think the doctor?
A. syndrome, inflammation of the gallbladder
B. bleeding from stomach ulcers
C. syndrome of peptic ulcers
D. malabsorption syndrome
E. syndrome maldigestion

40. Hyperchlorhydria hypersecretion and gastric ulcers have characteristic localization


in :
A. esophagus
B. small curvature of the stomach
C. body
D. duodenum
E. cardiac part of the stomach

41. Black, tarry stool found in bleeding from:


A. stomach and duodenum
B. blind, ascending colon
C. the transverse colon
D. descending and sigmoid colon
E. rectum

42. A direct indication of the presence of ulcers on X-ray examination is:

A. scar body deformation


B. folds convergence
C. "Niche" symptom
D. evacuation change
E. dyskinesia gatekeeper

43. On physical examination, patients with peptic ulcer syndrome marked tenderness to
percussion in epigastric. This is a symptom of :
A. Mendel
B. Murphy
C. Ortner
D. Kerr
E. Courvoisier
44. Vomiting 2-3 hours after meals is typical for:

A. ulcers cardiac part of the stomach


B. ulcer of the stomach body
C. ulcer of the stomach antrum
D. pyloric stenosis
E. duodenal ulcer

45. Pain during peptic ulcer decrease or disappear:

A. after vomiting
B. when applying cold
C. after fatty meal
D. when you change the patient's position
E. the application of heat

46. Pigment function of liver is in the serum levels of:


A. cholesterol
B. total protein and protein fractions
C. fibrinogen
D. bilirubin and its fractions
E. glucose

47. The normal serum bilirubin is:


A. 1-3,02 mol / l
B. 8,55-20,5 .mu.mol / l
C. 30,70-40,70 mol / l
D. 40,80-50,80 mol / l
E. 22-25,0 mmol / l

48. Disability, weakness, fatigue, malaise, headaches in patients with liver disease is
indicated by the syndrome:
A. dyspeptic
B. asthenovegetative
C. haemorrhagic
D. hepatolienalis
E. portal hypertension

49. The total protein content in the blood in normal:


A. 65-85 g / l
B. 60-80 g / l
C. 70-90 grams / liter
D. 80-100 g / l
E. 40-55 g / l

50. Localization of pain in liver disease :


A. the left back side
B. umbilical region
C. the right iliac region
D. right upper quadrant
E. left upper quadrant

51. Decreased appetite, heaviness in the epigastric region, nausea, vomiting, bloating, stool
disorders in patients with liver disease are the signs of :
A. dyspeptic syndrome
B. hemorrhagic syndrome
C. asthenovegetative syndrome
D. hepatolienal syndrome
E. portal hypertension

52. The rich fat mushy feces is observed when:


A. gastric hypersecretion
B. syndrome of peptic ulcer
C. portal hypertension
D. exocrine pancreatic insufficiency
E. syndrome of gallbladder inflammation

53. Normally albumin in blood:


A. 40-55 g / l
B. 36-50 g / l
C. 70-85 g / l
D. more than 80 g / l
E. less than 20 g / l
54. Normally, Thymol test is:
A. 0-5 units.
B. 5-9 units
C. 10-14 units
D. 15-19 units
E. more than 20

55. Ascites ,enlargement of the saphenous veins of abdominal wall, enlarged spleen,
esophageal varices in patients with liver disease syndrome denotes:
A. asthenovegetative
B. dyspeptic
C. haemorrhagic
D. portal hypertension
E. hepato-splenic

56. Normally, the blood pressure in the portal vein:


A. 50-115mm water column
B. 70-150mm water column
C. 150-220mm water column
D. 220-290mm water column
E. 290-360mm water column

57. "Spider nevi" represent:


A. dilation of small blood vessels in the skin
B. intradermal deposits of cholesterol in the form of plaques
C. dermatorrhagia
D. slightly elevated above the skin pulsating angioma
E. bruising

58. Patient M., 50 years old, complaints on admission: an increase in the abdomen, general
weakness, poor appetite, weight loss. From anamnesis: 2 years ago after a visit to the
dentist was ill, she suffered a viral hepatitis. Skin and mucous membrane are pale, palmar
erythema, chest and shoulders "spider nevi". On examination, the abdomen is in the
supine position with the flattened sides of the protrusion in, percussion dull sound is
determined in these departments, palpation - a positive symptom of "fluctuations". The
liver protruded from under the costal margin 1.5-2 cm, thick consistency. Lower edge of
the spleen is palpable. Select the basic syndrome.
A. hemorrhagic syndrome
B. suprarenal jaundice syndrome
C. obstructive jaundice syndrome
D. portal hypertension syndrome
E. maldigestion syndrome

59. Signs of liver failure syndrome:


A. albumin reduction
B. increase in the total protein content
C. albumin increase
D. increased activity of the enzymes ALT, AST
E. decrease in the activity of enzymes ALT, AST

60. Blood test in syndrome of hepatocellular insufficiency shows:


A. decreased albumin
B. increased hemoglobin levels
C. increased total protein level
D. increases the number of red blood cells
E. increased prothrombin index

61. In hepatic jaundice hyperbilirubinemia is due to:


A. conjugated and unconjugated bilirubin
B. conjugated bilirubin
C. bile acids
D. free bilirubin
E. glucose oxidase

62. Skin itching in Liver disease is characteristic for?

A. stock duodeno-gastric reflux


B. an increase of bile acids in the blood against the backdrop of severe cholestasis
C. violation synthetic (protein-) liver function
D. heart failure with the expressed accompanying myocardial dystrophy
E. reducing detoxifying liver function with respect to protein breakdown products

63. Cytolysis syndrome that develops during liver damage is characterized by:
A. increased ALT, AST, LDH
B. increase in prothrombin level
C. increase in total protein and albumin
D. no laboratory changes
E. reducing thymol

64. In the suprarenal jaundice in the blood is detected:


A. an increase of unconjugated bilirubin
B. lowering of unconjugated bilirubin
C. increase in ketone bodies
D. lowering ketone bodies
E. an increase of conjugated bilirubin

65. In suprarenal jaundice what is found in the urine:


A. elevated levels urobilin
B. reduced content urobilinoides
C. elevated levels of conjugated bilirubin
D. lowering the content of conjugated bilirubin
E. urine unchanged

66. In the suprarenal jaundice what is detected in the stool:


A. elevated levels stercobilin
B. reduced content stercobilin
C. aholich feces
D. tarry stool (melena)
E. lack stercobilin

67. In the horizontal position abdomen is soft, side sections swell (frog belly) suggests:
A. obesity
B. the presence of large intra-abdominal tumors
C. flatulence
D. fluid accumulation in the abdominal cavity
E. enlarged liver

68. "Palmar eritema", represent:

A. intradermal deposits of cholesterol in the form of plaques


B. bleeding into the skin
C. petechiasis
D. symmetrical reddening of thenar and hypothenar
E. bruising

69. Skin color with obstructive jaundice:


A. pale lemon yellow tinge
B. orange-yellow
C. dark greenish
D. Leather is not changed
E. bronzing

70. The projection (localization) of the gallbladder is equal to:


A. the point of intersection of the outer edge of the right rectus abdominis with the costal
arch
B. under the lower edge of the liver, outwards from the edge of the right lateral rectus
C. in the left upper quadrant
D. at the intersection of the outer edge of the left rectus abdominis with the costal arch
E. in the left iliac region

71. The bleeding from the nose, gums, uterine bleeding, cutaneous bleeding,
bleeding of esophageal varices in patients with liver diseases designate syndrome:

A. asthenovegetative
B. dyspeptic
C. haemorrhagic
D. portal hypertension
E. hepatolienal

72. The wall thickness of the gallbladder by ultrasound in the standard is:
A. not more than 2 mm
B. 5-6 mm
C. 3-4 mm
D. 7-8 mm
E. 8-10 mm

73. The gallbladder contracts under the influence of :


A. gastrin
B. pancreatic juice
C. cholecystokinin
D. secretin
E. of hydrochloric acid

74. Main function of bile is:


A. emulsification of fats
B. carbohydrate hydrolyzing
C. split proteins
D. split of fat
E. increases of the secretion gastrin

75. In hypomotor dyskinesia of gallbladder observed:


A. increase in the tone of the gall bladder
B. decreased tone of the gallbladder with bile stasis
C. reduction of the initial dimensions of the gallbladder
D. accelerated emptying of the gallbladder
E. increasing the motor activity of the gall bladder and biliary tract

76. "False jaundice" occurs when:


A. hemolytic jaundice
B. receiving a large amount of carotene from food
C. iron supplementation
D. liver failure
E. peptic ulcer

77. Which of the methods are used for the diagnosis of the biliary tract.
A. oral cholecystography
B. ultrasound procedure
C. nuclear magnetic resonance
D. endoscopic retrograde cholangiopancreatography
E. intravenous cholangiography

78. What can trigger the attack of biliary colic :


A. overheating or undercooling
B. abundant intake of fatty foods
C. smoking
D. bad dream
E. poor appetite

79. Cholecystography - is X-ray methodic of :


A. portal vein
B. gallbladder
C. splenic vein
D. celiac artery
E. bulbs of duodenum

80. Characteristic for hyperkinetic biliary dyskinesia:


A. acute colic pain in the right upper quadrant
B. body weight loss
C. increased body temperature
D. hepatomegaly
E. splenomegaly

81. The method of physical examination to determine the upper and lower boundaries of
the liver:
A. inspection
B. deep palpation
C. percussion
D. auscultation
E. estimated surface palpation

82. The following is true for hypokinetic biliary dyskinesia:


A. acute colic pain in the right upper quadrant radiating to the right shoulder blade, shoulder
B. hepatomegaly
C. amount of bile in the portions B rises to 100-150 ml at duodenal intubation
D. Body temperature decrease
E. sharp pain in the left upper quadrant

83. In hypermotor gallbladder dyskinesia we observe:


A. decreased tone of the gall bladder
B. spasm of the sphincter of Oddi and Lyutkense
C. increasing the initial size of the gallbladder
D. slowing down the emptying of the gallbladder
E. reduction in motor activity of the gallbladder and bile ducts

84. Pain like a band in the upper abdomen is observed in pathology:


A. stomach
B. liver
C. pancreas
D. cecum
E. sigmoid colon

85. Point De Garden’s - is:


A. the projection of the sphincter of Oddi
B. Pancreatic body projection
C. the initial part of the pancreas
D. the projection of the pancreatic head
E. projection cardiac of the stomach

86. Intradermal deposition of cholesterol in the century as yellow plaques called?


A. petechial rash
B. xanthelasmatosis
C. telangiectases
D. hematoma
E. palmar erythema

87. Gold standard evaluation failure of exocrine pancreatic function is:


A. determination of the activity of pancreatic elastase in feces
B. microscopic examination of feces
C. secretin-probe test pankreazimin
D. determination of blood amylase and trypsin
E. determination of urine amylase and trypsin

88. Exocrine pancreatic insufficiency is characterized by:


A. increased alkaline phosphatase
B. presence of unconjugated bilirubin in the urine
C. hypoglycemia
D. steatorrhea, kreatorrhea, amylorrhea
E. raising the general level of protein in the blood

89. The defeat of the pancreatic head pain is defined:


A. point De Garden
B. at Mayo-Robson
C. at Kacha
D. point Mallet-Guy
E. zone Shaffer's

90. Zone Gubergriz’s:


A. It reflects the initial portion of the body of the pancreas
B. It reflects the tail of the pancreas
C. It corresponds to the projection of the sphincter of Oddi
D. corresponds to the body of the pancreas
E. It reflects the terminal portion of the body of the pancreas

91. A sharp pain during effleurage in the projection of the gallbladder at the height of
inspiration - is a symptom of :
A. Zakharyin
B. Vasilenko
C. Obraztsov - Murphy
D. Ortner
E. Musso - George's

92. The pain of effleurage edge of the right costal arch brush - it's a symptom:
A. Zakharyin
B. Vasilenko
C. Obraztsov - Murphy
D. Ortner
E. Musso George's

93. Pain with pressure in between the legs of sternocleidomastoid muscle on the right - it's a
symptom:
A. Zakharyin
B. Vasilenko
C. Obraztsov - Murphy
D. Ortner
E. Musso - George's

94. In duodenal intubation detection of portion "B" of a large amount of leucocytes and
increase of gallbladder epithelial cells indicates:
A. an inflammation of the gall bladder
B. peptic ulcers
C. inflammation of the duodenum
D. inflammation of the pancreas
E. dyskinesia of the gallbladder

95. Stool frequency constipation:

A. 1 time per day


B. 1 time in 2 days
C. stool delay more than 3 days
D. 2 times a day
E. 3 times per day

96. Pain in the right iliac region arise in pathology:

A. cecum
B. sigmoid colon
C. transverse colon
D. rectum
E. stomach

97. In the normal intestine percussion determines


A. tympanic sound
B. dull sound
C. dull sound
D. box sound
E. resonance sound
98. Superficial palpation reveals:
A. the consistency of the body
B. intestinal localization
C. voltage abdominal wall
D. body diameter
E. body surface

99. Flatulence - is:

A. frequent stools
B. false urge to defecate
C. abdominal distention
D. tarry stools
E. pain around the navel.

100. A sigmoidoscopy examines the colon mucosa in the distance (cm):


A. 45
B. 50
C. 35
D. 40
E. 50

101. A colonoscopy allows to explore:

A. Only the cecum


B. only the sigmoid colon
C. only the rectum and part of the sigmoid colon
D. Only rectum
E. the entire large intestine

102. X-ray examination of the large intestine - is:

A. colonoscopy
B. sigmoidoscopy
C. irrigoscopy
D. laparoscopy
E. duodenography

103. Signs of maldigestion syndrome according to the analysis of feces:


A. steatorrhea, kreatoreya
B. aholic feces
C. desquamated epithelium
D. melena
E. positive reaction Gregersen

104. Gregersen's reaction makes it possible to determine in the stool:

A. leukocytes
B. occult bleeding from the gastrointestinal tract
C. eggs worm
D. dysbiosis
E. neutral fat

105. If a large number of muscle fibers is determined in the stool, it's:

A. stearrhea
B. kreatorrhea
C. lientereya
D. melena
E. amylorrhea

106. Patient S., 46 years old, complaints on admission: an enlargement of the abdomen,
general weakness, heaviness in the right upper quadrant, jaundice of the skin. From the
history of the disease. During the year, there is a gradual increase of the above symptoms.
As a child, he moved Botkin's disease, alcohol abuse. What research tools are needed to
clarify the syndrome?

A. cholecystography

B. duodenal intubation

C. Ultrasonography of the abdomen

D. laparoscopic abdominal organs

E. microscopic analysis of feces

107. Malabsorption is :
A. syndrome caused by malabsorption of nutrients in the small intestine
B. syndrome caused by biliary dyskinesia
C. syndrome caused by impaired blood flow in the portal vein
D. syndrome caused by violation of bilirubin formation and excretion of bile ducts
E. syndrome caused by amplification in bile Incoming intestines
108. Primary malabsorption occurs when:
A. vitamin B12 deficiency
B. hereditary deficiency of lactase
C. hypoholii
D. enteritis
E. gastritis

109. In malabsorption syndrome swelling is associated with:


A. hypoproteinemia
B. portal hypertension
C. circulatory failure
D. anemia
E. thyroid dysfunction

110. The sample-Triboulet Vishnevsky is performed to:


A. identifying squirrel exudation
B. determination of occult blood in the stool
C. determining the bacterial overgrowth
D. identifying lactase deficiency
E. detection of Helicobacter pylori infection

111. Feces of an adult normally release:

A. unconjugated bilirubin
B. conjugated bilirubin
C. urobilin
D. stercobilin
E. urobilinogen

112. The complete absence of hydrochloric acid and pepsin in gastric juice is called:
A. hyperacidity
B. Achille
C. hypochlorhydria
D. achlorhydria
E. hypersecretion

113. On gross examination feces in normal:


A. polifekaliya
B. unformed liquid
C. elegant, brown
D. shine stool
E. mushy

114. Malabsorption syndrome of Vitamin A deficiency leads to:


A. night blindness
B. subcutaneous hemorrhage
C. neurological disorders
D. dermatitis
E. esophagitis
115. In a normal sigmoid colon is palpated in:

A. 90-95%
B. 80-85%
C. 60-70
D. 50-60%
E. not detectable

116. If in the feces are determined by a large number of neutral fat, it's this:
A. stearrhea
B. kreatoreya
C. lientereya
D. amylorrhea
E. melena

117. In diseases of the small intestinal pain is observed:


A. around the navel
B. the right-hand side of the area
C. in the left iliac region
D. epigastric
E. in the right upper quadrant

118. What information does the positive symptom waves (fluctuations) in the percussion
bimanual palpation of the abdomen give?

A. symptom is detected normally;


B. there is free fluid in the abdominal cavity;
C. It has pyloric stenosis;
D. a large amount of gases in the colon (flatulence in patients with colitis);
E. has accumulated liquid contents and gases in the colon (e.g., a patient with acute enteritis).

119. During Diarrhea stool is :

A. 1 time per day


B. 1 time on the second day
C. more than 3 times a day
D. 1 time per day Zoe
E. 1 time per week

120. Schmidt sample was carried out to:

A. identify protein exudation

B. determination of occult blood in the stool

C. determining the complete cessation of the flow of bile into the intestine

D. identifying novoobrozovaniya

E. identifying ulceration
6
CARDIOLOGY
1. Choose the most correct interpretation of palpation data - a spilled high (dome-shaped)
apical impulse in the VI intercostal space 2 cm outwards from the mid-clavicular line:
A. hypertrophy and dilatation of the left ventricle
B. hypertrophy and dilatation of the right ventricle
C. hypertrophy of the left atrium
D. right atrial hypertrophy
E. right ventricular hypertrophy

2. Choose the most correct interpretation of palpation data - a pronounced cardiac impulse
and epigastric pulsation:
A. hypertrophy and dilatation of the left ventricle
B. hypertrophy and dilatation of the right ventricle
C. hypertrophy of the left atrium
D. right atrial hypertrophy
E. Left ventricular hypertrophy

3. Palpation of the heart at the apex reveals a jitter that does not coincide with the pulsation
a. carotis. What heart disease is it typical for?
A. aortic valve insufficiency
B. mitral valve insufficiency
C. aortic stenosis
D. mitral stenosis
E. tricuspid valve insufficiency

4. What clinical situation is characterized by the following changes in the auscultative


picture of the heart: at the apex I tone is weakened, systolic murmur is heard, which is
conducted in the axillary region. What is it?
A. aortic valve insufficiency
B. mitral valve insufficiency
C. aortic stenosis
D. mitral stenosis
E. tricuspid valve insufficiency

5. Mitral valve is better heard where?


A. in the fifth intercostal space on the left 1.5 cm inwards from the midclavicular line
B. in the second intercostal space to the left of the sternum
C. in the second intercostal space to the right of the sternum
D. to the right of the sternum in the fourth intercostal space
E. at Botkin-Erb Point

6. The apical impulse is formed by:


A. aortic arch
B. abdominal aorta
C. right ventricle
D. left ventricle
E. left atrium.

7. Apical impulse with aortic valve insufficiency is going to be:


A. small, weak, limited
B. small, reinforced, limited
C. high, reinforced, spilled
D. small
E. limited

8. For what clinical situation are the following options for changing the boundaries of the
relative dullness of the heart: the right border is 1 cm to the right of the right edge of the
sternum, the left one along the anterior axillary line, the top edge is the third edge?
A. mitral stenosis
B. mitral insufficiency
C. tricuspid valve insufficiency
D. aortic valve insufficiency
E. stenosis of the pulmonary valve

9. What clinical situation is characterized by the following options for changing the
boundaries of the relative dullness of the heart: the right border is 3 cm to the right of the
sternum edge, the left one is 1 cm medially from the left mid-clavicle line, the upper one is
the upper edge of the second rib?
A. mitral stenosis
B. mitral insufficiency
C. tricuspid valve insufficiency
D. aortic stenosis
E. aortic valve insufficiency

10. What clinical situation is characterized by the following options for changing th e
boundaries of the relative dullness of the heart: the right border is 1 cm to the right of the
sternum edge, the left one is 2 cm outward from the left mid-clavicle line, the top one is II
rib?
A. mitral stenosis
B. mitral insufficiency
C. tricuspid valve insufficiency
D. aortic malformations
E. stenosis of the pulmonary valve

11. Indicate the most characteristic signs of arterial pulsus pulsus dificiens:
A. a sharp weakening or absence of pulsation in a single radial artery
B. a sharp decrease in the magnitude of the pulse in both radial arteries
C. The number of pulse waves in the radial artery is greater than the number of heartbeats.
D. The number of pulse waves in the radial artery is less than the number of
heartbeats.
E. sharp weakening or absence of pulsation in both radial arteries

12. The area of the mitral orifice is normally:


A. 1-2 cm2
B. 3-4 cm2
C. 4-5 cm2
D. 4-6 cm2
E. 2-3 cm2
13. Critical mitral stenosis is:
A. 1 cm2
B. 1.3 cm2
C. 1.5 cm2
D. 1.9 cm2
E. 2 cm2

14. Left ventricular hypertrophy is established when:


A. RV5 + SV1 ≥ 38 mm, R aVL> 11mm
B. RV1> 7mm
C. h. RV5 <RV4
D. R V5-6 <15 mm
E. Availability S.S V5-6

15. Rhythm "quail" in mitral stenosis is:


A. clapping tone I, enhanced tone II and the tone of the opening of the mitral
valve
B. I tone is weakened, II tone is not changed, pathological III tone in diastole
C. I tone is not changed, II tone is weakened, systolic murmur
D. I tone is enhanced, II tone is not changed, mesodiastolic noise
E. I tone is not changed, II tone is not changed, systolic murmur

16. Specify the most informative method for diagnosing mitral valve insufficiency:
A. Auscultation of the heart
B. phonocardiography
C. X-ray of the heart
D. echoppler cardiography
E. coronaroangiophagia

17. The main auscultatory sign of mitral valve insufficiency:


A. diastolic noise in the second intercostal space on the right
B. systolic murmur throughout systole at the top
C. diastolic noise at the Botkin point
D. systolic murmur in the second intercostal space
E. systolic murmur in the second intercostal space on the left

18. A direct sign of mitral valve insufficiency in EchoCG Doppler study:


A. turbulent diastolic flow in the projection of the mitral valve
B. Blood regurgitation from the left ventricle to the left atrium during
systole
C. Regurgitation jet from the aorta to the left ventricle
D. Blood regurgitation from the left ventricle to the left atrium during
diastoles
E. Stream of regurgitation from the right ventricle to the right atrium

19. A direct sign of aortic valve insufficiency in Doppler examination:


A. Regurgitation jet from the right atrium to the right ventricle
B. Regurgitation jet from the left ventricle to the left atrium
C. Regurgitation jet from the right ventricle to the right atrium
D. Regurgitation jet from the aorta to the left ventricle
E. Stream of regurgitation from the pulmonary trunk to the right ventricle
20. Which symptom of the following is pathognomonic for left ventricular heart failure?
A. swelling of the neck veins
B. ascites
C. enlarged liver
D. orthopnea
E. swelling in the legs

21. A pronounced pulsation of the carotid arteries is observed in patients with:


A. aortic stenosis
B. aortic valve insufficiency
C. mitral valve insufficiency
D. tricuspid valve insufficiency
E. Insufficiency valve of the pulmonary artery

22. Which of the heart defects most significantly increases the left atrium?
A. pulmonary artery stenosis
B. Aortic valve insufficiency
C. mitral stenosis
D. Aortic stenosis
E. Tricuspid valve insufficiency

23. When accent of II tone on the aorta occurs:


A. hypertension syndrome
B. syndrome of acute coronary insufficiency
C. myocardial inflammation syndrome
D. Pericardial inflammation syndrome
E. chronic coronary insufficiency syndrome

24. Forced position orthopnea facilitates the patient's condition with:


A. hypertension syndrome
B. syndrome of acute coronary insufficiency
C. chronic coronary insufficiency syndrome
D. left ventricular heart failure syndrome
E. Syndrome of arterial hypotension

25. Typical localization of pain in chronic coronary syndrome


failure:
A. behind the sternum
B. in the left half of the chest, in the apex of the heart
C. in the right side of the chest
D. in the epigastric region
E. in the right hypochondrium

26. Sinus tachycardia is a sinus rhythm with a heart rate.


abbreviations:
A. More than 85 per minute
B. more than 60 per minute
C. more than 75 per minute
D. more than 90 per minute
E. more than 120 per minute
27. What is meant by "mitral face"?
A. Erythematous rash on cheeks and nasal back
B. pallor of face
C. Cyanotic cheek blush
D. hyperemia of the left cheek
E. cyanosis of nasolabial triangle

28. A typical manifestation of heart failure is:


A. dizziness
B. shortness of breath
C. chest pain
D. high blood pressure
E. Headaches

29. When are the atria and ventricles contracted by their own rhythm?
A. with atrioventricular blockade of III degree
B. atrial fibrillation
C. with atrial paroxysmal tachycardia
D. with ventricular premature beats
E. atrial flutter

30. What is characteristic radiographically for aortic valve insufficiency:


A. The aorta is dilated, pulsating deeply, ejects the IV arch
B. protrudes the left atrial appendage
C. protrudes pulmonary trunk
D. Availability of Curly lines
E. aorta dilated in ascending division

31. The appearance on the ECG of a pathological Q wave in acute coronary insufficiency
syndrome is a reflection of:
A. subendocardial myocardial ischemia
B. myocardial necrosis
C. myocardial damage
D. Subepicardial myocardial ischemia
E. scarring myocardium

32. The ECG criterion for chronic coronary insufficiency syndrome is:
A. appearance of abnormal Q wave
B. reduction of the amplitude of the R wave
C. ST segment displacement below the isoelectric line by 1 mm or more
D. ST segment elevation above the isoelectric line more than 1 mm
E. Negative T wave in all chest leads

33. What is the pulse characteristic of aortic valve insufficiency?


A. pulsus celer et altus
B. pulsus rarus et parvus
C. pulsus differens
D. pulsus filiformis
E. pulsus parvus

34. What is the cause of noise in aortic stenosis:


A. Blood regurgitation from the aorta to the left ventricle
B. The expulsion of blood from the left ventricle to the aorta.
C. Blood regurgitation from the left ventricle to the left atrium
D. Expulsion of blood from the left atrium to the left ventricle
E. Blood regurgitation from the right ventricle to the right atrium

35. The occurrence of noise in mitral stenosis is caused by:


A. Expulsion of blood from the left ventricle to the aorta
B. Regurgitation of blood from the left ventricle to the left atrium
C. Regurgitation of blood from the right ventricle to the right atrium
D. Expulsion of blood from the left atrium to the left ventricle
E. Regurgitation of blood from the aorta to the left ventricle

36. The cause of the noise in the aortic valve insufficiency:


A. Blood regurgitation from the aorta to the left ventricle
B. Blood regurgitation from the right ventricle to the right atrium
C. blood regurgitation from the left ventricle to the left atrium
D. Expulsion of blood from the left ventricle to the aorta
E. Expulsion of blood from the left atrium to the left ventricle

37. At what defect is the diastolic murmur at the apex heard?


A. mitral stenosis
B. mitral valve insufficiency
C. tricuspid valve insufficiency
D. Aortic stenosis
E. aortic valve insufficiency

38. What changes in echocardiography of the following are a sign of left ventricular hea rt
failure:
A. EF <80%, EDDLV> 5.0 cm
B. EF <50%, EDDLV> 5.5 cm, diffuse hypokinesis of the LV walls
C. EF = 60%, LVPWT-1,2cm
D. EF = 60%, local hypokinesis IVST
E. Echonegative space for LVPWT 10mm

39. For what pathological condition is the “triangular” shape of the heart configuration
characteristic?
A. mitral stenosis
B. tricuspid valve insufficiency
C. aortic valve insufficiency
D. pericardial effusion
E. acute coronary insufficiency

40. Positive venous pulse is characteristic:


A. for mitral stenosis
B. for aortic stenosis
C. for aortic valve regurgitation
D. For tricuspid valve insufficiency
E. for heart failure

41. Pulmonary valve is better listened at:


A. in the fifth intercostal space on the left 1.5 cm inwards from the midclavicular line
B. in the second intercostal space to the left of the sternum
C. in the second intercostal space to the right of the sternum
D. to the right of the sternum in the fourth intercostal space
E. at Botkin-Erb Point

42. The symptom of "cat's purr" is determined by the method:


A. percussion
B. Auscultation
C. palpation
D. phonocardiography
E. electrocardiography

43. The symptom of "cat's purr" in the apex of the heart is determined by:
A. acute coronary insufficiency
B. mitral valve insufficiency
C. stenosis of the left AV opening
D. aortic valve regurgitation
E. tricuspid valve insufficiency

44. The following ECG indicators: the correct rhythm with a heart rate of <60 per min. And
unchanged QRS complexes are characteristic of:
A. sinus arrhythmia
B. blockade of bundle branch foot
C. sinus bradycardia
D. extrasystolic arrhythmia
E. atrial fibrillation

45. Palpation on the basis of the heart reveals a tremor, which coincides with the pulsation
on a. carotis. This is typical for:
A. mitral stenosis
B. mitral valve insufficiency
C. aortic stenosis
D. aortic valve regurgitation
E. tricuspid valve insufficiency

46. The value of blood pressure measured in a patient at rest and equal to 140/85 mm Hg,
should be considered in accordance with modern guidelines, as
A. 1st degree arterial hypertension
B. 2nd degree of arterial hypertension
C. high normal blood pressure
D. normal blood pressure
E. optimal blood pressure

47. When auscultation of the heart in patients with severe heart failure can be identified:
A. rhythm "quail"
B. protodiastolic gallop rhythm
C. systolic gallop rhythm
D. Additional pericardium tone
E. gain I tone at the top

48. The pulsation of the carotid arteries ("carotid dancing") is a sign:


A. aortic valve regurgitation
B. tricuspid valve insufficiency
C. stenosis of the mirtal valve
D. mitral valve insufficiency
E. Aortic stenosis

49. Complete AV blockade is characterized by:


A. increase the PQ interval
B. independent atrial and ventricular contractions
C. at different RR intervals
D. severe deformity of the ventricular complex
E. falling out of the next h. P and QRS complex

50. II tone with aortic stenosis:


A. weakened on the aorta
B. strengthened on the aorta
C. strengthened at the top
D. weakened at the top
E. not changed on the aorta

51. When aortic stenosis is heard:


A. diastolic murmur at the apex
B. diastolic noise in the second intercostal space on the right
C. systolic murmur in the second intercostal space on the right
D. systolic murmur at the apex
E. systolic murmur on tricuspid valve

52. Paradoxical pulse is determined by


A. heart failure syndrome
B. syndrome of arterial hypertension;
C. coronary insufficiency syndrome;
D. myocardial inflammation syndrome
E. Pericardial inflammation syndrome

53. ECG signs of atrioventricular block I degree:


A. prolongation of the interval PQ> 0.12 sec
B. prolongation of the interval PQ> 0.20 s
C. no P wave
D. deformation of the QRS complex
E. negative T wave

54. Name the components of the first tone.


A. muscular, valvular, vascular
B. muscular, valvular, vascular, atrial
C. muscle, valve
D. Valve, vascular
E. muscular vascular

55. When aortic valve insufficiency is heard


A. systolic murmur at the base of the xiphoid process
B. Protodiastolic noise at Botkin-Erb point
C. systolic murmur at the Botkin - Erb point
D. diastolic murmur at the base of the xiphoid process
E. diastolic murmur at the apex of the heart
56. Amplification of the apical impulse indicates
A. right ventricular hypertrophy
B. hypertrophy of the right atrium
C. hypertrophy of the left atrium
D. Left ventricular hypertrophy
E. dilatation of the left atrium

57. Accent of tone II on the pulmonary artery is noted when


A. Arterial hypertension
B. pulmonary hypertension
C. left ventricular hypertrophy
D. dilatation of the left ventricle
E. pulmonary valve insufficiency

58. Dysphagia in mitral stenosis is due to:


A. spasm of the esophagus;
B. compression of the esophagus by the enlarged right atrium;
C. compression of the esophagus because of enlarged left atrium;
D. compression of the esophagus by the enlarged pulmonary artery;
E. compression of the esophagus by the dilated left ventricle.

59. Select the symptom complex characteristic of left ventricular heart failure syndrome:
A. swelling of the legs, cough, shortness of breath
B. cough, hemoptysis, choking, orthopnea
C. hemoptysis, swelling of the legs, enlarged liver
D. swelling and pulsation of the neck veins
E. cough, swelling of the legs

60. When mitral valve insufficiency auscultatory is determined:


A. Strengthening of the I tone at the apex of the heart
B. weakening of the I tone at the apex of the heart
C. "click open" mitral valve
D. diastolic murmur at the apex
E. systolic aortic murmur

61. Systolic murmur with stenosis of the aortic mouth is performed


A. To the point of Botkin-Erb
B. on the xiphoid process
C. to the armpit
D. on the neck vessels
E. to the top of the heart

62. Which of the following factors predisposes to the development of coronary insufficiency
syndrome:
A. fasting blood sugar 4.5 mmol / l
B. Blood pressure 120/80 mm Hg.
C. blood cholesterol 5.0 mmol / l
D. smoking
E. Increased high density lipoprotein

63. Subjective characterization of pain in chronic coronary insufficiency syndrome:


A. compressive, pressing
B. stabbing
C. aching
D. dagger
E. cutting

64. Symptom Musset is observed at


A. Aortic stenosis
B. stenosis of the mirtal valve
C. aortic valve regurgitation
D. tricuspid valve insufficiency
E. mitral valve insufficiency

65. "Heart Hump" occurs when


A. severe left ventricular hypertrophy
B. severe left atrial hypertrophy
C. severe right ventricular hypertrophy
D. severe right atrial hypertrophy
E. severe dilatation of the left atrium

66. A sign of acute left ventricular failure is:


A. choking with choking breath;
B. swelling and pulsation of the veins of the neck;
C. enhanced epigastric ripple;
D. liver enlargement and tenderness
E. swelling of the lower limbs

67. Forced position of the patient sitting, bending forward, is observed when:
A. coronary insufficiency
B. effusion pericarditis;
C. mitral valve insufficiency
D. myocarditis
E. Arterial hypertension

68. "Deficit" pulse occurs when


A. Arterial hypertension
B. heart failure
C. atrial fibrillation
D. aortic valve regurgitation
E. atrioventricular block

69. The upper limit of relative cardiac dullness increases due to


A. right ventricular hypertrophy
B. Left ventricular hypertrophy
C. hypertrophy of the right atrium
D. Left atrial hypertrophy
E. dilatation of the left ventricle

70. Which symptom of the following is characteristic of left ventricular heart failure:
A. Lower limb edema
B. enlarged liver
C. inspiratory dyspnea
D. ascites
E. positive venous pulse

71. What symptom of the following is characteristic of right ventricular heart failure:
A. hepatomegaly
B. inspiratory dyspnea
C. symptom Musset
D. dry cough
E. moist rales in the lungs during auscultation

72. Absolute ECG sign of acute coronary insufficiency syndrome:


A. pathological Q wave
B. negative T wave
C. ST interval offset
D. Absence of P-wave
E. prolongation of the PQ interval

73. Diastolic noise with aortic valve insufficiency is better heard:


A. in the second intercostal space on the right
B. at Botkin-Erb Point
C. on the xiphoid process
D. at the apex of the heart
E. on the vessels of the neck

74. The most informative method for diagnosing coronary insufficiency is:
A. ECG
B. veloergometry
C. Echocardiography
D. Coronary angiography
E. Daily ECG monitoring

75. The main method for the diagnosis of cardiac arrhythmia syndrome is:
A. ECHO
B. ECG
C. chest X-ray
D. phonocardiography
E. coronary angiography

76. The most informative method for confirming necrotic changes in the myocardium is
A. determination of erythrocyte sedimentation rate and leukocytes
B. determination of the level of alkaline phosphatase
C. Determination of transaminase level
D. determination of troponin level
E. determination of creatinine level

77. Identification on the ECG of the interval PQ, equal to 0.28 seconds. indicates the
presence of the patient
A. sinoatrial blockade
B. blockade of atrioventricular conduction
C. extrasystolic arrhythmia
D. atrial fibrillation
E. paroxysmal ventricular tachycardia
78. The most reliable sign of stenosis of the left atrioventricular orifice is
A. increase the left border of the heart
B. facies mitralis
C. mesodiastolic murmur and the presence of a "opening click" mitral
valve at the top
D. smoothness of the waist of the heart during X-ray
E. weakening of the apical impulse

79. For mitral valve insufficiency it is typical:


A. clapping the first tone at the top
B. systolic murmur at the apex
C. increase the boundaries of the heart to the right
D. weakening of the II tone on the aorta
E. diastolic murmur at the apex

80. Blood pressure in aortic valve insufficiency:


A. only systolic increases
B. only diastolic increases
C. systolic and diastolic increases
D. Systolic decreases and diastolic increases
E. does not change

81. Indicate aortic valve insufficiency:


A. 120/90 mm Hg
B. 190/140 mm Hg
C. 160/100 mm Hg
D. 160/40 mm Hg
E. 90/60 mm Hg

82. What can be observed in endocardial inflammation syndrome?


A. leukopenia, lymphocytosis
B. anemia, leukocytosis, accelerated ESR
C. eosinophilia, accelerated ESR
D. anemia, lymphocytosis, accelerated ESR
E. anemia, leukopenia, thrombocytopenia

83. Specify the level of blood pressure, corresponding to the I degree of arterial
hypertension:
A. Blood pressure - 150/95 mm Hg
B. Blood pressure - 138/88 mm Hg
C. Blood pressure - 174/108 mm Hg
D. Blood pressure - 202/114 mm Hg
E. Blood pressure - 200/90 mm Hg

84. For the development of coronary insufficiency:


A. increased levels of low density lipoprotein
B. Age over 45
C. hypotension
D. male
E. genetic predisposition
85. Pulsation of the pupils is characteristic of:
A. mitral stenosis;
B. tricuspid insufficiency;
C. Aortic stenosis;
D. tricuspid stenosis;
E. aortic insufficiency

86. Which method is most effective in verifying left ventricular myocardia l hypertrophy:
A. ECG
B. veloergometry
C. Echocardiography
D. Coronary angiography
E. transesophageal pacing

87. Which of the following is an unmodified risk factor for the development of coronary
insufficiency:
A. increased levels of low density lipoprotein
B. diabetes
C. Arterial hypertension
D. smoking
E. genetic predisposition

88. When stenosis of the mouth of the aorta is heard


A. systolic murmur in the apex of the heart, radiating to the axillary region
B. diastolic murmur at the apex of the heart, radiating to the base of the xiphoid process
C. systolic murmur in the second intercostal space to the right of the sternum,
radiating to the carotid arteries
D. diastolic murmur in the second intercostal space to the right of the sternum radiating to
the carotid arteries
E. systolic murmur at the apex of the heart radiating to the base of the xiphoid process

89. The weakening of the I tone is observed at:


A. mitral insufficiency
B. tricuspid insufficiency
C. aortic insufficiency
D. mitral stenosis
E. in a healthy person

90. Specify the correct definition of the term "heart push":


A. ripple to the right of the xiphoid process
B. Local ripple in the third intercostal space at the left edge of the sternum
C. diffuse ripple throughout the precardiac area
D. Pulsation in the fourth, fifth, sixth intercostal space at the left border of the relative
dullness of the heart
E. ripple to the left of the xiphoid process

91. To detect pericardial effusion, the most informative method is


A. ECG
B. Coronary angiography
C. Echocardiography
D. X-ray examination of the chest organs
E. phonocardiography
92. Pulse pressure is:
A. Difference between systolic and diastolic blood pressure
B. stroke volume of the left ventricle
C. amount of blood released by the ventricles to systole
D. The amount of blood released by the atria to the ventricles
E. minute heart volume

93. Small, slow, rare pulse is characteristic for:


A. mitral stenosis
B. mitral insufficiency
C. aortic valve insufficiency
D. Aortic stenosis
E. thyrotoxicosis

94. Apical impulse for aortic insufficiency:


A. weakened, localized;
B. reinforced, localized;
C. tall, reinforced, spilled;
D. weakened, spilled
E. Reinforced, localized

95. Specify the cause of acute coronary insufficiency:


A. Coronary artery thrombosis
B. Inflammation of the coronary arteries
C. inflammation of the coronary veins
D. Arterial hypertension
E. Left ventricular myocardial hypertrophy

96. A 42-year-old patient has a significant shift in the boundaries of the relative dullness of the
heart to the left, increased apical impulse, a pronounced symptom of “systolic tremor” in the 2nd
intercostal space to the right of the sternum and ibid weakening of the II tone. Which of the
following auscultatory phenomena must necessarily occur in this patient?
A. systolic murmur at the apex
B. diastolic murmur at the apex.
C. systolic murmur above the aorta
D. systolic murmur at xiphoid process
E. diastolic murmur over the aorta

97. The most informative method for confirming the presence of heart disease is:
A. ECG
B. Echocardiography
C. roentgenoscopy of the chest
D. Coronary angiography
E. Bicycle ergometry test

98. Large pulse pressure, double Traube tone and duozier noise on the vessels, fast and high
pulse, Musset’s symptoms are characteristic
A. for aortic valve regurgitation
B. for aortic stenosis
C. for tricuspid valve insufficiency
D. for mitral stenosis
E. for mitral valve insufficiency

99. Indicate the physical sign of pericardial inflammation syndrome:


A. appearance of 3 tones
B. pansystolic noise at the top
C. appearance 4 tones
D. Pericardial friction noise
E. attenuation 1 tone

100. The orthopneic position is:


A. forced sitting position to relieve shortness of breath
B. forced sitting position to relieve pain
C. forced lying position to relieve pain
D. forced standing position to relieve hiccups
E. forced standing position to relieve shortness of breath

101. In case of pathology of which organ of the following, secondary arterial hypertension may
develop:
A. liver
B. adrenal glands
C. stomach
D. vessels of the lower extremities,
E. lungs

102. What is the peculiarity of pain in acute coronary insufficiency:


A. is associated with physical activity
B. Duration over 30 minutes
C. Docked with NTG
D. passes in peace
E. associated with breathing

103. The increase in pulse pressure is observed at:


A. Aortic stenosis
B. aortic insufficiency
C. mitral insufficiency
D. tricuspid insufficiency
E. mitral stenosis

104. Which echocardiography sign is the criterion for the diagnosis of endocardial inflammation
syndrome:
A. vegetation on the valves
B. mitral regurgitation
C. dilatation of the left ventricle
D. cusp perforation
E. calcification of valve cusps

105. Echocardiographic sign of acute coronary insufficiency:


A. diffuse hyperkinesis of the walls of the left ventricle
B. diffuse hypokinesis of the walls of the left ventricle
C. Local hypokinesis of the walls of the left ventricle
D. Local hyperkinesis of the walls of the left ventricle
E. Left ventricular dilatation
106. Target organs for hypertension:
A. Heart, kidneys, brain, fundus vessels and aorta
B. Liver, lungs
C. heart, liver
D. fundus vessels, lungs
E. kidney, heart

107. The area of the mouth of the aorta in the norm is:
A. 1.5-2.5 cm2
B. 4-4.5 cm2
C. 4.5-5.5 cm2
D. 2.5-3.5 cm2
E. 1-2 cm2

108. The tone of the opening of the mitral valve is heard:


A. at the apex of the heart
B. in the second intercostal space on the right at the sternum
C. in the second intercostal space to the left of the sternum
D. at the base of the xiphoid process
E. at Botkin-Erb Point

109. For stenosis of the left atrioventricular orifice characteristic


A. systolic murmur at the apex of the heart, aggravated by exhalation
B. Accent and split II tone over the aorta
C. systolic murmur at the apex of the heart, aggravated by inspiration
D. Additional high-frequency tone in diastole, separated from tone II by 0.07-0.12 seconds
E. weakening of the I tone at the top

110. The most informative method for the diagnosis of myocardial inflammation syndrome:
A. ECG
B. ECHO
C. Bicycle ergometry
D. Coronary angiography
E. radiography

111. What is the symptom characteristic of the appearance of patients with stenosis of the
aortic mouth:
A. diffuse cyanosis of the skin
B. pallor of the skin
C. symptom Musset
D. "dancing carotid"
E. capillary pulse

112. Of the listed risk factors for coronary insufficiency, the most significant “triad” is:
A. obesity, hypodynamia, male
B. hypercholesterolemia, hypertension, smoking
C. Arterial hypertension, hypodynamia, obesity
D. Impaired carbohydrate tolerance, obesity, smoking
E. menopause, obesity, hypodynamia
113. A common symptom in the objective status of patients with aortic valve insufficiency
and aortic stenosis is:
A. pallor of the skin
B. symptom Musset
C. pulsation of the carotid, subclavian, temporal, brachial arteries
D. capillary pulse
E. Decrease in diastolic blood pressure

114. For patients with myocardial inflammation syndrome, the following complaints are
most characteristic:
A. pain in the heart, palpitations, shortness of breath
B. pain in the heart, palpitations, fainting
C. pain in the heart, shortness of breath, ascites
D. pain in the heart, fever, dry cough
E. fainting, fever

115. Specify the anti-atherogenic class of lipoproteins:


A. total cholesterol
B. HDL-cholesterol
C. LDL cholesterol
D. triglycerides
E. chylomicrons

116. Click open mitral valve


A. occurs 0.06-0.12 seconds after the closure of the aortic valves
B. characteristic of mitral insufficiency
C. characteristic of aortic stenosis
D. best heard at Botkin-Erb
E. characteristic of aortic insufficiency

117. Paradoxical pulse is:


A. alternation of pulse waves of large and small filling;
B. decrease or disappearance of inhalation pulse waves;
C. discrepancy between the number of pulse waves and the number of heartbeats;
D. unequal pulse wave intervals
E. amplification of the pulse wave on the exhale

118. Name the ECG-sign, which is a reflection of myocardial ischemia:


A. decrease in the amplitude of the R wave
B. Deep, broad Q wave
C. ST segment depression
D. deformation of the QRS complex
E. deformation of the P wave

119. Quincke precapillary pulse occurs when:


A. stenosis of the mouth of the aorta;
B. aortic valve insufficiency;
C. mitral stenosis;
D. mitral insufficiency;
E. pulmonary insufficiency.
120. One of the following laboratory indicators confirms the syndrome of acute coronary
insufficiency in a patient with an anginal attack:
A. decreased hemoglobin
B. Hyperlipidemia
C. Increased ESR
D. Increased activity of serum troponins
E. lymphocytosis

121. Conduction disturbances include:


A. Extrasystolic arrhythmia
B. atrial fibrillation
C. paroxysmal tachycardia
D. AV block
E. sinus tachycardia

122. Hypertension is characterized by the following level of increase in blood pressure:


A. ≥130 / 90 mm Hg
B.> 140/100 mm Hg
C. ≥140 / 90 mmHg
D. ≥130 / 80 mm Hg
E. ≥120 / 80 mm Hg

123. The appearance of a large-wave undifferentiated line on the ECG in the absence of a P wave
and a QRS complex indicates:
A. Atrial fibrillation
B. ventricular fibrillation
C. heart asystoles
D. ventricular tachycardia
E. extrasystolic arrhythmia

124. When asystole on the ECG is recorded:


A. small wave undifferentiable line
B. large-wave undifferentiable line
C. straight line
D. Absence of P wave and change of QRS form
E. the presence of pauses lasting up to 3 seconds

125. The right border of the absolute dullness of the heart is normally located:
A. on the right edge of the sternum;
B. on the left edge of the sternum;
C. 3 cm medially from the right mid-clavicular line;
D. along the percutaneous right line.
E. on the midclavicular line

126. For the second tone of the heart, the following statement is true:
A. between II and I heart tones a short pause;
B. louder at the base of the heart;
C. coincides with the apical impulse;
D. coincides with the pulse of the carotid arteries;
E. slightly lower and longer than I tone.

127.Acrocyanosis is determined by:


A. on the back
B. on mucous membranes
C. on auricles, nose tip, fingers and toes
D. in the forearm area
E. on the lower limbs

128. The most informative laboratory test confirming the diagnosis of endocardial inflammation
syndrome is:
A. sharp acceleration of ESR
B. anemia
C. neutrophilic leukocytosis
D. leukopenia
E. Positive blood culture at blood culture

129. Accent II tone in the 2nd intercostal space on the right is noted when:
A. Aortic stenosis
B. aortic insufficiency
C. Arterial hypertension
D. pulmonary arterial hypertension
E. tachycardia

130. What is the level of blood pressure of the following corresponds to the 3rd degree of arterial
hypertension
A. 140/90 mm Hg
B. 180/110 mm Hg
C. 160/100 mm Hg
D. 150/99 mm Hg
E. 170/100 mm Hg

131. Normally, the duration of the ventricular QRS complex:


A. more than 0.12 "
B. More than 0.11 "
C. less than 0.10 "
D. less than 0.14 "
E. less than 0.02 "

132. Wave P on ECG reflects:


A. Depolarization of the interventricular septum
B. depolarization of the left ventricle
C. repolarization of the right ventricle
D. Atrial depolarization
E. repolarization of the left ventricle

133. In mitral stenosis, the borders of the heart are displaced:


A. Left and up
B. Left and down
C. right and left
D. Right and up
E. not shifted

134. In mitral insufficiency, the borders of the heart are displaced:


A. Left and up
B. Left and down
C. right and left
D. not shifted
E. Right and up

135. In case of insufficiency of the aortic valve, the border of the heart is displaced:
A. Left and up
B. Left and down
C. right and left
D. not shifted
E. Right and up

136. With compensated stenosis of the aortic aperture, the borders of the heart are displaced:
A. Left and up
B. Left and down
C. right and left
D. not shifted
E. Right and up

137. What level of blood pressure of the following do you consider normal?
A. 160/70 mm Hg.
B. 180/80 mm Hg
C. 120/90 mm Hg
D. 140/80 mm Hg
E. 130/80 mm Hg

138. The mitral configuration of the heart is:


A. unchanged borders of the heart
B. enlargement of the right ventricle
C. enlargement of the left ventricle
D. enlargement of both ventricles
E. enlargement of the left atrium and cone of the pulmonary artery

139. On the ECG, the absence of P-waves, waves f and different distances RR are characteristic
of:
A. Atrial flutter
B. ventricular flutter
C. atrial fibrillation
D. supraventricular paroxysmal tachycardia
F. extrasystolic arrhythmia

140. On an ECG, ventricular extrasystolic arrhythmia is characterized by the appearance of:


A. ST segment depression
B. widened and deformed QRS after unchanged P wave
C. premature extraordinary altered ventricular QRS complex
D. Premature extraordinary appearance of unchanged ventricular QRS complex
E. periodic loss of the QRS complex

141. Affection of the heart in the syndrome of primary arterial hypertension is manifested:
A. hypertrophy of the left atrium
B. Right ventricular hypertrophy
C. Left ventricular hypertrophy
D. dilatation of the left atrium
E. hypertrophy of the right atrium

142. Select the criterion for left ventricular hypertrophy:


A. R / S V1> 1
B. R / S V5 <1
C. Rv5 + Sv1 ≥38mm
D. Sv1 + Rv5 ≥10.5mm
E. Rv1> 7mm

143. What changes on the ECG do you expect to get in a patient with chronic coronary
insufficiency syndrome outside the attack:
A. the appearance of negative T waves
B. ST elevation above isoline
C. Shift down the ST segment
D. normal ECG
E. Pathological Q wave

144. The most characteristic complaints in chronic coronary insufficiency are:


A. pain in the heart of stabbing nature
B. pain in the left half of the chest for up to several hours, associated with the act of breathing
C. pain in the left half of the chest, resulting from a change in body position
D. burning pains behind the sternum, radiating to the neck and left arm, occurring during
physical exertion, arrested at rest or after taking nitroglycerin in 1-2 minutes
E. Stitches in the precordial region, at night, up to 3-4 hours

145. In mitral stenosis I tone:


A. weakened at the top
B. strengthened at the base of the xiphoid process
C. “flapping” at the top
D. weakened in the second intercostal space on the right
E. strengthened in the second intercostal space on the left

146. The tone of the opening of the mitral valve is heard:


A. at the apex of the heart;
B. in the second intercostal space on the right at the sternum;
C. in the second intercostal space to the left of the sternum;
D. at the base of the xiphoid process
E. at Botkin-Erb Point

147. The patient at the 4th auscultation point revealed a systolic murmur, which increases with
inspiration. This is typical for:
A. mitral valve insufficiency;
B. stenosis of the right atrioventricular orifice;
C. tricuspid valve insufficiency;
D. mitral stenosis;
E. pulmonary valve insufficiency.

148. ECG changes with stenosis of the aortic mouth:


A. signs of left ventricular hypertrophy
B. signs of left atrial hypertrophy
C. signs of right ventricular hypertrophy
D. Signs of right atrial hypertrophy
E. blockade of the right bundle branch block

149. Sinus bradycardia is often observed:


A. when eating
B. well-trained athletes at rest
C. hypotension
D. emotional stress
E. Arterial hypertension

150. For pericardial friction noise is typical:


A. listened in places auscultation valves;
B. held in the axillary region;
C. increases with pressure on the chest with a stethoscope;
D. is heard in the zone of relative dullness of the heart;
E. disappears when inhaling.

151. ECG is a sign of atrial extrasystolic arrhythmia:


A. the presence of a P wave in front of an extraordinary unchanged QRS complex followed
by a compensatory pause
B. irregular rhythm with QRS complex deformation
C. loss of every other QRS complex
D. the absence of a P wave in front of an extraordinary QRS complex, an extension of the QRS
complex
E. F waves

152. Three-leaved valve is better heard


A. in the fifth intercostal space on the left 1.5 cm inwards from the midclavicular line
B. in the second intercostal space to the left of the sternum
C. in the second intercostal space to the right of the sternum
D. to the right of the sternum in the fourth intercostal space
E. at Botkin-Erb Point

153. Paroxysmal supraventricular tachycardia on ECG is manifested:


A. increase in heart rate to 160 min
B. the presence of wide deformed QRS complexes with a frequency of 140-220 per minute
C. frequent irregular appearance of ventricular QRS complexes without s.P
D. normal unchanged QRS complexes with a frequency of> 160 per minute
E. the emergence of extraordinary QRS complexes.

154. Pathological Q wave on the ECG is detected when:


A. for chronic coronary insufficiency
B. with left ventricular hypertrophy;
C. with right ventricular hypertrophy;
D. Acute coronary insufficiency
E. inflammation of the pericardium

155. Determine the characteristic color of the skin with endocarditis:


A. pallor
B. yellowness
C. Limited cyanotic blush
D. Color "coffee with milk"
E. unilateral hyperemia of the cheeks

156. What level of blood pressure of the following corresponds to arterial hypertension of II
degree:
A. Blood pressure - 150/94 mm Hg
B. AD - 138/80 mm Hg
C. AD - 174/108 mm Hg
D. Blood pressure - 160/114 mm Hg
E. Blood pressure - 200/100 mm Hg

157. The composition of atherosclerotic plaque includes:


A. tire and plaque core
B. spikes, tire
C. nucleus and processes
D. podocytes
E. Podocytes and nucleus

158.Laboratory indicators characteristic of acute coronary insufficiency:


A. an increase in troponin I or T
B. anemia
C. proteinuria
D. lymphopenia
E. leukocyturia

159. The most common causes of myocardial inflammation syndrome are:


A. Streptococcus
B. viruses
C. Staphylococcus
D. fungi
E. parasites

7
Eralieva M.O., Assistant Course Hematology, Department of Hospital Therapy,
Occupational Pathology with a course of Hematology, Ph.D., coordinator of the
hemopoietic system for foreign students

Exam questions of test control in the module “Hematopoietic system” for 3-year
students Foreign Language

The normal content of iron in serum is:


A. 2-10 µmol / l

B. 6-15 µmol / l

C. 10-28 μmol / l

D. 12-30 µmol / l

E. 15-40 µmol / l

Normal blood levels of Hb in adults are:

A. 90-140 g / l

B. 100-150 g / l

C. 110-160 g / l

D. 120-170 g / l

E. 130-180 g / l

Anemic syndrome manifested:

A. perversion of taste

B. dry skin

C. causeless general weakness

D. hair loss

E. deformation of nails

Hyposiderosis is characterized by:

A. perversion of smell

B. dizziness

C. causeless general weakness

D. palpitations

E. shortness of breath

5
Sideropenic syndrome develops with:

A. hemolysis of erythrocytes

B. folic acid deficiency

C. vitamin B12 deficiency

D. chronic blood loss

E. aplasia of the bone marrow

Body containing the main reserves of iron in the body:

A. liver

B. Spleen

C. kidney

D. brain

E. lungs

What research is needed for hypochromic anemia:

A. blood bilirubin

B. creatinine

C. rheumatological tests

D. serum ferritin

E. blood sugar

Iron deficiency is characterized by:

A. megaloblastic type of blood formation

B. hypochromia of erythrocytes

C. normochromia of erythrocytes

D. pancytopenia

E. hyperbilirubinemia
9

The normal content of ferritin in the serum is:

A. 10 - 12 µg / l

B. 80 - 350 µg / l

C. 10 - 30 μg / l

D. 15 - 300 µg / l

E. 5 - 175 µg / l

10

Erythrocytic poikilocytosis is:

A. change in the average diameter of erythrocytes

B. change in the form of red blood cells

C. red blood cells with basophilic point

D. Jolly red blood cells

E. Erythrocytes with kebot rings

11

Erythrocytic anisocytosis is:

A. red blood cells with pathological inclusions

B. target red blood cells

C. red blood cell hyperchromia

D. oval red blood cells

E. Presence in the blood of erythrocytes of various sizes

12

Organ, containing vitamin B12 in the body:

A. brain

B. adrenal glands

C. liver

D. kidneys
E. spleen

13

The absorption of vitamin B12- and folic acid occurs mainly:

A. in the stomach

B. in the duodenum

C. in the upper small intestine

D. in the colon

E. in the lower small intestine

14

The syndrome of the funicular myelosis is associated with the formation of:

A. thymidine

B. methylmalonic acid

C. folic acid

D. ascorbic acid

E. nicotinic acid

15

Macrocytosis and hyperchromia of erythrocytes caused by:

A. hyposiderosis

B. folic acid deficiency

C. hemosiderosis

D. red blood cell hemolysis

E. erythrocytosis

16

Vitamin B12 deficiency is manifested:

A. macrocytes

B. microcytes

C. normocytes
D. microspherocytes

E. sickle cell erythrocytes

17

Hemolytic syndrome manifests is:

A. pale skin

B. dry skin

C. yellowness of the skin

D. leukemia on the skin

E. trophic ulcers on the skin

18

Erythrocyte hemolysis results in:

A. direct hyperbilirubinemia

B. indirect hyperbilirubinemia

C. hyperbilirubinemia of both fractions

D. Hypobilubinemia

E. abilirubinemia

19

Hemolysis of erythrocytes due to the formation of anti-erythrocyte antibodies is


confirmed by:

A. direct breakdown of Coombs

B. indirect Coombs breakdown

C. direct sucrose breakdown

D. hemosiderin urine

E. hemoglobinuria

20

Dysmetabolic degeneration of the lateral and partially posterior spinal cord of the
spinal cord with vitamin B12 deficiency leads to the development of:
A. anemic syndrome

B. neuro-anemic syndrome

C. Sideropenic syndrome

D. hemolytic syndrome

E. neuroleukemia

21

Reticulocytosis develops with:

A. anemic syndrome

B. sideropenic syndrome

C. hemolytic syndrome

D. iron overload syndrome

E. hyposiderosis

22

Areticulocythemia develops when:

A. the presence of antibodies against bone marrow erythrocaryocytes

B. the presence of antibodies against peripheral blood erythrocytes

C. fermentopathy of erythrocytes

D. hemoglobinopathies

E. erythrocyte membranopathyes

23

The cause of red blood cell microcytosis:

A. increased red blood cell sequestration in the spleen

B. excess of “old” red blood cells

C. increasing the number of young red blood cells

D. Reduction of Hb content in the erythrocyte

E. intravascular hemolysis of erythrocytes

24
When examining the patient revealed pancytopenia. Your actions in the survey
plan:

A. determination of ferritin

B. direct Coombs test

C. liver tests

D. Creatinine and other kidney tests

E. bone marrow examination

25

Syndrome of the funicular myelosis appears:

A. violation of tactile and pain sensitivity

B. general weakness

C. hyperplasia of lymphatic tissue

D. perversion of smell

E. dysphagia

26

The normal content of platelets in the blood is:

A. 100-200х109 / l

B. 150-300x109 / l

C. 170-350х109 / l

D. 150-400x109 / l

E. 150-450х109 / l

27

Platelet ancestors are:

A. reticulocyte

B. megakaryocytes

C. myelokaryocytes

D. Oxyphilic normocytes
E. polychromatophilic normocytes

28

Megakaryocytes are precursors to:

A. reticulocyte

B. neutrophils

C. lymphocyte

D. platelet

E. erythrocyte

29

The life span of platelets is:

A. 12-24 hours

B. 24-48 hours

C. 3-5 days

D. 8-11 days

E. 15-20 days

30

When thrombocytopenia develops:

A. petechial-bruise type of bleeding

B. hematoma type of bleeding

C. vasculitic-purple type of bleeding

D. petechial hematoma type of bleeding

E. angiomatous type of bleeding

31

Deficiency of coagulation factors VIII and IX leads to the development of:

A. petechial- bruise type of bleeding

B. vasculitic-purple type of bleeding

C. hematoma type of bleeding


D. angiomatous type of bleeding

E. petechial-hematoma type of bleeding

32

Vasculitis-purple type of bleeding due to:

A. thrombocytopenia

B. thrombocytopathy

C. pathology of capillary endotheliocytes

D. congenital inferiority of capillary walls

E. deficiency of coagulation factors VIII and IX

33

Petechial hematoma type of bleeding develops when:

A. thrombocytopenia

B. thrombocytopathy

C. deficiency of coagulation factors VIII and IX

D. Willebrand factor deficiency

E. congenital inferiority of the capillary walls

34

Congenital inferiority of the capillary walls with the expansion of the lumen of blood
vessels leads to the development of:

A. vasculitic-purple type of bleeding

B. angiomatous type of bleeding

C. hematoma type of bleeding

D. petechial-Sinyachkov type of bleeding

E. petechial-hematoma type of bleeding

35

Critical thrombocytopenia:

A. 50x109 / l
B. 30x109 / l

C. 20x109 / l

D. 15x109 / L

E. 10x109 / l

36

Platelets are formed from one megakaryocyte:

A. 1000-2000

B. 2000-4000

C. 3000-6000

D. 4000-8000

E. 5000-10000

37

Hemorrhagic syndrome in aplasia of the blood due to:

A. anomalous structure of vessel walls

B. Increased destruction of platelets in the blood

C. deficiency of blood coagulation factors

D. increased platelet consumption

E. insufficient formation of megakaryocytes

38

Which of the laboratory signs is pathognomic for the deficiency of coagulation


factors?

A. prolonged bleeding time

V. decrease in the number of platelets

C. lengthening of blood clotting time

D. Platelet dysfunction

E. reducing the number of megakaryocytes


39

In what type of bleeding are intermuscular, subcutaneous, retroperitoneal


hematomas observed?

A. vasculitic purple

B. angiomatous

C. hematomic

D. Microcirculatory

E. petechial-bruise

40

Extending of the activated partial thromboplastin time (APTT) is observed with:

A. petechial-bruising type of bleeding

B. vasculitic-purple type of bleeding

C. hematoma type of bleeding

D. Angiomatous type of bleeding

E. microcirculatory type of bleeding

41

Lengthening the duration of bleeding time by Duke occurs when:

A. thrombocytopenia

B. Intravascular thrombosis

C. congenital coagulopathy

D. Acquired coagulopathy

E. intravascular hemolysis of erythrocytes

42

Platelet dysfunction:

A. thrombocytopenia

B. thrombocytopathy

C. angiopathy
D. coagulopathy

E. Vasopathy

43

The first stage of coagulation hemostasis ends with the formation of:

A. active prothrombinase

B. thrombin

C. plasmin

D. fibrin

E. prothrombin

44

The second stage of coagulation hemostasis ends with the formation of:

A. active prothrombinase

B. prothrombin

S. thrombin

D. fibrin

E. plasmin

45

Physiological anticoagulant is:

A. plasminogen

B. plasmin

C. trombomodulin

D. Kallikrein

E. von Willebrand factor

46

Hyperplastic syndrome:

A. hypoplasia of lymphoid tissue

B. hyperplasia of lymphoid tissue


C. hypoplasia of myeloid tissue

D. hyperplasia of myeloid tissue

E. aplasia of myeloid tissue

47

Plasma cells are formed from:

A. B-lymphocytes

B. T-lymphocytes

C. monocytes

D. neutrophils

E. eosinophils

48

Botkin-Humprecht cells:

A. abnormal monocytes

B. hemolyzed erythrocytes

C. polysegmented neutrophils

D. destroyed lymphocytes

E. Activated platelets

49

The spleen is located at the level of:

A. VIII-X ribs

B. IX-XI ribs

C. X-XII ribs

D. XI-XII ribs

E. VIII-XII ribs

50

Spleen is covered with peritoneum:

A. on the one hand


B. from two sides

C. from three sides

D. all around

E. not covered on one side

51

Bone marrow is located in:

A. in the diaphysis of the bones

B. in bone metaphysis

C. cells of spongy bone

D. in compact bone matter

E. in the periosteum

52

The central organs of the hematopoietic system include:

A. lymph nodes

B. Spleen

C. thymus gland

D. liver

E. lymphocytes

53

Which organ has white and red pulp?

A. thymus gland

B. lymph node

C. bone marrow

D. spleen

E. Liver

54

What tissue makes up the soft marrow stroma?


A. Stratified squamous nonthreshold epithelium

B. reticular

C. mucous

D. lymphoid

E. ferrous

55

Hypersplenism:

A. splenomegaly + cytopenia

B. hepatosplenomegaly

C. splenomegaly + lymphadenopathy

D. Splenomegaly + Erythrocytosis

E. splenomegaly + viral infection

56

Patient I., 34 years old, complaints of general weakness, fatigue, dizziness, headaches,
shortness of breath and palpitations with little exertion
From the anamnesis: heavy, prolonged menstrual bleeding.
What syndrome can you think of?
A. sideropenic syndrome

B. hemolytic syndrome

C. anemic syndrome

D. hemorrhagic syndrome

E. hyperplastic syndrome

57

Patient S., 28 years old, went to a doctor's office with complaints of taste perversion, hair
loss, brittle nails, difficulty in swallowing food, dry skin.

CBC: HB 128 g / l, erythr. 4,7 h1012 / l, MCV 62 fl, MCH 18 pg.

What syndrome are we talking about?

A. anemic syndrome

B. hemolytic syndrome
C. funicular myelosis

D. Sideropenic syndrome

E. hyperplastic syndrome

58

Patient J., 42 years old, appealed to the outpatient clinic. Complaints when going to the
doctor: yellowness of the skin and visible mucous membranes, change in color of urine as
a strongly brewed tea, feeling of heaviness in hypochondria, occasionally itchy skin.

Total bilirubin 78.4 µmol / l, mainly due to the indirect fraction.

What syndrome does the patient have?

A. anemic syndrome

B. sideropenic syndrome

C. funicular syndrome

D. hemorrhagic syndrome

E. hemolytic syndrome

59

Patient D., 19 years old, was found to have: HB 58 g / l, eryth.3.7x10 12 / l, MCV 58 fl,
MCH 15 pg, thromb.325.0x109 / l, leuk.4.3h109 / l.

What changes are there in the total blood count?

A. normocytic, normochromic anemia

B. macrocytic, hyperchromic anemia

C. microcytic, hypochromic anemia

D. megalocytic, hyperchromic anemia

E. normocytic, hypochromic anemia

60

Patient S., 70 years old, complains of general weakness, fatigue, shortness of breath and
palpitations at the slightest physical exertion, unsteadiness of gait, feeling of crawls all
over the body, change of color of urine.

CBC: HB 62 g / l, erytr.1.7x1012 / l, MCV 118 fl, MCH 36pg, thromb. 155.0x109 / l,


leuk.3.9x109 / l.
What changes are there in the total blood count?

A. normocytic, normochromic anemia

B. macrocytic, hyperchromic anemia

C. microcytic, hypochromic anemia

D. normocytic, hyperchromic anemia

E. normocytic, hypochromic anemia

61

Patient A., 26 years old, was admitted to the hospital with complaints of marked general
weakness, fatigue, headaches, dizziness, blurred vision, shortness of breath and
palpitations at the slightest physical exertion, yellowness of the skin, feeling of heaviness
and pain in the left hypochondrium, change in urine color .

CBC: erythrocytes 1.6x1012 / l, Hb 48 g / l, MCV 92 fl, MCH 31 pg, platelets 187.4x10 9 /


l, leukocytes 8.6x109 / l: fell. 6, segm.72, mon. 6, lymph 16, ESR 32 mm / hour.

What is determined by the general blood test?

A. normocytic, normochromic anemia

B. macrocytic, hyperchromic anemia

C. microcytic, hypochromic anemia

D. normocytic, hyperchromic anemia

E. normocytic, hypochromic anemia

62

Patient I., 16 years old, during the study found: peripheral blood platelet count 20.0x10 9 /
l; the number of bone marrow megakaryocytes in 1 μl of 185 cells.

What type of bleeding can a patient develop?

A. petechial-bruise

B. vasculitic purple

C. hematoma

D. angiomatous

E. petechial hematoma

63
Patient I., 17 years old, was admitted to the clinic with complaints of nasal bleeding, pain
in the right knee joint, aggravated by the slightest movement.

History: ill from early childhood.

CBC: erythrocytes 4,6x1012 / l, Hb -148 g / l, MCV 64 fl, MCH 30pg, platelets 410x109 /
l, leukocytes 4,6x109 / l: fell. 6, segm.72, mon. 6, lymph 16, ESR 12 mm / hour.

What type of hemorrhagic syndrome does the patient have?

A. petechial-bruise

B. vasculitic purple

C. hematoma

D. angiomatous

E. petechial hematoma

64

Patient U., 20 years old, was admitted to the clinic with complaints of multiple lesions on
the skin of the lower extremities, back, pain in the ankle and knee joints, aggravated by
physical exertion, abdominal pain, nausea, single vomiting, liquid stool of a dark color,
increase in body temperature to 37.8 - 38.00 C, general weakness,

Anamnesis: ill acutely, the onset of the disease is associated with transferred flu.

CBC: HB 130 g / l, erythrocytes 4.2x1012 / l, platelets 416.6x109 /l, leukocytes 13.8x109 /


l: neut - 67, basoph. 2, eosin. 3 mon 4, lymph. 24, ESR 24 mm / hour.

To determine the option of hemorrhagic syndrome?

A. petechial-bruise

B. vasculitic purple

C. hematoma

D. angiomatous

E. petechial hematoma

65

Patient I., 26 years old, came to the clinic. According to the patient, it became clear that
during the last 6 months she began to notice the appearance of petechial rashes on the
extremities, bruises after minor injuries, an increase in the duration of menstrual bleeding
to 10-12 days.
What research is shown?

A. blood coagulation factors VIII, IX

B. APTT

C. platelet count, megakaryocytes

D. Prothrombin time, INR

E. coagulation factors I, VII

66

Patient Z., 26 years old, complaints of an increase in cervical, submandibular, axillary,


inguinal lymph nodes, a feeling of heaviness and pain in the right and left hypochondria.

CBC: HB 126 g / l, erytr. 3,9x1012 / l, thromb. 176.0 x109 / l, leuk -26.2 x109 / l: neut -
38, eosin. 2, lymph.- 60, ESR 24 mm / hour.

What is the hematopoietic lesion syndrome in a patient?

A. hemorrhagic

B. hemolytic

C. necrotic ulcer

D. Funicular myeosis

E. hyperplastic

67

Patient K., 35 years old, complaints of a feeling of heaviness and pain in the left
hypochondrium, aggravated by physical activity, after eating, and general weakness.

Objectively: The general condition is severe. Peripheral lymph nodes are not palpable.
The tongue is coated with white bloom, dry. The abdomen is enlarged, asymmetric due to
the enlarged spleen. The lower edge of the spleen at the entrance to the pelvis, extends
beyond the median line by 3-4 cm, dense, flat surface, sensitive to palpation. The liver is
not palpable.

What syndrome are we talking about?

A. hyperplastic

B. hepatomegaly
C. splenomegaly

D. hepatolienal

E. hepatorenal

68

A patient with lymph proliferative disease has generalized lymphadenopathy.

What blood cells can be involved in this case?

A. neutrophils

B. monocytes

C. lymphocytes

D. eosinophils

E. basophils

69

Patient Z., 68 years old. During 6 years it is observed by a hematologist. According to the
testimony receives treatment in an outpatient hospital setting. Upon admission, the
condition is severe: a diffuse increase in all groups of peripheral lymph nodes in large
sizes, dense consistency; splenohepatomegaly; manifestations of exudative pleurisy on
the right.

What research to do first?

A. complete blood count + platelets

B. radiography of the chest

C. liver tests

D. renal tests

E. Ultrasonography of internal organs

70

Patient K., 67 years old, was diagnosed with peripheral blood: HB-138 g / l, erythrocytes-
4,1x1012 / l, MCV 84 fl, MCH 30 pg, platelets-216,8x109 / l, leukocytes-6,4x109 / l: neut
24, lymph. 70, mon. 6, ESR 15 mm / hour.

What changes are there in the total blood count?

A. erythrocytosis
B. thrombocytosis

C. leukocytosis

D. Lymphocytosis

E. monocytosis
Q. A 71-year-old male patient with type 2 diabetes complained of frequent episodes of excess
sweating, feeling of hunger, and tremor in the previous week manifesting as. His diabetes control
was excellent (HbA1c 6.5%) with glimepiride 4 mg daily and metformin 2 g daily. His renal function
was normal and he did not have microalbuminuria. No changes in dietary habits and exercise
program were reported.

A. Hypoglycaemia

Q. Patient B. went to the clinic with complaints of severe weakness, rapid fatigue, disorders in the
activity of the gastrointestinal tract. Lost in the last 4 months weight 18 kg. An objective
examination revealed: a sharp depletion of the body, the turgor of the skin is reduced, the
phenomenon of dehydration of the body, the skin on the palms, in the places of folds and the
greatest friction on clothes, are pigmented much more strongly than those surrounded

A. Hypercortisolism

Q. Patient N. 25 years old suffered a severe form of acute respiratory infections. Already during
the recovery period, there was a feeling of pressure in the left neck region. There were pains when
swallowing, body temperature 38. The left lobe of the thyroid gland is enlarged, dense, sharply
painful.

A. Subacute parathyroidism

Q. Patient M is a white woman, 32 years of age, presenting to her primary care physician with
complaints of polyuria for the past four weeks. Further assessment reveals an upcoming
appointment with her optometrist for new onset blurred vision. The patient is 5 feet 5 inches tall
and weighs 142 pounds; her calculated BMI is 23.6 kg/m2. She confirms a family history of
diabetes on her father's side and admits to a generally sedentary occupation and lifestyle. A
random finger stick reveals a blood glucose level of 257 mg/dL.

HbA1c: 9% (estimated average glucose: 208 mg/dL)

Fasting blood glucose: 151 mg/dL

A. Syndrome of hyperglycaemia

Q. Patient S., 28 years old, came to the clinic with complaints of general weakness, malaise,
headaches, hair loss, dry skin, an increase in the size of the thyroid gland. From the anamnesis:
during the year, the above-described complaints are worried, gradually progressing. Objectively:
The skin is dry, pale, body mass index - 32. Pulse 59 in min., BP - 125/70 mm Hg. Art., muffled
heart tones. Thyroid increased to I degree according to WHO, compacted, painless. Regional
lymph nodes are not enlarged

A. Hyperthyroidism

Q. Patient J., 32 years old. Complains of constant headache, pain in lumbar spine, increased blood
pressure, general weakness, irritability, overweight. The first symptoms of the disease appeared 2
years ago. Within 3 years observed by a therapist for hypertension. Antihypertensive therapy is
not effective. Objectively. Height - 178 cm, body weight - 101 kg., Disproportionate deposition of
subcutaneous adipose tissue (mainly in the face area, neck, torso. On the inner surface of the
shoulders, in the lateral areas abdomen, in the area of the thighs purple-red stripes. The skin is
dry. Pulse-82 in 1 min. BP - 180/95 mm Hg Additional research. On ultrasound - an increase in the
size of both adrenal glands. The level of cortisol, ACTH in the blood is increased. On The
radiograph of the Turkish saddle did not reveal any pathology. OGTT (glucose blood): 6.0 - 11.1
mmol / l.

A. Hypercortisolism

Q. History: A 35 year old nurse complained of nervousness, weakness, and palpitations with
exertion for the past 6 months. Recently, she noticed excessive sweating and wanted to sleep with
fewer blankets than her husband. She had maintained a normal weight of 120 pounds but was
eating twice as much as she did 1 year ago. Menstrual periods have been regular but there was
less bleeding. Physical examination: Pulse was 92/minute and BP was 130/60. She appeared
anxious, with a smooth, warm, and moist skin, a fine tremor, a bounding cardiac apical impulse, a
pulmonic flow murmur, and she couldn't rise from a deep knee bend without aid. Her thyroid
contained 3 nodules, 2 on the right and one on the left with a total gland size of 60 grams (3 times
normal size), all nodules being of firm consistency and there was no lymphadenopathy. Her eyes
were not prominent (proptotic) and she had no focal skin thickening.

Laboratory studies: Serum T4=15.6 ug/dl and serum T3=250 ng/dl (N=80-160).

A. Hyperthyroidism

Q. Patient R., 25 years old, average height. The face is moon-shaped, the skin on it is crimson.
Excessive fat deposition on the abdomen and thighs. The bones are thin. Red stripes are marked
stretching on the skin of the abdomen and shoulders. Blood pressure 160/90 mm Hg. Art. Blood
sugar 7.0mmol / l. X-ray: the Turkish saddle is dilated.

Hypercortisolism
Q. An 82-year-old male patient was taken to the emergency room in the afternoon for loss of
consciousness in the previous hour. The patient had hypertension, chronic ischemic heart disease,
and mild diabetes treated with 30 mg gliclazide MR daily. On examination the patient had coma
(Glasgow scale 5) and right hemiplegia. An immediate blood glucose determination in the
emergency room using a portable glucose meter was 30 mg/dl (1.7 mmol/L).

A. Hypoglycaemia

Q. A 71-year-old male patient with type 2 diabetes complained of frequent episodes of excess
sweating, feeling of hunger, and tremor in the previous week manifesting as. His diabetes control
was excellent (HbA1c 6.5%) with glimepiride 4 mg daily and metformin 2 g daily. His renal function
was normal and he did not have microalbuminuria. No changes in dietary habits and exercise
program were reported.

A. Hyperglycaemia

Q. A 53-year-old woman presents to her primary care provider with complaints of fatigue, weight
gain, and constipation. She states that her weight has gradually increased over the last year
despite no change in her activity level or eating habits. She works roughly 8 hours a day as a
nutrition assistant, but she falls asleep as soon as she gets home in the afternoon. She lacks
motivation to do anything during the weekend, and notes that she does not feel like herself. Upon
questioning, the patient reports the following changes: thinning of her scalp hair, brittle nails, dry
skin, and cold intolerance. Physical examination confirms dry skin, brittle nails, coarse and dry
hair, and abdominal distension. The only medication she takes is a multivitamin daily. She has a
positive family history for autoimmune disorders and stroke.

A. Hypothyroidism

Patient N., 27 years old, after childbirth, complicated by massive bleeding, began to develop
exhaustion, the skin became dry, wrinkled, wax-colored. Body temperature - 360 С, BP - 100/60
mm Hg. Art., blood glucose - 3.3 mmol / l, the content of 17-ketosteroids in urine are lowered.

A. Hypocortisolism

Q. Patient A. 40-year-old applied to a polyclinic with complaints of an increase in body


temperature up to 39˚C, pain in the thyroid gland, aggravated by swallowing, coughing, chills,
tachycardia, weakness. Anamnesis: frequent tonsillitis. Objectively: On palpation, the thyroid
gland is sharply painful and dense. Laboratory: Complete blood count: leucocytosis with a shift to
the left, increased ESR. Choose the most informative thyroid examination method in this situation:
A. determination of T3, T4 in blood

B. thyroid scan with 131 I

C. ultrasound with biopsy

D. lateral radiography of the neck.

E. auscultation of the thyroid gland

A. True answer is: “C”

Ultrasound and biopsy. Because in case of acute thyroiditis, functionally it will stay stable and there
is no need to estimate levels of T3, T4.We need to do ultrasound in order to determine
inflammation, abscess and etc, and do biopsy in order to cultivate bacteria’s and determine
sensitivity of bacterias to certain group of antibiotics

Q. Patient D., 42 years old. Complains of headache, increased blood pressure, general weakness,
irritability, persistent cough. First symptoms diseases appeared 4 years ago. I did not go to the
doctor. Smokes since 18 years (pack of cigarettes per day). Objectively. Height is 188 cm, body
weight is 97 kg., Disproportionate deposition of subcutaneous adipose tissue (mainly in the face
area, neck, torso. Crimson red stripes on the thighs. The skin is dry. Pulse-82 in 1 min. BP 155/90
mm Hg Additional research. On ultrasound, the adrenal glands are not are visualized. The level of
cortisol, ACTH in the blood is increased. MRI pituitary gland - no pathology was revealed. OGTT
(blood glucose): 4.9 - 7.5 mmol / l. FLG of the chest - a mass in the upper lobe of the left lung.

A. Hypercorticism , Because red stripes

Patient S., 47 years old. I went to the doctor with complaints of weight loss, thirst, nausea,
weakness in the lower extremities, bone pain. Objectively: low nutrition, skin of normal color and
moisture, atrophy of the muscles of the legs, deformation of the chest. Vesicular breathing in the
lungs. Rhythmic heart sounds, pulse 74 beats per minute. The abdomen is soft, painful in the
epigastric region. Constipation. There is a "duck gait". During examination: in the general blood
test: hemoglobin -100 g / l, erythrocytes - 4.0x10 '/ l, leukocytes - 4.9 x 109 / l, ESR - 18 mm / h,
blood sugar - 4.8 mmol / l , blood calcium - 3.5 mmol / l (norm 2.1-3.0 mmol / l). X-ray of the spine
shows signs of osteoporosis. Ultrasound of the parathyroid glands - hyperplasia of these glands.

A. Hyperparathyroidism
Q. Patient A is a white man, 19 years of age, with a history of type 1 diabetes. He is currently in the
end of his first year of college and studying for his final examinations. He has been unable to
exercise due to the amount of time devoted to studying. His eating has been erratic, and he has
forgotten to cover his meals with insulin aspart. He presents to the emergency department with a
six-hour history of abdominal pain, nausea, and confusion as reported by his roommate. The
roommate also recalls a fruity odor to Patient A's breath and knows he has not been consuming
alcohol. When asked questions regarding his blood glucose levels, Patient A is confused and
unable to recall any past history of testing. Blood glucose: 531 mg/dL

A. Hypoglycaemia

Q. Patient M is a white woman, 32 years of age, presenting to her primary care physician with
complaints of polyuria for the past four weeks. Further assessment reveals an upcoming
appointment with her optometrist for new onset blurred vision. The patient is 5 feet 5 inches tall
and weighs 142 pounds; her calculated BMI is 23.6 kg/m2. She confirms a family history of
diabetes on her father's side and admits to a generally sedentary occupation and lifestyle. A
random finger stick reveals a blood glucose level of 257 mg/dL.

HbA1c: 9% (estimated average glucose: 208 mg/dL)

Fasting blood glucose: 151 mg/dL

A. Hyperglycemia with type 2

Q. Patient V is a Hispanic man, 61 years of age, presenting to the primary care physician with
complaints of polydipsia, polyphagia, and fatigue for the past month. Upon further assessment, he
states that he has recently had his eyeglass prescription adjusted due to blurred vision. He is 5 feet
10 inches tall and weighs 245 pounds; his calculated BMI is 35.2 kg/m2. The patient reveals a
family history of diabetes on both sides. He works a physical job in a warehouse lifting 50-pound
boxes. A random finger stick reveals a blood glucose level of 220 mg/dL. As previously discussed,
this is sufficient information to diagnose diabetes.

HbA1c: 8.7% (estimated average glucose

A. Hyperglycemia

Q. Patient A. came to the clinic with complaints of progressive obesity, characterized by the
deposition of fat in the face (moon-shaped face), upper body and abdomen, hirsutism (excessive
growth of hair on the body and face in women according to the male pattern), menstrual
irregularities, pain in the lumbar region, polyphagia, polydipsia. Recently, pustular skin diseases
have often been observed, an objective examination revealed: blood glucose - 7.0 mmol / l, blood
pressure - 140/80 mm Hg. Art., changes from side of the blood - moderate leukocytosis (an
increase in the number of leukocytes in the blood), absolute lymphopenia (a decrease in the
number of lymphocytes in the blood). Radiographically revealed enlargement of the left adrenal
gland, radiography of the skull did not enlarge the Turkish saddle.

A. Hypercorticism becz moon shape

Q. Patient K is a white woman, 71 years of age, presenting to her primary care physician with
complaints of fatigue, polyuria, polyphagia, and polydipsia for the past few months. She is 5 feet 4
inches tall and weighs 205 pounds, with a calculated BMI of 35.3 kg/m2. She confirms a family
history of diabetes on her mother's side, a personal history of having given birth to a child
weighing more than 9 pounds, and arthritis of her bilateral knees. A finger stick reveals a blood
glucose level of 198 mg/dL.

HbA1c: 8.0% (estimated average glucose: 183 mg/dL)

Fasting blood glucose: 195 mg/dL

Two-hour glucose level (after 75 g oral glucose tolerance test): 160 mg/dL

A. Hyperglycemia

Q. Patient S., 28 years old, came to the clinic with complaints of general weakness, malaise,
headaches, hair loss, dry skin, an increase in the size of the thyroid gland. From the anamnesis:
during the year, the above-described complaints are worried, gradually progressing. Objectively:
The skin is dry, pale, body mass index - 32. Pulse 59 in min., BP - 125/70 mm Hg. Art., muffled
heart tones. Thyroid increased to I degree according to WHO, compacted, painless. Regional
lymph nodes are not enlarged

A. Hyperthyroidism

Q. Patient T., 40 years old, came for a consultation with an endocrinologist. Complains of fatigue,
dry skin, apathy. Objectively: Height - 164 cm, weight - 68 kg. The skin is dry, warm, pale. Pulse -
58 per minute, blood pressure - 120/80 mm Hg. Art. The thyroid gland is not visible to the eye. On
palpation of the thyroid gland reduced in volume, diffusely heterogeneous, dense, painless.
Regional lymph nodes are not enlarged.

A. O grade classification of hyperthyroidism becaz skin should be warm and moist and fatigue

Q. Patient M., 39 years old. Complains of joint pain, increased blood pressure, general weakness,
headache, overweight. First symptoms diseases appeared 5 years ago, gradually progressing. Later
there was dry mouth, thirst, pain in the epigastric region. Objectively. Height - 165 cm, body
weight - 88 kg., Disproportionate deposition of subcutaneous adipose tissue (mainly in the face
area, neck, torso), climacteric hump. On the inner surface of the shoulders, in the lateral areas of
the abdomen, in the thighs, purple-red stripes. Enhanced facial hair growth. The skin is dry. Pulse-
82 in 1 min. AD - 165/95 mmHg. Additional research. OGTT (blood glucose): 6.3 - 9.6 mmol / l. X-
ray shows minor osteoporosis of the chest and lumbar spine. On ultrasound - an increase in size

A. Hypercorticism

Q. A 53-year-old woman presents to her primary care provider with complaints of fatigue, weight
gain, and constipation. She states that her weight has gradually increased over the last year
despite no change in her activity level or eating habits. She works roughly 8 hours a day as a
nutrition assistant, but she falls asleep as soon as she gets home in the afternoon. She lacks
motivation to do anything during the weekend, and notes that she does not feel like herself. Upon
questioning, the patient reports the following changes: thinning of her scalp hair, brittle nails, dry
skin, and cold intolerance. Physical examination confirms dry skin, brittle nails, coarse and dry
hair, and abdominal distension. The only medication she takes is a multivitamin daily. She has a
positive family history for autoimmune disorders and stroke.

A. Hypothyroidism

Q. A 71-year-old male patient with type 2 diabetes complained of frequent episodes of excess
sweating, feeling of hunger, and tremor in the previous week manifesting as. His diabetes control
was excellent (HbA1c 6.5%) with glimepiride 4 mg daily and metformin 2 g daily. His renal function
was normal and he did not have microalbuminuria. No changes in dietary habits and exercise
program were reported.

A. Hyperglycemia

Q. Patient K., 34 years old, complains of pain in the region of the heart, palpitations, shortness of
breath, appearing with accelerated walking, physical. stress, neck thickening (appeared in the last
6 months), irritability, tearfulness. Celebrates weight loss up to 10kg. Considers himself a patient
for 7 years. She was treated for heart disease and neurasthenia. Objectively: a patient with low
nutritional status with an expression of "frozen fear" on her face. Pulse 120 beats / min, BP 130 \
70 mm Hg. Exophthalmos. The thyroid gland is enlarged evenly with a smooth surface, soft-elastic
consistency, painless, displaced when swallowing.

A. Hyperthyroidism
Q. Patient R., 25 years old, average height. The face is moon-shaped, the skin on it is crimson.
Excessive fat deposition on the abdomen and thighs. The bones are thin. Red stripes are marked
stretching on the skin of the abdomen and shoulders. Blood pressure 160/90 mm Hg. Art. Blood
sugar 7.0 mmol / l. X-ray: the Turkish saddle is dilated.

A. Hypercorticism

Q. Patient L is an African American man, 52 years of age, presenting to his primary care physician
with complaints of frequent urination and excessive thirst, particularly in the late afternoon and
early evening. He is a truck driver and works 12 hours each day. Patient L is 6 feet 1 inch tall and
weighs 215 pounds, with a calculated BMI of 28.4 kg/m2. He confirms a family history of diabetes
on both his father's and mother's sides. A random finger stick results in a blood glucose level of
243 mg/dL.

HbA1c: 8.7% (estimated average glucose: 203 mg/dL)

Fasting blood glucose: 151 mg/dL

A. Hyperglycemia
Vi Indialldea
Oi&ll 80 11:28

Be6 TecTWPOBaHne
X avn.kgma.kg

TynTa PaBMKaHT Kg Ru

Bonpoc: N93
Name the main screening method of imaging the thyroid gland:

OTBeTbI(oqMH OTBeT)
1
radiography

2 thermalimaging
ultrasound

magnetic resonance imaging

radioisotope

50
11 23 4 5 6

10
11:24 AM Mon 11 Jan

A avn.kgma.kg

BHHan KyMap ArapBan Kg Ru


Bonpoc: N914
The reason for the hyperfunction of the sex glands in adults is:

OTBeTbli(oAMH OTBET)
1 Increased binding of testosterone to
plasma proteins
Decrease in the formation of gonadoliberin in the
hypothalamus
3 Central (dysregulatory) disorders

Properglandular disorders
5 Tumors and inflammation in
the hypothalamus

10 11 12 13 14 15 16 17
18 50

1 2 3 4567 8 910
111213115161718 19 20
21 2223 24 25 26 27 28 29 30
31 32 3334 3536 37 38 3940
414243 4445 4647 48 49
50
OTBeHeHHbi
Bonpoc
nponyueHHl BOnpoc
Texyu Bonpoc

+
11:22 O
0 <. U 2.00
t l 80%

OA avn.kgma.kg/webtest/te
pOupU. N°TrO

Patient B., 50 years old, complains that recently the ears,


nose, hands began to increase in size. Which gland is
overactive will give similar symptoms:

OTBeTbI(ogMH OTBET)
1 Gonads

2 Adrenalglands

3 Epiphysis

4 Pituitary gland

5 Thyroid

1 14 15 16 17 18

19 20 21 22 50

1 2 3 45 6 7 89 10
11 12 13 14 15 16 17 18 19 20

21 22 23 24 25 26 27 28 29 30
31 32 33 34 35 36 37 38 39 40
41 42 43 44 45 46 47 48 49 50

OTBeyeHHblM BOnpoc

nponyueHHblM BOnpoc
X
Be6 TecTWPOBaHne
avn.kgma.kg
<
TynTa PaBMkaHT Kg Ru

Bonpoc: N°33
Cushing syndrome is characterized by

OTBeTbi(oqMH OTBeT)
1 hyperthyroidism

2 hypercortisolism
3 hyperprolactinemia

4 hyperpituitarism

5 hyperparathyroidism

29 30 31 32 33 34 35 36 37
50
Vi Indiajldea
P O l 83 11:30

Be6 TecTWPOBaHMe
avn.kgma.kg

TynTa PaBMKaHT Kg Ru

Bonpoc: N°29
What is the characteristic symptom for the syndrome of acute inflammation of the thyroid gland:

OTBeTbi(oqMH OTBeT)
1
no pain on palpation

2 bradycardia, hypotension

3 increase in body temperature

4 headaches, weakness

5 attacks of asphyxia

25 26 27 28 29 30 31 32 33

50

1 2 3 45 6 7 8 9 10
11 12 13 14 15 16 17 18 19 20
11:24 AM Mon 11 Jan

Ov A avn.kgma.kg

BMHan KyMap Arapsan. Kg

Bonpoc: N99
Hypoparathyroidism is manifested by:

OTBeTbI(oAMH OTBer)
1 Deposition of Ca phosphate in soft tissues

2 Demineralization of bones

3 Massive edema

4 Sharp tetany, bronchospasm


5 Decreased tubular water reabsorption

7 8
9 10 11 12 13
50

1 2 3 4 5 67
9 10 8
11 121314 15 16 17 181920
21 22 232425 26 27 28 29 30
31323334 35 36 37 38 39 40
4142434445 4647 4849 50
OTBeyeHHbi
Bonpoc
nponyueHHbi Bonpoc
TeKyuwi BOnpoc
SAMSUN
44%-12:44 p
CHHTx KynAun Kg

Bonpoc: N 39
A white sclera line between the iris and the upper eyelid when looking down with thyroid
hyperfunction syndrome is a symptom:

OTBeTbI(oAMH OTBeT)

1 Kocher

2 Shtelvag

3 Grefe
4 Elinek

5 Krause

35 36 37 38 39 40 41 42 43
50

1 23 45 67 8 9 10

11 14 1516 17 18 19 20
13
12

21 22 23 24 25 26 27 28 29 30
31 32 33 34 35 36 37 38 39 40
41 42 43 44 45 46 47 48 49 50
OTBe4eHHblM
Bonpoc
nponyuyeHHbiM Bonpoc
TeKyuwn Bonpoc

Ao 3aBepuueHna TecTa oCTanocb

0:10:39
3aBepuwTb TeCT
11:24 AM Mon 11 Jan

A avn.kgma.kg

BMHan KyMap ArapBan. Kg

Bonpoc: N98
With an insufficient amount of iodine in food,
it develops:

OTBeTbI(OAMH OTBET)
1 Autoimmune thyroiditis

2 Hypoparathyroidism

3 Diffuse toxic goiter


4 Endemic goiter
5 Hyperthyroidism

50
6
8 9 10 11 12

1 2 3 4 5 6 7
8 9 10
11 1213 14 15 16 17 18 1920
21 22 2324 25 2627 28 29 30
31 32 33 34 35 36 37 38 39 40
4142 4344 454647 48 49 50
OTBeveHHbi
Bonpoc
nponyueHitul Bonpoc
Teryujni 6onpoc
Kymap OM Hapane

/webtest/testing

Bonpoc: Ne27
hypothyroidism
syndrome
in
«myxedema heart»
the
is the basis of
Highlight, what OTBeT)
OTBeTbi(oAMH

regulation
nervous
disorder of the
content
potassium
increase in
muscle fibers
dysfunction of
eosingophilia
50
mucoid edema 31
myocardial 30
29
intersticial
25
26 28
23 24
11:24 AM Mon 11 Jan

O A avn.kgma.kg

BMHan Kymap ArapBan Kg Ru

Bonpoc: N916
On the basis of which pituitary hormone are drugs used to stimulate the
contractile activity of the uterus:

OTBeTbI(oAMH OTBeT)
1 Thyroid stimulating hormone

Vasopressin
3 Oxytocin

Follicle-stimulating hormone
OProlactin

12
12 13 14 15 16 17 18 19
20 50

12 3456789 1
11 1213 14 151617 18 1920
2122 2324 25 26 27 28 29 30
31 32 3334 35 36 37 38 3940
4142434445 46474849 50
9TBeeHHtal Bonpoc
nponyueHHln Bonpoc
Te ad nonno
11:24 AM Mon 11 Jan

OT A avn.kgma.kg

BMHan KyMap Arapsan. Kg Ru

Bonpoc: N917
The patient has clinically all the signs of thyrotoxicosis. What research
needs to be done to clarify the diagnosis:

OTBeTbl(oAMH oTBET)
1 Determine the level of thyroxine

2 Blood test for sugar, ketone bodies


3 LDL, HDL, bloodcholestero

Oprotein composition of plasma, residual


blood nitrogen,
5 General analysis of blood
and urine

13 14 15 16 17 18 19 20
21 50

1
2 3 4 567
11 12 13 14 15
89
10
1617 1819 20
21 22 23 24 25 26 27 28 29 30
31 32 33 3435 36 3738 39 40
414243444546474849 50
OTBeyeHHbl Bornpoc
nponyueHHbl
Bonpoc
rercvnain Bannoc
12:42 e r al 91%
OA avn.kgma.kg/webtest/testing

Bonpoc: N°11
Hyperproduction of glucocorticoids causes:

OTBeTbI(oguH OTBeT)
Increased blood pressure

2 Hypoglycemia

3 Increased bone ossification

4 Positive nitrogen balance

5 Decrease in the excitability of the nervous system

7 8 9 10 11 12
13 14 15 50

1 2 3 4 5 6 7 8 9
10
12 14 15 16 17 18 19 20
13
21 22 23 24 25 26
27 28 29 30

ehich micro 0ofo >


A. Periorbital edema with purplish-purple
erythema
B. Cyanotic eyelid skin staining
C. Dark circles under the eyes
D. Hyperpigmentation of the upper eyelids
E. Eye constriction with erythema of eyelids
4. Sjogren syndrome is:
A avn.Kgma.kg/webtest/testung

KyMap HwpaHaaH Kg

Bonpoc: N°36
What complication does hyperthyroidism give:

OTBeTbI(oAMH OTBeT)
1 atrophic gastritis

2 biliary dyskinesia

3 B12 deficiency anemia

atrial fibrillation

5 addison's diseasee

1
32 33 34 35 36 37

38 39 40 50

ehich micro 0 of 0

A. Periorbital edema with purplish-purple erythema


B. Cyanotic eyelid skin staining
C. Dark circles under the eyes
D. Hyperpigmentation of the upper eyelids
E. Eye constriction with erythema of eyelids
4. Sjogren syndrome is:
A. "dry" syndrome due to dehydration
New Tab
Ope 5

Kyauaeaxa Paxyn KyMap

Bonpoc: NO9 C. sympatihes


Thehighest content of antibodies D parathyro
to microsomal antigen in blood
is observed when: E.
mfencr l

OTBeTbi(oAMH OoTBeT)
3. Name w
pihutary sy

1O Thyrotoxic adenoma
A Magneti
2 Diffusetoxicgoiter
B.Positron
C. Craniog
3 Subacute thyroiditis
D. Compu
4 Thyroid cancer E Endosco

5OAutoimmunethyroiditis 6. On com
structure E
PRDerea
6 10 11 12 13 50

A endothe

23 4 5 6 7 89 10 B. netast=

12 13 14 15 16 17 18 19 20
11 Cncrm
A 43%=1250 pm
Crx Kynqen

Bonpoc: NP44
complaints of
Woman 53 years old turned to a therapist with
thirst,frequent urination, itching of the perineum. Anamnesis,
disturb the above complaints for 1 month. Objectively: The
skin is dry and clean. BMI = 33.2 kg/m2. BP 135/95 mm.Hg.
-
Laboratory blood glucose 9.3 mmol/1, after meals 15.0
mmol/1L. Doctor diagnosed with type 2 diabetes mellitus
Explain why the patient developed hyperglycemia syndromein
this case?

OTBeTbI(oAMH OTBeT)

1 obesity

2 hypodynamia

3 heredity

4 underweight
5 stress

40 41 42 43 4445 46 47
50

1 2 3 456 7 89 10
11 12 13 14 15 16 17 18 19 20
21 22 23 24 25 26 27 28 29 30
1 32 33 34 35 36 37 38 39 40
1 42 43 44 45 46 47 48 49 50
OTBeYeHHbl Bonpoc
nponyuyeHHblM BOnpoOc
Tekyuznin Bonpoc
Vi Indialldea P
O l 1130
X Be6 TecTWpOBaHWne
A avn.kgma.kg

TynTa PaBMKaHT Ru

Bonpoc: N°26
Explain the reason for the large tongue in a child with hypothyroidism

OTBeTbi(oqMH OTBeT)

1 disorder of the salivary glands

2 innervation disorder

3 mucous swelling of the tongue

4 disorder of the musculature of the tongue

5
impaired circulation of the oral cavity

22 23 24 25 26 27 28 29 30

50
LTEl 927%
12:38

A avn.kgma.kg/webtest/testing

Kg R
KyMap HwpaHamaH

Bonpoc: N913
With an insufficient amount of iodine in food, it develops:

OTBeTbi(oAMH OTBeT)

1 Endemic goiter

2 Hypoparathyroidism

3 Hyperthyroidism

4 Autoimmune thyroiditis

5 Diffuse toxicgoiter

1 9 10 11 12 13 14

15 16 17 50

ehich micro O of 0

A. Periorbital edema with purplish-purple erythema


B.Cyanotic eyelid skin staining
C. Dark circles under the eyes
D. Hyperpigmentation of the unner eveli
11:21 0<. U kars t 80%

Pan HwTHI Kg Ru

Bonpoc: Ne14
Hypoparathyroidism is manifested by:

OTBeTbI(ogMH OTBer)
1
Decreased tubular water reabsorption

2 Sharp tetany, bronchospasm

33 Deposition of Ca phosphate in soft tissues

4 Demineralization of bones

5 Massive edema

1 10 11 12 13 14

15 16 17 18 50

1 2 3 4 5 6 7 89 10
1112 13 14 15 16 17 18 19 20
21 22 23 24 25 26 27 28 29 30
31 32 33 34 35 36 37 38 39 40
41 42 43 44 45 46 47 48 49 50
OTBeyeHHblM BOnpoc
nponyueHHblM BOnpoc
Tekyuinn Bonpoc

O
tSung
Gmail YouTube Maps How to Study Gene

KyMap ApAwTR

567 8 9 10
15 16 17 18 19 20 Bonpoc: N91
25 26 27 28 29 30
Patient A, 51 years old, complains of irritability, trembling of fingers, insomnia for more than 2
35 36 37 38 39 40 months after suffering psychotrauma. On examination, the patient
had a normal diet, the pulse was 84 beats per minute. BP 120/70 mm Hg Eye symptoms are
45 46 47 48 49 50 not detected. In the area of the isthmus of the thyroid gland, a mass of 2
cm is determined by palpation, What method of radiological diagnosis should be performed to visualize the palpable mass and determine its structure
HHbli Bonpoc
ieHHbl Bonpoc
BOnpoc
OTBeTbI(oAUH OTBeT)
eHHR TeCTa ocTanoCb 1 doppler study
12:31 2 radioisotope study of thyroid hormone in a test tub

epuTb TecT 3 0 x-ray of the neck

4O radioisotope scanning of the thyroid gland


thyroid gland).
5 ultrasound (two-dimensional sonography of the

2 34 5 6 7 50

10:22
d) ENG
11-01-20212
ype here to search
43% =12:47 pm
CHHrx Kynaun Kg Ru
Bonpoc: N944

Woman 53 years old turned to a therapist with complaints of


thirst, frequent urination, itching of the perineum. Anamnesis,
disturb the above complaints for month. Objectively: The
1

skin is dry and clean. BMI = 33.2 kg/m2. BP 135/95 mm.Hg.


Laboratory blood glucose-9.3 mmol/1, after meals 15.0
mmol/1. Doctor diagnosed with type 2 diabetes mellitus.
Explain why the patient developed hyperglycemia syndromein
this case?

OTBeTbi(ogMH OTBer)
1 obesity

Ohypodynamia

3 heredity
underwelght

5 stress

40 41 42 43 44 45 46 47 48
50

21112 133 144 15


5

8 210
7

16 17 18 19 20
21 22 23 24 25 26 27 28 29 30
31 32 33 3435 36 37 38 39 40
41 42 43 44 45 46 47 48 49 50
OTBeeHHbl
BOnpoc
nponyuyeHHbi
BOpoc
TeKyunn Bonpoc
Vi Indialldea O 3) 11:30

Be6 TecTHpoBaHe
A avn.kgma.kg

TynTa PaBHKaHT Ru

Bonpoc: N 31
Thyroid-stimulating hormone is normal (mU/)?

OTBeTbI(opnH OTBeT)
1 from 0.1 to 3.0

from 1.4 to 6.1

3 from 2.1 to 7.5


4 from 1.0 to 5.0

5 from 0.4 to 4.0

27 28 29 30 31 32 33 34 35
50

56 7
89 10
ndiajldea
Ot l 33 11:29
X Be6 TecTWpOBaHne
avn.kgma.kg

TynTa PaBMKaHT Ko Ru

Bonpoc: N99
The cause of secondary aldosteronism is:

OTBETbl(OqMH OTBeT)

1 Tumor of the fascicular zone of the adrenal cortex

2 Increased secretion of aldosterone under the influence of angiotensin

3 Tumor of the reticular adrenal cortex

Tumor of the glomerular zone of the adrenal cortex

5
Tumor of the adrenal medulla

5 6 7 89 10 11 12 13 50

2 3 4 56 7 89 10
11:21
0< 0 2e 4 80%

Pan HWTWLI Kg Ru

Bonpoc: Ne13
The cause of secondary aldosteronism is:

OTBeTbI(oAMH OTBer)

1 Tumor of the reticular adrenal cortex

2 Increased secretion of aldosterone under the


influence of angiotensin

3 Tumor of the fascicular zone of the adrenal


cortex

4 Tumor of the adrenal medulla

5 Tumor of the glomerular zone of the adrenal


cortex

1 9 10 11 12 13

14 15 16 17 50

1 2 3 4 56 7
89 10
11 12 1314 15 16 17 18 19 20
21 22 23 2425 26 27 28 20 220

G 13
11:25 AM Mon 11 Jan

A avn.kgma.kg

BMHan KyMap ArapBan. Kg Ru

Bonpoc: N923
Describe the stool for hypothyroidism in early
childhood:

OTBeTbl(oAMH oTBeT)
1 form as pea puree
2 frequent, frothy with mucus

3 frequent with blood


4 form of rice water

5 persistent constipation

19 20 21 22 23 24 25 26
27 50

1 2 34 567 89 10
11 121314 1516 17
18 19 20
2122 232425 26 27 28 29 30
3132 333435 36 3738 39
40
41 42 434445 4647 4849 50
OTBeveHHblM
Bonpoc
nponyuyeHHbl
Bonpoc
TeKyuuna
BOnpoc
11:22 0 <. I0 4l 81%

OA avn.kgma.kg/webtest/te

Bonpoc: Ne49
A 55-year-old woman consulted a family doctor at the
place of residence with complaints of thirst, dry mouth,
weight gain, pain in the lumbar region, sweating.
Objectively: height - 170 cm, body weight- 120 kg, moon-
shaped face, crimson-red, excessive growth of facial
hair, acne, excessive deposition of fatty tissue in the
shoulder, chest, abdomen, red-violet color, atrophy of the
muscles ofthe arms and legs. BP- 190/100 mm Hg.
Blood glucose 12 mmol/I. Choose, what hormonal
blood test is necessary to conduct any obesity disease?

OTBeTbI(oAMH OTBeT)
1
vasopressin

2 thyroxine

3 insulin

4 prolactin

5 cortisol

1 43 44 45 46 A1
47

48 49 50

O
1Z.37

A avn.kgma.kg/webtest/testing

KyMap HwpaHaaH Kg Ru

Bonpoc: N94
Patient J., 34 years old, had a bloody nipple discharge the
in

period of absence of pregnancy and lactation. Previously, the


study of the mammary glands was not performed. Has been
smoking for several years. What radiation methods will you
prescribe to the patient in the first place?

OTBETbI(oAMH OTBeT)

magnetic resonance imaging

ultrasound, doppler

3 multi spiralcomputed tomography

4 chest xray

mammography, ductography

1 2 3 4 5 67 8

ehich micro 0ofo

A. Periorbital edema with purplish-purple erythema


B.Cyanotic eyelid skin staining
C. Dark circles under the eyes

D. Hyperpigmentation of the upper eyelids


E. Eye constriction with erythema of eyelids
4. Sjogren syndrome is:
A. "dry" syndrome due to dehydration
B. "dry" syndrome due to lesions of the endocrine glands
CMar condroma due to lesions of the exocrine glands
KyuaBaxa

Paxyn Map

Bonpoc: N°49
Patient 29 years old, consulted a family doctor with complaints of headaches, weight gain, weakness,
palpitations, menstrual irregularities. On examination: rounded face, acne. Weight 98 kg, height 162 cm.
Body mass index- 37.6 kg /m2. Conclusion: obesity ll degree. Choose, which method isused to determine of
obesity?

OTBeTbi(oAuH OTBeT)
1 O Brock's index

2 Borngardt index

3 Jackson-Pollock index

4 Breitman index

Quetelet index
5
46 47 48 49 50
43 44 45
Vi Indialldea

Be6 TecTHpOBaHMe
X avn.kgma.kg

Kg Ru
TyTaPaBMKaHT

Bonpoc: N°41
A 38-year-old womanwas admitted to the clinic with complaints of decreased appetite,
constipation, hearing loss, difficulty in nasal breathing, weakness. Anamnesis: Subtotal
strumectomy was performed 3 years ago. Objectively: The skin is dry, the height is 167 cm, the
weight is 68 kg, there is dense swell ing of the legs. diagnosed with hypothyroidism syndrome.
Explain why hearing is reduced and nasal breathing is difficultdue to?

OTBeTbI(oAMH OTBeT)

1
inflammation

2 infection

3 muscle swelling

4 swelling of the skin

5 mucous edema

37 38 39 40 41 42 43 44 45
Kyneap Ons
Hapate

na.kg/webtest/testin9
increased biood
mouth, cm,weight
Ne49 headache, diy height-167
Bonpoc: weakness, rubeosis, according by
complaintsof cheek obesity
face, degree of
hospital with Objectively.rounded Determine,what
clinical thighs.
exercise.
department of a during abdomenand
endocrinology lumbar spine the
skin of
admitted to the the heart, in the striae on the
region of cm, pale
woman was in the circumference 100
57-year-old mm Hg. pain hip
A 160/90 cm,
pressure to circumference 120 OTBeTbi(OAMH
OTBer)

92 kg, waist
BMI?

O
50
48
47
46
45
44
43
11:25 O 0<. 009 Y 4 84%

OA avn.kgma.kg/webtest/te

Pan HuTWI Kg
Ru

Bonpoc: N°38
Waist circumference in men is normally less than (cm)

OTBETbi(oaMH OTBeT)
1 110
2 94

3 86

4 78
5 102

1 34 35 36 37 38

39 40 41 42 50

1 23 4 5 6 7 89 10
11 12 13 14 15 16 17 18 19 20
21 22 23 24 25 26 27 28 29 30
31 32 33 34 35 36 37 38 39 40
41 42 43 44 45 46 47 48 49 50
ATDAUAILIŽ pannA
M50% 11:54 am
CHHrxKynAn Ke
Ru
Bonpoc: N4

Patient J., 34 years old, had a bloody nipple discharge in the


period of absence of pregnancy and lactation. Previously, the
study of the mammary glands was not performed. Has been
smoking for several years. What radiation methods will you
prescribe to the patient in the first place?

OTBETbl(oAMH OTBET

1 mammography, ductography

2 chest x-ray

3 magnetic resonance imaging

4 ultrasound, doppler

5 multispiralcomputed tomography

1
2 3 7 8 50

1 2 3 45 67 8 9 10
11 12 13 14 15 16 17 18 19 20
21 22 23 24 25 26 27 28 29 30
31 32 33 34 35 36 37 38 39 40
41 42 43 44 45 46 47 48 49 50
OTBeeHHbiM
Bonp0c
nponyuieHHbI
BOnpoc
Tekyunn Bonpoc

Ao 3aBepweHnA TeCTa
oCTanocb:

1:00:49
3aBepuwTb TeCT
11:25 AM Mon 11 Jan

Ov A avn.kgma.kg

BMHan Kymap ArapBan. Kg Ru


Bonpoc: N924
Indicate, in what period of childhood symptoms of hyperthyroidism
syndrome appear:

OTBeTbI(OAWH OTBeT)
1 newborn
2 senior nursery age
3 puberty
4 preschool
5 brest

20 21 22 23 24 25 26 27
28 50

1 23 4567 9 8 10
1112131415 1617 18 19
20
212223 24252627 28
29 30
31 32 3334 35 36 3738 39 40
41424344 4546474849
50
OTBeveHHb
Bonpoc
nponyuieHHbi
Bonpoc
Tonmman popnnc
Vi IndialldeaP ©
OR ill a33 1130

Be6 TecTMpOBaHMe
A avn.kgma.kg

TynTa PaBHKaHT Ru

Bonpoc: N932
Firstaid for hypoglycemia, what kind of remedy is taken?

OTBETbI(ogMH OTBeT)

1 metformin 500mg
2 insulin 4 unit

3 furosemide 40 mg

calcium chloride 100 mg

5 glucose 40%

28 29 30 31 32 33 34 35 36
50

12 3 4567 8 9 10
avn.kgma.kg

rynTa PaBMKaHT Ru

Bonpoc: N°20
A 3-year-old boy
was admitted to the clinic for childhood diseases. On examination:
the child's
heightis much lower than the age norm, short neck, arms and legs, big belly; the
head is
the nasal bridge is sunken, the face is puffy, pale, mask-like, the mouth is open, the tonguelarge,
is
enlarged, there are many carious teeth, the speech is slurred, the spoken words are
distorted,
the syllables in them are often rearranged; the child does not always answer questions, he is
capricious and irritable. BP 80/50 mm Hg. Art., pulse 45 per minute.
The thyroid gland is painless
onpalpation, not enlarged. The child's mother took iodine preparations during pregnancy.
Diagnosis "Congenital hypothyroidism. Cretinism". The initial
and pathogenetic link of this
disease is:

OTBeTbli(oAMH OTBeT)
1
Low TSH levels
2 Increase in FSH and LH

3 Lack of micronutrients

4 Deficiency of T3 T4

5 Lack of iodine

16 17 18 19 20 21 22 23 24
50
Vi IndialldeaP

Beó TecTWPOBaHe
X A avn.kgma.kg

TynTa PaBMKaHT Kg Ru

Bonpoc: N°47
20-year-old boy was admitted to the clinic unconscious. According to relatives, she previously
A

complained of intense thirst, frequent urination and general weakness. Objectively: the smell of
acetone from the mouth, the skin is dry. Rapid breathing. Heart sounds are muffled. The
abdomen is tense. The doctor had previously diagnosed acute hyperglycemia. What analysis is
needed first?

OTBETbI(oqMH OTBeT)

total cholesterol

2 C- peptide
3 triglycerides

Oglycohemoglobin

5 blood glucose

43 44 45 46 47 48 49 50
7 l51%u 10:15
Be6 TeCTWpOBaHMe

C avn.kgma.kg/webtest/testing

AHaHA MpMHanmHM Kg Ru
Bonpoc: N°43
A man V. 40 years old, was admitted to the clinic with multiple pathological, spontaneous
fractures of tubular claws, pronounced curvature of the spine and deformation of the skeleton.
Upon admission, the patient complained of general weakness, extremely rapid fatigue,
muscle
hypotonia, and bone pain. In the anamnesis there are indications presence of kidrney stones,
severe polyuria and an increased content of phosphates in the urine. Laboratory examination
revealed an increase in the calcium content in the blood to 15.1 mmol /L and a decrease in
the
phosphorus content to 0.24 mmol/L. What is your preliminary syndrome?

OTBeTbI(oAMH OTBeT)

1 thyroid inflammation syndrome


2 hypercortisol syndrome

3 hyperthyroidism syndrome

4 hyperparathyroidisrm syndrome

5 hyperglycemic syndrome

39 40 41 42 43 44 45 46 47

50

45 67
tEE1pt2 18 19 2

44 45 46 47 48 49 50
42%-127 p
complaints of
generalpracitione at an poicinic wit
constipation, hoarseress, memory loss, weakness
drowsiness, and swzallowirng of face Fro the anammesis it is
krow tese symgioms have been disturoirg tor the lat
12
months. bjectiveiy. The skin is dry, elbow hyperkeratosis
Swalliowing of face. The thyroid gland is not enlarged by
palpatior, pairless, nodular fomations are nat determined
Hormonal profile and antibody determinationc TSH-25.7 pU
mil T4 free-6.1 prmol /LAB 1o TP0-271 U/mil, AB to
yrogicbulin-310 ug /mil. Utrasound of the thyroid gland: V.
igt iobes -30cm3, Vieft. lobes-21 cm, V total-5.1 cm3.
Echogenictyis increased. Pronounced diffusely
eserogeneous structure, nodular formations are not
determined. What oyndrome does the hormonal profile data
indicane

OTBETE(OBVH OTBET)

hyaid iefamaion srdome


2 ndime of hypetypaidim
desme of yprptiutarism

symdecme f tyoparatsraidiam

tfyid scddle sysdeome

45 49 50

23 4 89 1
11 12 131416 1
waps New Tab en

Kyuraeanca Paag
Kyaap

Bonpoc: Ne12
An excess of thyroid hormones
occurs when:

OTBeTbi(oAMH OoTBET)
1 Diffuse toxic goiter
2 Acromegaly
3O Miksedeme

4 Endemiccretinisr
5 nsulinoma

14
8 10 11
12 13 15 16 50

211l12 133 144567 9910


15 16 1718 19 20
KyMap Ou Hapam

wn.kgma.kg/webtest/testing

disturbingfor 1
complaintsare
Bonpoc: N940 palpitations.The
above
breasts,
purple-red
gain,
weakness,
cm, enlargement of the patient
headaches, weight height -162 symptoms ofthe
complaints of acne,
weight-110 kg, descriptionthe objective
therapist with rounded,
a polyclinic examination: the face is Findout from the
turned to arms and legs.
A
40-year-old man hastraumatic brain injury. On thighs, thinning of the
suffering a abdomen,
year after stretching of the skin on the OTBer)
OTBeTbI(oAMH
streaks of

obstetrician hand
Ofish mouth, diastema
noce macrognathia,
2 0 macroglossia
hirsutism,
3 50
gynecomastia, striae
44
43
42
41
4 myxedema 39 40
exophthalmos, 38
37
30
SAMSUNU

l 51%10:16
Be6 TecTpoBaHe

O C A avn.kgma.kg/webtest/testing

AHaHA MpMHannH Kg Ru

Bonpoc: Ne48
Patient, 48 years old, at a doctor's appointment complains of muscle weakness, rapid physical
fatigue, lack of appetite, nausea, muscle pain, addictlon to salty foods, menstrual irregularities.
On examination, weight 48 kg, height 166 cm. The skin has a dark brown color, dark spots on
the mucous membrane of the oral cavity, subcutaneous fat is thinned. BP 70/50 mm Hg. The
pulse on the radial artery is soft, small filling. Choose the most informative blood test for your
diagnosIS

OTBETbi(oqMH OTBeT)
1 adrenocorticotropichormone

2 growth hormone

3 thyroid-stimulating hormone

4 melanotropic hormone

5 follicle-stimulating hormone

43 45 46 48 49 50

1 3 13 14
45 67 8
11 12 18 15 16 17 19 20
21 22 2324 25 26 27 28 29 3
31 32 33 34 35 36 37 36 39 40
41 42 43 44 45 46 47 48 49 50

OTBeeHHbIRBOnpoc
nponyuteHHbIM BOnpoc
TeKyuynn Bonpoc

Ao saBepueHMa TeCTa gCTA9S


Kpap Oe Hapa

Bonpoc: Ne27
hypothyroidism syndrome:
«myxedema heart» in
basis of the
Highlight, what is the
OTBeTbl(OAMH OTBeT)

regulation
nervous
disorder of the potassium.content
increase in
muscle fibers
dysfunction of
eosinophilia
Omucoid 30
myocardtal edema
31
30
29
29
Ointersticial 28

2A 26
KyuaBaxa Paxyn KymMap

Bonpoc: Ne38
What is the characteristic symptom for the syndrome of acuteinflammation ofthethyroid gland:

OTBeTbI(oanH OTBeT)
1Obradycardia, hypotension

2 O
increase in body temperature

3Oheadaches, weakness

4 Ono pain on palpation

5Oattacksof asphyxia
42
34 35 36 37 39 40 41

1 23 45 6 781R 91910
20
17
11:25 O 0 <. U 2.00
t l 84%

OA avn.kgma.kg/webtest/te

Pan HWTWu Kg
Ru

Bonpoc: Ne39
Inhypercortisolism syndrome, skin changes are
characterized by

OTBeTbI(oqMH OTBeT)
1 myxedema

2 darkening
3 striae

4 yellowness

5 vitiligo

1 35 36 37 38 39

40 41 42 43 50

1 23 4 5 6 7 89 10
11 12 13 14 15 16 17 18 19 20
21 22 23 24 25 26 27 28 29 30
31 32 33 34 35 36 37 38 39 40
41 42 43 44 45 46 47 48 49 50
O
11:23 AM Man 11 Jan

A avn.kgma.kg

BMHanKyMap Arapsan. Ru

Bonpoc: N91
Patient A., 51 years old, complains of iitability, trembling of fingers,
insomnia for more than 2 months. after suffering psychotrauma. On
examination, the patient had a normal diet, the pulse was 84 beats per
minute. BP 120/70 mm Hg Eye symptoms are not detected. In the area of
the isthmus ofthe thyroid gland, a mass of 2 cm is determined by
palpation. What method of radiological diagnosis should be performed to
visualize the palpable mass and determine its structure?

OTBeTbI(oqnH OTBeT)
1 x-ray of the neck.

2 doppler study.

3 radioisotope scanning of the thyroid gland.

4 radioisotope study of thyroid hormone in a test tube.

ultrasound (two-dimensional sonography of the thyroid gland)

2 3 4 5 6 50

2 3 4 5 6789 10
11 12 13 14 151617 1819 20
21222324 25 26 2728 29 30
31 32 3334 35 36 37 38 3940
41424344 4546474B4950
O
11:24 AM Mon 11 Jan

A avn.kgma.kg

BHHa KyMap Arapsan. Kg Ru

Bonpoc: N913
Basophilic adenoma of the adenohypophysis leads to the
development of:

OTBeTbi(oguH OTBeT)
1 Simmonds disease
2 Hyperthyroidism 6
3Gigantism
Acromegaly
5 Cushing'sdisease

17
910 11 12 13 14 15 16
50

1 2 3 4
89 5.67 10
11121314 15 16 17 18 19 20
2122 2324 25 2627 28 29 30
31 32 33 34 35 36 37 38 39
40
4142434445 4647 4849 50
QTBeueHHtl
Bonpoc
nponyuieHHl Bonpoc
Texyuw Bonpoc
Aavn.kgma.kg/webtest/testing
O
Kg Ru
Kymap HupaHamaH

Bonpoc: N°6
A 63-year-old patient complains of difficulty passing solid food.
On
Objectively, the patient is lethargic, his voice is hoarse.
fat. A
examination, bradycardia, pasty skin and subcutaneous
barium swallow was performed, which showed a moderate
to
deviation of the pharynx and the upper third of the esophagus
the the
right, contours of the esophagus are normal. What is the
most informative research method you can use?

OTBETbI(oqMH OTBeT)

1 multi spiralcomputed tomography

2 endoscopPy

3 overview Radiography

4 computed tomographywith contrasting thyroid gland

5 thyroid ultrasound

ehich micro 0 of 0

A. Periorbital edema with purplish-purple erytheema


B. Cyanotic eyelid skin staining
C. Dark circles under the eyes
D. Hyperpigmentation of the upper eyelids
E. Eye constriction with erythema of eyelids

4. Sjogren syndrome is:


A. "dry" syndrome due to dehydration
B. "dry" syndrome due to lesions of the endocrine glands

C."dry" syndrome due to lesions of the exocrine glands


Vi Indial|ldea P IOt l831 11:30
Be6 TecTMpOBaHne
avn.kgma.kg

TynTa PaBMKaHT Ka Ru

Bonpoc: N°30
Waist circumference in men is normally less than (cm)

OTBeTbl(OAMH OTBeT)

1 102

2 110

3 94

4 78
5 86

26 27 28 29 30 31 32 33 34
50

1 23 4 5 6 7 89 10
11 12 13 14 15 16 17 18 19
20
wTUTeCiMpOBaHne
avn.kgma.kg

rynTa PaBMKaHT KgRu

Bonpoc: N914
Conn's syndrome (primary aldosteronism)
is manifested by

OTBeTbli(oAMH OTBeT)

1 Oliguria
2 Hypotension

3 Accumulation of H ions

4 Na loss and K delay

5 Na retention and loss of K

10 11 12 13 14 15 16 17 18

50

123 4 56 7
89 10
11 12 13 14 15 16 17 18 19 20
21 22 23 24 25 26 27 28 29 30
O l 831 11:31
Vi IndialldeaP O

Be6 TecTMpOBaHne
X A avn.kgma.kg
Ru
TynTa PaBMKaHT

Bonpoc: N945
present any
44-year-old woman was admitted to the clinic for examination and doesn't
A
with increased
complaints. Anamnesis: dad suffers from diabetes. Objectively: a patient
Breathing is vesicular.
nutrition, normal color leather. Height 170 cm2, body weight- 85 kg.
blood) 5.8 mmol / after
l
Heartsounds are saved. Laboratory: fasting blood glucose (capillary
-

meals after 2 hours 6.8 mmol/1, interpret this result

OTBETbI(oaMH OTBeT)

1 hypoglycemia

2 acute hyperglycemia

3 diabetes mellitus

4 impaired fasting glucose

55 impaired glucose tolerance

41 42 43 44 45 46 47 48 49 50
Vi IndialldeaP 01 l ) 11:300

ABe6 TeCTWpoBaHme
X avn.kgma.kg

TynTa PaBMKaHT Ru

Bonpoc: N°40
A man V. 40 years old, was admitted to the clinic with multiple pathological, spontaneous
fracturesof tubular claws, pronounced curvature of the spine and deformation of the skeleton.
Upon admission, the patient complained of general weakness, extremely rapid fatigue, muscle
hypotonia, and bone pain. In the anamnesis there are indications presence of kidney stones,
severe polyuria and an increased content of phosphates in the urine. Laboratory examination
revealedan increase in the calcium content in the blood to 15.1 mmol/L and a decrease in the
phosphorus content to 0.24 mmol / L. What is your preliminary syndrome?

OTBETbl(oAMH OTBeT)

1 thyroid inflammation syndrome

2 hypercortisol syndrome

3 hyperparathyroidism syndrome

4 hyperglycemic syndrome

5 hyperthyroidism syndrome

36 37 38 39 40 41 42 43
SAMSNG
l 43%=12:50 pm
CWHrx Kyngun Kg Ru

Bonpoc: N°43
complaints of
Patient K., 48 years old, went to the clinic with
severe weakness, weight loss, bone pain, especially in the feet,
long-term healing fractures, loss of appetite, nausea, vomiting
not associated with food intake, diarrhea. From the
anamnesis: suffers from adenoma of the right parathyroid
gland. Laboratory data: TBC-anemia, Calcium-3.4 mmol /1,
phosphates-0.7 mmol/1 What is your preliminary syndrome?

OTBeTbI(oAWH OTBeT)

1 thyroid inflammation syndrome

2 hypercortisol syndrome

3 hyperthyroidism syndrome

4 hyperparathyroidism syndrome

5 hyperglycemia syndrome

39 40 41 42 43 44 45 46 47
50

1 23 4
5 6

7 8 910
11 12 13 1415 16 17 1819 20
2122 23 24 25 26 2728 29 30
31 32 33 34 35 36 37 38 39 40
4142 43 44 45 46 47 48 49 50
OTBeveHHbI Bonpoc
nponyueHHbin BOnpoc
TeKyuMn Bonpoc

Ao saBepiieHwa TecTa oCTanocb


l 93%

Bonpoc: N946
The man40 year old was admitted to the clinic, complaints
of
palpitations, interruptions in the work of the heart,
sweating,
trembling in the body, weakness. Anamnesis: has been ill for
several years. Examination: the apical impulse is enhanced.
Heart
sounds are loud, arrhythmic. Pulse 98 in 1 minute. blood pressure
170/60 mm Hg.ECG: tachysystolic form of atrial fibrillation.The
doctor diagnosed hyperthyroidism syndrome. Explain, what
caused the atrial fibrillation in this case, because of the increase?

OTBeTbI(oAMH OTBeT

1 parathyroid hormone

2 calcitonin

3 increased TSH

4 antibodies TPO

5 T3 and T4

42 43 44 45 46 47
48 49 50

ehich micro 0 of 0

A. Periorbital edema with purplish-purple erythema


B. Cyanotic eyelid skin staining
C. Dark circles under the eyes
D. Hyperpigmentation of the upper eyelids
E. Eye constriction with erythema of eyelids
4. Sjogren syndrome is:

A. "dry" syndrome due to dehydration


B. "dry" syndrome due to lesions of the endocrine glands
c "dr" sundromn duo t
l 43%-12:49 pm

TeCTMpOBaHW X
Be6 O
CAavn.kgma.kg/webtes
CMHTxKynAnn Kg Ru

Bonpoc: Ne21
headache, increased
complains of nausea,
A 25-year-old man gastrointestinal
polyuria. Gastroscopy revealed no
thirst and
pathology. The condition
worsened thirst and polyuria
-

a
amount of liquid drunk increased to 8 liters,
increased, the a
headache accompanied by vomiting, there was
constant
Urine analysis: a decrease the
in
lateral visual fields.
loss of
of urine to 1002, blood
specific gravity in the morning portion
mosm/kg.
osmolality-315 mosm/kg, urine osmolality-270
Which of the following
Fasting blood sugar-3.2 mmol/1.
methods is priority:

OTBeTbI(oAMH OTBeT)

1 Determination of blood sugar during the day

2 Excretory urography

3 Fluid restriction test

4 MRI of the brain

5 ultrasound of the kidneys

17 18 19 20 21 22 23 24 25

50

12 3 4 56789
a1.12.13
10
341s16
17.1819.20
11:24 AM Mon 11 Jan

A avn.kgma.kg

BMHanKymap ArapBan. Kg Ru
Bonpoc: N918
A 39-year-old patient complains of
irritability, palpitations and pain in the
region of the heart, increased appetite. On examination,
attention is drawn
to bulging9, muscle tremors, blood pressure
150/70.
Indicatewhichglandfunctionisimpaired?

OTBeTbI(oquH OTBeT)
1 Pancreas

2 Epiphysis

3 Thyroid

4 Pituitary gland.
Parathyroid

14 15 16 17 18 19 20 21
22 50

12 3 4567
11121314 151617119 89 10
20
21 222324 25 2627 28
29 30
31 32 33 34 35 363738 39 40
41424344 45 464748 49
50
rAeve
Vi Indialldea
TecTHpoBaHne
X ABe6
avn.kgma.kg

Tynta PaBMKaHT Kg Ru

Bonpoc: N°37
Indicate in which syndrome arterial hypotension is most often observed?

OTBeTbi(OAMH OTBeT)

1
hypocorticism

2 hypoparathyroidism

3 acromegaly

4 hyperthyroidism

5 hyperglycemia

33 34 35 36 37 38 39 40 41

50

23 4 5 67 8 9 10
O
11:25 0<. 009 4l 84%

O A avn.kgma.kg/webtest/te

Bonpoc: N942
Patient G., 42 years old, after a traumatic injury to the
neck, notes the periodic appearance of paresthesias
(pathological unpleasant sensations, for example,
"running creeps") with the subsequent development of
seizures. Before the development of seizures, he has
nausea, vomiting. Convulsions are tonic in nature,
beginning in the upper limbs and face. Consciousness
during this period is preserved. Examination of the
patient revealed that the content of calcium in the blood
is 0.6 mmol /I, phosphorus -8.0 mmol/1. What is your
preliminary syndrome?

OTBeTbI(oAMH OTBeT)
1 hypoparathyroidism syndrome

2 hypocorticism syndrome

3 hypopituitarism syndrome

4 hypothyroidism syndrome

5 hypoglycemic syndrome

1 38 39 40 41 42

43 44 45 46 50

O
<
11:25 0 009 4l 84%

OA avn.kgma.kg/webtest/te

Bonpoc: Ne43
A man V. 40 years old, was admitted to the clinic with
multiple pathological, spontaneous fractures of tubular
claws, pronounced curvature of the spine and
deformation of the skeleton. Upon admission, the
patient complained of general weakness, extremely
rapid fatigue, muscle hypotonia, and bone pain. In the
anamnesis there are indications presence of kidney
stones, severe polyuria and an increased content of
phosphates in the urine. Laboratory examination
revealed an increase in the calcium content in the blood
to 15.1 mmol/ L and a decrease in the phosphorus
content to 0.24 mmol/L. What is your preliminary
syndrome?

OTBeTbI(oqMH OTBeT)
1 hyperthyroidism syndrome

2 hyperglycemic syndrome

3 hyperparathyroidism syndrome

4 thyroid inflammation syndrome

5 hypercortisol syndrome

1 39 40 41 42 43

AA A5 A6 50
O
DEU CUTMpOBaHIE
avn.kgma.kg

TynTa PaBMKaHT Ru

Bonpoc: N°21
A 56-year-old woman was admitted to surgery for stomach ulcers in an unconscious state.
Objectively: a patient of average height. The face is moon-shaped, crimson-red, vellus hair on the
chin and above the upper lip. There is a selective deposition of fat in the chest and upper
abdomen. The limbs are thin. On the skin of the abdomen, shoulders, mammary glands, there
are wide stretch stripes of red-purple color, a lot of acne. The borders of the heart are expanded.
Pulse 92 per minute, blood pressure 200/100 mm Hg, blood levels of hyperglycemia,
hypernatremia, hypokalemia; in plasma, a decrease in corticotropin and an increase in
17
ketosteroids in the urine. Which of the following preliminary diagnoses is most likely

OTBETbi(OqMH OTBeT)
1
Hypertensive crisis
2 Diabetes insipidus

3 Peptic ulcer

4 Cushing's syndrome

5 Diabetes mellitus

1
17 18 19 20
21 22 23 24 25
50
Vi Indialldea O1 l 31 11:29

X Be6 TecTMpOBaHne
avn.kgma.kg

TynTa PaBkaHT Kg Ru
Bonpoc: Ne16
A patient was brought to the admission department of the hospital, who lost consciousness in
thestreet. The examination revealed the smell of acetone from the mouth. What is the
preliminary diagnosis that can be made:

OTBeTbl(OAMH OTBeT)
1 Internal bleeding
2 Uremic coma

3 Pulmonary embolism

4 Diabetic coma

5 Hepatic coma

1 12 13 14 15
16 17 18 19 20
50

2 3 4 5 67 89 10
l 43%12:49 pm
Cerx Kynan

Bonpoc: N42
with
Patient A., 40-year-old was admitted to the hospital
complaints of chills, weakness, difficulty in swallowing, pain in
when
the neck with irradiation to the ears, head, aggravated
turning, body temperature 38 C, sweating. On palpation, the
thyroid gland is enlarged, painful, not soldered to the
surrounding tissues, compacted, there are no fluctuating
areas. Submandibular, cervical lymph nodes are not enlarged.
In the analysis of blood: ESR up to 21 mm/h, relative
lymphocytosis. In the study of the accumulation of iodine by
the thyroid gland on the scanned areas of enlightenment, the
accumulation is reduced. What is your preliminary syndrome?

OTBeTbi(oAMH OTBeT)

1 syndrome of hypofunction of the parathyroid glands

2 adrenal hypofunction syndrome

3 syndrome of hypothyroidism

4 thyroid nodule syndrorme

5 thyroid inflammation syndrome

38 39 10 41 42 43 44 45
50

1 23 456 7
11 1213 141516 17 18891920
10

2122 23 24 25 26 2723 29 30
32 33 34 35 36 37 38 39 40
Be6 TecTWpOBaHMe
avn.kgma.kg

TynTa PaBMKaHT Ru

Bonpoc: N°49
Patient, 29 years old, consulted a family doctor with complaints of headaches, weight gain,
98B
weakness, palpitations, menstrual irregularities. On examination: rounded face, acne. Weight
kg,height- 162 cm. Body mass index 37.6 kg/ m2. Conclusion: obesity degree. Choose,
lI
-

which method is used to determine of obesity?

OTBeTbi(oqMH OTBET)

1 Borngardt index

2 Brock's index

3 Quetelet index

4 Jackson-Pollock index

5 Breitman index

43 44 45 46 47 48 49 50

1 234 567 89 10
11 12 13 14 15 16 17 18 19 20
21 22 23 24 25 26 27 28 29 30
avn.kgma.kg

TynTa PaBMKaHT Ru

Bonpoc: N°23
Name the hormone that regulates the function of
the thyroid gland:

OTBeTbi(OqMH OTBeT)
1 adrenocorticotropic hormone

2 thyroid-stimulating hormone

3 thyroglobulin
4 thyrocalcitonin

5 thyrotropin releasing hormone

19 20 21 22 2324 25 26 27
50

12 3 456 789 10
11 12 13 14 15 16 17 18 19 20
21 22 23 24 25 26 27 28 29 30
31 32 33 34 35 36 37 38 39 40
41 42 43 44 45 46 47 48 49 50
Mymap Om Hapa
gma.kg/webtest/testing

Bonpoc: N°50 memory loss,drowsiness.


rapid fatigue,
the neck, apparentreasonand developed
pressure in the front of no
complains of a feeling of complaints arose years ago for gland is
with a local
therapist
that these
6gland are palpated, more on the right: theanti-TPO-364
appointment anamnesis thyroid pmol/ml,
years old, at an It is known
from the
and both lobes
of the
detected: 8-miU/L
T4fr.-8.6 clarifythe
Patient M., 44 autoimmune thyroiditis. touch. The isthmus laboratory tests, TSH was intervention should beused to
was to the
Anamnesis: Mother is clean, dry, cold
mm Hg. In What diagnostic
examination: the skin rhythmic. BP 90/70 hemoglobin 96g /L
gradualy. On per minute, leukocytes 5.8 x 109/,
moderately painful. Pulse 53 1012/1,
dense, erythrocytes 3.5
count:
mi. Total blood
IU/
diagnosis? OTBeTbI(oAuH OTBer)

1311
thyroidscan with
thyroid gland
auscultation of the
thyroid gland
ultrasound of the
thyroid gland 50
MRI of the 48
49
neck 47
tadiography of the 44 45 46
lateral
TynTa PaBMKaHT Ru

Bonpoc: N 22
A 60-year-old
man was admitted to the endocrinology department with
suspected basophilic
adenoma of the anterior pituitary gland. Complains of general weakness, frequent bronchitis,
headache, pain in the back and limbs. Objectively: the clinical picture corresponds
to Cushing's
syndrome. In the analyzes, there is an increase in the basal level of corticotoropin in
blood
plasma and glucocorticoids in plasma and urine. With fluoroscopy, pronounced
osteoporosis
thebones of the skull, vertebral bodies, hyperplasia of both adrenal glands. Which of the of
following answers is most likely

OTBeTbi(ogMH OTBeT)

1 Estrogen deficiency

2 Imbalance between bone formation and destruction

3 High glucocorticoid levels

4 Vitamin D deficiency

5 Aging of the body

18 19 20 21 22 23 24 25 26
50
11:25 AM Mon 11 Jan

O A avn.kgma.kg

BMHan KyMap Arapsan. Kg Ru


Bonpoc: N920
A 25-year-old man complains of nausea, headache, increased
thirst and
polyuria. Gastroscopy revealed no gastrointestinal pathology.
The condition
worsened thirst and polyuria increased, the amount of liquid
drunk
increased to 8 liters, a constant headache accompanied by
vomiting, there
was a loss of lateral visual fields. Urine analysis: a
decrease in the specific
gravity in the morning portion of urine to 1002, blood
osmolality 315
mosm/ kg, urine osmolality 270 mosm/ kg. Fasting blood sugar 3.2
mmol/1. Which of the following methods is priority:

OTBeTbl(oAMH oTBET)
1 Determination of blood sugar during the day

2 MRI of the brain


3 ultrasound of the kidneys
4 Fluid restriction test
5 Excretory urography

16 17 18 19 20 21 22 23
24 50

1 2 3 4 567
89 10
11 121314 151617 181920
A Be6 TeCTMpOBaHMe
A avn.kgma.kg

TynTa PaBMKaHT Kg Ru

Bonpoc: N935
A white sclera line between the iris and the upper eyelid when looking down with thyroid
hyperfunction syndrome is a symptom:

OTBeTbI(oaMH OTBeT)
1 Kocher

2 Grefe

3 Elinek
4 Krause

5 Shtelvag

31 32 33 34 35 36 37 38 39

50
11:23 AM Mon 11 Jan

O Aavn.kgma.kg

BMHanKyMap Arapsan. Kg Ru
Bonpoc: N94
A mother consulted a doctor, whose son had grown by 10 cm over
the
summer. When examining a 19-year-old boy: height
180 cm, weight 68
kg. Which research method is more applicable
for diagnosing a disease:

OTBeTbi(oAMH OTBET)
1 target X-ray of the skull

2 magnetic resonance imaging


3 computed tomography
4 ultrasound
5 craniography

1 2 3
456 7 50

1 2 3 456 7 8 9 10
11 12 13 14 15 16 17 18 19 20
21 222324 25 26 2728 29 30
31 3233 34 35 36 37 3839
40
4142 4344454647 4849 50
OTBeHeHHbi Bonpoc
nponyueHHbi Bonpoc
TEKyuw Bonpoc
Bonpoc: N°29
Laboratory sign of hypercortisolism syndrome

OTBeTbi(oAMH OTBET)
1 hyperproteinemia

2 hypernatremia

3 hypocalcemia

4 O hypoglycemia

5 hyperkalemia

25 26 27 28 29 30 3132 33
U il 83 11:29
X Be6 TecCTWpOBaHWe
avn.kgma.kg

TynTa PaBMKaHT Ru

Bonpoc: N°11
The initial product for the synthesis of thyroid hormones
is:

OTBeTbi(oqMH OTBeT)
1
Tyrosin

2 Wallin
3 Phenylalanine

4 Leucine

5 Oxyproline

1
8 9
10 1112 13 14 15
50

23 4 5 67
89 10
12 13 14 15 16 17 18 19 20
Beó TecTMPOBaHe
XA avn.kgma.kg <
rynTa PaBMKaHT Ru

Bonpoc: N°17
Onthe basis of which pituitary hormone are drugs used to stimulate the contractile activity of
the uterus:

OTBeTbl(oAMH OTBeT)
Oxytocin

2 Prolactin

3 Thyroid stimulating hormone

4 Vasopressin
5 Follicle-stimulating hormone

13 14 15 16 17 18 19 20 21

50

1 23 456789 10
11 12 13 14 15 16 17 18 19
20
21 22 23 24 25 26 27 28 29 30
HapanH
Kymap OM

ma.kg/webtest/testing

from diabetes
Bonpoc: N944 training,
beensuffering
the boy haspreserved.BP 120/90 mm Hg
goodbefore soundsare consciousness.
trainer, he felt vesicular.Heart regained
According to the Breathing is and the patient
unconscious state. skin is moist. glucose solution,
to the clinic in an consciousness. Objectively The
hypoglycemia,
prescribed a
was admitted suddenly lost diagnosed him with
A
17-year-old boy training, he physician
childhood; during attending
since /1. The
glucose 2.7 mmol case?
Laboratory hypoglycemia in this
cause of OTBETb1(oanH oTBeT)
Explain the

Odrinkalcohol
overdose
metformin
2
untimelymeal
50
activity 47
physical 46

overdose 42
43 44 45
insulin 40
41

1731
Vi Indialldea
0 l 83 11:28
Be6 TecTMpOBaHe
avn.kgma.kg

TynTa PaBMKaHT Kg Ru

Bonpoc: N95
Woman M., 35 years old, who has been
smoking for 18 years, found in her left the mammary
gland has a nodular formation, up to 2 cm in diameter. She turned
to the mammologist, who
confirmed the presence of a mass. Predict which radiological
examinations the patient should be
referred to for diagnostic information?

OTBeTbi(oqMH OTBeT)
1 X-ray of the chest

2 mammography
3
magnetic resonance imaging

4 scintigraphy

5 multi spiralcomputed tomography

1 2 3 67 8 50

1 2 3 45 6 7 89 10
x Eriddocint linalpd

Bonpoc: N93
Indicate what volume of contrast agent is injected with intravenous contrasting of the adrenal gland:

OTBeTbI(oaMH OTBeT)
1 90-120 ml
2 O 120-150 ml

3 30-60 ml

4 O60-90 ml
5O150-180ml

1
234 5 6 7 50

prt sc delete

&
8 9 backspace
avn.kgma.kg/webtest/testing
Kymap OM HapaH

Bonpoc: N96
examination, bradycardia, pasty skin and
lethargic, his voice is hoarse. On
solid food. Objectively, the patient is upper third of the esophagus to the
right, the
patient complains of difficulty passing deviation of the pharynx and the
A 63-year old which showed a moderate
swallow was performed, you can use?
0 subcutaneous fat. A barium informative research method
the most
normal. What is
0 contours of the esophagus are
OTBeTbi(ogMH OTBeT)

tomography
1 mnulti spiralcomputed
AoCb
thyroid gland
tomographywith contrasting
2 computed
endoscopy
3
thyroid ultrasound

Ooverview Radiography 9 10
S0
5 8
3 4
IO 3
X A Be6 TecTMpOBaHe 1129
avn.kgma.kg

TynTa PaBHKaHT
Ru

Bonpoc: N°10
Violation
of theprocesses of secretion in
the endocrine gland can occur
due to

OTBeTbI(oqMH OTBeT)
1
Changes in the number of specific
receptors
2
Disorders of hormone metabolism

3 Violations of the feedback


mechanism

4 Weak transport connection


with blood proteins

5 Genetic defect in hormone synthesis

1 6 8 9 10 11 12 13 14
50

1 2 3 4567 89 10
1112 13 14 15 16 17 18 19 20
Kywap Ow Hapans

t/testing9

Bonpoc: N99
can lead to:
endocrine disorders
peripheral form of
To
OTBeTbl(oAMH OTBer)

hormone-protein binding
Weak
hypothalamus
Alteration of the
alteration
Pituitary
endocrine gland
Alteration of the proteins 50
hormones with 12
13
Strong bond of 10
11
11:21
< 0 2e 4 80%

Pan HwTWLI Kg Ru

Bonpoc: Ne16
On the basis of which pituitary hormone are drugs used
to stimulate the contractile activity of the uterus:

OTBeTbI(oqMH OTBeT)
1 Oxytocin

2 Follicle-stimulating hormone

3 Thyroid stimulating hormone

4 Prolactin

5 Vasopressin

12 13 14 15 16

17 18 19 20 50

1 3
4 5 67 8 9 10
2

11 12 13 14 15 16 17 18 19 20
21 22 23 24 25 26 27 28 29 30
31 32 33 34 35 36 37 38 39 40
41 42 43 44 45 46 47 48 49 50
OTBeyeHHbIM BOnpoc
nponyueHHbl BOnpoc

O
12:42
l 914
A avn.kgma.kg/webtest/testing

Kymap HupaHaaH

Bonpoc: N°12
is:
The most common cause of Addison's disease

OTBeTbI(ogMH OTBeT)

1 Adrenal atrophy

2 Pituitary tumor

3 Autoimmune thyroiditis

4 Hyperplasia of the pineal gland

5 Ovary hypertrophy

1 8 9 10 11 12

14 15 16 50

1 2 3 45 6 78 9 10
ehich micro O of 0
) Porionbitol
HapanH
Kymap OM

hypothyroidism?
Bonpoc: Ne26 babieswith
membranesin
mucous
and
facial skin
pallor of the
explains the OTBeT)
Highlight, what OTBeTbl(oquH

deficiencCy
vitamin «C
deficiency
iron
2 hypovitaminosis B12
activity 50
physical
decreased vessels
blood

deep
arrangement of

22
24
25 27
Vi Indialldea
O l 1 11:28

Be6 TecTMpoBaHMe
X avn.kgma.kg

TynTa PaBMKaHT Kg Ru

Bonpoc: N94
patient with complaints of abdominal pain was delivered to the admission department of the
hospital. An ultrasoundof the pancreas:head 18 mm, body 16 mm, tail 17 mm. It is visualized
satisfactorily. The contours are clear, smooth. The structure is moderately
heterogeneous, the
echogenicity is moderately increased. There is a volumetric formation in the head of the
pancreas in the form of an anechoic formation up to 12 mm. Conclusionis

OTBeTbi(oaMH OTBeT)

1 nodular formation

2 malignant neoplasmn

3 sclerosis

4 calcification

5 fluid cyst

12 2 3 4 5 6 7 50
A Be6 TecTHpoBaHme <
avn.kgma.kg

TynTa PaBMKaHT KRu


Bonpoc: N°38
Normal level of body mass index (kg/ m2) is

OTBETbl(OAMH OTBeT

1 30.0 -34.9
2 35.0 -39.9
3 25.0-29.9

4 40.0 45.9

5 18.5-24.9

34 35 36 37 38 39 40 41 42

50
SatseuG
l 43%12:45 pm
Be6TeCTMpOBaHn x

C A avn.kgma.kg/webtes o
CHHrx KynaAMn Kg Ru

Bonpoc: N941

woman consulted a family doctor at her place of


A 55-year-old
residence with complaints of thirst, dry mouth, weight gain,
headache, pain in the lumbar region, sweating. Objectively:
height-170 cm, weight- 120 kg, moon-shaped face, crimson-
red,excessive growth of facial hair, acne, excessive deposition
of fatty tissue in the shoulder, chest, abdomen, striae of red-
violet color, atrophy of the muscles of the arms and legs. BP -

190/100 mm Hg. Explain, what caused the development of


obesity syndrome?

OTBeTbi(oaMH OTBeT)
1 hypercortisolism
2 hyperprolactinemia

3 hyperparathyroidism

4 hyperglycemia

5 hyperthyroidism

37 38 39 40 41 42 43 44 45
50

2 3 456 7 8 9 10
11 12 13 14 15 16 17 18 19 20
1 2223 24 25 26 27 28 29 30
pm
S1245

Bonpoc: N41
A 55-year-old woman consulted a family doctor at her place of
residence with complaints of thirst, dry mouth, weight gain
headache, pain in the lumbar region, sweating. Objectively
height-170 cm,weight-120 kg moon-shaped face, crimson
red, excessive growth of facial hairt,acne, excessive deposition
of fatty tissuein the shoulder,chest,abdomen, striae ofred
violet color, atrophy of the muscles of the arms and legs. BP-
190/100 mm Hg. Explain, what caused the development of
obesity syndrome?

OTBeTbi(OAMH OTBeT)

1 yperc olism

yperpn

3 byperparathyroidism

byperglycema

5 hyperthyroidism

37 38 39 41 42 3 5
50

23 45
11 12 13 141516678 9 10
17 18 19 20
21 22 23 24 25 27 29
26 28 30
32 33 34 35 3 37 38 39 40
42 43 44 45 46 47 48 49 50
oTBeveHblN BONpoc
ponyueHHb
Bonpoc
TynTa PaBMkaHT Ru

Bonpoc: N924
Indicate the main symptom in hyperglycemia syndrome:

OTBeTbi(oqMH OTBeT)
1
bad appetite
2 cold sweat
3 polydipsia

4 hyperactivity

5 tremor of the hands, body

1
20 21 22 23 24 25 26 27 28
50

1 2 3 4 5 6 7 89 10
11 12 13 14 15 16 17 18 19 20
21 22 23 24 25 26 27 28 29 3
31 32 33 34 35 36 37 38 39 40
41 42 43 44 45 46 47 48 49 50

-OTBE
11:24 AM Mon 11 Jan

A avn.kgma. kg

BMHan KyMap ArapBan Kg

Bonpoc: N97
The highest content of antibodies to microsomal antigen in blood
is
observed when:

OTBeTbl(OAMH OTBeT)
1 Diffusetoxicgoiter

2 Thyroid cancer
3 Subacute thyroiditis
4 Autoimmune thyroiditis
5 Thyrotoxic adenoma

89 10 11
50

A234 5 678 10
11 12 13 14 151617 18 9
19 20
21 22 232425 26 27 28 29 30
31 323334 35 36 3738 3940
41424344 454647 484950
OTBeHeHHbl 5onpoc
iponyuueHHl BOnpoc
Vi Indialldea
O1 11.30
Be6 TecTWpOBaHe
avn.kgma.kg
<
rynTa PaBMKaHT Ru

Bonpoc: N°25
Indicate the hormone that regulates the secretion of thyroid-stimulating
hormone by the
pituitary gland:

OTBeTbi(ogMH OTBeT)

1 adrenocorticotropic hormone

2 thyrotropin releasing hormone

3 thyroglobulin

4 thyrocalcitonin

5 thyroxine

21 22 23 24 25 26 27 28
28 29
50
KywaBaxa Paxyn KymMap

Bonpo: N98
Changes in oxytocin secretion play a role in pathogenesis:

OTBeTbi(oAuH OTBET)
1 Disorders of carbohydrate metabolism in diabetes mellitus

2 Diffuse toxic goiter


sleep
3 Disorders of circadian rhythms "wakefulness-

4 Disorders of labor

5 Myxedem
11 12 50
10
1
6

10

1212 313 144 5 67R


9
17 18
15 16 19 20
11
11:24 AM Mon 11 Jan

A avn.kgma.kg

BHHan KyMap ArapBan. Kg Ru

Bonpoc: N915
Patient N., 45 years old, complains of weakness, rapid fatigue, lack of
appetite, weight loss, pain in the abdomen. Objective
examination: the
skinand visible mucous membranes of bronze color, blood pressure is
reduced. Dysfunction of which endocrine gland can cause
the following
symptomsS:

OTBeTb(oAMH OTBeT)
1 Ovary
2 Pancreas

3 Epiphysis

4 Adrenal glands
5 Pituitary gland

11 12 13 14 15 116 17 18
19 50

12 3 4 56789 10
11121314151617 181920
212223 24 25 26 27 28 29 30
31 32 33 34 35 36 37 38 3940
4142434445 4647 4849 50
KyMapOM Hapamy
ma.kg/webtest/testin9

Bonpoc: N92 operation?


structure was damaged during the
Determine which
patient developed convulsions.
of the thyroid gland, the
After removal
OTBeTb(ognH OTBeT)

sympathetictrunk
nerve
inferior laryngeal
2
parathyroidgland
3
vagus nerve
4 the
laryngeal artery 50
Oinferior 4 5 6
7
11:23 AM Man 11 Jan

A avn.kgma.kg

BHHan KyMap ArapBan. R

Bonpoc: N92
On a CT scan, the pancreas is of the usual location, its contours are clear,
the structure is homogeneous, without focal changes.Determine the
condition of the pancreas:

OTBETbI(oqMH OTBeT)
1 pancreatitis

2 normal
3 tumor
4 cancer

5 Cystadenomna

1 2 3 4 5 6 7 50

23 4 5 67 8 9 10
11 12 13 14 15 16 17 18 19 20
21 22 23 24 25 26 27 28 29 30
3132 3334 3536 37 38 39 40
4142 43 4445 4647 4849 50
OTBeueHHbl Bonpoc
rnponyuieHHbiM BOnpoc
TEYuwi Bonpoc
11:25 AM Mon 11 Jan

A avn.kgma.kg

BMHa Kymap Arapsan Kg Ru


Bonpoc: N922
A 30-year-old man complains of recurrent seizures of
the muscles of the
limbs and face, numbness of the fingers and toes,
pain in the chest and in
the epigastric region, difficulty in breathing. The
disease developed after
removal of part of the thyroid gland for nodular
goiter. The patient is pale.
The hair is sparse, gray. Brittle nails. Neurological
examination a sharp
increase in the excitability of the nervous and
muscular systems. The
content of Ca in the blood is significantly
reduced, the content of
phosphorus is increased. Indicate which
endocrine glands function is
impaired:

OTBeTbi(oAnH oTBeT)
1 Neurohypophysis

2 Adrenal glands
3 Adenohypophysis

Thyroid

Parathyroid

18 19 20 21 22 23 24 25
26 50

1 2 3 4 567 8 910
11:21
<. 0 2e 4 80%

Pan HWTWLI Kg Ru

Bonpoc: Ne15
After removal of the thyroid gland, the patient developed
convulsions. What was damaged during the operation:

OTBeTbI(ogMH OTBeT)
1 Vagus nerve

2 Sympathetic trunk

3 Parathyroid gland

4 Inferior laryngeal artery

5 Inferior laryngeal nerve

1 11 12 13 14 15

16 17 18 19 50

1 2 3 4 5 67 8 9 10
11 12 13 14 15 16 17 18 19 20
21 22 23 24 25 26 27 28 29 30
31 32 33 34 35 36 37 38 39 40

G 15
ebtest/testing Kymap Ot Hapan

Bonpoc: N934
hypercortisolism is
cause of secondary
The most common

OTBeTbi(oAuH OTBeT)

corticosteroma
1O
corticotropinoma
2
hypophysectomy
3
pheochromocytoma
4
meningoencephalitis 38
50
37
5
30 31 32 33
34 35 36
(0

in
11:25 0 0 09 KB/S
Y 4l 84%

Bonpoc: Ne45
A 40-year-old woman, at an appointment with an
endocrinologist in a polyclinic, complains of muscle
weakness, rapid physical fatigue, weight los by 12 kg in
the last 2 months, lack of appetite, nausea, muscle pain,
addiction to salty foods, menstrual irregularities. On
examination, weight -48 kg, height- 166 cm. The skin
has a dark brown color, dark spots on the mucous
membrane of the oral cavity, subcutaneous fat is
thinned. BP -70/50 mm Hg. The pulse on the radial
artery is soft, small filling. What explains the decrease in
blood pressure in this patient?

OTBeTbi(oaMH OTBeT)
1 hypothyroidism

2 hypoglycemia

3 hypocorticism

4 hypopituitarism

5 hypoparathyroidism

1 41 42 43 44 45

46 47 48 49 50

G 45

O
Bonpoc: Ne26
Highlight, what explains the pallor of the facial skin and mucous membranes babies with hypothyroidism?
in

OTBeTbI(oAnH OTBeT)

1C iron deficiency

2 hypovitaminosis B12

3 vitamin «C» deficiency


4 deep arrangement of blood vessels

5 decreased physical activity


50
22 23 24 25 27 2829 30
Be6 TecTWpOBaHMe
avn.kgma.kg

TynTa PaBHKaHT Kg R
Bonpoc: N912
Hormone metabolism is disturbed in diseases

OTBeTbl(oqMH OTBET)
1 Liver

2 Hearts

3 Lungs
4 Spleen
5 Nervoussystem

1 8 9 10 11 12 13 14 15 16
50

3 45 67 8 9 10
13 14 15 16 17 18 19 20
1 00
12:39 tall 92%

OA avn.kgma.kg/webtest/testing

KyMap HupaHamaH Kg Ru

Bonpoc: Ne14
Violation of the processes of secretion in the endocrine gland
can occur due to:

OTBeTbI(oqMH OTBeT)
1
Genetic defect in hormone synthesis

2 Weak transport connection with blood proteins

3 Violations of the feedback mechanism

4 Changes in the number of specific receptors

5 Disorders of hormone metabolism

10 11 12 13 14 15
16 17 18 50

ehich micro 0 of 0

A. Periorbital edema with purplish-purple erythema


B. Cyanotic eyelid
skin staining
C.Dark circles under the eyes
D. Hyperpigmentation of the upper eyelids
E. Eye constriction with erythema
of eyelids
4. Sjogren syndrome is:
A. "dry" syndrome due to dehydration
B. "dry syndrome due tn lesionr nf
LJE 33111:30
ll
ViIndialldeaP

x Be6 TecTMpOBaHne
avn.kgma.kg
<
TynTa PaBMKaHT Ko Ru

Bonpoc: N936
Obesity is a risk factor for the development of the syndrome

OTBeTbI(oAMH OTBeT)

1 gigantism

2 hyperglycemia

3 hypercortisolism

hyperthyroidism

5 hyperparathyroidism

32 33 34 35
3637 38 39 40

50

12 3 456 7 89 10
ViIndialldea IO ll 83 11

Be6 TecTWPOBaHne
X A avn.kgma.kg

TynTa PaBMKaHT Ka

Bonpoc: N°27
Identify the main cause of constipation and flatulence in children with hypothyroidism:

OTBeTbI(oAMH OTBeT)
1
low water consumption

2 poor appetite

3 decrease metabolism of substances

4 decreased physical activity

5 motor function of the intestine

23 24 25 26 27 28 29 30 31
50
al 43%1245pm
Ceerx Kyna

Bonpoc: N40
A29-year-old man, was admitted to the clinic with complaints
ofpalpitations, sweating, tremors inthe body, weakness
diarhea, darkening of the skin in places of skin friction.
Anamnesis: has been ill for several years. Examination: the
apical impulse is enhanced. Heart sounds are loud,
arhythmic. Pulse 98 in 1 minute. blood pressure 170/60 mm
Hg.TSH is reduced, T3, T4 is increased. The doctor
diagnosed
hyperthyroidism syndrome. Explain what
is the connection
with the appearance of skin darkening due to0?

OTBeTbi(0AMH OTBeT)

1 yms iciency

parathyro insufficien

3 renail failur
adrenal insufficiency

liver failuree

37 38 39 40 41 42 43 44
50

12 3 4
11 12 13 1415161789
10
m 1819 20
22 23 24 25 26 27
2 29 30
32 33 34 3s 6 37 39
41 42 43 44 45 45 47 48 49 50
OTBeves
BOnpoc
penyuEHHi BonpoC
A 43%12:49 pm
CHrxKynAun 2Ru

Bonpoc: e41
55-year-old woman consulted a family doctor at her place of
A

residence with complaints of thirst, dry mouth, weight gain,


headache, pain in the lumbar region, sweating. Objectively:
height-170 cm, weight-120 kg, moon-shaped face, crimson-
red, excessive growth of facial hair, acne, excessive deposition
of fatty tissue in the shoulder, chest, abdomen, striae of red-
violet color, atrophy of the muscles of the arms and legs. BP -
190/100 mm Hg. Explain, what caused the development of
obesity syndrome?

OTBeTbI(oAMH OTBET)

1 hypercortisolism

2 hyperprolactinemia

3 hyperparathyroidism

4 hyperglycemia
5 hyperthyroidism

37 38 39 40 41 42 43 AA A5
50

12 3 45 6 7 8 9 10
11 12 13 14 151617 18 19 20
2122 23 24 25 26 27 28 29 30
31 32 33 34 35 36 37 38 39 40
43 42 43 44 45 46 47 48 49 50
OTBeveHHbIM
BOnpoc
nponyujeHHbA Bonpoc
11:24 AM Mon 11 Jan

A avn.kgma.kg

BMHan Kymap Arapsan. Kg Ru

Bonpoc: N911
Changes in oxytocin secretion play a role in pathogenesis:

OTBeTb(oAMH OTBET)
1 Myxedema

2 Disorders of labor
3 Disorders of circadian rhythms "wakefulness sleep"
4 Disorders of carbohydrate metabolism
in diabetes mellitus
5 Diffuse toxic goiter

7 8 9 10 11 12 13 14 15
50

1 3 45 6 7
2
89 10
1112 131415 16 1718 1920
21 22 23 24 25 26 27 28 29 30
31 32 333435 36 37 38 3940
41 42 4344454647 4849 50
OTBeHeHHbl
BOnpoc
nponyuieHHbl
BOnpoc
Tekyuinn BOnpoc

+
12:26 ®o00 ll 93%

A avn.kgma.kg/webtest/testing

Kymap HupaHaaH Kg Ru

Bonpoc: N948
Patient, 29 years old, consulted a family doctor with complaints
of headaches, weight gain, weakness, palpitations, menstrual
irregularities. On examination: rounded face, acne. Weight 98 kg,
height 162 cm. Body mass index 37.6 kg / m2. Conclusion:
obesity IIl degree. Choose, which method is used to determine of
obesity?

OTBeTbI(oAMH OTBeT)
1 Breitman index

2 Borngardtindex

3 Jackson-Pollock index

4 Quetelet index

5 Brock's index

5- A9

ehich micro 0 of 0

A. Periorbital edema with purplish-purple erythema


B. Cyanotic eyelid skin staining
C. Dark circles under the eyes
D. Hyperpigmentation of the upper eyelids
E. Bye constriction with erythema of eyelids
4.Sjogren syndrome is:
A. "dry syndrome due to dehydration
B.'dry" syndrome due to lesions of the endocrine glands
C.dry" syndrome due to lesions of the exocrine glands
12:17 PM Mon 11 Jan

O A avn.kgma.kg

GG Go
A patient girl, 14years of life, turned with mothers to the family doctor
with complaints on heartbeat, feeling of pressure on the neck, sweating, gle cholester
weight loss. Anamnesis: the above complaints appeared 2 weeks after
the death of the father. Objectively: height 160 sm (N 160), weight
- aA
- 44 kg (N- 51 kg). Nervous, irritable. The skin and palms are very About 11.

moist, tremor of the hands. The thyroid gland is I-II degree. Heart
tones increased, tachycardia. Heart rate 132 in 1 min. Arterial Lower
chole
pressure- 130/72 mm.Hg. Increased appetite, but losing weight. Often LDL

loose stools. What diagnostic study should be done to diagnose the www.
syndrome? Pota

OTBeTbi(OAMH OTBeT) Peo


Wha
1 cholesterol, potassium
Is it
2 common analysis of blood
Wha

3 TSH, T4 Wha

4 glucose blood www.in


Hype
5 common protein, protein fractions
The m
chiann
by AN
25 26 27 28 29 30 31
24
pubm

50 Athe
32
Theb
ester
by

1 2 3 4 5 6 78 9 10 aca
11121314 15 16 17 18 1920 In
weel
21222324 2526 272e 2930 9ro
by

CeOTO9y
Transcript-based notes on video lectures in Ma
Dr. Mayur Arun Kulkarni, MD, DNB3
12:39 ®8 all 92%

OA avn.kgma.kg/webtest/testing

Bonpoc: N°21
A 25-year-old man complains of nausea, headache, increased
thirst and polyuria. Gastroscopy revealed no gastrointestinal
pathology. The condition worsened -thirst and polyuria
increased, the amount of liquid drunk increased to 8 liters, a
constant headache accompanied by vomiting, there was a loss of
lateral visual fields. Urine analysis: a decrease in the specific
gravity in the morning portion of urine to 1002, blood osmolality
315 mosm / kg, urine osmolality-270 mosm / kg. Fasting blood
sugar- 3.2 mmol/1. Which of the following methods is priority:

OTBETbl(OAWH OTBeT

1 MRI of the brain

2 Fluid restriction test

Determination of blood sugar during the day

4 Excretory urography

5 ultrasound of the kidneys

ehich micro O of 0
A. Periorbital edema with purplish-purple erythema
B.Cyanoticeyelid skin staining
C. Dark circles under the eyes
D. Hyperpigmentation of the upper eyelids
E. Eye constriction with erythema of eyelids
4.Siogren svndrome is
11:25 0 <. I0 RS t 4l 84%

Bonpoc: N940
Patient A., 64-year-old, female consulted a local general
practitioner complaining of dry skin, overweight (BMI
28 kg/ m2), decreased body temperature, decreased
appetite, constipation, decreased memory, drowsiness.
From the anamnesis: no surgery on the thyroid gland or
irradiation of the neck. Father had a thyroid disorder. On
examination: the skin is dry, cold to the touch. Swelling
of the face. Vesicular breathing, no wheezing. Heart rate
58 per minute. BP -110/70 mm Hg. Body temperature
36.3 C. Stool irregular, constipation tendency.
According to laboratory examination data: total blood
test: hemoglobin 104 g/1, cholesterol-9.4 mmol /,
-

triglycerides -5.5 mmol/ I, TSH-15.3 mU/ 1. Whatisthe


likely cause of hypothyroidism syndrome?

OTBeTbI(oqMH OTBeT)
1 syndrome of hyperthyroidism

2 syndrome of hypofunction of the parathyroid


glands

3 thyroid nodule syndrome

4 syndrome of acute inflammation of the thyroid


gland

5 syndrome of chronic inflammation of the


thyroid gland

O
kgma.kg/webtest/testing
yseap Ow Hapasn

Bonpoc: N941
by 12 kg in the
of muscle weakness,
rapid physical fatigue, weightloss
in a polyclinic, complains 166 cm. The skin has a
endocrinologist 48 kg, height-
woman, at an appointment with
an irregularities. On examination, weight- on the radial artery is soft, small
A 40-year-old salty foods, menstrual
appetite, nausea, muscle pain, addiction to fat is thinned. BP - 70/50 mm Hg. The pulse
subcutaneous
last 2 months, lack of membrane of the oral cavity,
spots on the mucous
dark brown color, dark pressure in this patient?
filling. Whatexplains the decrease in blood

OTBeTbI(oanH OTBeT)

Ohypothyroidism
hypopituitarism
2
hypocorticism
3
hypoglycemia
40
hypoparathyroidism 44 45
43
5 39 40 1 42
37 38
Vi Indialldea 01 l3 11:28

X A Be6 TecTHpOBaHMe
avn.kgma.kg

rynTa PaBMKaHT Kg Ru

Bonpoc: N°7
An increase in the concentration of thyroid-stimulating hormone in the blood during
hypothyroidism indicates the localization of the pathological process in:

OTBETbi(OAMH OTBeT)

1
Thyroid gland
2 Timuse

3 Parathyroid glands

44 Pituitary gland

5 Hypothalamus

1 3
45 6 9 10 11 50

23
4 5 67 8 910 10
12 14 15 16 17 19 20
Rl 49N12:00pm
Curx KyAAH

Bonpoc: Ne49
appointment with a local
Patient M., 44 years old, at an
therapist complains of a feeling of
pressure in the front of the
AnamnesisS
neck, rapid fatigue, memory loss, drowsiness.
Mother was autoimmune thyroiditis. It is known from the
anamnesis that these complaints arose 6 years ago for no
apparent reason and developed gradually. On examination the
skin is clean, dry, cold to the touch. The isthmus and both
lobes of the thyroid gland are palpated, more on the right the
gland is dense, moderately painful. Pulse- 53 per minute,
rhythmic. BP-90/70 mm Hg. In laboratory tests, TSH was
detected: 8-miU/L, T4 fr.- 8.6 pmol/ ml, anti-TPO-364 IU/
ml. Total blood count: erythrocytes 3.5 x 1012 /1, leukoeytes
5.8x109/1, hemoglobin-96g/1. What diagnostic
intervention should be used to clarify the diagnosis?

OTBeTbi(oAWH OTBeT)
1 lateral radiography of the neck

2 auscultation of the thyroid


gland
3 thyroid scan with 131 1

4 MRI of the thyroid


gland
5 ultrasound of the thyroid
gland

43 44 45 46 47 48 49

345
am
49%11:59
TeCTWpOBaHMX
Be6
C A
avn.kgma.kg/webtes
CWHrx Kyngun
o
Kg Ru

Bonpoc: N 40
admitted to the clinic with complaints
A 29-year-old man, was
palpitations, sweating, tremors in the body, weakness,
of skin friction.
diarrhea, darkening of the skin in places of
Anamnesis: has been ill for several years.
Examination: the
apical impulse is enhanced. Heart sounds are loud,
arrhythmic. Pulse 98 in 1 minute. blood pressure 170/60 mm
Hg. TSH is reduced, T3, T4 is increased. The doctor diagnosed
hyperthyroidism syndrome. Explain what is the connection
with the appearance of skin darkening due to?

OTBeTbI(oAMH OTBeT)

1 thymusinsufficiency

2 parathyroid insufficiency

3 renalfailure
4 8adrenalinsufficiency
5 liver fallure

36 37 38
50
39 40
4142 43 44
49%11:
TeCTWpOBaHX
Be6
A avn.kgma.kg/webtes
C
CHHrx Kynqun

Bonpoc: N°36
Laboratory sign of hypercortisolism syndrome

OTBETbI(oAMH OTBeT)

1 hyperkalemia

2 hypernatremia

hypocalcemia
4 hypoglycern.a

5 hyperproteinemia

32 33 34 35 36 37 38 39 40
50

23 45678 9 10
1 12 13 14 15 16 17
18 19 20
2122 2324 25 26 27 28 29 30
31 32 33 34 353 37 38 39
41 42 43 44 45 46 47 40
48 49 50
OTBeveHHbli
BonpocC
niponyuueHHbl
BOnpoc
TeKyuw
49%11:59 am

TeCTWpOBaHWX
Be6
C
avn.kgma.kg/webtes o
Kg Ru
CHHrx Kynaun

Bonpoc: N°45

60 years old, applied for a consultation


to a local
Patient K.,
complaints of
general practitioner at an policlinic with
constipation, hoarseness, memory loss, weakness,
it is
drowsiness, and swallowing of face. From the anamnesis
known: symptoms have been disturbing for the last 12
these
months. Objectively: The skin is hyperkeratosis.
dry, elbow
Swallowing of face. The thyroid gland is not enlarged by
palpation, painless, nodular formations are not determined.
Hormonal profile and antibody determination: TSH- 26.7 uIU/
ml. T4 free 6.1 pmol/1, AB to TPO-271U/ml, AB to
thyroglobulin -310 ug/ ml. Ultrasound of the thyroid gland: V-
right lobes 3.0 cm3, V -left. lobes -2.1 cm3, V total. 5.1 cm3.
Echogenicity is increased. Pronounced diffusely
heterogeneous structure, nodular formations are not
determined. What syndrome does the hormonal profile
data
indicate?
hypothyroidism
OTBeTbl(oAMH OTBeT)
1 thyroidinflammation
syndrome
2 syndrome of hyperthyroidism

syndrome of hypopituitarism

4 syndrome of hypoparathyroidism
49%
A
Be6TeCTMpoBaHWX
C avn.kgma.kg/webtes
CHurx Kyngan

Bonpoc: N°33
Thyroid-stimulating hormone is normal (mU/ 1)?

OTBETbi(oAMH OTBeT)
1 from 0.1 to 3.0

2 from 0.4 to 4.0

3 from 2.1 to 7.5


4 from 1.0to 5.0

5 from 1.4 to 6.1

29 30 31 32 33 34 35 36
50

23 45678
11 12 13 14 15 16 17 18 9
10
19 20
21 22 2324 25 26 27 28 29 30
31 32 33 34 35 36 37 38 39 40
41 42 43 44 45 46 47
48 49 50
OTBeveHHbi
BOnpoc
nponyueHHbi Bonpoc
Texyuim BOnpoc
49%12:00 pm
CHHrx KynAun Kg Ru

Bonpoc: N948
at the place of
Patient V., 50 years old, consulted a doctor
weight, pain in
residence with complaints of increased body
200/1000
the lumbar region, increased blood pressure up to
mm Hg. Objectively: height 165 cm, body weight- 121 kg.
Obesity with redistribution of subcutaneous fat according to
the 'cushingoid" type, redness of the cheeks, striae of red-
violet color in the hips and abdomen, multiple acne on the face
and back, hypotrophy of the limbs. Fasting blood glucose 15 -

mmol/1, urine sugar 6%. What hormonal research method will


help in making a diagnosis?

OTBETbi(oAMH OTBeT)
1 testosterone
2 vasopressin

3 cortisol

4 thyroxine
5 Insulin

43 44 45 46 47 48 49 50

2345
1 12 13 14 15
6
7 5 9 10
16 17 18
212223 24 25 26 27 19 20
31 92 28 29 30
3334 35 38 37 38
A1 42 39 40
45 4& A546
474 49 50

OTBeeHHbi BOnpoc
iponyujeHHbl
BOnpoc
am
49%11:59
TecTWpOBaHMX
Be6
A avn.kgma.kg/webtes
C Ru
Kg
CHHrx Kyngun

Bonpoc: N°34
Obesity 1st degree, the body mass index (kg/m2) is

OTBETbI(oAMH OTBeT)

1 18.5-24.9

2 35.0-39.9

3 40.0-45.9

4 30.0-34.9

5 25.0-29.9

30 31 32 33 34 35 36 37 38
50

123 4
12 13 14 56789
10
11 15 16 17 18 19 20
21 23 24 25 2627 28 29 30
22
1 32 334 35 36 37 38 39 40
41 4243 44 45 46 47 48 49 50
OTBeNOHHbI
Bonpoc
IponyujewHbl
Bonpoc
TeKyun BOnpoc
49% =11:58 a
TeCTMpOBaHWX
Be6
C A avn.kgma.kg/webtes
CHHrx Kynqun Kg R

Bonpoc: N°24
Describe the stool for hypothyroidism in early childhood:

OTBeTbi(oqMH OTBeT)
1 frequent with blood

2 frequent, frothy with mucus

form as pea puree

4 persistent constipation

5 form of rice water

20 21 22 23 24 25 26 27 28
50

212 3 4567 89 10
11 1314 15 16 17 18 19 20
2122 23 225 26 27
28 29 30
31 32 33 34 35 36
37 38 39 40
41 42 43 44 45
46 47 48 49 50
OTBeHeHHbIM
BOnpoc
1DOnVLLleuu
49% 11:58 am
TeCTWpOBaHM X

Be6
avn.kgma.kg/webtes
CA Kg Ru
CHHrxKynAMn

Bonpoc: N°30
Indicate,what change in the cardiovascular system is most typical for the obesity syndrome?

OTBeTbI(oAMH OTBeT)

1 displacement of the right border

2 apical impulse is weakened, low

3 increase blood pressure


4 pulse soft, low tension

5 arrhythmia of heart sounds

26 27 28 29 30 31 32 33 34
50

2 3 45
11 12 13 14 15
678 9 10
16 17 18 19 20
21 22 23 24 25 26 27 28
29 30
31 32 33 34 35 36 37 38
39
41 42 43 44 45 46 47 48 40
49 50
OTBEveHHbi
Bonpoc
TIponmyujewHbin
Bonpoc
Texyuna Bonpoc
D D

D
l 49%11:58 am

TeCTWpOBaHMX
Be6
avn.kgma.kg/webtes
C
Kg Ru
CWHrx Kyngun

Bonpoc: N931

symptom is characterized by increasing the irritability of


What
and metacarpal
the nerves, and a flexion of the wrist joints
phalangeal joints with extension of the interphalangeal
a
and adduction of the thumb (carpal spasm) after placing
blood pressure cuff on the patient's arm and inflating to 20
mm Hg above systolic blood pressure for 3-5 minutes?

OTBETbI(oguH OTBeT)
1 Trousseau

2 Weiss
3 Schlesinger

4 'fish mouth
5 Khvostek

1 27 28 29 30 31 32 33 34 35
50

23 45 57 59 10
11 1213 14 15
16 17 18 19 20
21222324 25 26 27 28 29
30
32 3334 3E
49% 11:57 am
TeCTHpOBaHWX
Be6
avn.kgma.kg/webtes
CA CHHrx Kyngn Kg Ru

Bonpoc: N°15
appetite andgreat thirst.Diuresisis 10liters per day
A patient with complaints of decreased
Blood sugar 4mmol /1. What disease can you think of in this case?

OTBeTbi(oAuH OTBeT)

1 Diabetes insipidus

2 Diabetes mellitus

3 Gestational diabetes

Renaldiabetes

5 Non-sugaranti-diabetes

11 12 13 14 15 16 17 18 19
50

23 45 678
11 12 1314 15 16 17 18
9 10
19 20
21 22 23 24 25 26 27 28
31 32 33 34 35 36 37 29 30
38 39 40
41 42 43 44 45
46 47 48 49
50
OTBeHeHHblM
BOnpoc
npomyuueHHbi
BOnpoc
Texyutin
Bonpoc
C
49%11:59
Be6TeCTWpoBaHx
A avn.kgma.kg/webtes
CHHrx Kyngwn Kg

Bonpoc: N°44

Woman 53 years old turned to a therapist with complaints


o-

thirst, frequent urination, itching of the perineum. Anamnesis


disturb the above complaints for 1 month. Objectively: The
skin is dry and clean. BMI = 33.2 kg/m2. BP 135/95 mm.Hg.
Laboratory blood glucose -9.3 mmol/1, after meals 15.0
mmol/1. Doctor diagnosed with type 2 diabetes mellitus.
Explain why the patient developed hyperglycemia syndromei
this case?

OTBeTbi(oquH OTBET)
1 obesity

2 hypodynamia
3 heredity

4 underweight

stress

40 41 42 43 44 45 46 47
50 48
A 49% 11:
TeCTWpOBaHMX
Be6
A avn.kgma.kg/webtes
C
CHHrx KynAMn

Bonpoc: N°14
Basophilic adenoma of the adenohypophysis leads to the development of:

OTBeTbi(oguH OTBET)
1
Simmonds disease

2 Acromegaly
3 Cushing'sdisease

4 Hyperthyroidism

5 Gigantism

10 11 12 13 14 15 16 17 18
50

211124131 15 16 17 18 19
89 10
21 22 23 24 25 20
26
31 32 33 34 35 27 28 29 30
36 37 38 39
41 42 43 44 40
45 46 47
D

D
TeCTHpoBaHM X
Be6
avn.kgma.kg/webtes
C
CHHrx Kynaun

Bonpoc: N°37
(BMI)
Formula for calculating body mass index

OTBETbI(oAWH OTBET)

1 weight, kg/ height, m2

2 weight,kg / height, m

3 weight, kg2/ height, m

4 height, m /weight, kg2

5 height, m2/weight, kg

33 34 35
3637 38 39 40
50

2 3 45
11 12 13 14 15
6 78 9 10
16 17 18 19 20
122 23 24 25 26 27 28 29 30
132 33 34 35 36 3 38 39 40
41 42 43 44 45 46
47 48 49 50
OTBeHeHHbin
BOnpoc
n

O
on

Q
49%11:59 am

TeCTHpOBaHMX
Beó
C avn.kgma.kg/webtes
CWHrx Kyngun Kg Ru

Bonpoc: Ne41

A 55-year-old woman consulted a family doctor at her place of


residence with complaints of thirst, dry mouth, weight gain,
headache, pain in the lumbar region, sweating. Objectively:
height- 170 cm, weight -120 kg, moon-shaped face, crimson-
red, excessive growth of facial hair, acne, excessive deposition
of fatty tissue in the shoulder, chest, abdomen, striae of red-
violet color, atrophy of the muscles of the arms and legs. BPP
190/100 mm Hg. Explain, what caused the development of
obesity syndrome?

OTBETbi(oAMH OTBeT)
1 hypercortisolism
2 hyperprolactinemia

3 hyperparathyroidism

4 hyperglycemia

5 hyperthyroidism

1
37 38 39 40 41
S0 42 43 44 45
49% 11:59 am
TeCTWpOBaHMX
Be6
avn.kgma.kg/webtes
CA CHHrxKynAMn
Kg Ru

Bonpoc: Ne42
hospital with
Patient A., 40-year-old was admitted to the
in
complaints of chills, weakness, difficulty in swallowing, pain
the neck with irradiation to the ears, head, aggravated when
turning, body temperature 38 C, sweating. On palpation, the
thyroid gland is enlarged, painful, not soldered to the
Surrounding tissues, compacted, there are no fluctuating
areas. Submandibular, cervical lymph nodes are not enlarged.
Inthe analysis of blood: ESR up to 21 mm/h, relative
lymphocytosis. In the study of the accumulation of iodine by
the thyroid gland on the scanned areas of enlightenment, the
accumulation is reduced. What is your preliminary
syndrome?

OTBeTbI(oAMH OTBeT)
syndrome of hypofunction of the
parathyroid glands
adrenal hypofunction syndrome

3 Syndrome of hypothyroidism
4 thyroid nodule
syndrome
5 thyroid inflammation
syndrome

38 39 40
50
41
42 43 44 45 46
49% 11:57 am
TeCTMpoBaH X
Be6
C avn.kgma.kg/webtes O
Kg Ru
CHrxKyngwn

Bonpoc: N°9
Damage to the neurohypophysis is accompanied by impaired secretion:

OTBeTbi(oqMH OTBeT)

1 Vasopressin

2 Thyroid stimulating hormone

3 Adrenocorticotropic hormone

4 Prolactin

5 Growth hormone

5 6 78 10 11 12 13 50

1 2 34 5 6 7
B
9 10
11 12 13 14 15 16 17 18 19 20
21 22 23 24 25 26 27 28 29 30
31 32 33 34 35 36 37 38 39 40
41 42 43 44 45 46 47 48 49 50
0TBeieHHbl Bonpoc
nponiyujeHHM
BOnpoc
Tekyuwn Bonpoc

AO 3a8epuieHwA
TeCTa oCTaneai
49% -11:59 am
l
CHHrx KynaMn Kg Ru

Bonpoc: N947

Patient M., 40 years old, a salesman, applied to the doctor of


weakness, rapid
policlinic with complaints of general
fatigability, sensations of pressure in the
front ofthe neck, loss
memory, facial swelling, constipation. From the anamnesis:
suffering from rheumatoid arthritis for many years.
Objectively: Body temperature 36.1 C. The skin is pale, dry,
peeling is expressed on the legs. Swelling of face, dense
edema on the legs, lower third of the lower leg. Pulse 55 per
minute, BP-126/80 mm Hg. Tongue clean, teeth marks on
tongue. The thyroid gland is unevenly enlarged on
examination, on palpation it is dense, mobile when swallowing,
painful, nodular formations are not determined. Uitrasound of
thethyroid gland: V total. -6.3 cm3 (8.3-18.0). Increased
echogenicity. The structure is clearly diffusely
heterogeneous.
What laboratory test proves the presence of an
autoimmune
syndrome?

OTBeTbi(oqWH OTBeT)
1 T3 fr-3.0 pmol/I
2 AB-TPO-250U/ml
3 AB TG 9U/ml
4
TA fr-28 pmol/1

5 TSH-0.3 mu/1

43 44 45 46
47 48 49 50
am
49%11:57
TeCTHpOBaHMX
Be6 avn.kgma.kg/webtes
CA CHHrx KynAMn
Kg Ru

Bonpoc: N°17

A17-year-old girl considers herself for 6


ill months.
area and
Complaints about the growth of facial hair in the chin
on the upper lip, coarsening of the voice, absence of
menstruation. An objective examination revealed: male-type
hair growth, the mammary glands are not developed, the
external genital organs are underdeveloped. Which of the
following preliminary diagnoses is most likely:

OTBETbI(oAMH OTBeT)

1 Hyperaldosteronism

2 Shereshevsky-Turner
syndrome
3 Ovarian hyperfunction
4 Hypogonadism
5 Ovarian hypofunction

13 14 15
50
16
17 18 19 20 21
49%49%11:557
Be6TeCTWpOBaHMX
C avn.kgma.kg/webtes
CWHrx Kynaun

Bonpoc: Ne11
Conn's syndrome (primary aldosteronism) is manifested by

OTBeTbi(oAuH OTBeT)
1 0liguria
2 Na loss and K delay

3 Accumulation of H ions
4
Naretention and loss of K
5 Hypotension

7 89 10 11 12 13 14 15
50

12
11 12
34 567 8 9 10
13 14 15 16 17
18
2122 23 24 25 26 27 28 19 20
31 32 33 34 35 36 29 30
37 38 39 40
41 42 43 44 45
46 47 48 49 50
OTBeyeHHblM
BOnpoc
nponyujeHHbi
BoIpoc
eRv
49%11:58
Be6TecTWpOBaHX
C
avn.kgma.kg/webtes
CHHrx Kynawn Kg

Bonpoc: N°27
What disorders lead to the development of hyperglycemia syndrome?

OTBeTbI(oAMH OTBer)

1 intestinal digestion
2 absorption of dietary fats

3 protein-synthesizing function of the liver

4 deficiency of insulin secretion

5 assimilation of fats

1 a 23 24 25 26 27 28 29 30 31
50

123 4567 8 9 10
11 12 13 14 15 16 17 18 19 20
2122 23 24 2526 27 28 29 30
3132 33 34 35 36 37 38 39 40
41 42 43 A4 45 46 47 48 49 50
OTBeeHHbi Bonpoc
iponyugeHHbIM
BOnpoc
N

en

Q
N

C
pm
49%12:00
CHHrx Kynaun Kg Ru

Bonpoc: N°50
admitted to the endocrinology
Patient S., 39 years old, was complaints of weakness,
hospital with
department of a clinical
occipital region when the weather changes,
a headache in the
pressure up to 190/100 mm Hg. pain in
anincrease in blood
physical exertion,
the heart, in the lumbar spine during
menstrual irregularities. Objectively: "moon face,
cheek
-
rubeosis, height 165 cm, weight 112 kg, waist
circumference 120 cm, thigh circumference 100 cm,
numerous small subcutaneous hemorrhages on the skin of
the arms, neck, abdomen, legs, abdominal skin streaks of
purple-red stretch. What is the most informative examination
method for making a diagnosis?

OTBeTbI(oAWH OTBeT)

1 Ultrasound examination

2 Electrocardiogram

3 Computedtomography

4 Echocardiography

5 Radiography

43 44 A5 46 47 48 49 50

2345617
1 21314 15 16 12 69 10
1h 19 20
212223 2A 25
26272 20 30
3132 3334 35 36-37
36 39 40
49% 11:50 am
ll
TeCTWpOBaHM
Beó avn.kgma.kg/webtes
O CA CHurx Kynpm

Bonpoc: N°28
family doctor
A her mother turned to the
girl, 9 years old with
memory and
with complaints of constipation, decreased
appeared
academic performance, overweight, drowsiness that
over the past six months. Anamnesis: grew and developed
satisfactorily. The family uses little seafood. Objectively:
Hight-127,0 cm (N-135cm), weight-46cm (N-42 cm). Thyroid
gland 1st degree, soft. Heart tones are clear and rhythmic.
Survey: General blood analysis: erythrocytes #,7x1012/1, Hb
107g/1,leukocytes 4,8x10 9/1. Hormones: T4 -norm, TSH
norm. ECG- no pathology. Ultrasonography: the total
volume of
the thyroid gland 16 cm3. Summarize all the data and decide
what pathology the patient has?

OTBeTbi(opMH OTBeT)
vegetative-vascular dystonia

2 hypothyrolidism

3 iodine deficiency state

neurasthenia
deficiencyanemia

24 25 26 27 28 29 30 31 32
A49%11:58 am

TeCTMpOBaHneX
Be6
CA avn.kgma.kg/webtes 0
CHHrx Kyngun Kg Ru

Bonpoc: N°23
Determine, at what age you need to identify the first signs of
hypothyroidism

OTBeTbI(oaMH OTBeT)
1
4-6years
2 6-9month
3 1-3month
4 1-3 years
5 years
7-9

1 19 20 21 22 23 24 25 26
26 27
50

1 23 4567 8 9 10
11 12 13 14 15 16.17 18 19 20
21 22 2 24 25 26 27 28 29
30
31 32 33 34 35 36 37
38 39 400
41 42 43 44 45 46
47 48 49 50
OTBeHeHHbsl
Bonpoc
nponyujeHHbi
Bonpoc
texyiunn Bonpoc

Ao 3aBepuieHwa
TeCTa oCTanocb

0:57:00
amm
49%11:57

Be6TeCTWpOBaHM
X

C avn.kgma.kg/webtes
CHHrx KynAMn
Kg Ru

Bonpoc: N°10
Transhypophyseal regulation is the basis for

OTBETbI(oqMH OTBeT)
1 Parathyroidglands

2 Gonads
3 Pancreas
4 Adrenalcortex

5 Thyroid gland

67 78 9 10 11 12 13 14
50

23 4 67
11 12 13 14 15 16
8 9 10
17 18 19 20
21 22 23 24 25 26 27
28 29 30
31 32 33 34 35
36 37 38 39
41 42 43 44 45 40
46 47 48 49 550
oTBEHeHHbl
BOnpoc
nponyujeHHbI
BOnpoc
TekyuM B
N

888

(D

5
49%m11:58
9
TeCTMpOBaHMX
Be6
A avn.kgma.kg/webtes
C
CHHrx Kynaun Kg

Bonpoc: N°26
hyperthyroidism:
children -leading symptom of
Explain,what causes weight loss in

OTBeTbi(oAMH OTBeT)

1 polyuria

disorder of carbohydrate metabolism

3 functional liver failure

4 intestinal absorption is impaired

5 frequent loose stools

22 23
2425 26 27 28
28 29 30
50

12 3 4 56
11 12 13 14
7
8 9 10
15 16 17 18 19 20
21 23 25 26 27
22 2A 28 29 30
31 32 33 34 35 36 37 38 39 40
41 42 43 44 45 46 47 48 49 50
OTBeHeHHbI
BOnpoc
nporyugewHbi0 Bonpoc
Teryunn Bonpoc

Ao 3aBepuienua
TeCTa OCTanocb
49%=11:59 am
CHHrx Kyngun Kg Ru

Bonpoc: N°47

years old, a salesman, applied


to the doctor of
Patient M., 40
weakness, rapid
policlinic with complaints of general
fatigability, sensations of pressure the
in front of the neck, loss
memory, facial swelling, constipation. From the anamnesis:
years.
suffering from rheumatoid arthritis for many
dry,
Objectively: Body temperature 36.1 C. The skin is pale,
peeling is expressed on the legs. Swelling of face, dense
edema on the legs, lower third of the lower leg. Pulse 55 per
minute, BP-126/80 mm Hg. Tongue clean, teeth marks on
tongue. The thyroid gland is unevenly enlarged on
examination, on palpation it is dense, mobile when swallowing,
painful, nodular formations are not determined. Ultrasound of
thethyroid gland: V total. 6.3 cm3 (8.3-18.0). Increased
echogenicity. The structure is clearly diffusely heterogeneous.
What laboratory test proves the presence of an
autoimmune
syndrome?

OTBETbI(oAMH OTBeT)
1 13 fr- 3.0 pmol/

2 AB-TPO-250 U/ml

3 AB-TG-9U/ml
4
T4 fr-28 pmol/1
5 TSH-0.3 mU/1

43 44 45 46 47 48 49 50
49%11:58 am

Be6TeCTWpOBaHMX
C A avn.kgma.kg/webtes
CHHrx KynAHn Kg Ru

Bonpoc: NP29
Which microorganism is the main pathogen in acute thyroiditis?

OTBETbi(oAMH OTBeT)

1 mushrooms
2 rickettsia

3 staphylococcus

4 protozoa
5 viruses

25 26 27 28 29 30 31 32 33
50

1112 3 4567 8 9 10
12 13 14 15 16 17 18 19 20
21 22 23 24 25 26 27 28
29 30

31 32 33 34 35 36 37 38 39 40
41 42 43 44 45 46 47
48 49 50
OTBeHeHHbA
Bonpoc
nporiyujeHHbl
Bonpoc
Tekyuni BOnpoc

Ao 3aBepuieHwa
TeCTa oCTanocb

0:56
Q

N
49%=11:58 am
l
TeCTWpOBaHe X

Be6 avn.kgma.kg/webtes
CC Kg Ru
CWHrx Kynann

Bonpoc: N°32
(venous plasma):
following corresponds to impaired glucose tolerance
Which of the

OTBeTbI(OAMH OTBeT)

1 fasting glucose 10.0 mmol/1, 2 hours after OGTT 16.8 mmol/1

2 fasting glucose 6.8 mmol/1,2 hours after OGTT 10.9 mmol/1

3 fasting glucose-9.6 mmol/1,2 hours after 0GTT-14.4 mmol/1

fasting glucose 5.4 mmol/I,2 hours after OGTT 9.2 mmol/1

5 fasting glucose 8.9 mmol/1,2 hours after OGTT 12.7 mmol //

50
28 29 30 31
32 33 34 35 36

2 12 313 456789 10
11 14 15 16 17 18 19 20
2122 232A 25 26 27 28 29 30
33 34 35 36 37 38 39 40
41 42 43 A4 45 46 47 48 49 50
OTBeYeHHLIN
BOnpog
nponyuaesHtln
6onpoc
Texyuiw Bonpoc
am
49% 11:57
TeCTWpOBaHMEX
Be6
C
avn.kgma.kg/webtes O
Kg Ru
CHHrx KynAMn

Bonpoc: Ne16
flow of urine
Patient A., 39 years old, complains of copious
(daily urine output is 22 liters per day). Blood sugar is
within
thenormal range. With a violation of the production of which
hormone polyuria is associated:

OTBeTbI(oAMH OTBET)

1 insulina

2 Parathyroid hormone

3 ADH Vasopressin

4 Glucagon

5 Aldosterone

12 13 14
50
15
16 17 18 19 20

12
34567 8

1 121314 151 17 18 9 10
21 22 23 24 25 26 19 20
27 28 29 30
31 32 33
34 35 36 37 38
39 An
am
A 49%11:57
TeCTWpOBaHWE X

Be6
avn.kgma.kg/webtes
C
CHHrx Kyngun Kg Ru

Bonpoc:N°18
drugs used to stimulate the contractile activity of
the
On the basis of which pituitary hormone are
uterus

OTBETbI(OAMH OTBeT)

1 Thyroid stimulating hormone

2 Follicle-stimulating hormone

3 Vasopressin

4 Oxytocin
5 Prolactin

14 15 16 17 18 19 20 21 22
50

23 45 6789 10
11 12 13 1415 16 17/18 19 20
21 22 23 24 25 26 27 28
29 300
31 32 33 34 35 36 37 38
39 40
41 42 43 44 45 46 47
48 49 50
-OTBeueHHblW
BonpoC
nponyueHHbl Bonpoc
Texyuina BOnpoc
am
49%11:58
TecTMpOBaHM X

Be6
avn.kgma.kg/webtes
C A
Kg Ru
CHHrx Kyngun

Bonpoc: NP21

complains of nausea, headache,


increased
A 25-year-old man
polyuria. Gastroscopy revealed no gastrointestinal
thirst and
polyuria
pathology. The condition worsened thirst and
-

8 liters, a
increased, the amount of liquid drunk increased to
constant headache accompanied by vomiting, there was
a

loss of lateral visual fields. Urine analysis: a decrease in the


specific gravity in the morning portion of urine to 1002, blood
osmolality-315 mosm/kg, urine osmolality 270 mosm/ kg.
Fasting blood sugar - 3.2 mmol/I. Which ofthe following
methods is priority:

OTBETbI(oAMH OTBeT)

1 Determination of blood sugar during the day


2 Excretory urography

3 Fluid restrictiontest
4 MRI of the brain

5 ultrasound of the kidneys

17 18 19 20 21 22 23 24 25
50
l 49%11:5

Be6TeCTWpOBaHX
C
avn.kgma.kg/webtes
CWHrx Kyngun Kg

Bonpoc: N°35
(capillary plasma):
Which of the following corresponds to impaired fasting glucose

OTBeTbI(oaMH OTBeT)

1 fasting glucose 5.9 mmol /,2 hours after OGTT 6.8 mmol/I

2 fasting glucose 9.5 mmol/ 1, 2 hours after OGTT 12.8 mmol/I

fasting glucose 6.8 mmol/1, 2 hours after OGTT-9.5 mmol/1

fasting glucose 8.6 mmol/1,2 hours after OGTT 11.4 mmol/I

5 fasting glucose 7.9 mmol / 1,2 hours afterOGTT-10.7 mmol/I

31 32 33 34 35 36 37 38 39
50

2 3 4 5 6 7 8 9 10
11 12 13 14 15 16 17 18 19 20
21 22 23 24 25 26 27 28 29 30
31 32 33 34 3 36 37 38 39 40
41 42 43 44 45 46 47 48 49 50
OTBeMeHHbl
Bonpoc
nponyugeHHbI
BOnpoc
Texyumn uonpoc
49%1
TeCTWpOBaHM X
Be6
C avn.kgma.kg/webtes
CHHx Kyngun

Bonpoc: N°38
What complication does hyperthyroidism give:

OTBeTbi(oAuH OTBeT)
1
addison's disease
2 atrial fibrillation

3 biliary dyskinesia

4 B12 deficiency anemia

5 atrophicgastritis

34 35 36 37 38 39 40 41 42
50

1212 3 45 678 9 10
11 13 14 15 16 17 18 19
20
21 22 23 2425 26 27 28 29
30
31 32 3334 35 36 37 38 39 40
41 42 43 44 45 46 47 48
49 50
OTBeHeHHbl
BOnpoc
nponyuzeHHbi BOnpoc
TeKyunn BonpocC
am
49%11:58
TeCTWpOBaHWe X
Beó
O CA avn.kgma.kg/webtes o
CHHTx Kynan Kg Ru

Bonpoc: N 20

Female, 52 years old, onset of the disease 1.5 years ago after
a course of radiation therapy. Complains of memory
impairment, general weakness, drowsiness, baldness, lack of
appetite, weight gain. An objective examination revealed: the
skin is dry, flaky, the face is puffy, hypomimic, there are areas
of baldness on the head, the voice is low, rough, speech is
slowed down, the body temperature is 35.7, the pulse is 52
beats. per minute, blood pressure 110/65 mm Hg. Art., the
levelof glucose in the blood 3.5 mmol/1, the basal
-

metabolism is reduced by 30%. Diagnosed with


Myxedema.
What is the most likely cause of this disease:

OTBETbI(oAMH OTBeT)
1 lodine deficiency
2 Decreased TSH levels
3 Formation of reverse
T3
Formation of
mucin
5 Increase in T3 T4
l 49% 11:59 am

TeCTWpOBaHWX
Be6 O
avn.kgma.kg/webtes
CA CwHrx KynAn Kg Ru

Bonpoc: N°43
with complaints of
Patient K., 48 years old, went to the clinic
severe weakness, weight loss, bone pain, especially in the feet,
long-term healing fractures, loss of appetite, nausea, vomiting
not associated with food intake, diarrhea. From the
anamnesis: suffers from adenoma of the right parathyroid
gland. Laboratory data: TBC-anemia, Calcium-3.4 mmol /1,
phosphates-0.7 mmol /1. What is your preliminary syndrome?

OTBeTbi(oguH OTBeT)
1 thyroid inflammation syndrome

2 hypercortisol syndrome

3 hyperthyroidism syndrome

4 hyperparathyroidism syndrome

5 hyperglycemia syndrome

39 40 41 42 43 44 45 46
50
47

2 3 45670 9.10
1 12 13 14 15
16 12 18 19 20
49%11:59
Be6TeCTWpOBaHMX
avn.kgma.kg/webtes
CA Kg
CHHrx Kyngun

Bonpoc: N 39
with thyroid
A white sclera line between the iris and the upper eyelid when looking down
hyperfunction syndrome is a symptom

OTBeTbI(oAMH OTBeT)

1 Kocher
2 Shtelvag

3 Grefe
4 Elinek
5 Krause

35 36 37 38 39 40 41 42 43 #
50

211 3 4567 8 9 10
12 13 14 15 16 17 18 19 20
21 22 23 24 25
26 27 28 29 30
31 32 33 34 35 36 37 38 39 40
41 42 43 44 45 46 47
48 49 50
TEe4eHML
49%1
Be6TecTMpOBaHnx
C A avn.kgma.kg/webtes
CHHrx Kynaun

Bonpoc: Ne13
Dysfunction of the kidneys in thyrotoxicosis may manifest itself

OTBeTbI(oquH OTBeT)
1 Proteniuria
2 Impaired renal blood flow

3 Impaired reabsorption of Ca and phosphorus

4 Concentration dysfunction

5 Increasing the filtration capacity of the kidneys

9 10 11 12 13 14 15 16 17
50

212 3 4
5 6 78 9 10
11
14 15 16
2122 23 24 25 26
17 18 19 20
27 28 29 30
31 32 33 34 35
36 37 38 39 40
41 42 43 44 45 46
47 48 49 50
OTBeeHHbIM
BOnpoc
nporyujenHbi
Bonpoc
Tekyunn Boripoc
am
49%11:59
CHHTx Kynawn Kg Ru

Bonpoc: N946

17-year-old boy was admitted to the clinic


in an unconscious
A
training, the
state. According to the trainer, he felt good before
childhood; during
boy has been suffering from diabetes since
training, he suddenly lost consciousness. Objectively: The skin
is moist. Breathing is vesicular. Heart sounds are preserved.
BP120/90 mm Hg. Laboratory glucose-2.7 mmol /1. The
attending physician diagnosed him with hypoglycemia,
prescribed a glucose solution, and the patient regained
consciousness. Explain the cause of hypoglycemia in this
case?

OTBeTbI(oAMH OTBeT)
1 metformin overdose

2 physical activity

3 drink alcohol

untimely meal

insulin overdose

1 42 A3 44 45 46 17 48 49 50

234
112 13 18 15 16 17 1t 19 20
2122 232a 25 26 27
28 29 30
21 42334 5 30 37 38 39
40
E 34aina 47 48 49 50
OTBeHeAR
9
l 49%
1
TeCTHpoBaHn x
Be6
C A avn.kgma.kg/webtes
CHHrx Kyngun

Bonpoc: N°25
Indicate the main symptom in hypoglycemia syndrome

OTBeTbi(oAMH OTBeT)
1 hungery

2 polydipsia
3 loss of weight

4 nocturia
5 polyuria

21 22
50
23
2425 26 27 28 29

1212 3 456 7 89 10
11 13 14 15 16 17
18 19 20
1 22 232425 26 2728
31 32 33 34 35 29 30
36 37 38 39 40
41 42 43 44 45
46 47 48 49 50
OTBeHeHHbl
nponyue BOnpoc
HHE
am
49%11:57
TeCTWpOBaHMX
Be6
avn.kgma.kg/webtes
C A
Kg Ru
CHHrx Kyngun

Bonpoc: N°12
plays a role:
Inthe pathogenesis of acute thyroiditis

OTBeTbI(oAMH OTBeT)

1 Injury of the thyroid gland

22 lodinedeficiency

3 Penetration of infection into the thyroid gland

4 Genetic predisposition

5 Violation of the mechanisms of immunological


protection

9
50
8 10 11
1213 14 15 16

2 3 45 67 8 9 10
1112 13 14 15 16 17 18 19 200
21 22 23 24 25 26 27 28
29
31 32 33 34 35 36 37 38 39 30
41 42 43 A4 45 A6 40
47 48 49 50
OTBe4eHHbi
Bonpoc
nponyueHHbi
Bonpoc
Tekyuu
BOnDOC
am
49%11:58
TeCTWpOBaHMX
Be6
avn.kgma.kg/webtes
CA CHHrx KynAMn
Kg Ru

Bonpoc: N°19

45 years
N., old, complains of weakness, rapid fatigue,
Patient
Objective
lackof appetite, weight loss, pain in the abdomen.
examination: the skin and visible mucous membranes of
bronze color, blood pressure is reduced. Dysfunction of which
endocrine gland can cause the following symptoms:

OTBETbi(oAMH OTBeT)

1 Adrenal glands

2 Ovary

3 Epiphysis

4 Pituitary gland

5 Pancreas

15 16 17 18 19 20 21 22 23
50

B9.10
20
am
TA4911:59
BeóTeCTHpoBaHX 0
A
avn.kgma.kg/webtes
C
O
Bonpoc: Ne43
with complaints of
Patient K, 48 years old, went to the clinic
severe weakness, weight loss, bone pain, especially in the feet
long-term healing fractures, loss of appetite, nausea, vomiting
notassociated with food intake, diarthea. From the
anamnesis: suffers from adenoma of the right parathyroid
gland. Laboratory data: TBC-anemia, Calcium-3.4 mmol/L
phosphates-0.7 mmol/1. What is your preliminary syndrome?

OTBeTbI(oAMH OTBeT)

1 thyroid inflammation syndrome

2 hypercortisol syndrome

3 hyperthyroidism syndrome

4 hyperparathyroidism syndrome

5 hyperglycemia syndrome

39 A0
40 41
41
4243 44 5 46 47
50

22456709 10
1137 131A 15 1617 18
19 20
amn
49%11:58
TeCTWpOBaHX
Be6
CA
avn.kgma.kg/webtes O
O CHHrx Kyngun Kg Ru

Bonpoc: N°22

A ambulance team
45-year-old man, taken to the clinic by an
with adiagnosis of "Hypertensive crisis". BP- 180/120 mm
Hg.Art.,myocardial hypertrophy, tachycardia, weakening of
memory and intelligence, blood glucose 6.5 mmol /I. A
history of sudden crises with a rise in blood pressure up to
200/140 mm Hg. Art., the appearance of tachycardia,
Sweating, sharp agitation. Nuclear magnetic resonance
imaging of the lumbar region revealed an increase in the size
of the left adrenal gland, the presence of a rounded formation
inthe medulla of the gland. Which of the following diagnoses
is most likely:

OTBeTbI(oAMH OTBeT)
1 Pheochromocytoma

2 Adrenalcrisis
3 Arterial hypertension
4 Addison's disease

Diabetes mellitus

18 19 20 21
10:56 EIE .ll 53%

Bonpoc: N931
Pigmentation of the eyelids (upper) with thyroid hyperfunction
syndrome is a symptom:

OTBETbI(oAMH OTBeT)

1 Raceman

2 Kocher

3 Elinek
Moebius

Krause

27 28 29 30 31 3
33 3435 50

1 23 4 567 8
11 12 13 14 15 16 17 18 19 20
21 22 23 24 25 26 27 28 29 30
31 32 33 34 35 36 37 38 39 40
41 42 43 44 45 46 47 48 49 50

OTBeyeHHblM BOnpoc
nponyyeHHblM BOrIpoc
TekyunA BOnpoc

Ao saBepuieHna TeCTa oCTanocb

0:20:03
G 31

K
O
Scanned with CamScanner
P
10:54 G l 53%

A avn.kgma.kg/webtest/1
MIEER

AxacaH Myxammaa Hagum Kg Ru

Bonpoc: N932
Indicate, what change the cardiovascular system is
in
characteristic of the syndrome of hypercortisolism?

OTBETbl(oAMH OTBeT)

pulse soft, low tension


-

2 weakening of heart sounds

3 increase blood pressure

4 apical impulse is weakened, low

5 displacement of the right border

28 29 30 31 32 33

34 35 36 50L

1 23 4 5 6 7 8
11 12 13 14 16 17 18 192
15
21 22 23 24 25 26 27 28
31 32 33 34 35 36 3738 39 40
41 42 43 44 4647 48 49 50
45 .
OTBeyEHHbi Boripoc
nponyyeHHbIM BOnpoc
TeKyunn Bonpoc

O3aBEPIeHMA TeATa0CTanoCb

Scanned with CamScanner


O
N

LOLO

Scanned with CamScanner


10:48 i I E l 53%
AxacaH MyxaMMaA HaquM

Bonpoc: N934
Thyroid-stimulating hormone is normal (mU/1)?

OTBeTbi(OAMH OTBET)

from 2.1 to 7.5

2 from 1.4 to 6.1

from 1.0to 5.0

4 from 0.4to 4.0


5 from 0.1to 3.0

30 31 32 33
34 35
36 37 38 50

1 2 3 4 5 6 7 8 9 10
11 12 13 14 15 16 17 18 19 20
21 22 23 24 25 26 27 28
29 30
31 32 33 34 35 36 37 38 39 40
41 42 43 44 45 46 47 48 49 500
OTBeyeHHLIM
BONpoc
nponyuyeHHbIM
BOnpoc
TeKyunM BonpoC

o 3aBepweHnA TECTa oCTanocb


Scanned with CamScanner
A avn.kgma.kg/webtest/1
AxacaH MyxaMmaa HaguM Kg Ru

Bonpoc: Ne35
Normally, the volume of the thyroid gland (according to
ultrasound) in men is (ml):

OTBETbI(oAWH OTBeT)

from 45 to 48

2 from 38 to 40

3 more than55

4 from 30 to 35

5 Up to 25

K
31 36
37 38 39 50

1 2 3 4 5
11 12 13 14 15
16 17 18 19 20
21 22 23 24
25 26 27 28 29 30
31 32 33 34 3536 8738 39 40
41 42 43 44 45 46 47 48 49 50
OTBeyeHHbIM
Boipoc
nponyeHHbI BOnpoc

G 35

Scanned with CamScanner


10:43 X ll 54%
AxacaH MyxaMmaA Hagum Kg Ru

Bonpoc: Ne36
A white sclera line between the iris and upper
eyelid when looking up with thyroid hyperfunction
syndrome is a symptom:

OTBETbI(oquH OTBeT
1
Raceman

2 Shtelvag

3 Elinek

4 Grefe
5 Kocher

32 33 34 35 36 37

38 39 40 50

1 2 3 4 5 6 7 89 10
11 12 13 14 15 16 17 18 19 20
21 22 23 24 25 26 27 28 29 30
3132 33 34 35 36 37 38 39 40
41 42 43 44 45 46 47 18 49 50 A
OTBeyeHHblM BONpoc
nponyujeHHblM BOipoc
TeKyuun Bonpoc

Ao 3aBepweHwA TeCTa oCTanocb

Scanned with CamScanner


10:39 n il 54%

avn.kgma.kg/webtest/1

Bonpoc: N937
Free thyroxine is normal (pmol /1)?

OTBETbI(oqMH OTBeT)

1 from 10.0to 26.0

2 from 6.0 to 15.0

from 7.0 to 20.0

4 from 9.0 to 22.0

5 from 12.0 to 29.0

38
33 34 35 36 37
A1 50
39 40

2 3
17 18 19 20
11 12 131415 16
27 28 29 30
21 22 23 24 25 26
33 34 35 36 37 38 39 40
31 32
43 44 45
4647 48 49 50
41 42
OTBeYeHHblM BOnpoc .

nponyuEHHbI Bonpoc
Texyuyun Bonpoc
ocTanocb
3aBepuieHna TecTa
Ao
0:36:34
u
AnllUA
Scanned with CamScanner
10:37 Ei Ei E il 54%

Bonpoc: N938
A white sclera line between the iris and the upper
eyelid when looking down with thyroid
hyperfunction syndrome is a symptom:

OTBeTbi(oAMH OTBeT)

1 Kocher

2 Grefe

Shtelvag

4 Elinek

5 Krause

1
34 35 36 37 38 39

40 41 42 50

1 2 3 4 5 6 7 8 9 10
11 12 13 14 15 16 17 18 19 20
21 22 23 24 25 26 27 28 29 30
31 32 33 34 35 36 37 38 39 40
41 42 43 44 45 46 47 48 49 50
OTBeyeHHblM BOnpoc
nponyuyeHHblM BOnpoc
Tekyuwn Bonpoc

o 3aBepuweHuA TeCTa oCTanocb

0:38:35

Scanned with CamScanner


10:34 PG * TE ll 55
AxacaH Myxammag HaaMM Kg Ru

Bonpoc: Ne39
Characteristics of the pulse on the radial artery in hypocorticism
syndrome

OTBETbI(oAMH OTBET)

1
tense

2 arrhythmic

3 high

4 soft

5 resistant

35 36 37 38 39 40

41 42 43 50

1 2 3 45 6 7 89
10
11 12 13 14 15 16 17 18 19 20
21 22 23 24 25 26 27 28 29 30
31 32 33 34 35 36 37 38 39 40
41 42 43 44 45 46 47 48 49 50

OTBeeHHblM BOnpoc
nponyueHHblM Bonpoc
TEKyyn BONpOc

o saBepueHna TeCTa oCTanocb:

G 39

Scanned with CamScanner


N

(D
(D

(D

8 D

Scanned with CamScanner


N

O
(D

(D

Scanned with CamScanner


N

10

Scanned with CamScanner


10:22 G ll 56%

A avn.kgma.kg/webtest/1

Bonpoc: N943
Patient A. 34-year-old, applied to a polyclinic with
complaints of an increase in body temperature up
to 39 C, pain in theright side of the thyroid gland,
aggravated by swallowing, coughing, chills,
tachycardia, weakness. Anamnesis: frequent
tonsillitis. Objectively: On palpation, the thyroid
gland is sharply painful, dense, regional lymph
nodes are enlarged Laboratory: Total blood test:
leukocytosiswith a shift to the left, increased ESR.
What is your most likely syndrome?

OTBeTbi(OAMH OTBET

Syndrome of acute inflammation of thethyroid gland

2 syndrome of hypofunction ofthe parathyroid glands

3 syndrome of chronic inflammation of the thyroid gland

4 thyroid nodule syndrome

5 syndrome of hypothyroidism

1 39 40 41 43 44

45 46 47 50

Scanned with CamScanner


10:20 r l 56%
AxacaH MyxaMMaA HapuM Kg Ru
Bonpoc: N944
A 43-year-old man, a truck driver, was admitted to
the clinic with complaints of an increase in blood
pressure up to 190/100 mm Hg, accompanied by
headaches, dizziness, tinnitus. Parents have
obesity, diabetes mellitus, arterial hypertension.
On examination: abdominal obesity, body weight-
110 kg, height 160 cm, Broca's index 60 kg.
-

Waist circumference 130 cm, hip circumference


110 cm. Palpation: the liver is painless, even, not
enlarged. Auscultation: accent I tone above the
aorta, BP 180/95 mm Hg. Laboratory: fasting
venous blood glucose -4.5 mmol/1,2 hours after
eating-7.1 mmol/1. What is the most typical
syndrome of a patient that has developed as a
result of obesity?

OTBETbI(oguH OTBeT)
1 gynecomastia

2 hypercortisolism

3 hepatomegaly

4 hyperglycemia

5 hypertenslon

1 40 41 42 43 44 45

O
Scanned with CamScanner
10:17 ll 56%

OA avn.kgma.kg/webtest/1 0
Bonpoc: N945
The man was admitted to the clinic, complaints of
palpitations, interruptions in the work of the heart,
sweating, trembling in the body, weakness.
Anamnesis: has been ill for several years.
Examination: the apical impulse is enhanced.
1
Heart sounds are loud, arrhythmic. Pulse 98 in
minute. blood pressure 170/60 mm Hg. ECG:
tachysystolic form of atrial fibrillation. Your
suspected syndrome?

OTBeTbi(oAMH OTBeT)

1 hyperparathyroidism

2 hyperglobinemia

3 hyperglycemia

4 hyperthyroidism

5 hypercorticism

**
41 42 43 44 46

47 48 49 50 .

G 45
Scanned with CamScanner
1015
AXacaH MyxaMMan HaMM

Bonpoc: N946
.A 29-year-old female patient turned to a
polyclinic therapist with complalnts of headaches,
weight gain, weakness, palpitations, menstrual
irregularities. The above complaints are disturbing
for 6 months after the head injury. On
examination: moon face, excessive facial hair
growth, acne. Weight-98 kg, height 162 cm. Find
-

out from the description the objective symptom of


this patient

OTBeTbI(oAMH OTBeT)
1 diastemna

2 fish mouth

exophthalmos

4 macrognathia

5 hirsutism

42 45 46 47

48 49 50

1
2 3 4 5 6
11 12 13 14 15 16 17 18 19 20
21 22 23 24 25 26 27 28 29 30
31 32 33 34 35 36 37 38 3940
41 42 43 44 45 46 47 48 49 50

Scanned with CamScanner


L il 56%
10:12 V
AxacaH MyxamMag Hapum Kg Ru

Bonpoc: N947
A 16-year-old boy was admitted to the clinic in an
unconscious state according to the mother,
previously complained of severe thirst, frequent
urination and general weakness. Objectively: the
smell of acetone from the mouth, the skin is dry.
Rapid breathing. Heart sounds are muffled. The
abdomen is tense. What is the most informative
laboratory examination method (blood) for
making a diagnosis?

OTBETbI(opMH OTBeT)

1 proteim

2 calcium

3 cholestetol

4 cretinine

5 glucose

43 44 45 46 47 48

49 50

1 2 3 4 5 6 7 89 10
11 12 13 14 15 16 1718.19.20

G 47

O
Scanned with CamScanner
.l 56%
LTE1

A avn.kgma.kg/webtest/1
AxacaH MyxaMmaA HaauM Kg Ru
Bonpoc: N°48
Patient G., 42 years old, after a traumatic injury too
the neck, notes the periodic appearance of
paresthesias (pathological unpleasant sensations,
for example, "running creeps") with the
subsequent development of seizures. Before the
development of seizures, he has nausea, vomiting.
Convulsions are tonic in nature, beginning in the
upper limbs and face. Consciousness during this
period is preserved. Examination of the patient
revealed that the content of calcium in the blood
is 0.6 mmol/ I, phosphorus 8.0 mmol/ 1. Which
hormone is deficient in this situation?

OTBeTbi(oAMH OTBET)
1 thyroxine hormone

2 vasopressin hormone

3 cortisol hormone
4 somatotropin hormone

5 parathyroid hormone

1 43 44 45 46 47 48

49 50

O
Scanned with CamScanner
AAC
NO49
Bonpoc:
doctor at the
consulted a increased
old,
V., 50
years complaints of increased
Patient residence with region,Objectively:
place of
lumbar
the
weight, pain in 200/100 mm Hg. Obesityy
body pressure up to
weight 121 kg. according
blood
165 cm, bodysubcutaneous fat
height cheeks,
redistribution of the
with 'cushingoid" type, redness of abdomen,
the the hips and
to ofred-violet color in hypotrophy of
striae and back,
face
multiple acne
on the
glucose 15 mmol7,
blood
the limbs. Fasting
hormonal research method
6%. What
urine sugarmaking a diagnosis?
will help in
OTBETbl(oAMH OTBeT)

1 insulin
2 thyroxine

3 vasopressin

4 testosterone

5 cortisol

45
45 46 47 48
43 44

49 50

Scanned with CamScanner


10:07 F 9 l 56%m

Bonpoc: N°50
Patient M., 44 years old, at an appointment with a
local therapist complains of a feeling of pressure
in the front of the neck, rapid fatigue, memory
loss, drowsiness. Anamnesis: Mother was
autoimmune thyroiditis. It is known from the
anamnesis that these complaints arose 6 years
ago for no apparent reason and developed
gradually. On examination: the skin is clean, dry,
cold to the touch. The isthmus and both lobes of
the thyroid gland are palpated, more on the right
the gland is dense, moderately painful. Pulse - 53
per minute, rhythmic. BP-90/70 mm Hg. In
laboratory tests, TSH was detected: 8 -mlU/ L, T4
fr.-8.6 pmol/ ml, anti-TPO - 364 IU/ml. Total
blood count: erythrocytes-3.5x 1012/1,
leukocytes 5.8 x 109/1, hemoglobin -96g/I.
What diagnostic intervention should be used to
clarify the diagnosis?

OTBeTbI(oAMH OTBET)
1
thyroid scan with 131

2 MRI ofthe thyroid gland


3 lateral radiography ofthe neck

4 ultrasound of the thyroid gland


5 auscultation of the thyroid gland

O
Scanned with CamScanner
N
N

N N N

Scanned with CamScanner


N

Scanned with CamScanner


N

3 D

Scanned with CamScanner


SNN

Scanned with CamScanner


N

so:0

&

Scanned with CamScanner


10:05 g Yll 57%

Bonpoc: N°25
Name the hormone that regulates the function of the thyroid-
gland:

1-
OTBeTbI(oAWH OTBET)

thyroid-stimulating hormone
-

thyrocalcitonin

thyrotropin.releasinghormone

4 thyroglobulin

5 adrenocorticotropic hormone

21 22 23 24 25 26

27 28 29 50

12 3 4567 8 9 10
11 12 13 14 15 16 17 18 19 20
2122 23 24 25 26 27 28 29 30
31 32 33 34 35 36 37 38 39 40
41 42 43 44 45 46 47 48 49 50
OTBeyeHHblM Bonpoc
nponyuEHHbiÀ Bonpoc
TexyuynA BOnpocC

Scanned with CamScanner


10:05 V9 X ll 57%

Bonpoc: N°24
Indicate the hormone that regulates the secretion ofthyroid-
stimulating hormone by the pituitary gland:

OTBeTbi(oauH OTBeT)

1 thyroglobulin

2 thyrotropin releasing hormone

3 thyrocalcitonin

thyroxine

5 adrenocorticotropic hormone

20 2122 2223 24 25

28 50
26 27

2 3 4 5 7 86 9 10
1 19 20
11 12 13 14 15 1617 18
29 30
2122 23 24 25 26 27 28
39 40
31 32 33 34 35 36 37 38
41 42 43 44 45 46 47 48 49 50

OTBeYeHHbIM BONpoc
nponyueHHblM BOnpoc
TEKyuuñ Bonpoc

oCTanocb:
Ao 3aBepueHMA TeCTa

1:10:40
Scanned with CamScanner
10:05 9 il 57%m
AxacaH MyxaMMaA HaqMM

Bonpoc: N°23
Determine, which of the following- is the main thyroid hormone:

OTBeTbI(oAMH OTBeT)
1
thyrotropin releasing hormone

2 thyrocalcitonin

3 thriodthytonine

4 thyroxine

5 thyroid- stimulating hormone

1 19 20 21 22 23 24

25 26 27 50

1 2 3 4 5 6 7 89 10
11 12 13 14 15 16 17 18 19 20
21 22 23 24 25 26 27 28 29 30
31 32 33 34 35 36 37 38 39 40
41 42 43 44 45 46 47 48 49 50

OTBevEHHblM Bonpoc
nponyueHHblM BOnpoc
TEKyunn Bonpoc

Ao saBepweHnA TECTa oCTanocb:

Scanned with CamScanner


N

D
LO
D
O
D IO

Scanned with CamScanner


Bonpoc: N°21
A25-year-old man complains of nausea,
headache, increased thirst and polyuria.
Gastroscopy revealed no gastrointestinal
pathology. The condition worsened thirst and -

polyuria increased, the amount of liquid drunk


increased to 8 liters, a constant headache
accompanied by vomiting, there was a loss of
lateral visual fields. Urine analysis: a decrease in
the specific gravity in the morning portion of urine
to 1002, blood osmolality 315 mosm /kg, urine
osmolality - 270 mosm/ kg. Fasting blood sugar
3.2 mmol /1. Which of the following methods is
priority:

OTBETbl(oAMH OTBET
1 MRI of the brain

2 Determination of blood sugar during the day


3 Excretory urography

4 Fluid restriction test

5 ultrasound of the kidneys

17 18 19 20 21 22

23 24 25 50

G 21

O
Scanned with CamScanner
10:04 9 l 57%

Bonpoc: NP20
A 25-year-old woman complains of insomnia,
rapid fatigue, irritability, unreasonable anxiety,
absent-mindedness, tearfulness, excessive
sweating, poor heat tolerance, menstrual
irregularities. The patient is fussy, verbose,
pronounced bulging. The skin is warm, moist, thin.
The face is pink. The subcutaneous fat layer is
poorly developed. There is a diffuse enlargement
of the thyroid gland, tremor of the fingers. BP
130/60 mm Hg, pulse 108 per minute. Basic
metabolism increased by 40%. What disease is
the clinical picture typical for:

OTBeTbi(ogMH OTBET)

1 Autoimmune thyroiditis

2 Endemicgoiter
3 Nodular toxic goiter
4 Thyrotoxicosis

5 Subacute thyroiditis

1 16 17 18 19 20 21
22 23 24 50

1 2 3 4 5 6 7 89 10
11 12 13 14 15 16 17 18 19 20
Scanned with CamScanner
10:03 V 9 A
.l 57%
A avn.kgma.kg/webtest/1
AxacaH MyxaMMa4 HaauM Kg Ru

Bonpoc: N918
A 39-year-old patient complains of irritability,
palpitations and pain in the region of the heart,
increased appetite. On examination, attention is
drawn to bulging, muscle tremors, blood pressure
150/70. Indicatewhichglandfunctionisimpaired?

OTBETbI(oAMH OTBET)

1
Thyroid

2 Pituitary gland.

3 Parathyroid
4 Epiphysis.

5 PancreasS

14 15 16 17
18 19

20 21 22 50

1 2 3 4 5 6 7 8 9 10
11 12 13 14 15 16 17 18 19 20
21 22 23 24 25 26 27 28 29 30
31 32 3 34 35 36 37 38 29 40

G 18

Scanned with CamScanner


10:03 9
7%
OA avn.kgma.kg/webtest/1
AxacaH MyxamMaA HaguM
Kg Ru

Bonpoc: N917
After removal of the thyroid gland,
the patient developed
convulsions. What was damaged during
the operation:

OTBeTbI(oqMH OTBeT)
1 Parathyroid gland

2 Inferior laryngeal artery

3 Inferior laryngeal nerve

4 sympathetic trunk

5 Vagus nerve

1 13 14 15 16 17 18

19 20 21 50

1
2 3 45 6 7 8 9 10
11 12 13 14 15 16 17 18 19 20
21 22 23 24 25 26 27 28 29 30
31 32 33 34 35 36 37 38 39 40
41 42 43 44 45 46 47 48 49 50
OTBEYEHHblM Bonpoc
nponyeHHblM Bonpoc
TONmu naanAn

G Convulsions

K
O
Scanned with CamScanner
N
D
G
N

9 D
D

N
IO

Scanned with CamScanner


N

D
O1

Scanned with CamScanner


G NI
O

N
O

O
C

Scanned with CamScanner


10:02 9 l 57%

A avn.kgma.kg/webtest/1

AxacaH MyxaMmaa HaguM Kg Ru

Bonpoc: N913
An increase inthe concentration of thyroid-
stimulating hormone in the blood during
hypothyroidism indicates the localization of the
pathological process in:

OTBeTbi(ogMH OTBeT)

1 Hypothalamus
2 Timuse

3 Thyroid gland

4 Ptuitary gland
5 Parathyroid glands

1 10 11 12 13 14

15 16 17 50

1 2 3 4 5 6 7 8 9 10
11 12 13 14 15 16 17 18 19 20
21 22 23 24 25 26 27 28 29 30
31 32 33 34 35 36 37 38 39 40
41 42 43 44 45 46 47 48 49 50

OTBeyeHHbl Bonpoc
nponyueHHbIM BOnpoc
TeKvIn BONDOC

Scanned with CamScanner


10:02 9 .ll 57%m

OA avn.kgma.kg/webtest/1
AxacaH MyxaMmag HaquM Kg Ru

Bonpoc: N°12
The highest content of antibodies to microsomal antigen in blood
is observed when:

OTBeTbI(onuH OTBeeT)

1
Diffusetoxicgoiter

2 Autoimmune thyroiditis

3 Thyrotoxic adenoma

Thyroid cancer

5 Subacute thyroiditis

1 8 9 10 11
12 13

14 15 16 50

1 2 3 4 5 6 7 8 9 10
11 12 13 14 15 16 17 18 19 20
21 22 23 24 25 26 27 28 29 30
31 32 33 34 35 36 37 38 39 40
41 42 43 44 45 46 47 48 49 50

OTBeuEHHbIM BONpoc
nponyueHHblM BOnpoc
TeKyun Bonpoc

4o 3aBepweHuA TECTa ocTanocb

I
O
Scanned with CamScanner
10:02 9 ill 57%

O A
avn.kgma.kg/webtest/1
AxacaH Myxammaa HanuM Kg Ru

Bonpoc: Ne11
Primary aldosteronism (Connes syndrome) is:

OTBETbi(oAMH OTBeT)
1
Increased secretion of aldosterone under the influence
of angiotensin

2 Tumor of the fascicular zone of the adrenal cotex

3 Tumor of the reticular adrenal cortex

4 Tumor of the glomerular zone of the adrenal cortex

5 Tumor of the adrenal medulla

89 10 11 12

13 14 15 50

1 2 3 4 5 67 8 9 10
11 12 13 14 15 16 17 18 19 20
21 22 23 24 25 26 27 28 29 30
31 32 33 34 35 36 37 38 39 40

41 42 43 44 45 46 47 48 49 50

OTBeueHHblM BOnpoc
nponyueHHbiM Bonpoc

G 11

O
Scanned with CamScanner
10:02 V 9
l 57%

A avn.kgma.kg/webtest/1

AxacaH Myxammaa HaguM Kg Ru

Bonpoc: N°10
The pathogenesis of tertiary hypothyroidism is due to:

OTBeTbI(oqMH OTBeT)
1
Secretion of biological inactive TSH

2 Increased synthesis of FSH

3 Autoimmune process in the thyroid gland

4 Decrease in the synthesis of thyroliberin

5 Increased TSH secretion

6 7 8 9 10 11

12 13 14 50

1 2 3 45 67 89 10
11 12 13 14 15 16 17
18 19 20
21 22 23 24 25 26 27 28
29 30
31 32 33 34 35 36 37 38
39 40
41 42 43 44 45 46 47 48
49 50
OTBeYEHHblM Bonpoc
nponyuyeHHbIM BOnpoc
Texyuw Bonpoc

Ao 3aBepweHua TeCTa oCTanocb

1:13:46

Scanned with CamScanner


10:02 9 X ll 57%
A avn.kgma.kg/webtest/1 0
AxacaH MyxaMMaA HaauM
Kg

Bonpoc: N99
The cause of secondary aldosteronism is:

OTBeTbI(oqWH OTBeT)

1 Tumor of the adrenal medulla

2 Tumor of the reticular adrenal cortex

3 Tumor of the fascicular zone of the adrenal cortex

4 Tumor of the glomerular


zone of the adrenal cortex

5 Increased secretion of aldosterone under the influence


of angiotensin

6 8 10

11 12 13 50

4 567 818 919 20


10
2312 13 14 15 16 17
11
21 22 23 24 25 26 27 28 29 30
40
31 32 33 34 35 36 37 38 39
41 42 43 44 45 46 47 48 49 50
OTBeyeHHbIM BOnpoc
nponyyeHHbiM Bonpoc

G 91

K
O
Scanned with CamScanner
10:02 9 l 57%

A avn.kgma.kg/webtest/1 0
AxacaH MyxaMMaA HaguM Kg Ru

Bonpoc: N°8
Conn's syndrome (primary aldosteronism) is manifested by

OTBeTbi(oAWH OTBeT)
1
Hypotension
2 Na loss and K delay

3 Oliguria

4 Na retention and loss of K

5 Accumulation of H ions

1 O 9 10

11 12 50

1 2 3 4 5 6 7
8 9 10

11 12 13 14 15 16 17 18 19 20
21 22 23 24 25 26 27 28 29 30
31 32 33 34 35 36 37 38 39 40
41 42 43 44 45 46 47 48 49 50

OTBeyeHHbiM BOnpoc
nponyyeHHbIM BOnpoc
Tekyun BOnpoc

o 3aBepuweHnA TECTa ocTanocb

1:14:02

Scanned with CamScanner


10:01
i9 ll 57%

O A avn.kgma.kg/webtest/1
Kg Ru
AxacaH MyxaMMaa HaaMM

Bonpoc: Ne7
The appearance at the age of 3 to 7 years of secondary sexual
characteristics is observed:

OTBeTbI(OAMH OTBET)

1
Pituitary dwarfism

.2 Adrenogenital syndrome

3 Acromegaly

4 Adiposogenital dystrophy

5 Klinefelter Syndrome

9
**
3 45 6
7 8

10 11 50

1 2 3 4 5 67 89 10

11 12 13 14 15 16 17 18 19 20
21 22 23 24 25 26 27 28 29 30
31 32 33 34 35 36 37 38 39 40
41 42 43 44 45 46 47 48 49 50

OTBeYeHHblM BOnpoc
nponyuyeHHbIM BOnipoc
TEKynn BOnpoc

4o 3aBepweHnA TECTa oCTanocb

O
Scanned with CamScanner
10:01 Vg
l 57%

A avn.kgma.kg/webtest/1

AxacaH MyxaMmaa Haqum Kg Ru

Bonpoc: N96
Patient S., 56 years old, diagnosed with
autoimmune thyroiditis on the basis of
examination, palpation and laboratory tests.
Choose which thyroid test method is more
applicable here:

OTBeTbi(oAMH OTBeT)

1. ultrasound

2 positron emission tomography

3 K-ray examination

4 scintigraphy
5 endoscoPY

1 2 3 4 5
6 7 8 9

10 50

1 2 3 4 5 6 7 89 10
11 12 13 14 15 16 17 18 19 20
21 22 23 24 25 26 27 28 29 30
31 32 33 34 35 36 37 38 3940
41 42 43 44 45 46 47 48 49 50

OTBeyeHHbIM BOnpoc

O
Scanned with CamScanner
10:01 g rll 57%

A avn.kgma.kg/webtest/1
AxacaH MyxaMMaA HaguM Kg Ru

Bonpoc: N95
A mother consulted a doctor, whose son had
grown by 10 cm over the summer. When
examining a 19-year-old boy: height 180 cm,
weight 68 kg. Which research method is more
applicable for diagnosing a disease:

OTBETbI(ogMH OTBeT)

1 computed tomography

2 Craniography
3 ultrasound
4 target X-ray of the skull

5 magnetic resonance imaging

1 23 4 6 7 8 9

50

1 2 3 4 5 6 7 8 9 10
11 12 13 14 15 16 17 18 19 20
21 22 23 24 25 26 27 28 29 30
31 32 33 34 35 36 37 38 39 40
41 42 43 44 45 46 47 48 49 50

OTBeyeHHblM BOnpog

Scanned with CamScanner


10:01 eV 9 * l57%
A avn.kgma.kg/webtest/1

AxacaH MyxaMMaa HaauM Kg Ru

Bonpoc: N94
OnComputed Tomography.on the anterior wall of
the trachea to the jugular notch of the sternum, a
tumor-like formation extending into the anterior
mediastinum is determined. Explain which organ
is affected.

OTBETbI(oqWH OTBeT)
1
Pparathyroid lymph nodes.
2 perotracheal lymph nodes

3 thyroid gland.

4 thymus gland.

5 parathyroid glands.

11 2 3
4 5 6 7 8

50

1 2 3 4 5 6 7 89 10
12 13 14 15 16 17 18 19 20
11

21 22 23 24 25 26 27 28 29 30
31 32 33 34 35 36 37 38 39 40
41 42 43 44 45 46 47 48 49 50

OTBeueHHblM BOnpoc

Scanned with CamScanner


10:01 V9 ll 57%

avn.kgma.kg/webtest/1
OA
AxacaH MyxaMMaA HaguM Kg Ru

Bonpoc: N°3
which is
Name a thyroid disease, the main method of detecting
scintigraphy:

OTBETbi(oAMH OTBET)

1 thyroid cance

2 toxic adenoma of the thyroid gland

3 diffuse toxic goiter

4 thyrotoxic goiter

5 parathyroid tumors

1 2 4 5 6 7 50

7 8 9 10
2 3 4 5 6
1

11 12 13 14 15 16 17 18 19 20

21 22 23 24 25 26 27 28 29 30
31 32 33 34 35 36 37 38 39 40
41 42 43 44 45 46 47 48 49 50

OTBeyeHHbIM BOnpoc
nponyeHHblM Bonpoc
TeKyÀ BOnpOc

Ao 3aBepweHuA TECTa oCTanocb

Scanned with CamScanner


10:01 9 * l 57%

A avn.kgma.kg/webtest/1

AxacaH MyxaMMan HaguM g Ru

Bonpoc: N°2
Specify the method of radiological diagnostics used for
screening breast diseases in women older 40:

OTBeTbI(oAMH OTBeT)

1 mammography

2 ultrasound

radioisotope

4 scintigraphy

thermography

5 6 50

111 2 3 4 567 8 9 10
12 13 14 15 16 17 18 19 20
21 22 23 24 25 26 27 28 29 30

31 32 33 34 35 36 37 38 39 40
41 42 43 44 45 46 47 48 49 50
OTBEvEHHbli Bonpoc
nporyuueHHblM BOnpoc

G 2

O
Scanned with CamScanner
10:01 9 x l 57%

OA avn.kgma.kg/webtest/
AxacaH MyxaMMaA HaguM KgRu

Bonpoc: Ne1

Name the main screening method of imaging the thyroid gland:

OTBeTbi(oAMH OTBeT)

1 radiography

2 ultrasound
3 thermalimaging
4 radioisotope

5 magnetic resonance imaging

1 2 3 4 5 6 50

7 8 9 10
1 2 3 4 5 6

12 13 14 15 16 17 18 19 20
11
21 22 23 24 25 26 27 28 29 30

31 32 33 34 35 36 37 38 39 40
41 42 43 44 45 46 47 48 49 50

OTBeyeHHblM BOnp0C
nponyuEHHbIn Bonpoc
TeKyuinM Bonpoc

Ao 3aBepuweHnA TECTa oCTanocb

1:14:59
O
Scanned with CamScanner
samjo pass irformatiox|S Pe6 TecTe ponanne GMade 30 years old, He *G Patient 8. 50 years oe t4 test i done-
S endacrine Mainpdf *S Fnddocrine fralpd# * xGhow X

C A Not secure | avn.kgma.kg/webtest/testing


Arps G Espert QAI Cheg.. t Linkedin
oina Masting-Zo. Mal SHIVENDRA WhatsApp co Chancigarh Urihvert O Campaign URL Euil. Applications| 3Pa. Locatton- Noids
CuNx MaNApa Mparan

1 2 3 4 5 6 7 8 9 10
11 12 13 14 15 16 17 18 19 20
Bonpoc: Ne46
21 22 23 24 25 26 27 28 29 30
Patient A. 34-year-old, applied to a polyclinic with complaints of an increase in body temperature up to 39C, pain in the right side of the thyroid gland, aggravated
31 32 33 34 35 36 37 33 39 40
by swallowing, coughing, chills, tachycardia, weakness. Anamnesis: frequent tonsilitis. Objectively. On palpation, the thyroid gland is sharply painful, dense, regional
41 42 43 44 4547 48 49 50
lymph nodes are enlarged. Laboratory: Total blood test: leukocytosis with a shift to the left, increased ESR. What is your most likely syndrome?
01eeHHIR BOnpOr
porrywe-HM BOIpoc
Tekyubni Donpoc
OTBeTbi(oguH OTBeT)
Ao 3acepueuuR TeCTa oCTaNDCL:
1O thyroid nodule syndrome

0:00:44 2 0 syndrome of hypothyroidism


3apepuwTb TeCT 3 0syndrome of chronic inflammation of the thyroid gland
4 syndrome of acute inflammation of the thyroid gland

5 syndrome of hypofunction of the parathyroid glands

1 42 43 44 45 46 47 48 49 50

endocrine Main pd lectures.pdf samjo pass inform..pdf Enddocrine final pdf Show al
3 endocrie Mainpdt x amjo pass intformatior x Enddocrine fralpd x ee TeT poein G Wale 30 years c . H Paient M Y1s :A tw

C A Not secure | avmkgma.kg/webtest/testing


co Chandigarh LUrivers Campaign URL Puil.. Apnlications| 3Pila O loeation Noida
Apps g Expert 98A | Cheg. n Linkedn loin a Meeting- Z. Mail-5HIVENDRA. WhattApp
CuMr HanApa nparan KaR

1 2 34567 8 9 10 Bonpoc: N930


1 12 13 14 15 16 17 18 19 20

1 22 23 24 25 26 27 28 29 symptom
A white sclera line between the iris and the upper eyelid when looking down with thyroid hyperfunction syndrome is a
31 40
32 33 34 35 36 37 38 39
1 42 43 44 45 46 47 43 49 50
-OTRe4OH4HIA NOnpoc OTBeTbi(oAMH OTBeT)
riporyuetHi BOrIpoc
Toxyuun Donpoc 1OElinek
Ao 3acepuenan recTa oCTanoa: 2 OKrause
0:01:42 3 Grefe

3asepuwnbTeCT 4 0Kocher
5OShtelvag

1 26 27 28 29 30 31 32 33 34 50
ndocrine Main pdf x samjo pass informatic x Enddocrine finalpdf x Beb TecMponaw G Male 30 years cld. He xG Patient B, S0 yæars ols x G
how t4 test is dane x
C A Not secure | am.kgma.kg/webtest/testing
Apps pert QA| Cheg.
in Linkedn Join a Meeting-7o. Mail-SHIVENDRA WhatsApp co Chancigarh Urivers.. Campaign URL Bui.. Applications | 3Pla. locatian - Nolda
CHM

12 3 4 5 6 7 6 910
11 12 13 14 15 16 17 18 19 20
Bonpoc: N99
21 22 23 24 25 26 27 20 29 30
Dysfunction of the kidneys in thyrotoxicosis may manifest itself:
31 32 33 34 35 36 37 38 39 40
41 42 43 44 A5 46 47 43 49 50

OTPe4eHHhÅ FOnpoc OTBeTbi(oAWH OTBer)


ipornyueHbM BOrIpoc
TCKyurii Dompoc 1 Increasing the filtration capacity of the kidneys

Ao 3acepuweHMn TeCTa ocTanoCL 2 Impaired renal blood flow

0:02:52 3OProteniuria
3asepuxT TeCT 4 O Concentration dysfunction

5OImpaired reabsorption of Ca and phosphorus

56 7 8 10 11 12 13 50
endocrine Main.pdf X 5samjo pas informatior X| Fnddecrine finalpdf x es TecTWEOBEHA=
GMale 30 years cld. He X G Patiert E, 50 years old x G haw 14 test is done- X+
C A Not secure avnkgmakg/webtest/testing
Apps Epert aaA Chag iLinkedin Join a Meeting -Zo Mail-SHIVENDRAL WhattApp co Chancigarh Uriver.. O Campaign URL B.. Applications | 3PIla. Location Noida-

CuMr aHApa Nparan

111 122 133 144S 678 9 10 Bonpoc: N943


15 16 17 18 19 20
21 22 23 24 25 26 27 28 29 30
The man was admitted to the clinic, complaints of palpitations, interruptions in the work of the heart, sweating, trembling in the body, weakness. Anamnesis: has
31 32 33 34 35 36 37 38 39 40
been ill for several years. Examination:. the apical impulse is enhanced. Heart sounds are loud, arrhythmic. Pulse 98 in 1 minute. blood pressure 170/60 mm Hg. ECG
41 42
44 45 46 47 43 49 50
tachysystolic form of atrial fibrillation. Your suspected syndrome?
oTReeHiHHI BONpoc
nporyueHbii BOripoc
Teyuni Donpoc
OTBerbi(oAMH OTBeT)
Ao 3anepweHMA TeLTa oCTahocu 1
hyperparathyroidism

0:00:55 20 hyperthyroidism

3abepuwT TeCT 3 hypercorticism

4Ohyperglobinemia

SO hyperglycemia

39 40 41 42 44 45 46 47 50

endocrinc Main pdf lectures pdf samjo pass informpdf Enddocrine fnalpd Show
CHHr

1 2
4 S 6 7 8 9 10
Bonpoc: N3
11 12 13 14 15 16 17 18 19 20
21 22 23 24 25 26 27 20 29 30
31
Indicate which method of examining the thyroid gland is the first priority
32 33 34 35 36 37 38 39 40
4142 43 44 45 45 47 48 49 50
OTRPPHHHi BOnpOC
OTBETbI(oAMH OTBeT)
nporyueHHEi BOrTpoOc
Texyuini nonpoc 1O magnetic resonance imaging

Ao 3arepeHMR TOCTa oCTanocu: 20 endoscopy

0:03:17 3 0 computed tomography

3aBepuTb TeCT 4 O fluoroscopy

5 ultrasound

1 2 3 45 6 7 50
endocrine Main pdf x smio pass informatice x Enddocrine Firalpd X Ba5 TecTpcaaHna xGMale 30 years cid. He * G Patient E. S0 years olc x G how 14 test is donm
x+ X

C A Not secure | amkgma.kg/webtest/testing

ApEs C Espert QA Cheg.1

taLnkedin loin a Meting-7o Mail-SH1VENDRA WhatsApp co Chandigarth LUriers.. Campaign URL Bui. Arplications | 3Pila. loation Noida
CMMr HamApa Nparan

1 2 3 4 56 7 8 9 10
11 12 13 14 15 16 17 18 19 20 Bonpoc: N 22
21
23 24 25 25 27 28 29 30
A man of 30 years old, height is 120 cm. Body proportions are typical for a child 3-4 years old. The skin is pale. The head is small in size, the facial features are small
31 32 33 34 35 36 37 33 39 40
with a childlike ratio of individual parts (relatively small sizes of the upper jaw and chin). Excessive fat deposition on the chest and abdomen. The voice is high. Facial
41 42 43 44 45 46 47 43 49 50
and trunk vegetation is absent. Psychophysical develapment is nomal. There is some infantilism in behavior, memory loss. Basal metabolism is within normal limits.
EOnpoc
OTEeueHHhIÅ
There are abnormalities in the reproductive sphere. What pathology of the endocrine system can be assumed:
rnporryueHHbi borlpoc
TexyuMi Donpoc

Ao 3arepuwenua TeCTa oCTanocu


OTBeTbI(oAH OTBeT)
1OHypofunction of the adenohypophysis
0:02:06
20 Hyperfunction of the ovary
3aBepuwTL TeCT
3OHypofunction of the thyroid gland
4 Hyperfunction of the adrenal gland

5 Hyperfunction of the neurohypophysis

** 18 19 20 21 22 23 24 25 26 50

endocrine Main pdf lectures pdt samja pass inform.-pdf Enddorine final pdf Show ai
Main.pd X samjo pass informatio * S Enddocrine tnalpai x Es TrcT E.AnlE
endocine

A Nat secure | avn.kgma.kg/wehtest/testing


Expert QA Cheg in linked n Join a Meeting-Zo. Mal -SHrVENDRA WhatsAPp co Chandigarth Univers O Campaigm URL BLil Applications | 3 Pilla locaticn Noida
AppsC
CuH Haapa nparan

1 2 3 4 5 6 7 B 9 10 Bonpoc: Ne19
11 12 13 14 15 16 17 1820
in the back and legs. With a dysfunction of which gland it may be
21 22 23 24 25 26 27 28 29 30
Over the past 6 months, the patient has had several bone fractures, muscle weakness, pain
31 32 33 34 35 35 37 3a 39 40 associated:
11 42 43 44 45 46 47 43 49 50
OTaeHeHHhih Eonpoc
poryuEHHL BONpoc OTBeTbI(oAMH OTBeT)
Texyuwi Donpoc
1O Pituitary gland
Ao saucpuwewwn TeCTa ocTanotb:
Thyroid
0:02:16
3 OAdrenalglands
3asepuT TeCT
4 O Parathyroid

5OPancreas

15 16 17 18 19 20 21 22 23 50
CMMI

34 56 7 618 9 10
1 122 13 Bonpoc: Ne14
11
115
16 17 19 20
21 22 23 24 25 26 27 28 29 30
Transhypophyseal regulation is the basis for
31 32 33 34 35 36 37 38 39 40
41 42 43 44 45 46 47 48 49 50

OTRAL4eHHI RIONpoc
OTBeTbi(oquH OTBeT)
nponyeHHbii uoripoC
TekyuMi Donpoc 1 Adrenalcortex

Ao 3acepuenn TeCTa ocTanocu: 2OThyroid gland

0:02:32 3 OGonads

3asepwT TeCT OPancreas


5OParathyroidglands

10 11 12 13
14 15 16 17 18 50
CMH

1 2 3 4 5 67 8 9 10
11 12 13 14 15 16 17 18 19 20 Bonpoc: N939
21 22 23 24 25 26 27 28 29 30
Obesity 2nd degree, the body mass index (kg/ m2) is
31 32 33 34 35 36 37 33 40
41 42 43 44 45 46 47 43
49 50
oTReyaHHhld ROnpoc
OTBeTbI(oAuH OTBer)
porryueHbM BOOc
TCKyuni DOmpoc
1O 35.0 39.9
Ao saccpuwewn TeCTa ocTanoc: 2 25.0 29.9

0:01:09 3O 40.0 45.9

3aBepuWTb TeCT O30.0 34.9

5O18.5 24.9

1 35 36 37 38 39 40 41 42 43 50
endocrine Main petf x samja pass informatior x| Fnddocrine final pd X Heti TectpcEHE x GMale 30 years old. He x G Patient E. 50 yæars olc x G owt4 tet is done x +
C A Not secure | avmkgma.kg/webtest/testing
Apps
G Expert GaA|Cheg
n Linkedin
oin a Meeting-Zo. Mail-SHIVENDRAL. WhatsApp co Chandigarh Urivers.. OCampaign URL Bil.. Appnlications 1 3Plla..
location- Noida -
CwMr anApa nparan Kg Ru

1 2 3 45 67 8 9 10 Bonpoc: Ne16
11 12 13 14 1517 18 19 20
21 22 23 24 25 26 27 28 29 30
Patient B, 50 years old, complains that recently the ears, nose, hands began to increase in size. Which gland is overactive will give similar symptoms:
31 32 33 34 35 36 37 38 39 40
41 42 43 44 45 45 47 48 49 50

oTBeeHHhl BOnpoc OTBeTbI(oAMH OTBeT)


nporryueHbii Bonpoc
Texyuun Bonpoc
10 Adrenalglands

AD sacepucHMA TeCTa oCTano 2 Pituitary gland

0:02:25 3 Gonads

3aeepuwTb TeCT 4O Thyroid

5 Epiphysis

1 12 13 14 15 16 17 18 19 20 ** 50
1234 5 G
78
11 12 13 14 15 16 17 18 19 20
910
Bonpoc: N927
21 22 23 24 25 2628 29 30
Distinguish, which function in babies with hypothyroidism is refilected by muscle hypotonia:
31 32 33 34 35 36 37 38 39 40
41 42 43 44 45 46 47 48 49 50

-OTReeHH Onpoc OTBeTb(oAMH OTBeT)


nporryuewHE BOnpoc
Tenni Donpoc 1 O on the formation of speech

Ao 3aBepwHMR TecTa ocTanocL: 2 metabolism of substances

0:01:51 3 O on the development of static functions

3aBepuwTL TeCT 4 0 of peripheral circulation


5 at work bowel

1 *** 23 24 25 26 27
28 29 30 31 50
CHH

1 2 3 45 67 8 9 10
Bonpoe: N933
11 12 13 14 15 16 17 18 19 20
21 22 23 24 25 26 27 28 29 30
Normal level of body mass index (kg/ m2) is
31 32
34 35 36 37 33 39 40
41 42 43 44 4S 46 47 43 49 50

OTReMOHHÀ BOTpOC
OTBeTbi(oAMH OTBer)
porryueHE BONIpoC
TCHL Dompoc 1 18.5-24.9

Ao 3aucpuueHa TECTa octanocL: 2O25.0-29.9


0:01:31 3 30.0- 34.9

3aaepwwTb TeCT 4 0 35.0 399

5 O 40.0 45.9

1 29 30 31 32
33 34 35 36 37 50
endocrine Mainpd x samja pas informatic x Frddocrine fralpdf X Be TecTypoBZHta X Male 30 years aid. HE X G Patient E. 50 yæars olcxG how t4 tst i dona x +
C A Not secure |
am.kgmakg/webtest/testing9

E App g Expert 4A I Cheg aLnked n loin a Meeting Zo. Mail-SHIVENDRAL WhatsAppco Chancigarh Unierr. OCampaign URL Bul. Appicationt | 3Plla.. Location Noida-

CMMrx HaApa nparan u

2 3 4s 67 8 9 10
1

11 12 13 14 15 16 17 18 19 20 Bonpoc: Ne45
21 22 23 24 25 26 27 28 29 30
Patient G, 42 years old, after a traumatic injury to the neck, notes the periodic appearance of paresthesias (pathological unpleasant sensations, for example, "running
31 32 33 34 35 36 37 38 39 40
creeps) with the subsequent development of seizures. Before the development of seizures, he has nausea, vomiting. Convulsions are tornic in nature, beginning in
41 42 43 44
46 47 43 49 50
the upper limbs and face. Consciousness during this period is preserved. Examination of the patient revealed that the content of calcium in the blood is 06 mmol/
OTReueHHHIX EOnpoc
phosphorus 8.0 mmol/1. What is your preliminary syndrome?
ponyueHHblR BOrIpOC
TexyumM DOnpoc

OTBeTbI(oAMH OTBeT)
Ao 3aepuweHwR TeCTa ocTanoc:

1 O hypopituitarism syndrome
0:00:48
2 0 hypog'ycemic syndrome
3aBepuTbTeCT
3 Ohypoparathyroidism syndrome

4 hypocorticism syndrome

hypothyroidism syndrome
50
1 41 42 43 44 45 46 47 48 49 50

endocrine Main.pdf lectures.pdf amo pavs infom..pdf Enddocririe fralpdf Show a


1
23 4 5 67 8 9 10 Bonpoc: N12
1113 14 15 16 17 18 19 20
21 22 23 24 25 25 27 28 29 30
Violation of the processes of secretion in the endocrine gland can occur due ta:
31 32 33 34 35 36 37 38 39 40
41 42 43 44 45 46 47 48 49 50

oreeueHHh Bonpoc OTBeTbI(oAMH OTBer)


nporyuebi vonpoc
Texyni eOnpoc 1 Violations of the feedback mechanism

A 3acepuwenun TOCTa oCTanocL 20 Weak transport connection with blood proteins

0:02:40 3 O Genetic defect in hormone synthesis

3asepuTb TeCT 4OChanges in the number of specific receptors

5ODisorders of hormone metabolism

89 10 11 12 13 14 15 16 50
23 4 5 6 7 8 9 10
11 12 13 14 15 16 17 18 19 20
Bonpoc: N923
21 22 24 25 26 27 28 29 30
Name the main laboratory criterion of hyperglycetmia syndrome:
31 32 33 34 35 36 37 38 19 40
41 42 43 4A A5 A6 47 48 49 50

0TRe4eww ponpoc
OTBeTbi(oAMH OTBeT)
nporyujetHbii Bonpoc
Texyui Donpoc 1 hypernatriemia
Ao 32cepuueHn TOCTA ocTanocL: 2 hypoproteinemia

0:02:03 3 hypokaliemia

3abepuT TeCT 4 high fasting glycemia

5Ohypercholesterolemia

19 20 21 22 23 24 25 26 27 50
Endocrine Mainpd jo pass informatior x Fnddocrine finalpd x Ba6 TecTpoEIH x GMale 30 years cid. He xG Patienit E, 50 years olc x G how t4 rext is done - x X

C A Not secure | avn.kgma.kg/webtest/testing


Apps Expert QMA| Cheg inLinkedin Join a Meeting-Zo. Mail -SHVENDRA. WhatsApp co Chandigarh Uriver. O Carpaign URL Bl. Applicarions | 3Pilla locaricn- Noicta

CMMrx anApa NpaTan Ru

12 3 4 5 67 8 9 10 Bonpoc: Ne49
11 12 13 14 15 16 17 18 19 20
21 22 23 24 25 26 27 28 29 30
A 55.year-old woman consulted a family doctor at the place of residence with complaints of thirst, dry mouth, weight gain, pain in the lumbar region, sweating.
31 32 33 34 35 36 37 3a 39 40
Objectively: height 170 cm, body weight 120 kg, moon-shaped face, crimson-red, excessive growth of facial hair, acne, excessive deposition of fatty tissue in the
41 42 43 44 45 46 47 43 E50
shoulder, chest, abdomen, red violet color, atrophy of the muscles of the arms and legs. BP 190/100 mm Hg. Blood glucose 12 mmol/1. Choose, what hormonal
-oraeyeH RONpoc
blood test is necessary to conduct any obesity disease?
nporyuetH BOnpoc
-TexyuMü Donpoc
OTBETbI(OAMH OTBeT)
Ao 3atcpuweiwn TeCTa oCTanocu

1
O thyroxine
0:00:28
2 cortisol
3aBepwTh TeCT
3 O insulin

4 Ovasopressin

5 O prolactin

43 44 45 46 47 48 49 S0

endocrine Main pd lectures.pdt sajo pass inform.pdf EnddoEr irie firalpdf Show a
endocrine Main.pdf x samja pass informatior x| Enddocrine firnalpd X Pe6 TeTpomaHna GMale 30 years cid, He: xG Patient 8. 50 yars olc. x G hew 14 test is dona- x+
C A Not seure | amkgmakg/wehtest/testing
Apps C Expert
0A Cheg. n Linkedin loin a Meeting Zo. Mail-SHIVENDRA WhatsApp co Charcigarh Lnivers. OCampwign URL Bil.. Apptications | 3Pilla.. locticn- Noida

CMArx 4aApa nparan Ro

12
11
3 4 5678 9 10 Bonpoc: Ne28
12 13 14 15 16 17 18 19 20
21 22 23 24 25 26 27229 30
A girl, 9 years old with her mother turned to
the family doctor with complaints of constipation, decreased memory and academic performance, overweight,
31 32 33 34 35 36 37 38 39 40
drowsiness that appeared over the past six months. Anamnesis: grew and developed satisfactorily. The family uses little seafood. Objectively: Hight-127,0 cm (N
41 42 43 44 45 46 47 43 49 50
135cm), weight-46cm (N-42 cm). Thyroid gland 1st degree, soft. Heart tones are clear and rhythmic. Survey: General blood analysis: erythrocytes #,7x1012/1n, Hb
01eHeHHh EOnpoc 107g/l leukocytes 4,8x10 91. Hormones: T4 -norm, TSH norm. ECG no pathology. Ultrasonography: the total volume of the thyroid gland 16 cm3. Summarize all
ypoinyueHHb BOrIpoc
the data and decide what pathology the patient has?
TCKyuLMN Donpoc

Ao acepuweHUA TOCTa octanoch


OTBeTbI(OAMH OTBeT)
0:01:48 1 hypothyroidism

3aBepuiTb TeCT
2 iodine deficiency state

3 Ovegetative-vascular dystonia

4 O neurasthenia

5 O deficiency anemia

24 25 26 27 28 29 30 31 32 50

endoca ine Mainpdf lectures.pdf samjo pass inform.pdf aEnddocine final.pd Show a
1 2 3 4 5 67
11 12 13 14 15 16 17 1
8919 10 Bonpoc: N938
20
21 22 23 24 25 26 27 28 29 30
Which microorganism is the main pathogen in acute thyroiditis?
31 32 33 34 35 36
3739 40
41 42 43 4A 45 46 47 48 49 50

oTR4eHhii pOnpoc OTBeTbi(OAMH OTBer)


nporyueHHb BOrIpoc
TORyLLMM uonpoc 1
staphylococcus
Ao 33ccpuweuMT TECTa oCTanoC 2 O viruses

0:01:13 30mushrooms

3anepuwT TeCT 40 rickettsia

5Oprotozoa

** 34 35 36 37 38 39 40 41 42 * 50
1 2 3 4 5 67 8 9 10 Bonpoc: N917
11 12 13 14 15 1618 19 20
21 22 23 24 25 26 27 23 29 30
On the basis of which pituitary hormone are drugs used to stimulate the contractile activity of the uterus:
31 32 33 34 35 36 37 38 39 40
41 42 4344 45 46 47 48 49 50
-oTaeueHHhli Bonpoc OTBETbI(OAWH OTBer)
nporyuwetikbi BoIpoc
Texyum nonpoc 1OVasopressin
Ao 3acepuCHuA TOCTA oCTa/1oCL: 2 Follicle-stimulating hormone

0:02:22 3O Thyroid stimulating hormone

3aaepum TeCT 4 Oxytocin

5 OProlactin

1 13 14 15 1617 18 19 20 21 50
23 45 678 9 10 Bonpoc: Ne18
11 12 13 14 15 16 17 1E 19
20
21 22 23 24 25 26 27 28 29 30
The patient has clinically all the signs of thyrotoxicosis. What research needs to be done to clarify the diagnosis:
31 32 33 34 35 36 37 38 39 40
41 42 43 44 45 46 47 43 49 50

OTReeHHHÀ ROnpoc OTBeTbi(AMH OTBe7)


porryuwenti eopoc
Texyui Donpoc
1OLDL, HDL, bloodcholesterol

Ao 3anepueHMR TOCTa oCTanocu 2 Determine the level of thyroxine

0:02:19 3Blood test for sugar, ketone bodies

BasepuwT TeCT 40 General analysis of blood and urine

prote in composition of plasma, residual blood nitrogen,


5O
14 15 16 17 18 19 20 21 22 50
CMHT

1 2 3 4 5 6 7 8 9
11 12 13 14 15 16 17 18 19 20 Bonpoc: Ne10
21 22 23 24 25 26 27 28 29 30
To peripheral form of endocrine disorders can lead to:
31 32 33 34 35 36 37 38 39 40
41 42 43 44 A5 A6 47 43 49 50

OTRPueHHt sonpoc
OTBeTbi(oAMH OTBeT)
nporryusetei BOrIoC
TeKyL DOnpoc Pituitary alteration
10

AD 33epueHMR TeCTa ocTanocL


20Weakhormane-protein binding
0:02:49 3 OStrong bond of hormones with proteins
3a8epuWT TeCT Alteration of the endocrine gland
5OAlteration ofthe hypothalamus

6 7 9 10 11 12 13 14 50
1 234 567 8
9 10
11 12 13 14 15 16 17 18 19 20 Bonpoc: N931
21 22 23 24 25 26 27 28 29 30 Which of the following corresponds to impaired glucose tolerance (venous plasma)
32 33 34 35 36 37 38 39 40
41 42 43 44 45 45 47 43 49 50

oTReeHhai ROnpo OTBeTbI(OAMH OTBeT)


nporyuetHbii vonpoc
TOKyni Donpoc 1Ofasting glucose 54 mmol/ ,2 hours after OGTT 9.2 mmol/

A 3cpweHMR TOCTa octanocu: 2 fasting glucose 6.8 mmol/,2 hours after OGTT 10.9 mmol/
0:01:37 3 fasting glucose 9.6mmol /, 2 hours after OGTT 14.4 mmol/

3aBepuvT TeCT 40fasting glucose- 10.0 mmol/,2 hours after OGTT 16.8 mmol/1

SO fasting glucose 89 mmol/ 1,2 hours after OGTT 12.7 mmol/

1 27 28 29 30 31 32 33 34 35 50
endocrine Mainpdf * amjo pass informatio x6 Frddocrine firal pdf X Bei TecTepcazHn X GMale 30 years cld. He X G Patient B., 50 years olc. X G howt4 tæst is done-G X +
C A Not secure | avm.kgma.kg/webtest/testing
App g Expert QaA |Cheg nLinkedin loin a Meeting-Z.. Mail-5HIVENDRA WhatsApp cChancig arh Uriver O Campaign LRL Buil Applications | 3Pilla. Location Noida

CHr HanApa nparan Ru

1 2 3 4 5 67 8 9 10
11 12 13 14 15 16 17 138 19 20 Bonpoc: N941
21 22 23 24 25 26 27 28 29 30
A 43-year old man. a truck diver, was admitted to the clinic with complaints of an increase in blood pressure up to 190/100 mm Hg,
31 32 33 34 35 36 37 38 39 40 accompanied by headaches
diztinesstinitu Parents have obesity, diabetes melitus, arterial hypentension. On examination: abdominal obesity, body weight-110 kg, height- 160 cm, Broca's
42 43
44 45 45 47 48 49 50 cm.
index-6o kg, Woist circumference - 130 cm, hip circumierence - 110 not enlarged. Auscultation: acent tone above the
Palpation: the liver is painles, even,
-01a4@uH BOnpoc
aorta, BP 180/95 mm Hg. Laboratory: fasting venous blood glucose 4.5 mmol/ , 2 hours a fter eating 7.1
l
nporryeHHbii BOnpoc mmol/ 1. What is the most typical syndrome of a
TeKyuni DonpoC
patient that has developed as a result of obesity?

Ao 3anepuieHun tecTa oCTanoCL


OTBETbI(oAuH OTBeT)
0:01:01
10hepatomegaly
3anepuT TeCT
2 Ohyperglycemia

3 hypercortisolism

4 0 hypertension

5O gynecomastia

37 38 39 40
41 42 43 44 45 50

erndocrine Main.pdf
lectures.pdf samjo pas inform.pdf Enddocrine finat pd
Show all
Endocrine Main pedf * 6 samja pas informaticor * Enddocrine finalpdf x Be6 TecTypoRAHVnE X G Male 30 years old. He
*G Patiert E, 50 years olr x G haw t4 test is done -
x+
C A Not secure | am.kgma.kg/webtest/testing
Ars a Meeting-Zo
G Exper GaA ICheg nlinkedn cin Mail-SHIVENDRA WhatsApp c Chancigarh Urivers O Campaign URL Bl.. Applications | 3Pila. Location Noida

CMH 4aApa nparan Kg

1 23 4 SG7 89 10
11 12 13 14 15 16 17 18 19 20 Bonpoc: Ne6
21 22 23 24 25 26 27 28 29 30 A 63-year-old patient camplains of difficulty passing solid food. Objectively, the patient is lethargic, his voice is hoarse. On examination, bradycardia, pasty skin and
31 32 33 34 35 36 37 33 39 40
41 42 43 44 45 46 47 48 49 50
subcutaneous fat. A barium swallow was performed, which showed a moderate deviation of the pharynx and the upper third of the esophagus to the right, the
contours of the esophagus are normal. What is the most informative research method you can use?
OTneucHHÅ ROnpoc
riporyuieHHbiM BOnpOC
TexyuiMi Donpoc
OTBeTbI(oAMH OTBer)
Ao 3apcpweHMA TECTa oCTA1OC 1 thyroid ultrasound

0:03:04 2 0 computed tomographywith contrasting thyroid gland

3asepuNTh TeCT 3 O endoscopy

4 O multi spiralcomputed tomography

5Ooverview Radiography

12 3
456 7 89 10 50

MMain pul
endoerine lectures,p!f samjo pass inforn.- pdf Enddeerine fnalpdf Show all
CMM

13 45 67 B9 10
11 12 13 14 15 16 17 Bonpoc: Ne2
1a 19 20
21 22 23 24 25 26 27 28 29 30
Specify the method of radiological diagnostics used for screening breast diseases in women older 40:
31 32 33 34 35 36 37 38 39 40
41 42 43 4 45 46 47 43 49 50
orneyeHHbIM BOnpac OTBeTbI(oAnH OTBer)
nponryueHblM BOnpo
TexyLIMÄ Donpoc
mamrmography

Ao 33nepuieHuA TeCTa oCTa1oC 20 radioisotope

0:03:22 3 ultrasound

3aBepuwTb TECT 4 thermography

5 scintigraphy

1 45 6 7 50
x samjo pass informatio x x Be6 Tectpcanae Parient B., 50 years olc x G how t4 test is done x|+
edocrine Main pd Fnddoerine final pd G Male 30 yers cid.
HexG
C A Not secure | avm.kgmakg/webtect/testing

Apps Epert 0A Cheg. nLinkedin a Weeting Zo co Chancigarh Urivers


oin Mal-SHIVENDRA. WhatsApp Campaign UEL Blil Applicatians | 3Filla. Loeation -
Noida-

CHurx anApa Npa

2 34 56 7 8 9 10
11 12 13 14 15 16 17 18 19
Bonpoc: Ne1
20
21 22 23 24 25 26 27 28 29 30
After removal of the thyroid gland, the patient developed convulsions. Determine which structure was damaged during the operation?
31 32 33 34 35 36 37 33 39 40
41 42 43 44 45 46 47 48 49 50

oTReHeHHLIÅ ROnpoc OTBeTbi(oAMH OTBer)


nporyuenteii Eopoc
reryuutä Donpoc 1
the vagus nerve
Ao 3acepuuevuR TeCTa ocTa1oa: inferior laryngeal nerve

0:03:26 3 sympathetic trunk

3aBepuwTb TecCT 4 Oinferior laryngeal artery


5parathyroid gland

2 3 4 5 6 50
CHH

1 2 3 4 5 67 8 9 10 Bonpoc: N 35
11 12 13 14 15 16 17 18 19 20
21 22 23 24
25 26 27 28 29 30
Laboratory criteria for obesity syndrome
31 32 33
A1
3436 37 39 39 40
42 43 44 45 45 47 43 49 50

OTReveHhIÀ RIOnpoc OTBeTbi(oAMH OTBer)


nporyuuetHei Bonpo
TOKyni Dompoc 1 low LDL cholesterol

Ao sacepuweHn TeCTa ocTanocu 2 high HDL cholesteral

0:01:24 3 Olow triglycerides


3aBepuTb TeCT 4O low total cholesterol

50 high glucose

31 32 33 34 35 36 37 38 39 50
samja pass informatio X
€ Enddecrine final pa* X Eei TerTAFOA*
endocrine Main pd X S

C A Not secure | avmkgma.kg/webtest/testing


co Chandigarh Univers.. Campaign URL Buil Applications | 3Pilla. location- Noida
Apps g Expert QA| Cheg. tn Linkedin loin a Meeting-Za.. Mail SHIVENDRA WhatsApp
CuHrx HaApa nparan Ka

1 2
35 67 8
9 10 Bonpoc: Ne4
11 12 13 14 15 16 17 18 19 20
is painful to touch, the pain intensifies during
21 22 23 24 25 26 27 28 29 30 A26-year-old woman came to the clinic with complaints of pain in the right marnmary gland, the mammary gland
31 32 33 34 35 36 37 38 39 40
menstruation. The doctor ordered a mammogram, Indicate in your answers when to get a mammogram:
41 42 43 44 A5 46 47 43 49 50

OTReeHHhM Ronpoc
11ponyueHtbit uonpoc OTBeTbI(oaMH OTBeT)
TCKyn vonpoc
10 only after menopause
Ao 3acepueHMB TeCTa oCTanOCL:
2regardless of the phase of the cycle
0:03:12 O from the 6th to the 12th day of the menstrual cycle
3aBepuwT TeCT
4 O from the 1st to the 5th day of the menstrual cycle

5Oin the second half of the menstrual cycle

1
2 7 8 50
CMn 4aApa NpaTan Kg

1 2
3 45 6 7 8 9 10
Bonpoc: N913
11
1214 15 15 17 18 19 20
21 22 23 24 25 26 27 28 29 30
31 32 33 34 35 36 37 30 3940
An excess of thyroid hormones occurs when
41 42 43 44 45 46 47 48 49 50
oTRPeHHR nOnpoc OTBeTbi(OAMH OTBeT)
porryuennbi BOpO
-TexryuM nonpoc 1 Diffuse toxic goiter

Ao sacepweHun TeCTa oCTanoc


20 Acromegaly

0:02:35 3 OMiksedeme

3anepuwTb TeCT 40Insulinoma


5OEndemiccretinism

1 ** 910 11 12 13 14 15 16 17 50
Endocrine Mainpdf X samjo pass informatior *| SFnddocrine firalpd x Be5 TeTpORaHne X GMale 30 years ald. He x|G Patiert E. 50 years ole G baw 14 test is done-X +
C A Nat secure | ankgma.kg/webtest/testing
App GExpert Q&A| Chag. tn Linikedin Jaina Meeting Zo. Mall-SHVENORA. WhatsAep co Chancigarh L/ivers.. OCampaign URL Buit.. Applicarions | 3Pll. tocation Noida

CHurx 4anApa npaTan

1 2 3 4 5 6 7 8 9 10
11 12 13 14 15 16 17 1a 19 20 Bonpoc: Ne47
21 22 23 24 25 26 27 28 29 30
A 52-year-old woman consulted a therapist with complaints of thirst, frequent urination, itching of the perineum. In the anamnesis, the above complaints are worried
31 32 33 34 35 36 37 33 39 40
for month. Objectively. The skin is dy and clean. BMI 32.2 kg/ m2. BP 130/90 mm Hg. Laboratory blood glucose 10.3 mmol/, after meals 15.0 mmol/
1
41 42 43 44 45 4643 49 50
What analysis can you use to find out blood glucose in 3 manths?
OTReeH-hIR RONpoc
porryuHHii BOrnpoc
Texywi Donpoc
OTBeTbi(oAMH OTBET)
A ancpuiewia TeCTa ocTaica 1 total cholesterol

0:00:41 2 blood glucose


3aBepuTb TeCT 3 Otriglycerides

4 O glycohemaglobin

5OC- peptide

1 43 44 45 46 47 48 49 50

endouine Mairnpdl lecturespdf sarrjo pass inforn..pdf Enddocine finalpdf Show a


endocrine Main.pdf X samja pass informatio x Enddocrine finalpd x Bes TecTupaRAHA X GMale 30 years cld, He x GPatient E., 50 years old x G how 14 test is done-G X +
C A Not senure | avmkgma.kg/webtest/testing
Appe Fspert 08A Cheg. in Linkedin Join a Meeting Zo.. Mail-SHIVENDRA WhateAprp co Chancigarth Univers. O Carpaign URL Buil.. Applications 1 3Pila. 1ocation Noida

CMMx HaApa npaTan Ru

1
23 4 5 67 8
9 10 Bonpoc: Ne49
11 12 13 14 15 16 17 18 19 20
21 22 23 24 25 26 27 28 29 30
A 55-year-old woman consulted a family doctor at the place of residence with complaints of thirst, dry mouth, weight gain, pain in the lumbar region, sweating.
31 32 33 34 35 36 37 33 39 40
Objectively. height 170 cm, body weight 120 kg, moon-shaped face, crimson-red, excessive growth of facial hair, acne, excessive deposition of fatty tissue in the
81 42 43 44 45 46 47 483
150 shoulder, chest, abdomen, red-violet color, atrophy of the muscles of the arms and legs. BP 190/100 mm Hg. Blood glucose 12 mmol /1. Choose, what hormonal
OT84eHH BOnpoc blood test is necessary to conduct any obesity disease?
1porryuenbi bonpoc
TexyunÅ DonpOc

OTBeTbI(oqMH OTBeT)
Ao acepueHMA TCCTa OCTanotL

1 0 thyroxine
0:00:34
2 cortisol
3apepuTb TeCT

3 Oinsulin
4 Ovasopressin

5 prolactin

43 44 45 46 47 48 49 50

endocrine Main pdf lectures pdf asamjo pass infompdf Enddocaine final pdf Show all
endocrine Main potf x sarja pazs informatier x Frddocrine finalpdf X Ba6 TerTmpaRaHnE X GMale 30 years ald. He x GPatient E, 50 yars olo x G how t4 sEt is done-x +
C A Not secure | an.kgma.kg/webtest/testing
Apps g Expert Q&AI Cheg.n linkadin Join a Menting- 7o. Mail-SHIVENDRA WhatsAnp co Chandigarh Urivers.. OCampaign URL Buit Applicarione 3Pla.. location Nolda

CMMr 4aNApa nparan KgRu

1 2 3 45 6 7 8 9 10 Bonpoc: N920
11 12 13 14 15 16 17 18 19
21 22 23 24 25 26 27 28 29 30
Female, 52 years old, onset of the disease 1.5 years ago after a course of radiation therapy. Complains of memory impairment, general weakness, drowsiness,
31 32 33 34 35 36 37 38 39 40
baldness, lack of appetite, weight gain. An objective examination revealed: the skin is dry, flaky, the face is puffy, hypomimic, there are areas of baldness on the head,
4142 43 44 45 46 47 48 49 50 ,
the voice is low, rough, speech is slowed down, the body temperature is 35.7 the pulse is 52 beats. per minute, blood pressure 110/65 mm Hg. Art, the level of
-OTReveHHIÀ BOnpoc glucose in the blood 3.5 mmol/1, the basal metabolism is reduced by 30%. Diagnosed with Myxedema. What is the most likely cause of this disease
nporryueHbli Bonpoc
Toxyui DOnpoc

OTBeTbI(oAWH OTBeT)
Ao 3abepueHMA TeCTa oCTa1ocL:

1 O lodine deficiency
0:02:13
2O Formation of reverse T3
3asepuwTb TeCT

3 O Decreased TSH levels

OFormation of mucin
5 Increase in T3 T4

16 17 18 19 20 21 22 23 24 50

endocrine Mainpdl lectures.pdf samjp pas inform.pdf Enddecrine finalpdf Showa


CHM

1 2 3 4 5 6 7 9 10
11 12 13 14 15 16 17 18 19 20 Bonpoc: N8
21 22 23 24 25 26 27 28 29 30
Hyperproduction of glucocorticoids causes:
31 32 33 34 35 36 37 33 39 40
41 42 43 44 45 46 47 48 49 50

OTBeyeHHbi EOnpoc OTBeTbI(oauH OTBer)


nporyueHbi BOrnpo
TOyuni eonpoc 1 Increased blood pressure

Ao 3apcpuewA TecTa ocTanocb 2 0 Decrease in the excitability of the nervous system

0:02:56 3 O Increased bone ossification

3aBepuwT TeCT 4 OHypoglycemia


5 OPositive nitrogen balance

45 6 7 89 10 11 12 50
ndocrine Main pdf x amjo pass informatior * Frddocrine finat paf x Be6 Terruponante x GMale 30 yeas cld, He x G Patient E. 50 years ole x G haw14 test is done x + X

6 C A Not secure | avn.kgma.kg/webtest/testing


App Expert C&A Cheg.
| in a Maeting -Zo.. WhatsAr co Chandigarh Univer. URL Bull Applicatians | 3Pila.. Locaticn-Noicda
Linkedn loin Mail-SHIVENDRA Campaign

CHMrx HanApa Mparan Kg Ru

12 3 4 5 6 78 9 10
14 Bonpoc: N915
11 12 13
21 22 23 24
16
2526
17 18 19 20
27 28 29 30
A patient with complaints of decreased appetite and great thirst. Diuresis is 10 liters per day. Blood sugar 4mmol/1 What disease can you think of in this case?
31 32 33 34 35 36 37 33 39 40
41 42 43 44 45 46 47 43 49 50

-OTRe4PHHÅ Eonpoc OTBeTbI(oAMH OTBeT)


iporyueHHbIN BOTIpoc
Texyunii vornpoc 1O Diabetes insipidus

AO 1BDepweHIA TOCTa OCTaTDCL 2 Non-sugaranti-diabetes

0:02:29 3 ORenal diabetes


3aBepuuvTh TeCT 4ODiabetes mellitus
5O Gestational diabetes

1 *
11 12 13 14
15 16 17 18 19 50

endocrine Mainpdf lecturespdf pass inform..pdt


sanjo Enddocrine final pdf
Show al
CHHr

1 2 3 4 5 6 7 8 9 10
Bonpoc: N911
12
21
13 14 15 16 17 18 19 20
22 23 24 25 26 27 28 29 30
Adrenogenital syndrome develops with increased secretion:
31 32 33 34 35 36 37 38 39 40
41 42 43 4445 46 4748 49 50
oTEPueHHhlÅ ROnpoc OTBeTbI(oAMH OTBer)
nportyueHE1 BOIPOc
Teryuni nonpoc 1OMineralocorticoids
Ao 33ccpuwcHWA TeCTa ocTanoc 2O Sex hormones testes

0:02:44 3 O Catecholamines

3aBepuwTb TeCT Corticosteroids


5OSex hormones in the reticular adrenal cortex

7 8 9 10 11 12 13 14 15 50
CwH

1 2 3 4 5 67 B9 10 Bonpoc: N929
11 12 13 14 15 16 17 18 19 20
21 22 23 24 25 26 27 2830 Waist circumference in men is normally less than (cm)
31 32 33 34 35 36 37 38 39 40
11 42 43 44 45 45 47 48 49 50

oTReHPH-ti ROnpo OTBeTbi(oAMH OTBeT)


poryuebli bOnpoc
Teryunii nonpoc
1 102

Ao 3acepueHHn TeCTa ocTanoc 2O78


0:01:45 3O 94
3aBepwwT TeCT 40110
5 086

1 25 26 27 28 29 30 31 32 33 50
1 2 3 4 5 68 9 10
Bonpoc: N97
11 12 13 14 15 16 17 18 19 20
21 22 23 24 25 26 27 28 29 30
In the onset of Basedow's disease, the main role belongs to:
31 32 33 34 35 36 37 38 39 40
41 42 43 44 A5 46 47 48 49 50

oreeeHHl EOnpoc OTBeTbi(OAMH OTBeT)


nporyuewHb BOnpoc
Texyuni Donpoc 1 Increased production of somatotropic hormones

Ao 3aepweHuA TeCTa ocTanoCL: 2Insufficient aldosteron production


0:03:00 3 Insufficient action of parathyroid hormone

3aeepuT TeCT Excessive action of thyroid hormones


S Increased production of cortisol

**

45 67 89 10 11 50
endocrine Main pdf
xS samjo pats infarmaticr x Encdoerine firalpdf x Bas TecTpoRAHM
XGMale 30 year old. He x GPatient B. S0 yæsrs alc x G
how t4 t=st it donE- X + X

C A Not serure| avn.kgma.kg/webtest/testing


Apps CExpert Q&A| Cheg. in Linkedin Jin a Meeting - Zo.. Mal - SHIVENDRA WhattApp co Chandigarh Univer.. O Campaign URL Buil. Appications | 3PIla location Noida
CMMx aHApa Mpatan KgRu

1 2 3 4 5 6 7 B
9 10
11 12 13 14 15 16 17 18 19 20
Bonpoc: Ne42
21 22 23 24 25 26 27 28 29 30 A 29-year-old man, was admitted to the clinic with complaints of palpitations, sweating. tremors in the body, weakness, diarrhea, darkening of the skin in places of
31 32 33 34 35 36 37 33 39 40
skin friction. Anamnesis: has been ill for several years. Examination: the apical impulse is enhanced. Heart sounds are loud, arrhythmic. Pulse 98 in 1 minute. blood
43 44 45 46 47 43 49 50
pressure 170/60 mm Hg. TSH is reduced, T3, T4 is increased. The doctor diagnosed hyperthyroidism syndrome. Explain what is the connection with the appearance
OTBeieHHiÀ BOnpOC of skin darkening due to?
poryueHHIM BOIIPoC
rexyuui nonpoc

OTBeTbI(oAWH OTBeT)
Ao 3anepueHAR TeCTZa oCTanoc

1 adrenal insufficiency
0:00:58
2 O liver failure
3aeepwwTb TeCT

3 O renal failure

4O thymus insufficiency

5 parathyroid insuficiency

38 39 40 41
42 43 44 45 46 50

endocririe Mgin,pdf lectures.pdt s.arjo pass infampdf Enddocrine Fnal pilf Show
1 2 34 5 6 7 6 9 10
11 12 13 14 15 16 17 18 19 20
Bonpoc: N937
21 22 23 24 25 25 27 28 29 30
Indicate, what change in the cardiovascular system is characteristie of the syndrome of hypercortisolism?
31 32 33 34 35
3638 39 40
41 42 43 44 45 45 47 48 49 50

OTReHHÅ EOnpoc OTBeTbi(oAMH OTBET)


nporryueE sonpoc
Texyunä Donpoc 1 weakening of heart sounds

Ao 3apepweHun TCCTa ocTanocu 2 increase blood pressure

0:01:16 3 pulse soft, low tension

3aaepuT TeCT 4 apical impulse is weakened, low

5 0 displacement of the right border

1 33 34 35 36 37 38 39 40 41 ** 50
111 122 133 1445 16678 9 10
17 18 19
Bonpoc: Ne25
15 20
21 22 23 24E 26 27 28 29 30
Determine, which of the following is the main thyroid hormone:
31 32 33 34 35 36 37 38 39 40
41 42 43 44 A5 46 47 49 49 50

OTRAIHHii BOnpo OTBeTbI(OAMH OTBer)


nporryueblñ BorIpoc
TOKYLI Donpor 1Othriodthyronine
A 3ccpucHWA TecTa oCTaocu 2 thyrotropin releasing hormone

0:01:57 3Othyroid- stimulating hormone

3anepuwn TecT thyroxine


5 Othyrocalcitonin

** 21 22 23 24 25 26 27 28 29 50
1 2 3 45 6 7
6 9 10
11 12 13 14 15 16 17 18 19 20 Bonpoc: N926
21 22 23 24
2527 28 29 30 Identify the main cause of constipation and flatulence in children with hypothyroidism:
31 32 33 34 35 36 37 33 39 40
41 42 43 44 45 45 47 43 49 50

-oTBeueHHLIÅ EIOMpoc OTBeTbI(oAMH OTBET)


ripornyueHHbM BOrIpoc
Texyun vonpoc 10 decrease metabolism of substances

Ao 3apepweHnn TecTa oCTanoCL 2 poor appetite

0:01:54 3 motor function of the intestine

3aBepuwTL TeCT 4Odecreased physical activity


5 Iow water consumption

1 22 23 24 25 26 27 28 29 30 50
1 2
3 4567 8
11 12 13 14 15 16 17 18 19 20
9 10
Bonpoc:N 32
21 22 23 24 25 26 27 28 29 30
What is the most informative method for examining the thyroid gland in acute thyroiditis:
3133 34 35 36 37 38 39 40
A1 42 43 44 45 46 47 48 49 50

OTReeHHhli ROnpoc
OTBeTbI(OAMH OTBeT)
11porryueHbi BOnpOc
TOKyint Donpoc 10lateral radiography of the neck

Ao 2ApepueHMA TeCTa oCTanocu 2 radioisotope scanning

0:01:34 3 inspection and palpation

3anepuT TeCT 4Oultrasound with biopsy


5 auscultation of the thyroid gland

28 29 30 31
32 33 34 35 36 ** 50
CHH

12 3 45 67 8 9 10 Bonpoc: N936
11 12 13 14 15 16 17 18 19 20
21 22 23 24 25 26 27 28 29 30
What are the causes of primary hypothyroidism?7
31 32 33 34
3537 38 39 40
41 42 43 44 45 46 47 43 49 50

OTRAeHHIM AOTpOc
OTBeTbI(oAnH OTBer)
ponyuenHbi Bonpoc
TCKyunn aonpoc
1Opituitary adenoma
Ao 3aepuweHMA TOCT oCTaNOCL: 2 autoimmune thyroiditis

0:01:20 3 Opituitary irradiation

3aBepuTb TeCT 4 0skulltrauma


5Oneuroinfection

* 32 33 34 35
36 37 38 39 40 50
1 2 3 4 567 8 9 10 Bonpoc: N924
11 12 13 14 15 16 17 18 19 20
21 22 232 25 26 27 28 29 30 Indicate the blood glucose level in healthy children (mmol/l):
31 32 33 34 35 36 37 38 39 40
41 42 43 44 45 46 47 48 49 50

OTEeHHHi EOnpoe OTBeTbi(oAMH OTBer)


nporryuee Bonpoc
Texyuni sonpoc 1
144 2,22

Ao 33pepuweHun TOCTa ocTanocb


2O 7,78-8,88

0:02:00 3O 3.33-5,55

3anepuT TeCT 4O 666 7,77


5 O889 9,99

1 20 21 22 23 24 25 26 27 28 50
endoenine Main pdf x mjo pas informatior x Erddocrine fralpdfx Be6 TeCTPpoRawn xGMale 30 years eld. He: x| G Patient B, 50 yeart ol x | G haw14 test in done- Gx+
C Not sequre| avn.kgma.kg/webtest/testing
Apps Expert Q8A| Cheg in -Zo.. co Chancigarh Urivers OCampaign URL Bi
Linkedin loin a Meeting MailSHIVENDRA. WhattArp Applications 3Plla. Location- Noida-

Cwwrx HanApa nparan Ru

1 2 3 4 5 6 7 8 9 10
11 12 13 14 15 16 17 18 19 20
Bonpoc: N244
21 22 23 24 25 26 27 28 29 30
A woman, 50 years old, consulted a local therapist with complaints of thirst, dry mouth, weight gain, headache, pain in the lumbar region, sweating. From the
31 32 31 34 35 36 37 38 39 40
anamnesis: the patient's mother suffered from type 2 diabetes. Objectively: hcight 165 cm, body weight 100 kg. rubeosis of the cheeks, excessive deposition of
41 42 43 45 46 47 48 49 50
fatty tissue in the area of the shoulders, chest, abdomen, pale striae on the skin of the abdomen, thighs, deformation of the nails legs. Waist circumference 102 cm,
OTREPHHIÀRONpoc hip circumference 95 cm. BP 160/90 mm Hg. Interpret, how much the patient's waist circumference should be? Less than..
nporryuieHmbit borpoc
TeKyuuMi Donpoc
OTBeTbi(oAMH oTBer)
Ao 3arepuweHMn TecTa ocTanocu:

1 95
0:00:51
2 0 85
3aBepuTb TeCT
3 0 80

4 O75

5 O90

1
40 1 42
4344 45 46 47 48 50

endocrine Main.pdf lectures.pdt Samjo pass inform.pdf Enddacaine finalpd how ai


endocrine Main pdf samjo pass informatior finalpd Male 30 years cld. He G PaTient E. 50 years olc
*G how t4 tett is done- x
x|Fnddocrine Bes TeceponaHte xG X

C A Not seaure | am.kgma.kg/webtest/testing


Apps Espert QaA 1Cheg in Linkedin
JoinaMeeting Zo. Mall-SHIVENDRA. wharsApp co Chancigarh Univer O Campaign URL Buil.. Applications | 3illa location Noida -

CMNTx 4anApa paran

1 2 3 45678 9 10 Bonpoc: N240


11 12 13 14 15 16 17 18 19 20
21 22 23 24 25 26 27 28 29 30
Patient K, 48 years old, went to the clinic with complaints of severe weakness, weight loss, bone pain, especially in the feet, long-term heafing fractures, loss of
31 32 33 34 35 36 37 38 39
appetite, nausea, vomiting not a3sociated with food intake, diarrhea. From the anamnesis: suffers from adenoma of the right parathyroid gland. Laboratory data:
41 42 43 44 45 46 47 48 49 50
TBC-anemia, Calcium-3.4 mmol/, phosphates-0.7 mmol/1. What is your preliminary syndrome?
-aTRe 4eHHR RONpOC
iponyuenHb BONPO
TeKy Donpoc OrBeTbi(oAMH OTBeT)
Ao accpueHAn TeCTa oCTanoc: 1 0 hyperparathyroidism syndrome

0:01:05 2 O thyroid inflammation syndrome

3aBepuwTb TeCT 3Ohyperglycemia syndrome


4 hypercortisol symdrome

5 O hyperthyroidism syndrome

1
36 37 38 39 40 41 42 43 44 ** 50
S endocrina Mainpdf x amjo pas infarmatior x Fnddocrine firalpd x Bes TecTpoaAve G Male 30 yesrs cl. He xG Pariert E 50 years olr x G haw14test is done- x+
C A Not secure | avn kgma.kg/webtest/testing

App Exper Q8A| Cheg.. n Linkedn


loin aMeeting-7a. Mall-SHIVENDRA WhatsApp co Chandigarh lUrivers O Campaign URL. Bil,. Applicahions | 3Pila loction- Noida

CMMx 4anapa nparan Fiu

1 2 3 4 5
678 9 10 Bonpoc: N950
11 12 13 14 15 16 17 18 19 20
21 22 23 24 25 26 27 28 29 30
Patient V, 50 years old, consulted a doctor at the place of residence with complaints of increased body weight, pain in the lumbar region, increased blood pressure
31 32 33 34 35 36 37 38 39 40
up to 200/100 mm Hg. Objectively. height 165 cm, body weight 121 kg. Obesity with redistribution of subeutaneous fat according to the "cushingoid" type,
41 42 43 44 45 46 47 40 49
redness of the cheeks, striae of red-violet color in the hips and abdomen, multiple acne on the face and back, hypotrophy of the limbs. Fasting blood glucose 15
OTaeeHHkÅ BOnpOC
mmol/1, urine sugar 6%. What hormonal research method will help in making a diagnosis?
nporyuetHbli BOnppoc
rexyui Dompoc

OTBeTbI(oAMH OTBeT)
Ao 3aepwewun TeCTa OcTancCL

10 testosterone
0:00:25
20vasopressin
3aepuwTb TeCT

3 O insulin

4 O thyroxine

5 cortisol

1 * 43 44 45 46 47 48 49 50

endocrine Main pdf lectures.pdf samjo pass inform..pdf Enddocine fnalpdf Show all
Endarina Man pdf * samja pass informatior x|Erddocrine finalpdf x BeS TeCTepoREHMa x GMale 30 years ald. He x GPariant E, 50 yæars tle x G hw t4 test is done x +
C A Not secure | avn.kgma.kg/webtest/testing

Apps apet O8A Cheg. nLinkrdin loin a Meeting-7o.. Mal - SHIVENDRA.. hatsApp co Chancigarth Univers..O Campain URL Bui.. Applications| 3Pila location-Noida

CHn 4anApa npaTan Ru

12 3 45 6
789 10
Bonpoc: N 21
11 12 13 14 15 16 17 1a 19 20

22 23 24 25 26 27 28 29 30
Male 30 years old. He was admitted to the clinic with complaints of headache, drosiness, change in appearance over the past 2 years, during which he considers
31 32 33 34 35 36 37 38 39 40
himself ill. Examination: an increase in the brow ridges, chin, nose, auricles, tooth divergence. The hands and feet are disproportionately enlarged. AD 160/95 mm
41 42 43 44 45 46 47 43 49 50
Hg, heart rate 90 beats per minute, an increase in the size of the liver, the borders of the heart are expanded, blood glucose 10 mmol/1, total protein 90 g
0TReeHHtÅ ROnpOC
/
urine glucose+. Which of the following preliminary diagnoses is most likely:
nporryuetHb BOnpoc
-TexyunÄ DonpoC

OTBeTbI(OAMH OTBeT)
Ao 3arepuweHMR TeCTa ocTanoc:

0:02:10 1OPituitary gigantism


2 0 Adiposogenitaldystrophy
3aaepuT& TeCT

3 O Pituitary dwarfsm

Acromegaly

5 OPituitary hypogonadism

1 17 18 19 20
21 22 23 24 25 50

endocrine Main pdf lecturespdf samjo pass inform.pdf Enddeaine finalpdf Show a
endocrine Main pdf * samja pazs informatic: x Enddocrine finalpdf x Be6 Tectepcaawe xGMate 30 years cld, He X G Patient B, 50 years olc x G how t4 test it done-
X+
C Not secure | ankgma.kg/webtest/testing

Arps C Fxpert QAAI Cheg. n Linkedn O loin a Meeting-7o Mail-SHIVENDR WhatsApp coChanicigarh Urivers. Campaign URL Bil Appilications| 3Pla. Locmion-Noids

CuHx HanApa nparan

7 8
12 3
46
11 12 13 14 15 16 17 18 19 20
9 10 Bonpoc: N95
21 22 23 24 25 26 27 28 29 30
Patient K, 32 years old, complains of unreasonable aniety, increased mental excitability, weight loss, sweating, frequent stools. She fell ill for 6 months. back after
31 32 33 34 35 36 37 38 39 40
suffering the flu. Revealed eye symptoms (Grafe, Kraus). Pulse 100 in 1 minute. The thyroid gland is enlarged to 3 tbsp. What is the method of radiaticn diagnostics
4142 43 44 45 46 47 48 49 50
that alliows in this case to assess the function of the thyroid gland:
-OTRejeHHHIA Eonpoc
nponyuueHEl BOrIpOC
TeKyun ponpoc OTBeTbI(oAMH oTBer)
Ao 3accpucN TeCTa OCTanoCb
10 doppler study.
0:03:08 2 O nuclear magnetic resonance.

3aBepuMTb TeCT
3 radioimmunoassay.
4 computed tomography of the thyroid gland.

5 O ultrasonic scanning

123 45 6 7 9 50
3 endocrine Main pdf *5 amjo pars informaticx Enddocrine fralpd x e6 TectepomaHe GMale 30 years cl. He GPatient E, 50 years olc *G haw t4 tet is done -
x+ X

C A Not secure | avn.kgma.kg/wehtest/testing


Aprs C Fperr Q8A Chag. n Linkedn loin a Meing-7o Mall-SHENDRA. WhatAp co Chandigarh Univer.. O Campaign URL Bil. Applications | 3Pilla Locaticn-Noida
CHHrx 4aHAPa nparan KgRu

1 2 3 45 67 B
9 10 Bonpoc: N934
11 12 13 14 15 16 17 1a 19 20
21 22 23
24 25 26 27 20 29 30
Cushing syndrome is characterized by
31 32 33 35 36 37 38 39 40
41 42 43 44 45 46 47 43 49 50

-oraee44hiR BOnpoc OTBeTbi(OAMH OTBeT)


nporryuetb BOpoc
D-Texy Donpoc 1 hypercortisolism

Ao 3accpueHn TeCTa ocTanocL: 2 hyperprolactinemia

0:01:27 3 hyperthyroidism

3anepuuT 4 hyperparathyroidism
TecT

5 hyperpituitarism

30 31 32 33 35 36 37 38 50

endocrine Mairnpdf lectures pdf samjo pass inform.pdf Enddactine finalpdf Show all
endocrine Mainpdf x samjo pass informatio x Fnddacrine final.pdf X Befi TerTupaRAHna X GMale 30 years eld. He X G Patient E. 50 yaars cdr x G hew t4 tet is done- x +
C A Not seure |
avn.kgma.kg/webtest/testing
Apps Exper 8A Cheg
Linkedn loin a Meting-Zo. Mail-SHIVENORAL WhateApp co Chancigarh Uriver.. Campaign URL Bui, Applications | 3Pilla. location Noida-

CMMrx4anApa nparan Ru

1 2 3 4 567 89 10
Bonpoc: N948
11 12 13 14 15 16 17 18 19 20
21 22 23 24 25 26 27 28 29 30
Patient A, 34ycar old applied to polylinic with complaints of an increase in body temperature up to 39'C, pain in the right side of the thyroid gland, aggravated by
31 32 33 34 35 36 37 34 39 40
swallowing, coughing, chills, tachycardia, weakness. Anamnesis: frequent tonsilitis. Objoctively: On palpation, the thyroid gland is sharply painful, dense, regional
41 42 43 4445 46 47 49 50
lymph nodes are enlarged. Laboratory: Total blood count: leukocytosis with a shift to the left, increased ESR. Choose the most informative method for examining the
E-oTaeuewH ROnpoc thyroid gland in this situation:
1porryueHHbiii BOnpoc
rexyuwÄ Dornpioc

OTBeTbi(oaMH OTBeT)
Ao sacepuenun TeCTa ocTanocu

Oauscultation of the thyroid gland


0:00:38
20 ultrasound with biopsy
3anepuuwm TeCT
3 O thyroid scan with 131

4 0 determination of T3, T4 in blood

5lateral radiography of the neck.

1 43 44 45 46 47
43 49 50

endocrine Mairn.pdl lectutespdf Sajo pass inform. pdf Endotine fnal pdf Show a X
l 98% 10:12
avn.kgma.kg

Quick.. Be6 X

Moxa DaM3aH Ru

Bonpoc: N°1

Specify the method of radiological diagnostics used for screening breast diseases in women
older 40

OTBeTbI(oAMH OTBeT)
1 mammograpny

2 thermography

3 radioisotope

4 scintigraphy
5 ultrasound

50

2 3 4 5 6 7 8 9 10
11 12 13 14 15 16 17 18 19 20
21 22 23 24 25 26 27 28 29 30
31 32 33 34 35 36 37 38 39 40
41 42 43 44 45 46 47 48 49 50

OTBeHeHHblM BOnpoC
nponyueHHblM BOnpoc
Texyunin Bonpoc

Ao 3aBepuweHMA TeCTa ocTanocb:

1:15:06
3aBepuiWTb TeCT

Scanned with CamScanner


dl 98% 10:12
avn.kgma.kg E

Quick... Be6... X

Moxg dan3aH Ru

Bonpoc: N°2
After removal of the thyroid gland, the patient developed convulsions. Determine which structure
was damaged during the operation?

OTBeTbi(oauH OTBeT)
1 the vagus nerve
2 parathyroid gland

inferior laryngeal artery

inferior laryngeal nerve

sympathetic trunk

7 50
2 3 4 5
6

1 23 4 56 7 89 10

11 12 13 14 15 16 17 18 19 20
21 22 23 24 25 26 27 28 29 30
31 32 33 34 35 36 37 38 39 40
41 42 43 44 45 46 47 48 49 50
OTBeeHHblMBonpoc
nponyujeHHbiM Bonpoc
TeKyuMi BOnpoC

Ao 3aBepweHMA TeCTa ocTanocb

1:14:58
3apepunTb TeCT

Scanned with CamScanner


IN H 98% 10:13

O avn.kgma.kg

Quick... Be6..

MoxA Dan3aH Ru

Bonpoc: N93
Indicate what volume of contrast agent is injected with intravenous contrasting of the adrenal
gland:

OTBeTbI(opMH OTBeT)
1 0-1120 ml

2 30-60 ml
3 120-150 ml
4 150-180 m

5 60-90 ml

1 2 34 5 6 50

7 8 9 10
1 2 34 56
11 12 13 14 15 16 17 18 19 20
21 22 23 24 25 26 27 28 29 30
31 32 33 34 35 36 37 38 39 40
41 42 43 44 45 46 47 48 49 50

OTBeyeHHblM Bonpo
nponyujeHHbIM BOnpoc
TeKyujmn BOnpoc

Ao 3aBepweHWA TeCTa oCTanoCb

1:14:51
3aBepuiTb TeCT

Scanned with CamScanner


N 98% 10:13

avn.kgma.kg E

Quick.. Be6... X

Moxg dana3aH Ka Ru

Bonpoc: Ne4
A 26-year-old woman came to the clinic with complaints of pain in the right
mammary gland, the
intensifies during menstruation. The doctor ordered
mammary gland is painful to touch, the pain
a mammogram. Indicate in your answers when to get a mammogram

OTBETbi(oAWH OTBeT)

1 from the 1st to the 5th day of the menstrual cycle

2 in the second half of the menstrual cycle

3 regardless of the phase of the cycle

4 from the 6th to the 12th day of the menstrual cycle

5 only after menopause

2 3 4 5 6 7 8

234 5 678 9 10
11 12 13 14 15 16 17 18 19 20
21 22 23 24 25 26 27 28 29 30
31 32 33 34 35 36 37 38 39 400
41 42 43 44 45 46 47 48 49 50

OTBeeHHBIM BOnpoc
nponyueHHbI BOnpoc
Tekyuynn BOnpoc

o 3aBepuweHnA TecTa oCTanocb

1:14:46
3aBepunTb TeCT

Scanned with CamScanner


l 98%10:13
avn.kgma.kg E

Quick. Be6.. X

MoxA DawaaH Kg Ru

Bonpoc: N°5
OnComputed Tomography.on the anterior wall of the trachea to the jugular notch of the sternum,
a tumor-like formation extending into the anterior mediastinum is determined. Explain which
organ is affected.

OTBeTbI(ogUH OTBeT)
1
thymus gland.
parathyroid lymph nodes.

perotracheal lymph nodes

4 parathyroid glands.

5 thyroid gland.

2 3 4 50

7 8 9 10
23 456
11 12 13 14 15 16 17 18 19 20
21 22 23 24 25 26 27 28 29 30
31 32 33 34 35 36 37 38 39 40
41 42 43 44 45 46 47 48 49 50

OTBeYeHHbiM BOnpoc

nponyueHHblM Bonpoc
TexyuMn Bonpoc

Ao 3aBepweHWA TeCTa oCTanoCb:

1:14:42
3agepuwTb TeCT

Scanned with CamScanner


NI l 98% 10:13
avn.kgma.kg

Quick.. Be..
MoxA dawaaH a Ru

Bonpoc: N°6
old did not undergo an annual
Patient B., 39 years old, 4 years old could not get pregnant, years
4

of iregular painful periods, The doctor prescribed a medical


medical examination. History
is it better to start the
examination of the pelvic organs. With which diagnostic method
examination?

OTBeTbI(oaMH OTBeT)

magnetic resonance imaging

2 computed tomography

3 endoscopy

4 radiography

5 ultrasound

2 3
45 67 89

1 23 4 5 67 8 9 10
11 12 13 14 15 16 17 18 19 20

21 22 23 24 25 26 27 28 29 30

31 32 33 34 35 36 37 38 39 40
41 42 43 44 45 46 47 48 49 50

OTBeyeHHbIM BONpOC
nponyujeHHBIM BONpoOC

Tekyunn BOnpoc

Ao 3aBepuweHna TeCTa oCTanocb

1:14:36

Scanned with CamScanner


N l 98% 10:13

avn.kgma.kg E

Quick.. Be6... X

MoxA Dan3aH Kg Ru

Bonpoc: Ne7
An excess of thyroid hormones occurs when

OTBeTbI(oAMH OTBeT)

1
Endemic cretinism
2 Insulinoma
3 Acromegaly

4 Diffuse toxic goiter


5 Miksedeme

3 4 5 6 8 10

2 3 6 7 8 9 10
1 45
11 12 13 14 15 16 17 18 19 20
21 22 23 24 25 26 27 28 29 30
31 32 33 34 35 36 37 38 39 40
41 42 43 44 45 46 47 48 49 50

OTBeyeHHbiM BOnpoC
nponyujeHHbIM BOnpOc
TeKyuMn Bonpoc

A0 3aBepweHnA TecTa oCTanocb

1:14:30
3aBepuinTb TeCT

Scanned with CamScanner


N l 98% 10:13

avn.kgma.kg E

Quick.. Be6. X

MoxA Dan3aH Kg Ru

Bonpoc: N°8
A decrease in the production of hormones of the sex glands is manifested:

OTBETbi(oAMH OTBeT)

1
Hirsutism
2 Cryptorchidism

3 Hermaphroditism

4 Virilism

Eunuchoidism

4 5 6
89 10 11 |2 50

1 2 3 4 5 6 7 89 10
11 12 13 14 15 16 17 18 19 20
21 22 23 24 25 26 27 28 29 30
31 32 33 34 35 36 37 38 39 40
41 42 43 44 45 46 47 48 49 50

OTBeyeHHblM BonpoC
nponyujeHHblM Bonpoc
TeEKyuwi Bonpoc

Ao 3aBepueHWA TecTa oCTanoCb

1:14:25
3aBepuinTb TeCT

Scanned with CamScanner


IN
l 98%1 10:13
avn.kgma.kg E

Quick. Be6..

MoxA Dan3aH Kg Ru

Bonpoc: Ne9
Conn's syndrome (primary aldosteronism) is manifested by:

OTBETbI(oaMH OTBeT)
1
Accumulation of H ions
2 Na loss and K delay

3 Hypotension

4 Dliguria

5 Na retention and loss of K

7 9 10 11 12 13 50

1 2 3 4 567 89 10
11 12 13 14 15 16 17 18 19 20
21 22 23 24 25 26 27 28 29 300

31 32 33 34 35 36 37 38 39 40
41 42 43 44 45 46 47 48 49 50

OTBeueHHbi Bonpoc
nponyuyeHHbiM Bonpoc
TeKyujnn BonpoC

0 3aBepweHnA TeCTa oCTanoCb

1:14:21
3aBepwMTb TeCT

Scanned with CamScanner


IN 98% 10:13

avn.kgma.kg

Quick... Be6..

MoxA dan3aH Kg Ru
Bonpoc: N°10
In severe cases of hypothyroidism in adults occurs

OTBETbi(ogMH OTBeT)
1 Eunuchoidism

2 Hypergonadism

Dwarfismn

Myxedema

5 Cretinism

T0 11 12 13 14

50

2 3 45 6 7 8 9 10
11 12 13 14 15 16 17 18 19 20

21 22 23 24 25 26 27 28 29 30
31 32 33 34 35 36 37 38 39 40
41 42 43 44 45 46 47 48 49 50

OTBeueHHbIM Bonpoc
nponyujeHHblM BOnpoc
TeKyumi BonpoC

Ao saBepueHMA TeCTa oCTanocb:

1:14:16
3aBepuunTb TeCT

Scanned with CamScanner


N l 98% 1 10:13
avn.kgma.kg E

Quick. Be6.
MoxA Dan3aH Kg Ru

Bonpoc: Ne11
Primary aldosteronism (Connes syndrome) is:

OTBeTbi(OAMH OTBeT)

1 Tumor of the glomerular zone of the adrenal cortex

2 Tumor of the adrenal medulla

3 Tumor of the reticular adrenal cortex

4 Tumor of the fascicular zone of the adrenal cortex

Increased secretion of aldosterone under the influence of angiotensin

9 0 12 13 14

50

45 6 7 8 9 10
2 3

12 13 14 15 16 17 18 19 20
1
21 22 23 24 25 26 27 28 29 30
31 32 33 34 35 36 37 38 39 40
41 42 43 44 45 46 47 48 49 50

OTBeeHHbiM BOnpoOC
nponyujeHHbIM BOnpoc
Tekyujin BOnpoc

Ao 3aBepweHWA TecTa oCTanoCb

1:14:11
3agepulwTb TeCT

Scanned with CamScanner


l 98% 10:13
O avn.kgma.kg

O Quick.
Be6...
MoxA Oan3aH

Bonpoc: N°12
Hypoparathyroldism is manifested by:

OTBETbi(oaMH OTBEeT)
1 Demineralization of bones

2 Deposition of Ca phosphate in soft tissues

3 Massive
edema

4 Decreased tubular water reabsorption

5 Sharp tetany, bronchospasm

9 10 11 12 13 14 5 16

50

123 4 5 6 78 9 10
12 13 14 15 16 17 18 19 20
21 22 23 24 25 26 27 28 29 30
31 32 33 34 35 36 37 38 39 40
41 42 43 44 45 46 47 48 49 50

oTBeyeHHbIM BOnpoc
nponyuyeHHbiM BOnpoc
Tekyunn BOnpoc

Ao 3aBepuweHs TecTa ocTanocb

1:14:07
3aBepuiTb TeCT

Scanned with CamScanner


INI l 98% 10:13
1 avn.kgma.kg

Qulck Be6 +
MoxA Dana3aH Ru

Bonpoc: N°13
The initial product for the synthesls of thyrold hormones is:

OTBeTbi(ogMH OTBeT)
Phenylalanine

Oxyproline

3 Tyroain

4 Leucine
5 Wallin

9 10 11 2 13 14 15 17

1
2 3 4 56 7 8 9 10
1112 13 14 15 16 17 18 19 20
21 22 23 24 25 26 27 28 29 30
31 32 33 34 35 36 37 38 39 40
41 42 43 44 45 46 47 48 49 50

OTBeuewHbih Bonpoc
aonpoc
nponyuyeHHbin
TeKyWn Bonpoc

Ao 3aBepweHnA TeCTa oCTanocb

1:14:02
GaBepuWTb TeCT

Scanned with CamScanner


N 98% 10:14

avn.kgma.kg

Quick.. Be6.

Moxa dawsaH Kg Ru
Bonpoc: N914
Diabetes insipidus develops when damaged:

OTBeTbi(oAMH OTBeT)
1
Adrenal glands
2 Pancreas
3 Thyroid gland

4 Posterior pituitary gland


5 Anterior pituitary gland

0 11 12 13 14 15 16 17 18

50

1
2 3 4 5 6 7
89 10
1112 13 14 15 16 17 18 19 20
21 22 23 24 25 26 27 28 29 30
31 32 33 34 35 36 37 38 39 40
41 42 43 44 45 46 47 48 49 50

OTBeyeHHblM BOnpoc
nponyueHHbIM BOnpoc
TeKyuMM BOnpoC

Ao saBepweHna TeCTa oCTanoCb

1:13:53
3aBepunTb TeCT

Scanned with CamScanner


INI 98% 10:14

P9 vn
avn.kgma.kg C E
OQuick.. Be6.. X

MoxA ban3aH Ru

Bonpoc: N°15
Patient B, 50 years old, complains that recently the ears, nose, hands began to increase in size
Which gland is overactive will give similar aymptoms;

OTBeTbI(onMH OTBeT)

Gonads

2 Pituitary gland
Thyroid

4 Adrenalglands

5 Epiphysis

11 12 13 14 15 16 17 18 19

50

345 678 9 10
1112 13 14 15 16 17 18 19 20
21 22 23 24 25 26 27 28 29 30
31 32 33 34 35 36 37 38 39 40
41 42 43 44 45 46 47 48 49 50

OTBeyeHHbIM BOnpoc
nponyujeHHbin Bonpoc
TeKyuMM BOnpoc

Ao 3aBepweHWA TecTa oCTanoCb

1:13:49
3ARenillTh TACT

Scanned with CamScanner


N 98%10:14
avn.kgma.kg

Quick. Be
..
MoxA Daw3aH Kg Ru
Bonpoc: NP16
Patient N., 27 years old, after childbirth, complicated by massive bleeding, began to develop
exhaustion, the skin became dry, wrinkled, wax color. Body temperature 360 C, blood pressure-
100/60 mm Hg. Art, the content of glucose in the blood is 3.3 mmol /1, the content of 17-
ketosteroids in the urine is lowered. What pathology of the endocrine system can be assumed:

OTBETbi(oaMH OTBeT)
1 Pituitary hyperfunction

2 Hypothyroiditis

3 Hyperfunction of the adrenal cortex

4 Hypofunction of the ovary

5 Pituitaryhypofunction

2 13 4 5 16 7 18 19 20

50

4 8 9 10
1112 13 14 15 16 17 18 19 20
21 22 23 24 25 26 27 28 29 30
31 32 33 34 35 36 37 38 39 40
41 42 43 44 45 46 47 48 49 50
OTBeeHHbiM BOnpoc
nponyujeHHbiM Bonpoc
TeKyujnn Bonpoc

Ao 3aBepweHWA TECTa oCTanocb

1-13:43

Scanned with CamScanner


l 98% 10:14

avn.kgma.kg

Quick.. Be6..

MoxA Dan3aH Kg Ru
Bonpoc: N°17
Patient N., 45 years old, complains of weakness, rapid fatigue; lack of appetite, weight loss, pain
in the abdomen. Objective examination: the skin and visible mucous membranes of bronze color,

blood pressure is reduced. Dysfunction of which endocrine gland can cause the following
symptoms:

OTBeTbI(oaMH OTBeT)
1
Pituitary gland

2 Adrenal glands

3 Epiphysis

4 Pancreas
5 Ovary

13 14 5 16 17 18 19 20 21

50

2
456 7 8 9 10
11 12 13 14 15 16 17 18 19 20
21 22 23 24 25 26 27 28 29 30
31 32 33 34 35 36 37 38 39 40
41 42 43 44 45 46 47 48 49 50

OTBeeHHbiM BOnpoc
nponyujeHHbiM BOnpoc
TexyuMn BOnpoc

Ao 3aBepuweHna TecTa oCTanocb

1:13:39

Scanned with CamScanner


N l 98% 10:14
K avn.kgma.kg O

Quick. Be6..

MoxA Dan3aH Ru
Bonpoc: N°18
After removal of the thyroid gland, the patient developed convulsions. What was damaged during
the operation:

OTBETbi(oguH OTBeT)
1 Inferior laryngeal artery

2 Inferior laryngeal nerve

3 Sympathetic trunk

Vagus nerve

5 Parathyroid gland

4 6 17 18 9 20 21 22

50

1
23 4 5 678 9 10
11 12 13 14 15 16 17 18 19 20

22 23 24 25 26 27 28 29 30
31 32 33 34 35 36 37 38 39 40
41 42 43 44 45 46 47 48 49 50

OTBeyeHHbIM BOnpoc
nponyujeHHbIM Bonpoc
TekyuMn BOnpoc

0 3aBepweHWA TeCTa oCTanocCb

1:13:35
RarenilUTh TeCT

Scanned with CamScanner


IN 98% 10:14
avn.kgma.kg

Quick. Be6.
MoxA DaM3aH Kg Ru
Bonpoc: N°19
What change in the biochemical composition of the blood is observed with an overdose of
insulin:

OTBETbi(ogMH OTBeT)

Decreased protein content

2 Decreased phosphorus levels


3 Increased calcium content

OLowering sugar levels

5 Increased sugar levels

16 17 18 19 20 21 2 23

50

10
3 67 89
11 12 13 14 15 16 17 18 19 20
21 22 23 24 25 26 27 28 29 30
31 32 33 34 35 36 37 38 39 40p
41 42 43 44 45 46 47 48 49 50

OTBeeHHbIM BOnpoc
nponyyeHHbIM Bonpoc
TeKyuMM Bonpoc

Ao 3aBepweHWA TeCTa oCTanocb

1:13:30
3aRenilITh TeCT

Scanned with CamScanner


N l 98% 10:14
avn.kgma.kg

Quick.. Be6...

MoxA Dan3aH Kg Ru
Bonpoc: N 20
Patient L., 45 years old, has been ill
for 6 months, the onset of the disease is associated with the
transferred sore throat. Complaints of general weakness, irritability, sweating, increased appetite,
drop in body weight, palpitations. On physical examination, the body temperature is
37.8
hyperhidrosis (ncreased sweating), tremor of the fingers, increased gitter of the eyes, diffuse
enlargement of the thyroid gland, small exophthalmos (bulging eyes), pulse 100 beats. per
minute, blood pressure- 140 mm Hg. Art., the level of glucose in the blood-5.5 mmol/1,
the
basal metabolic rate is increased by 40%. Diagnosed with Basedow's disease.
Indicate the
function of which endocrine gland is impaired:

OTBeTbi(oAMH OTBeT)
1
Parathyroid
2 Pituitary gland

Thyroid

4 Adrenalglands

5 Epiphysis

17 18 19 20 21 22 23 24

50

2 34 5 67 89
11 12 13 14 15 16 17 18 19 20
21 22 23 24 25 26 27 28 29 30
31 32 33 34 35 36 37 38 39 40
41 42 43 44 45 46 47 48 49 50

OTBeyeHHbiM BOnpoC
nponyuyeHHbiM BOnpoc
Tekyuin BOTpoc

Scanned with CamScanner


N dl 98% | 10:14
avn.kgma.kg

Quick. Be
.. X

Moxa dansaH Kg Ru

Bonpoc: N°21
Male 45 years old. The face is moon-shaped, the skin on it with a purple tint, acne. Excessive fat
deposition on the abdomen and thighs. The bones are thin. Thinning of the skin, red streaks of
stretch on the skin of the abdomen and shoulders are noted. Blood pressure 160/90 mm Hg. Art.
Blood sugar 9.0 mmol /1. Xray: the Turkish saddle is dilated. Which of the following preliminary
diagnoses is most likely:

OTBeTbi(OAMH OTBeT)

1 Myxedema

2 Pheochromocytoma

OCushing's Disease

Addison's disease

5 Diffuse toxic goiter

17 18 19 20 21 22 23 2425

50

1
2 3 4 56 7 89 10
11 12 13 14 15 16 17 18 19 20
21 22 23 24 25 26 27 28 29 30
31 32 33 34 35 36 37 38 39 40
41 42 43 44 45 46 47 48 49 50

OTBeyeHHbiM BOnpoC

nponyueHHbiM BOnpoc
TeKyumn BOnpoc

Ao saBepweHWA TeCTa oCTanocb

Scanned with CamScanner


IN l 98%10:14
avn.kgma.kg

Quick. Be6.. X

MoxA Daw3aH R

Bonpoc: N 22
A 30-year-old man complains of recurrent seizures of the muscles of the limbs and face,
numbness of the fingers and toes, pain in the chest and in the epigastric region, difficulty in
breathing. The disease developed after removal of part of the thyroid gland for nodular goiter
The patient is pale. The hair is sparse, gray. Britte nails. Neurological examinationa sharp
increase in the excitability of the nervous and muscular systems. The content of Ca in the blood
is significantly reduced, the content of phosphorus is increased. Indicate which endocrine glands
function is impaired:

OTBETbI(OAMH OTBeT)
1 Adenohypophysis
2 Adrenal glands
3 Parathyroid

Neurohypophysis

Thyroid

18 19 20 21 22 24 26

50

23 4 567 8 109
11 12 13 14 15 16 17 18 19 20
21 22 23 24 25 26 27 28 29 30
31 32 33 34 35 36 37 38 39 40
41 42 43 44 45 46 47 48 49 50

OTBeeHHbiM BOnpoc
nponyueHHbin BOnpoc
rekyunn Bonpoc

Scanned with CamScanner


I 98% 10:14

K avn.kgma.kg

Quick.. Be6. X

MoxA oansaH Kg Ru
Bonpoc: N°23
Indicate the blood glucose level in healthy children (mmol/):

OTBETbI(OAMH OTBeT)

1 8,89 -9,99
2 1,44-2,22

3 7,78-8,88
4 6,66-7,77

5 3,33-5,55

20 21 22 23 24 26 27
*

50

1
2 3 4 5 6 7 8 9 10
1 12 13 14 15 1b 17 18 19 20
21 22 23 24 25 26 27 28 29 30
31 32 33 34 35 36 37 38 39 40
41 42 43 44 45 46 47 48 49 50
OTBeHeHHbIM BOnpoC
nponyujeHHbiM Bonpoc
Texyunn BonpocC

40 3aBepuieHnA TecTa octanocb

1:13:11
3asepiMTb TeCT

Scanned with CamScanner


MIY7 98% 10:14

avn.kgma.kg E
Quick Be6.

MoxA daw3aH Ru

Bonpoc: N°24
Name the main laboratory criterion of hyperglycetmia syndrome

OTBeTbi(ogMH OTBeT)
1 hypernatriemia

2 hypokaliemia

3 hypoproteinemia

4 high fasting glycemia

5 hypercholesterolemia

20 21 22 23
24 25 26 27 28

1
2 3 4 5 6 7
89 10
11 12 13 14 15 16 17 18 19 20
2122 23 24 25 26 27 28 29 30
31 32 33 34 35 36 37 38 39 40
41 42 43 44 45 46 47 48 49 50

OTBeYeHHbiM BOnpoc
nponyujeHHbIM BOnpoOc
TeKyuyMn Bonpoc

Ao 3aBepweHWA TecTa oCTanocb

1:13:03
3aBepuiWTb TeCT

Scanned with CamScanner


98% 10:14
avn.kgma.kg

Quick.. Be6.
MoxA daM3aH Kg Ru
Bonpoc:N°25
Describe the stool for hypothyroidism in early childhood:

OTBeTbi(oAMH OTBeT)
1 persistent constipation
2 frequent with blood
3 form of rice water

4 frequent, frothy with mucus


5
form as pea puree

21 22 23 24 25 26 27 28 29

50

1
2 3 4 5 67 89 10
11 12 13 14 15 16 17 18 19 20
21 22 23 24 25 26 27 28 29 30
31 32 33 34 35 36 37 38 39 40
41 42 43 44 45 46 47 48 49 50

OTBeueHHbiM BOnpoC
nponyuyeHHbiM Bonpoc
TeKyuMM BOnpoc

40 3aBepuieHWA TeCTa oCTanoCb

1:12:58
3aBepuiTb TeCT

Scanned with CamScanner


IN 98% 10:14

avn.kgma.kg

Quick. Beo..

MoxA daM3aN Kg Ru

Bonpoc: N°26
Explain the cause of prolonged jaundice in babies with hypothyroidism

OTBeTbi(oqMH OTBeT)
1 imperfect function of the gallbladder

2 increased kidney function

3 insufficient secretion of bile by the liver

4 deficiency of vitamin A in the body

5 mucous swelling of the skin

22 3 25 26 27 28 29

50

1
2 3 456 7 8 9 10
11 12 13 14 15 16 17 18 19 200

2122 23 24 25 26 27 28 29 30
31 32 33 34 35 36 37 38 39 4

41 42 43 44 45 46 47 48 49 50

OTBeHeHHbIM BOnpoc
nponyujeHHbIM BOnpoc
TeKyuwn Bonpoc

Ao 3aBepweHwA TeCTa oCTanocb

1:12:54
3aBepuMTbTeCT

Scanned with CamScanner


TE l 98% | 10:15
avn.kgma.kg

Quick.. Be.. X

Moxa Dan3aH Ru
Bonpoc: N°27
Explain, as a result of which there is overweight in chldren with hypothyroidism syndrome:

OTBeTbi(oquH OTBeT)
1
acceleration of oxidative processes

2 disaccharide metabolism disorders

3 myxedema edema
4 violation of fat metabolism

liverdysfunction

23 24 25 26 27 20 29 30 31

1 2 3 45 6 7 8 9 10
11 12 13 14 15 16 17 18 19 20
21 22 23 24 25 26 27 28 29 30
31 32 33 34 35 36 37 38 39 40
41 42 43 44 45 46 47 48 49 50

OTBeyeHHbiM BOnpoc
nponyueHHbIM BOMpoc
TekyuM BOnpoc

Ao 3aBepueHna TeCTa oCTanocb

1:12:49
3aBepuiVTb TeCT

Scanned with CamScanner


N Tl 98% 10:15
avn.kgma.kg

9 Quick. Be6.. x
MoxA Dan3aH Kg Ru

Bonpoc: N°28
A patient gir, 12 years of life, turned with mother to the family'doctor with complaints on hunger,
rapid fatique, dizziness, pain in abdomen, hand tremors. Anamnesis: suffers from diabetes
melfutus 1 type/ since of 9, receives insulin therapy. 4 days ago, she began to attend training
Sessions for 2 hours daily. This complaints appeared 2 days ago, very tired She tries to follow the
diet.prepares herself. She studies well. Objectively: physical development corresponds to age
Theskin is palor, dry cold hands. Heart rate-82 in 1 minute. Arterial pressure 110/70 mm Hg
No thirst, does not urinate often. Blood glucose level is 4,2 mmol/l. In urine no glucose. Decide,
which syndrome the patient has?

OTBeTbi(oAMH OTBeT

1 intoxication

2 hypoglycemia

3 premenstrual
hyperthyroidism

5 "attention deficit"

24 25 26 27 28 29 30 31
32
50

1 2 3 4 5 67 8 9 10
I1 12 13 14 15 16 17 18 19 20
21 22 23 24 25 26 27 28 29 30
31 32 33 34 35 36 37 38 39 40
41 42 43 44 45 46 47 48 49 50

OTBeYeHHbiM BOnpoc
nponyueHHbin Bonpoc
TeKyuunn BOnpoc

Scanned with CamScanner


NI 98% 8 10:15

avn.kgma.kg E
Quick. Be6... X

MoxA Dan3aH Kg Ru
Bonpoc: N°29
What symptom is characterized by increasing the iritability of the nerves, and a flexlon of the
wrist and metacarpat phalangeal joints with extension of the interphalangeal joints and
adduction of the thumb (carpal spasm) after placing a blood pressure cuff on the patients arm
and inflating to 20 mm Hg above systolic blood pressure for 3-5 minutes?

OTBETbi(oAMH OTBeT)

1 Weiss
2 Schlesinger

Trousseau

Khvostek

5 "fish mouth

25 26 27 28 29 30 31 32 33

50

12 3 4 5 67 8 9 10
11 12 13 14 15 16 17 18 19 20
21 22 23 24 25 26 27 28 29 30
31 32 33 34 35 36 37 38 39 40
41 42 43 44 45 46 47 48 49 50

OTBeyeHHblM BOnpoc
nponyujeHHbiM BOnpoc
TEKyuMn BOnpo

Ao 3aBepuweHMs TecTa oCTanocb

1:12:37

Scanned with CamScanner


IN dl 98% 1015
avn.kgma.kg O
Quick. Be .
MoxA Da3aH

Bonpoc: N°30
Obesity is a risk factor for the development of the syndrome

OTBeTbi(oaMH OTBeT)
1 hyperparathyroidism
2 hyperthyroidism

gigantism

4 hypercortisolism

5
hyperglycemia

26 27 28 29 30 31 32 33 34

50

2 3 4 5 6 7 8 9 10
11 12 13 14 15 16 17 18 19 20
21 22 23 24 25 26 27 28 29 30
31 32 33 34 35 36 37 38 39 40
41 42 43 44 45 46 47 48 49 50
OTBeYeHHbIM BOnpoc
nponyuleHHbIM BOnpoc
Texyun BOnpoc

J0 3aBepuweHnA TeCta oCTanocb

1:12:32
3aBepuwTEb TeCT

Scanned with CamScanner


IN l 98% 10:15
avn.kgma.kg

Quick... Be6.. X

MoxA dan3aH RU
Kg

Bonpoc: N°31
ndicate, what change in the cardiovascular system is most typical for the obesity syndrome?

OTBeTbi(oAMH OTBeT)
1 pulse soit, low tension

2 apical impulse is weakened, low

3 displacement of the right border

arrhythmia of heart sounds

increase blood pressure

28 29 30 31 32 34 35

50

2 3 4 56 8 910
11 12 13 14 15 16 17 18 19 20
21 22 23 24 25 26 27 28 29 30
31 32 33 34 35 36 37 38 39 40
41 42 43 44 45 46 47 48 49 50

OTBeyeHHbiM BOnpoc
iponyujeHHbln Bonpoc
TeKyunn Bonpoc

Ao saBepweHWA TeCTa oCTanocb

1:12:27
3aBepuWTb TeCT

Scanned with CamScanner


INIR 989% 10:15

avn.kgma.kg

9 Quick.. Be6.. X

Moxn oan3aH Ru

Bonpoc: N°33
Freethyroxine is normal (pmol/1)?

OTBETbi(oqMH OTBeT)

1 from 12.0 to 29.0

2 from 9.0 to 22.0

3 from 7.0 to 20.0

4 from 10.0 to 26.0

from 6.0 to 15.0

29 30 31 32 33 34 35 36

50

1
2 3 45 6 8
TT 12 13 14 15 1b 17 18 19 20
21 22 23 24 25 26 27 28 29 30
3132 33 34 35 36 37 38 39 40
41 42 43 44 45 46 47 48 49 50

OTBeueHHbiM BOnpoc
nponyujeHHbIM BOnpoc
TeKyujnn Bonpoc

o 3aBepuieHMA TecTa ocTanocb

1:12:18
3aBepuwTb TeCT

Scanned with CamScanner


I l 98% 1015
avn.kgma.kg

Quick. Be . X

Moxa dan3aH Kg Ru

Bonpoc: N°34
Obesity 1st degree, the body mass index (kg/ m2) is

OTBETbI(oaMH OTBeT)

1 40.0-45.9

2 35.0-39.9

3
18.5-24.9
4 30.0-34.99

5 25.0-29.9

30 32
33 34 35 36 37 38

50

1 2 3 45 6 7 89 10
11 12 13 14 15 16 17 18 19 20
21 22 23 24 25 26 27 28 29 30
31 32 33 34 35 36 37 38 39 40
41 42 43 44 45 46 47 48 49 50
OTBeYeHHbiM BOnpoc
nponyujeHHbIM BOnpoc
Tekyuni Bonpoc

Ao 3aBepweHna TeCTa ocTanocb

1:12:13
3aBepunTb TeCT

Scanned with CamScanner


N l 98%10:15
avn.kgma.kg O E

Quick. Be6.. X +
Kg Ru
Moxg dawsaH

Bonpoc: N°35
Obesity 2nd degree, the body mass index (kg/ m2)is

OTBeTbI(oAMH OTBeT)

25.0-29.9

2 30.0-34.9

3 18.5-24.9

4 35.0-39.9

5 40.0-45.9

31 32 33 34 35 50 37 38 39

1
2 3 4 5 6 7 89 10
11 12 13 14 15 16 17 18 19 20

21 22 23 24 25 26 27 28 29 30
31 32 33 34 35 36 37 38 39 40
41 42 43 44 45 46 47 48 49 50

oTBeyeHHblM BOnpoc
nponyujeHHblM BOnpoc
Tekyuwn Bonpoc

Ao 3aBepuweHWA TeCTa oCTanocb

1:12:09
3aBepuwTb TeCT

Scanned with CamScanner


98% 10:15
N l
o avn.kgma.kg

Quick. Be6. X

MoxA Dan3aH Kg Ru

Bonpoc: Ne36
Waist circumference in women is normally
less than (cm)

OTBETbI(oqMH OTBeT)

1 75

2 BU

3 B5

4 665

5 70

37 38 39 40
32 33 34 35 36

50

1
2 3 4 5 6 7 8 9 10
11 12 13 14 15 16 17 18 19 20
21 22 23 24 25 26 27 28 29 30
31 32 33 34 35 36 37 38 39 40
41 42 43 44 45 46 47 48 49 50
OTBeYeHHbiM BOnpoc
nponyuyeHHbIM BONpocC
TeKyynn Bonpoc

Mo 3aBepweHWa TeCTa oCTanocb

1:12:04
3aBepuiwTb TeCT

Scanned with CamScanner


98%10:15
I
avn.kgma.kg

Quick. Be6.
Moxp Daw3aH Kg Ru

Bonpoc:N 37
(according to ultrasound) in women is (ml
Normally, the volume of the thyroid gland

OTBETbi(oaMH OTBeT)

from 35 to 38

2 more than 50

3 from 25 to 30

4 Up to 18

5 from 40 to 45

38 41
33 3435 3637 39 40

50

2 3 4 5 67 8 9 10
11 12 13 14 15 16 17 18 19 20
21 22 23 24 25 26 27 28 29 30
31 32 33 34 35 36 37 38 39 40

41 42 43 44 45 46 47 48 49 50

OTBeyeHHbi BOnpoc
nponyueHHbIM BOnpoC
TeKyn Bonpoc

Ao 3aBepueHna TeCTa oCTanocb

1:12:00
3aBepunTb TeCT

Scanned with CamScanner


N

ll 98% 10:15

avn.kgma.kg E

Quick.. Be6.. +
MoxA Dan3aH Kg Ru

Bonpoc: N°38
In hypercortisolism syndrome, skin changes are characterized by

OTBETbi(oAMH OTBeT)

1 yellowness
2 vitiligo
3 striae

4 darkening

5 myxedema

34 35 36 37 38 9 40 41 42

50

2 3 4 5 6 7 8 9 10
11 12 13 14 15 16 17 18 19 20
21 22 23 24 25 26 27 28 29 30
31 32 33 34 35 36 37 38 39 40
41 42 43 44 45 46 47 48 49 50

OTBeeHHbi BOnpoc
nponyujeHHbl BOnpoc
TexyuMn Bonpoc

Ao 3aBepueHMA TecTa oCTanoCb

1:11:54
3aBepunTb TeCT

Scanned with CamScanner


INI 98% 10:165

avn.kgma.kg

Quick.. Be6. X

MoxA daw3aH Kg

Bonpoc: N 39

First aid for hypoglycemia, what kind of remedy is taken?

OTBETbI(OgMH OTBeT)
1
calcium chloride 100 mng

2 glucose 40%
3 insulin 4 unit
4 metformin 500 mg

5 furosemide 40 mg

35 36 37 38 39 40 41 42 43

50

1 2 3 4 5 67 8 9 10
11 12 13 14 15 16 17 18 19 20
21 22 23 24 25 26
27 28 29
31 32 33 34 35 36 37 38 39 40
41 42 43 44 45 46 47 48 49 50

OTBeyeHHbI BOonpoc
nponyujeHHbi Bonpoc
TeKyuMn BOnpoc

Ao 3aBepweHWA TecCTa oCTanoCb

1:11:50
3aBepuwTb TeCT

Scanned with CamScanner


IN l 98%1016
avn.kgma.kg

Quick. Be6.. x
Moxa daM3aH
Kg
Ru

Bonpoc: N940
A 57-year-old woman was admitted to the endocrinology department of a clinical hospital with
complaints of weakness, headache, dry mouth, increased blood pressure up to 160/90 mm Hg,
pain in the region of the heart. in the lumbar spine during exercise. Objectively rounded face,
cheek rubeosis, height- 167 cm, weight 92 kg. BMI 34.0 kg /m2, waist circumference- 120
cm, hip circumference 100 cm, pale striae on the skin of the abdomen and thighs. Determine
what is the degree of obesity by body mass index?

OTBeTbI(oAMH OTBeT)
1
0
2

36 3738 39
4041 42 43 44

50

123 4 5 6 7 89 10
11 12 13 14 15 16 17 18 19 20
21 22 23 24 25 26 27 28 29 30
31 32 33 34 35 36 37 38 39 40
41 42 43 44 45 46 47 48 49 50

OTBeyeHHbIM BOnp0C
nponyuyeHHblM BOnpoc
TeKyuiM* Bonpoc

ao anoADULAuAa TAGTO ACTAnAAL

Scanned with CamScanner


98%1 1016
avn.kgma.kg E
Quick... Be6.

Moxp danaa4 Kg Ru
Bonpoc: N 41
A 40-year-old man has turned to a polyclinic therapist with complaints of headaches, vWeight gain,
weakness, palpitations. The above complaints are disturbing for 1 year after suffering a
traumatic brain injury. On examination: the face is rounded, acne, welght 110 kg, height- 162
cm, enlargement of the breasts, purple-red streaks of stretching of the skin on the abdomen,
thighs, thinning of the arms and legs. Find out from the description the objective syrnptomis of
the patient.

OTBETbi(opuH OTBeT)
exophthalmos, myxedema

fish mouth, obstetrician hand

3 macrognathia, diastema
4 gynecomastia, striae
5 hirsutism, macroglossia

37 38 39 41 42 3 44 45

12 3 4 5 6 78 9 10
11 12 13 14 15 16 17 18 19 20
21 22 23 24 25 26 27 28 29 30
31 32 33 34 35 36 37 38 39

142 43 44 45 46 47 48 49 50

OTBeeHHbIMBOnpoc
nporlyyeHHbiM BOnpoc
TeKyum- Boipoc

Scanned with CamScanner


l 98% | 10:16
avn.kgma.kg E

Quic Be6.... X

MoxA Da3aH Kg Ru

Bonpoc: Ne42
The man was admitted to the clinic, complaints of palpitations, interruptions in the work of the
heart, sweating, trembling in the body, weakness, Anamnesis: has been ill for several years.
Examination: the apical impulse is enhanced. Heart sounds are loud, arrhythmic. Pulse 98 in
minute. blood pressure 170/60 mm Hg. ECG: tachysystolic form of atrial fibrillation. Your
suspected syndrome?

OTBeTbI(ogMH OTBET)
1
hyperglobinemia

2 hyperthyroidism
3 hyperparathyroidism

4 hypercorticism
5 hyperglycemia

38 39 40 41 42 44 45

34 5 678 9 10
11 12 13 14 15 16 17 18 19 20
21 22 23 24 25 26 27 28 29 30
31 32 33 34 35 36 37 38 39 40
41 42 43 44 45 46 47 48 49 50
OTBeveHHbiM BOnpoC

nponyuyeHHbin BOnpoc
TeKyunn Bonpoc

Ao 3aBepweHMA TeCTa oCTanocb

Scanned with CamScanner


l 98%1 10:16
avn.kgma.kg

Quick.. Be6..

MoxA DaM3aH Kg Ru
Bonpoc: N°43
The man40 year old was admitted to the clinic, complaints of palpitations, interruptions in the
Work of the heart, sweating, trembling in the body, weakness. Anamnesis: has been ill for several
years. Examination: the apical impulse is enhanced. Heart sounds are loud, arrhythmic. Pulse 98
in 1 minute. blood pressure 170/60 mm Hg.ECG: tachysystolic form of atrial fibrillation. The
doctor diagnosed hyperthyroidism syndrome. Explain, what caused the atrial fibrillation in this
case, because of the increase?

OTBeTbi(ogMH OTBeeT)
1
calcitonin
2 parathyroid hormone
3 increased TSH
4 antibodies TPO
13 and T4

40 41 42 43 44 45

50

23 4 6 8 9 10
11 12 13 14 15 16 17 18 19 20
21 22 23 24 25 26 27 28 29 30
31 32 33 34 35 36 37 38 39 40
41 42 43 44 45 46 47 48 49 50

OTBeyeHHbiMBOnpoc
nponyujeHHbin Bonpoc
TeKyuyM: BOnpocC

o aapanuiLAn TATA AOTAnO

Scanned with CamScanner


Bonpoc: N°38
Which drug should be prescribed for acute hyperglycemia?

OTBETbI(oqnH OTBer)
1 gliclazide

2 short-acting insulin

3 long acting insulin

4 O glibenclamide
5 metformin

34 35 36 3738 39 40 41 42

50
BepMaCapseu Ko

Bonpoc: N936
Obesity is often observed in the syndrome

OTBeTbI(oquH OTBeT)
1 hypothyroidism

2 hypoglycemia

3 hypopituitarism
4 hypoparathyroidism

5 hypocorticism

32 33 34 35 36 37 38 39 40

50

asLLAGAaAV

2 3
1
4 5 6 7 8 9 10
11 12 13 15 16 17 18 19 20
14

21 22 2324 25 26 27 28 29 30
31 32 33 3435 36 37 38 39 40
41 42 43 44 45 46 47 48 49 50

OTBeYeHHbIÅ Bonpoc
nponyyeHHbIM Bonpoc
TeKyuin Bonpoc

Ao 3aBepuieHwa TeCTa ocTanocb

0:23:40
3apepuwTb TeCT
Bonpoc: N933
The most common cause of primary hypocorticism is

OTBeTbl(OAMH OTBeT)
1
0 meningitis

2 tuberculosis

3 corticotropinoma

4 hypophysectomy

5 Ocorticosteroma

1 29 30 31 32 33 34 35 36 37

50
Bonpoc: N°27
Explain the reason for the large tongue in a child with hypothyroidism:

OTBeTbi(oAMH OTBeT)
1 disorder of the musculature of the tongue

2 mucous swelling of the tongue

3 disorder ofthe salivary glands

4 innervation disorder

5
impaired circulation of the oral cavity

1
23 24 25 26 27 28 29 30 31

50

1 2 3 4 5 6 7 8 9 10
11 12 13 14 15 16 17 18 19 20
21 22 23 24 25 26 27 28 29 30
31 32 33 34 35 36 37 38 39 40
41 42 43 44 45 46 47 48 49 50

OTBeeHHbIM Bonpoc
nponyeHHblM Bonpoc
TeKyuyn BONpOcC

Ao 3aBepuieHuA TECTa oCTanocb:

0:32:52
3aBepuWTb TeCT
Bonpoc: N°13
Dysfunction of the kidneys in thyrotoxicosis may manifest itself:
T

OTBETbi(ogMH OTBeT)
1 Impaired reabsorption of Ca and phosphorus -

Concentration dysfunction

3 Impaired renal blood flow

Increasing the filtration capacity of the kidneys

5 Proteniuria

9 10 11 12 13 14 15 16 17

50

1 2 3 45 6 78 9 10
11 12 1314 15 16 17 18 19 20
21 22 23 24 25 26 27 28 29 30
31 32 33
34 35 36 37 38 39 40
41 42 43 44 45 46 47 48 49 50
OTBeeHHbIM BOnpoc
nponyueHHbIM BOnpoc
Bonpoc: N 11
With eosinophilicadenoma of the pituitary gland during the growth period of the body develops:

OTBETbi(OAMH OTBET)

1 Gigantism
2 Pituitarydwarfism

3 Cushing'sDisease

4 Dysplasia

5 Acromegaly

7B9 10 12 13 14 15 .

50

1 2 34 5 6 7 89 10
12 13 14 15 16 17 1819 20
21 22 23 24 25 26 27 28 29 30
31 32 33 34 35 36 37 38 39 40
41 42 43 44 45 46 47 48 49 50

OTBeHeHHbiM Bonpoc
nponyueHHbIM BOonpoc
TEKyun Bonpoc
BepMa CapBeu Kg Ru

Bonpoc: Ne10
An increase in the concentration of thyroid-stimulating hormone in the blood during
hypothyroidism indicates the localization of the pathological process in:

OTBeTbi(OAMH oTBET)
1 Hypothalamus

2 Thyroid gland

3 Timuse
4 Parathyroid glands

5 Pituitary gland

6
7 8 9 10 11 12 13 14
50

1 2 3 45 6 7 89
11 12 13 14 15 16 17 18 19 20
21 22 23 24 25 26 27 28 29 30
31 32 33 34 35 36 37 38 39 40
41 42 43 44 45 46 47 48 49 50
Bonpoc: N°1
Which of the following radiation methods is the most informative in the diagnosis of kidney
cysts:

OTBeTbl(oAMH 0TBET)
1 ultrasound

2 dynamic tomoscintigraphy
3 overview X-ray of the urinary tract

4 excretory urography

5
cystograpny

1 2 3 4 5 6 50
BepMa Capse Kg
Ru
Bonpoc: N940
Patient N., 35 years old, in hospital, a day after strumlectomy (complete or partial removal of the
thyroid gland) showed signs of hypertonicity of the symmetrical muscle group: spasm of the
facial muscles (in the form of a "sardonic smile"), lips in the form of a "fish mouth', trismus of the
chewing muscles. When the light is turned on, cramps in the muscles of the upper extremities
additionally occur a symptom of an obstetrician's hand. In which syndrome are the above
symptoms determined?

OTBETbi(OquH OTBeT)
1 hypopituitarism syndrome

2 hypoglycemic syndrome

3 hypoparathyroidism syndrome
4 hypothyroidism syndrome

5 hypocorticism syndrome

1 36 37 38 39 40 41 42 43 44

50
Endocrine system questions

Q1.An increase in the concentration of thyroid-stimulating hormone in the blood during


hypothyroidism indicates the localization of the pathological process in:

1.Parathyroid glands

2.Pituitary gland

3.Thyroid gland

4.Timuse

5.Hypothalamus

Q2. With an insufficient amount of iodine in food, it develops:

1.Hyperthyroidism

2.Hypoparathyroidism

3.Endemic goiter

4.Autoimmune thyroiditis

5.Diffuse toxic goiter

Q3.A decrease in the production of hormones of the sex glands is manifested:

1 Hirsutism

2.Virilism

3.Eunuchoidism

4.Cryptorchidism

5.Hermaphroditism

Q4.Changes in oxytocin secretion play a role in pathogenesis:

1.Disorders of circadian rhythms "wakefulness - sleep"

2.Myxedema
3.Disorders of labor

4.Diffuse toxic goiter

5.Disorders of carbohydrate metabolism in

diabetes mellitus

Q5.In the onset of Basedow's disease, the main role belongs to:

1.Increased production of cortisol

2.Insufficient action of parathyroid hormone

3.Insufficient aldosteron production

4.Increased production of somatotropic hormones

5.Excessive action of thyroid hormones

Q6.The pathogenesis of tertiary hypothyroidism is due to:

1 Increased TSH secretion

2.Increased synthesis of FSH

3.Secretion of biological inactive TSH

4.Autoimmune process in the thyroid gland

5 Decrease in the synthesis of thyroliberin

Q7.Patient B., 50 years old, complains that recently the ears, nose, hands began to increase in
size. Which gland is overactive will give similar symptoms:

1.Gonads

2 Thyroid

3 Adrenalglands

4.Epiphysis
5.Pituitary gland

Q8.Patient A., 39 years old, complains of copious flow of urine (daily urine output is 22 liters per
day). Blood sugar is within the normal range. With a violation of the production of which
hormone polyuria is associated:

1.Glucagon

2.ADH - Vasopressin

3.Aldosterone

4.Insulina

5.Parathyroid hormone

Q9.A patient came to the appointment - thin, with pronounced exophthalmos (bulging eyes),
pronounced anxiety in behavior. Heart rate (HR) - 92 beats / min. Dysfunction of which
endocrine gland can cause the following symptoms:

1.Timus

2.Pituitary gland

3 Adrenalglands

4.Parathyroid

5.Thyroid

Q10.A patient was brought to the admission department of the hospital, who lost consciousness
in the street. The examination revealed the smell of acetone from the mouth. What is the
preliminary diagnosis that can be made:

1.Uremic coma

2.Diabetic coma

3.Pulmonary embolism

4.Internal bleeding

5.Hepatic coma
Q11.The patient has clinically all the signs of thyrotoxicosis. What research needs to be done to
clarify the diagnosis:

1.General analysis of blood and urine

2.Blood test for sugar, ketone bodies

3.LDL, HDL, bloodcholesterol

4.protein composition of plasma, residual blood nitrogen,

5.Determine the level of thyroxine

Q12.Female, 52 years old, onset of the disease 1.5 years ago after a course of radiation
therapy. Complains of memory impairment, general weakness, drowsiness, baldness, lack of
appetite, weight gain. An objective examination revealed: the skin is dry, flaky, the face is puffy,
hypomimic, there are areas of baldness on the head, the voice is low, rough, speech is slowed
down, the body temperature is 35.7 °, the pulse is 52 beats. per minute, blood pressure - 110/65
mm Hg. Art., the level of glucose in the blood 3.5 mmol / I, the basal metabolism is reduced by
30%. Diagnosed with Myxedema. What is the most likely cause of this disease:

1.Formation of mucin

2 Formation of reverse T3

3.Decreased TSH levels

4.Increase in T3 T4

5.lodine deficiency

Q13.Male 30 years old. He was admitted to the clinic with complaints of headache, drowsiness,
change in appearance over the past 2 years, during which he considers himself ill. Examination:
an increase in the brow ridges, chin, nose, auricles, tooth divergence. The hands and fe are
disproportionately enlarged. AD 160/95 mm Hg, heart rate - 90 beats per minute, an increase in
the size of the liver, the borders of the heart are expanded, blood glucose - 10 mmol / I, total
protein - 90 g/l, urine glucose +. Which of the following preliminary diagnoses is most likely:

1.Pituitary hypogonadism

2.Pituitary dwarfism

3.Acromegaly
4.Adiposogenitaldystrophy

5.Pituitary gigantism

Q14.Woman 44 years old. She suffers from diffuse toxic goiter and has been taking tyrosol 5
mg for 6 months. The condition improved, and the treatment was stopped on her own. Then her
health began to gradually deteriorate, she lost 10 kg. 2 weeks ago, she suffered a severe form
of flu, after which her condition worsened. The body temperature rose to 38.6 ° C, tachycardia
developed Objectively: pronounced emaciation, the patient is agitated, a sharp tremor of the
fingers, tremors of the whole body, pulse - 146 bpm, atrial fibrillation. BP 100/30 mm Hg. Art. In
the lower parts of the lungs, there are single humid unsonic rales, the liver is enlarged. What
disease is the clinical picture typical for:

1.Tyroidite Hashimoto

2.Thyrotoxic crisis

3.Acutethyroiditis

4.Diffuse toxic goiter

5.Myxedema

Q15.Indicate a pronounced disorder of the nervous system in children with hypothyroidism:

1.emotional instability

2.hysteria

3.mental retardation

4.convulsive readiness

5.neurasthenia

Q16.Name the main laboratory criterion of hyperglycetmia syndrome:

1.hypoproteinemia

2.hypokaliemia

3.hypernatriemia

4.high fasting glycemia


5.hypercholesterolemia

Q17.Highlight, what explains the pallor of the facial skin and mucous membranes in babies with
hypothyroidism?

1.vitamin «C» deficiency

2.decreased physical activity

3 deep arrangement of blood vessels

4 iron deficiency

5.hypovitaminosis B12

Q18.Explain, what causes weight loss in children - leading symptom of hyperthyroidism:

1.intestinal absorption is impaired

2.polyuria

3.frequent loose stools

4.functional liver failure

5.disorder of carbohydrate metabolism

Q19.A patient girl, 12 years of life, turned with mother to the family doctor with complaints on
hunger, rapid fatique, dizziness, pain in abdomen, hand tremors. Anamnesis: suffers from
diabetes mellutus 1 type/ since of 9, receives insulin therapy. 4 days ago, she began to attend
training sessions for 2 hours daily. This complaints appeared 2 days ago, very tired She tries to
follow the diet,prepares herself. She studies well. Objectively: physical development
corresponds to age. The skin is pallor, dry, cold hands. Heart rate - 82 in 1 minute. Arterial
pressure - 110/70 mm Hg. No thirst, does not urinate often. Blood glucose level is 4,2 mmol/l. In
urine no glucose. Decide, which syndrome the patient has?

1.premenstrual

2.intoxication

3."attention deficit"

4.hyperthyroidism
5.hypoglycemia

Q20.Obesity 2nd degree, the body mass index (kg / m2) is

1.30.0 - 34.9

2 35.0 - 39.9

3.25.0 - 29.9

4.18.5 - 24.9

5.40.0 - 45.9

Q21.Normally, the volume of the thyroid gland (according to ultrasound) in men is (ml):

1.from 45 to 48

2.more than55

3.from 30 to 35

4.from 38 to 40

5.Up to 25

Q22.Formula for calculating body mass index (BMI)

1.weight, kg / height, m2

2.weight, kg2 / height, m

3.height, m2 / weight, kg

4.weight, kg / height, m

5 height, m/ weight, kg2

Q23.What are the causes of primary hypothyroidism?

1.pituitary adenoma

2.skull trauma
3.neuroinfection

4.pituitary irradiation

5.autoimmune thyroiditis

Q24.The cause of obesity syndrome is

1.hypercortisolism

2.hyperparathyroidism

3.hyperglycemia

4.hyperthyroidism

5.gigantism

Q25.Which drug should be prescribed for acute hyperglycemia?

1.gliclazide

2 glibenclamide

3.metformin

4.short-acting insulin

5.long acting insulin

Q26.Describe the characteristic changes on palpation of the thyroid gland in acute inflammation
of the thyroid gland:

1.smooth elastic consistency

2.regional lymph nodes are not enlarged

3.thyroid gland of uneven density

4.absence of a focus of fluctuation

5.sharply painful, dense


Q27.First aid for hypoglycemia, what kind of remedy is taken?

1.metformin 500 mg

2.glucose 40%

3.calcium chloride 100 mg

4.furosemide 40 mg

5.insulin 4 unit

Q28.Normally, the volume of the thyroid gland (according to ultrasound) in women is (ml):

1.from 35 to 38

2.Up to 18

3.from 40 to 45

4.from 25 to 30

5.more than 50

Q29.Addison's disease is characterized by

1.hypoprolactinemia

2.hypopituitarism

3.hypoparathyroidism

4.hypothyroidism

5.hypocorticisolism

Q30.38-year-old woman was admitted to the clinic with A complaints of decreased appetite,
constipation, hearing loss, difficulty in nasal breathing, weakness. Anamnesis: Subtotal
strumectomy was performed 3 years ago. Objectively: The skin is dry, the height is 167 cm, the
weight is 68 kg, there is dense swelling of the legs. diagnosed with hypothyroidism syndrome.
Explain why hearing is reduced and nasal breathing is difficult due to?

1.mucous edema
2.swelling of the skin

3.inflammation

4.muscle swelling

5.infection

Q31.A 55-year-old man, a plumber at a tuberculosis hospital, called an ambulance, complaining


of severe weakness, decreased appetite, nausea, vomiting, and abdominal pain. Objectively:
the patient is adynamic, weight - 49 kg, height - 170 cm, darkening of the skin of the face, neck,
arms, breast nipples. The pulse on the radial artery is soft, low voltage. BP - 80/40 mm Hg.
Indicate the most likely predisposing factor for this syndrome.

1.hypotension

2.gender

3.tuberculosis

4.cachexia

5.age

Q32.The man 40 year old was admitted to the clinic, complaints of palpitations, interruptions in
the work of the heart, sweating, trembling in the body, weakness. Anamnesis: has been ill for
several years. Examination: the apical impulse is enhanced. Heart sounds are loud, arrhythmic.
Pulse 98 in 1 minute. blood pressure 170/60 mm Hg. ECG: tachysystolic form of atrial
fibrillation. The doctor diagnosed hyperthyroidism syndrome. Explain, what caused the atrial
fibrillation in this case, because of the increase?

1.calcitonin

2.parathyroid hormone

3.T3 and T4

4.increased TSH

5.antibodies TPO

Q33.Patient, 51 years old. At the endocrinologist's appointment for the first time. Complaints of
increased fatigue, overweight. From the anamnesis: the patient's mother was obese. Smokes
since 20 years. Objectively: height 173 cm, body weight 102 kg. The type of distribution of
adipose tissue is abdominal obesity. The skin is normal in color, dry to the touch. Blood
pressure - 110/70 mm Hg, heart rate - 76 per minute. Laboratory results: fasting glycemia 10.5
mmol / I. Explain, what caused the development of hyperglycemia in this patient?

1.smoking

2.hypodynamia

3.obesity

4.hypertension

5.heredity

Q34.The man was admitted to the clinic, complaints of palpitations, interruptions in the work of
the heart, sweating, trembling in the body, weakness. Anamnesis: has been ill for several years.
Examination: the apical impulse is enhanced. Heart sounds are loud, arrhythmic. Pulse 98 in 1
minute. blood pressure 170/60 mm Hg. ECG: tachysystolic form of atrial fibrillation. Your
suspected syndrome?

1.hyperglycemia

2.hyperglobinemia

3.hypercorticism

4.hyperparathyroidism

5.hyperthyroidism

Q35.A woman, 50 years old, consulted a local therapist with complaints of thirst, dry mouth,
weight gain, headache, pain in the lumbar region, sweating. From the
anamnesis: the patient's mother suffered from type 2 diabetes. Objectively: height - 165 cm,
body weight - 100 kg, rubeosis of the cheeks, excessive deposition of fatty tissue in the area of
the shoulders, chest, abdomen, pale striae on the skin of the abdomen, thighs, deformation of
the nails legs. Waist circumference - 102 cm, hip circumference - 95 cm. BP - 160/90 mm Hg.
Interpret, how much the patient's waist circumference should be? Less than....

1.85

2.90

3.80
4.95

5.75

Q36.A man V. 40 years old, was admitted to the clinic with multiple pathological, spontaneous
fractures of tubular claws, pronounced curvature of the spine and deformation of the skeleton.
Upon admission, the patient complained of general weakness, extremely rapid fatigue, muscle
hypotonia, and bone pain. In the anamnesis there are indications presence of kidney stones,
severe polyuria and an increased content of phosphates in the urine. Laboratory examination
revealed an increase in the calcium content in the blood to 15.1 mmol/L and a decrease in the
phosphorus content to 0.24 mmol / L. What is your preliminary syndrome?

1.hypercortisol syndrome

2.hyperparathyroidism syndrome

3.hyperglycemic syndrome

4.thyroid inflammation syndrome

5.hyperthyroidism syndrome

Q37.Patient A., 34year old applied to polyclinic with complaints of an increase in body
temperature up to 39°C, pain in the right side of the thyroid gland, aggravated by swallowing,
coughing, chills, tachycardia, weakness. Anamnesis: frequent tonsillitis. Objectively: On
palpation, the thyroid gland is sharply painful, dense, regional lymph nodes are enlarged.
Laboratory: Total blood count: leukocytosis with a shift to the left, increased ESR. Choose the
most informative method for examining the thyroid gland in this situation:

1.lateral radiography of the neck.

2.determination of T3, T4 in blood

3.thyroid scan with 131 I

4.auscultation of the thyroid gland

5.ultrasound with biopsy

Q38.Patient M., 40 years old, a salesman, applied to the doctor of policlinic with complaints of
general weakness, rapid fatigability, sensations of pressure in the front of the neck, loss
memory, facial swelling, constipation. From the anamnesis: suffering from rheumatoid arthritis
for many years. Objectively: Body temperature - 36.1 °C. The skin is pale, dry, peeling is
expressed on the legs. Swelling of face, dense edema on the legs, lower third of the lower leg.
Pulse 55 per minute, BP - 126/80 mm Hg. Tongue clean, teeth marks on tongue. The thyroid
gland is unevenly enlarged on examination, on palpation it is dense, mobile when swallowing,
painful, nodular formations are not determined. Ultrasound of the thyroid gland: V total. - 6.3
cm3 (8.3-18.0). Increased echogenicity. The structure is clearly diffusely heterogeneous. What
laboratory test proves the
presence of an autoimmune syndrome?

1.T3 fr.-3.0 pmol/l

2.TSH-0.3 mU/l

3.T4 fr.-28 pmol/l

4.AB-TPO - 250 U/ ml

5.AB-TG - 9 U/ml

Q39.A 16-year-old boy was admitted to the clinic in an unconscious state according to the
mother, previously complained of severe thirst, frequent urination and general weakness.
Objectively: the smell of acetone from the mouth, the skin is dry. Rapid breathing. Heart sounds
are muffled. The abdomen is tense. What is the most informative laboratory examination
method (blood) for making a diagnosis ?

1.proteim

2.glucose

3.cretinine

4.calcium

5.cholestetol

Q40.A 19-year-old girl was admitted to the clinic unconscious. According to her mother, the girl
has been suffering from diabetes for 4 years, at home after an injection of insulin she suddenly
lost consciousness. The ambulance doctor revealed hypoglycemia of 2,3 mmol / I. First aid was
given by intravenous glucose drip, but the patient did not regain consciousness. Indicate which
drug should be administered in this situation?

1.inulin

2.albumin
3.glucose

4.glucagon

5.insulin

Q41.Indicate which method of examining the thyroid gland is the first priority:

1.endoscopy

2.magnetic resonance imaging

3.computed tomography

4.ultrasound

5.fluoroscopy

Q42.A mother consulted a doctor, whose son had grown by 10 cm over the summer. When
examining a 19-year-old boy: height 180 cm, weight 68 kg. Which research method is more
applicable for diagnosing a disease:

1.craniography

2.target X-ray of the skull

3.magnetic resonance imaging

4.ultrasound

5.computed tomography

Q43.To peripheral form of endocrine disorders can lead to:

1.Pituitary alteration

2.Alteration of the hypothalamus

3.Strong bond of hormones with proteins

4.Alteration of the endocrine gland

5.Weak hormone-protein binding


Q44.In the onset of Basedow's disease, the main role belongs to:

1.Increased production of cortisol

2.Insufficient action of parathyroid hormone

3.Insufficient aldosteron production

4.Increased production of somatotropic

hormones

5.Excessive action of thyroid hormones

Q45.A patient came to the appointment - thin, with pronounced exophthalmos (bulging eyes),
pronounced anxiety in behavior. Heart rate (HR) - 92 beats / min. Dysfunction of which
endocrine gland can cause the
following symptoms:

1.Timus

2.Pituitary gland

3.Adrenalglands

4.Parathyroid

5 Thyroid

Q46.Female, 52 years old, onset of the disease 1.5 years ago after a course of radiation
therapy. Complains of memory impairment, general weakness, drowsiness, baldness, lack of
appetite, weight gain. An objective examination revealed: the skin is dry, flaky, the face is puffy,
hypomimic, there are areas of baldness on the head, the voice is low, rough, speech is slowed
down, the body temperature is 35.7 °, the pulse is 52 beats. per minute, blood pressure - 110/65
mm Hg. Art., the level of glucose in the blood - 3.5 mmol / I, the basal metabolism is reduced by
30%. Diagnosed with Myxedema. What is the most likely cause of this disease:

1.Formation of mucin

2.Formation of reverse T3

3.Decreased TSH levels

4.Increase in T3 T4
5.lodine deficiency

Q47.Male 30 years old. He was admitted to the clinic with complaints of headache, drowsiness,
change in appearance over the past 2 years, during which he considers himself ill. Examination:
an increase in the brow ridges, chin, nose, auricles, tooth divergence. The hands and feet are
disproportionately enlarged. AD - 160/95 mm Hg, heart rate 90 beats per minute, an increase in
the size of the liver, the borders of the heart are expanded, blood glucose - 10 mmol / I, total
protein - 90 g/l, urine glucose +. Which of the following preliminary diagnoses is most likely:

1.Pituitary hypogonadism

2.Pituitary dwarfism

3.Acromegaly

4.Adiposogenitaldystrophy

5.Pituitary gigantism

Q48.A patient girl, 12 years of life, turned with mother to the family doctor with complaints on
hunger, rapid fatique, dizziness, pain in abdomen, hand tremors. Anamnesis: suffers from
diabetes mellutus 1 type/ since of 9, receives insulin therapy. 4 days ago, she began to attend
training sessions for 2 hours daily. This complaints appeared 2 days ago, very tired She tries to
follow the diet,prepares herself. She studies well. Objectively: physical development
corresponds to age. The skin is pallor, dry, cold hands. Heart rate 82 in 1 minute. Arterial
pressure - 110/70 mm Hg. No thirst, does not urinate often. Blood glucose level is 4,2 mmol/l. In
urine no glucose. Decide, which syndrome the patient has?

1.premenstrual

2.intoxication

3."attention deficit"

4.hyperthyroidism

5.hypoglycemia

Q49.The immediate stimulant of the glomerular adrenal cortex is :

1.Angiotensin

2.Nicotin
3.Acetylcholine

4.Vasopressin

5.Adrenaline

Q50.What change in the biochemical composition of the blood is observed with an overdose of
insulin:

1.Decreased protein content

2.Decreased phosphorus levels

3.Increased sugar levels

4.Increased calcium content

5.Lowering sugar levels

Q51.A patient came to the appointment - thin, with pronounced exophthalmos (bulging eyes),
pronounced anxiety in behavior. Heart rate (HR) - 92 beats / min. Dysfunction of which
endocrine gland can cause the following symptoms:

1.Thyroid

2.Adrenalglands

3.Timus

4.Pituitary gland

5.Parathyroid

Q52.Patient B., 39 years old, who has been unable to get pregnant for 8 years, was advised to
consult an endocrinologist. Examination revealed exophthalmos, hand tremor, tachycardia in
the patient. The disease of which endocrine gland is accompanied by the following symptoms:

1.Thyroid

2.Pancreas

3.Epiphysis
4.Adrenal glands

5.Polovykh

Q53.After removal of the thyroid gland, the patient developed convulsions. What was damaged
during the operation:

1.Inferior laryngeal artery

2.Inferior laryngeal nerve

3.Sympathetic trunk

4.Parathyroid gland

5.Vagus nerve

Q54.A 45-year-old man, taken to the clinic by an ambulance team with a diagnosis of
"Hypertensive crisis". BP. 180/120 mm Hg. Art., myocardial hypertrophy, tachycardia,
weakening of memory and intelligence, blood glucose - 6.5 mmol/l. A history of sudden crises
with a rise in blood pressure up to 200/140 mm Hg. Art., the appearance of tachycardia,
sweating, sharp agitation. Nuclear magnetic resonance imaging of the lumbar region revealed
an increase in the size of the left adrenal gland, the presence a ded formation in the medulla of
the gland. Which of the following diagnoses is most likely:

1.Diabetes mellitus

2.Addison's disease

3.Adrenalcrisis

4.Arterial hypertension

5.Pheochromocytoma

Q55.A 60-year-old man was admitted to the endocrinology department with suspected
basophilic adenoma of the anterior pituitary gland. Complains of general weakness, frequent
bronchitis, headache, pain in the back and limbs. Objectively: the clinical picture corresponds to
Cushing's syndrome. In the analyzes, there is an increase in the basal level of corticotoropin in
blood plasma and glucocorticoids in plasma and urine. With fluoroscopy, pronounced
osteoporosis of the bones of the skull, vertebral bodies, hyperplasia of both adrenal glands.
Which of the following answers is most likely:

1.Imbalance between bone formation and destruction


2.Aging of the body

3.Estrogen deficiency

4.High glucocorticoid levels

5.Vitamin D deficiency

Q56.Indicate the main symptom in hyperglycemia syndrome:

1.hyperactivity

2.cold sweat

3.polydipsia

4.bad appetite

5.tremor of the hands, body

Q57.Determine, at what age you need to identify the first signs of hypothyroidism:

1.7-9 years

2.4-6 years

3.6-9 month

4.1-3 month

5.1-3 years

Q58.Baby, 2 months of life, with mother went to the family doctor with complaints of
constipation, yellowness of the skin, drowsiness, difficulty breathing. Anamneses: was born
full-term on time, delivery was normal, mass of body - 4300,0. Sucks sluggish. The umbilical
remains fell of late. Objectively: lenth-55cm, mass - 6 400,0. The face is edematous, the tongue
is large. Low voice. Baby is sluggish, does not hold his head. The skin is icteric, very dry. The
bridge of nose is sunken. Heart tones are weakened, bradycardia. Heart rate - 106 in 1 minute.
In the blood test: erythrocytes - 3,7x10121/1, Hb -106 g/l. Hormones: T4 - decreased, TSH
-increased. ESG: sinus bradycardia. Choose, using all the data, which syndrome this child has?

1.hypothyroidism
2.Daun

3.uric

4.hypoexcitability

5.anemic

Q59.Explain, why accelerated growth and ossification occurs in children with hyperthyroidism
syndrome?

1.high growth hormone levels

2.hypercalcemia

3.high levels of thyroglobulin

4.development of bone epiphyses

5.high level soft hyrocalcitonin

Q60.In hypercortisolism syndrome, skin changes are characterized by

1.vitiligo

2.yellowness

3.striae

4.darkening

5.myxedema

Q61.What is a symptom of polyphagia, is it an increase?

1.appetite

2.heart rate

3.thirst

4.weight

5.urination
Q62.Indicate the reason for the occurrence of hypoglycemia?

1.prednisone overdose

2.iodine overdose

3.insulin overdose

4.dexamethasone overdose

5.glucagon overdose

Q63.The cause of obesity syndrome is

1.hyperthyroidism

2.hypercortisolism

3.gigantism

4.hyperparathyroidism

5.hyperglycemia

Q64.A 48-yearold woman was admitted to the clinic for examination and doesn't present any
complaints. Anamnesis: mom suffers from diabetes. Objectively: a patient with increased
nutrition, normal color leather. Height - 172 cm2, body weight - 88 kg. Breathing is vesicular.
Heart sounds are saved. Laboratory: fasting blood glucose (venous plasma) - 6.9 mmol/l after
meals after 2 hours 10.1 mmol /I, interpret this result.

1.diabetes mellitus

2.hypoglycemia

3.impaired glucose tolerance

4.acute hyperglycemia

5.impaired fasting glucose

Q65.A 35-year-old man, was admitted to the clinic with complaints of palpitations, interruptions
in the work of the heart, sweating, trembling in the body, weakness. Anamnesis: has been ill for
several years. Examination: the apical impulse is enhanced. Heart sounds are loud, arrhythmic.
Pulse 110 in 1 minute. Blood pressure 180/60 mm Hg. Laboratory: TSH - decreased, T4, T3
increased. The doctor diagnosed hyperthyroidism syndrome. What method is necessary given
the severe tachycardia?

1.MRI

2.scintigraphy

3.ECG

4.X-ray

5.ultrasound

Q66.A 29-year-old man, was admitted to the clinic with complaints of palpitations, sweating,
tremors in the body, weakness, diarrhea, darkening of the skin in places of skin friction.
Anamnesis: has been ill for several years. Examination: the apical impulse is enhanced. Heart
sounds are loud, arrhythmic. Pulse 98 in 1 minute. blood pressure 170/60 mm Hg. TSH is
reduced, T3, T4 is increased. The doctor diagnosed hyperthyroidism syndrome. Explain what is
the connection with the appearance of skin darkening due to?

1.renal failure

2.adrenal insufficiency

3.thymus insufficiency
Hi
4.liver failure

5.parathyroid insufficiency

Q67.A 52-year-old woman consulted a therapist with complaints of thirst, frequent urination,
itching of the perineum. In the anamnesis, the above complaints are worried for 1 month.
Objectively: The skin is dry and clean. BMI = 32.2 kg/m2. BP - 130/90 mm Hg. Laboratory blood
glucose - 10.3 mmol/l, after meals 15.0 mmol / I. What analysis can you use to find out blood
glucose in 3 months?

1.glycohemoglobin

2.C- peptide

3.blood glucose

4.total cholesterol
5.triglycerides

Q68.Damage to the neurohypophysis is accompanied by impaired secretion:

1.Adrenocorticotropic hormone

2.Vasopressin

3.Thyroid stimulating hormone

4.Growth hormone

5.Prolactin

Q69. A mother consulted a doctor, whose son had grown by 10 cm over the summer. When
examining a 19-year-old boy: height 180 cm, weight 68 kg. Which research method is more
applicable for diagnosing a disease:

1.ultrasound

2.magnetic resonance imaging

3.craniography

4.computed tomography

5.target X-ray of the skull

Q70.Patient K., 32 years old, complains of unreasonable anxiety, increased mental excitability,
weight loss, sweating, frequent stools. She fell ill for 6 months. back after suffering the flu.
Revealed eye symptoms (Grafe, Kraus). Pulse 100 in 1 minute. The thyroid gland is enlarged to
3 tbsp. What is the method of radiation diagnostics that allows in this case to assess the
function of the thyroid gland:

1.nuclear magnetic resonance.

2.ultrasonic scanning.

3.radioimmunoassay.

4.computed tomography of the thyroid gland.

5.doppler study
Q71.Determine which CT-scan slice increment (distance between slices) should be used for
small adrenal masses (<1.5 cm):

1.5mm

2.24 mm

3.1.5 mm

4.36 mm

5.16 mm

Q72.Patient, 48 years old, at a doctor's appointment complains of muscle weakness, rapid


physical fatigue, lack of appetite, nausea, muscle pain, addiction to salty foods, menstrual
irregularities. On examination, weight - 48 kg, height - 166 cm. The skin has a dark brown color,
dark spots on the mucous membrane of the oral cavity, subcutaneous fat is thinned. BP - 70/50
mm Hg. The pulse on the radial artery is soft, small filling. Choose the most informative blood
test for your diagnosis:

1.follicle-stimulating hormone

2.growth hormone

3.adrenocorticotropic hormone

4.thyroid-stimulating hormone

5.melanotropic hormone

Q73.A 50-year-old man was admitted to the clinic with complaints of dry mouth, thirst, frequent
urination, weakness. Anamnesis, the above complaints bother for a month. Objectively: The
skin is dry and clean. Breathing is vesicular. Heart sounds are saved. Blood pressure 150/90
mm Hg. Laboratory: blood glucose 12.0 mmol. The doctor diagnosed hyperglycemia. Explain
the explanation for the elevated blood glucose level in this case?

1.impaired secretion of somatostatin

2.violation of enzyme secretion

3.impaired glucose secretion

4 violation of glucagon secretion


5.impaired insulin secretion

Q74.A 33-year-old woman consulted a therapist, according to the patient, six months after the
strumectomy, weakness and apathy appeared. drowsiness, constipation and menstrual
irregularities. Objectively: the skin is dry, the hair is dry, brittle, baldness in the outer areas.
Pulse 58 beats per minute. blood pressure 120/80 mm Hg. Heart sounds are weakened. Doctor
diagnosed hypothyroidism syndrome. Determine the cause of hypothyroidism?

1.tertiary

2.idiopathic

3.peripheral

4.secondary

5.primary

Q75.Describe the total blood count for acute inflammation of the thyroid gland:

1.reticulocytosis

2.thrombocytosis

3.leukocytosis

4.​lymphocytosis

5.erythrocytosis

Q76.A 55-year-old man, a plumber at a tuberculosis hospital, called an ambulance, complaining


of severe weakness, decreased appetite, nausea, vomiting, and abdominal pain. Objectively:
the patient is adynamic, weight - 49 kg, height - 170 cm, darkening of the skin of the face, neck,
arms, breast nipples. The pulse on the radial artery is soft, low voltage. BP - 80/40 mm Hg.
Indicate the most likely predisposing factor for this syndrome.

1.hypotension

2.tuberculosis

3.age

4.gender
5. cachexia

Q77.The most common cause of secondary hypercortisolism is :

1.hypophysectomy

2.corticosteroma

3.pheochromocytoma

4.meningoencephalitis

5.corticotropinoma

Q78.Normal level of body mass index (kg / m2) is:

1.35.0 - 39.9

2. 18.5 - 24.9

3.40.0 - 45.9

4.30.0 - 34.9

5.25.0 - 29.9

Q79.What complication does hyperthyroidism give:

1.biliary dyskinesia

2.atrophic gastritis

3.B12 deficiency anemia

4.addison's disease

5.atrial fibrillation

Q80.Remember, the main laboratory sign indicating


hypothyroidism is:

1.increased cholesterol
2.high thyroid-stimulating hormone

3.erythrocytosis

4.limphocytosis

5.increased triiodothyronine

Q81.A 60-year-old man was admitted to the endocrinology department with suspected
basophilic adenoma of the anterior pituitary gland. Complains of general weakness, frequent
bronchitis, headache, pain in the back and limbs. Objectively: the clinical picture corresponds to
Cushing's syndrome. In the analyzes, there is an increase in the basal level of corticotoropin in
blood plasma and glucocorticoids in plasma and urine. With fluoroscopy, pronounced
osteoporosis of the bones of the skull, vertebral bodies, hyperplasia of both adrenal glands.
Which of the following answers is most likely:

1.Imbalance between bone formation and destruction

2.Aging of the body

3.Estrogen deficiency

4.High glucocorticoid levels

5.Vitamin D deficiency
5-семестр

Модуль "Эндокринная система" [крд.-2]

Вопрос: №1
Indicate which radiopharmaceutical is used for thyroid radionuclide study:
Ответы(один ответ)
1 99mTc - pyrophosphate

2 99shTs-technetril

3 99mTc - diphosphonate

4 99mTs -pertechnetat

5 99mTc - colloid

Вопрос: №2
After removal of the thyroid gland, the patient developed convulsions. Determine which structure w
Ответы(один ответ)
1 parathyroid gland

2 inferior laryngeal nerve

3 inferior laryngeal artery

4 the vagus nerve

5 sympathetic trunk

1 2 3 4 5 6 7 8 9 10

11 12 13 14 15 16 17 18 19 20

21 22 23 24 25 26 27 28 29 30

31 32 33 34 35 36 37 38 39 40

41 42 43 44 45 46 47 48 49 50

- отвеченный вопрос

- пропущенный вопрос

- текущий вопрос
До завершения теста осталось:

0:16:29
Завершить тест

Вопрос: №3
Which research method is more informative for ovarian screening:
Ответы(один ответ)
1 endoscopy

2 ultrasound

3 magnetic resonance imaging

4 fluoroscopy

5 computed tomography

«1234567...50»

Мехди Тарик

Вопрос: №4
A 26-year-old woman came to the clinic with complaints of pain in the right mammary gland, the m
menstruation. The doctor ordered a mammogram. Indicate in your answers when to get a mammog
Ответы(один ответ)
1 from the 6th to the 12th day of the menstrual cycle

2 regardless of the phase of the cycle

3 from the 1st to the 5th day of the menstrual cycle

4 only after menopause

5 in the second half of the menstrual cycle

A mother consulted a doctor, whose son had grown by 10 cm over the summer. When examining a
method is more applicable for diagnosing a disease:
Ответы(один ответ)
1 ultrasound

2 target X-ray of the skull


3 computed tomography

4 magnetic resonance imaging

5 craniography

Patient S., 56 years old, diagnosed with autoimmune thyroiditis on the basis of examination, palpati
applicable here:
Ответы(один ответ)
1 positron emission tomography

2 x-ray examination

3 endoscopy

4 ultrasound

5 scintigraphy

Вопрос: №7
Violation of spermatogenesis in men, due to a decrease in the secretion of lutropin and follitropin b
Ответы(один ответ)
1 Thyroid adenoma

2 Hypoprolactinemia as a result of prolonged use of chlorpromazine

3 Hyperprolactinemia with pituitary adenoma

4 Congenital adrenal hypoplasia

5 Chronic adrenal insufficiency

Вопрос: №8
The most common cause of Addison's disease is:
Ответы(один ответ)
1 Adrenal atrophy

2 Hyperplasia of the pineal gland

3 Ovary hypertrophy
4 Autoimmune thyroiditis

5 Pituitary tumor

Вопрос: №9
Transhypophyseal regulation is the basis for:
Ответы(один ответ)
1 Adrenalcortex

2 Parathyroidglands

3 Gonads

4 Thyroid gland

5 Pancreas

Вопрос: №10
In severe cases of hypothyroidism in adults occurs:
Ответы(один ответ)
1 Cretinism

2 Myxedema

3 Dwarfism

4 Hypergonadism

5 Eunuchoidism

Вопрос: №11
With an insufficient amount of iodine in food, it develops:
Ответы(один ответ)
1 Hyperthyroidism

2 Autoimmune thyroiditis

3 Hypoparathyroidism

4 Endemic goiter
5 Diffuse toxic goiter

Вопрос: №12
What disorders can occur with partial hypofunction of the anterior pituitary gland:
Ответы(один ответ)
1 Basedow's disease

2 Gigantism

3 Hyperglycemia

4 Emaciation

5 Hypogonadism

Вопрос: №13
The appearance at the age of 3 to 7 years of secondary sexual characteristics is observed:
Ответы(один ответ)
1 Adrenogenital syndrome

2 Acromegaly

3 Adiposogenital dystrophy

4 Klinefelter Syndrome

5 Pituitary dwarfism

Вопрос: №14
Peripheral (extraglandular) mechanisms of hormone activity disturbance are:
Ответы(один ответ)
1 Congenital malformations of the glands

2 Blockade of hormonal receptors

3 Adenoma of neurosecretory cells of the hypothalamus

4 Deficiency of substrates for the formation of hormones

5 Hereditary defect in hormone biosynthesis enzymes


Вопрос: №15
On the basis of which pituitary hormone are drugs used to stimulate the contractile activity of the u
Ответы(один ответ)
1 Oxytocin

2 Prolactin

3 Thyroid stimulating hormone

4 Follicle-stimulating hormone

5 Vasopressin

Вопрос: №16
The patient has clinically all the signs of thyrotoxicosis. What research needs to be done to clarify th
Ответы(один ответ)
1 Determine the level of thyroxine

2 LDL, HDL, bloodcholesterol

3 Blood test for sugar, ketone bodies

4 protein composition of plasma, residual blood nitrogen,

5 General analysis of blood and urine

Вопрос: №17
Patient A., 39 years old, complains of copious flow of urine (daily urine output is 22 liters per day). B
production of which hormone polyuria is associated:
Ответы(один ответ)
1 Parathyroid hormone

2 Insulina

3 Glucagon

4 ADH - Vasopressin

5 Aldosterone
Вопрос: №18
Over the past 6 months, the patient has had several bone fractures, muscle weakness, pain in the ba
associated:
Ответы(один ответ)
1 Thyroid

2 Parathyroid

3 Pancreas

4 Pituitary gland

5 Adrenalglands

Вопрос: №19
A mother consulted a doctor, whose son had grown by 18 cm over the summer. When examining a
the activity of which endocrine gland is it associated:
Ответы(один ответ)
1 Epiphysis

2 Pituitary gland

3 Thyroid

4 Adrenal glands

5 Sex

Вопрос: №20
Patient L., 45 years old, has been ill for 6 months, the onset of the disease is associated with the tran
sweating, increased appetite, drop in body weight, palpitations. On physical examination, the body
the fingers, increased glitter of the eyes, diffuse enlargement of the thyroid gland, small exophthalm
140 mm Hg. Art., the level of glucose in the blood - 5.5 mmol / l, the basal metabolic rate is increas
of which endocrine gland is impaired:
Ответы(один ответ)
1 Thyroid

2 Epiphysis

3 Adrenalglands
4 Pituitary gland

5 Parathyroid

Вопрос: №21
A 45-year-old man, taken to the clinic by an ambulance team with a diagnosis of "Hypertensive cris
weakening of memory and intelligence, blood glucose - 6.5 mmol / l. A history of sudden crises with
appearance of tachycardia, sweating, sharp agitation. Nuclear magnetic resonance imaging of the lu
gland, the presence of a rounded formation in the medulla of the gland. Which of the following dia
Ответы(один ответ)
1 Arterial hypertension

2 Diabetes mellitus

3 Adrenalcrisis

4 Addison's disease

5 Pheochromocytoma

Вопрос: №22
Male 30 years old. He was admitted to the clinic with complaints of headache, drowsiness, change i
himself ill. Examination: an increase in the brow ridges, chin, nose, auricles, tooth divergence. The ha
heart rate - 90 beats per minute, an increase in the size of the liver, the borders of the heart are exp
glucose +. Which of the following preliminary diagnoses is most likely:
Ответы(один ответ)
1 Adiposogenitaldystrophy

2 Acromegaly

3 Pituitary hypogonadism

4 Pituitary dwarfism

5 Pituitary gigantism

Вопрос: №22
Male 30 years old. He was admitted to the clinic with complaints of headache, drowsiness, change i
himself ill. Examination: an increase in the brow ridges, chin, nose, auricles, tooth divergence. The ha
heart rate - 90 beats per minute, an increase in the size of the liver, the borders of the heart are exp
glucose +. Which of the following preliminary diagnoses is most likely:
Ответы(один ответ)
1 Adiposogenitaldystrophy

2 Acromegaly

3 Pituitary hypogonadism

4 Pituitary dwarfism

5 Pituitary gigantism

Вопрос: №23
Describe the stool for hypothyroidism in early childhood:
Ответы(один ответ)
1 frequent, frothy with mucus

2 form of rice water

3 frequent with blood

4 form as pea puree

5 persistent constipation

Вопрос: №24
Determine, at what age you need to identify the first signs of hypothyroidism:
Ответы(один ответ)
1 6-9 month

2 7-9 years

3 1-3 years

4 4-6 years

5 1-3 month

Вопрос: №25
Indicate the hormone that regulates the secretion of thyroid-stimulating hormone by the pituitary g
Ответы(один ответ)
1 adrenocorticotropic hormone
2 thyrotropin releasing hormone

3 thyroxine

4 thyrocalcitonin

5 thyroglobulin

Вопрос: №26
Identify the main cause of constipation and flatulence in children with hypothyroidism:
Ответы(один ответ)
1 motor function of the intestine

2 poor appetite

3 low water consumption

4 decrease metabolism of substances

5 decreased physical activity

Вопрос: №27
Explain, why accelerated growth and ossification occurs in children with hyperthyroidism syndrome
Ответы(один ответ)
1 highlevelsofthyrocalcitonin

2 high growth hormone levels

3 hypercalcemia

4 high levels of thyroglobulin

5 development of bone epiphyses

Вопрос: №28
A patient, 11 years old, turned with her mother to the family doctor with complaints of thirst, increa
Thesecomplaints appeared after a dog bite, she was scared after 3 days. Objectively: physical develo
breathing, the smell of «ratten apples». The mucous membranes of the mouth and lips are dry, alth
degree. The heart sounds are clear, rhythmic. The liver is at the edge of the costal arch. Frequent an
test to conduct immediately?
Ответы(один ответ)
1 general blood analysis

2 electroencephalography

3 common analysis of urine

4 fasting blood glucose

5 common protein, protein fractions

Вопрос: №29
Causes of primary hyperparathyroidism:
Ответы(один ответ)
1 lack of calcium in food

2 dysfunction of the hypothalamus

3 removal of the thyroid gland

4 pituitary adenoma

5 hyperplasia of the parathyroid glands

Вопрос: №30
Formula for calculating body mass index (BMI)
Ответы(один ответ)
1 height, m2 / weight, kg

2 weight, kg / height, m

3 height, m / weight, kg2

4 weight, kg2 / height, m

5 weight, kg / height, m2

Вопрос: №31
Which of the following corresponds to impaired glucose tolerance (venous plasma):
Ответы(один ответ)
1 fasting glucose - 9.6 mmol / l, 2 hours after OGTT - 14.4 mmol / l

2 fasting glucose - 6.8 mmol / l, 2 hours after OGTT - 10.9 mmol / l

3 fasting glucose - 8.9 mmol / l, 2 hours after OGTT - 12.7 mmol / l

4 fasting glucose - 10.0 mmol / l, 2 hours after OGTT - 16.8 mmol / l

5 fasting glucose - 5.4 mmol / l, 2 hours after OGTT - 9.2 mmol / l

Вопрос: №32
Laboratory criteria for obesity syndrome
Ответы(один ответ)
1 low total cholesterol

2 high glucose

3 low LDL cholesterol

4 low triglycerides

5 high HDL cholesterol

Вопрос: №33
What complication does hyperthyroidism give:
Ответы(один ответ)
1 atrial fibrillation

2 B12 deficiency anemia

3 addison's disease

4 biliary dyskinesia

5 atrophic gastritis

Вопрос: №34
Describe the total blood count for acute inflammation of the thyroid gland:
Ответы(один ответ)
1 lymphocytosis

2 erythrocytosis

3 reticulocytosis

4 thrombocytosis

5 leukocytosis

Вопрос: №35
Waist circumference in women is normally less than (cm)
Ответы(один ответ)
1 80

2 70

3 85

4 75

5 65

Вопрос: №36
The cause of obesity syndrome is
Ответы(один ответ)
1 hyperglycemia

2 hypercortisolism

3 hyperthyroidism

4 gigantism

5 hyperparathyroidism

Вопрос: №37
Which drug should be prescribed for acute hyperglycemia?
Ответы(один ответ)
1 long acting insulin

2 glibenclamide

3 short-acting insulin

4 gliclazide

5 metformin

Вопрос: №38
What is the most informative method for examining the thyroid gland in acute thyroiditis:
Ответы(один ответ)
1 auscultation of the thyroid gland

2 lateral radiography of the neck

3 radioisotope scanning

4 inspection and palpation

5 ultrasound with biopsy

Вопрос: №39
A white sclera line between the iris and the upper eyelid when looking down with thyroid hyperfunc
Ответы(один ответ)
1 Krause

2 Shtelvag

3 Grefe

4 Elinek

5 Kocher

Вопрос: №40
. A 29-year-old female patient turned to a polyclinic therapist with complaints of headaches, weight
complaints are disturbing for 6 months after the head injury. On examination: moon face, excessive
from the description the objective symptom of this patient.
Ответы(один ответ)
1 diastema

2 exophthalmos

3 macrognathia

4 hirsutism

5 fish mouth

Вопрос: №41
Patient G., 42 years old, after a traumatic injury to the neck, notes the periodic appearance of parest
creeps") with the subsequent development of seizures. Before the development of seizures, he has
upper limbs and face. Consciousness during this period is preserved. Examination of the patient rev
phosphorus - 8.0 mmol / l. What is your preliminary syndrome?
Ответы(один ответ)
1 hypoparathyroidism syndrome

2 hypocorticism syndrome

3 hypopituitarism syndrome

4 hypoglycemic syndrome

5 hypothyroidism syndrome

Вопрос: №42
Patient, 65 years old. Complaints - overweight, increased appetite, intermittent thirst, episodes of in
headaches, tinnitus. A history of cholecystectomy for cholelithiasis, heredity is aggravated by type 2
Objectively: height 170 cm, weight 134 kg. The skin is normal in color, dry. Distribution of subcutane
the face, multiple pale striae on the abdomen and buttocks, ulnar hyperkeratosis. Predict, what is th
Ответы(один ответ)
1 4,2

2 6,2

3 2.2

4 5,2

5 3,2
Вопрос: №43
Patient A., 40-year-old was admitted to the hospital with complaints of chills, weakness, difficulty in
aggravated when turning, body temperature 38˚C, sweating. On palpation, the thyroid gland is enla
there are no fluctuating areas. Submandibular, cervical lymph nodes are not enlarged. In the analysi
study of the accumulation of iodine by the thyroid gland on the scanned areas of enlightenment, th
Ответы(один ответ)
1 syndrome of hypothyroidism

2 thyroid inflammation syndrome

3 adrenal hypofunction syndrome

4 syndrome of hypofunction of the parathyroid glands

5 thyroid nodule syndrome

Вопрос: №44
A woman, 50 years old, consulted a local therapist with complaints of thirst, dry mouth, weight gain
anamnesis: the patient's mother suffered from type 2 diabetes. Objectively: height - 165 cm, body w
tissue in the area of the shoulders, chest, abdomen, pale striae on the skin of the abdomen, thighs,
circumference - 95 cm. BP - 160/90 mm Hg. Interpret, how much the patient's waist circumference s
Ответы(один ответ)
1 95

2 85

3 80

4 75

5 90

Вопрос: №45
Patient A., 64-year-old, female consulted a local general practitioner complaining of dry skin, overw
appetite, constipation, decreased memory, drowsiness. From the anamnesis: no surgery on the thyr
On examination: the skin is dry, cold to the touch. Swelling of the face. Vesicular breathing, no whee
temperature - 36.3 ° C. Stool irregular, constipation tendency. According to laboratory examination
mmol / l, triglycerides - 5.5 mmol / l, TSH - 15.3 mU / l. What is the likely cause of hypothyroidism s
Ответы(один ответ)
1 syndrome of acute inflammation of the thyroid gland
2 thyroid nodule syndrome

3 syndrome of hypofunction of the parathyroid glands

4 syndrome of chronic inflammation of the thyroid gland

5 syndrome of hyperthyroidism

Вопрос: №46
Woman 53 years old turned to a therapist with complaints of thirst, frequent urination, itching of th
Objectively: The skin is dry and clean. BMI = 33.2 kg/m2. BP 135/95 mm.Hg. Laboratory blood gluco
type 2 diabetes mellitus. Explain why the patient developed hyperglycemia syndromein this case?
Ответы(один ответ)
1 stress

2 underweight

3 heredity

4 obesity

5 hypodynamia

опрос: №47
A 55-year-old woman consulted a family doctor at the place of residence with complaints of thirst,
Objectively: height - 170 cm, body weight - 120 kg, moon-shaped face, crimson-red, excessive grow
shoulder, chest, abdomen, red-violet color, atrophy of the muscles of the arms and legs. BP - 190/1
blood test is necessary to conduct any obesity disease?
Ответы(один ответ)
1 cortisol

2 thyroxine

3 vasopressin

4 insulin

5 prolactin
Вопрос: №48
A 29-year-old man, was admitted to the clinic with complaints of palpitations, sweating, tremors in
years ago she received a brain injury. Examination: The skin is moist, the apical impulse is enhanced
pressure 160/60 mm Hg. ECG: Atrial fibrillation. TSH -0.01 mU / l, free T4-35 pmol / l. The doctor dia
Ответы(один ответ)
1 radiography

2 electrocardiography

3 ultrasound

4 endoscopy

5 echocardiography

Вопрос: №49
Patient, 48 years old, at a doctor's appointment complains of muscle weakness, rapid physical fatigu
menstrual irregularities. On examination, weight - 48 kg, height - 166 cm. The skin has a dark brown
subcutaneous fat is thinned. BP - 70/50 mm Hg. The pulse on the radial artery is soft, small filling. C
Ответы(один ответ)
1 follicle-stimulating hormone

2 thyroid-stimulating hormone

3 adrenocorticotropic hormone

4 growth hormone

5 melanotropic hormone

Вопрос: №50
Patient M., 40 years old, a salesman, applied to the doctor of policlinic with complaints of general w
neck, loss memory, facial swelling, constipation. From the anamnesis: suffering from rheumatoid art
skin is pale, dry, peeling is expressed on the legs. Swelling of face, dense edema on the legs, lower t
Tongue clean, teeth marks on tongue. The thyroid gland is unevenly enlarged on examination, on p
formations are not determined. Ultrasound of the thyroid gland: V total. - 6.3 cm3 (8.3-18.0). Increa
What laboratory test proves the presence of an autoimmune syndrome?
Ответы(один ответ)
1 T3 fr.- 3.0 pmol / l

2 AB-TPO - 250 U / ml
3 TSH-0.3 mU / l

4 Т4 fr.-28 pmol / l

5 AB-TG - 9 U / ml

- отвеченный вопрос
- пропущенный вопрос
- текущий вопрос
AnaM 2KaseA

Bonpoc: Ne1
Examination of the pituitary gland according to the standard technique, with a slice thickness of 3.0 mm in T1, T2 (Ax, Sag). The pituitary gland is not enlarged,
dimensions: sagittal 13.7 mm, transverse 16.3 mm, vertical 9.6 mm, pituitary pedicle 2.8 mm. The contours of the pituitary gland are smooth, the structure is
homogeneous. The pituitary funnel is not deflected. In the structure of the neurohypophysis (left), on postcontrast T1 scans, a hypovascular zone with dimensions of
3.1-2.7-4.1 mm is determined, Retrobulbar spaces without pathology.Areas of chiasm and cavernous sinuses are not changed. The pathology of the brain substance
in the scan area is not determined.In the cavity of the main, maxillary sinus, there is parietal fluid.What conclusion did you make based on the description:

OTBeTbI(oanH OTBeT)
10 MRI Signs of Infrasellar Pituitary Adenoma

2 0 MR signs of a pituitary cyst

3 0 MR signs of pituitary macroadenoma

MR signs of endosellar pituitary adenoma

5 MR signs of pituitary microadenoma

2 4 50

11:29
ENG
Bonpoc: N 2
Indicate what volume of contrast agent is injected with intravenous contrasting of the adrenal gland

OTBeTbI(oguH OTBeT)

1O120-150 ml
2O60-90 ml
3 O 30-60 ml

O90-120 ml
5 150-180 m

23 45 6 50
Anass a

Bonpoc: N93
women older 40
Specify the method of radiological diagnostics used for screening breast diseases in

OTBeTbi(ognH OTBeT)

mammography

2 ultrasound

3 thermography

4 O radioisotope

5O scintigraphy

1 2 4 5 6 50
avn.kgma.kg
AnaM KaseA

Bonpoc: N4
pregnancy and lactation. Previously, the study of the mammary glands was not
Patient J, 34 years old, had a bloody nipple discharge in the period of absence of
will you prescribe to the patient in the first place?
performed. Has been smoking for several years. What radiation methods

OTBeTbi(oqnH OTBeT)

1O mammography, ductography

2 magnetic resonance imaging

3O ultrasound, doppler

40 multi spiralcomputed tomography

5 Ochest x-ray
5 6 7 8

1129
g aD ENG 11-01-2021
Bonpoc: N95
Woman M, 35 years old, who has been smoking for 18 years, found in her left the mammary gland has a nodular formation, up to 2 cm in diameter. She turned to
the mammologist, who confirmed the presence of a mass. Predict which radiological examinations the patient should be referred to for diagnostic information?

OTBeTbI(oaMH OTBer)

10multi spiralcomputed tomography


20 magnetic resonance imaging
3 mammography

4 0 scintigraphy

X-ray of the chest

2 34
5 b6 7 8 9 50

1129
NG 11-01-d
avn.kgma.kg

Anass an

Bonpoc: N96
A 63-year-old patient complains of difficulty passing solid food. Objectively, the patient is lethargic, his voice is hoarse. On examination, bradycardia, pasty skin and
subcutaneous fat. A barium swallow was performed, which showed a moderate deviation of the pharynx and the upper third of the esophagus to the rnght, the
contours of the esophagus are normal. What is the most informative research method you can use?

OTBeTbI(oanH OTBeT)
10endoscopy

2 0multi spiralcomputed tomography

3 computed tomographywith contrasting thyroid gland

40 thyroid ultrasound

5 Overview Radiography

2 9 0

n ENG 1-01202
Anam Kaner

Bonpoc: Ne7
Changes in oxytocin secretion play a role in pathogenesiS:

OTBeTbI(oguH OTBeT)
1
Disorders of labor

2 0 Disorders of carbohydrate metabolism in diabetes mellitus

3ODiffusetoxic goiter
4 O Myxedema
"wakefulness sleep
5ODisorders of circadian rhythms
-

11 50
4 5 6 7 8 9 10

ENG1-01-2
Anam MaseA

Bonpoc: N98
With eosinophilic adenoma of the pituitary gland during the growth period of the body develops:

OTBeTbI(oquH OTBeT)

1 O Gigantism

20 Dysplasia

3O Cushing'sDisease

40Acromegaly

5 Pituitarydwarfism

10 11 12 50
4 5 6 7 8 9
Anaa anea

Bonpoc: N99
is:
The initial product for the synthesis of thyroid hormones

OTBeTbi(oAMH OTBeT)

Tyrosin

2 OLeucine

3 O Oxyproline

4 0 Phenylalanine

5 0Wallin
1213 50
6 7 8 1 11
Anana

Bonpoc: Ne11
What disorders can occur with partial hypofunction of the anterior pituitary gland:

OTBeTbi(oquH OTBeT)
1
OEmaciation

2 0 Hyperglycemia

3 OGigantism

4 OBasedow's disease
Hypogonadism

11 13 14 15 S0
9 10
1
Bonpoc: Ne10
The most common cause of Addison's disease iS

OTBETbI(oAuH OTBeT)

10 Pituitary tumor

OAutoimmune thyroiditi

3 O Adrenal atrophy

4 0 Hyperplasia of the pineal gland

5 Ovary hypertrophy

50
78 910 12 13 14

WhatsApp Desktop

Ruchi India
Paas kr gen
Bonpoc: N912
In severe cases of hypothyroidism in adults Occurs:

OTBeTbl(oqnH OTBeT)

1O Hypergonadism

2 O Cretinism

3 Myxedema

4 Eunuchoidism

5 Dwarfism
50
8 9 10 11
12 13 14 15 16
Bonpoc: N913
Primary aldosteronism (Connes syndrome) is

OTBeTbi(oAMH OTBeT)
O Tumor of the fascicular zone of the adrenal cortex

2 0Increased secretion of aldosterone under the influence of angiotensin

3 Tumor of the adrenal medulla

4O Tumor of the reticular adrenal cortex

5O Tumor of the glomerular zone of the adrenal cortex

9 10 11 12 13 14 1516 17 50
Bonpoc: Ne14
Anincrease in the concentration of thyroid-stimulating hormone in the blood during hypothyroidism indicates the localization of the pathalogical process in

OTBeTbi(oAMH OTBeT)

10 Parathyroid glands

2 0 Pituitary gland

3 Timuse

4 0 HypothalamuUs

5 OThyroid gland

10 11 12 13 14 15 16 17 18 0

O
AnaM

Bonpoc: N915
uterus:
On the basis of which pituitary hormone are drugs used to stimulate the contractile activity the
of

OTBETbi(oAuH OTBeT)
1 O Thyroid stimulating hormone

20 Follicle-stimulating hormone

3 0Prolactin
O Oxytocin

5 Vasopressin

17 50
11 12 13 14
15 16 18 19 **
AnaM 2XaseA

Bonpoc: Ne16
A patient with complaints of decreased appetite and great thirst. Diuresis is 10 liters per day. Blood sugar 4mmol /1. What disease can you think of in this case?

OTBeTbI(ogMH OTBeT)

10 Diabetes insipidus

2 Non-sugaranti-diabetes

30 Diabetes mellitus

4 ORenal diabetes

50Gestational diabetes

12 13 14 15 16 17 18 19 20 50
wn.kgma.kg

Anam Mae

Bonpoc: Ne17
skin became dry, wrinkled, wax color. Body temperature
Patient N., 27 years old, after childbirth, complicated by massive bleeding, began to develop exhaustion, the
is 3.3 mmol /1, the content of 17-ketosteroids in the urine is lowered. What
360C, blood pressure 100/60 mm Hg. Art, the content of glucose in the blood
pathology of the endocrine system can be assumed:

OTBeTbI(oAMH OTBeT)

1OHyperfunction of the adrenal cortex


Pituitaryhypofunction
Hypothyroiditis
3
4 Pituitary hyperfunction

5 OHypofunction of the ovary


14 15 16 17 18 19 20 21 50
AnaM asen

Bonpoc: Ne18
Over the past 6 months, the patient has had several bone fractures, muscle weaknes, pain in the back and legs. With a dysfunction of which gland it may be
associated:

OTBeTbI(oAuH OTBeT)
1 O Thyroid

2 0 Pituitary gland

30 Adrenalglands

4 Pancreas

O Parathyroid

22
14 15 16
17 1e 19 20 21 50
AnaM Kase

Bonpoc: Ne19
Which of the following situations can lead to the development dementia?
of

OTBeTbi(oAVH OTBeT)

Hypofunction of the thyroid gland in adulthood

2 Hypofunction of the pineal gland at an early age


early childhood
Hypofunction of the thyroid gland
in
3 0
gland in early childhood
4 0Hyperfunction of the thyroid
eariy childhood
Hyperproduction of growth hormone
in
5

21 22 23 50
17 18 19 20
15 16

11:31

G 1-01-202
Anaw Kanen

Bonpoc: Ne19
Which of the following situations can lead to the development of dementia?

OTBeTbl(oquH OTBeT)
1
Hypofunction of the thyroid gland in adulthood

2 O Hypofunction of the pineal gland at an early age

3 OHypofunction of the thyroid gland in early childhood

in early childhood
4 Hyperfunction of the thyroid gland
early childhood
5 O Hyperproduction of growth hormone in

15 16 17 18 19 20 2122 23 50

aENG
kgma.kg

AnaM 2KabeA

Bonpoc: N°20
A 45-year-old man, taken to the clinic by an ambulance team with a diagnosis of "Hypertensive crisis". BP 180/120 mm Hg. Art, myocardial hypertrophy,
-

tachycardia, weakening ot memory and intelligence, blood glucose - 6.5 mmol /1. A history of sudden crises with a rise in blood pressure up to 200/140 mm Hg. Art,
the appearance of tachycardia, sweating, sharp agitation. Nuclear magnetic resonance imaging of the lumbar region revealed an increase in the size of the left
adrenalgland, the presence of a rounded formation in the medulla of the gland. Which of the following diagnoses is most likely:

OTBeTbI(OAuH OTBeT)
10 Arterial hypertension

20 Adrenalcrisis

3 Pheochromocytoma

4 0Diabetes mellitus
50 Addison's disease

19 20
16 17 18
22 **
0

52
Bonpoc: N 21
A 30-year-old man complains of severe weakness, rapid physical fatigue, darkening of the skin, loss of appetite, nausea, vomiting, diarhea. When examned, the sen
palms) and areas
turgoris reduced, has a golden brown color. Particularly pronounced pigmentation on exposed parts of the body (face, palmar folds, back of the
weak filling. In the blood, anemia, lymphocytosis, eosinophilia,
subject to friction from clothing. BP 90/60 mm Hg, muffled tones, pulse 82 per minute of
cortisol is reduced, a high titer of antibodies to the adrenal tissue. Which
hyponatremia, hyperkalemia, hypoglycemia. The content of corticotropin increased, and
is

of the following diagnoses is most likely:

OTBeTbi(oAMH OTBeT)

1OHemochromatosis

20 Itsenko-Cushing's disease

3 0 Essential arterial hypotension

-4 O Addison's disease
Primarybiliarycholangitis
50
17 18 9 20 2122 2 24 25 50

WhatsApp DesktopP

Omnarayan Bhai Airtel


Fail nu
kgma.kg

Ana Kane

Bonpoc: N922
cm was
with normal weight from parents with normal height. At 6 months, he
weighed 13.5 kg, at 9 years old, his height was 186 and his weight
Patient M. was born normal, there is no coarsening of facial features, he has great
physical
80 kg. At the age of 18, his height was equal to 243 cm. The proportions of the body are
strength. What pathology ot the endocrine system can be assumed:

OTBeTbI(oAMH OTBeT)

1 0 Hypofunction of the thyroid gland

2 0 Hyperfunctionofthe adrenal gland


ovary
3 0Hypofunction of
O Hyperfunction of the adenohypophysis

50Hyperfunction of the neurohypophysIS

25 26 0
18 19 20 21
22 23 24 .

e ENG
Bonpoc: N923
Determine, which of the following is the main thyroid hormone:

OTBeTbi(oAnH OTBeT)
1O thriodthyronine

2 thyroid- stimulating hormone

3 thyroxine

4 thyrocalcitonin

5 0 thyrotropin releasing hormone

19 20 21 22
23 24 25 26 27 0
gma kg

Aaw aneg

Bonpoc: N924
Describe the stool for hypothyroidism in early childhood:

OTBeTbI(oqnH OTBeT)
1 0 frequent, frothy with mucus

2 0 form of rice water

3 0 form as pea puree

persistent constipation

5 0frequent with blood

20 21 22 23
24 2526 27 28 50

O
AnaM

Bonpoc: Ne25
Indicate the main symptom in hyperglycemia syndrome:

OTBeTbI(oquH OTBeT)
polydipsia

2 0 cold sweat

3 O bad appetite

4 0tremor of the hands, body

5 O hyperactivity

21 22 23 24 25 2627 28 29 50
Bonpoc: N°26
Explain the cause of prolonged jaundice in babies with hypothyroidism

OTBeTbi(oanH OTBeT)
1 Omucous swelling of the skin

2Oincreased kidney function


3O deficiency of vitamin A in the body

40 imperfect function of the gallbladder

insufficient secretion of bile by the liver

30 50
1 22 24 25 26 27 28 29
a.kg

AnaM Xasea

Bonpoc: N°27
Explain the reason for the large tongue in a child with hypothyroidism:

OTBeTbI(oquH OTBeT)
1 mucous Swelling of the tongue
2O innervation disorder

disorder of the musculature of the tongue

4 impaired circulation of the oral cavity

5O disorder of the salivary glands

23 24 25 26 27 29 30 31 50
kgma.kg
AnaM Kasea

Bonpoc: Ne28
A girl, 9 years old with her mother turned to the family doctor with complaints of constipation, decreased memory and academic pertormance, overwelgnt
cm
drowsiness that appeared over the past six months. Anamnesis: grew and developed satisfactorily. The family uses little seafood. Objectively: Hight-127,0 (N
135cm), weight-46cm (N-42 cm). Thyroid gland 1st degree, soft. Heart tones are clear and rhythmic. Survey: General blaod analysis: erythrosytes #,Txi012/1, Hb
-
107g/lleukocytes 4,8x10 9/1. Hormones: 14 norm, TSH-norm. ECG no pathology. Ultrasonography. the total volume of the thyroid gland 16 cm3. Summarize all
the data and decide what pathology the patient has?

OTBeTbI(oAVH OTBeT)

1O getative-vascular dystonia

20 deficiency anemia

3 neurasthenia

hypothyroidism

50 iodine deficiency state

24 25 26 27
28 293031 32
50

1133
Ehd 11-01-2021
O
Bonpoc: N 29

Which of the following coresponds to impaired fasting glucose (capllary plasma)

OTBeTbI(oanH OTBeT)
10fasting glucose 9.5 mmol /1, 2 hours after
- OGTT 12.8 mmol/I
fasting glucose- 7.9 mmol /1, 2 hours after OGTT 10.7 mmol /I

3 0 fasting glucose 8.6 mmol /1, 2 hours after OGTT- 11.4 mmol / I

4 O fasting glucose- 5.9 mmol / 1, 2 hours after OGTT 6.8 mmol /I

50 fasting glucose- 6.8 mmol /, 2 hours after OGTT -9.5 mmol /I

28 29 39 31 32 33 50
25 26 27
Bonpoc:N930
Waist circumference in men is normally less than (cm)

OTBeTbI(oguH OTBeT)
1
O110
2 94

3 O86

4 0102
5 O 78

33 34
26 27 28 9
3031 32
Bonpoc: N931
Describe face change in hypercortisolism syndrome

OTBeTbi(oAuH OTBeT)
1 O sardonic

2 Omasklike

30 suffering

4 excited

moon

34 35
27 28 29 30
Bonpoc: N932
Pigmentation of the eyelids (upper) with thyroid hyperfunction syndrome is a symptom

OTBeTbI(oquH OTBeT)

OMoebius

2 O Raceman

3 OElinek

4 O Krause

5 OKocher

28 29 30 31
32334 35 36 0
Bonpoc: N933
Obesity is often observed in the syndrome

OTBeTbi(oqUH OTBeT)

1 Ohypopituitarism

2 O hypocorticism

hypothyroidism

4 0 hypoparathyroidism

5 Ohypoglycemia

29 30 31 32
3334 35 36 37 50
Bonpoc: N934
Indicate, what change in the cardiovascular system is most typical for the obesity syndrome?

OTBeTbI(oqMH OTBeT)
1 displacement of the right border

2Opulse soft, low tension

3 arrhythmia of heart sounds

4 apical impulse is weakened, low

5 increase blood pressure

38 50
30 31 32 3334 36 37
Bonpoc: N935
Causes of primary hyperparathyroidism:

OTBeTbi(oquH OTBeT)
1 Olack of calcium in food

2 pituitary adenoma

3 Odysfunction of the hypothalamus

4 emoval of the thyroid gland

nyperplasia of the parathyroid glands

37 38 39 50
31 32 33
3435
Bonpoc: N936
is normal (mmol 7 ):
The fasting glucose level in the plasma of venous blood

OTBeTbI(oauH OTBeT)

1O4.0-6.1

2O9.3- 10.0

3O 3.3-5.5

4 8.1-8.9

O 6.4-7.0

39 40 50
37 38
32 33 34 35
36
Bonpoc: Ne37
Indicate in which syndrome arterial hypotension is most often observed?

OTBeTbI(oanH OTBeT)
1 O hyperthyroidism

2 hyperglycemia

3 Oacromegaly

4 Ohypoparathyroidism

5 OhypocortiCism

39 40 41 50
34 35 36 37
1 33
Bonpoc: N 38
The most common cause of primary hypocorticism is

OTBeTbi(oAuH OTBeT)

tuberculosis

2 Ocorticotropinoma

3O meningitis

4 corticosteroma
hypophysectomy
5O
50
34 35 36 37 38
40 41 42
Bonpoc: N939
acute thyroiditis:
the most informative method for examining the thyroid gland
is
in
What

OTBETbi(oaMH OTBer)

1 O radioisotope scanning

nspection and palpation

3 lateral radiography of the neck

40 auscultation of the thyroid gland

5 ultrasound with biopsy


43 50
41 42
35 36 37 38 39
4
Bonpoc: N940
A 55-year-old man, a plumber at a tuberculosis hospital, called an ambulance, complaining of severe weakness, decreased appetite, nausea, vomiting, and
abdominal pain, Objectively: the patient is adynamic, weight 49 kg, height 170 cm, darkening of the skin of the face, neck, arms, breast nipples. The pulse on the
radialartery is soft, low voltage. BP 80/40 mm Hg. Indicate the most likely predisposing factor for this syndrome

OTBeTbi(ognH OTBeT)
cachexia

20tuberculosis
3 0 hypotension

4 0 age

5 gender

36 37 38 39 40 41 42 43 44 50

1134
O 01-2021
Bonpoc: N941
A 57-year-old woman was admitted to the endocrinology department of a clinical hospital with complaints of weakness, headache, dry mouth, increased blood
pressure up to 160/90 mm Hg. pain in the region of the heart, in the lumbar spine during exercise. Objectively: rounded face, cheek rubeosis, height- 167 cm,
weight - 92 kg. BMI 34.0 kg / m2, waist circumference 120 cm, hip circumference 100 cm, pale striae on the skin of the abdomen and thighs. Determine what is
-

the degree of obesity by body mass index?

OTBETbi(oanH OTBeT)
1 0 V

20 0
3 OI
4 O

5OI
1 37 38 39 40
41 4343 44 45 50
Bonpoc: NO42
A 17-year-old boy was admitted to the clinic in an unconscious state. According to the trainer, he felt good before training, the boy has been sufering from diabetes
sincechildhood; during training, he suddenly lost consciousness. Objectively: The skin is moist. Breathing is vesicular, Heart sounds are preserved. BP 120/90 mm Hg
Laboratory glucose- 2.7 mmol71. The attending physician diagnosed him with hypoglycemia, prescribed a glucose solution, and the patient regained consciousness.
Explain the cause of hypoglycemia in this case?

OTBeTbi(oAMH OTBeT)
10 drink alcohol

2 metformin overdose
insulin overdose

4 physical activity

50 untimely meal

1 38 39 40 41
2 44 45 46 50

11:34
A ENG 11-01-202
Bonpoc: N943
palpitations.
weakness, The above complaints are disturbing for 1
A 40-year-old man has turned to a polyclinic therapist with complaints of headaches, weight gain, - 162 cm, enlargement of the breasts, purple-red
examination: face is rounded, acne, weight- 110 kg, height
year after sutfering a traumatic brain injury. On the
legs. Find out from the description the objective symptoms of the patient.
streaks of stretching of the skin on the abdomen, thighs, thinning of the arms and

OTBeTbI(oquH OTBeT)

1 O fish mouth, obstetrician hand

2 hirsutism, macroglossia

3 Omacrognathia, diastema

40 exophthalmos, myxedema

5 gynecomastia, striae

45 46 47 50
39 40 41 42 45 44

134
ENG
e
n.kgma.kg
AnaM KaseA

Bonpoc: Ne44
Anamnesis: fell ill 6 months agO after receiving a
man, consulted a therapist, complaints: weight loss, irritability, neck thickening, fever, palpitations.
A 2/-year-old kg, moist skin, tremors of the hands and whole body. Heart sounds
are increased.
brain injury. Objectively: a patient with low nutrition, height -168 cm, weight 53 analysis is necessary in this case first of all
minute. The doctor diagnosed hyperthyroidism syndrome. What
Blood pressure 110/70 mm Hg. Heart rate 100 beats per

OTBeTbi(oguH OTBeT)

TSH, T4 free

0 vasopressin
2

3 parathyroid hormone

4 0prolactin
5 Ocortisol
48 50
40 41 42 43
44 45 46 47

1134
ENG
e án 11-01-2021
Bonpoc: Ne45
Patient, 51 years old. At the endocrinologist's appointment for the first time. Complaints of increased fatigue, overweight. From the anarmresiu the patert s mgher
was obese. Smokes since 20 years. Objectively height 173 cm, body weight 102 kg The type of distribution of adipose tissue is abdomindi otety, The
mmo
in s rormal
incolor, dry to the touch. Blood pressure-110/70 mm Hg, heart rate - 76 per minute. Laboratory resuits: fasting g'ycemia 10.5 fL Expan, utat caxied te
development of hyperglycemia in this patient?

OTBETbi(oAMH OTBeT)

smoking

heredity

3O hypertension

40 hypodynamia

5 obesity

41 42 43 44 45 647 48 49 50

139
NG
e
n.kgma.kg
ARaM Casea

Bonpoc: N946
a
A 50-year-old man was admitted to the clinic with complaints of dry mouth, thirst, frequent urination, weakness. Anamnesis, the above complaints bother tor
pressure 150/90 mm Hg. Laboratory. blood glucose-12.0 mmol.
month. Objectively: The skin is dry and clean. Breathing is vesicular. Heart sounds are saved. Blood
The doctor diagnosed hyperglycemia. Explain the explanation for the elevated blood glucose level
in this case?

OTBeTbI(oAUH OTBeT)

1 0violation of enzyme secretion

2 0Impaired secretion of somatostatin

O impaired insulin secretion

4 0 impaired glucose secretion

5 O violation of glucagon secretion

47 48 49 50
+2 43 44 45
46

ENG
&
AanaEA

Bonpoc: N947
Patient S, 39 years old, was admitted to the endocrinology department of a clinical hospital with complaints of weakness, a headache in the occipital region when
the weather changes, an increase in blood pressure up to 190/100 mm Hg. pain in the heart, in the lumbar spine during physical exertion, menstrual iregulantes,
Objectively: "moon" face, cheek rubeosis, height- 165 cm, weight- 112 kg, waist circumference 120 cm, thigh circumference 100 cm, numerous small
subcutaneous hemorhages on the skin of the arms, neck, abdomen, legs, abdominal skin streaks of purple-red stretch. What is the most informatve examinaton
method for making a diagnosis?

OTBeTbi(ogMH OTBeT)

1 0Ultrasound examination

20Radiography

30Electrocardiogram

Computed tomography

5 Echocardiography

44 45 46 47 48 49 50
5
Bonpoc: N 48
A man V. 40 years old, was admitted to the clinic with multiple pathological, spontaneous fractures of tubular claws, pronounced curvature of the spine and
deformation of the skeleton. Upon admission, the patient complained of general weakness, extremely rapid fatigue, muscle hypotonia, and bone pain. In the
anamnesis there are indications of the presence of kidney stones in the patient, severe polyuria and an increased content of phosphates in the urine. What
laboratory changes will this syndrome have?

OTBeTbI(oAMH OTBeT)

1O hypercalcemia

20 hypernatremia

3 hyperfluoremia

40 hyperphosphatemia

50 hyperkalemia

43 14 45 46 47 48 49 50

O T35
Bonpoc:N949
A 35-year-old female patient complains of muscle weaknes, rapid physical fatigue, lack of appetite, nausea, muscle pain, addiction to salty food, menstrual
irregularities (delayed menstruation), She had a history of pulmonary tuberculois 3 years ago, received inpatient treatment in a tuberculosis hospital. Weight 49 1g.
height- 168 cm. The skin has a smoky bronze color, hyperpigmentation in open areas of the body, friction areas of cdothing, niples of the mammary glands, datk
spots on the oral mucosa, subcutaneous fatty tissue is thinned. BP 90/60 mm Hg. What laboratory research method will help establish
a diagnosis?

OTBETbI(oAMH OTBer)

1 estrogen

2 vasopressin

3Oinsulin

40 cortisol

5 0 thyroxine

43 44 45 46 7 48 49 p

1135
E 11-01-20T
Bonpoc: NO50
A 19-year-old girl was admitted to the clinic unconscious. According to her mother, the girl has been suffering from diabetes for 4 years, at home after an injection
of insulin she suddenly lost consciousness. The ambulance doctor revealed hypoglycemia of 2,3 mmol /I. First aid was given by intravenous glucose drip, but the
patient did not regain consciousness. Indicate which drug should be administered in this situation?

OTBeTbi(oAMH OTBeT)

10glucose

2 0 albumin

glucagon.
4 0 insulin

5 inulin

43 44 45 46 47 48 49
50

1135
NG
11-01-2042T
O Aavn.kgma.kg/webtest/testing

LWaxees AnaM Kg Ru

Bonpoc: N91
Examination of the pituitary
gland according to the standard
slice thickness of 3.0 mm in technique, with a
T1, T2 (Ax, Sag). The pituitary
dimensions: sagittal 13.7 mm, transverse gland is not enlarged,
16.3 mm, vertical 9.6 mm, pituitary
pedicle 2.8 mm. The contours
of the pituitary gland are smooth,
homogeneous. The pituitary funnel the structure is
is not deflected. In the structure
neurohypophysis (left), on postcontrast of the
T1 scans, a hypovascular
dimensions of 3.1-2.7-4.1 mm is determined. zone with
Retrobulbar spaces without
pathology.Areas of chiasm and cavernous
sinuses are not changed.The
pathology of the brain substance in the
scan area is not determined.In the cavity
of the main, maxillary sinus,
there is parietal fluid.What conclusion did you
based on the description: make

OTBETbI(OAMH OTBeT)
1
MR signs of a pituitary cyst

2 MR signs of pituitary macroadenoma

3 MR signs of endosellar pituitary


adenoma
4 MRI Signs of Infrasellar Pituitary Adenoma

5 MR signs of pituitary microadenoma

2 3 4 5 6 50

12 3 4 5 6 7 8 9 10
11 12 13 14 15 16 17 18 19 20
21 22 23 24 25 26 27 28 29 30
31 32 33 34 35 36 37 38 39 40
41 42 43 44 45 46 47 48 49 50

OTBeyeHHblM BONpoc

nponyueHHbIM BOnpoc
TeKyuu Bonpoc

Ao aaBepuweHna TeCTa oCTanocb:

1:15:06
Scanned with CamScanner
11 10:06 "U" LTE 44)

O AA avn.kgma.kg/webtest/testing
LLWaxBe3 AnaM Kg Ru

Bonpoc: Ne2
Name the main screening method of imaging the thyroid gland:

OTBeTbI(oqMH OTBeT)
1
radioisotope

2 ultrasound
3 magnetic resonance imaging

4 thermalimaging
5 radiography

2 3 4 5 6 7 50

12 3 4 5 6 7 8 9 10
11 12 13 14 15 16 17 18 19 20
21 22 23 24 25 26 27 28 29 30
31 32 33 34 35 36 37 38 39 40
41 42 43 44 45 46 47 48 49 50

OTBeueHHblM BOnpoc
nponyujeHHblM Bonpoc
Tekyuinn Bonpoc

Ao 3aBepweHnA TECTa oCTanocb:

1:15:03
3apepwnTb TeCT

Scanned with CamScanner


10:06 U" LTE1 4)

A avn.kgma.kg/webtest/testing

LLWaxBes AiaM Kg Ra

Bonpoc: N93
Name which diagnostic method provides reliable information about the pituitary
gland:

OTBETbi(ogMH OTBET)

1 x-ray

2 endoscopy

3 magnetic resonance imaging

4 sonography
5 computed tomography

2 3 4 5 7 *** 50

1 2 3 4 5 6 7 8 9 10
11 12 13 14 15 16 17 18 19 20
21 22 23 24 25 26 27 28 29 30
31 32 33 34 35 36 37 38 39 40
41 42 43 44 45 46 47 48 49 50

OTBeyeHHbiM BonpOC
nponyueHHblM Bonpo
TeKyuuÄ BonpoOc

Ao 3aBepweHnA TeCTa oCTanocb

1:15:01
3aBepunTb TECT

Scanned with CamScanner


11 10:06 "U" LTET (44)

A avn.kgma.kg/webtest/testing
LLWaxBes AnaM Kg Ru

Bonpoc: N94
Patient B, 39 years old, who for 8 years has not been able to get pregnant, was
advised to consult an endocrinologist. Examination revealed exophthalmos,
hand tremor, tachycardia in the patient. Ultrasound showed an increase in the
size of the gland. Explain what disease is depicted on the ultrasound image?

OTBETbi(OAMH OTBET)
1 hypothyroidism

2 chronic autoimmune thyroiditis

3 Otoxic adenoma
4 diffusetoxic goiter
5 diffuse non-toxic goiter

1 2 3 4 5 6 7 50

1 2 34 5 6 7 89 10
11 12 13 14 15 16 17 18 19 20
21 22 23 24 25 26 27 28 29 30
31 32 33 34 35 36 37 38 39 40
41 42 43 44 45 46 47 48 49 50

OTBeyeHHblM BOnpoc
nponyueHHbl Bonpoc
TeKyuun Bonpoc

Ao 3aBepweHnA TECTa oCTanoCb:

1:14:58
SaBepunTb TeCT

Scanned with CamScanner


11 10:06 "U" LTE1 (44)

O AA avn.kgma.kg/webtest/testing
LWaxBes AnaM Kg R&

Bonpoc: N95
Patient K.,32 years old, complains of unreasonable anxiety, increased mental
excitability, weight loss, sweating, frequent stools. She fell ill for 6 months. back
after suffering the flu. Revealed eye symptoms (Grafe, Kraus). Pulse 100 in 1
minute. The thyroid gland is enlarged to 3 tbsp. What is the method of radiation
diagnostics that allows in this case to assess the function of the thyroid gland:

OTBeTbI(OqMH OTBET)

1 radioimmunoassay.

2 computed tomography of the thyroid gland.

3 doppler study.

4 nuclear magnetic resonance.


5 ultrasonic scannng.

1 2 3 4 5 6 7 8 9 ** 50

1 2 3 4 5 6 7 89 10
11 12 13 14 15 16 17 18 19 20
21 22 23 24 25 26 27 28 29 30
31 32 33 34 35 36 37 38 39 40
41 42 43 44 45 46 47 48 49 50

OTBeYeHHbli Bonpoc
nponyueHHblM BONpoc
TeKyuu BOnpoc

Ao 3aBepweHna TecTa oCTanocb

1:14:56
3aBepuwTb TeCT

Scanned with CamScanner


11 10:06 U LTE1 44)

A avn.kgma.kg/webtest/testing
LlaxBes AnaM Kg Ru

Bonpoc: N96
The thyroid gland is of a typical type and location, the contours are well-defined,
the total volume is 23 cubic cm. The isthmus is 8 mm thick, the contours are
even, theechostructure is isoechoic, homogeneous. The right lobe is 50 x 24 x
23 mm, with a volume of 13 cm3, the contours are even, clear, the echo
structure of diffusely increased echogenicity, heterogeneous due to anechoic
inclusions and linear hyperechoic signals that impart lobularity to the structure.
The left lobe measures 52 x 18 x 22 mm, with a volume of 10 cm3, the contours
are even, clear, the echo structure of diffusely increased echogenicity,
heterogeneous due to anechoic inclusions and linear hyperechoic signals,
imparting lobularity to the structure. Regional lymph nodes are not visualized.
Choose the correct conclusion option:

OTBETbI(oAMH OTBET)

1
echographic signs of subacute thyroiditis

2 echographic signs of focal thyroiditis

3 echographic signs of chronic autoimmune thyroiditis

4 echographic signs of acute non-purulent thyroiditis

5 echographic signs of diffuse toxic thyroiditis

1
2 3 4 5 6 7 8 9 10

50

1 2 3 4 5 6 7 89 10
11 12 13 14 15 16 17 18 19 20
21 22 23 24 25 26 27 28 29 30
31 32 33 34 35 36 37 38 39 40
41 42 43 44 45 46 47 48 49 50

OTBeyeHHblM BONnpoc
nponyyeHHblM BOnpo0c
TeKyun Bonpoc

Ao 3aBepweHnA TCTa oCTanocb

Scanned with CamScanner


10:06 U" LTE1 44

O A avn.kgma.kg/webtest/testing
laxees AnaM Kg Ru

Bonpoc: Ne7
With eosinophilic adenoma of the pituitary gland during the growth period of the
body develops:

OTBeTbi(ogMH OTBET)

1 Gigantism

2 Cushing'sDisease

3 Dysplasia
4 Pituitarydwarfism

5 Acromegaly

1 3 4 5 6 7 8 9 10 11

50

1 2 3 4 5 6 7 89 10
11 12 13 14 15 16 17 18 19 20
21 22 23 24 25 26 27 28 29 30
31 32 33 34 35 36 37 38 39 40
41 42 43 44 45 46 47 48 49 50
OTBeYeHHblM BonpoOc
nponyueHHblM Bonpoc
TexyuuM BONpoc

Ao saBepweHnA TEcTa oCTanocb:

1:14:51
3aBepuTb TECT

Scanned with CamScanner


10:06 LTE1 44)

O Aavn.kgma.kg/webtest/testing

LLWaxBes AnaM Kg R&

Bonpoc: N98
The highest content of antibodies to microsomal antigen in blood is observed
when:

OTBeTbl(ogMH OTBET)
1 Thyrotoxic adenoma

2 Subacute thyroiditis

3 Diffusetoxicgoiter

4 Thyroid cancer

5 Autoimmune thyroiditis

1
** 4 5 67 8 9 10 11 12

50

1 23 4 5 6 7 9 10
8
11 12 13 14 15 16 17 18 19 20
21 22 23 24 25 26 27 28 29 30
31 32 33 34 35 36 37 38 39 40
41 42 43 44 45 46 47 48 49 50

OTBeyeHHbiM Bonpoc
nponyueHHbIÅ BOnpoc
TEKyunÄ Bonpoc

Ao saBepweHuA TeCTa oCTanoCb

1:14:49
3aBepwTb TeCT

Scanned with CamScanner


l 10:06

A avn.kgma.kg/webtest/testing
LlWaxBe3 AnaM Kg
Ru

Bonpoc: Ne9
A decrease in the production of hormones of the sex glands is manifested:

OTBETbI(OqMH OTBET)

1
Eunuchoidism

2 Hirsutism
3 Virilism

4 Hermaphroditism
5 Cryptorchidism

*** 5 6 7 8
9 10 11 12

13 50

1 2 3 4 5 6 7 8 9 10

11 12 13 14 15 16 17 18 19 20
|21 22 23 24 25 26 27 28 29 30
31 32 33 34 35 36 37 38 39 40
41 42 43 44 45 46 47 48 49 50
OTBeueHHblM BONnpoc
nponyujeHHblÅ BOnpoc
TekyunÀ BOnpOc

Ao 3asepweHwA TECTa oCTaniocb

1:14:47
3aBepuTb TECT

Scanned with CamScanner


l1 10:06

O A avn.kgma.kg/webtest/testing

Llaxee3 AnaM Kg Ru

Bonpoc: Ne10
Primary aldosteronism (Connes syndrome) is:

OTBeTbi(oAMH OTBeT)
1
Tumor of the fascicular zone of the adrenal cortex

2 Increased secretion of aldosterone under the influence of angiotensin

3 Tumor of the adrenal medulla

4 Tumor of the reticular adrenal cortex

5 Tumor of the glomerular zone of the adrenal cortex

** 6 7 8 9 10 11 12 13

14 50

1 2 3 4 5 6 7 8 9 10
11 12 13 14 15 16 17 18 19 20
21 22 23 24 25 26 27 28 29 30
31 32 33 34 35 36 37 38 39 40
|41 42 43 44 45 46 47 48 49 50

OTBeuEHHbliM Bonpoc

nponyujeHHblM Bonpoc
TexyuM BOnpoc

Ao 3aBepweHnA TEcTa oCTanocb:

1:14:44
3aBepunTb TeCT

Scanned with CamScanner


ill T0:07

OA avn.kgma.kg/webtest/testing

LlaxBe3 AnaM Kg Ru

Bonpoc: N911
Disorders of transhypophyseal regulation underlie changes in production:

OTBeTbI(oAMH OTBET)

1Ocatecholamines
2 Glucagon

3OThyroid hormone

Parathyrine

5 Insulin

*** 8 9 10 11 12 13 14

15 50

1 4 5 6 7
2 3 8 9 10
11 12 13 14 15 16 17 18 19 20
|21 22 23 24 25 26 27 28 29 30
31 32 33 34 35 36 37 38 39 40
41 42 4344 45 46 47 48 49 50

OTBeYeHHblM BOnpoc

nponyeHHbiM Bonpoc
TeKyni Bonpoc

Ao 3aBepweHwA TECTa oCTanoCb

1:14:42
SaBepuwTb TeCT

Scanned with CamScanner


10:07 "U LTE1 4)

A avn.kgma.kg/webtest/testing
LLWaxBes AnaM Kg Ru

Bonpoc: N912
In the pathogenesis of acute thyroiditis plays a role:

OTBeTbI(oqMH OTBET)
1
Injury of the thyroid gland

2 Genetic predisposition
3 Violation of the mechanisms of immunological protection

Penetration of infection into the thyroid gland

5 lodine deficiency

1 8 9 10 11 12 13 14 15

16 50

2 3 4 5 6 7 8 9 10
11 12 13 14 15 16 17 18 19 20
21 22 23 24 25 26 27 28 29 30
31 32 33 34 35 36 37 38 39 40
|41 42 43 44 45 46 47 48 49 50

OTBEYEHHblM Bonpoc

nponyuueHHblM Bonpoc
Texyunn Bonpoc

Ao 3aBepweHnA Tecta oCTaocb

1:14:40
3aBepunTb TeCT

Scanned with CamScanner


i 10:07

O Aavn.kgma.kg/webtest/testing

LllaxBe3 AiaM Kg Ru

Bonpoc: N®13

The immediate stimulant of the glomerular adrenal cortex is:

OTBETbI(OAMH OTBET)

1 Adrenaline
2 Nicotin
3 Angiotensin

4 Vasopressin
5 Acetylcholine

** 9 10 11 12 13 14 15 16

17 50

1 2 3 4 5 6 7 8 9 10
213 14 15 16 17 18 19 20
21 22 23 24 25 26 27 28 29 30
31 32 33 34 35 36 37 38 39 400
41 42 43 44 45 46 47 48 49 50

OTBeyeHHblM Bonpoc

nponyujeHHblM Bonpoc
TekyuMÀ Bonpoc

Ao 3aBepweHnA TECTa oCTanocb:

1:14:38
3aBepunTb TeCT

Scanned with CamScanner


10:07

O Aavn.kgma.kg/webtest/testing

LlaxBes AnaM Kg Ruy

Bonpoc: NP14
An excess of thyroid hormones occurs when:

OTBETbI(OAMH OTBET)

1 Endemic cretinism

2 Diffuse toxic goiter

3 Miksedeme
4 Insulinoma
5 Acromegaly

1 10 11 12 13 14 15 16 17

18 ** 50

1 2 3 4 5 67 89 10
11 12 13 14 15 16 17 18 19 20
21 22 23 24 25 26 27 28 29 30
31 32 33 34 35 36 37 38 39 40
|41 42 43 44 45 46 47 48 49 50
OTBeyeHHbiM Bonpoc

nponyujeHHbIM BOnpoc
TeKyunM Bonpoc

Ao 3aBepweHwa TeCTa oCTanoCb:

1:14:36
3aBepunTb TeCT

Scanned with CamScanner


10:07 U LTE1 44)

A avn.kgma.kg/webtest/testing 0
WaxBes AnaM Kg
R

Bonpoc: N915
A patient was brought to the admission department of the hospital, who lost
consciousness in the street. The examination revealed the smell of acetone
from the mouth. What is the preliminary diagnosis that can be made

OTBeTbI(oqMH OTBET)
1
Hepatic coma

2 Diabetic coma

3 Pulmonary embolism

4 Uremiccomma

5 Internal bleeding

11 12 13 14
15 16 17 18

19 50

1 2 3 4 5 6 7 89 10
11 12 13 14 15 16 17 18 19 20
21 22 23 24 25 26 27 28 29 30
31 32 33 34 35 36 37 38 39 40
41 42 43 44 45 46 47 48 49 50

OTBeuEHHblM Bonpooc
nponyujeHHblM Bonpoc
TeKyuwi Bonpoc

Ao 3aBepweHna TecTa oCTanoCb

1:14:34
3aBepuTb TeCT

Scanned with CamScanner


10:07 U" LTE1 (44

A avn.kgma.kg/webtest/testing

laxBes AnaM Kg Ru

Bonpoc: N®16

After removal of the thyroid gland, the patient developed convulsions. What was
damaged during the operation:

OTBETbI(ogMH OTBET)

1 Inferior laryngeal artery

2 Parathyroid gland

3 Vagus nerve
4 sympathetictrunk
5 Inferior laryngeal nerve

1 ** 12 13 14 15 16 17 18 19

20 50

1 2 3 4 5 6 7 8 9 10
1112 13 14 15 16 17 18 19 20
21 22 23 24 25 26 27 28 29 30
31 32 33 34 35 36 37 38 39 40
41 42 43 44 45 4647 48 49 50

OTBeyeHHbiMBonpoc
nponyueHHbIÅ BOnpoc
TEKyLMM Bonpoc

Ao 3aBepweHnA TeCTa oCTanocb:

1:14:32
3aBepuwTb TeCT

Scanned with CamScanner


1 10:07 "U LTE1 4)

A avn.kgma.kg/webtest/testing

LlaxBes AnaM Kg Ru

Bonpoc: Ne17
Patient N, 27 years old, after childbrth, complicated by massive bleeding.
began to develop exhaustion, the skin became dry, wrinkled, wax color. Body
temperature -360 C, blood pressure -100/60 mm Hg. Art, the content of
glucose in the blood is 3.3 mmol /1, the content of 17-ketosteroids in the urine is
lowered. What pathology of the endocrine system can be assumed:

OTBeTbI(oqMH OTBET)
1
Hypofunction of the ovary

2 Hyperfunction of the adrenal cortex

3 Hypothyroiditis
4 Pituitaryhypofunction

5 Pituitary hyperfunction

1 * 13 14 15 16 17 18 19 20

21 50

1
2 3 4 5 6 7 10 89
1112 13 14 15 16 17 18 19 20
21 22 23 24 25 26 27 28 29 30
31 32 33 34 35 36 37 38 39 40
41 42 43 44 45 46 47 48 49 50

OTBeeHHbiM Bonpoc
nponyueHHblM Bonpoc
TeKyuu BOnpoc

Ao 3aBepweHna TecTa oCTanocb

1:14:30
3aBepuTb TeCT

Scanned with CamScanner


10:07 "U LTE1 )

A avn.kgma.kg/webtest/testing
LLWaxBe3 AnaM Kg Ra

Bonpoc: Ne18
A 30-yearold woman complains of dry mouth and severe thirst, which appeared
after a severe nervous shock. Laboratory examination revealed an increase in
blood sugar up to 10 mmol/ 1. Which endocrine gland disease the patient has:

OTBeTbI(oqMH OTBET)
1
Sex

2 Epiphysis
3 Adrenal glands

4OThyroid
5 Pancreas

1 14 15 16 17 18 19 20 21

22 50

1 2 3 4 5 6 7 89 10
11 12 13 14 15 16 17 18
19 20
21 22 23 24 25 26 27 28 29 30
31 32 33 34 35 36 37 38 39 40
41 42 43 44 45 46 47 48 49 50

OTBeYeHHblM Bonpoc
nponyujeHHblM Bonpoc
TeKyuwi Bonpoc

Ao saBepweHMA TECTa oCTanocb:

1:14:28
3aBepunTb TeCT

Scanned with CamScanner


l 10:07 U' LTE1 44)

O AA avn.kgma.kg/webtest/testing
LWaxBe3 AnaM Kg Ru

Bonpoc: Ne19
Patient B, 39 years old, who has been unable to get pregnant for 8 years, was
advised to consult an endocrinologist. Examination revealed exophthalmos,
hand tremor, tachycardia in the patient. The disease of which endocrine gland is
accompanied by the following symptoms:

OTBeTbI(OAMH OTBeT)
1
Epiphysis
2 Polovykh

3 Thyroid

4 Adrenal glands
5 Pancreas

** 15 16 17 18 19 20 21 22

23 50

2 3 4 5 6 7
1
10 89
11 12 13 14 15 16 17 18 19 20

21 22 23 24 25 26 27 28 29 30
31 32 33 34 35 36 37 38 39 40
41 42 43 44 45 46 47 48 49 50
OTBeyeHHblM BOnpoc
nponyuyeHHblM BOnpoc
TekyuuM BOnpoc

Ao 3aBepueHnA TecTa oCTanocb:

1:14:25
3asepiWTb TeCT

Scanned with CamScanner


1 10:07 U' LTE1 44)

O AA avn.kgma.kg/webtest/testing
Llaxee3 AnaM Kg Ru

Bonpoc: N 20
A 45-yearold man, taken to the clinic by an ambulance team with a diagnosis of
Hypertensive crisis". BP-180/120 mm Hg. Art, myocardial hypertrophy,
tachycardia, weakening of memory and intelligence, blood glucose 6.5 mmol/ -

I.A history of sudden crises with a rise in blood pressure up to 200/140 mm Hg.
Art, the appearance of tachycardia, sweating, sharp agitation. Nuclear magnetic
resonance imaging of the lumbar region revealed an increase in the size of the
left adrenal gland, the presence of a rounded formation in the medulla of the
gland. Which of the following diagnoses is most likely:

OTBETbI(oAMH OTBET)

1
Diabetes mellitus
2 Adrenalcrisis
3 Pheochromocytoma

40Addison'sdisease
5 Arterial hypertension

1 16 17 18 19 20 21 22 23

24 50

1 2 3 4 5 6 7 89 100
11 12 13 14 15 16 17 18 19 20
21 22 23 24 25 26 27 28 29 30
31 32 33 34 35 36 37 38 39 40
41 42 43 44 45 46 47 48 49 50

OTBeeHHbli Bonpoc
nponyujeHHblM Bonpoc
TeKyn Bonpoc

Ao 3aBepweHnA TecTa oCTanocb

1:14:23
BaBepuMTb TeCT
Scanned with CamScanner
l 10:07 "U LTE1 44)

O Aavn.kgma.kg/webtest/testing

Llaxee3 AnaM Kg Ru

Bonpoc: Ne21
A 3-year-old boy was admitted to the clinic for childhood diseases. On
examination: the child's height is much lower than the age norm, short neck,
arms and legs, big belly; the head is large, the nasal bridge is sunken, the face is
puffy, pale, mask-like, the mouth is open, the tongue is enlarged, there are many
carious teeth, the speech is slurred, the spoken words are distorted, the
syllables in them are often rearranged; the child does not always answer
questions, he is capricious and irritable. BP 80/50 mm Hg. Art., pulse 45 per
minute. The thyroid gland is painless on paipation, not enlarged. The child's
mother took iodine preparations during pregnancy. Diagnosis "Congenital
hypothyroidism. Cretinism". The initial and pathogenetic ink of this disease is:

OTBeTbi(OAMH OTBET)
1
Increasein FSH and LH
2 Deficiency of T3 T4

3 Low TSH levels


4 Lack ofiodine
5 Lack of micronutrients

17 18 19 20 21 22 23 24

25 50

2 3 4 5 6 7 8 9 10
1

11 12 13 14 15 16 17 18 19 20
21 22 23 24 25 26 27 28 29 30
31 32 33 34 35 36 37 38 39 40
41 42 43 44 45 46 47 48 49 50

OTBeYeHHblM Bonpoc
nponyueHHbiM Bonpoc
TexyunM BOnpoc

o 3aBepweHnA TecTa oCTanocb

1:14:21
Scanned with CamScanner
U LTE1 44)
10:07

O A avn.kgma.kg/webtest/testing
LWaxees AnaM Kg Ru

Bonpoc: N 22
Female, 52 years old, onset of the disease 1.5 years ago after a course of
radiation therapy. Complains of memory impairment, general weakness,
drowsiness, baldness, lack of appetite, weight gain. An objective examination
revealed: the skin is dry, flaky, the face is puffy, hypomimic, there are areas of
baldness on the head, the voice is low, rough, speech is slowed down, the body
.
temperature is 35.7 the pulse is 52 beats. per minute, blood pressure 110/65 -

mm Hg. Art, the level of glucose in the blood-3.5 mmol/, the basal
metabolism is reduced by 30%. Diagnosed with Myxedema. What is the most
likely cause of this disease:

OTBeTbi(oquH OTBET)
1 Increase in T3 T4

2 Formation of mucin

3 Formation of reverse T3

4 lodine deficiency
5 Decreased TSH levels

18 19 20 21 22 23 24 25

26 50

1 2 3 4 5 6 7 89 10
11 12 13 14 15 16 17 18 19 20
21 22 23 24 25 26 27 28 29 30
31 32 33 34 35 36 37 38 39 40
|41 42 43 44 45 46 47 48 49 50

OTBeEHHblM BONpoc
nponyuyeHHblM BOnpoc
TeKyunn Bonpoc

Ao 3aBepweHMA TECTa oCTanocb

1:14:18
Scanned with CamScanner
U LTET 44)
10:07

O A avn.kgma.kg/webtest/testing

LWaxBe3 AnaM Kg Ru

Bonpoc: N°23
Indicate the main symptom in hyperglycemia syndrome

OTBeTbI(oAMH OTBET)

1
bad appetite
2 polydipsia

3 cold sweat

4 tremor of the hands, body

5 hyperactivity

1 19 20 21 22 23 24 25 26

21 50

1 2 3 4 5 6 7 8 9 10
11 12 13 14 15 16 17 18 19 20
21 22 23 24 25 26 27 28 29 30
31 32 33 34 35 36 37 38 39 40
41 42 43 44 45 46 47 48 49 50

OTBeYeHHblM Bonpoc
nponyueHHbIM BOnpoc
TexyumM BOnpoc

0 3aBepweHuA TECTa oCTanocb

1:14:15
3aBepuwTb TeCT

Scanned with CamScanner


10:07 U LTE1 44)

A avn.kgma.kg/webtest/testing

LlaxBe3 AnaM Kg R&

Bonpoc: Ne24
Indicate the hormone that regulates the secretion of thyroid-stimulating
hormone by the pituitary gland:

OTBETbI(ogMH OTBET)

1 thyroxine
2 adrenocorticotropic hormone

3 thyrocalcitonin

4 thyroglobulin
5 thyrotropin releasing hormone

1
** 20 21 22 23 24 25 26 27

28 50

1 2 3 4 5 6 7 8 10 9
11 12 13 14 15 16 17 18 19 20
21 22 23 24 25 26 27 28 29 30
31 32 33 34 35 36 37 38 39 40
41 42 43 44 45 4647 48 49 50

OTBEyeHHbin BOnpoc
nponyuyeHHblM BOnpoc
TeKyunn Bonpoc

Ao saBepweHuA TeCTa oCTanocb

1:14:13
3aBepunTb TeCT

Scanned with CamScanner


10:07

O A avn.kgma.kg/webtest/testing

LWaxBes AnaM Kg Ru
Bonpoc: Ne25
Name the main laboratory criterion of hyperglycetmia syndrome:

OTBeTbI(oAMH OTBET)

1
hypercholesterolemia

2 hypoproteinemia
3 high fasting glycemia
4 hypernatriemia
5 hypokaliemia

21 22 23 24 25 26 27 28

29 * 50

1 2 3 4 5 6 7 8 9 10
11 12 13 14 15 16 17 18 19 20
2122 23 24 25 26 27 28 29 30
31 32 33 34 35 36 37 38 39 40
41 42 43 44 45 46 47 48 49 50
OTBeYeHHblM Bonpoc
nponyueHHblM Bonpoc
TeKyunM Bonpoc

Ao aa8epuweHuA TecTa oCTanoCb:

1:14:11
3aBepuuwTb TeCT

Scanned with CamScanner


10:07 U LTE1 44)

A avn.kgma.kg/webtest/testing
LlaxBe3 AnaM Kg Ru

Bonpoc: N 26
Identify the main cause of constipation and flatulence in children with
hypothyroidism:

OTBeTbI(oquH OTBET)
1
decreased physical activity

2 decrease metabolism of substances

3 poor appetite

4 motor function of the intestine

5 low water consumption

1
* 22 23 24 25 26 27 28 29

30 50

1
2 3 4 5 6 7 8 9 10
11 12 13 14 15 16 17 18 19 20
2122 23 24 25 26 27 28 29 30
31 32 33 34 35 36 37 38 39 40
41 42 43 44 45 4647 48 49 50

OTBeyeHHbiMBonpoc
nponyueHHbIÅ BOnpoc
TEKyunÄ Bonpoc

Ao 3aBepweHnA TeCTa oCTanocb:

1:14:09
3aBepuwTb TeCT

Scanned with CamScanner


10:07 " LTE1

A avn.kgma.kg/webtest/testing

laxBes AnaM Kg
Ru

Bonpoc: N 27
Explain, as a result of which there is overweight in children with hypothyroidism
syndrome:

OTBeTbI(oqMH OTBET)
1
acceleration of oxidative processes

2 violation of fat metabolism

3 myxedema edema

4 OIiver dysfunction

5 disaccharide metabolism disorders

1 23 24 25 26 27 28 29 30

31 50

1 2 3 4 5 6 7 8 9 10
11 12 13 14 15 16 17 18 19 20
21 22 23 24 25 26 27 28 29 30
31 32 33 34 35 36 37 38 39 40
41 42 43 44 45 46 47 48 49 50

OTBeyeHHbiMBonpoc
nponyuyeHHblM BOnpoc
TEKyunÄ Bonpoc

Ao 3apepweHnA TECTa oCTanoCb:

1:14:07
3aBepuTb TECT

Scanned with CamScanner


10:07 "U LTE1 44)

O AA avn.kgma.kg/webtest/testing
LLWaxee3 AiaM Kg Ru

Bonpoc: N 28
Baby, 3 month, with mother turned to the family doctor with complaints on rare
stool 1 time in 2 days, drowsiness, difficulty swallowing. Anamnesis: 2-nd
pregnancy with toxicosis, 2nd delivery, urgent, sucked sluggishly. The umbilical
residue disappeared on the 16th day, jaundice was noted up to 1,5 months.
Objectively: physical development: height 52 sm, weight 6200,0 The face is
edematous, the tongue is large, sticking out. The skin is pale with yellowness,
dry. Heart tones very weak, bradycardia. Heart rate 100 in 1 min. What is the first
diagnostic test to use?

OTBeTbi(onMH OTBeT)
1
transaminases
2 Chest Xray

3 general urine analysis

4 cholesterol, bilirubin

5 T4, TSH

1 ** 24 25 26 27 28 29 30 31

32 50

1 2 3 4 5 6 7 10 89
11 12 13 14 15 16 17 18 19 20
21 22 23 24 25 26 27 28 29 30
31 32 33 34 35 36 37 38 39 40
41 42 43 44 45 46 47 48 49 50

OTBeYeHHbl Bonpoc
nponyueHHblM Bonpoc
TeKyn Bonpoc

Ao 3aBepweHnA TecTa oCTanocb

1:14:05
BaBepulMTb TeCT
Scanned with CamScanner
Aavn.kgma.kg/webtest/testing

WaxBe3 Anam
Kg Ra

Bonpoc: N929
The wide opening of
the eye slits for thyroid
symptom: hyperfunction syndrome
is a

OTBETbI(OAMH OTBeT)
1 Shtelvag

2 Kocher
3 Grefe
Delrymple

5 Elinek

25 26 27 28 29 30 31 32
33 50

1 2 3 4 5 67
10 89
11 12 13 14 15 16 17 18 19 20
21 22 23 24 25 26 27 28 29 30
31 32 33 34 35 36 37 38 39 40
41 42 43 44 45 46 47 48 49 50

OTBeyeHHbii Bonpoc
nponyueHHblM BOnpoc
TeKyuni Bonpoc

o 3aBepweHna TecTa oCTanocCb:

1:14:03
3aBepuTb TeCT

Scanned with CamScanner


10:07 "U" LTE )

A avn.kgma.kg/webtest/testing
LlaxBes AnaM Kg Ru

Bonpoc: N 30
Obesity is often observed in the syndrome

OTBeTbI(oqMH OTBeT)
1 hypoglycemia

2 hypoparathyroidism

3 hypothyroidism

hypopituitarism

5 hypocorticism

1 26 27 28 29 30 31 32 33

34 50

1 2 3 4 5 67 10 89
11 12 13 14 15 16 17 18 19 20
21 22 23 24 25 26 27 28 29 30
31 32 33 34 35 36 37 38 39 40
|41 42 43 44 45 46 47 48 49 50

OTBeueHHblM Bonpoc
nponyueHHbIM Bonpoc
TeKyunn Bonpoc

Ao 3aBepueHnA Tecta ocTanocb

1:14:01
3aBepuTb TeCT

Scanned with CamScanner


10:07 "U" LTE1 44)

O AA avn.kgma.kg/webtest/testing

LlaxBes AnaM Kg Ru

Bonpoc: N 31
What symptom is characterized by increasing the irritability of the nerves, and a
flexion of the wrist and metacarpal phalangeal joints with extension of the
interphalangeal joints and adduction of the thumb (carpal spasm) after placing
a blood pressure cuff on the patient's arm and inflating to 20 mm Hg above
systolic blood pressure for 3-5 minutes?

OTBeTbl(oAMH OTBET)
1 Khvostek

2 Schlesinger

3 Weiss
4O Trousseau

5 fish mouth
27 28 29 30 31 32 33 34

35 50

1
2 3 4 5 6 7 89 10
11 12 13 14 15 16 17 18 19 20
21 22 23 24 25 26 27 28 29 30
31 32 33 34 35 36 37 38 39 40
41 42 43 44 45 46 47 48 49 50

OTBeeHHbiM Bonpoc
nponyujeHHblM BONpoc
TeKyuuÅ BOnpoc

Ao 3aBepweHna TecTa oCTanocb

1:13:58
3aBepunTb TeCT

Scanned with CamScanner


11 10:07

O A avn.kgma.kg/webtest/testing

LWaxaea AnaM Kg R

Bonpoc: N932
Indicate in which syndrome arterial hypotension is most often observed?

OTBETbi(qMH OTBET)
1
hyperthyroidism
2 acromegaly
3 hypoparathyroidism

hypocorticism

5 hyperglycemia

***| 28 29 30 31 32 33 34 35

36 50

1 2 3 4 5 67 8 9 10
11 12 13 14 15 16 17 18 19 20
21 22 23 24 25 26 27 28 29 30
31 32 33 34 35 36 37 38 39 40
41 42 43 44 45 46 47 48 49 50

OTBeyeHHblM BOnpoc
nponyujeHHblÅ BOnpoc
TekyunM BOnpoc

Ao 3aBepweHua TECTa oCTanocb:

1:13:55
3aBepuwTb TeCT

Scanned with CamScanner


10:07 U LTE1 44)

A avn.kgma.kg/webtest/testing
LLWaxBes Anam Kg Ru

Bonpoc: N933
The most common cause of secondary hypercortisolism is

OTBeTbi(oAMH OTBET)
1
corticotropinoma
2 corticosteroma
3 meningoencephalitis
4 pheochromocytoma
5 hypophysectomy

** 29 30 31 32 33 34 35 36

37 50

1 2 3 4 5 67 89 10
11 12 13 14 15 16 17 18 19 20
21 22 23 24 25 26 27 28 29 30
31 32 33 34 35 36 37 38 39 40
41 42 4344 45 46 47 48 49 50

OTBeueHHblM Bonpoc
nponyuyeHHbIM BOnpoc
TeKyunM BoNnpoc

o 3aBepweHna TecTa oCTanocb

1:13:52
3aBepuMTb TeCT

Scanned with CamScanner


10:07

A avn.kgma.kg/webtest/testing
LlWaxBe3 AiaM Kg Ru

Bonpoc: N934
The most common cause of primary hypocorticism is

OTBETbI(OqMH OTBeT)

1
hypophysectomy
2 tuberculosis

3 corticosteroma
4 meningitis
5 corticotropinoma

1 30 31 32 33 34 35 36 37

38 50

1 2 3 4 5 6 7 8 9 10
11 12 13 14 15 16 17 18 19 20
21 22 23 24 25 26 27 28 29 30
31 32 33 34 35 36 37 38 39 40
|41 42 43 44 45 46 47 48 49 50

OTBeyeHHblM BOnpoc

nponyueHHbIM BOnpoc
TeKyuni Bonpoc

Ao 3aBepweHWA TeCTa oCTanoCb

1:13:49
3aBepuTb TeCT

Scanned with CamScanner


A avn.kgma.kg/webtest/testing

LlaxBe3 AnaM Kg Ru

Bonpoc: N935
Characteristics of the pulse on
the radial artery in hypocorticism syndrome

OTBeTbi(ogMH OTBET)
1
soft

2 arrhythmic
3 tense
4 high
5 resistant

31 32 33 34 35 36 37 38
39 * 50

1
2 3 4 5 6 7 10 89
11 12 13 14 15 16 17 18 19 20
21 22 23 24 25 26 27 28 29 30
31 32 33 34 35 36 37 38 39 40
41 42 43 44 45 46 47 48 49 50
OTBeyeHHbiM BOnpoc
nponyujeHHbiM BOnpoc
TeKyunM Bonpocc

o 3aBepweHwa TeCTa oCTanocb:

1:13:48
3aBepuuUTb TeCT

Scanned with CamScanner


l 10:07

A avn.kgma.kg/webtest/testing

LlaxBes AnaM Kg Ru

Bonpoc: N936
Which drug should be prescribed for acute hyperglycemia?

OTBeTbi(oAMH OTBET)
1 short-acting insulin

metformin

3 gliclazide
4 glibenclamide
5 long acting insulin

1 32 33 34 35 36 37 38 39

40 50

1 2 3 4 5 6 7 10 89
11 12 13 14 15 16 17 18 19 20
21 22 23 24 25 26 27 28 29 30
31 32 33 34 35 36 37 38 39 40
|41 42 43 44 45 46 47 48 49 50
OTBeyeHHbiM BOnpoc
nponyujeHHbli Bonpoc
TeKyuni Bonpoc

Ao 3abepuweHMA TECTa oCTanocb

1:13:44
3aBepwTb TeCT

Scanned with CamScanner


l 10:08 U' LTE1 44)

A avn.kgma.kg/webtest/testing
LLWaxBes AiaM Kg Ru,

Bonpoc: N937
In hypercortisolism syndrome, skin changes are characterized by

OTBETbi(ogMH OTBET)
1
myxedema
2 vitiligo

3 striae

4 yellowness

5 darkening

33 34 35 36 37 38 39 40

41 50

1 2 3 4 5 6 7 8 9 10
11 12 13 14 15 16 17 18 19 20
21 22 23 24 25 26 27 28 29 30
31 32 33 34 35 36 37 38 39 40
41 42 43 44 45 46 47 48 49 50

OTBeueHHblM BONnpoc
nponyujeHHblM Bonpoc
TexyuuM Bonpoc

Ao 3aBepweHua TecTa oCTanocb

1:13:42
3aBepunTb TeCT

Scanned with CamScanner


11 10:08 "U LTE1 44)

A avn.kgma.kg/webtest/testing
LLWaxBes AnaM Kg Ru

Bonpoc: N938
Normally, the volume of the thyroid gland (according to ultrasound) in women is
(ml):

OTBETbi(ogMH OTBET)
1
from 25 to 30

2 from 35 to 38

3 from 40 to 45

4 Up to 18

5 more than 50

34 36 37 38 39 40 41
1 35

42 50

1 2 3 4 5 6 7 8 10 9
11 12 13 14 15 16 17 18 19 20
21 22 23 24 25 26 27 28 29 30
31 32 33 34 35 36 37 38 39 40
41 42 43 44 45 46 47 48 49 50
OTBeyeHHbiMBonpoc
nponyuyeHHblM BOnpoc
TeKyuwi Bonpoc

Ao 3aBepweHnA TeCTa oCTanocb:

1:13:39
3aBepuwTb TeCT

Scanned with CamScanner


11 10:08 U LTE1 44)

O Aavn.kgma.kg/webtest/testing

LLWaxBe3 AnaM Kg Ru

Bonpoc: N939
Cushing syndrome is characterized by

OTBeTbI(oqMH OTBeT)
1 hyperthyroidism

2 hyperprolactinemia
3 hyperparathyroidism

hypercortisolism

5 hyperpituitarism

1 35 36 37 38 39 40 41 42
43 50

1 2 3 4 5 6 7 8 9 10
11 12 13 14 15 16 17 18 19 20
21 22 23 24 25 26 27 28 29 30
31 32 33 34 35 36 37 38 39 40
41 42 43 44 45 46 47 48 49 50
OTBeYeHHblM Bonpoc
nponyueHHblM Bonpoc
TEKyuM Bonpoc

Ao 3aBepweHuA Tecta oCTanocb

1:13:37
SaBepunTb TeCT

Scanned with CamScanner


1 10:08 "U LTE1 44)

A avn.kgma.kg/webtest/testing

LLWaxBes AnaM Kg Ru

Bonpoc: Ne40
A 44-year-old woman was admitted to the ckinic for examination and doesn't
present any complaints. Anamnesis: dad suffers from diabetes. Objectively: a
patient with increased nutrition, normal color leather. Height-170 cm2, body
weight-85 kg. Breathing is vesicular. Heart sounds are saved. Laboratory:
fasting blood glucose (capillary blood) - 5.8 mmol/ after meals after 2 hours -
I

6.8 mmol/1, interpret this result.

OTBeTbI(OAuH OTBET)
1
impaired glucose tolerance

2 hypoglycemiaa
3 diabetes mellitus

4 acute hyperglycemia
5 impaired fasting glucose

1 36 37 38 39 40 41 42 43

44 * 50

1 2 3 4 5 6 7 8 10 9
11 12 13 14 15 16 17 18 19 20
21 22 23 24 25 26 27 28 29 30
31 32 33 34 35 36 37 38 39 40
41 42 43 44 45 46 47 48 49 50

OTBeueHHblM Bonpoc
nponyueHHbiM Bonpoc
Texyuu Bonpoc

Ao 3aBepweHnA TECTa oCTanocb

1:13:34
3aBepuwTb TeCT

Scanned with CamScanner


10:08 LTE C44

O A avn.kgma.kg/webtest/testing
WaxBes AnaM Kg Ru

Bonpoc: Ne41
A 43-year-old man, a truck driver, was admitted to the clinic with complaints of
an increase in blood pressure up to 190/100 mm Hg, accompanied by
headaches, dizziness, tinnitus. Parents have obesity, diabetes mellitus, arterial
hypertension. On examination: abdominal obesity, body weight -110 kg, height
160 cm, Broca's index -60 kg. Waist circumference- 130 cm, hip circumference-
110 cm. Palpation: the liver is painless, even, not enlarged. Auscultation: accent
l tone above the aorta, BP-180/95 mm Hg. Laboratory: fasting venous blood
glucose 4.5 mmol/ 2 hours after eating -7.1 mmol/ What is the most
1, 1.

typical syndrome of a patient that has developed as a result of obesity?

OTBeTbI(oqMH OTBET)
1
gynecomastia
2 hypercortisolism

3 hyperglycemia

4 hepatomegaly
5 hypertension

37 38 39 40 41 42 43 44

45 50

1 4 5 6 7
2 3 10 89
11 12 13 14 15 16 17 18 19 20
21 22 23 24 25 26 27 28 29 30
31 32 33 34 35 36 37 38 39 40
41 42 43 44 45 46 47 48 49 50
OTBeYeHHblM Bonpoc
nponyeHHblM Bonpoc
TexynM BonpocC

o 3aBepweHna TecTa oCTanocb

1:13:32
Scanned with CamScanner
1 10:08 "U LTE1 44)

A avn.kgma.kg/webtest/testing

LlaxBe3 AnaM Kg Ru

Bonpoc: Ne42
Patient K., 48 years old, went to the clinic with complaints of severe weakness,
weight loss, bone pain, especially in the feet, long-term healing fractures, loss of
appetite, nausea, vomiting not associated with food intake, diarrhea. From the
anamnesis: suffers from adenoma of the right parathyroid gland. Laboratory
data: TBC-anemia, Calcium-3.4 mmol / I, phosphates-0.7 mmol/ I. What is your
preliminary syndrome?

OTBeTbl(oAMH OTBET)

1 hyperthyroidism syndrome

2 hypercortisol syndrome

hyperparathyroidism syndrome

4 thyroid inflammation syndrome

5 hyperglycemia syndrome

1 38 39 40 41 42 43 44 45

46 * 50

1 2 3 4 5 6 7 89 10
11 12 13 14 15 16 17 18 19 20
21 22 23 24 25 26 27 28 29 30
31 32 33 34 35 36 37 38 39 40
41 42 43 44 45 46 47 48 49 50

OTBeYeHHbiM Bonpoc
nponyujeHHbiM BOnpoc
Texyuun Bonpobc

Ao saBepweHnA TECTa oCTanoCb:

1:13:30
3aBepunTb TeCT

Scanned with CamScanner


10:08

A avn.kgma.kg/webtest/testing
LLWaxee3 AnaM Kg Ru

Bonpoc: N43
Patient A. 34-year-old, applied to a polyclinic with complaints of an increase in
body temperature up to 39C, pain in the right side of the thyroid gland,
aggravated by swallowing, coughing, chills, tachycardia, weakness. Anamnesis:
frequent tonsillitis. Objectively: On palpation, the thyroid gland is sharply painful,
dense, regional lymph nodes are enlarged. Laboratory: Total blood test:
leukocytosis with a shift to the left, increased ESR. What is your most likely
syndrome?

OTBETbi(oqUH OTBET)
1
syndrome of hypothyroidism

syndrome of acute inflammation of the thyroid gland

3 syndrome of hypofunction of the parathyroid glands

4 syndrome of chronic inflammation of the thyroid gland

5 thyroid nodule syndrome

39 40 41 42 43 44 45 46

47 50

2 3 4 5 6 7 8 9 10
11 12 13 14 15 16 17 18 19 20
21 22 23 24 25 26 27 28 29 30
31 32 33 34 35 36 37 38 39 40
4142 4344 45 46 47 48 49 50
OTBeyeHHbiM BONpoc

nponyueHHbIM BOnpoc
TeKyuMM Bonpoc

Ao 3aBepweHnA TEcTa oCTanocb

1:13:27
3aBepuTb TeCT

Scanned with CamScanner


10:08

A avn.kgma.kg/webtest/testing
LLWaxBes AnaM Kg Ru

Bonpoc: N944
A 38-year-old woman was admitted to the ckinic with complaints of decreased
appetite, constipation, hearing loss, difficulty in nasal breathing, weakness.
Anamnesis: Subtotal strumectomy was performed 3 years ago. Objectively: The
skin is dry, the height is 167 cm, the weight is 68 kg, there is dense swelling of
the legs. diagnosed with hypothyroidism syndrome. Explain why hearing is
reduced and nasal breathing is difficultdue to?

OTBeTbl(ogMH OTBET)

1 inflammation

2 muscle swelling
3 mucous edema
4 infection

5 swelling of the skin

1 40 41 42 43 44 45 46 47

48 * 50

1 2 3 4 5 6 7 8 9 10
11 12 13 14 15 16 17 18 19 20
21 22 23 24 25 26 27 28 29 30
31 32 33 34 35 36 37 38 39 40
4142 43 44 45 46 47 48 49 50

OTBeyeHHblM Bonpoc
nponyujeHHblM BONpoc
Texyuu Bonpoc

Ao 3aBepweHnA TECTa oCTanocb

1:13:15
3aBepunTb TeCT

Scanned with CamScanner


10:08

A avn.kgma.kg/webtest/testing

LlaxBes AnaM Kg a
Bonpoc: Ne45
Woman 53 years old turned to a therapist with complaints of thirst, frequent
urination, itching of the perineum. Anamnesis, disturb the above complaints for
1
month. Objectively: The skin is dry and clean. BMI = 33.2 kg/m2. BP 135/95
mm.Hg. Laboratory blood glucose-9.3 mmol/1, after meals 15.0 mmol/ 1.
Doctor diagnosed with type 2 diabetes mellitus. Explain why the patient
developed hyperglycemia syndromein this case?

OTBeTbl(oAMH OTBET)

1 hypodynamia

2 stress
obesity

4 heredity

5 underweight

1 41 42 43 44 45 46 47 48

49 50

1 2 3 4 5 6 7 8 9 10
11 12 13 14 15 16 17 18 19 20
21 22 23 24 25 26 27 28 29 30
31 32 33 34 35 36 37 38 39 40
41 42 43 44 45 46 47 48 49 50
OTBeueHHblM Bonpoc
nponyujeHHbiM BOnpoc
Texyuywn Bonpobc

Ao 3aBepweHnA TECTa oCTanocb

1:13:13
3aBepunTb TeCT

Scanned with CamScanner


10:08 U LTE 43)

O A avn.kgma.kg/webtest/testing

LWaxees AnaM Kg Ru

Bonpoc: Ne46
Patient 42 years old, after a traumatic injury to the neck, notes the periodic
G.,
appearance of paresthesias (pathological unpleasant sensations, for example,
"running creeps) with the subsequent development of seizures. Before the
development of seizures, he has nausea, vomiting. Convulsions are tonicin
nature, beginning in the upper limbs and face. Consciousness during this period
is preserved. Examination of the patient revealed that the content of calcium in
,
the blood is 0.6 mmol/ phosphorus-8.0 mmol /L.What is your preliminary
syndrome?

OTBETbi(oqMH OTBET)

1 hypothyroidism syndrome

2 hypopituitarism syndrome

3 hypocorticism syndrome

4 hypoglycemic syndrome

5 hypoparathyroidism syndrome

* 42 43 44 45 46 47 48 49

50

1 2 3 4 5 6 7 8 9 10
11 12 13 14 15 16 17 18 19 20
21 22 23 24 25 26 27 28 29 30
31 32 33 34 35 36 37 38 39 40
|41 42 43 44 45 46 47 48 49 50
OTBeyeHHbliM Bonpoc
nponyujeHHbIM BOnpoc
TeKyunM Bonpoc

Ao 3aBepuweHwa TeCTa oCTanoCb

1:13:11
3aBepuMTb TECT
Scanned with CamScanner
1 10:08 "U LTE 43)

A avn.kgma.kg/webtest/testing
LlaxBe3 AnaM Kg Ru,

Bonpoc: Ne47
A 19-year-old girl was admitted to the ckinic unconscious. According to her
mother, the girl has been suffering from diabetes for 4 years, at home after an
injection of insulin she suddenly lost consciousness. The ambulance doctor
revealed hypoglycemia of 2,3 mmol/ 1. First aid was given by intravenous
glucose drip, but the patient did not regain consciousness. Indicate which drug
should be administered in this situation?

OTBETbi(oqMH OTBeT)

1Oinulin
2 glucose
3 albumin

4 insulin

5 glucagon

1 43 44 45 46 47 48 49 50

1 2 3 4 5 6 7 8 9 10
11 12 13 14 15 16 17 18 19 20
21 22 23 24 25 26 27 28 29 30
31 32 33 34 35 36 37 38 39 40
41 42 43 44 45 46 47 48 49 50
OTBeueHHblM Bonpoc
nponyujeHHblM BOnpoc
Texyuui Bonpoc

Ao 3aBepweHnA TECTa oCTanocb

1:13:08
3aBepunTb TeCT

Scanned with CamScanner


U LTE1 43)
10:08

O A A avn.kgma.kg/webtest/testing

LlaxBe3 AnaM Kg Ru

Bonpoc: Ne48
A 16-year-old boy was admitted to the clinic in an unconscious state according
to the mother, previously complained of severe thirst, frequent urination and
general weakness. Objectively: the smell of acetone from the mouth, the skin is
dry. Rapid breathing. Heart sounds are muffled. The abdomen is tense. What is
the most informative laboratory examination method (blood) for making a
diagnosis ?

OTBETbi(oAMH OTBET)

1
cholestetol
2 glucose

3 calcium

4 cretinine
5 proteim

43 44 45 46 47 48 49 50

1 2 3 4 5 6 7 8 9 10
11 12 13 14 15 16 17 18 19 20
21 22 23 24 25 26 27 28 29 30
31 32 33 34 35 36 37 38 39 40
41 42 43 44 45 46 47 48 49 50

-OTBeueHHbiM Bonpoc
nponyuueHHblM Bonpoc
TexyujmM Bonpoc

40 3asepueHnA TECTa oCTanoCb:

1:13:07
3aBepuTb TECT

Scanned with CamScanner


11 10:08

A avn.kgma.kg/webtest/testing

LLWaxBes AnaM Kg Ru

Bonpoc: Ne49
A 55-yearold woman consulted a family doctor at the place of residence with
complaints of thirst, dry mouth, weight gain, pain in the lumbar region, sweating.
Objectively: height- 170 cm, body weight 120 kg, moon-shaped face, crimson-
red, excessive growth of facial hair, acne, excessive deposition of fatty tissue in
the shoulder, chest, abdomen, red-violet color, atrophy of the muscles of the
arms and legs. BP- 190/100 mm Hg. Blood glucose- 12 mmol/ 1. Choose, what
hormonal blood test is necessary to conduct any obesity disease?

OTBeTbI(oqMH OTBET)
1
insulin
2 vasopressin
3 prolactin
4 thyroxine
5 cortisol

T 43 44 45 46 47 48 49 50

2 3 4 5 6 7 8 9 10
11 12 13 14 15 16 17 18 19 20
21 22 23 24 25 26 27 28 29 30
31 32 33 34 35 36 37 38 39 40
|41 42 43 44 45 46 47 48 49 50
OTBeyeHHblM BOnpooc
nponyeHHbIM Bonpoc
TexyunM BOnpoc

Ao 3aBepweHnA TEcTa oCTanocb

1:13:04
3aBepuTb TeCT

Scanned with CamScanner


1 10:08

O Aavn.kgma.kg/webtest/testing

Llaxees AnaM Kg Ru

Bonpoc: N®50

Patient S., 39 years old, was admitted to the endocrinology department of a


clinical hospital with complaints of weakness, a headache in the occipital region
when the weather changes, an increase in blood pressure up to 190/100 mm
Hg. pain in the heart, in the lumbar spine during physical exertion, menstrual
irregularities. Objectively: "moon' face, cheek rubeosis, height- 165 cm, weight
112 kg, waist circumference 120 cm, thigh circumference 100 cm, numerous
-

small subcutaneous hemorrhages on the skin of the arms, neck, abdomen, legs,
abdominal skin streaks of purple-red stretch. What is the most informative
examination method for making a diagnosis?

OTBeTbl(ogMH OTBET)

Computed tomography

2 Electrocardiogram

3 Radiography
4 Ultrasound examination

5 Echocardiography

1 43 44 45 46 47 48 49 50

1 2 3 4 5 6 7 8 9 10
11 12 13 14 15 16 17 18 19 20
21 22 23 24 25 26 27 28 29 30
31 32 33 34 35 36 37 38 39 40
|41 42 43 44 45 46 47 48 49 50
OTBeueHHblM Bonpoc
nponyuyeHHblM BOnpoc
TeKyuwi Bonpoc

Ao 3aBepuweHMA TeCTa oCTanOCb:

1:13:02
Scanned with CamScanner
PeaynbTaT

64 banOB
OTBETM npaBnIbHO Ha 32 u3 50

BepHyTbCA B BblXoA
MYHbIM KaónHeT

Scanned with CamScanner


Exam test questions in the discipline of radiation diagnostics for 3rd
year students of the Faculty of Foreign Citizens by the module
"Cardiovascular System"

1. The lower arch of the right contour of the heart on the radiograph in a straight line projection

presented

A.the left atrium

B. right atrium

C.left ventricle

D.right ventricle

E.aorta

2. Cardiomegaly is defined when the cardiothoracic index is more than

A. 40%

B. 45%

C. 50%

D. 55%

E. 60%

3. What is the hemodynamic sign of mitral stenosis?

A. enlargement of the left atrium

B. increase in the right atrium

C. enlargement of the left ventricle

D. aortic enlargement

E. absence of pulmonary hypertension

4. Indicate the hemodynamic sign of aortic stenosis

A. enlargement of the left atrium


B. increase in the right atrium
C. enlargement of the left ventricle

D. enlargement of the right ventricle

E. pulmonary hypertension

5. Indicate the ultrasound sign of aortic stenosis

A. incomplete opening of the aortic valve cusps

B. midsystolic convergence of the aortic valve cusps

C. separation of the aortic valve cusps

D. Oscillation of mitral leaflets

E. oscillation of the aortic valve leaflets

6.Preliminary diagnosis: thromboembolism of the right femoral artery.

Specify the optimal method of radiation diagnostics

A. radiography

B. ventriculography

D. selective angiography

C. X-ray computed tomography

E. perfusion scintigraphy

7. Diagnosis: ischemic heart disease. Surgical treatment of coronary artery disease is


recommended.

Specify the optimal method of radiation diagnostics

A. radiography

B. coronary angiography

D. X-ray computed tomography

C. echocardiography

E. dopplerography
8.Preliminary diagnosis: pericardial effusion. Please indicate optimal

method of radiation diagnostics

A. radiography

B. ventriculography

C. selective angiography

D. echocardiography

E. perfusion scintigraphy

9. A 19-year-old patient complains of fainting, chest pain during exertion, shortness of


breath, weakness. The palpable impulse of the left ventricle is displaced to the left, the radial
pulse is reduced. A chest x-ray was taken (Fig. 1). What is the most likely condition?

Fig.1
A. aortic insufficiency
B. tricuspid insufficiency
C. mitral insufficiency
D. mirtal stenosis
E. aortic stenosis

10. A 31-year-old patient complains of an increase in body temperature of 39oС, severe


fatigue and weakness. History of antecedent mitral regurgitation. Auscultation revealed
abnormal heart murmurs. Echocardiography was performed (Fig. 2). Which condition is most
likely?
Fig. 2
A. endocardial vegetation
B. sagging of the leaflets into the left atrium
C. aortic valve insufficiency
D. valve prosthesis
E. valvular calcification

11. A 16-year-old girl fell during a cycling competition and developed pain in the left side of her
chest. History of leg lameness (lameness). To rule out a fractured rib, she was sent to the
trauma department. There is no evidence of rib fracture or lung pathology on a chest x-ray.
However, the radiologist drew attention to the unevenness and notch of the lower edges of the
posterior segments of the IV-VII ribs and the "number 3" sign. What visualization method
should be the most informative?

A. fluoroscopy
B. angiography
C. echocardiography
D. coronary angiography
E. aortography

12. A 53-year-old patient with a history of coronavirus pneumonia was admitted to the hospital.
However, his condition deteriorated sharply, dizziness and shortness of breath appeared, oxygen
saturation decreased. Pulmonary thrombosis suspected. What imaging techniques need to be
clarified?

A. CT angiography
B. computed tomography
C. echocardiography
D. magnetic resonance imaging
E. chest x-ray
Tests Pathophysiology
1. The substances of Depressor action include
A) adrenalin
B) angiotensin-II
C) aldosterone
D) Prostaglandins E and A

2. Substances of Depressor action include:


A) nitrogen oxide
B) angiotensin-II.
C) aldosterone
D) endothelin

3. Pressor Action possesses:


A) adrenaline
B) kallikrein
C) prostaglandin E.
D) nitrogen oxide

4. Pressor Action has


A) kallikrein
B) angiotensin-II
C) prostaglandin E
D) nitrogen oxide

5. Pressor Action has


A) caplicrein
B) endothelin- I
C) prostaglandin E
D) nitrogen oxide

6. What blood pressure values indicate the presence of hypertension


A) 170/110
B) 110/70
C) 120/80
D) 115/60

7. Secondary (symptomatic) arterial hypertension is one of the symptoms of


diseases A) the liver
B) Gastrointestinal tract
C) kidney
D) spleen

8. Secondary arterial hypertension is a symptom: A)


Chronic adrenal insufficiency.
B) Gastric ulcer disease.
C) Primary aldosteronism.
D) hypocorpism

9. Secondary arterial hypertension is a symptom:


A) Itsenko-Cushing diseases.
B) Gastric ulcer disease.
C) Chronic adrenal insufficiency.
D) hypocorticoidism

10. Secondary arterial hypertension is a symptom:


A. Chronic adrenal insufficiency;
B. Gastric ulcer disease;
B. primary aldosteronism;
D. Diseases of Itsenko-Cushing (hypercorticoidism);
E. Intestinal autointoxication
A) A, b ANS-B,D
b) B, C)
in, G
D) G, D

11. Among the symptomatic arterial hypertension, hypertension is most often found
in
A) pheochromocytoma
B) Kidney lesion
C) primary hyperaldosteronism
D) hyperthyroidism

12. Among the symptomatic arterial hypertension most commonly found


A) endocrine
B) renal
C) neurogenic centrogenic
D) neurogenic reflex

13. The critical factor in the pathogenesis of renovascular renal hypertension is the
(A) Activation of the renin-angiotensin-aldosterone system;
B) increased secretion of glucocorticoids;
C) decrease in the production of depressor utilize substances in the kidneys; D)
increase of adrenaline secretion;

14. The leading role in the pathogenesis of renal hypertension is played by:
A) decrease in the production of depressor utilize substances in the kidneys.
B) increased secretion of glucocorticoids.
(C) Activation of the renin-angiotensin-Aldosteron system. D)
increase of adrenaline secretion.

15. Renal Arterial hypertension is observed


(A) in chronic glomerulonephritis;
B) In the case of congenital absence of one kidney;
C) in case of narrowing of the renal artery;
D) with thrombosis of renal artery;

16. In the pathogenesis of Renal graft (Parenchymatous) hypertension is


important
(A) Activation of the renin-angiotensin-Aldosterone system
B) increased secretion of vasopressin
C) Increased adrenaline secretion
D) Violation of kidney secretion, prostaglandin in the kidneys

17. Arterial hypertension in the aortic coarctation refers


A) to cardio-vascular hypertension
B) neurogenic arterial hypertension
C) to endocrine arterial hypertension
D) to medicinal arterial hypertension

18. The pathogenesis of primary arterial hypertension involves:


A) persistent increased excitability and hypotension of higher sympathetic nerve
centers.
B) Prolonged braking of the emotional centers.
C) Increasing the braking influence of the cerebral cortex on the pressure centers.
D) Increase of sodium-urethric hormone production.
19. The pathogenesis of primary arterial hypertension involves:
A) persistent increased excitability and hypotension of higher sympathetic
Nerve centers;
b) long-term re-excitation of emotional centers;
b) Reduction of the braking influence of the cerebral cortex on the pressor
Centres
d) Hereditary deficiency of membrane ionic pumps of smooth-muscle vascular
cells; E. Insufficiency of the adrenal cortex function
A) A B) A, b
c) A, D
D) A, B, C,D

20. Genetic defect of cell membranes in hypertensive disease results in


A) to increase the calcium content in the cytoplasm of cells;
B) to increase the electrical potential of cell membranes;
C) to increase the speed of the mediators &#39; reuptake by nerve endings; D) to
suppress ATP-ase activity of Myosin;

21. Orthostatic collapse occurs when


A) massive blood loss
B) Crushing the pancreas
C) rapid reduction of oxygen in the inhaled air
D) abrupt transition from horizontal position to vertical

22. Symptomatic arterial hypotension is observed in the case of


A) anemia
B) pheochromocytome
C) hyper aldosteronism
D) acute diffuse glomerulonephritis

23. Symptomatic arterial hypotension is observed at:


A) cholemia
B) pheochromocytoma
C) hyperaldosteronism
D) acute diffuse glomerulonephritis

24. The cause of sinus tachycardia is


enhancing
A) strengthening sympathetic influences on the heart
B) increased parasympathetic influences on the heart
C) weakening of sympathetic influences on the heart
D) lowering the body temperature

25. Sinus Bradycardia develops at


A) Increase in body temperature
B) strengthening of parasympathetic influences on the heart
C) improving the tone of the sympathetic nervous system
D) hypoxia

26. In the pathogenesis of sinus Bradycardia, it is important to


A) slowing down the spontaneous depolarization of the cardiomyocytes membrane
B) acceleration of spontaneous depolarization of the membrane of cardiomyocytes
C) Occurrence of damage currents in the myocardium
D) Lengthening of the period of cardiac platelet refractive

27. In the pathogenesis of sinus (respiratory) arrhythmia, it is important to


A) Formation of an ectopic impulse
B) The fluctuation of the Vagus tone
C) disturbance of excitation from atrial fibrillation to ventricles
D) mechanism &quot;re-entry&quot;

28. The transverse blockade of the heart is a


A) Violation of the excitation on the right leg of the Gissa beam
B) Disturbance of the excitation on the left leg of the Gissa beam
C) disruption of the pulse from the atrial to the ventricles through the
Atrioventricular knot
(D) Violation of the initiation of pericardium

29. Ventricular fibrillation is a


A) chaotic reduction of certain groups of cardiomyocytes
B) Violation of the excitation on the right foot of the beam of Gissa
C) Violation of the excitation on the left leg of the Gissa beam
(D) Violation of the initiation of pre-cordium

30. Severe violations of hemodynamics develop in the


A) sinus tachycardia
B) sinus arrhythmia
C) extrasystoles
D) Ventricular fibrillation
Practice tests for CVS in children:

1. What is heart disease has blood pressure on the upper extremities above than on
the lower extremities?
A. mitral stenosis
B. aortic coarctation
C. myocarditis
D. with pulmonary artery stenosis
E. tetrad Fallot

2. The QRS complex on the ECG reflects:


A. propagation of arousal in the Atria
B. propagation of arousal in the ventricles
C. repolarization
B. phase of cessation of arousal in the Atria
E. phase of cessation of arousal in the ventricles

3. What is week finished formation of the fetal heart?


A. 1-2 weeks
B. 3-4 weeks
C. 5-6 weeks
D. 7-8 weeks
E. 9-10 weeks

4. When lightly pressing on the tip of the nail and holding in this position, the
white spot in its center (respectively systole and diastole) then turns red, and then
turns pale. Which is pathology of the heart is this?
A. mitral valve stenosis
B. aortic valve insufficiency
C. aortic coarctation
D. tricuspid valve stenosis
E. anemia

5. Expansion of the right border of the heart is observed in:


A. left ventricular hypertrophy
B. right ventricular hypertrophy
C. left atrial hypertrophy
D. right atrial hypertrophy
E. stenosis of the mouth of the pulmonary artery
6. Increased 1-tone in the aortic artery is noted in stenosis:
A. mitral opening
B. aortic mouth
C. mouth of the pulmonary artery
B. inferior Vena cava
E. tricuspid valve

7. When will appears diastolic murmur is heard?


A. open ductus arteriosus
B. mitral stenosis
C. ventricular septal defect
D. stenosis of the mouth of the pulmonary artery
E. atrial septal defect

8. Determine CHD with impoverishment of the large circulatory circle:


A. open aortic ductus
B. stenosis of the mouth of the pulmonary artery
C. mitral insufficiency
D. aortic stenosis
E. tetrad of Fallot

9. What is main cause of rare pulse in children:


A. paralysis of the diaphragm
B. insufficient of blood supply to the heart
C. predominance of the parasympathetic nervous system
D. inflammation of pericardium
E. inflammation of myocardium

10. Specify the main risk factor for the development of heart disease in children:
A. the presence of mother’s infection
B. functional disorders of the cardiovascular system
C. hereditary predisposition to rheumatic diseases
D. poor nutrition, poor housing and seasonal clothing
E. bad habits of parents, their education
11. What is heart disease low diastolic blood pressure on the lower part of body, up
to zero, and cyanotic of legs observed:
A. ventricular septal defect
B. atrial septal defect
C. mitral stenosis
D. open ductus arteriosus
E. stenosis of the mouth of the pulmonary artery
Training exam tests
for 3rd year students of the faculty of "General Medicine" on the subject of
Propedeutics of Internal Diseases of the Cardiovascular System

For memorization:

1. What is the most reliable diagnostic method to confirm syndrome of coronary insufficiency is?
A. Resting ECG
B. Treadmill test
C. Echocardiography
D. Bronchodilation test
E. Holter ECG test

2. What symptom is typical for patients with endocardial inflammation syndrome?


A. Lukin-Liebman
B. Machiavelli
C. Oppenheimer
D. Ortner-Grekov
E. Mussey-Georgievsky

3. Which of the following is the center pulse?


A. Popliteal pulse
B. Radial pulse
C. Femoral pulse
D. Pulse Dorsalis pedis
E. Cibialis posterior pulse

4. Which of the following is the peripheral pulse?


A. Femoral pulse
B. Carotid pulse
C. Aortic pulse
D. Popliteal pulse
E. Brachial pulse

5. What is characteristic of edema in patients with heart failure syndrome?


A. Usually appears on the right leg
B. Typically appears on the left leg
C. Reduced size of the abdomen
D. On both lower extremities
E. Decreased thickness of legs

6. What defines Holter monitoring?


A. Blood pressure variability
B. Nocturnal blood pressure
C. Oxygen saturation
D. Rhythm disturbances
E. Blood gases` variability

7. Which of the following numbers is optimal blood pressure level?


A. 119/80 mm Hg
B. 135/90 mm Hg
C. 140/92 mm Hg
D. 160/70 mm Hg
E. 180/100 mm Hg

8. Why chronic coronary insufficiency is dangerous in the long term?


A. It may cause pulmonary hypertension
B. It may cause thromboembolism of cerebral vessels
C. It may cause fatal arrhythmias
D. It may cause myocardial infarction
E. It may cause venous insufficiency
9. Target organs of arterial hypertension?
A. Eyes, brain, lungs, spleen
B. Liver, lungs, heart, aorta
C. Kidney, heart, liver, eyes
D. Stomach, kidneys, heart, brain
E. Heart, kidneys, brain, eyes and aorta

10. What is the auscultation point of aortic valve?


A. 2nd intercostal space, midclavicular line
B. 2nd intercostal space, right border of sternum
C. 2nd intercostal space, left border of sternum
D. 4th intercostal space, midclavicular line E. 5th intercostal space, midclavicular line

11. Normal mitral valve area?


A. 0.5-1 cm2
B. 1-2 cm2
C. 2-4 cm2
D. 4-6 cm2
E. 6-9 cm2

12. Normal aortic valve area?


A. 0.5-1 cm2
B. 1-2 cm2
C. 2-4 cm2
D. 4-6 cm2
E. 6-9 cm2

13. What is dyspnea?


A. fatigue
B. shortness of breath
C. dizziness
D. sensation of heartbeat
E. fainting

14. Where is the auscultation point of pulmonary valve?


A. 2nd intercostal space, midclavicular line
B. 2nd intercostal space, right border of sternum
C. 2nd intercostal space, left border of sternum
D. 4th intercostal space, midclavicular line
E. 5th intercostal space, midclavicular line

15. Which of the following is ECG criteria of ischemia of inferior wall of left ventricle?
A. ST segment depression in I, aVL leads
B. ST segment depression in II, III, aVF leads
C. ST segment depression in V1-V2 leads
D. ST segment depression in V3-V4 leads
E. ST segment depression in V5-V6 leads

16. The pressure that the heart must work against to eject blood during systole is called? A.
A.Afterload
B. Preload
C. Pulse
D. Ejection fraction
E. Cardiac output

17. This condition can be recognized as physiological, particularly in athletes, sinus node
dysfunction; it can be a consequence of using of drugs (beta blockers, calcium channel blockers);
at ECG: rate < 60 bpm, normal P wave before every QRS complex. Select correct answer:
A. Right bundle branch block
B. Left bundle branch block
C. AV block, Mobitz type I
D. AV block, Mobitz type II
E. Sinus bradycardia

18. Which of the following is the gold standart in diagnostic of Infective myocarditis?
A. Coronary angiography
B. Endomyocardial byopsy
C. Echocardiography
D. MRI
E. ECG

For understanding

19. Choose the most accurate definition: what is the advantage of the diagnostic method -
ambulatory blood pressure monitoring?
A. Variability of blood pressure
B. Blood pressure in outpatients
C. Blood pressure in inpatients
D. Serious rhythm disturbances
E. Conduction disturbances

20. A patient with tricupid valve infective endocarditis was admitted to inpatient hospital. What
is the most likely complication for the patient?
A. Ischemic stroke
B. Septic kidney emboli
C. Septic emboli of the spleen
D. Pulmonary embolism
E. Embolism of mesenteric artery

21. A patient with mitral valve`s infective endocarditis has been admitted to inpatient hospital.
What is the most likely complication for the patient?
A. Distal pulmonary arterioles` emboli
B. Septic emboli of the spleen
C. Bilatery emboli of pulmonary artery
D. Right pulmonary embolism
E. Left pulmonary embolism

22. What is the main mechanism by which edema can develop in patients with chronic heart
failure syndrome and decreased cardiac output? Choose the correct answer:
A. Increase in extracellular fluid volume
B. Elevated serum albumin levels
C. Increased serum triglyceride levels
D. Reducing serum cholesterol
E. Increased potassium retention

23. According to NYHA classification of heart failure, which of the following is characterized by
appearance of symptoms at rest?
A. Class I
B. Class II with preserved EF
C. Class II with reduced EF
D. Class III
E. Class IV

24. A 63-year-old man over the past 5 months has been complaining on squeezing pain in the
area behind the sternum that occurs during walking of 500 m, or when climbing the stairs to the
2nd floor. Pain reliefs at rest within 5 minutes. Risk factors: obesity, arterial hypertension. What
is your diagnosis? A. Unstable angina
B. Progressive angina
C. Myocardial infarction
D. Vasospastic angina
E. Stable angina
25. 65 years old male, complaints of headaches, dizziness. Risk factors: heavy smoker, diabetes,
positive family history. Examination: blood pressure 140/90 mm Hg. What degree of arterial
hypertension would you put?
A. I degree arterial hypertension
B. II degree arterial hypertension
C. III degree arterial hypertension
D. Isolated systolic hypertension
E. Prehypertension

26. 42 years old male admitted to the hospital. After hypothermia, the body temperature
increased to 38.7 ° C, accompanied by chills and sweat. Body temperature returned to normal for
a short time, then increased again to high numbers. When examining the heart, the picture of
combined mitral defect. Respiratory rate (RR) - 22 per minute. Heart rate (HR) 90 per minute,
blood pressure (BP) 110/70 mm Hg. Which laboratory test is needed here?
A. Urinalysis
B. Blood culture
C. Troponin
D. CRB
E. Serum iron

27. Patient S, 58 years old, with complaints like dyspnea at the rest, orthopnea and paroxysmal
nocturnal dyspnea, palpitations, chest pain admitted to hospital. The symptoms are worsening over
the years. In anamnesis he had an Coronary heart disease. Echocardiography showed ejection
fraction 20%. What is your diagnosis?
A. Acute Heart failure
B. Chronic Heart failure Class I
C. Chronic Heart failure Class II
D. Chronic Heart failure Class III
E. Chronic Heart failure Class IV

28. A 39 year old male admitted to the hospital with complaints of fever for 2 weeks up to 39
degrees, severe weakness. In anamnesis congenital heart disease since childhood – bicuspid aortic
valve. RR – 22 per minute. Auscultation – diastolic murmur in the aorta region. BP – 120/40 mm
Hg. CBC: increased ESR. Echocardiography: mobile vegetation of large sizes on the aortic valve.
Blood culture results: Staphylococcus aureus. What is your preliminary diagnosis?
A. Syndrome of inflammation endocardium
B. Syndrome of inflammation pericardium
C. Syndrome of inflammation myocardium
D. Syndrome of mitral valvular diseases
E. Syndrome of acute coronary insufficiency
29. 35 years old male admitted the physician with complaints for the chest pain, orthopnea, fatigue.
In anamnesis - rheumatic fever. Doctor performed the auscultation and discovered loud S1 and
opening snap after S2. What is your preliminary diagnosis?
A. Tricuspid stenosis
B. Aortic stenosis
C. Aortic regurgitation
D. Mitral stenosis
E. Mitral regurgitation

30. Patient K. 40 years old is describing his complaints as: “my heart flip-flops, skips beats,
especially if I carry stuff up his stairs or when I’m bending down.”. ECG showed no visible P
waves and an irregular narrow QRS complexes. What do you suspect?
A. atrial flutter
B. atrial fibrillation
C. ventricular fibrillation
D. ventricular tachycardia
E. supraventricular tachycardia

For applying

31. 58 years old male patient applied to outpatient clinic with complaints for squeezing
retrosternal chest pain occurring at doing physical work; pain releases at the rest position in
a few minutes. Patient is a smoker; the patient's left leg was amputated last year due to car
accident. Which diagnostic method would you recommend to confirm syndrome of coronary
insufficiency?
A. Resting ECG
B. Treadmill test
C. Echocardiography
D. Bicycle stress test
E. Holter ECG test

32. What method of investigation would you recommend using in the diagnosis of patients with
suspected white coat hypertension?
A. Resting ECG
B. Treadmill test
C. 24h Holter monitoring
D. Bicycle stress test
E. 24h BP-monitoring
33. 46-year old female admitted to the hospital with dyspnea and hemoptysis. Patient history:
rheumatic fever. Physical examination: orthopnea, facies mitralis, loud S1, opening snap of
mitral valve have been detected. What diagnostic method would you administer to confirm your
diagnosis?
A. MRI
B. Chest x-ray
C. Angiography
D. Echocardiography
E. ECG

34. 57 years old female complains on retrosternal chest pain radiating to the left shoulder, arises
with an increase of blood pressure. Pain worries her for 2 months. During last week, pain
episodes occurred more often, mainly in the morning and were accompanied by shortness of
breath. She is a heavy smoker. Suffers from hypertension. The father and older brother of the
patient suffered from myocardial infarction. Examination: overweight. Pulse 88 in 1 min,
rhythmic. Echocardiography: global and local myocardial contractility is preserved. BP 145/100
mm Hg. Which examination is necessary for differential diagnosis?
A. ECG
B. Kidney ultrasound
C. Chest X-ray
D. CT scan
E. 24-hour ECG monitor

35. 46-year-old male was admitted to the hospital with pericardial inflammation syndrome. He
associates his illness with hypothermia. Examination: fever, chills, sweating, pain in the heart
associated with the breathing and changes of body position, pericardial friction rub, which
increases during inspiration. What examinations need to be administered? A. Analysis of
urine
B. BNP
C. Blood glucose
D. AST/ALT
E. Cardiac enzymes

36. 49 years old male complains on retrosternal chest pain radiating to the left back, neck, jaw
arises after physical exertion. Pain worries him for 2 months. During last week, pain episodes
occurred more often, mainly in the morning and were accompanied by shortness of breath. He is
a heavy smoker. Examination: Pulse 78 in 1 min. BP 145/100 mm Hg. On ECG at the rest no
changes. Which examination is necessary to determine diagnosis?
A. Chest x ray
B. MRI
C. Echo
D. CT scan
E. Stress ECG
37. Patient without signs heart failure has complaints for angina pectoris, dizziness. At ECG:
signs of left ventricular hypertrophy high R peak in left chest leads (I, aVL and V₄₋₆), deep S
peak in right chest leads (III, aVR, V₁₋₃), widened QRS complex (> 100 ms). Heart rate - 84 per
minute. Which diagnostic method should be prescribed at this case to confirm your preliminary
diagnosis?
A. MRI
B. Spirometry
C. Chest x-ray
D. Daily ECG monitoring
E. Echocardiography

38. A 5-year-old man admitted to the hospital through the ambulance with complaints of
retrosternal chest pain which lasts more than 1 hour. Patient is diaphoretic. Examination: Pale skin.
BP – 120/70 mmHg, HR – 78 bpm. ECG showed ST segment elevation in II, III, avF. Which
laboratory test may help you to confirm diagnosis?
A. ALT, AST
B. Troponin I/T
C. ESR
D. CBC
E. Creatinine CRP
(pedia previous year)
Сardiovascular system 190.
Pulse rate in newborns (per 1 min):
A. 115-120
B. 24-26
C. 140-160
D. 85-90
E. 68-72

191. Pulse rate in at 10-12 months (per 1 min):


A. 140-160
B. 24-26 C. 85-90
D. 68-72
E. 115-120

192. Pulse rate in child at 5 year of age (per 1 min):


A. 115-120
B. 140-160
C. 100
D. 85-90
E. 68-72

193. Pulse rate in child at 8-10 years (1min):


A. 140-160 B. 115-120
C. 130-135
D. 85-90
E. 100

194.Embriogenesis of CVS is started (weak):


A.1
B.3-4
C.2-3
D.10-12
E.6-7 195.Formula for calculating systolic pressure in babies before 12
month of life (mmHg):
A. 90+2n
B. 76+2n
C. 60+n
D. 75+n
E. 75+2n
196. Formula for calculating middle systolic pressure in children after 1 year of
age (mm Hg):
A. 76+n
B. 75+2n
C. 90+2n
D. 60+n
E. 105+2n

197. Formula for calculating middle diastolic pressure in children after 1 year
of age (mm Hg):
A. 76+n
B. 75+2n
C. 90+2n
D. 60+n
E. 105+2n

198. Mass of the heart in newborns brom mass of body (%):


A. 0,8 B. 0,5 C. 0,4 D. 0,9
E. 0,2
199. By palpation of the heart define:
A. borders
B. diameter
C. apical impulse
D. murmurs
E. tones

200.By percussion of the heart define:


A. borders
B. diameter
C. apical impulse
D. tones
E. murmurs

210. In intrauterine period first organ, which gets oxygen:


A. extremities
B. lungs
C. liver
D. spleen
E. kidney

211. Position of the heart in babies before 1-2 years of age:


A. vertical
B. gorizontal
C. obligue
D. right
E. high

212. Myocardial blood supply in children:


A. bad
B. middle
C. abundant
D. good
E. moderate

213. Diameter of pulmonary artery versus aorta in newborns:


A. more
B. same
C. less
D. better
E. much less

214. Thickness of right and left ventricles in newborns same and consist (mm):
A. 9-10
B. 5-7
C. 2-3
D. 11-12
E. 1-2

215.Right border of the heart formed by:


A. right atrium
B. left atrium
C. right ventricle
D. left ventricle
E. aorta
216. Left border of the heart formed by:
A. right atrium
B. left atrium
C. right ventricle
D. left ventricle
E. aorta

217. Upper border of the heart formed by:


A. right atrium
B. left atrium
C. right ventricle
D. left ventricle
E. aorta

218. Left border of the heart is in babies before 2 years of age (intercostal space):
A. 2 B. 3 C. 4 D. 5
E. 1

219. Right border of the heart in babies before 2 years of age (line):
A. right middle clavicular
B. right parasternalis
C. left parasternalis
D. left middle clavicular
E. anterior axillaris

220. Left border of the heart in child at 7 years of age is (on):


A. left middle clavicular line B. right middle clavicular line
C. left parasternalis line
D. right parasternalis line
E. left anterior axillaris line

221. Mitral valve function is heard:


A. in the 2-nd itercostal space to the right of the sternum
B. in the 2-nd itercostal space to the left of the sternum
C. on apex of the heart
D. in the 3-4 itercostal space to the right of the sternum
E. based on the xiphoid process
222.”Heart hump” develops in:
A. ventricular septal defect
B. patent ductus arteriosus
C. aortic stenosis
D. coarctation of aorta
E. atrial septal defect

223. “Cardiac asthma” in sick children is noted in insufficiency of:


A. right atrium
B. right ventricle
C. left atrium
D. left ventricle
E. aorta

224. Left ventricular failure is seen in:


A. critical pulmonary stenosis
B. tricuspid valve disease
C. coarctation of aorta
D. pulmonary hypertension
E. large atrial septal defects

225. Main symptom in Tetralogy of Fallot is:


A. nosebleeds
B. cyanosis of skin
C. permanent moist rough
D. delay of physical development
E. polyuria

226. In coarctation of aorta systolic murmur is head:


A. on apex of the heart
B. in the 2-nd intercortal space to the left of the sternum
C. in the 2-nd intercostal space to the right of the sternum
D. based on the xiphoid process
E. in 4-th intercostal space to the left of the sternum

227. The intensification of both tones is noted at:


A. miocarditis
B. mitral valve insufficiency
C. aortic valve disease
D. physical excitement
E. hypothyroidism

228. In coarctation of aorta in elder children main symptom:


A. pallor of the skin
B. “Heart hump”
C. acrocyanosis
D. disproportion of development of upper and lower parts of body
E. narrow chest

229.”Catʹs purr” define by: A.


inspection
B. palpation
C. percussion
D. auscultation
E. ultrasonography

230. For organic murmur characteristic:


A. soft timbre
B. inconstancy
C. harsh timbre
D. short
E. only systolic

231.”Cappillary pulse” is observed in:


A. mitral stenosis
B. mitral valve insufficiency
C. collapse
D. aortic valve insufficiency
E. tricuspid valve insufficiency

232. Apical impulse in children not palpable in:


A. dry pericarditis
B. exudative pericarditis
C. hyperthyroidism
D. fever
E. mental urge

233. Symptom of positive venous pulse manifested by:


A. pulse deficit
B. loss of pulse
C. tremor of the chest
D. coincidence of pulsation of the veins with the pulse on the common
carotid artery
234. In babies 0-2 years of age apical impuls is:
A. 1,0 cm outside from left middle clavicular line, in 5-th intercostal space
B. 0,5-1,0 cm inside from middle clavicular line, in 3-d intercostal space
C. 3,0 cm outside from parasternalis line
D. 1,5-2,0 cm outside from middle clavicular line, in 4-th intercostal space
E. 3-d rib
235. In babies 2-7 years of age apical impuls is:
A. 1,5-2,0 cm outside from middle clavicular line, in 4-th intercostals space
B. 2-nd rib
C. 1,0 cm outside from middle clavicular line, in 5-th intercostals space
D. 3,0 cm outside from parasternalis line
E. 3-d rib

236. In babies 0-2 years of age upper border is on:


A. 4-th rib
B. 2 –nd rib
C. 2 – nd intercostals space
D. 5-th rib
E. 1 –st rib
237. In babies 2-7 years upper border is on is:
A. 4-th rib
B. 3 –d rib
C. 2 – nd intercostals space
D. 2-nd rib
E. 5 –th rib
238. Pulse rate in babies at 10 month (1 min) is:
A. 90-95
B. 140-160
C. 125
D. 115-120
E. 100
239. Pulse rate in child at 5-6 years (1 min) is:
A. 90-95
B. 140-160
C. 125
D. 115-120
E. 100
240. Right border of the heart in child 7-12 year is:
A. right parasternalis line
B. a little inside from parasternalis line
C. a little ant side from parasternalis line
D. on middle between parasternalis line and right border of sternum
E. right border of sternum
241. Formula systolic arterial pressure in children after 1 year – the upper age:
A. 90+2n
B. 75+2n
C. 105+2n
D. 88+2n
E. 60+n

242. Formula diastolic arterial pressure in children after 1 year – the middle age:
A. 60+n
B. 75+n
C. 45+n
D. 105+n
E. 50+n
243. Formula systolic arterial pressure after 1 year – the middle age:
F. 90+2n
G. 100+2n
H. 60+2n
I. 80+2n
J. 88+n
244.Right border of the heart is formed by:
A. Left ventricle
B. right ventricle
C. right atrium
D. left atrium
E. aorta

245.Left border of the heart is formed by:


A. left ventricle
B. right ventricle
C. right atrium
D. left atrium
E. aorta
246 .Upper border of the heart is formed by:
A. left ventricle
B. right ventricle
C. right atrium
D. left atrium
E. aorta
247. In auscultation of the heart there are points (quantity):
A. 1 B. 2 C. 3
D. 4
E. 5
248. At 1 – st point of auscultation of the heart there is head working of (valve):
A. mitral
B. tricuspid
C. aortic
D. pulmonary artery
E. vesseled

249. At 2 – nd point of auscultation of the heart there is heard working of (valve):


A. mitral
B. tricuspid
C. aortic
D. pulmonary artery
E. vesseled

250. At 3 – st point of auscultation of the heart is heard working of (valve):


A. mitral
B. tricuspid
C. aortic
D. pulmonary artery
E. vesseled
251. At 4 – st point of auscultation of the heart is heard working of (valve):
A. mitral
B. tricuspid
C. aortic
D. pulmonary artery
E. vesseled
252.Anatomical closing of the foramen ovale is noted at (age):
A. 2-3 month
B. 1-11/2 year
C. 5-6 month
D. 2-3 year
E. 5-6 year
253. Anatomical closing of the ductus arteriosus is noted at (age): A.
2-3 month
B. 1-11/2 year
C. 5-6 month
D. 2-3 year
E. 5-6 year
254 .Pulse rate in child at 2-3 month is (1 min):
A. 140-160
B. 130-135
C. 100
D. 85-90
E. 72-74
255. Pulse rate in child at 11-12 month is (1 min):
A. 140-160
B. 130-135
C. 100
D. 115-120
E. 125-130

256.Pulse rate in child at 3-6 month is (1 min):


A. 140-160
B. 130-135
C. 100
D. 115-120
E. 125-130
257.Position of the apical impuls in healthy babies 0-2 year of age is (intercostals
space):
A. 2
B. 3
C. 4
D. 5
E. 7
258.Position of the apical impuls in healthy children after 2 years of age is
(intercostals space):
A. 2
B. 3
C. 4
D. 5
E. 7
259.Thickening of left and right ventricles consist (mm):
A. 1-2
B. 3-4
C. 4-5
D. 5-7
TEXAS UNIVERSITY CVS SAMPLE QUES
1. Blockage of which of the following arteries would lead to ischemia of the apex
of the heart?

Anterior interventricular (descending)

Left circumflex

Posterior interventricular (descending)

Right marginal

Right coronary
2. If the ductus arteriosus does not spontaneously close off soon after birth (to
become the ligamentum arteriosum), it may have to be surgically ligated.
When clamping or ligating it, what important structure immediately behind it
must be identified and saved? arch of the azygos vein internal thoracic
artery left phrenic nerve left recurrent laryngeal nerve

left superior intercostal vein


3. A hand slipped behind the heart at its apex can be extended
upwards until stopped by a line of pericardial reflection that forms the:

Cardiac notch

Costomediastinal recess

Hilar reflection

Oblique pericardial sinus

Transverse pericardial sinus


4. A stethoscope placed over the left second intercostal space just lateral to the
sternum would be best positioned to detect sounds associated with which
heart valve?

aortic

pulmonary

mitral

tricuspid

5. Which valves would be open during ventricular systole?

Aortic and pulmonary


Aortic and tricuspid

Mitral and aortic

Tricuspid and mitral

Tricuspid and pulmonary


6. Which chamber's anterior wall forms most of the sternocostal
surface of the heart?

Left atrium

Left ventricle

Right atrium

Right ventricle
7. A 3rd-year medical student was doing her first physical exam. In order to
properly place her stethoscope to listen to heart sounds, she palpated bony
landmarks. She began at the jugular notch, then slid her fingers down to the
sternal angle. At which rib (costal cartilage) level were her fingers?

4
8. A patient involved in an automobile accident presents with a sharp object
puncture of the middle of the sternum at about the level of the 4th or 5th
costal cartilage. If the object also penetrated pericardium and heart wall,
which heart chamber would most likely be damaged?

Left atrium

Left ventricle

Right atrium

Right ventricle
9. Which statement is true of the right atrioventricular valve?

it is also called the mitral valve

it is open during ventricular diastole

it transmits oxygenated blood

it is opened by the pull of chordae tendineae

it consists of 2 leaflets
10. A 23-year-old male injured in an industrial explosion was found to have
multiple small metal fragments in his thoracic cavity. Since the pericardium
was torn inferiorly, the surgeon began to explore for fragments in the
pericardial sac. Slipping her hand under the heart apex, she slid her fingers
upward and to the right within the sac until they were stopped by the cul-
desac formed by the pericardial reflection near the base of the heart. Her
fingertips were then in the:

Coronary sinus

Coronary sulcus

Costomediastinal recess

Oblique sinus

Transverse sinus
11. An elderly lady suffers a coronary occlusion and subsequently it is noted that
there is a complete heart block (that is, the right and left bundles of the
conduction system have been damaged). The artery most likely involved is
the:

acute marginal branch

anterior interventricular (Left anterior


descending)
circumflex branch

obtuse marginal

posterior interventricular (posterior descending)


12. During fetal life and sometimes persisting into the adult there is an opening
between the right and left atria; this opening is called the:

atrioventricular canal

coronary sinus

foramen ovale

sinus venosus

truncus arteriosis
13. The heart sound associated with the mitral valve is best heard:
In the jugular notch

In the second left intercostal space

In the second right intercostal space

In the fifth left intercostal space

To the right of the xiphoid process

14. Which heart valve has leaflets described as "anterior, left and right"?
Aortic

Pulmonary

Left atrioventricular

Right atrioventricular
15. In preparation for thoracic surgery, a median sternal splitting procedure was
carried out. But an improper depth setting on the saw blade resulted in a
slight nick on the underlying sternocostal surface of the heart. Which heart
chamber would most likely have been opened had the blade completely
penetrated this wall?

Left atrium

Left ventricle

Right atrium

Right ventricle
16. The sound associated with tricuspid stenosis (narrowing) in a 40-year-old
male would be best heard at which location on the anterior chest wall?

Below the left nipple

In the right 2nd intercostal space near the sternum

Over the apex of the heart

Over the sternal angle

Xiphoid area, just off the sternum


17. Blockage of blood flow in the proximal part of the anterior interventricular
artery could deprive a large area of heart tissue of blood supply, unless a
substantial retrograde flow into this artery develops via an important
anastomosis with which other artery?

Circumflex

Left marginal
Posterior interventricular

Right coronary

Right marginal
18. Traumatic, acceleration/deceleration injuries to the aorta usually
occur where its mobile and fixed portions meet. This would be at the:

at the ligamentum arteriosum

junction of aortic arch with the descending portion


junction of the ascending aorta with the heart

origin of the brachiocephalic artery on the arch

point where the descending aorta passes through the diaphragm


19. Which structure does NOT lie in the coronary sulcus?

circumflex artery
coronary sinus
right coronary
artery right
marginal artery

left coronary artery


20. Which structure contains postganglionic sympathetic fibers?

greater thoracic splanchnic nerve


recurrent laryngeal nerve
sympathetic trunk

ulnar nerve

vagus nerve
21. Which posterior mediastinal structure is most closely applied to the posterior
surface of the pericardial sac?

Aorta

Azygos vein

Esophagus

Thoracic duct

Trachea
22. In obstruction of the superior or inferior vena cava, venous blood is returned
to the heart by an alternate route via the azygos vein, which becomes dilated
in the process. Which of the following structures might it compress as a
result?
trachea root of

the left lung

phrenic nerve

thoracic duct

descending aorta
23. Elevated systolic blood pressure in the right ventricle suggests stenosis of
which valve?

Aortic
Mitral

Pulmonary

Tricuspid
24. During examination of a 62-year-old man, the senior resident tells you to put
your stethoscope on the left 5th intercostal space, slightly below the nipple,
and listen for a clearly audible murmur. You hear it distinctly and know it must
be associated with severe stenosis of which heart valve?

Aortic

Mitral

Pulmonary

Tricuspid
25. A 26-year-old male is brought into the emergency room after having been
kicked in the chest by a horse. After examination, it is concluded that the most
likely immediate danger is cardiac tamponade (bleeding into the pericardial
sac). You prepare to draw off some of the blood from the sac to relieve the
pressure on the heart. The safest site at which to insert the needle of the
syringe in order to miss the pleura would be:

Just below the nipple on the


left
Just to the left of the xiphisternal junction

Near the sternal angle

Through the jugular notch

4th left intercostal space in the midaxillary line


26. A 22-year-old male involved in an automobile accident presents with
symptoms suggestive of myocardial contusion due to blunt trauma,
specifically compression of the sternocostal surface of the heart by the
sternum when his chest hit the steering wheel. Which heart chamber was
most likely damaged?

Left atrium

Left ventricle

Right atrium

Right ventricle
27. While attempting to suture the distal end of a coronary bypass onto the
anterior interventricular artery, the surgeon accidentally passed the needle
through the adjacent vein. Which vein was damaged?

Anterior cardiac vein

Coronary sinus

Great cardiac vein


Middle cardiac vein

Small cardiac vein


28. While listening to a patient's heart sounds with a stethoscope, you identify a
high-pitched sound in the second right intercostal space, just lateral to the
edge of the sternum. Your correct conclusion is that you have detected
stenosis of which heart valve?

Aortic

Mitral

Pulmonary

Tricuspid
Pathophysiology CVS task

An example of a normal ECG (normal sinus rhythm; HR - 68 bpm):

Task 1.

A) sinus bradycardia;
B) sinus tachycardia;
C) sinus arrhythmia;
D) atrioventricular block I degree.

Reason - Sinus bradycardia is a slow, regular heartbeat It happens when your heart's
pacemaker, the sinus node, generates heartbeats less than 60 times in a minute. Sinus
bradycardia occurs on an ECG when there is a normal upright P wave in lead Il (sinus
P wave) preceding every QRS complex.

Task 2.
A) sinus bradycardia;
B) sinus tachycardia;
C) sinus arrhythmia;
D) paroxysmal supraventricular tachycardia

Reason- The ECG criteria to diagnose sinus arrhythmia is a variation of the P-P
interval, from one beat to the next, of at least 0.12 seconds, or 120 milliseconds. Sinus
arrhythmia if not in a young person and not occurring with respiration may be a sign
of-sick sinus syndrome

Task 3.

A) sinus tachycardia;
B) atrial extrasystole;
C) paroxysmal supraventricular tachycardia;
D) atrial flutter.

Reason- SVTS can be classified based on site of origin (atria or AV node)


Classification based on QRS width is unhelpful as this is also influenced by the
presence of pre-existing bundle branch block Regular tachycardia -140-280 bpm.

Task 4.
A) atrial flutter;
B) ventricular flutter;
C) atrial fibrillation;
D) ventricular fibrillation.
Reason - Atrial flutter is a type of supraventricular tachycardia caused by a re-entry circuit within the
right atrium, trial rate of around 300 bpm (range 200-400). AV blocks can occur usually due to
medications or underlying heart disease.

Task 5.

A) sinus arrhythmia;
B) atrioventricular block I degree; C) complete atrioventricular
block;
D) atrial extrasystole.

Reason- These are premature P waves which look different from a normal P wave. They may be
hidden in the ST segment or T wave of the preceding sinus beat. They may be followed either by a
normal QRS complex, or the PR interval may be prolonged, or the impulse may not be conducted at
all.They can also occur when there is increased pressure on the atria such as in cardiac failure or
mitral valve disease and may occur prior to the development of atrial fibrillation.

Task 6.
A) sinus bradycardia;
B) sinus tachycardia;
C) sinus arrhythmia;
D) paroxysmal supraventricular tachycardia.

Reason- Sinus tachycardia is recognized on an ECG with a normal upright P wave in


lead Il preceding every QRS complex, indicating that the pacemaker is coming from
the sinus node and not elsewhere in the atria, with an atrial rate of greater than 100
beats per minute.

Task 7.

A) atrial premature beats;


B) ventricular extrasystole of the bigeminy type; C) ventricular
extrasystole of the trigeminic type;
D) atrioventricular block I degree.

Reason- These are wide, abnormally shaped QRS complexes. Extrasystoles occurring
at every third beat are called trigeminy respectively.

Task 8.
A) atrial flutter;

B) ventricular flutter;
C) atrial fibrillation;

D) ventricular fibrillation.

Reason - Chaotic irregular deflections of varying amplitude. No identifiable P waves,


QRS complexes, or T waves

Task 9.

A) atrioventricular block I degree;


B) atrioventricular block II degree (type I Mobitz);
C) atrioventricular block II degree (type II Mobitz);
D) atrioventricular block III degree (complete AV block).

Reason - A First degree AV node BlOCK occurs when conduction through the AV node is slowed, thus delaying
the time it takes for the action potential to travel from the SA node, through the AV node, and to the ventricles.
A first degree AV block is indicated on the ECG by a prolonged PR interval. Recall that the P wave indicates atrial depolarization
(initiated by firing of the SA node). The atrial depolarization eventually spreads to the AV node where there is a slight delay before the
electrical impulse is conducted to the ventricles. If the AV nodal conduction (dromotropy) is decreased, it will take longer for the
impulse to reach the ventricles, so there will be a greater distance between the P wave and the QRS complex (remember the QRS
complex indicates ventricular depolarization). Thus the PR interval will be prolonged.
Task 10.

A) atrioventricular block I degree;


B) atrioventricular block II degree (type I Mobitz);
C) atrioventricular block II degree (type II Mobitz);
D) atrioventricular block III degree (complete AV block).

Reason - Second-degree atrioventricular (AV) block or second-degree heart block is a


disorder characterized by disturbance delay or interruption of atrial impulse
conduction to the ventricles through the atrioventricular node AVN and bundle of His
Electrocardiographically, some P waves are not followed by a QRS complex.

A) atrial flutter;
B) ventricular flutter;
C) atrial fibrillation;
D) ventricular fibrillation.

Reason- Atrial flutter is a type of supraventricular tachycardia caused by a re-entry


circuit within the right atrium, trial rate of around 300 bpm (range 200-400). AV
blocks can occur usually due to medications or underlying heart disease.

Task 12.
A) atrioventricular block I degree;
B) atrioventricular block II degree (type I Mobitz);
C) atrioventricular block II degree (type II Mobitz);
D) atrioventricular block III degree (complete AV block).

Reason- Some P waves are not followed by a QRS complex. It is Second-degree


atrioventricular (AV) block, or second-degree heart block, a disorder characterized by
disturbance, delay, or interruption of atrial impulse conduction to the ventricles
through the atrioventricular node (AVN) and bundle of His.

Task 13.

A) atrioventricular block I degree;


B) atrioventricular block II degree (type I Mobitz);
C) atrioventricular block II degree (type II Mobitz);
D) atrioventricular block III degree (complete AV block).

Reason- Extra beat is seen after QRS Complex with fixed P-R Interval, indicates failure of
conduction at the level of Purkinje fibres.
It works on "all or nothing" phenomenon where Purkinje Fibres fail to conduct a supraventricular
impulse

Task 14.
A) atrial flutter;
B) ventricular flutter;
C) ventricular fibrillation;
D) paroxysmal ventricular tachycardia.

Reason - Continuous Sine Wave, No identifiable P waves, QRS complexes, or T


waves, Rate usually > 200 beats / min

CARDIOLOGY (previous year internal medicine)

1. Choose the most correct interpretation of palpation data – a spilled high (dome-shaped)
apical impulse in the VI intercostal space 2 cm outwards from the mid-clavicular line:
A. Hypertrophy and dilatation of the left ventricle
B. hypertrophy and dilatation of the right ventricle
C. hypertrophy of the left atrium
D. right atrial hypertrophy
E. right ventricular hypertrophy

2. Choose the most correct interpretation of palpation data -a pronounced cardiac impulse
and epigastric pulsation:
A. Hypertrophy and dilatation of the left ventricle
B. hypertrophy and dilatation of the right ventricle
C. hypertrophy of the left atrium
D. right atrial hypertrophy
E. Left ventricular hypertrophy

3. Palpation of the heart at the apex reveals a jitter that does not coincide with the
pulsation a. carotis. What heart disease is it typical for?
A. aortic valve insufficiency
B. mitral valve insufficiency

C. aortic stenosis

D. mitral stenosis
E. tricuspid valve insufficiency

4. What clinical situation is characterized by the following changes in the auscultative


picture of the heart: at the apex I tone is weakened, systolic murmur is heard, which is
conducted in the axillary region. What is it?
A. aortic valve insufficiency
B. mitral valve insufficiency
C. aortic stenosis
D. mitral stenosis

E. tricuspid valve insufficiency

5. Mitral valve is better heard where?


A. in the fifth intercostal space on the left 1.5 cm inwards from the midclavicular line
B. in the second intercostal space to the left of the sternum

C. in the second intercostal space to the right of the sternum


D. to the right of the sternum in the fourth intercostal space E. at Botkin-Erb

Point

6. The apical impulse is formed by:


A. aortic arch

B. abdominal aorta
C. right ventricle

D. left ventricle

E. left atrium.

7. Apical impulse with aortic valve insufficiency is going to be

A. small, weak, limited


B. Small, reinforced, limited
C. high, reinforced, spilled

D. small

E. limited

8. For what clinical situation are the following options for changing the boundaries of the
relative dullness of the heart: the right border is 1 cm to the right of the right edge of the
sternum, the left one along the anerior axillary line, the top edge is the third edge?

A. mitral stenosis
B. mitral insufficiency

C. tricuspid valve insufficiency

D. aortic valve insufficiency

E. stenosis of the pulmonary valve

9. What clinical situation is characterized by the following options for changing the boundaries
of the relative dullness of the heart: the right border 1s 3 cm to the right of the sternum edge,
the eft one is 1 cm medially from the left mid-clavicle line, the upper one 1s the upper edge of
the second rib?

A. mitral stenosis

B. mitral insuficiency
C. tricuspid valve insufficiency

D. aortic stenosis
E. aortic valve insufficiency

10. What clinical situation is characterized by the following options for changing the boundaries
of the relative dullness of the heart: the right border is 1 cm to the right of the sternum edge,
the left one is 2 cm outward irom the left mid-clavicle line, the top one is II rib?

A. mitral stenosis
B. mitral insufficiency

C. tricuspid valve insufficiency

D. aortic malformations
11. Imdicate the most characteristic signs of arterial pulsus pulsus dificiens:

A. a sharp weakening or absence of pulsation in a single radial artery

B. a sharp decrease in the magnitude of the pulse in both radial arteries


C. The number of pulse waves in the radial artery is greater than the number of heartbeats.

D. The number of pulse waves in the radial artery is less than the number of heartbeats. E.
sharp wealkening or absence of pulsation in both radial arteries

12. The area of the mitral orifice is normally:

A. 1-2 cmn2
B. 3-4 cm2 C. 4-5 cm2 D. 4-6 cm2
E. 2-3 cm2

13. Critical mitral stenosis is:

A 1 cm2

B. 1.3 cm2

C. 1.5 cm2
D. 1.9 cm2

E.2 cm2

14. Left ventricular hypertrophy is established when:

A. RV5+ SV1≥ 38 mm, R aVL> 11mm


B. RV1> 7mm
C. HRVS <RV4
D. R V5-6 <15 mm
E. Availability S.S V5-6

15. Rhythm “quail” in mitral stenosis is:


A clapping tone I, enhanced tone II and the tone of the opening of the mitral Valve

B. I tone is weakened, il tone is not changed, pathological Il tone in diastole


C. I tone is not changed, II tone is weakened, systolic murmur

D. I tone is enhanced, II tone is not changed, mesodiastolic noise

E. I tone is not changed, II tone is not changed, systolic murmur

16. Specify the most informative method for diagnosing mitral valve insufficiency:
A. Auscultation of the heart

B. phonocardiography

C. Xray of the heart


D. echoppler cardiography

E. coronaroangiophagia

17. The main auscultatory sign of mitral valve insuficiency:

A. diastolic noise in the second intercostal space on the right


B. systolic murmur throughout systole at the top diastolic noise at the top

C.diastolic noise at the Botkin point

D. systolic murmur in the second intercostal space

E. systolic murmur in the second intercostal space on the left

18. A direct sign of mitral valve insufficiency in EchoCG Doppler study:


A. turbulent diastolic flow in the projection of the mitral valve

B. Blood regurgitation from the left ventricle to the left atrium during systole

C. Regurgitation jet from the aorta to the left ventricle


D. Blood regurgitation from the left ventricle to the left atrium during Diastoles F. Stream of

regurgitation from the right ventricle to the right atrium

19. A direct sign of aortic valve insufficiency in Doppler examination:


A. Regurgitation jet from the right atrium to the right ventricle
B. Regurgtaion jet rom the let ventricle to the left atrium

C. Regurgitation jet from the right ventricle to the right atrium

D. Regurgitation jet from the aorta to the left ventricle


E. Stream of regurgitation from the pulmonary trunk to the right ventricle

20. Which symptom of the following is pathognomonic for left ventricular heart failure?

A. Swelling of the neck veins


B. ascites
C. Enlarged liver
D. orthopnea
E. E Swelling in the legs

21. A pronounced pulsation of the carotid arteries is observed in patients with:

A. Aortic stenosis
B. Aortic valve insufficiency
C. C. mitra valve insufficiency
D. D. tricuspid valve insuffīciency
E. E. Insufficiency valve of the pulmonary artery

22. Which of the heart defects most significantly increases the left atrium?

A. Pulmonary artery stenosis


B. Aortic valve insufficiency
C. mitral stenosis
D. Aortic stenosis
E. Tricuspid valve insufficiency

23. When accent of II tone on the aorta occurs:

A. Hypertension syndrome
B. syndrome of acute coronary insufficiency
C. myocardial inflammation syndrome
D. Pericardial inflammation syndrome
E. chronic coronary insuficiency syndrome
24. Forced position orthopnea facilitates the patient’s condition with:
A. hypertension syndrome

B. syndrome of acute coronary insufficiency

C.chronic coronary insufficiency syndrome


D. left ventricular heart failure syndrome

E. Syndrorne of arterial hypotension

25. Typical localization of pain in chronic coronary syndrome Failure:

A. behind the sternum

B.In the left half of the chest, in the apex of the heart
C. in the right side of the chest

D. in the epigastric region

E. in the right hypochondrium

26. Sinus tachycardia is a sinus rhythm with a heart rate.


Abbreviations:

A. More than 85 per minute


B. more than 60 per minute
C. more than 75 per minute
D. more than 90 per minute
E. more than 120 per minute

27. What is meant by “mitral face”?

A. Erythematous rash on cheeks and nasal back


B. pallor of face

C. Cyanotic cheek blush

D. hyperemia of the left cheek


E. cyanosis of nasolabial triangle
28.dyspnea;

A. Dizziness
B. Shortness of breath
C. Chest pain
D. high blood pressure
E. Headaches

29. When are the atria and ventricles contracted by their own rhythm?

A .with atrioventricular blockade of III degree

B. atrial fibrillation
C.With atrial paroxysmal tachycardia

D. with ventricular premature beats

E. atrial futter

30. What is characteristic radiographically for aortic valve insafficiency:


A. The aorta is dilated, pulsating deeply, ejects the IV arch

B.protrudes the left atrial appendage

C. protrudes pulmonary trunk


D. Availability of Curly lines

E aorta dilated in ascending division

31. The appearance on the ECG of a pathological Q wave in acute coronary insufficiency
syndrome is a reflection of:

A. subendocardial myocardi al ischemia

B. myocardial necrosis
C. myocardial damage

D. Subepicardial myocardial ischemia

E. scarring myocardium

32. The ECG criterion for chronic coronary insufficiency syndr omeis:
A. appearance of abnormal Q wave

B. reduction of the amplitude of the R wave

C. ST segment displacement below the isoelectric line by 1 mm or more


D. ST segment elevation above the isoelectric line more than 1 mm

E. Negative Twave in all chest leads

33. What is the pulse characteristic of aortic valve insufficiency?

A. pulsus celer et altus

B. pulsus rarus et parvus


C.pulsus differens

D. pulsus filiformis

E. pulsus parvus

34. What is the cause of noise in aortic stenosis:

A. Blood regurgitation from the aorta to the left ventricle B.


The expulsion of blood from the left ventricle to the aorta.
C. C. Blood regurgitation from the left ventricle to the left atrium
D. D. Expulsion of blood from the left atrium to the left ventricle
E. E. Blood regurgitation from the right ventricle to the right atrium

35. The occurrence of noise in mitral stenosis is caused by:

A. Expulsion of blood from the left ventricle to the aorta


B. Regurgitation of blood from the left ventricle to the left atrium

C. Regurgitation of blood from the right ventricle to the right atrium


D. Expulsion of blood from the left atrium to the left ventricle

E. Regurgitation of blood from the aorta to the left ventricle

36. The cause of the noise in the aortic valve insufficiency:


A .Blood regurgitation from the aorta to the left ventricle
B. Blood regurgitation from the right ventricle to the right atrium

C. blood regurgitation from the left ventricle to the left atrium

D. Expulsion of blood from the left ventricle to the aorta


E. Expulsion of blood from the left atrium to the left ventricle

37. At what defect is the diastolic murmur at the apex heard?

A. mitral stenosis

B.Mitral valve insufficiency


C. tricuspid valve insufficiency

D. Aortic stenosis

E.Aortic valve insufficiency

38. What changes in echocardiography of the following are a sign of left ventricular heart
failure:

A. EF <80%, EDDLV> 5.0 cm


B. EF <50%, EDDLV> 5.5 cm, diffuse hypokinesis of the LV walls
C. EF 60%, LVPWT-1,2cm
D. EF = 60%, local hypokinesis IVST
E. Echonegative space for LVPWT 10mmn

39. For what pathological condition is the “triangular” shape of the heart configuration
characteristic?

A. Mitral stenosis
B. Tricuspid valve insufficiency
C. aortic valve insufficiency
D. pericardial effusion

E. acute coronary insufficiency

40. Positive venous pulse is characteristic:


A. for mitral stenosis

B.For aortic stenosis

C.For aortic valve regurgitation


D. For tricuspid valve insufficiency

E. for heart failure

41. Pulmonary valve is better listened at:


A. in the fifth intercostal space on the left 1.5 cm inwards from the Midclavicular line

B.In the second intercostal space to the left of the sternum

C. in the second intercostal space to the right of the sternum


D. to the right of the sternum in the fourth intercostal space

E.At Botkin-Erb Point

42. The symptom of “cat’s purr” is determined by the method:

A. percussion
B.Auscultation

C.palpation

D. phonocardiography
E. electrocardiography

43. The symptom of “cat’s purr” in the apex of the heart is determined by:
A. acute coronary insufficiency

B. mitral valve insufficiency

C. stenosis of the left AV opening


D. aortic valve regurgitation

E. tricuspid valve insufficiency


44. The following ECG indicators: the correct rhythm with a heart rate of <60 per min. And
unchanged QRS complexes are characteristic of: A. sinus arrhythmia .
B. blockade of bundle branch foot

C.sinus bradycardia

D. extrasystolic arrhythmia
E. atrial fibrillation

45. Palpation on the basis of the heart reveals a tremor, which coincides with the pulsation on
a. carotis. This is typical for: A. mitral stenosis

B. mitral valve insufficiency

C. aortic stenosis

D. aortic valve regurgitation


E. tricuspid valve insufīciency

46. The value of blood pressure measured in a patient at rest and equal to 140/85 mm Hg,
should be considered in accordance with modern guidelines,as
A .1st degree arterial hypertension

B. 2nd degree of arterial hypertension

C. high normal blood pressure


D. normal blood pressure

E. optimal blood pressure

47. When auscultation of the heart in patients with severe heart failure can be identified:

A. rhythm "quail"
B. protodiastolic gallop rhythm

C. systolic gallop rhythm

D. Additional pericardium tone

E. gain I tone at the top


48. The pulsation of the carotid arteries (“carotid dancing”) is a sign:

A aortic valve regurgitation

B. tricuspid valve insufficiency


C. stenosis of the mirtal valve

D.Mitral valve insufficiency

E. Aortic stenosis

49. Complete AV blockade is characterized by:


A. increase the PQ interval

B. independent atrial and ventricular contractions

C. at different RR intervals
D.Severe deformity of the ventricular complex

E. falling out of the next h. P and QRS complex

50. II tone with aortic stenosis:

A .weakened on the aorta

B.Strengthened on the aorta


C.strenghened at the top

D. weakened at the top

E. not changed on the aorta

51. When aortic stenosis is heard:


A. diastolic murmur at the apex

B. diastolic noise in the second intercostal space on the right

C. systolic marmur in the second intercostal space on the right

D. systolic murmur at the apex


E. systolic murmur on tricuspid valve
52. Paradoxical pulse is determined by
A. heart failure syndrome

B. syndrome of arterial hypertension;

C. coronary insufficiency syndrome;


D. myocardial inflaxnnmation syndrome

E. Pericardial inflammation syndrome

53. ECG signs of atrioventricular block I degree:

A. prolongation of the interval PQ> 0.12 sec

B. prolongation of the interval PQ> 0.20s


C.No P wave

D.Deformation of the QRS complex

E. negaxive T wave

54. Name the components of the first tone.


A. muscular, valvular, vascular

B. muscular, valvular, vascular, atrial

C. muscle, valve
D. Valve, vascular
E. muscular vascular

55. When aortic valve insufficiency is heard

A. systolic murmur at the base of the xiphoid process

B. Protodiastolic noise at Botkin-Erb point


C. systolic murmur at the Botkin -Erb point

D. diastolic murmur at the base of the xiphoid process

E. diastolic murmur at the apex of the heart


56. Amplificatica of the apical impulse indicates

A. right ventricular hypertrophy

B. hypertrophy of the right atrium


C. hypertrophy of the left atrium

D. Left ventricular hypertrophy

E. dilatation of the left atrium

57. Accent of tone II on the pulmonary artery is noted when


A. Arterial hypertension

B.pulmonary hypertension

C. left ventricular hypertrophy

D. dilatation of the left ventricle


E.Pulmonary valve insufficiency

58. Dysphagia in mitral stenosis is due to;


A.spasm of the esophagus

B.compression of the esophagus by the enlarged right atrium;

C. compression of the esophagus because of enlarged left atrium; D.


compression of the esophagus by the enlarged pulmonary artery;
E. compression of the esophagus by the dilated left ventricle.

59. Select the symptom complex characteristic of left ventricular heart failure syndrome:
A. Swelling of the legs, cough, shortness of breath

B.cough, hemoptysis, choking, orthopnea

C. hemoptysis, swelling of the legs, enlarged liver

D. swelling and pulsation of the neck veins


E. cough, swelling of the legs
60. When mitral valve insufficiency auscultatory is determined:

A. Strengthening of the I tone at the apex of the heart

B. weakening of the I tone at the apex of the heart


C. “click open” mitral valve

D. diastolic murmur at the apex

E.Systolic aortic murmur

61. Systolic murmur with stenosis of the aortic mouth is perfor med

A. To the point of Botkin-Erb


B. on the xiphoid process
C. to the armpit
D. On the neck vessels
E. To the top of the heart

62. Which of the following factors predisposes to the development of coronary insufficiency
syndrome:

A. Lasting blood sugar 4.5 mmol/1


B. Blood pressure 120/80 mm Hg.
C. Blood cholesterol 5.0 mmol /1
D. Smoking
E. Increased high density lipoprotein

63. Subjective characterization of pain in chronic coronary insufficiency syndrome:


A .compressive, pressing

B. stabbing

C. aching
D.Dagger

E. cutting

64. Symptom Musset is observed at


A. Aortic stenosis
B. Stenosis of the mirtal valve
C. Aortic valve regurgitation
D. tricuspid valve insufficiency
E. E. mitral valve insufficiency

65. “Heart Hump” occurs when

A. severe left ventricular hypertrophy


B. severe left atrial hypertrophy

C. severe right ventricular hypertrophy

D. severe right atrial hypertrophy

E. severe dilatation of the left atrium

66. A sign of acute left ventricular failure is:


A. choking with choking breath;

B. swelling and pulsation of the veins of the neck;

C. enhanced epigastric ripple;


D. liver enlargement and tenderness

E. swelling of the lower limbs

67. Forced position of the patient sitting bending forward, is observed when:

A. coronary insufficiency
B. effusion pericarditis;

C. mitral valve insufficiency

D. myocarditis
E. Arterial hypertension

68. “Deficit” pulse occurs when

A. Arterial hypertension
B. heart failure
C. Atrial fibrillation
D. Aortic valve regurgitation
E. atrioventricular block

69. The upper limit of relative cardiac dullness increases due to

A. Tight ventricular hypertrophy


B. Left ventricular hypertrophy

C. hypertrophy of the right atrium

D. Left atrial hypertrophy

E.dilatation of the left ventrirle

70. Which symptom of the following is characteristic of left ventricular heart failure:

A. Lower limb edema


B. Enlarged liver
C. inspiratory dyspnea
D. Ascites
E. Positive venous pulse

71. What symptom of the following is characteristic of right ventricular heart Failure:

A. Hepatomegaly
B. inspiratory dyspnea
C. symptom Musset
D. dry cough
E. moist rales in the lungs during auscultation

72. Absolute ECG sign of acute coronary insufficiency syndrome:

A. pathological Q wave
B. negative T wave
C. ST interval offset
D. Absence of P-wave
E. Prolongation of the PQ interval

73. Diastolic noise with aortic valve insufficiency is better heard:

A. in the second intercostal space on the right

B. at Botkin-Erb Point

C.On the xiphoid process


D. at the apex of the heart

E. on the vessels of the neck

74. The most informative method for diagnosing coronary insufficiency is:

A. ECG
B. Veloergometry
C. Echocardiography
D. Coronary angiograpny
E. Daily ECG monitoring

75. The man metnod for tne diagnosis of cardiac arrhythmia syndrome is ;

A. ECHO
B. ECG
C. chest X-ray

D. phonocardiography
E. coronary angiography

76. The most informative method for confirming necrotic changes in the myocardium is

A. determination of erythrocyte sedimentation rate and leukocytes

B. determination of the level of alkaline phosphatase


C. Determination of transaminase level

D. determination of troponin level

E. determination of creatinine level


77. Identification on the ECG of the interval PQ, equal to 0.28 seconds. indicates the presence of
the patient

A. Sinoatrial blockade
B. blockade of atrioventricular conduction

C. extrasystolic arrhythmia

D. atrial fibrillation
E. paroxysmal ventricular tachycardia

78. The most reliable sign of stenosis of the left atrioventricular orifice is

A. increase the left border of the heart


B. facies mitralis

C. mesodiastolic murmur and the presence of a "opening click" mitral valve at the top

D. simoothness of the waist of the heart during X-ray


E. weakening of the apical impulse

79. For mitral valve insufficiency it is typical:

A. clapping the first tone at the top

B. systolic murmur at the apex

C. increase the boundaries of the heart to the right


D. weakening of the II tone on the aorta

E. diastolic murmur at the apex

80. Blood pressure in aortic valve insufficiency:

A. only systolic increases


B. only diastolic increases

C. systolic and diastolic increases

D. Systolic decreases and diastolic increases


E. does not change
81. Indicate aortic valve insufficiency:

A. 120/90 mm Hg

B. 190/140 mm Hg
C. 160/100 mm Hg

D. 160/40 mm Hg

E. 90/60 mm Hg
82. What can be observed in endocardial inflammation syndrome?

A. leukopenia, lymphocytosis

B.Anemia, leukocytosis, accelerated ESR


C. eosinophilia, accelerated ESR

D. anemia, lymphocytosis, accelerated ESR

E. anemia, leukopenia, thrombocytopenia

83. Specify the level of blood pressure, corresponding to the I degree of arterial hypertension:
A. Blood pressure – 150/95 mm Hg

B. Blood pressure -138/88 mm Hg


C. Blood pressure – 174/108 mm Hg

D. Blood pressure – 202/114 min Hg

E. Blood pressure – 200/90 mm Hg

84. For the development of coronary insufficiency:


A. increased levels of low density lipoprotein

B. Age over 45

C. hypotension

D. male
E.genetic predisposition

85. Pulsation of the pupils is characteristic o

A. mitral stenosis
B. Tricuspid insufficiency:
C. Aortic stenosis;
D. tricuspid stenosis;
E. aortic insufficiency

86. Which method is most effective in verifying left ventricular myocardial hypertrophy:

A. ECG
B. Veloergometry
C. Echocardiography
D. Coronary angiograpny
E. transesophageal pacing

87. Which of the following is an unmodified risk factor for the development of coronary
insufficiency:

A. increased levels of low density lipoprotein


B. diabetes

C. Arterial hypertension

D. smoking
E. genetic predisposition

88. When stenosis of the mouth of the aorta is heard

A. Systolic murmur in the apex of the heart, radiating to the axillary region
B. diastolic murmur at the apex of the heart, radiating to the base of the xiphoid process
C. Systolic murmur in the second intercostal space to the right of the sternum, Radiating
to the carotid arteries
D. diastolic murmur in the second intercostal space to the right of the sternum raciating to
the carotid arteries
E. systolic murmur at the apex of the heart radiating to the base of the xiphoid Process

89. The weakening of the I tone is observed at:

A. mitral insufficiency

B. tricuspid insuficiency
C. aortic insufficiency

D. mitral stenosis

E. in a healthy person
90. Specify the correct definition of the term “heart push”:

A. ripple to the right of the xiphoid process


B. Local ripple in the third intercostal space at the left edge of the sternum

C. diffuse ripple throughout the precardiac area

D. Pulsation in the fourth, fifth, sixth intercostal space at the left border of the relative dullness
of the heart

E. ripple to the left of the xiphoid process

91. To detect pericardial effusion, the most informative method is

A. ECG
B. Coronary angiography
C. Echocardiography
D. X-ray examination of the chest organs
E. phonocardiography

92. Pulse pressure is:

A. Difference between systolic and diastolic blood pressure


B. stroke volume of the left ventricle
C. Amount of blood released by the ventricles to systole
D. The amount of blood released by the atria to the ventricles
E. minute heart volume

93. Small, slow, rare pulse is characteristic for:

A. mitral stenosis
B. Mitral insufficiency
C. aortic valve insufficiency
D. Aortic stenosis
E. Thyrotoxicosis
94. Apical impulse for aortic insufficiency:
A. weakened, localized;

B.reinforced, localized;

C. tall, reinforced, spilled;


D. weakened, spilled

E. Reinforced, localized

95. Specify the cause of acute coronary insuficiency:

A Coronary artery thrombosis

B. Inflammation of the coronary arteries


C. inflammation of the coronary veins

D. Arterial hypertension

E. Left ventricular myocardial hypertrophy

96. A 42-year-old patient has a significant shift in the boundaries of the relative dullness of the
heart to the left, increased apical impulse, a pronounced symptom of “systolic tremor” in the
2nd intercostal space to the right of the sternum and ibid weakening of the II tone. Which of the
following auscultatory phenomena must necessarily occur in this patient?

A. systolic murmur at the apex


B. diastolic murmur at the apex.

C. systolic murmur above the aorta

D. systolic murmur at xiphoid process

E. diastolic murmur over the aorta

97. The most informative method for confirming the presence of heart disease is:
A. ECG

B.Echocardiography
C. roentgenoscopy of the chest

D. Coronary angiograpny

E. Bicycle ergometry test

98. Large pulse pressure, double Traube tone and duozier noise on the vessels, fast and high
pulse, Musset’s symptoms are characteristic

A. For aortic valve regurgitation


B. for aortic stenosis
C. For tricuspid valve insufficiency
D. for mitral stenosis
E. For mitral valve insufficiency

99. Indicate the physical sign of pericardial inñammation syndrome:

A. Appearance of 3 tones
B. pansystolic noise at the top
C. appearance 4 tones
D. Pericardial friction noise

E. attenuation 1 tone

100. The orthopneic position is:

A. forced sitting position to relieve shortness of breath

B. forced sitting positon to relieve pain


C. forced lying position to relieve pain

D. forced standing position to relieve hiccups

E. forced standing position to relieve shortness of breath

101. In case of pathology of which organ of the following, secondary arterial hypertension may
develop:

A. liver
B. Adrenal glands
C. stomach
D. Vessels of the lower extremities,
E. Lungs

102. What is the peculiarity of pain in acute coronary insuficiency:

A. is associated with physical activity


B. Duration over 30 minutes
C. Docked with NTG
D. Passes in peace
E. associated with breathing

103. The increase in pulse pressure is observed at:

A. Aortic stenosis
B.Aortic insufficiency

C. mitral insufficiency

D. tricuspid insufficiency
E. mitral stenosis

104 Which echocardiography sign is the criterion for the diagnosis of endocardial inflammation
syndrome:
A .vegetation on the valves

B. mitral regurgitation

C. dilatation of the left ventricle


D.Cusp perforation

E. calcification of valve cusps

105. Echocardiographic sign of acute coronary insufficiency

A. Diffuse hyperkinesis of the walls of the left ventricle


B. diffuse hypokinesis of the walls of the left ventricle
C. Local hypokinesis of the walls of the left ventricle
D. Local hyperkinesis of the walls of the left ventricle
E. E. Left ventricular dilatation

106. Target organs for hypertension:


A Heart, kidneys, brain, fundus vessels and aorta

B. Liver, lungs

C. heart, liver
D. fundus vessels, lungs

E .kidney, heart

107. The area of the mouth of the aorta in the norm is:

A. 1.5-2.5 cm2
B. 4-4.5 cm2
C. 4.5-5.5 cm2
D. 2.5-3.5 cm2
E. 1-2 cm2

108. The tone of the opening of the mitral valve is heard:

A .at the apex of the heart

B. in the second intercostal space on the right at the sternum


C. in the second intercostal space to the left of the sternumn

D. at the base of the xiphoid process

E. at Botkin-Erb Point

109. For stenosis of the left atrioventricular orifice characteristic


A. systolic murmur at the apex of the heart, aggravated by exhalation

B. Accent and split II tone over the aorta

C. systolic murmur at the apex of the heart, aggravated by inspiration


D. Additional high-frequency tone in diastole, separated from tone II by 0.07-0.12 Seconds
E.Weakening of the I tone at the top

110. The most informative method for the diagnosis of myocardial inflammation syndrome:

A. ECG
B. ECHO
C. Bicycle ergometry
D. D. Coronary angiography
E. E radiography

111. What is the symptom characteristic of the appearance of patients with Stenosis of the
aortic mouth:

A. Diffuse cyanosis of the skin


B. pallor of the skin
C. symptom Musset
D. “dancing carotid”
E. capillary pulse

112. Of the listed risk factors for coronary insufficiency, the most significant “triad” is:

A. Obesity, hypodynamia, male


B. hypercholesterolemia, hypertension, smoking
C. Arterial hypertension, hypodynamia, obesity
D. Impaired carbohydrate tolerance, obesity, smoking
E. menopause, obesity, hypodynamia

113. A common symptom in the objective status of patients with aortic valve insufficiency and
aortic stenosis is:
A .pallor of the skin

B. symptom Musset

C. pulsation of the carotid, subclavian, temporal, brachial arteries

D. capillary pulse
E. Decrease in diastolic blood pressure

114. For patients with myocardial inflammation syndrome, the following complaints are most
characteristic:

A. pain in the heart, palpitations, shortness of breath


B. pain in the heart, palpitations, Iainüng
C. pain in the heart, shortness of breath, ascites

D. pain in the heart, fever, dry cough

E. fainting, fever

115. Specify the anti-atherogenic class of lipoproteins:

A. total cholesterol
B. HDL-cholesterol
C. LDL cholesterol
D. Triglycerides
E. chylomicrons

116. Click open mitral valve

A.occurs 0.06-0.12 seconds after the closure of the aortic valves


B.Characteristic of mitral insufficiency

C.characteristic of aortic stenosis

D.Best heard at Botkin-Erb


E.Characteristic of aortic insufficiency

117. Paradoxical pulse is:

A. alternation of pulse waves of large and small filling:

B. decrease or disappearance of inhalation pulse waves;

C. discrepancy between the number of pulse waves and the number of heartbeats;
D. unequal pulse wave intervals

E. amplification of the pulse wave on the exhale


118. Name the ECG-sign, which is a reflection of myocardial ischemia:

A. decrease in the amplitude of the R wave


B. Deep, broad Q wave
C. ST segment depression
D. deformation of the QRS complex
E. Deformation of the P wave
119. Quincke precapillary pulse occurs when:

A. stenosis of the mouth of the aorta;

B.Aortic valve insufficiency;


C. mitral stenosis;

D. mitral insufficiency;

E. pulmonary insufficiency.

120. One of the following laboratory indicators confirms the syndrome of acute coronary
insufficiency in a patient with an anginal attack:

A. decreased hemoglobin
B. Hyperlipidemia

C. Increased ESR

D.Increased activity of serum troponins


E. lymphocytosis

121. Conduction disturbances include:

A. Extrasystolic arrhythmia
B. atrial fibrillation
C. Paroxysmal tachycardia
D. AV block
E. Sinus tachycardia

122. Hypertension is characterized by the following level of increase in blood Pressure:


A. ≥130/90 mm Hg

B.> 140/100 mm Hg

C. ≥140/90 mmHg
D. ≥130/80 mm Hg

E ≥120/80 mm Hg
123. The appearance of a large-wave undifferentiated line on the ECG in the absence of a P
wave and a QRS complex indicates:

A. Atrial fibrillation
B. Ventricular fibrillation
C. Heart asystoles
D. ventricular tachycardia
E. extrasystolic arrhythmia

124. When asystole on the ECG is recorded:


A. SImall wave undifferentiable line
B. large-wave undifferentiable line

C. straight line

D. Absence of P wave and change of QRS form

E. the presence of pauses lasting up to 3 seconds

125. The right border of the absolute dullness of the heart is normally located:
A. on the right edge of the sternum;

B. on the left edge of the sternum;

C. 3 cm medially from the right mid clavicular line;


D. along the percutaneous right line.

E on the midclavicular line

126. For the second tone of the heart, the following statement is true:
A. between II and I heart tones a short pause;

B. louder at the base of the heart;

C.Coincides with the apical impulse;


D. coincides with the pulse of the carotid arteries;

E. slightly lower and longer than I tone.


127.Acrocyanosis is determined by:

A. on the back

B. on mucous membranes
C. on auricles, nose tip, fingers and toes

D. in the forearm area

E.On the lower limbs

128.The most informative laboratory test confirming the diagnosis of endocardial intlammation
syndrome is:

A. sharp acceleration of ESR


B. anemia

C.Neutrophilic leukocytosis
D. leukopenia

E. Positive blood culture at blood cudture

129. Accent II tone in the 2nd intercostal space on the right is noted when:

A. Aortic stenosis
B.Aortic insufficiency

C. Arterial hypertension

D. pulmonary arterial hypertension


E. tachycardia

130. What is the level of blood pressure of the following corresponds to the 3 rd degree of
arterial hypertension A. 140/90 mm Hg

B. 180/110 mm Hg

C. 160/100 mm Hg
D. 150/99 mm Hg

E.170/100 mm Hg

131. Normaky, the duration of the ventricular QRS complex:

A. more than 0.12”


B. More than 0.11”
C. Less than 0.10”
D. Less than 0.14”
E. Less than 0.02

132. Wave P on ECG reflects:

A. Depolarization of the interventricular septum


B. B. depolarization of the left ventricle
C. C. repolarization of the right ventricle
D. D. Atrial depolarization
E. E. repolarization of the left ventricle

133. In mitral stenosis, the borders of the heart are displaced:


A. Left and up

B. Left and down

C.Right and left


D. Right and up

E. not shifted

134. In mitral insufficiency, the borders of the heart are displaced:

A. Left and up
B. Left and down
C. Right and left
D. not shifted
E. Right and up

135. In case of insufficiency of the aortic valve, the border of the heart is displaced:

A. Left and up

B. Left and down


C.Right and left

D. not shifted

E.Right and up
136. With compensated stenosis of the aortic aperture, the borders of the heart are displaced:
A. Left and up

B. Left and down


C.Right and left

D. not shifted

E.Right and up

137. What level of blood pressure of the following do you consider normal?
A. 160/70 mm Hg

B. 180/80 mm Hg

C.120/90 mm Hg
D. 140/80 mm Hg

E. 130/80 mm Hg

138. The mitral configuration of the heart is:


A. unchanged borders of the heart

B. enlargement of the right ventricle


C. enlargement of the left ventricle

D. endargement of both ventricles

E. enlargement of the left atrium and cone of the pulmonary artery

139. On the ECG, the absence of P-waves, waves f and diferent distances RR are Characteristic
of:

A. Atrial futter
B. Ventricular flutter
C. Atrial fibrillation
D. Supraventricular paroxysmal tachycardia
E. extrasystolic arrhythmia
140. On an ECG, ventricular extrasystolic arrhythmia is characterized by the appearance of:

A. ST segment depression

B. widened and deformed QRS after unchanged P wave


C. premature extraordinary altered ventricular QRS complex

D.Premature extraordinary appearance of unchanged ventricular QRS complex E.

periodic loss of the QRS complex

141. Affection of the heart in the syndrome of primary arterial hypertension is manifested:

A. hypertrophy of the left atrium

B. Right ventricular hypertrophy


C. Left ventricular hypertrophy

D.Dilatation of the left atrium

E.Hypertrophy of the right atrium

142. Select the criterion for left ventricular hypertrophy:

A. R/S V1>1
B. R/S VS5 <1
C. Rv5+Sv1 ≥38mm
D. Sv1+ Rv5 ≥10.5mm
E. Rv1> 7mm

143. What changes on the ECG do you expect to get in a patient with chronic coronary
insuffciency syndrome outside the attack:

A. the appearance ofnegative T waves


B. ST elevation above isoline
C. Shift down the ST segment
D. normal ECG
E. Pathological Q wave

144. The most characteristic complaints in chronic coronary insufficiency are:

A. pain in the heart of stabbing nature


B. pain in the left half of the chest for up to several hours, associated with the act of breathing
C. pain in the left half of the chest, resulting from a change in body position

D. burning pains behind the sternum, radiating to the neck and left arm, occurring during
physical exertion, arrested at rest or after taking nitroglycerin in 1-2 minutes E. Stitches in
the precordial region, at night, up to 3-4 hours

145. In mitral stenosis I tone:

A. Weakened at the top


B. strengthened at the base of the xiphoid process
C. “flapping’ at the top
D. weakened in the second intercostal space on the right
E. strengthened in the second intercostal space on the left

146. The tone of the opening of the mitral valve is heard:

A .at the apex of the heart

B. in the second intercostal space on the right at the sterrnum;

C. in the second intercostal space to the left of the sternum;


D. at the base of the xiphoid process

E. at Botkin-Erb Point

147. The patient at the 4th auscultation point revealed a systolic murmur, which increases with
inspiration. This is typical for:

A. mitral valve insufficiency;

B. stenosis of the right atrioventricular orifice;


C. tricuspid valve insufficiency

D. mitral stenosis;

E pulmonary valve insufficiency.

148. ECG changes with stenosis of the aortic mouth:

A. signs of left ventricular hypertrophy


B. .Signs of left atrial hypertrophy
C. signs of right ventricular hypertrophy
D. Signs ofright atrial hypertrophy
E. blockade of the right bundle branch block

149. Sinus bradycardia is often observed:

A. when eating
B. well-trained athletes at rest
C. Hypotension
D. emotional stress
E. Arterial hypertension

150. For pericardial friction noise is typical:

A. listened in places auscultation valves;


B. held in the axillary region;

C. increases with pressure on the chest with a stethoscope;

D. is heard in the zone of relative dullness of the heart;

E. disappears when inhaling.

151. ECG is a sign of atrial extrasystolic arrhythmia:


A. the presence of a P wave in front of an extraordinary unchanged QRS complex followed by a
compensatory pause

B. irregular rhythm with QRS complex deformation

C. loss of every other QRS complex


D. the absence of a P wave in front of an extraordinary QRS complex, an extension of the QRS
complex

E. Fwaves

152. Three-leaved valve is better heard

A. In the fifth intercostal space on the left 1.5 cm inwards from the midclavicular line
B. In the second intercostal space to the left of the sternum
C. in the second intercostal space to the right of the sternum
D. to the right of the sternum in the fourth intercostal space
E. At Botkin-Erb Point

153. Paroxysmal supraventricular tachycardia on ECG is manifested:

A. increase in heart rate to 160 min


B. The presence of wide deformed QRS complexes with a frequency of 140-220 per
minute
C. Frequent irregular appearance of ventricular QRS complexes without s.P
D. Normal unchanged QRS complexes with a frequency of >160 per minute
E. the emergence of extraordinary QRS complexes

154. Pathological Q wave on the ECG is detected when:

A. for chronic coronary insufficiency


B. with let ventricular hypertrophy;

C. with right ventricular hypertrophy;


D. Acute coronary insufficiency

E. inflammation of the pericardium

155. Determine the characteristic color of the skin with endocarditis:

A. Pallor
B. yellowness
C. Limited cyanotic blush
D. Color “coffee with milk”
E. Unilateral hyperemia of the cheeks

156. What level of blood pressure of the following corresponds to arterial hypertension of II
degree: Blood pressure A. 150/94 mm Hg
B. AD-138/80 mm Hg
C. AD -174/108 mm Hg

D. Blood pressure – 160/114 mm Hg

E Blood pressure -200/100 mm Hg8


157. The composition of atherosclerotic plaque includes:

A. tire and plaque core

B. spikes, tire
C. nucleus and processes

D. podocytes

E.Podocytes and nucleus

158.Laboratory indicators characteristic of acute coronary insufficiency:

A. an increase in troponin I or T
B. Anemia
C. Proteinuria
D. lymphopenia
E. leukocyturia

159. The most common causes of myocardial inflammation syndrome are:

A. Streptococcus

B. viruses

C.Staphylococcus
D. fung

E. parasites
CVS module. Examination control tests (MCQ) for
pathophysiology for 3 course students.
1. Substances with depressor action

A. adrenaline

B. angiotensin II

C. leukotriens C4, D4

D. prostaglandins E, A

2. Substances with depressor action

A. nitric oxide

B. angiotensin II

C. aldosterone

D. endothelins

3. Substances with presser action

A. adrenaline

B, kallikrein

C. prostaglandin E

D. nitric oxide

4. Substances with presser action

A. kallikrein

B. angiotensin II

C. prostaglandin E

D. nitric oxide

5. Substances with presser action

A. kallikrein

B. endothelin I

C. prostaglandin E

D. nitric oxide
6. What blood pressure values indicate the presence of

hypertension?

A. 170/110

B. 110/70

C. 120/80

D. 115/60

7. The secondary arterial hypertension is usually the

symptom of:

A. Liver disease

B.GIT disease

C. Renal disease

D. Lung disease

8. The secondary arterial hypertension is the symptom

of:

A. chronic adrenal insufficiency

B. gastric ulcer

C. primary aldosteronism

D. hypocorticism

9. Pathogenesis of renovascular arterial hypertension:

A. activation of rennin-angiotensin-aldosterone system

B. increase of secretion of glucocorticoids

C. decrease of secretion of depressor substances in

kidneys

D. increase of secretion of adrenaline


10. Pathogenesis of renoparenchymal arterial

hypertension:

A. activation of rennin-angiotensin-aldosterone system

B. increase of secretion of ADH

C. increase of secretion of adrenaline

D. inappropriate secretion of kinins, prostaglandins in

kidney

11.Arterial hypertension due to the coarctation of the

aorta relates to:

A. cardiovascular arterial hypertension

B. neurogenic arterial hypertension

C. endocrine arterial hypertension

D. drug arterial hypertension

12. Effects of Vasopressin in hypertension

development is:

A. increased natriursis

B. antagonize the pressure effects of angiotensin II

C. activate RAAS

D. increase blood volume

13. Renoprival hypertension is result of:

A. pathological dominant in cortical vasomotor area

B. excess of ADH

C. activation of RAAS

D. loss of renal depressor function


14. Increased systemic arteriolar resistance (TPR) is

major trigger mechanism of:

A. renoprival hypertension

B. primary (essential) hypertension

C. endocrine hypertension

D. renal hypertension

15. Hypertension due to Renal trigger mechanism is:

A. pathological dominant in cortical vasomotor area

B. excess of ADH

C. activation of RAAS

D. excess of atrial natriuretic peptide (ANP)

16. Vessel autoregulation imbalance in essential

hypertension is characterized by:

A. increased production of endothelin

B. increased production of Nitric oxide

C. increased arteriolar lumen

D. decreased basal arteriolar tone

17. Secondary hyperaldesteronism in essential

hypertension is result of:

A. increased sympathetic output

B. activation of depressor renal function

C. activation of RAAS

D. hyperplasia of adrenal cortex (Conn’s disease)


18. Hypertension may be developed due to:

A. pathological dominant in cortical vasomotor area

B. deficiency of ADH

C. excess of atrial natriuretic peptide (ANP)

D. excess of renal kinine family synthesis

19. Major mechanism of primary (essential)

hypertension is:

A. increased systemic arteriolar resistance

B. coarctation of aorta

C. left ventricular hypertrophy

D. impairment of pressure buffer mechanism

20. Primary hyperaldesteronism, which cause

hypertension is result of:

A. increased sympathetic output

B. activation of RAAS

C. hyperplasia of adrenal cortex (Conn’s disease)

D. release of ADH

21. Hypertension due to excess of catecholamines is

result of:

A. Decreased cardiac output (CO)

B. Increased CO

C. Decreased TPR

D. Activation ANP synthesis


22. Mechanism of secondary hypertension resulting

from kidney diseases is:

A. coarctation of aorta

B. left ventricular hypertrophy

C. impairment of pressure buffer mechanism

D. activation of RAAS

23. Orthostatic collapse may be due to:

A. massive bleeding

B. pancreas crushing

C. intestinal infections

D. rapid transition from horizontal to vertical position

24. Symptomatic arterial hypotension may be due to:

A. anemia

B. pheohromocytoma

C. hyperaldosteronism

D. Cushing’s disease

25. Symptomatic arterial hypotension may be due to:

A. cholemia

B. pheohromocytoma

C. hyperaldosteronism

D. Cushing’s disease
26. Etiology of sinus tachycardia is:

A. increased sympathetic effects on the heart

B. decreased sympathetic effects on the heart

C. hypothermia

D. hyperoxaemia

27. Etiology of sinus bradycardia:

A. hypertermia

B. increased parasympathetic effects on the heart

C. increased sympathetic effects on the heart

D. hypoxia

28. Pathogenesis of respiratory arrhythmia:

A. formation of ectopic impulses

B. disturbance of tone of n. vagus

C. violation of excitation from the atria to the

ventricles

D. re-entry mechanism

29. What is transverse heart block?

A. violation of excitation by right bundle branch block

B. violation of excitation by left bundle branch block

C. violation of impulse conduction from the atria to the

ventricles through the atrioventricular node

D. formation of ectopic focus in the ventricles


30. Hemodynamic disorders appear due to:

A. sinus tachycardia

B. sinus arrhythmia

C. extrasystole

D. atrial fibrillation

31. Severe hemodynamic disorders appear due to:

A. sinus tachycardia

B. sinus arrhythmia

C. extrasystole

D. ventricular fibrillation

32. Choose characteristic feature of venricular

fibrillation:

A. instead P wave many low voltage waves in ECG

B. appearance of ectopic focus

C. re-entry mechanism

D. attack of fast heart rate

33. Atrial Flutter is characterized by:

A. sudden onset and disappearance of increased heart

rate

B. pacemaker for heart is S-A node

C. pulse deficit

D. delayed impulse transmission


34. Complete A-V block is characterized by:

A. PR interval is more 0.2 sec

B. QRS intermittently dropped

C. P waves rate faster than QRS

D. QRS complex rate faster than P wave

35. Characteristic of sinus tachycardia is:

A. increased heart rate above 180 beats per minute

B. sympathetic nerves stimulation

C. vagal-acetylcholine effects

D. sudden onset and disappearance

36. Characteristics of paroxysmal tachycardia are:

A. increased heart rate but not above 180 beats per

minute

B. lasting for days

C. sudden onset and disappearance

D. pacemaker for heart is S-A node

37. Fall of ventricular pumping function is result of:

A. A-V complete block

B. ventricular fibrillation

C. atrial flutter

D. ventricular extrasystole

38. Ventricular extrasystole is result of premature

impulses from:

A. S-A node

B. A-V node
C. ectopic focus in atrium

D. ectopic focus in ventricle

39. Choose ECG characteristic features of ventricular

extrasystole:

A. P wave after QRS

B. P wave before QRS

C. P wave absence

D. PR interval is longer

40. Complete A-V block is characterized by:

A. atrial and ventricular pacemaker is the same

B. atrial and ventricular pacemaker is different

C. atrial pacemaker is ectopic focus

D. ventricular pacemaker is sinus node

41. Etiology of relative coronary insufficiency

A. infectious myocarditis

B. atherosclerosis of coronary arteries

C. thromboembolism of coronary arteries

D. hyperproduction of adrenaline

42. Injury of coronary arteries leads to:

A. primary arterial hypertension

B. myocardial infarction

C. heart hypertrophy

D. heart defects
43. Etiology of absolutely coronary insufficiency

A. oxygen deficiency in arterial blood

B. atherosclerosis of coronary arteries

C. stimulation of n. vagus

D. hyperproduction of glucocorticoids

44. Etiology of coronary myocardial necrosis

A. hormonal disturbances

B. effect of biogenic amines

C. effect of catecholamine

D. thrombosis of coronary arteries

45. Compensatory mechanism in myocardial ischemia

A. arrythmia

B. the occurrence of aneurysm

C. hyperproduction of glucocorticoids

D. intensification of collateral circulation

46. Consequences of acute myocardial ischemia

A. heart defects

B. pericarditis

C. cell necrosis

D. coarctation of aorta
47. Atherosclerosis is characterized by:

A. disease of vessel wall intima

B. disease of vessel wall media

C. disease of vessel wall endothelium

D. disease of vessel wall adventitia

48. Choose reversible injury in Myocardial infarction:

A. pycnosis,

B. karyorrhexis

C. cellular swelling

D. karryolysis

49. Manifestations of angina pectoris include:

A. sudden onset of severe “crushing” substernal chest

pain

B. pain felt within the chest area

C. Neutrophilic leukocytosis

D. ECG changes: wide, deep Q waves

50. Atherosclerosis is characterized by:

A. low level of triglycerides

B. high level of low-density lipoproteins

C. high level of high-density lipoproteins

D. low level of low-density lipoproteins


51. Choose irreversible injury in Myocardial

infarction:

A. cellular swelling

B. swelling of mitochondria

C. accumulation of Na+, Ca++

D. karryolysis

52. Manifestations of angina pectoris include:

A. Fever

B. ECG changes: P waves absence

C. episodic chest pain often exertional or emotional

D. Serum cardiac markers :CK-MB, LD, AST, cTnT,

cTnI

53. Characteristic sign of atherosclerotic-damaged

coronary arteries is:

A. increased arterial wall stiffness

B. reduced arterial wall stiffness

C. increased arterial lumen

D. increased arteria elasticity

54. Angina attack pathogenesis is characterized by:

A. local acidosis

B. cardiomyocytes death

C. liberation of lysosomal enzymes

D. local ischemia more than 20 min


55. Manifestations of Myocardial infarction include all

following signs EXCEPT:

A. sudden onset of severe “crushing” substernal chest

pain

B. Fever, Neutrophilic leukocytosis

C. ECG changes: wide, deep Q waves

D. episodic chest pain often exertional or emotional

56. Characteristic sign of atherosclerotic-damaged

coronary arteries is:

A. increased arterial lumen

B. increased synthesis of relaxing factors

(NO,prostacyclin)

C. decreased synthesis of relaxing factors

(NO,prostacyclin)

D. decreased synthesis of contracting factors (TXA2,

endothelin)

57. Angina attack pathogenesis is characterized by:

A. karryolysis

B. liberation of lysosomal enzymes

C. release of mediators: histamine, bradykinin,

prostaglandin

D. pycnosis

58. Etiology of right heart ventricular failure:

A. aortic insufficiency

B. mitral insufficiency

C. coarctation of aorta

D. pulmonary stenosis
59. Etiology of right heart ventricular failure:

A. aortic insufficiency

B. mitral insufficiency

C. coarctation of aorta

D. hypertension of the pulmonary circulation

60. Sign of right heart ventricular failure:

A. attack of a dyspnea

B. cyanosis, ascites

C. hemoptysis

D. pulmonary edema

61. Sign of right heart ventricular failure:

A. attack of a dyspnea

B. pulmonary edema

C. pallor

D. swelling of the lower extremities, ascites

62. Etiology of left heart ventricular failure:

A. lung diseases

B. tricuspid valve insufficiency

C. right ventricular infarction

D. mitral insufficiency
63. Etiology of left heart ventricular failure:

A. pulmonary hypertension

B. stenosis of the pulmonary artery

C. tricuspid valve insufficiency

D. primary arterial hypertension

64. Sign of left heart ventricular failure:

A. acrocyanosis

B. swelling of the lower extremities

C. throbbing neck veins

D. cardiac asthma

65. Sign of left heart ventricular failure:

A. ascites

B. swelling of the lower extremities

C. pulmonary edema

D. jugular venous distention

66. Etiology of overload form of heart failure:

A. hypervolemia

B. myocardial ischemia

C. myocarditis

D. extrasystoly

67. Etiology of overload form of heart failure:

A. myocarditis

B. anemia

C. systemic arterial hypertension

D. systemic arterial hypotension


68. Etiology of overload form of heart failure:

A. coronary insufficiency

B. pulmonary hypertension

C. myocardial dystrophy

D. avitaminosis

69. Preload of the heart may be due to:

A. hypervolemia

B. arterial hypertension

C. arterial hypotension

D. mitral stenosis

70. Afterload of the heart may be due to:

A. heart valves insufficiency

B. erythremia

C. arterial hypertension

D. exercise

71. Overload of the heart with resistance may be due

to:

A. coarctation of aorta

B. heart valves insufficiency

C. dehydration

D. hypervolemia

72. Etiology of overload form of heart failure:

A. hypovolemia

B. myocardial ischemia

C. myocarditis

D. mitral insufficiency
73. Long-term adaptation of cardiac function may be

due to:

A. tachycardia

B. myocardial hypertrophy

C. bradycardia

D. formation of connective tissue

74.What kind of changes takes place in compensation

stage of heart failure?

A. tonogenic dilatation

B. tachycardia

C. myocardial hypertrophy

D. myogenic dilatation

75. What is the primary change in hemodynamic

parameters in the development of heart failure?

A. decreased BP (blood preasure)

B. increased BP

C. decreased SV (stroke volume)

D. decreased CO (cardiac output)

76. Sign of emergency (extra) phase of cardiac

hyperfunction by Meerson:

A. hyperfunction of non-hypertrophy myocardium

B. hyperfunction of hypertrophy myocardium

C. myogenic dilatation

D. formation of connective tissue


77. Etiology of myocardial form of heart failure

A. heart defects

B. arterial hypertension

C. coarctation of aorta

D. myocardial infarction

78. Etiology of myocardial form of heart failure

A. hypertension of pulmonary circulation

B. hypertension of systemic circulation

C. aortic stenosis

D. myocarditis

79. Side effect of myocardial hypertrophy resulting

from valve defects is:

A. nucleus/cytoplasma ratio in cardiomyocytes is 1/5

B. nucleus/cytoplasma ratio in cardiomyocytes is 1/15

C. increased capillary/myocyte ratio

D. increased myocardial compliance during diastole

80. Diastolic heart failure means:

A. ability of heart chambers to fill with blood is

limited

B. heart chambers fill with too much blood

C. myogenic dilation

D. usually in eccentric hypertrophy


81. Choose characteristic sign of chronic congestive

right heart failure:

A. severe dyspnea

B. legs region edema

C. lungs interstitial edema

D. central cyanosis

82. Manifestation of congestive right heart failure is:

A. severe dyspnea

B. orthopnea

C. central cyanosis

D. hepatomegaly

83. Manifestation of congestive left heart failure is:

A. severe dyspnea

B. edema

C. acrocyanosis

D. hepatomegaly

84. Characteristic sign of heart valve insufficiency is:

A. incomplete valve closure

B. incomplete valve opening

C. cause concentric heart hypertrophy development

D. cause afterload of the heart chamber


85. Characteristic sign of heart valve stenosis is:

A. incomplete valve closure

B. cause eccentric heart hypertrophy development

C. cause afterload of the heart chamber

D. cause preload of the heart chamber


Examination questions of pathological physiology to
Blood system module for 3rd course students.
1. Choose characteristic sign of anemic syndrome:

A) Hypertension

B) Skin and mucosa pallor

C) Skin and mucosa redness

D) Hyperoxemia

2. The main characteristic sign of anemic syndrome is:

A) Increased number of RBCs

B) Increased amount of Hb

C) Hypoxemia

D) Hyperoxemia

3. The main factor determining clinical manifestation

of anemic syndrome is:

A) Decreased oxygen level in the blood

B) Decreased plasma protein level

C) Decrease heart blood supply

D) Decreased erythropoiesis

4. Pathological type of RBCs, which is related to

Anisocytosis include:

A) Sickle cells

B) Orthochromatic erythroblast

C) Macrocytes

D) Reticulocytes
5. Pathological type of RBCs, which is related to

Poikilocytosis include:

A) Spherocytes,

B) Target like cell

C) Megalocyte

D) Hypochromic cell

6. Pathological type of RBCs, which is related to

Anisochromia include:

A) Sickle cells,

B) Target like cell

C) Megalocyte

D) Spherocytes

7. Indicate etiologic factor of dyserythropoietic

anemia:

A) Hemorrhage

B) Genetic defect of Hb synthesis

C) Increased RBCs hemolysis

D) Suppression of BM by radiation

8. Development of dyserythropoietic anemia may be

due to:

A) Membranopathy

B) Hemoglobinopathy

C) Deficiency of iron

D) Rhesus (Rh) conflict


9. Usual consequence of acute posthemorrhagic

anemia is appearance of:

A) Poikilocytosis

B) Reticulocytosis

C) Anisocytosis

D) Megalocytosis

10. Choose pathological type of RBCs:

A) Orthochromatic erythroblast

B) Reticulocytes

C) Sickle cells

D) Proerythroblast

11. Reticulocytes are characterized by presence in

cytoplasm:

A) Pathological toxic granules

B) Heinz bodies

C) Large pallor center

D) Residual ribosomes and mitochondria

12. Increased erythropoietin synthesis is result of

hypoxic stimulation of:

A) Chemoreceptors of aortic arch

B) Chemoreceptors of carotid bodies

C) Kidney JGA cells

D) Liver cells
13. Regenerative anemia is characterized by:

A) Ineffective erythropoiesis

B) Decreased erythropoietin (EPO) production

C) Stimulation of special receptors of erythroblast by

EPO

D) Stimulation of special receptors of CFU-E stem cells by EPO

14. Hyporegenerative anemia is characterized by:

A) Decreased number of reticulocytes

B) Increased number of reticulocytes

C) Effective erythropoiesis

D) Stimulation of special receptors of CFU-E stem

cells by EPO

15. Megaloblastic erythropoiesis is:

A) Normal physiological process

B) Disorders of RBCs precursors iron uptake

C) Disorders of RBCs precursors division

D) Disorders of RBCs precursors maturation

16. Compensatory reaction to hypoxemia in anemic

syndrome is:

A) Dizziness

B) Thachycardia

C) Decreased blood pressure tendency

D) Skin and mucosa pallor


17. Choose etiologic factor to development of intrinsic

hemolytic anemia:

A) G6PD deficiency (favism)

B) "Warm" and "cold" autoantibodies against RBCs

C) Rhesus (Rh) conflict

D) Lead poisoning (plumbism)

18. Choose etiologic factor to development of extrinsic

hemolytic anemia:

A) Point mutation in Hb synthesis

B) G6PD deficiency (favism)

C) Lead poisoning (plumbism)

D) Α-globin synthesis defect

19. Indicate hemolytic anemia type, related to

membranopathy:

A) Hereditary spherocytosis (Minkowski–Chauffard

syndrome)

B) Thalassemia

C) Sickle-cell disease

D) G6PD deficiency anemia

20. Indicate hemolytic anemia type, related to

hemoglobinopathy:

A) G6PD deficiency anemia

B) Thalassemia

C) Autoimmune hemolytic anemia

D) Hereditary spherocytosis (Minkowski–Chauffard

syndrome)
21. Indicate hemolytic anemia type, related to

enzymopathy:

A) Sickle-cell disease

B) Plumbism

C) Favism

D) Thalassemia

22. What from these dates about iron balance is false?

A) Best absorbed forms of iron come from animal

products

B) Daily iron absorption in adulthood is 1-2 mg

C) For iron absorption IF is needed

D) Ferritin is body stores of iron

23. Iron deficiency causes:

A) Impairment of DNA synthesis

B) Impairment of Hb synthesis

C) Impairment of BM stem cells division

D) Impairment of myelin synthesis

24. Choose etiology of iron deficiency anemia, related

to loss of iron:

A) Uterine chronic hemorrhage

B) Pregnancy

C) Lack of dietary iron

D) Iron malabsorption
25. Choose etiology of iron deficiency anemia, related

to increased demand for iron:

A) Duodenitis

B) Hemorrhoids

C) Mother with breast-fed of infant

D) Nutritional iron deficiency

26. Indicate major mechanism of iron deficiency

anemia development:

A) Megaloblastic erythropoiesis

B) Disorder of cells maturation in erythropoiesis

C) Disorder of cells division in erythropoiesis

D) Increased hemoglobinization process in

erythropoiesis

27. Choose correct mechanism of iron deficiency

anemia development:

A) Increased synthesis of cytochromes

B) Increased synthesis of myoglobin

C) Ineffective erythropoiesis constitutes about 15%

D) Ineffective erythropoiesis constitutes about 30%

28. Indicate characteristic signs of peripheral blood

picture in iron deficiency anemia:

A) Macrocytic erythrocytes

B) Macrocytic ovalocytes

C) Cabot's rings

D) Large pale centers’ erythrocytes


29. Iron deficiency anemia is characterized by

appearance in peripheral blood of:

A) Hypochromic erythrocytes

B) Hyperchromic erythrocytes

C) Unstable Hb precipitates as Heinz bodies in the

center of cell

D) Normochromic erythrocytes

30. Characteristic features of iron deficiency anemia

are following EXCEPT:

A) Koilonychias

B) Defects in myelination

C) Lack of myoglobin

D) Increased epithelial desquamation

31. Choose etiologic factor for Pernicious (Addison-

Biermer) anemia development:

A) Gastrectomy

B) Diphyllobotriosis

C) Anti-intrinsic factor antibodies

D) Chronic hepatitis

32. Vitamin B12 deficiency causes impairment of:

A) Cytochromes synthesis

B) Hb synthesis

C) DNA synthesis

D) Myoglobin synthesis
33. Choose etiologic factor of B12 deficiency anemia,

which don't relate immune mechanism:

A) Atrophy of parietal cells

B) Destruction of parietal cells by autoantibodies

C) Anti-IF antibodies

D) Anti-IF receptors antibodies

34. Deficiency of vitamin B12 leads to:

A) Normoblastic erythropoiesis

B) Megaloblastic erythropoiesis

C) Stimulated erythropoiesis

D) Decreased hemohlobinization of erythropoiesis

related cells

35. Lack of adenosylcobalamin as metabolic form of

vitamin B12 may cause to:

A) Impairment of activation of folic acid

B) Impairment of DNA synthesis

C) Impairment of myelin synthesis

D) Atrophy of tongue's papillae

36. Lack of methylcobalamin as metabolic form of

vitamin B12 may cause to:

A) Accumulation of methylmalonic acid

B) Activation of folic acid

C) Decrease of tetrahydrofolate (THF) formation

D) Activation of DNA synthesis


37. Folic acid deficiency anemia in compare with B12

deficiency anemia cannot cause:

A) Megaloblastic erythropoiesis

B) Neurological symptoms

C) Gastrointestinal symptoms

D) Skin atrophy

38. Indicate characteristic signs of peripheral blood

picture in B12 deficiency anemia:

A) Large pale centers’ erythrocytes

B) Miacrocytic erythrocytes

C) Spherocytes

D) Erythrocytes with nuclear remnants

39. Choose etiologic factor of thrombocytopathy:

A) B12 deficiency

B) Radiation

C) Decreased synthesis of vWF

D) Splenomegaly

40. Choose etiologic factors of Henoch-Schonlein

perpura:

A) Cross-reactive IgA after streptococcal infection

B) Synthesis of vWF with pathological structure

C) Decreased platelet production

D) Vitamin C deficiency
41. What from these dates about Verlhov's disease is

false?

A) Primarily thrombocytopenia

B) Presence autoantibodies (Ig G)

C) Increased platelet destruction

D) Decreased platelet production – thrombocytopoiesis

42. Von Willebrand's disease (VWD) belongs to:

A) Primary thrombocytopenia

B) Primary thrombocytopathy

C) Vasopathy

D) Secondary coagulopathy

43. Determine etiologic factor for hemophilia B

development:

A) Deficiency of IXF, which is X-linked recessive trait

B) Deficiency of XIF, which is autosomal dominant

trait

C) Deficiency of VIIIF, which is X-linked recessive

trait

D) Defects in platelet glycoprotein receptors (GPIb)


44. Indicate etiologic factor for hemophilia A

development:

A) Deficiency of XIF, which is autosomal dominant

trait

B) Deficiency of IXF, which is X-linked recessive trait

C) Deficiency of VIIIF, which is X-linked recessive

trait

D) Defects in platelet glycoprotein receptors (GPIb)

45. Choose specific features, which characterized

hemophilia C:

A) Deficiency of IXF, which is X-linked recessive trait

B) Bleeding into joints (hemarthroses) and closed

Spaces

C) Appearance of symptoms in childhood in boys

D) Appearance of symptoms in adults (male and


female)

46. More characteristic sign of pathology related to


coagulation disorders is:
A) Petechia
B) Hemarthroses
C) Capillary hemoangiomas
D) Mucosal bleeding
47. Indicate more characteristic sign of hemorrhagic
syndrome related to vascular defects- vasopathy:
A) Petechia
B) Hemarthroses
C) Late rebleeding
D) Hematoma

48. Determine etiologic factor to secondary


coagulopathy:
A) Deficiency of vWF
B) Deficiency of Vitamin C
C) Deficiency of Vitamin K
D) Deficiency of VIIIF, which is X-linked recessive
trait

49. Choose platelet count which lead to purpura


appearance, but without bleeding development:
A) 200 000/mcL
B) 100 000/mcL
C) 60 000 /mcL
D) 20 000/mcL

50. Hemorrhagic syndrome related to vascular defects-


vasopathy is:
A) Osler-Weber-Rentu diseases
B) Verlhov's disease
C) Von Willebrand's disease
D) Hemophilia C
(DISSCUSSION) QUESTIONS
57. This ahythmia occurs when a reentrant” circuit is present, causing a repeated loop of electrical activity to
depolarize the atriums at a rate of about 250 to 350 beats per minute. This produces a characteristic
“sawtooth” patterm of the P waves. Which condition do you think about?
A. Ventricular fibrillation

B. Ventricular tachycardin
C. Atrial premature beats
D. Atrial futter
E. Atrial fibrillation

58. A 62-year-old patient was refered to outpatient hospital due to atrial fibrillation. Patient has undergone
ECG. Which ECG changings do you expects in this case?

A. no visible P waves and irregular narow QRS complexes


B. no visible P waves and irregular wide QRS complexes
C. visible P waves and irregular narow QRS complexes
D. visible P waves and irregular wide QRS complexes
E. irregular in timing and morphology ultra-rapid baseline waves

59. This arhythmia is presented as a fast. Abnomal heart rate. It starts in heart’s ventricles, and is defined as
3 or more heartbeats in a row, at a rate of more than 100 beats per minute. How is it called?
A. Sinus brndycardia
B. Atrial fhutter

E. Pulmonary embol

67. What is the nomal range of left ventricle ejection fraction?


A. 55%
B. 45-549%
C. 45-5496
D. 25-34%
E. 15-249%

68. The volume of blood the heart pumps through the circulatory system per minute is called?
A. Preload
B. Afterload
C. Ejection fraction
D. Stroke volume
E. Cardiac output

69. What is the auscultation point of aortic valve?


A. 2nd intercostal space, midclavicular line
B. 2nd intercostal space, right border of sternum
C. 2nd intercostal space, left border of sternum
D.4th intercostal space, midclavicular line

E. 5th intercostal space, midclavicular line

70. The most common symptom of left heart failure


A. Hepatomegaly
B. Qrthopnea, dyspnea

C. Ascites

81. A 56-year-old man complains of squeezing pain in the chest that occurs when walking after 200-00
meters. Pain reliefs at rest within 5 minutes, The symptoms are noted for several months. He went to his
family doctor. ECG was obtained at rest no changes detected. What dingnostic method will be the most
infomative for clarifying the diagnosis?
A. Lipid spectrum
B. Daily ECG monitoring
C. Echocardiograplly
D. Bicycle or treadmill test
E. Troponins

82. A 63-year-old womam complains on the chest pain developed at rest and lasted for 1.5 hours
accompanied by cold sweat, weakness. Nitroglycerin didn’t help to relief the pain. Patient history:
retrostemial chest pain wories her for about a year, which usually happens during moderate physical
exertion amd lasts for about 5 minutes. What is your preliminary diagnosis?
A. Unstable angina

B. Stable angina FC Iv
C. Acute myocardial infarction
D. New-onset angina pectoris
E. Vasospastic angina

83. A 52-year-old man admitted to the intensive care unit because of squeezing chest pain radiating to the
left am, interscapular space. The pain has started 1.5 hours ago. Examination: BP-120/70 mm HR., HR 88
beats per min. ECG: ST elevation in V1-V4 and ST depression in

B.Myocardial infarction
93. 36 years old male admitted to the hospital. After hypothermia, the body temperatıre increased to 38.7
C. accompanied by chills and sweat. Body temperature returmed to nomal for a short time, then increased
again to high numbers. When examining the heart, the picture of combined mitral defect. Respiratory rate
(RR) 22 per minute. Heart mte (HR) 90 per minute blood pressure (BP) 110/70 mm Hg. The liver protrudes 3
cm from the edge of the costal arch. What is your preliminary diagnosis?
A. Syndrome of endocardium inflammation
B. Syndrome of pericardium inflammation
C. Syndrome of myocardium inflammation
D. Syndrome of mitral valvular diseases
E. Syndrome of acute coronary insufficiency

94. 38 years old female admitted to the hospital. After hyypothermia, the body temperature increased to
38.7 C, accompanied by chills and sweat. Body temperature retumed to nomal for a short time. Then
increased again to high numbers. When examining the heat, the picture of mitral regurgitation detected.
Respiatory rate (RR)- 22 per minute. Heart rate (HR) 90 per minute, blood pressure (BP) 110/70 mm Hg.
Which laboratory test is needed to confirm the Syndrome?

A.Blood culture
101. 35 years old male admitted to the hostital with complaints of dyspnea, fatigue, palpitations. On cardiac
examinatiou there is a lateral displacement of the apical impulse, quiet $1, holosystolic murmur. What is
your preliminary dingnosis?
A. Mitral regurgitation

B. Mitral stenosis
C. Aortic regurgitation
D. Aortic stenosis
E. Tricuspid stenosis

102. 34-year-old male admitted to playsician with complaints for the chest pain, noctumal dyspuea. fatigue.
Ou the auscultation: loud S1 and the opening suap after S2. Which of the following diagnostic tools is the
most infomative to confim the diagnosis?

A. ECG
B. Chest X-ray
C. Spirometry
D. Pulseoximetry
E. Echocardiography

103. Patient with symptoms of exertional dyspnea, fatigue, palpitatious has undergone ECG and P mitrale,
LV hypertrophy, atrial fibrillation has been revenled. On echocardiography: LV and LA hypertrophy. What is
your diagnosis?

A. Mitral regurgitation
B. Mitral stenosis
C. Aortic regurgitation

121. A 56-year old man admitted to the hospital with complaints of squeezing pain behind the sternum for
more than 1 hour. Patient is pale and anxious. Examination. BP-125/85 mmHg, HR - 82 bpm. ECG showed ST
segment elevation in V1-V4,Which laboratory test may help you to confirm diagnosis?

A.CRP
B. ESR
C.ALT, AST

D. Troponins
E. Creatinine

122. 62 years old male complains of retrosternal chest pain, radiating to the left am, jaw. Pain develops after
200-300 meters of walking or at rest and lasts 3-5 minutes. BP -130/70 mm Hg. HR-82 beats per min. Which
diagnostic method is necessary to confim diagnosis?
A. 24-hour ECG motnitor
B. 24-hour blood pressure monitor

C. Stress ECG test


D. Coronary angiogrnphy
E Chest X-ay

123. A 62-year-old woman admitted to the hospital through the ambulance with complaints of retrosternal
chest pain which lasts more than 1 hour. Patient is diaphoretic. Examination: Pale skin. BP125/85 mmHg.
HR82 bpm. ECG showed ST segment elevation in II, III, avF Which laboratory test may help you to confirm
diagnosis?
A. ALT, AST
B. Troponin I/T

C. ESR
D. CBC
E. Creatinine CRP

130. 50-year-old male complains of shortness of breath, swelling of the cervical veins, enlarged liver, ascites,
edema of the lower extremities. He was previously treated for tuberculosis. Heart Rate -96 per minute Heart
sounds are muffled. BP 100/80 mm Hg. ECG a decrease in Voltage. What is your preliminary diagnosis

A. Syndrome of pericardium inflammation


B. Syndrome of myocardium inflammation
C. Syndrome of endocardium inflammation
D. Syndrome of mitral valvular diseases
E. Syndrome of acute coronary insufficiency

131. A 25-year-old male was admitted to the hospital because of pericardium inflammation syndrome. She
said that she fell ill 2 weeks ago, after a cold (shortness of breath, weakness, chest tightness appeared).
What examinations need to be administered?

A. Coronary angiography
B. Ultrasound of the liver
C. ECG and echocardiography
D. Chest x-ray
E. Pleural cavities ultrasound

132. 46-year-old male was admitted to the hospital with pericardial inflammation syndrome. He associates
his illness with hypothermia. Examination: fever, chills, sweating. Pain in the heart associated with the
breathing and changes of body position, pericardial friction rub, which increases during inspiration. What
examinations need to be administered?
A. Cardiac enzymes
B. Thyroid hormones
C. Analysis of urine
D. Total protein in the blood
E. Albumin in the blood
138. Patient with history rheumatic fever complaints for dyspnea, orthopnea, hemoptysis. ECG has detected
atrial fibrillation, left atrial enlargement. On Echo mitral valve area 0.8. cm2 What is your preliminary
diagnosis?
A. Mild mitral stenosis
B. Moderate mitral stenosis
C. Severe mitral stenosis

D. Mild mitral insufficiency


E. Moderate mitral insufficiency

146. 17. In a patient with complaints of dyspnea for differential diagnosis between chronic heart failure and
other altemative causes of dyspnea, an analysis should be conducted for:
A. Detemination of natriuretic peptide

B. Detemination of catecholamines
C. Detemination of blood serum glucose
D. Detemination of serum electrolytes
E. Determination of the lipid spectrum

147. Ambulatory patient K. with anamnesis of rheumatic fever has symptoms like dyspnea, orthopnea,
hemoptysis. On examination mitral facies, loud first heart sound, opening snap of mitral valve have been
detected. What diagnostic method would you do to confim your preliminary diagnosis;
A. Chest x-ray
B. Angiography
C. ECG
D. Echocardiography

E. MRI

148. In a patient with complaints of dyspnea, edema, decreased tolerance to physical activity. On
examination jugular vein distention, murmurs. On Echo we determine akynesia of inferior wall of left
ventricle. What was the disease that caused heart failure:
A. Mitral regurgitation

B. Mitral stenosis
C. Myocardial infarction
D. Aortic insufficiency
E. Aortic stenosis

150. 38 years old female was transferred to the cardiology department of the infectious diseases hospital,
where she stayed for 12 days due to enterovirus infection. Due to the appearance of changes in the ECG, she
was transferred to the cardiology department. CBC: leukocytes 10.4x10 9/L, ESR 22 mm/h. Biochemical
blood test: total protein 70 g1, albumin 59% globulins: al – 3.9%, a2-10.3%, p- 10.5%%, Y- 16.3%, creatine -
88 umol1, total bilirubin- 14.34umoll, fibrinogen-4 g1; CRP – increased. ALT and cardiac enzymes are not
increased. ECG: sinus rhythm. Heart rate 100 beats per min. Sinus tachycardia. Slightly negative T in all
precordial leads. What is your preliminary diagnosis
A. Syndrome of inflammation pericardium
B. Syndrome of inflammation myocardium
C. Syndrome of inflammation endocardium

D. Syndrome of mitral valvular diseases


E. Syndrome of acute coronary insufficiency

77. The alternątion of strong and weak pulses caused by alterations in the stroke volume called?

A. Pulsus differens
B. Pulsus paradoxus
C. Pulsus deficiens
D. Pulsus parvus
E. Pulsus alternans

78. According to NYHA classification of heart failure, which of the following is associated wit no limitation of
physical activity?
A. Heart failure Class IV
B. Heart failure Class II
C. Heat failure Class II with preserved ejection fraction

D. Heart failure Class II with reduced ejection fraction


E. Heart failure Class I

79. The abnormal accumulation of fluid in the abdomen is called?


A.Ascits
85.65 years old male, complaints of headaches, dizziness. Risk factors: heavy smoker, diabetes positive
fanmily history. Examination: blood pressure 140/90 mm Hg. What degree of arterial hypertension would
you put?
A. I degree arterial hypertension
B. II degree arterial hypertension

C. III degree arterial hypertension


D. Isolated systolic hypertension
E. Prehypertension

86. 52 years old female complains on headaches, tinnitus. Anamnesis: she has been sufferin from diabetes
about for five years. Risk factors: heavy smoker, abdominal obesity, dyslipidemi Examination: blood pressure
150/90 mm. Hg. What is the risk of cardiovascular disease development in this patient?

A. no risk
B. mild
C. moderate
D. severe
E. high severe

87. 50 years old male complains on headaches, tinnitus. Risk factors: smokes about 10 years, overweight.
Examination: blood pressure 170/100 mm Hg. What degree of arterial hypertension will you diagnose? (ESC
classification)

A. I degree arterial hypertension


B. II degree arterial hypertension
C. III degree arterial hypertension
D. Isolated systolic hypertension
E. Prehypertension

88. According to NYHA classification of heart failure, which of the following is characterized
by appearance.of symptoms at rest?

E.ClassIV
89. 60 years old male complains on headaches, dizziness, nausea. Risk factors: smoking, age, increased
weight. Examination: Blood pressure 180/110 mm Hg. What degree of arterial hypertension do you put?
(ESC classification)
A.I
B.II

C. III
D. Isolated systolic hypertension
E. Prehypertension
90. A patient 66 years old was admitted to the hospital with complaints of headaches, dizziness. Memory
loss. History: Coronary heart disease. Stable angina was diagnosed 5 years ago. Risk factors: diabetes
mellitus, age, heredity. Examination: BMI 32 kg/ m2. Heart rate 76 per minute. Blood pressure 150/100 mm
Hg. What is your preliminary diagnosis?
A. I degree arterial hypertension, high risk
B. I degree arterial hypertension, very high risk
C. II degree arterial hypertension, high risk
D. II degree arterial hypertension, very high risk
E. III degree arterial hypertension, high risk

91. 58 years old female, complaints of headaches, tinnitus. Risk factors: smoker for 14 years, overweight,
heredity. Examination: blood pressure 140/90 mm Hg. What degree of arterial hypertension do you put?
(ESC classification)
A.I
IMPORTANT POINTS
Typical pain in angina - squeezing
typical localization of angina - Retrosternal
the typical duration of pain in coronary insufficiency – 3-5 minutes
if pain is more than 30 minutes it is - Myocardial infarction
Myocardial infarction on Echocardiography – akynesia/hypokinesia of walls
pathological (deep or wide) Q wave at ECG - Myocardial Necrosis
ECG sign that represents myocardial ischemia - ST-segment depression
Ig ECG at rest is normal to confirm diagnosis we should do - Treadmill test / Stress test
most informative instrumental diagnostics method for coronary insufficiency – angiography
the most informative laboratory diagnostics method for determining necrotic changes (in
myocardial infarction) – troponins (cardiac enzymes/biomarkers)
Holter monitoring defines - Rhythm disturbances
ST segment depression on ECG:
in I, aVL leads – high lateral
in II, III, aVF – inferior wall

optimal blood pressure <120/80 mm hg


normal blood pressure 120-129/80-84 mm hg
high normal blood pressure (prehypertension) 130-139/85-89 mm hg
hypertension grade 1 140-159/90-99 mm hg
hypertension grade 2 160-179/100-109 mm hg
hypertension grade 3 >180/110 mm hg
isolated hypertension >180/<90 mm hg
In hypertension if in anamnesis there is other disease like Angina or Diabeties – it is High severe
risk or Very high risk (of cardiovascular complications)
etiology of arterial hypertension – multifactoral or unknown
duration of treatment of hypertension - lifelong
target organs of arterial hypertension - Heart, kidneys, brain, eyes and aorta
ejection fraction normally - >55%
volume of blood the heart pumps through in 1 minute is called - Cardiac output
diagnosics of hypertension - 24-hour blood presure monitoring

Auscultation points :
Mitral valve – 5th intercostal space
Aortic valve – 2nd intercosal space right border of sternum
pulmonary valve – 2nd intercosal space left border of sternum
tricuspid valve – 4th intercostal space near the xiphoid process
(additional for aortic valve) Erb’s auscultation point - 3rd intercostal space, left sternal border

Normal area of mitral valve (bicuspid valve) – 4-6 cm2


Normal area of aortic valve – 2.5-3.5 cm2
De Musset's sign (rhythmic nodding of the head) - Aortic regurgitation
holosystolic murmur best heard at the apex – mitral regurgitation

normal left ventricular end-diastolic pressure? - 6-12 mm Hg

for diagnostics of valve diseases most informative is - Echocardiography

Single ventricular impulses caused by reentry mechanism within the ventricle or abnormal
automaticity of ventricular cells. They may present in both healthy patients and patients with a
heart disorder; it may be asymptomatic or cause palpitations. Diagnosed by
electrocardiography. What is it? - Ventricular extrasystoles
A patient with atrial fibrillation has high probability of development of - heart failure

prolonged PR interval on ECG. - Atrioventricular block I degree


at ECG: progressive lengthening of the PR interval until a beat is dropped. Select correct
answer: - AV block, Mobitz type I/Wenckebach
On the ECG: irregular dropped beats. - AV block, Mobitz type II/Wenckebach
This condition can be recognized as a Complete block: at ECG - AV dissociation: no
relationship between P waves and QRS complexes. - Atrioventricular block, third-degree block

NYHA classification of heart failure:


Class 1 – no limitation in physical activity
Class 2 – slightly limitation
Class 3 – significant limitation
Class 4 – symptoms appear at the rest

For endocardium inflammation : Fever >38C


For diagnostic – blood culture

myocardium inflammation syndrome mostly - in young men , etiology frequently - bacteria


In which population the fulminant course of myocardium inflammation syndrome is more
common - in newborns and children;

For diagnostic of myocardium inflammation - byopsy


For pericardium inflammation - QRS complex is reduced, negative T wave in I, II, III, V2-V6
leads, friction rub, edema, ascities
Pericardial friction rub in pericardial inflammation syndrome - Is better heard in a sitting
position
for differential diagnosis between chronic heart failure and other alternative causes of
dyspnea - Determination of natriuretic peptide
Propaedeutics Internal Medicine 3rd year exam tests 2019-20

1.Respiratory system – 66
2.Cardiovascular system – 150
3.Digestive system – 120
4.Endocrine System - 81
5.Urinary System - 59
6.Hematopoietic system – 71
7.Skeletal muscle system – 100
TOTAL - 647

RESPIRATORY SYSTEM

1. Indicate the most characteristic changes in the chest during inflammatory


compaction of the lobe of the lung:
A. decrease in half of the chest, its retraction and lag in breathing
B. lag in breathing, an increase in half of the chest and smoothing of the intercostal
spaces
C. only half of the chest is involved in breathing
D. hyperstenotic chest
E. an increase in the anteroposterior and transverse dimensions of the chest,
retraction of the intercostal spaces in the lower lateral parts on both sides

2. What percussion sound appears in the inflammatory compaction of lung tissue?


A. blunt or dull
B. clear pulmonary
C. tympanic
D. dullness with a tympanic note
E. boxed

3. For what purpose coughing is used as an additional technique for auscultation of


the lungs?
A. to distinguish pleural friction rub from crepitus and wheezing
B. to detect latent bronchial obstruction
C. in order to distinguish between dry wheezing and wet wheezing
D. to distinguish wheezing from crackles
E. for better listening to pathological bronchial breathing

4. What is the most common respiratory sound heard in lobar inflammatory


compaction?
A. impaired vesicular breathing
B. amphoric breathing
C. bronchial breathing
D. hard breathing
E. mixed bronchovascular breathing

5. What is the cause of crepitation?


A. the presence in the alveoli (parietal) of a small amount of exudate or
transudate
B. inflammation of the pleura
C. alveoli completely filled with exudate or transudate
D. viscous bronchial secretion in large bronchi
E. viscous bronchial secretion in the small bronchi and / or spasm

6. Crackesoccure in:
A. alveoli
B. small bronchial tubes
C. middle bronchi.
D. large bronchi.
E. bronchioles.

7. What kind of sputum in patients with focal pulmonary compaction syndrome?


A. Vitreous
B. Mucopurulent
C. Three-layer
D. "rusty"
E. Serous, foamy pink.

8. What is observed during an X-ray examination of the lungs with lung tissue
compaction syndrome?
A. Lobar dimming of lung tissue.
B. Increased transparency of the pulmonary fields
C. Lobular enlightenment of the tissue.
D. The presence of a round focus of enlightenment with fluid level.
E. Dimming along the interlobar sulcus.

9. What is the difference between crackles and moist bubbly rales?


A. occur on expiration;
B. occur on inspiration;
C. occur during inhalation and exhalation;
D. no difference
E. disappear after inspiration

10. Select the symptoms of the inflammatory compaction syndrome of the lung
tissue:
A. chest pain radiating to the left arm
B. cough with sputum discharge "full mouth", mainly in the morning
C. fever, chills, cough with "rust-colored" sputum
D. asthma attacks with distant wheezing
E. Leg swelling, enlarged liver

11. Indicate the most typical changes in the chest in obstructive atelectasis:
A. half of the chest decreased, is retracted and lag in breathing
B. lag in breathing, half of the chest increased and smoothing of the intercostal
spaces
C. Only lag in breathing of the half of the chest
D. hyperstenotic chest
E. an increase in the anteroposterior and transverse dimensions of the chest,
retraction of the intercostal spaces in the lower lateral parts on both sides

12. What is the main respiratory noise most often heard in the presence of a cavity
connected to the bronchus?
A. impaired vesicular breathing
B. bronchial breathing
C. Amphoric breathing
D. hard breathing
E. mixed bronchovascular breathing

13. Increased vocal fremitus, tympanic sound, bronchial, sometimes amphoric


breathing, an abundance of moist sonorous rales in a limited area, profuse purulent
sputum is observed in the syndrome:
A. increased airiness of the lungs
B. lung tissue compaction
C. fluid in the pleural cavity
D. air in the pleural cavity
E. presence of the lung cavity
14. A macroscopic examination of sputum in a syndrome of the cavity of the lung
tissue:
A. Profuse, excreted with full mouth, with an unpleasant odor with an
admixture of blood, purulent, has two layers.
B. It is allocated in a large number no in the evening, has mucopurulent character.
C. Lean, viscous.
D. "Rusty", odorless mucopurulent.
E. Mucopurulent in a small amount without impurity of blood.

15. What is the cause of the wet coarse wheezing?


A. viscous bronchial secretion in large bronchi
B. viscous bronchial secretion in the small bronchi and / or their spasm
C. liquid bronchial secretion in large bronchi or cavities communicating with
the bronchus
D. liquid bronchial secretion in the small bronchi with preserved airiness of the
surrounding lung tissue
E. liquid bronchial secretion in the small bronchi and inflammatory compaction of
the surrounding lung tissue

16. Which of the sputum elements reliably indicates the destruction of lung tissue?
A. Charcot-Leiden crystals;
B. white blood cells;
C. Kurshman spirals;
D. elastic fibers;
E. red blood cells.

17. Characterize wet wheezing:


A. is heard on inhalation and exhalation, crackling or “bursting of bubbles”,
     disappears when simulating respiratory movements
B. is heard on inspiration, the sound "f", above all pulmonary fields,
     disappears when simulating respiratory movements
C. is heard on the inhale and exhale over the larynx, the sound is "ha ha,
    disappears when simulating respiratory movements
D. is heard on the inhale and exhale above the pulmonary fields, the sound "f",
     disappears when simulating respiratory movements
E. is heard only on inspiration, crackling or creaking of snow, disappears on
inspiration

18. The most informative method for the presence of a cavity in the lung:
A. blood test
B. Sputum analysis
C. spirometry
D. radiography
E. bronchophony

19. What is typical for the syndrome of bronchiectasis?


A. caverns
B. tumors
C. pus in dilated bronchi
D. fluid in the pleural cavity
E. Thin-walled closed cavities in the lung

20. What kind of shortness of breath is most typical for spasms of the small
bronchi?
A. stridor respiration
B. expiratory dyspnea
C. Kussmaul breath or Chayne-Stokes breathing
D. Chayne-Stokes breathing or Biot’s respiration
E. inspiratory dyspnea

21. What is causing the appearance of dry buzzing (bass) rales?


A. the presence in the alveoli (parietal) of a small amount of exudate or transudate
B. inflammation of the pleura (constrictive pleurisy)
C. Alveoli completely filled with exudate or transudate
D. viscous bronchial secretion in large bronchi
E. Viscous bronchial secretion in the small bronchi and / or spasm

22. For what purpose is used forced expiration as an additional method for
auscultation of the lungs?
A. In order to distinguish pleural friction noise from crackles and wheezing
B. to detect latent bronchial obstruction
C. in order to distinguish between dry wheezing and wet wheezing
D. to distinguish wheezing from crepitus or pleural friction noise
E. for better listening to pathological bronchial breathing

23. For the syndrome of bronchial obstruction, the most characteristic auscultation
signs are:
A. bronchial respiration and crackles
B. prolonged expiration breathing, the appearance or intensification of
wheezing during forced expiration
C. Vesicular respiration and wet rales
D. impaired vesicular breathing
E. Bronchovascular breathing with prolonged inhalation

24. The functional sign of bronchial obstruction is:


A. Tidal volume change
B. Tiffno Index decrease
C. Decreased FVC
D. increased residual volume
E. increased PEFR

25. In what phase of breathing are wheezing heard?


A. on inhalation;
B. at the height of inhalation;
C. on exhalation;
D. on inhalation and exhalation.
E. end of exhalation

26. What is gasping?


A. subjective feeling of lack of air, accompanied by a change in the frequency,
depth and rhythm of breathing
C. an arbitrary or involuntary reflex act of protecting and self-cleaning the
respiratory tract from foreign bodies, mucus, pus and sputum
C. secretion of blood in sputum
D. Suddenly onset of lack of air
E. A painful barking cough

27. Which type of cough features in bronchial obstruction syndrome?


A. with mucus or purulent sputum discharge in the morning
B. simultaneous discharge of purulent sputum in large quantities
C. A painful cough with sputum
D. Cough with rusty sputum
E. Cough with foamy pink sputum

28. What syndrome is characterized by mucous vitreous sputum with the detection
of eosinophils, Kurshman spirals and Charcot-Leiden crystals in them?
A. pleural inflammation syndrome
B. lung compaction syndrome
C. cavity syndrome in lung tissue
D. pleural cavity syndrome
E. bronchial obstruction syndrome
Syndrome of increased airiness of the lung tissue:

29. What percussion sound appears in emphysema?


A. blunt or dull
B. clear pulmonary
C. tympanic
D. dullness with a tympanic hue
E. boxed

30. Indicate the most typical changes in the chest in emphysema:


A. Decrease in half of the chest, its retraction and lag in breathing
B. lag in breathing, an increase in half of the chest and smoothing of the intercostal
spaces
C. Only half breath in the chest
D. hypersthenic chest
E. an increased dimension of the chest, shortening of the intercostal spaces in
the lower lateral sections on both sides in inspiration.

31. The leading complaint of patients with the syndrome of increased airiness of
the lung tissue is:
A. Cough
B. Dyspnea
C. Chest pain
D. Hemoptysis
E. Fever

32. When vocal fremitus is weakened?


A. In a normal condition
B. during increased airiness of the lung tissue
C. during compression atelectasis
D. above the cavity
E. during lung infiltration
33. What changes in lung structure are associated with the development of
pulmonary emphysema?
A. Overstretching of elastic fibers in the interstitium of the lung
B. Change in mucociliary index
C. Blood overflow of the cavernous venous plexus of the lower nasal passage
D. Spasm of the muscle plate of the mucous membrane of the small bronchi
E. Impaired Surfactant Synthesis

34. What is dyspnea?


A. subjective feeling of lack of air, accompanied by a change in the frequency,
depth and rhythm of breathing
C. an arbitrary or involuntary reflex act of protecting and self-cleaning the
respiratory tract from foreign bodies, mucus, pus and sputum
C. secretion of blood in sputum
D. Suddenly onset of lack of air
E. sudden loss of consciousness

35. During comparative lung percussion, the doctor determined the box sound
above both lungs, the lower border of the lungs is lowered by 3 cm, the active
mobility of the pulmonary edges is limited. What syndrome are we talking about?
A. pleural inflammation syndrome
B. lung compaction syndrome
C. cavity syndrome in lung tissue
D. pleural cavity syndrome
E. Syndrome of increased airiness of the lung tissue

36. X-ray signs of the syndrome of increased airiness of the lung tissue:
A. homogeneous infiltration
B. increased transparency of lung tissue
C. Inhomogeneous infiltration
D. cloudy infiltrates
E. ring-shaped shadows with horizontal liquid level

37. What causes predisposition to the development of increased airiness of the lung
tissue?
A. α-1-antitrypsin deficiency
B. excess α-1-antitrypsin
C. excess collagenesis
D. deficiency of creatine phosphokinase
E. excess phospholipase

38. What causes the appearance of pleural friction rub?


A. the presence in the alveoli (parietal) of a small amount of exudate or transudate
B. inflammation of the pleura
C. Alveoli completely filled with exudate or transudate
D. Viscous sputum in large bronchi
E. Viscous sputum in the small bronchi and / or spasm

39. Indicate the most typical changes in the chest in pneumothorax:


A. Decrease in half of the chest, its retraction and lag in breathing
B. lag in breathing, an increase in half of the chest and smoothing of the
intercostal spaces
C. Only half breath is involved in the act of breathing
D. hyperstenotic chest
E. an increase in the anteroposterior and transverse dimensions of the chest,
retraction of the intercostal spaces in the lower lateral parts on both sides

40. What is the most common respiratory sound most often heard in
pneumothorax?
A. impaired vesicular breathing
B. amphoric breathing
C. bronchial breathing
D. hard breathing
E. mixed bronchovascular breathing

41. Indicate the most characteristic changes in the chest with unilateral
hydrothorax:
A. Decrease in half of the chest, its retraction and lag in breathing
B. lag in breathing, an increase in half of the chest and smoothing of the
intercostal spaces
C. Only half breath is involved in the act of breathing
D. hyperstenotic chest
E. an increase in the anteroposterior and transverse dimensions of the chest,
retraction of the intercostal spaces in the lower lateral parts on both sides
42. Patient N., was referred to outpatient hospital with preliminary diagnosis:
“Syndrome of air in the pleural cavity;Pneumothorax”.What do you think, which
percussion sound did the doctor listen during percussion of patient`s thorax?
A. blunt or blunt
B. clear pulmonary
C. tympanic
D. dullness with a tympanic hue
E. boxed

43. Patient M., was admitted to local hospital. Doctor inspected him, performed
auscultation, and put preliminary diagnosis: “Syndrome of fluid in the pleural
cavity;Hydrothorax”.What do you think, whichmain respiratory soundlistened by
physician during auscultation of patient`s thorax, did help to doctor to suspect
correct diagnosis?
A. impaired vesicular breathing
B. amphoric breathing
C. bronchial breathing
D. hard breathing
E. mixed bronchovascular breathing

44.Patient E., was referred to outpatient hospital with preliminary diagnosis:


“syndrome of fluid in the pleural cavity, hydrothorax”.What do you think, which
percussion sound did the doctor listen during percussion of patient`s thorax?
A. blunt or dull
B. clear pulmonary
C. tympanic
D. dullness with a tympanic hue
E. boxed

45. What is the difference between pleural friction rub and fine bubbling rales?
A. by nothing;
B. heard only on inspiration;
C. heard only on exhalation;
D. does not change when coughing;
E. changes when coughing.

46. Evaluate the result of the study of pleural contents: relative density - 1027,
turbid liquid, greenish-yellow color, protein - 60 g / l, Rivalt test +++, sediment
microscopy - neutrophils cover all fields of view:
A. transudate;
B. purulent exudate;
C. hemorrhagic exudate;
D. serous exudate
E. chylous exudate

47. Patient O., was referred to outpatient hospital with preliminary diagnosis:
“syndrome of fluid in the pleural cavity”. Please select appropriate signs at
patient`s chest X-ray film:
A. homogeneous dimming of the lower part of the lung with the
Damoiseau line.
B. increased transparency of the pulmonary fields.
C. bright pulmonary field without pulmonary pattern.
D. spotted dimming of the pulmonary field.
E. dimming the approach of the interlobar sulcus in the form of a spindle.

48. In a syndrome of fluid accumulation in the pleural cavity above the fluid
accumulation area, the following is noted:
A. tympanic sound.
B. boxed sound.
C. pulmonary sound.
D. tympanic sound with a metallic touch
E. dull sound

49. X-ray signs of the syndrome of air in the pleural cavity:


A. homogeneous dimming of the lower part of the lung.
B. increased transparency of the pulmonary fields.
C. bright pulmonary field without pulmonary pattern.
D. spotted dimming of the pulmonary pattern.
E. dimming along the interlobar pleura in the form of a spindle.

50. In which direction does the mediastinum move with the syndrome of fluid in
the pleural cavity:
A. to the healthy side
B. to the sick side
C. does not move anywhere
D. moves down
E. moves up
51. In which direction does the mediastinum move with the syndrome of air in the
pleural cavity:
A. to the healthy side
B. to the sick side
C. does not move anywhere
D. moves down
E. moves up

52. A characteristic change in blood composition in respiratory failure is:


A. decrease in red blood cells and hemoglobin
B. increase in residual nitrogen
C. Hypoxemia and hypercapnia
D. hyperlipidemia
E. hypoproteinemia

53. Obstructive type of respiratory failure syndrome is observed in:


A. Bronchospasm
B. Hydrothorax
C. Pneumosclerosis
D. Pneumothorax
E. Kyphoscoliosis

54. Patient N., was admitted to department of Pulmonology with diagnose:


“Respiratory Failure”. Please select blood test`s laboratory signs (mentioned
below), which could serve as a compensatory mechanism of respiratory failure:
A. lymphocytosis
B. thrombocytosis
C. erythrocytosis
D. eosinophilia
E. leukopenia

55. Patient K., has underwent clinical and instrumental investigation. The III-
degree of respiratory failure has been confirmed. Please select the level of SpO2 in
this clinical case:
A.> 90%
B. 90-94%
C. 75-89%
D. <75%
E. <60%

56. In case of II-degree respiratory failure the partial tension of oxygen (PaO2) in
arterial blood is:
A. 60-79mmHg Art.
B.> 80 mmHg Art.
C. 40-59 mm Hg Art.
D. <40 mmHg Art.
E. 20-39 mm RT. Art.

57. Patient O., was referred to department of Pulmonology with diagnose:


“Hypoxemic form of Respiratory Failure”. What do you think, which leading
pathogenic link took part in development of the hypoxemic form of respiratory
failure?
A. Diseases that directly affect the pulmonary parenchyma
B. Mechanical defects of the muscular and rib cage of the chest
C. Abdominal obesity
D. Violation of the regulatory functions of the respiratory center
E. Obstruction of the bronchi

58. Violation of external respiration is evidenced by a symptom detected during a


general examination, which is…
A. Chest changes
B. Pallor of the skin
C. Cyanosis
D. Facial hyperemia
E. Erythematous rash

59. Patient N., 46 y.o., worker at the factory, need full examination to rule out
respiratory failure. Upon conversation doctor gathered complaints and
suspectedthe first degree of chronic respiratory failure. What did the patient
complain?
A. Lack of shortness of breath during exercise
B. Shortness of breath with significant physical exertion
C. Shortness of breath with slight physical exertion and the inclusion of
compensatory mechanisms at rest
D. Dyspnea and cyanosis at rest
E. Shortness of breath and cyanosis in a dream
60. Indicate the point of the best auscultation of the pulmonary valve:
A. second intercostal space to the left
B. second intercostal space to the right
C. fifth intercostal space to the right
D. at the base of the xiphoid of the sternum
E. fifth intercostal space to the left

61. How will II heart tone change with increased pressure in the pulmonary
artery and severe hypertrophy of the right ventricle?
62. weakening of II tone in the pulmonary artery
B. accent of II tone on the pulmonary artery
C. accent and splitting of the II tone on the pulmonary artery
D. only splitting of II tone on the pulmonary artery
E. strong I tone over the pulmonary artery

63. A complication of the cardiovascular system that develops as a result of


chronic respiratory failure is:
A. arterial hypertension
B. hypertrophy of the right ventricle
C. arterial hypotension
D. Left ventricular hypertrophy
E. Dilation of the left atrium

64. A characteristic radiological sign of a pulmonary heart is:


A. expansion of the heart to the right because of the RV
B. expansion of the heart to the left due to LV
C. Mediastenal displacement to the right
D. displacement of the mediastinum to the left
E. thickening of the pulmonary pattern

65. Patient L. admitted to internal medicine department of hospital N. Doctor


suspects pulmonary hypertension. Which instrumental method would you
prescribe at this clinical case to help doctor in confirming pulmonary
hypertension?
A. Radiography
B. Fluorography
C. ECG
D. Echocardiography
E. Spirography

66. Patient M. applied to outpatient hospital for checkup his respiratory system.
Patient underwent full investigation, and functional tests revealed normal level
of pulmonary artery pressure. What do you think, which values of patient`s
pulmonary artery pressure had been obtained?
A. 25-30mmHg
B. 50-60 mmHg
C. 100-200mm water column
D. 25-30mm water column
E. 70-80 mmHg

EXAMINATION TESTS "THERAPY" CVS

1. Select correct interpretation of palpation – enlarged high apex beat at VI


intercostal space 2 cm lateral from midclavicular line:
a. hypertrophy and dilatation of left ventricle (LV)
b. hypertrophy and dilatation of right ventricle (RV)
c. hypertrophy of left atrium (LA)
d. hypertrophy of right atrium (RA)
e. hypertrophy of left and right atrium (LA and RA)

2. Choose correct interpretation of palpation – significant cardiac impulse and


epigastric pulsation:
a. hypertrophy and dilatation of left ventricle
b. hypertrophy and dilatation of right ventricle
c. hypertrophy of left atrium
d. hypertrophy of right atrium
e. hypertrophy of left and right atrium

3. Palpation of heart at the apex beat revealed thrill which is not equal with carotid
pulsation. Which valvular disease can you suspect?
a. aortic regurgitation
b. mitral regurgitation
c. aortic stenosis
d. mitral stenosis
e. tricuspid regurgitation
4. What is the normal area of the aortic valve?
a. 1-2 cm2
b. 2-4 cm2
c. 4-6 cm2
d. 6-8 cm2
e. 8-10 cm2

5. What is the normal area of the mitral valve?


a. 1-2 cm2
b. 2-4 cm2
c. 4-6 cm2
d. 6-8 cm2
e. 8-10 cm2

6. Which heart disease is known for an increased systolic blood pressure and
decreased diastolic blood pressure?
a.Mitral stenosis
b. Aortic valve regurgitation
c. Mitral valve regurgitation
d. Mitral valve prolapse
e. Aortic stenosis

7. What is the most common cause of mitral stenosis?


a. Atherosclerosis
b. Pulmonary hypertension
c. Invective endocarditis
d. Acute rheumatic fever
e. Chest trauma

8. For which valvular disease following changings of relative dullness of heart is


typical: right border – 1 cm medial from right sternal border, left – by the anterior
axillary line, upper - III ICS?
a. mitral stenosis
b. mitral regurgitation
c. tricuspid regurgitation
d. aortic valve disorders
e. pulmonary stenosis
9. For which valvular disease following changings of relative dullness of heart is
typical: right border – 3 cm to the right of sternum, left - 1 cm medial from left
midclavicular line, upper border - II ICS?
a. mitral stenosis
b. mitral regurgitation
c. tricuspid regurgitation
d. aortic valve disorders
e. tricuspid reguurgitation

10. For which valvular disease following changings of relative dullness of heart is
typical: right border – 1 cm lateral from sternum border, left - 2 cm lateral from left
midclavicular line, upper - II ICS?
a. mitral stenosis
b. mitral regurgitation
c. tricuspid regurgitation
d. aortic valve disorders
e. tricuspid regurgitation

11. What are the most typical signs of arterial pulse “pulsusdifferens”?
a. significantly decreased or absence of pulsation at one artery radialis
b. significant decreased degree of pulse wave at both radial arteries
c. number of pulse waves at the artery radialis more than heart rate
d. pulse waves number at the artery radialis less than heart rate
e. moderately decreased degree of pulse wave at both radial arteries

12. What is the normal mitral valve area?


a. 1-2 сm2
b. 3-4 сm2
c. 4-5 сm2
d. 4-6 сm2
e. 7-8 cm2

13. What is the mitral valve area during critical mitral stenosis?
a. 1 сm2
b. 1,3 сm2
c. 1,5 сm2
d. 1,9 сm2
t. 2,0 cm2
14. Sinus bradicardia is – sinus rhythm with heart rate:
a. less than 60 per min
b. less than 75 per min
c. less than 85 per min
d. less than 90 per min
e. less than 100 per min

15. Auscultation results in mitral stenosis?


a. Loud S1; opening snap after S2; diastolic murmur
b. Holosystolic murmur; quiet S1; S3
c. Late systolic murmur; soft S2; S4
d. Early diastolic murmur; S3
e. Diastolic murmur with a decrescendo

16. What symptom is typical for left ventricular heart failure?


a. Hepatomegaly
b. Orthopnea, dyspnea
c. Ascites
d. Swelling of the feet
e. Anorexia

17. What is the main auscultative sign of mitral regurgitation?


a. diastolic murmur at the II intercostal space to the right
b. systolic murmur at the V ICS
c. diastolic murmur at the III ICS to the left
d. systolic murmur at the II ICS to the right
e. systolic murmur at the II ICS to the left

18. What is not typical for mitral regurgitation?


a. diastolic murmur at the 2nd ICS
b. systolic murmur at the 5th ICS
c. quiet S1
d. increased and enlarged apex beat, left ventricular hypertrophy signs
e. S3 gallop in advanced stages of disease

19. What is the direct sign of aortic regurgitation at the Doppler-


echocardiography?
a. regurgitation jet from right atria to right ventricle
b. regurgitation jet from left ventricle to left atria
c. regurgitation jet from right ventricle to right atria
d. regurgitation jet from aorta to left ventricle
e. regurgitation jet from pulmonary artery to right ventricle

20. What is the most common cause of myocarditis?


a. Viral infection
b. Fungi of the genus Candida
c. Staphylococcus aureus
d. Pneumococcus
e. E. coli

21. At what age myocarditis typically develops?


a. in the children's
b. in the elderly
c. in adolescent
d. in young
e. does not depend on age

22. What happens with S1 during significant mitral regurgitation?


a. decreased
b. increased
c. unchanged
d. splits up
e. doubled

23. Which syndrome is characterized by loud S2?


a. syndrome of hypertension
b. syndrome of acute coronary insufficiency
c. syndrome of myocardial inflammation
d. syndrome of pericardial inflammation
e. syndrome of chronic coronary insufficiency

24. Position change in orthopnea alleviates symptoms of patients with…?


a. syndrome of hypertension
b. syndrome of acute coronary insufficiency
c. syndrome of chronic coronary insufficiency
d. syndrome of left heart failure
e.syndrome of pericardial inflammation
25. What is the typical localization of chest pain in patients with syndrome of
chronic coronary insufficiency?
a. retrosternal
b. left part of chest, at the apex area
c. right part of chest
d. epigastria
e. upper part of abdomen

26. Sinus tachycardia is - sinus rhythm with heart rate?


a. more than 55 per min
b. more than 60 per min
c. more than 75 per min
d. more than 85 per min
e. more than 90 per min

27. What does the “mitral flush” mean?


a. appearance of erythematous skin rash on the cheeks and nose
b. pale face
c. cyanosis of cheeks
d. left cheeks hyperemia
e. puffy face

28. Which symptom is typical for heart failure?


a. dizziness
b. dyspnea
c. chest pain
d. increased blood pressure
e. memory loss

29. Which rhytm disorder is characterized by independent contraction of atriums


and ventricles?
a. AV block III degree
b. atrial fibrillation
c. atrial form of paroxysmal tachycardia
d. ventricular extra systole
e. atrial flutter

30. AV block I degree on ECG is characterized by:


a. increased PQ interval> 0.20 sec
b. increased QT interval
c. increased QRS complex
d. the premature appearance of the QRS complex
e. presence of Delta wave

31. Appearance of pathological Q wave at ECG in the syndrome of acute coronary


insufficiency – is a reflection of?
a. subendocardial myocardial ischemia
b. myocardial necrosis
c. myocardial injury
d. subepicardial myocardial ischemia
e. transient myocardial ischemia

32. What is not typical for mitral regurgitation?


a. diastolic murmur
b. systolic murmur at the apex area
c. decreased S1
d. increased and enlarged apex beat, left ventricular hypertrophy signs
e. accent of S2 at second ICS to the left

33. Which ECG-sign is not typical for left ventricle hypertrophy?


a. left axis deviation - R1>R2>R3
b. prolonged PQ segment
c. increased amplitude of R wave in V5, V6, deepening of S wave in V1, V2
d. displacement of ST segment at V5, V6, I and аVL leads below isoline
e. right axis deviation

34. What is the cause of cardiac murmur in aortic stenosis?


a. blood regurgitation from aorta to left ventricle
b. expulsion of blood from left ventricle to aorta
c. blood regurgitation from left atrium to left ventricle
d. expulsion of blood from left atrium to left ventricle
e. blood regurgitation from left ventricle to left atrium

35. What is the reason of cardiac murmur in mitral stenosis?


a. turbulent blood flowfrom left ventricle to aorta
b. turbulent blood flow from right atrium to right ventricle
c. blood regurgitation from aorta to left ventricle
d. turbulent blood flow from left atrium to left ventricle
e. blood regurgitation from left ventricle to left atrium

36. What is the cause of cardiac murmur in aortic regurgitation?


a. blood regurgitation from aorta to left ventricle
b. pushing blood from left ventricle to aorta
c. blood regurgitation from left ventricle to left atrium
d. pushing blood from left atrium to left ventricle
e. blood regurgitation from right ventricle to right atrium

37. Which level of blood pressure is typical for aortic valve regurgitation?
a. 120/90 mmHg
b. 130/80 mmHg
c. 160/100 mmHg
d. 190/120 mmHg
e. 160/40 mmHg

38. What is the typical duration of pain during chronic coronary insufficiency?
a) 3-5 minutes
b) 15-25 minutes
c) 25-30 minutes
d) 30-60 minutes
d) more than 60 minutes

39. Which syndrome is known for paradoxical pulse (Pulsusparadoxus)?


a. syndrome of inflammation of the myocardium
b. syndrome of inflammation of the pericardium
c. syndrome of inflammation of the endocardium
d. syndrome of cardiac arrhythmias
e. syndrome of chronic coronary insufficiency

40. Which symptom not typical in heart failure?


a. edema
b. dyspnea
c. chest pain
d. orthopnea
e. ascites

41. On examination of the heart can be identified:


A. pulsation of the apex beat
B. sizes of heart
C. sizes of the vascular bundle
D. symptom murmur
E. diastolic murmur

42. Symptom of systolic or diastolic murmur can be defined by:


A. percussion
B. auscultation above 2nd intercostal space
C. palpation of chest
D. auscultation above 3rd intercostal space
E. auscultation above apex beat

43. Symptom of diastolic thrill is determined by:


A. acute coronary insufficiency
B. mitral regurgitation
C. mitral stenosis
D. aortic regurgitation
E. aortic stenosis

44. Patient N., 34 years old, admitted to inpatient hospital. Doctor performed
objective examination and revealed auscultate murmur above the apex ofheart.
Lesion of which valve should be suspected at this clinical case?
A. aortic stenosis
B. pulmonary
C. mitral
D. tricuspid
E. aortic regurgitation

45. On palpation on the basis of the heart revealed systolic or diastolic thrill with a
surge on carotids. This is typical for:
A. mitral stenosis
B. mitral insufficiency
C. aortic stenosis
D. aortic insufficiency
E. tricuspid insufficiency

46 "Triangular" shape configurations of the heart character for:


A. mitral stenosis
B. tricuspid insufficiency
C. aortic insufficiency
D. exudative pericarditis
E. aortic stenosis

47. Auscultation of the heart in patients with severe heart failure can be identified:
A. rhythm of "quail"
B. protodiastolic gallop
C. systolic gallop rhythm
D. extra-pericardial tone
E. atrial gallop

48 Carotid-shudder is a sign:
A. aortic valve insufficiency
B. tricuspid valve insufficiency
C. mitral stenosis
D. mitral regurgitation
E. aortic stenosis

49. Full A-V block characterized by:


A. increase of the PQ interval
B. unrelated rhythm of atrials and ventricles
C. different RR intervals
D. expressed deformation of the ventricular complex
E. pathological Q wave

50. II tone in aortic stenosis:


A. weakened at the aorta
B. loud at the aorta
C. loud at the apex
D. weakened at the apex
E. does not change

51. When present aortic stenosis we can auscultate:


A. diastolic murmur at the apex
B. diastolic murmur at the aorta
C. systolic murmur at the aorta
D. systolic murmur at the apex
E. diastolic murmur at the apex

52. Positive venous pulse occurs when:


A. aortic stenosis
B. mitral regurgitation
C. tricuspid valve insufficiency
D. aortic insufficiency
E. mitral stenosis

53. ECG signs of AV block I degree:


A. the PQ interval < 0,12 sec
B. the PQ interval > 0,20 sec
C. no P wave
D. “f” waves
E. wide QRS complex

54. How many components I tone consist of:


A. 1
B. 2
C. 3
D. 4
E. 5

55. In aortic valve insufficiency we can auscultate:


A. systolic murmur at the base of the xiphoid process
B. protodiastolic murmur at Botkin - Erba
C. systolic murmur at Botkin – Erba
D. diastolic murmur at the base of the xiphoid process
E. systolic murmur at the apex

56. Enhancing of the apex beat testifies to


A. the right ventricular hypertrophy
B. hypertrophy of the right atrium
C. hypertrophy of the left atrium
D. the left ventricular hypertrophy
E. hypertrophy of both atriums

57. Accent of the II sound on the pulmonary artery occurs when


A. the increase in pressure in the systemic circulation
B. hypertension of pulmonary circulation
C. the left ventricular hypertrophy
D. the dilatation of the left ventricle
E. aortic stenosis

58. ECG signs of atrial fibrillation


A. P wave high, ventricular complexes recorded at different time intervals
B. the disappearance of the P wave, ventricular complexes recorded at
different time intervals, multiple waves f
C. multiple small waves f, P wave high
D. the disappearance of the P wave, ventricular complexes recorded at regular
intervals, multiple waves f
E. wide QRS, tall R waves

59. Signs of left ventricular heart failure:


A. edema of shins, cough, dyspnea
B. cough, hemoptysis, dyspnea, orthopnea
C. edema of shins, hepatomegaly
D. swelling and pulsation neck veins
E. ascities, hepatomegaly
60. In case of insufficiency of the mitral valve auscultation is defined:
A. loud I sound at the apex of the heart
B. weakening I tone at the apex of the heart
C. "opening snap" of the mitral valve
D. diastolic murmur at the apex
E. systolic murmur at the apex

61. The systolic murmur in aortic stenosis is held


A. in point Botkin-Erb
B. on the xiphoid
C. to armpits
D. on the vessels of the neck
E. to the abdominal aorta

62. Character the pain associated with the syndrome of chronic coronary
insufficiency:
A. sharp
B. shooting
C. burning
D. pressing
E. burning

63. Subjective characteristics of pain in the syndrome of chronic coronary


insufficiency:
A. constricting and pressing
B. stabbing
C. gnawing
D. knife-like pain
E. burning

64. De Musset's sign is observed at


A. aortic stenosis
B. mitral stenosis
C. aortic insufficiency
D. tricuspid valve insufficiency
E. mitral insufficiency

65. "Cardiac hump" occurs when


A. severe hypertrophy of left ventricular
B. severe hypertrophy of the left atrium
C. severe right ventricular hypertrophy
D. severe hypertrophy of the right atrium
E. severe biventricular hypertrophy
66. Diastolic thrill determined when
A. mitral insufficiency
B. mitral stenosis
C. aortic insufficiency
D. aortic stenosis
E. tricuspid stenosis

67. Systolicthrill determined when


A. mitral insufficiency
B. mitral stenosis
C. aortic insufficiency
D. aortic stenosis
E. tricuspid stenosis

68. Pulse deficiency occurs at


A. arterial hypertension
B. heart Failure
C. atrial Fibrillation
D. the aortic insufficiency
E. mitral stenosis

69. The upper border of relative cardiac dullness increased due


A. right ventricular hypertrophy
B. left ventricular hypertrophy
C. biventricular hypertrophy
D. hypertrophy of the right atrium
E. hypertrophy of the left atrium

70. For the left ventricular heart failure is characterized by:


A. edema of the feet
B. hepatomegaly
C. inspiratory dyspnea
D. ascites
E. hepatomegaly

71. For the right heart failure is characterized


A. hepatomegaly
B. inspiratory dyspnea
C. De Musset's sign
D. chest pain
E. low BP

72. Absolute ECG sign of acute coronary insufficiency syndrome:


A. deep pathologic Q wave
B. negative T waves
C. displacement of the ST interval
D. absence of P wave
E. wide QRS

73. Diastolic murmur at the aortic valve is held


A. interscapulum
B. at the apex
C. on the xiphoid
D. on the vessels of the neck
E. at the right 2nd intercostal space

74. The most informative method for diagnosis of coronary insufficiency is:
A. ECG
B. veloergometry
C. echocardiography
D. coronary angiography
E. Ct-scan

75. The main method of diagnosis of the syndrome of heart rhythm disorders:
A. echocardiography
B. ECG
C.X-ray of the chest
D. phonocardiography
E. CT-scan

76. The most informative method determining necrotic changes in the myocardium
is:
A. evaluation of leukocytes and ESR
B. evaluation of the level of alkaline phosphatase
C. evaluation of the level of transaminases
D. evaluation of troponins
E. evaluation of WBC

77. On the ECG PQ elongation equal 0,28sec. It indicates the presence of


A. sinoatrial block
B. atrioventricular block
C. extrasystols
D. atrial fibrillation
E. bundle bruch block

78. The most reliable sign of mitral stenosis is


A. The increase in the left border of the heart
B. faciesmitralis
C. mesodiastolic murmur and the presence of "opening snap" of mitral
     valve at the apex
D. flatness waist of the heart on X-ray examination
E. loud I tone at the apex

79. For the mitral valve insufficiency is not typical


A. loud first tone at the apex
B. systolic murmur at the apex
C. increase of left borders of the heart
D. weakening I tone at the apex
E. loud I tone at the left 2nd intercostal space

80. Blood pressure in aortic insufficiency:


A. increased only systolic
B. increased only diastolic
C. increased systolic and decreased diastolic blood
D. decreased systolic and increased diastolic
E. doesn’t change

81. For the aortic valve insufficiency is not typical


A. pulsation of pupils
B. decrease of pulse pressure
C. fast and high pulse
D. high systolic blood pressure
E. low diastolic blood pressure

82. For the syndrome of heart failure is not typical


A. increase of the sizes of the heart
B. increase of blood pressure
C. decrease of cardiac output
D. acrocyanosis
E. hepatomegaly

83. ECG signs of left ventricular hypertrophy does not apply:


A. negative T waves in leads V5-V6
B. tall R waves in right precordial leads
C. tall R waves in the left precordial leads
D. deep S wave in right precordial leads
E. R in V5-6 + S in V1-2 > 38mm

84. Select the symptom, not the appropriate for right heart failure:
A. increase in liver
B. decrease in venous pressure
C. edema of the lower extremities
D. ascites
E. hepatomegaly
85. Uncharacteristic sign for left ventricular heart failure is:
A. cardiac asthma
B.hepatomegaly
C. moist rales in the lungs
D. cough
E. dyspnea

86. The causes of heart failure may include the following drugs:
A. ACE-inhibitors
B. NSAIDs
C. Statins
D. β-blockers
E. PPI

87. Which of the following is not a risk factor for coronary heart disease:
A. increase in high density lipoprotein
B. diabetes
C. arterial hypertension
D. smoking
E. hyperlipidemia
88. Аortic stenosis is heard:
A. systolic murmur at the cardiac apex
B. diastolic murmur at the apex of the heart
C. systolic murmur in the second intercostal space to the right of the sternum
D. diastolic murmur in the second intercostal space to the right of the sternum
E. systolic murmur at the apex

89. The weakening of the first tone is not observed in:


A. mitral regurgitation
B. insufficiency of tricuspid valve
C. aortic insufficiency
D. mitral stenosis
E. tricuspid stenosis

90. Specify the correct definition of the term "cardiac impulse":


A. pulsation right of the xiphoid
B. pulsation left of the xiphoid
C. local ripple in the third intercostal space at the left sternal border
D. diffuse ripple throughout precardiac region
E. surge in the fourth, fifth, sixth intercostal space at the left border of the relative
dullness of the heart

91. In order to identify pericardial effusion is the most informative method:


A. ECG
B. coronary angiography
C. echocardiography
D. X-ray examination of the chest
E. CT-scan

92. Pulse pressure is:


A. the difference between systolic and diastolic blood pressure
B. stroke volume of the left ventricle
C. the amount of blood ejected by the ventricles in systole
D. the amount of blood ejected by the Atria to the ventricles
E. the difference between pulse and heart beat rate

93. Risk factors for hypertension does not include:


A. family history
B. hypocholesterolemia
C. abuse of table salt
D. alcohol abuse
E. smoking

94. What are the symptom characteristic of aortic stenosis:


A. De Musset’s sign
B. regular carotid pulsation
C. systolic murmur at the aortic with irradiation to the neck vessels
D. accent II tone on the pulmonary artery
E. diastolic murmur at the apex

95. Specify the cause of the syndrome acute coronary insufficiency:


A. coronary artery thrombosis
B. inflammation of the coronary arteries
C. inflammation of the coronary veins
D. arterial hypertension
E. trauma

96. Patient 42 years a significant shift of borders of relative dullness of the heart to
the left, increasing the apical impulse, pronounced symptom of "systolic jitter" in
the 2nd intercostal space to the right of the sternum and the same - weakening II
tone. Which of the following auscultatory phenomena must necessarily be the case
in this patient?
A. systolic murmur at the apex
B. diastolic murmur at the apex
C. systolic murmur over the aorta
D. diastolic murmur over the aorta
E. systolic murmur at the xiphoid

97. The most informative method to confirm or exclude the diagnosis of heart
disease is:
A. ECG
B. echocardiography
C. chest x-ray
D. coronary angiography
E. MRI

98. For a syndrome of insufficiency of the aortic valve is not typical:


A. ripple pupils
B. the decrease in pulse pressure
C. fast and high pulse
D. high systolic blood pressure
E. low diastolic pressure

99. For the syndrome of heart failure is not typical:


A. increase in heart size
B. gallop
C. chest psin
D. weakening of the apical impulse
E. systolic murmur

100. For the syndrome of inflammation of the endocardium is not typical:


A. positive blood culture
C. high fever
C. the appearance of pathological Q wave on the ECG
D. increase in ESR
E. vegetations on valves

101. To the targets organs in hypertension does not include:


A. kidney
B. heart
C. liver
D. the vessels of the ocular fundus
E. lungs

102. Patients with hypertension in the study of the ocular fundus is not
characteristic detection:
A. narrowing of the arteries of the retina
B. arterial tortuosity
C. vascular tortuosity
D. expansion retinal arteries
E. dilated veins

103. ECG sign of chronic coronary insufficiency is:


A. prolongation of the interval Q-T
B. prolongation of the interval P-Q
C. Q-wave amplitude of more than a third of R and longer than 0.03 seconds
D. change in the ST segment and T wave
E. pathological Q

104. ECG sign of acute coronary insufficiency is:


A. pathological Q
B. premature appearance of the P wave
C. Q-wave amplitude of more than a third of R and longer than 0.03 seconds
D. decrease in the amplitude R
E. change in the final part of the ventricular complex (ST segment) and T
wave

105. Echocardiographic sign of acute coronary insufficiency:


A. diffuse hyperkinesis
B. diffuse hypokinesis
C. local hypokinesis
D. local hyperkinesis
E. diffuse hyperkinesis

106. Atrial fibrillation is characterized by:


A. QRS>0.20 ms
B. availability of premature QRS complexes
C. presence of f - waves.
D. wide P waves
E. inverted T wave

107. For ventricular arrhythmias is not typical:


A. premature QRS complex
B. extrasystolic QRS complex expanded and deformed
C. have a full compensatory pause
D. Presence of P wave before extrasystolic complex
E. pathological Q wave

108. For supraventricular arrhythmia is not typical:


A. premature QRS complex
B. extrasystoly similar to the main QRS
C. the presence of a full compensatory pause
D. the presence of strain, and P wave before ekstrasystolichniy complex
E. pathological Q wave
109. Mitral stenosis characteristic symptom:
A. systolic murmur at the apex of the heart, and the increasing expiratory
B. accent and split II tone of the aorta
C. systolic murmur at the apex of the heart, worse on inspiration
D. additional high-frequency tone in diastole, distant from II tone on
     0.07-0.12 seconds
E. Soft S2 at aorta

110. The clinical manifestations of the syndrome of tricuspid valve diseases does
not apply:
A. ascites
B. pulmonary edema
C.gepatomegaliya
D. edema
E. dyspnea

111. What are the symptoms in the objective status of patients with aortic stenosis:
A. diffuse cyanosis of the skin
B. pale skin
C. De Musset’s sign
D. "dance carotid"
E. orthopnea

112. Pulse pressure in patients with aortic stenosis:


A. does not change
B. increases
C. decreases
D. increases sharply
E. decreases sharply

113.Typical symptom in the objective status of patients with aortic valve


insufficiency and aortic stenosis is:
A. pale skin
B. De Musset’s sign
C. pulsation of the carotid, subclavian, temporal, brachial arteries
D. capillary pulse
E. orthopnea

114. For patients with a syndrome of inflammation of the myocardium are most
common following complaints:
A. heart pain, palpitations, shortness of breath
B. pain in the heart, palpitations, syncope
C. pain in the heart, shortness of breath, ascites
D. heart pain, fever, dry cough
E. dry cough, shortness of breath

115. For stenosis of the left atrioventricular orifice is typical:


A. hypertrophy of the left ventricle
B. hypertrophy and dilatation of the right ventricle
C. systolic murmur at the apex
D. systolic tremor on the apex of the heart
E. Dilation of the left ventricle

116. Click mitral valve opening:


A. occurs through 0.06-0.12 seconds after closing the aortic valve
B. harakteren for mitral regurgitation
C. harakteren for aortic stenosis
D. harakteren for mitral and aortic stenosis
E. best auscultated at Botkin

117. What are heart disease, which is characterized by the following options for
changing the boundaries of the relative dullness of the heart: the right - to 3cm.
outwards from the right edge of the sternum, the left - 1 cm. medially from the left
midclavicular line, top - the top edge of the 2nd rib:
A. mitral stenosis
B. mitral insufficiency
C. tricuspid regurgitation
D. aortic stenosis
E. aortic regurgitation

118. Name ECG sign, which is a reflection of myocardial ischemia:


A. decrease in the amplitude of the R wave
B. deep broad Q wave
C. ST-segment depression
D. deformation of the QRS complex
E.PQ interval prolongation

119. Signs of acute coronary insufficiency on the ECG is:


A. ST-segment depression
B. high equilateral T wave
C. presence of pathological Q in combination in conjunction with the rise
     ST-segment
D. deformation of the QRS complex
E. Prolongation of PQ interval

120. confirmed acute coronary insufficiency in a patient with angina attacks and
ST-segment elevation is one of the following laboratory parameters:
A. decrease in hemoglobin
B. hyperlipidemia
C. increased ESR
D. increase in the activity of serum troponin
E. increase creatinine

121. Violations of excitability does apply:


A. sino auricular block
B. atrial fibrillation
C. intraatrial block
D. A-V blockade
E. Left bundle branch block

122. Arterial hypertension is characterized by the following levels of blood


pressure:
A. ˃ 130/90 mmHg
B. ˃140/100 mm Hg
C. ≥ 140/90 mm Hg
D. ≥ 160 /100 mm.rt.st.
E. ˃140/80 mm Hg

123. Pericardial murmur can be listened during:


A. chronic coronary insufficiency
B. heart failure
C. inflammation of myocardium
D. inflammation of pericardium
E. endocardial inflammation

124. For the syndrome of chronic coronary insufficiency typical pain:


A. which occurred after using antihypertensive drugs
B. pointing pain
C. with irradiation to the left hand
D. after using nitroglycerine
E. stitching pain

125. The order of heart auscultation:


A. apex beat, right 2nd intercostal space next to the sternum, left 2nd intercostal
space next to the sternum, ensisternum, Botkin-Erba point
B. apex beat, ensisternum, left 2nd intercostal space next to the sternum, right 2nd
intercostal space next to the sternum, Botkin-Erba point
C. ensisternum, apex beat, Botkin-Erba point, right 2nd intercostal space next to the
sternum, left 2nd intercostal space next to the sternum.
D. right 2nd intercostal space next to the sternum, left 2nd intercostal space next to
the sternum, apex beat, ensisternum, Botkin-Erba point.
E. apex beat, left 2nd intercostal space next to the sternum, right 2nd intercostal
space next to the sternum, ensisternum, Botkin-Erba point.

126. Functional murmur is preferably located:


A. at the apex beat
B. at the Botkin point
C. at aorta
D. on the base of ensisternum
E. above the truncus pulmonale

127. Displacement of the apex beat to left and low direction, outside from the
midclavicular line (at VI-VII intercostal spaces) occurres during:
A. aortic regurgitation.
B. mitral stenosis.
C. tricuspid stenosis.
D. tricuspid regurgitation.
E. aortic stenosis.

128. Which changings of heart percussion are more typical for mitral configuration
of heart?
A. «triangle» shape of heart;
B. displacement of right heart border to the right side;
C. displacement of left heart border to the left side with highlighted heart`s
“waist”;
D. displacement of upper heart border to the up side and diminishing of
heart`s “waist”;
E. displacement of left heart border to the left side and right heart border - to the
right side.

129. Diastolic murmur at the mitral stenosis


A. irradiates to left axillar region
B. is listened fine at the right-side position
C. is listened fine at the left side position at expiration phase
D. is listened fine at vertical position
E. is listened fine at vertical and at the right-side position

130. «Gibbuscardiacus» occurs at


A. early childhood
B. young period
C. mature age
D. old age
E. mature age and old age
131. Paroxysmal tachycardia – is:
A. heart rhythm abnormality as sudden starting and also sudden finishing
acute palpitation with heart rate 140-250 per minute at the presence of
right regular rhythm
B. irregular sinus rhythm, which characterized with periods of increased and
decreased heart rate
C. condition with frequent (up to 400-700 per minute) irregular, chaotic excitation
of separate myofibrils of atria
D. unexpected contraction of heart, which occurs under the influence of ectopic
excitation focuses from different places of conductive system.
E.an abnormal heart rhythm (arrhythmia) characterized by rapid and irregular
beating of the atrial chambers of the heart.

132. The main clinical manifestations of the syndrome cardiac arrhythmias:


A. palpitations, and disruption of the heart
B. edema the lower extremities
C. nose bleed
D. diarrhea
E. numbness of the legs

134. Conduction disorders does not apply:


A. sinuauricular block
B. sinus arrhythmia
C. intraatrial block
D. A-V blockade
E. Left bundle branch block

135. Atrial fibrillation - is:


A. heart rhythm disturbances in the form of a sudden beginning and as suddenly
     ending with a heart attack heart rate = 140-250 per minute at
     maintaining proper regular rhythm
B. irregular sinus rhythm, characterized by periods of more frequent and
     Deceleration
C. the condition in which there is frequent (up to 400-700 per minute),
disorderly, chaotic excitation of individual muscle fibers of the atrial
D. premature contraction of the heart that occurs under the influence of ectopic
     foci of excitation from different parts of the conduction system
E. an arrhythmia, more specifically a tachycardia affecting the ventricles with a
rate over 250-350 beats/min, and one of the most indiscernible.
136. Sinus arrhythmia - is:
A. heart rhythm disturbances with a sudden beginning and as suddenly
     ending with a heart attack heart rate = 140-250 per minute at
     maintaining proper regular rhythm
B. irregular sinus rhythm, characterized by periods of more frequent and
     deceleration
C. The condition when observed frequently (up to 400-700 per minute), disorderly,
     chaotic excitation of individual muscle fibers of the atria
D. premature contraction of the heart that occurs under the influence of ectopic
     foci of excitation from different parts of the conduction system
E. an arrhythmia, more specifically a tachycardia affecting the ventricles with a
rate over 250-350 beats/min, and one of the most indiscernible.

137. The cause of arrhythmias and conduction disturbances is:


A. inflammation of the myocardium
B. kidney cysts
C. nephrotic syndrome
D. stroke
E. viral hepatitis

138. Нeart block - is:


A. slowing or complete cessation of the electrical pulse
     any department of the conduction system
B. irregular heartbeat with suddenly starts and suddenly
     ending with a heart attack heart rate = 140-250 per minute at
     maintaining proper regular rhythm
C. irregular sinus rhythm, characterized by periods of more frequent and
     deceleration
D. decrease of heart rate to 59-40 per minute while maintaining regular sinus
rhythm
E. the condition in which there is frequent, disorderly, chaotic excitation of
individual muscle fibers of the atrial

139. Laboratory diagnosis of the syndrome of inflammation of the myocardium


includes:
A. leukocytosis, accelerated erythrocyte sedimentation rate
B. increased creatinine
C. hyperfibirinogenemia
D. hypoglycemia
E. hypercholesterolemia

140. What is a manifestation of pain in inflammation of the pericardium:


A. localization - behind the sternum or on the right of it
B. ST-segment depression
C. enhanced in the supine position, with a deep breath, coughing and
swallowing
D. stopped by nitroglycerine
E. diastolic murmur at the apex

141. Main auscultatory signs of pericardial friction noise:


A. place auscultation area is absolute stupidity of the heart, it is better
     is heard during exhalation, with the patient inclined forward or
     lying on left side
B. place the tip of the heart auscultation is better auscultation during
     inhalation, with the patient supine
C. place auscultation is the base of the heart, the noise is carried out on the carotid
     artery
D. place auscultation is the point Botkin-Erb better auscultation in
     with the patient standing with a forward lean
E. listened fine at the right-side position

142. paradoxical pulse is typical for:


A. syndrome of inflammation of the myocardium
B. syndrome of inflammation of the pericardium
C. syndrome of inflammation of the endocardium
D. syndrome cardiac arrhythmias (atrial fibrillation)
E. AV blockade

143. Paradoxical pulse - is:


A. distinct inspiratory decrease in the amplitude of the pulse wave, the best
perceived on carotid and femoral artery
B. asymmetrical pulse on different hands
C. is the difference between heart rate and pulse rate measured simultaneously
D. is the medical term for shortness of breath, sometimes described as “air
hunger.”
E. defined as the pulse of your heart, or a single short moment, or something that
acts as a unifying force 

144. The most common ECG signs of inflammation of the pericardium syndrome
are:
A. ST-segment elevation in many leads, pathological Q wave is absent;
B. ST-segment elevation in many leads, pathological Q wave is present;
C. ST-segment depression
D. negative T waves
E. prolongation of PQ segment

145. Echocardiography-sign syndrome of inflammation of the pericardium (the


presence of exudate) is:
A. increase in end-diastolic left ventricular size
B. increase in the thickness of the interventricular septum and posterior wall of the
left ventricle
C. appearance between the sheets of pericardial echonegative space
D. right ventricular dilation
E. increased systolic pulmonary arterial pressure

146. The clinical picture of the syndrome of inflammation of the endocardium


includes:
A. fever with chills and copious sticky sweat
B. swelling of the lower extremities
C. hepatomegaly
D. persistent bradicardia
E. paradoxical pulse

147. In laboratory and instrumental examination of patients with the syndrome of


pericardium lesions can be identified:
A. positive blood culture
B. normal cardiac enzymes
C. vegetation on the valves
D. appearance between the sheets of pericardial echonegative space
E. fever with chills and copious sticky sweat

148. Plan examination of the patient with the syndrome of hypertension includes:
A. bacteriological culture from the throat
B. determination of blood sugar and serum cholesterol
C. determination of the level of cardiospecific enzymes
D. determination of rheumatoid factor
E. consultation by a urologist

149. Instrumental diagnostics of a syndrome of hypertension includes:


A. Ultrasound of the thyroid gland
B. Coronary angiography
C. Chest x-ray
D. Urography
E. ECG

150.Еxtrasystole arrhythmia - is:


A. heart rhythm disturbances in the form of a sudden beginning and as suddenly
     ending with a heart attack, heart rate = 140-250 per minute at
     maintaining proper regular rhythm
B. wrong sinus rhythm, characterized by periods of more frequent and
     deceleration
C. a condition in which there is frequent (up to 400-700 per minute), disorderly,
     chaotic excitation of individual muscle fibers of the atria
D. premature contraction of the heart that occurs under the influence of
ectopic excitation centers from different parts of the conduction system
E. irregular sinus rhythm, characterized by periods of more frequent and
     Deceleration.

“Digestive system”
1. Under physiological conditions, food is in the oral cavity:
A. 3-5 min
B. 5-10 sec
C. 10-15 sec
D. 15-20 sec
E. more than 20 sec
2. An adult tongue normally is :
A. wet, clean and pink
B. dark, mild rash and raids on her
C. pale pink coloration, scurfy
D. papillae on the tongue mild
E. dry, crimson
3. Where located upper border of the liver on midclavicular line in
normally?
A. in the V intercostal space
B. at the VI rib
C. in the VII intercostal space
D. in the VIII intercostalspace
E. in the IX intercostal space

4. Normal length of the esophagus:


A. 16-17 cm
B. 40-60 cm
C. 25-30 cm
D. 10-20 cm
E. 60-70 cm

5. Percussion size of the liver by Kurlov in normal:


A. 9-8-7 cm
B. 3-4-5cm
C. 11-12-13cm
D. 12-13-14cm
E. 6-7-8 cm

6. Intraventricular pH-metry estimates:


A. the most accurate acid-producing function of the stomach
B. It determines the amount of gastric juice
C. enzyme secretion function of the stomach
D. pepsin-forming function
E. determines the amount of intestinal juice

7. Deep sliding palpation of the cecum is carried out:


A. in the right iliac region
B. in the left iliac region
C. In the umbilical region
D. in the right flank
E. in the left flank

8. Dysphagia is:
A. violation of passage of food through the esophagus
B. chest pain
C. nausea
D. bitter taste in the mouth
E. excessive salivation

9. Painful swallowing is called:


A. dysphagia
B. odynophagia
C. dyspepsia
D. ataxia
E. melena

10.When duodenal sounding portion "A" - is the content of:


A. 12 duodenal ulcer
B. common bile duct
C. the neck of the gall bladder
D. stomach
E. gallbladder

11.When duodenal sounding portion of the "C" - is the content of:


A. 12 duodenal ulcer
B. stomach
C. common bile duct
D. intrahepatic bile ducts
E. gallbladder
12.The nature of pain in the syndrome of exocrine pancreatic
insufficiency:
A. cramping pain
B. Pain of uncertain nature
C. severe, persistent pain
D. sharp, stitching pains
E. localized pain

13.What polling method is used to determine “splash noise”?

A. inspection
B. palpation
C. percussion
D. auscultation
E. laboratory

14.A doctor examining patient with bent fingers to produce jerky


movements without moving away from the abdominal wall. This
technique is called:
A. Surface palpation
B. deep palpation
C. Determination of "splashing noise"
D. Definition of "symptom Mendel"
E. auskultation

15.In which lines define the size of the liver by Kurlov?


A. midclavicular line, the middle and the right costal arch
B. midclavicular line, the middle and the left costal arch
C. anterior axillary, median and right costal arch
D. midsternal line, medial, anterior axillary
E. posterior axillary, front axillary and median

16.In diseases of the liver and biliary tract pain irradiates to:
A. left shoulder
B. right scapula
C. hearts
D. waist
E. the left hand

17.Normal spleen length on X rib is:


A. 3-6 cm
B. 5-6 cm
C. 6-8 cm
D. 6-10 cm
E. 6-12 cm

18.Normal spleen diameter is equal to:


A. 4-6 cm
B. 4-8 cm
C. 4-10 cm
D. 6-8 cm
E. 8-10 cm

19.The most typical complain characteristic for the pathology of the


esophagus is:
A. dysphagia
B. belching
C. epigastric pain
D. nausea
E. vomiting

20.GERD patients have complaints:


A. diarrhea
B. heartburn
C. nausea
D. burp air
E. abdominal distention

21.Diseases of the esophagus are characterized by:


A. dysphagia, hypersalivation
B. constipation
C. flatulence
D. pain around the navel
E. tenesmus
22.A 58-year-old man turned to a local doctor with complaints of
difficulty swallowing solid food. Anamnesis: fell ill 6 months ago, 3
months later there were pains in the interscapular region, weakness,
only semi-liquid food began to pass, lost 6 kg. Objectively: the skin is
pale, with palpation moderate pain in the epigastric region. An X-ray
examination of the esophagus revealed a narrowing of its lumen in
the middle third due to a filling defect over 6 cm. What research
should be done to clarify the syndrome?
A. ultrasound of the chest cavity
B. еsophagoscopy with biopsy
C. computed tomography
D. laparoscopy
E. bronchoscopy
23.Dyspeptic complaints include:
A. nausea, discomfort in the epigastric region
B. pain in the iliac region
C. dizziness
D. tarry stools
E. generalized weakness

24.What research method is indicated to detect esophageal


diverticulum?
A. Radiography
B. mediastinoscopy
C. Electro-Kimography
D. esophagoscopy
E. Esophagomanometr

25.The estimated surface palpation usually begin with:


A. epigastric
B. the left iliac region
C. left hypochondrium
D. the right iliac region
E. right hypochondrium

26.Parietal cells secrete:


A. hydrochloric acid
B. lactic acid
C. gastric mukoprotein
D. mucin
E. pepsin

27.What can be heard normally on auscultation of the stomach an adult


?
A. noise peritoneum friction
B. splashing sound
C. a sharp increase in the peristalsis of bowel sounds
D. slight rumbling, transfusion fluid
E. the absence of sounds and noises

28.Pain that are characteristic of gastric ulcer:


A. early pain
B. later pain
C. night pain
D. hunger pains
E. stabbing pain

29.Pathognomonic signs of gastric bleeding:


A. pallor, weakness
B. headache, dizziness
C. Coffee grounds vomiting, tarry stools
D. tachycardia, decreased blood pressure
E. stool with mucus, bradycardia

30.Gastric bleeding is manifested by :


A. hemoptysis
B. hematuria
C. bloody vomit
D. the noise of splashing epigastric empty stomach
E. leukocyturia

31.Which method is the most informative to determine the cause and


location of bleeding from the upper gastrointestinal tract:
A. radiography
B. portomanometriya
C. laparoscopy
D. gastroduodenoscopy
E. tseliakografiya

32.The patient, 23 years old, came to the hospital with complaints of


heaviness and a feeling of fullness in the epigastric region after
eating, acid reflux, nausea. From the anamnesis: the patient smokes a
lot, abuses coffee, sometimes he takes diclofenac for headaches. Sick
for about 4 years. Not examined and not treated. What syndrome can
be assumed?
A. stomach inflammation syndrome
B. esophageal inflammation syndrome
C. pancreatic inflammation syndrome
D. gallbladder inflammation syndrome
E. liver inflammation syndrome

33.The most common ulcers in the stomach are located:


A. in subcardiac part
B. in the cardiac part
C. on a small curvature in the antrum
D. on the greater curvature
E. gastric body
34.In healthy individuals the volume of gastric juice secretion in basal
(in ml) is:
A. up to 50
B. 50-100
C. 100-140
D. 140-180
E. 100-200

35.A 37-year-old patient was admitted to the hospital with complaints of


pain in the epigastric region after 20-30 minutes. After eating,
heartburn, nausea and vomiting, which bother for a week. From the
anamnesis: suffers from chronic gastritis for 5 years, from which she
was not treated. On endoscopy: the esophagus is blown away, the
mucous membrane is not changed, the cardia is of normal shape and
size. The stomach is of normal shape and size. The mucous
membrane is hyperemic, in the antrum there is a deep ulcerative
defect of 1.5-2.0 cm in size, folds of usual shape and size. The
duodenal bulb of the usual shape and size, the mucous membrane is
not changed. What additional tests are needed to identify the causes
of peptic ulcer?
A. X-ray examination
B. electrogastroenterography
C. study of gastric secretion
D. helicobacter pylori tests
E. total protein and its fractions

36.A woman, 56 years old, went to the clinic complaining of bitterness in


her mouth, nausea, sometimes vomiting, aching pain in the right
hypochondrium, poor tolerance of fatty foods, alcohol. The above
complaints are disturbing for a year, have not been previously
examined and have not been treated. Objectively: subictericity of the
sclera, in the right hypochondrium and right scapular region - the
area of skin hyperesthesia, a positive symptom of Grekov-Ortner.
Your suspected syndrome:
A. stomach inflammation syndrome
B. esophageal inflammation syndrome
C. pancreatic inflammation syndrome
D. gallbladder inflammation syndrome
E. liver inflammation syndrome

37. The largest size of gastric ulcers:


A. to 0.3 cm
B. to 0.5 cm
C. 0.3-0.4 cm
D. 2-3 cm and above
E. 0.2sm less

38.Choose localization of ulcers that are characteristic for hunger


pains in 1.5 hours after meal:
A. pyloric ulcer of the stomach, duodenum ulcer
B. ulcers subcardial part of the stomach
C. ulcers, middle and lower third of the stomach.
D. stomach ulcer of the body
E. ulcers of the greater curvature of the stomach

39.Patient M., aged 35, complained of aching, hungry pain at night in


the epigastric region. Pains go away after a meal. From the history of
the disease: for many years observed with a diagnosis of chronic
gastritis. Eats irregularly, often fast foods. On examination: belly is
symmetrical, involved in the act of breathing. On palpation:
Shchetkin-Blumberg symptom is negative, Mendel's positive
symptom. What kind of syndrome is to think the doctor?
A. syndrome, inflammation of the gallbladder
B. bleeding from stomach ulcers
C. syndrome of peptic ulcers
D. malabsorption syndrome
E. syndrome maldigestion

40.Hypersecretion and hyperchlorhydria is characteristic for the


localization of ulcers in:
A. esophagus
B. small curvature of the stomach
C. body of the stomach
D. duodenum
E. cardiac part of the stomach

41.Black, tarry stool found in bleeding from:


A. stomach and duodenum
B. blind, ascending colon
C. the transverse colon
D. descending and sigmoid colon
E. rectum

42.A direct indication of the presence of ulcers on X-ray examination


is:
A. scar body deformation
B. folds convergence
C. "Niche" symptom
D. evacuation change
E. dyskinesia gatekeeper

43.On physical examination, patients with peptic ulcer syndrome


marked tenderness to percussion in epigastric. This is a symptom of
:
A. Mendel
B. Murphy
C. Ortner
D. Kerr
E. Courvoisier

44.Vomiting 2-3 hours after meals is typical for:


A. ulcers cardiac part of the stomach
B. ulcer of the stomach body
C. ulcer of the stomach antrum
D. pyloric stenosis
E. duodenal ulcer

45. Pain during peptic ulcer decrease or disappear:


A. after vomiting
B. when applying cold
C. after fatty meal
D. when you change the patient's position
E. the application of heat

46. Skin itching in diseases of the hepatobiliary system is caused by:


A. increase in blood bilirubin
B. increase in blood cholesterol
C. increase in blood bile acids
D. increase in blood levels of urea
E. increase in blood glucose

47.The normal serum bilirubin is:


A. 1-3,02 mol / l
B. 8,55-20,5 .mu.mol / l
C. 22-25,0 mmol / l
D. 30,70-40,70 mol / l
E. 40,80-50,80 mol / l
48. Disability, weakness, fatigue, malaise, headaches in patients with
liver disease is indicated by the syndrome:
A. dyspeptic
B. asthenovegetative
C. haemorrhagic
D. hepatolienalis
E. portal hypertension

49. The total protein content in the blood in normal:


A. 65-85 g / l
B. 60-80 g / l
C. 70-90 g / l
D. 80-100 g / l
E. 40-55 g / l

50. Localization of pain in liver disease :


A. the left back side
B. umbilical region
C. the right iliac region
D. right upper quadrant
E. left upper quadrant

51. Decreased appetite, heaviness in the epigastric region, nausea,


vomiting, bloating, stool disorders in patients with liver disease are
the signs of :
A. hemorrhagic syndrome
B. asthenovegetative syndrome
C. hepatolienal syndrome
D. portal hypertension
E. dyspeptic syndrome

52.The rich fat mushy feces is observed when:


A. gastric hypersecretion
B. syndromeof peptic ulcer
C. portal hypertension
D. exocrine pancreatic insufficiency
E. syndrome of gallbladder inflammation

53.The normal content of total protein in the blood:


A. less than 20 g / l
B. 25-45 g / l
C. 45-65 g / l
D. 65-85 g / l
E. more than 85 g / l
54.Normally, Thymol test is:
A. 0-5 units.
B. 5-9 units
C. 10-14 units
D. 15-19 units
E. more than 20

55.Ascites ,enlargement of the saphenous veins of abdominal wall,


enlarged spleen, esophageal varices in patients with liver disease
syndrome denotes:
A. asthenovegetative
B. dyspeptic
C. haemorrhagic
D. portal hypertension
E. hepato-splenic

56.Normally, the blood pressure in the portal vein:


A. 50-115mm water column
B. 70-150mm water column
C. 150-220mm water column
D. 220-290mm water column
E. 290-360mm water column

57."Spider veins" represent:


A. intradermal deposits of cholesterol in the form of plaques
B. dermatorrhagia
C. slightly elevated above the skin pulsating angioma
D. bruising
E. dilation of small blood vessels in the skin

58. A 50-year-old man was admitted to the hospital with complaints: of


an increase in the abdomen, general weakness, poor appetite, weight
loss in the last 2 weeks. From the anamnesis of the disease: 2 years
ago, after visiting a dentist, he fell ill, suffered viral hepatitis.
Objectively: swelling of the lower extremities, skin and visible
jaundice mucosa. On examination, the abdominal cavity in the
supine position is spread out with a protrusion in the lateralsections,
the sound of dull is determined by percussion in these sections, upon
palpation - a positive symptom of “oscillation”. The liver protrudes
1.5-2 cm from the edge of the costal arch, with a dense texture. The
lower pole of the spleen is palpable. Laboratory: total bilirubin-34
μmol / l, total protein-55 g / l. Highlight the underlying syndrome.
A. hepatic cell failure syndrome
B. suprahepatic jaundice syndrome
C. hepatic jaundice syndrome
D. portal hypertension syndrome
E. gallbladder inflammation syndrome

59. Signs of liver failure syndrome:


A. increase in the total protein content
B. albumin increase
C. increased cholestorol
D. decrease in the activity of enzymes ALT, AST
E. decrease of albumin

60. A possible cause of the appearance of «spider veins» in liver failure


syndrome is:
A. dysproteinemia
B. hyperestrogenemia
C. thrombocytopenia
D. Hypocholesterolemia
E. hypofibrinogenemia

61. In hepatic jaundice hyperbilirubinemia is due to:


A. conjugated and unconjugated bilirubin
B. conjugated bilirubin
C. bile acids
D. free bilirubin
E. glucose oxidase

62.Skin itching in Liver disease is characteristic for?


A. stock duodeno-gastric reflux
B. an increase of bile acids in the blood against the backdrop of severe
cholestasis
C. violation synthetic (protein-) liver function
D. heart failure with the expressed accompanying myocardial dystrophy
E. reducing detoxifying liver function with respect to protein breakdown products

63.Cytolysis syndrome that develops during liver damage is


characterized by:
A. increased ALT, AST, LDH
B. increase in prothrombin level
C. increase in total protein and albumin
D. no laboratory changes
E. reducing thymol
64.In suprahepatic jaundice in the blood is detected:
A. lowering of unconjugated bilirubin
B. increase in ketone bodies
C. lowering ketone bodies
D. an increase of conjugated bilirubin
E. an increase of unconjugated bilirubin

65. In suprahepatic jaundice what is found in the urine:


A. elevated levels urobilin
B. reduced content urobilinoides
C. elevated levels of conjugated bilirubin
D. lowering the content of conjugated bilirubin
E. urine unchanged

66. In the suprarenal jaundice what is detected in the stool:


A. elevated levels stercobilin
B. reduced content stercobilin
C. aholich feces
D. tarry stool (melena)
E. lack stercobilin

67.In the horizontal position soft abdomen, side sections swell (frog
belly) suggests:
A. obesity
B. the presence of large intra-abdominal tumors
C. flatulence
D. fluid accumulation in the abdominal cavity
E. enlarged liver

68."Palmar eritema", represent:


A. intradermal deposits of cholesterol in the form of plaques
B. bleeding into the skin
C. petechiasis
D. symmetrical reddening of thenar and hypothenar
E. bruising

69.Skin color in obstructive jaundice:


A. pale lemon yellow tinge
B. orange-yellow
C. dark greenish
D. leather is not changed
E. bronzing

70.The projection (localization) of the gallbladder is equal to:


A. the point of intersection of the outer edge of the right rectus abdominis
with the costal arch
B. under the lower edge of the liver, outwards from the edge of the right lateral
rectus
C. in the left upper quadrant
D. at the intersection of the outer edge of the left rectus abdomen with the costal
arch
E. in the left iliac region

71. The bleeding from the nose, gums, uterine bleeding, cutaneous
bleeding,
bleeding of esophageal varices in patients with liver diseases
designate syndrome:
A. asthenovegetative
B. dyspeptic
C. haemorrhagic
D. portal hypertension
E. hepatolienal

72.The wall thickness of the gallbladder by ultrasound in normally:


A. not more than 2mm
B. 3-4 mm
C. 5-6 mm
D. 7-8 mm
E. 8-10 mm

73. The gallbladder contracts under the influence of :


A. gastrin
B. pancreatic juice
C. cholecystokinin
D. secretin
E. of hydrochloric acid

74.Main function of bile is:


A. emulsification of fats
B. carbohydrate hydrolyzing
C. split proteins
D. split of fat
E. increases of the secretion gastrin

75. In hypomotor dyskinesia of gallbladder observed:


A. increase in the tone of the gall bladder
B. decreased tone of the gallbladder with bile stasis
C. reduction of the initial dimensions of the gallbladder
D. accelerated emptying of the gallbladder
E. increasing the motor activity of the gall bladder and biliary tract

76.With exogenous jaundice, in contrast to true jaundice:


A. only sclera are painted
B. scleral staining is not characteristic
C. only palms and feet are painted
D. characterized by coloration of the skin, mucous membranes and sclera
E. only oral mucous membranes are stained

77. Soreness in striking along the costal arches indicates a positive


symptom ....
A. Ortner
B. Kera
C. Vasilenko
D. Boas
E. Mussey-Georgievsky

78. What can trigger the attack of biliary colic :


A. overheating or undercooling
B. abundant intake of fatty foods
C. smoking
D. bad dream
E. poor appetite

79.Cholecystography - is X-ray methodic of :


A. portal vein
B. gallbladder
C. splenic vein
D. celiac artery
E. bulbs of duodenum

80.Skin itching in diseases of the hepatobiliary system is caused by:


A. increase in blood bilirubin
B. increase in blood cholesterol
C. an increase in blood bile acids
D. increase in blood levels of urea
E. increase in blood glucose

81.The method of physical examination to determine the upper and


lower boundaries of the liver:
A. inspection
B. deep palpation
C. percussion
D. auscultation
E. estimated surface palpation

82.In diseases of the hepatobiliary system, pain is most often gives


A. to the left shoulder
B. down the abdomen, in the thigh
C. up, left, back
D. up, right, back
E. in the left shoulder blade

83. In hypermotor gallbladder dyskinesia be observe:


A. decreased tone of the gall bladder
B. spasm of the sphincter of Oddi and Lyutkense
C. increasing the initial size of the gallbladder
D. slowing down the emptying of the gallbladder
E. reduction in motor activity of the gallbladder and bile ducts

84.Pain like a band in the upper abdomen is observed in pathology:


A. stomach
B. liver
C. pancreas
D. cecum
E. sigmoid colon

85.Point De Garden’s- is:


A. the projection of the sphincter of Oddi
B. Pancreatic body projection
C. the initial part of the pancreas
D. the projection of the pancreatic head
E. projection cardiac of the stomach

86.Intradermal deposition of cholesterol in the century as yellow


plaques called?
A. petechial rash
B. xanthelasmatosis
C. telangiectases
D. hematoma
E. palmar erythema
87.Gold standard evaluation failure of exocrine pancreatic function is:
A. determination of the activity of pancreatic elastase in feces
B. microscopic examination of feces
C. secretin-probe test pankreazimin
D. determination of blood amylase and trypsin
E. determination of urine amylase and trypsin

88.Exocrine pancreatic insufficiency is characterized by:


A. increased alkaline phosphatase
B. presence of unconjugated bilirubin in the urine
C. hypoglycemia
D. steatorrhea, kreatorrhea, amylorrhea
E. raising the general level of protein in the blood

89.The defeat of the pancreatic head pain is defined:


A. point De Garden
B. at Mayo-Robson
C. at Kacha
D. point Mallet-Guy
E. zone Shaffer's

90.Zone Gubergriz’s:
A. It reflects the initial portion of the body of the pancreas
B. It reflects the tail of the pancreas
C. It corresponds to the projection of the sphincter of Oddi
D. corresponds to the body of the pancreas
E. It reflects the terminal portion of the body of the pancreas

91.A sharp pain during effleurage in the projection of the gallbladder at


the height of inspiration - is a symptom of :
A. Zakharyin
B. Vasilenko
C. Obraztsov - Murphy
D. Ortner
E. Musso - George's

92.Tenderness when the hand taps the right costal arch -it's a positive
symptom of:
A. Zakharyin
B. Vasilenko
C. Obraztsov - Murphy
D. Ortner
E. Musso George's

93.Pain on the right hypohondrium when pressure in between the legs


of sternocleidomastoid muscle - it's a positive symptom:
A. Zakharyin
B. Vasilenko
C. Obraztsov - Murphy
D. Ortner
E. Musso - George's

94.In duodenal intubation detectionof in portion "B" large amount of


leucocytes and increase of gallbladder epithelial cells indicates:
A. an inflammation of the gall bladder
B. peptic ulcers
C. inflammation of the duodenum
D. inflammation of the pancreas
E. dyskinesia of the gallbladder

95.Stool frequency constipation:


A. 1 time per day
B. 1 time in 2 days
C. stool delay more than 3 days
D. 2 times a day
E. 3 times per day

96.Pain in the right iliac region arise in pathology:


A. cecum
B. sigmoid colon
C. transverse colon
D. rectum
E. stomach

97.In the normal intestine percussion determines


A. tympanic sound
B. dull sound
C. dullness sound
D. box sound
E. resonance sound

98.Superficial palpation reveals:


A. the consistency of the body
B. intestinal localization
C. voltage abdominal wall
D. body diameter
E. body surface

99.Flatulence - is:

A. frequent stools
B. false urge to defecate
C. abdominal distention
D. tarry stools
E. pain around the navel.

100. A sigmoidoscopy examines the colon mucosa in the distance (cm):


A. 45
B. 50
C. 35
D. 40
E. 50

101. A colonoscopy allows to explore:


A. Only the cecum
B. only the sigmoid colon
C. only the rectum and part of the sigmoid colon
D. Only rectum
E. the entire large intestine

102. X-ray examination of the large intestine - is:


A. colonoscopy
B. sigmoidoscopy
C. irrigoscopy
D. laparoscopy
E. duodenography

103. Signs of maldigestion syndrome according to the analysis of feces:


A. steatorrhea, kreatoreya
B. aholic feces
C. desquamated epithelium
D. melena
E. positive reaction Gregersen
104. About the defeat of the small intestine says:
A. Feces up to 15 times a day
B. blood stains in feces
C. abundant, soft, foamy feces
D. solid, black, sheep stool
E. discolored feces

105. If a large number of muscle fibers is determined in the stool, it's:


A. stearrhea
B. kreatorrhea
C. lientereya
D. melena
E. amylorrhea

106. Patient S., 46 years old, complaints on admission: an enlargement


of the abdomen, general weakness, heaviness in the right upper
quadrant, jaundice of the skin. From the history of the disease.
During the year, there is a gradual increase of the above symptoms.
As a child, he moved Botkin's disease, alcohol abuse. What research
tools are needed to clarify the syndrome?
A. cholecystography
B. duodenal intubation
C. ultrasonography of the abdomen
D. laparoscopic abdominal organs
E. microscopic analysis of feces
107. What is the name of the syndrome due to malabsorption of
nutrients in the small intestine:
A. fermentative dyspepsia syndrome
B. cholestatic syndrome
C. distal colitic syndrome
D. maldigestion syndrome
E. malabsorption syndrome

108. Primary malabsorption occurs when:


A. vitamin B12 deficiency
B. hereditary deficiency of lactase
C. hypoholii
D. enteritis
E. gastritis

109. In malabsorption syndrome swelling is associated with:


A. hypoproteinemia
B. portal hypertension
C. circulatory failure
D. anemia
E. thyroid dysfunction

110. Moderate polyfecalia, shaped or gruel, feces, grayish-white in


color, with a musty fetid odor, with an acid reaction, with an
abundance of fatty acids and soaps, a small amount of neutral fat,
occurs when:
A. fermentative dyspepsia syndrome
B. gastric dyspepsia
C. intestinal dyspepsia
D. Pancreatogenic dyspepsia
E. hepatogenic dyspepsia

111. Feces of an adult normally release:

A. unconjugated bilirubin
B. conjugated bilirubin
C. urobilin
D. stercobilin
E. urobilinogen

112. The complete absence of hydrochloric acid and pepsin in gastric


juice is called:
A. hyperacidity
B. Achille
C. hypochlorhydria
D. achlorhydria
E. hypersecretion

113. On macroskopy examination feces in normal:


A. polifekaliya
B. unformed liquid
C. elegant, brown
D. shine stool
E. mushy
114. Malabsorption syndrome of Vitamin A deficiency leads to:
A. night blindness
B. subcutaneous hemorrhage
C. neurological disorders
D. dermatitis
E. esophagitis

115. Patient, 64 years old, about a cough with profuse mucous sputum,
shortness of breath, fever up to 37.7 ° C. I received tetracycline inside
for 5 days, then replaced with kanamycin injections on the 10th day
of the disease, diarrhea occurred (7 times a day), blood in the stool,
pain throughout the abdomen, bloating, fever up to 39 ° C. Which
imagine method is need patient:
A. Fecal occult blood test
B. determination of urine amylase activity
C. Fecal analysis for ascariasis
D. colonoscopy
E. Irrigoscopy

116. If in the feces are determined by a large number of neutral fat,


it's this:
A. stearrhea
B. kreatoreya
C. lientereya
D. amylorrhea
E. melena

117. In diseases of the small intestinal pain is observed:


A. around the navel
B. the right-hand side of the area
C. in the left iliac region
D. epigastric
E. in the right upper quadrant

118. What information does the positive symptom waves (fluctuations)


in the percussion bimanual palpation of the abdomen give?
A. symptom is detected normally;
B. there is free fluid in the abdominal cavity;
C. It has pyloric stenosis;
D. a large amount of gases in the colon (flatulence in patients with colitis);
E. has accumulated liquid contents and gases in the colon (e.g., a patient with
acute enteritis).
119. During Diarrhea stool is :
A. 1 time per day
B. 1 time on the second day
C. more than 3 times a day
D. 1 time per day Zoe
E. 1 time per week

120. Schmidt sample was carried out to:


A. identify protein exudation
B. determination of occult blood in the stool
C. determining the complete cessation of the flow of bile into the intestine
D. identifying novoobrozovaniya
E. identifying ulceration

"Endocrine System"

1) What is the level of blood glucose corresponds to the hypoglycemia


syndrome in venous plasma:

A. 2.8 mmol / L
B. 4.0 mmol / L
C. 4.7 mmol / L
D. 5.6 mmol / L
E. 7.0 mmol \ L

2) Which of the following corresponds to impaired glucose tolerance (venous


plasma):

A. Fasting glucose - 7.4 mmol / l, 2 hours after OGTT - 12.2 mmol / l


B. Fasting glucose - 6.8 mmol / l, 2 hours after OGTT - 10.9 mmol / l
C. Fasting glucose- 6.2 mmol / l, 2 hours after OGTT - 7.7 mmol / l
D. Fasting glucose- 5.6 mmol / l, 2 hours after OGTT - 7.4 mmol / l
E. Fasting glucose- 4.0 mmol / l, 2 hours after OGTT - 6.8 mmol / l

3) List the triad of symptoms in hyperglycemia syndrome:

A. Polydipsia, polyphagy, polyuria


B. Hypertension, hyperpigmentation, obesity
C. Hypotension, anorexia, anuria
D. Exophthalmos, hypertension, hyperthermia
E. Alopecia, bradycardia, hypercholesterolemia

4) Complaints of patients with hypoglycemia syndrome may be:


A. Epigastric pain
B. Slow wound healing
C. Burning in the lower extremities
D. Pustular skin lesion
E. Tremor in the body and limbs

5) List the main risk factors for the development of hyperglycemia syndrome:
A. Heredity, obesity, lack of physical activity
B. Long-term stress, puberty, smoking
C. Age, Gender, Ethnicity
D. viral infections, radiation, injuries
E. Regular nutrition, pregnancy, alcoholism

6) Which of the following is the cause of hypoglycemic syndrome:


A. Hypercorticism syndrome
B. Overweight
C. Hypocorticism syndrome
D. Thyroid inflammation syndrome
E. Hyperthyroidism syndrome
 
7) Which of the following is the cause of hyperglycemic syndrome:
A. Hypercorticism syndrome
B. Hypothyroidism syndrome
C. Hypocorticism syndrome
D. Hypoparathyroidism syndrome
E. Anorexia Syndrome

8) The glucose level in the plasma of venous blood is normal:


A. Fasting 3.3-5.5 mmol / l, 2 hours after meals> 7.8 mmol / l
B. Fasting 5.6-6.1 mmol / l, 2 hours after meals 6.1-7.0 mmol / l
C. Fasting 5.6-6.1 mmol / l, 2 hours after meals<6.7 mmol / l
D. Fasting 4-6.1 mmol / l, 2 hours after a meal <7.8 mmol / l
E. Fasting 4.6-5.1 mmol / l, 2 hours after meals <8.5 mmol / l

9) The appearance of edema in hypercorticism syndrome is associated with:


A. Hypermagnesemia
B. Hypernatremia
C. Hypokalemia
D. Hypomagnesemia
E. Hyponatremia

10) An oral glucose tolerance test is performed:


A. On the background of acute illness
B. When glycogemoglobin is more than 6.5%.
C. In case of doubtful glycemia for clarification
the diagnosis
D. When blood glucose is greater than 11.0 mmol / L.
E. When fasting blood glucose is less than 3.3 mmol / L.

11) When viewed in the oral cavity with hyperglycemia syndrome can be
identified:

A. Loosening and early tееth loss


B. Geographic language
C. Tongue is covered with yellow bloom
D. Black spots on tongue and gums
E. Varnished language

12) Rare blinking in hyperthyroidism syndrome, this is a symptom:


A. Kocher
B. Shtelvag
C. Moebius
D. Krause
E. Elinek

13) The appearance of myxedema on the limbs of a patient with


hypothyroidism syndrome is due to a violation of:
A. Basic exchange
B. Carbohydrate Exchange
C. Lipid metabolism
D. Protein metabolism
E. Water-salt metabolism
14) A man, 36 years old, went to the clinic to the local doctor with complaints
of pain and swelling in the chest area from 2 sides. On examination: increased
nutrition, uneven distribution of the subcutaneous fat layer with a
predominant deposition in the upper body and on the face, on the front
surface of the abdomen and on the inner thighs of the strip of red-purple
color, palpation in the chest reveals dense painful formations with a diameter
of up to 5, blood pressure - 160/100 mm Hg
Highlight the main syndrome:
A. Thyroid hyperfunctions
B. Hypofunction of the thyroid gland
C. Hyperfunction of the adrenal cortex
D. Hypofunction of the adrenal cortex
E. Hyperfunctions of the parathyroid glands

15) On examination of the skin in a patient with hyperglycemia syndrome, it is


determined:

A. Hyperpigmentation
B. Humidity
C. Rubeosis of the cheeks
D. Cyanotic
E. Facial hair growth in women

16) For hyperfunction of the thyroid gland is characteristic:

A. Sleepiness
B. Bradycardia
C. Puffiness of the face and narrowing of the palpebral fissures
D. Hyperexcitability
E. Obesity

17) With acute hypoglycemia syndrome:


A. Skin is wet, motor agitation
B. Breathing is noisy and rare
C. Skin is dry
D. Reduced turgor
E. Thirst

18) Gynecomastia in patients with hypercorticism syndrome occurs due to


metabolic changes:
A. Glucocorticoids
B. Thyroid hormones
C. Sex hormones
D. Mineralocorticoids
E. Pituitary hormones

19) Symptoms of hypothyroidism include:

A. Diarrhea
B. Sweating
C. Constipation
D. Cyanosis
E. Heartbeat

20) Laboratory signs of hypothyroidism syndrome:

A. Decrease in T3 and T4 in the blood


B. Increase in T3 and T4 in the blood
C. Elevation of TSH and T4 in the blood
D. Decrease in the level of TSH in the blood
E. Increased T3 and T4, decreased TSH

21) Hypocorticism syndrome is characterized by:

A. Hyperpigmentation of the skin


B. Weight gain
C. Increased blood pressure
D. Tachycardia
E. Polyphagia

22) The volume of the waist in men normally should not exceed?
A. Not more than 94 cm
B. Not more than 106 cm
C. Not more than 86 cm
D. Less than 60 cm
E. Morethan 102 cm

23) How to calculate BMI (body mass index)?


A. Height / weight2
B. Weight / Height2
C. Height2/ weight
D. Weight2 / Height
E. Weight / Height

24) In which units is body mass index measured?


A. m2 / kg
B. m / kg2
C. kg / m2
D. kg2 / m
E. kg / m

25) Waist size in women is normal?

A. Not more than 65 cm


B. Not more than 70 cm
C. Not more than 80 cm
D. Less than 60 cm
E. More than 88 cm

26) The patient came to the endocrinologist. When calculating the body mass
index was 29.8 kg / m2. Determine the degree of impaired fat metabolism.

A. Underweight
B. Overweight
C. Obesity I degree
D. Obesity II degree
E. Obesity III degree

27) With the introduction of 1 mg of dexamethasone at 12 o'clock at night, a


decrease in cortisol by 50% or more the next morning indicates:

A. Hypocorticism
B. About the normal function of the adrenal cortex
C. About hypercorticism
D. About secondary hypocorticism
E. On increased secretion of ACTH

28) The following changes on an ECG are characteristic of a syndrome of a


hypofunction of a thyroid gland:
A. reduction of teeth voltage, bradycardia
B. tachycardia, ST segment reduction
C. atrial fibrillation
D. Pathological Q wave
E. AV blockade

29) Goiter is:

A. well palpable thyroid


B. thyroid disease, occurring in violation of its function
C. thyroid disease, complicated by tracheal compression
D. thyroid enlargement more than 18 ml in women and more than 25 ml in
men
E. An increase in volume of more than 16 ml in women and more than 18 ml in
men.

30) Laboratory criteria for hypoparathyroidism syndrome:

A. increased blood potassium levels


B. increased blood levels of parathyroid hormone
C. lower blood calcium levels
D. lower blood sodium levels
E. Increased blood glucose

31) With an enlargement of the thyroid gland of I degree (WHO):

A. The thyroid gland is invisible, palpable


B. The thyroid gland is not visible to the eye, is not palpable.
C. Thyroid visible on the eye, palpable
D. The thyroid gland is visible to the eye, not palpable.
E. Thyroid lobe smaller than the distal phalanx of the thumb studied

32) At the reception, patient Z., 36 years old. Height - 180 cm, weight - 95 kg.
Give an opinion on the violation of fat metabolism:

A. Deficiency of body weight


B. Overweight
C. Obesity I degree
D. Obesity II degree
E. Obesity III degree

33) A young man of 18 years old after a cold has thirst, polyuria, weakness.
Blood sugar level is 16mmol / l. What is the patient's syndrome?

A. hypoglycemia syndrome
B. hyperglycemia syndrome
C. thyroid hypofunction syndrome
D. Hypercortisolism syndrome
E. Thyroidhyperfunctionsyndrome

34) Palpation in adults in the normal thyroid gland:

A. soft, agile
B. Dense, welded to underlying tissues.
C. moderate soft, not mobile
D. soft, motionless
E. tight, painful

35) WHO classification of goiter:

A. 0,1,2,3,4 degrees
B. 0,1,2,3 degrees
C. 0,1,2 degrees
D. 1,2,3 degrees
E. 1,2,3,4,5 degrees

36) What changes in the content of thyroid-stimulating hormone,


triiodothyronine and thyroxin are characteristic of hypothyroidism
syndrome?

A. A decrease in serum thyroid-stimulating hormone (TSH) and an increase in


thyroxin (T4) and triiodothyronine (T3) levels
B. Normal levels of TSH and increased serum T3 and T4 levels
C. Elevated levels of TSH and a decrease in serum T3 and T4
D. Normal serum TSH, T3 and T4
E. Elevated serum TSH, T3 and T4
37) Indicate the characteristic electrolyte disturbances in hypocorticism
syndrome:

A. Hypochloremia, hypermagnesemia
B. Hypomagnesaemia, hyperchloremia
C. Hypochloremia, hypomagnesemia
D. Hypernatremia, hypokalemia
E. Hyperkalemia, hyponatremia

38) Atrial fibrillation can occur when:

A. hyperglycemia syndrome
B. hypoglycemia syndrome
C. hyperthyroidism syndrome
D. hypothyroidism syndrome
E. hypocorticism syndrome

39) Features of edema in thyroid hypofunction syndrome:

A. dense, do not leave a fossa


B. leaves a hole for palpation
C. pronounced in the evening on the lower limbs
D. expressed under the morning under the eyes
E. mild, painful

40) General weakness, fatigue, severe muscle weakness, darkening of the skin,
weight loss, loss of appetite, hypotension - these complaints are characteristic
of the syndrome:

A. Hypothyroidism
B. Hyperthyroidism
C. Hypocorticoidism
D. Hypercortisolism
E. Hypopituitarism

41) What syndrome is characterized by "matronism", "bull neck", "marble


skin pattern":
A. hyperglycemia
B. hyperthyroidism
C. Hypocorticoidism
D. hypercortisolism
E. hypothyroidism

42) Pigmentation of the eyelids (upper) with thyroid hyperfunction syndrome


is a symptom:
A. Kocher
B. Moebius
C. Krause
D. Elinek
E. Raceman

43) A white sclera line between the iris and the upper eyelid when looking
down with thyroid hyperfunction syndrome is a symptom:

A. Kocher
B. Shtelvag
C. Grefe
D. Krause
E. Elinek

44) A white sclera line between the iris and upper eyelid when looking up with
thyroid hyperfunction syndrome is a symptom:

A. Kocher
B. Shtelvag
C. Grefe
D. Elinek
E. Raceman

45) The wide opening of the eye slits for thyroid hyperfunction syndrome is a
symptom:

A. Kocher
B. Delrymple
C. Grefe
D. Elinek
E. Shtelvag

46) Enhanced pulsation of the carotid arteries, ascending apical impulse, pulse
110 beats. per minute, arrhythmic or rapid pulse are characteristic of:

A. Hyperglycemia syndrome
B. Hypoglycemia syndrome
C. Thyroid hyperfunction syndrome
D. Thyroid hypofunction syndrome
E. Hypopituitarism syndrome

47) In hyperthyroidism syndrome, the pulse:

A. Voltage, low speed


B. Tense, fast
C. Rare
D. Soft, slow
E. Large, rare
48) What is the amount of anhydrous glucose necessary for the oral glucose
tolerance test in adults?

A. 35 g
B. 50 g
C. 75 g
D. 50 mg
E. 100 mg
49) Indicate laboratory signs of primary hypocorticism syndrome:

A. Increased adrenocorticotropic hormone, decreased cortisol


B. Increased adrenocorticotropic hormone, increased cortisol
C. Decrease in adrenocorticotropic hormone, increased cortisol
D. Lowering adrenocorticotropic hormone, lowering cortisol
E. Adrenocorticotropic hormone is normal, lowering cortisol
50) The patient has hyperglycemia syndrome, after exercise there was a
tremor, palpitations, severe hunger, weakness, anxiety. Your tactics:

A. In/v the jet introduction of 40% glucose


B. I/v administration of glucagon + insulin
C. S/c administration of short-acting insulin
D. S/c administration of insulin ultrashort action
E. Urgent feeding of the patient, give sweet tea
51) It is typical for hyperglycemia syndrome:

A. Polyuria
B. Hypertension
C. Anuria
D. Shortness of breath
E. Oliguria
52) Male 30 years old, height 170 cm, body weight 105 kg, fat deposition
predominates in the trunk, on the skin of the inner surface of the shoulders
and thighs of purple streak, on the face and back multiple acne vulgaris. BP
steadily increased at the level of 150/90 - 165/110 mm Hg. Art. Daily excretion
of cortisol in the urine is higher than normal. Fasting glycemia 6.0 mmol / l,
sugar in urine - neg., The reaction of urine to acetone is negative. Highlight
the main syndrome.

A. Hyperglycemia syndrome
B. Hypoglycemia syndrome
C. Syndrome of chronic insufficiency of the adrenal cortex
D. Hypercortisolism syndrome
E. Overweight syndrome
53) Craving for salty foods is most characteristic of the syndrome:

A. Hypoparathyroidism
B. Hyperparathyroidism
C. Hypercorticism
D. Hypocorticism
E. Hypothyroidism
54) Patient L., 34 years old. Complains of overweight, fatigue. Body weight
increased significantly 5 years ago after childbirth. Menstruation from the age
of 13, regular. Loves flour products, sweets. Father and mother are obese 1-2
tbsp. The younger brother has obesity 1 tbsp. Objectively. Height - 168 cm,
body weight - 96 kg. BMI - 34kg / 2, waist circumference - 95 cm. Hip
circumference - 106 cm. Deposition of subcutaneous fat in the thighs.
Determine the types and degrees of obesity.

A. Android obesity type I


B. Guinea type II obesity
C. Cushingoid type of obesity grade II
D. Android type II obesity
E. Ginoid type of obesity I degree
55) What syndrome is manifested by an increase in the size of the thyroid
gland, bug-eyed, weight loss, tachycardia, increased blood pressure:

A. Hypothyroidism
B. Hyperthyroidism
C. Hypocorticoidism
D. Hypercortisolism
E. Hypopituitarism

56) For what syndrome are dry, wrinkled, cold, thickened skin, anemia,
narrowing of the palpebral fissures, thickening of the lips, tongue,
constipation, bradycardia, hypothermia?

A. Hypothyroidism
B. Hyperthyroidism
C. Pituitary Hyperfunction
D. Hypocorticoidism
E. Hypercortisolism
57) Hypoaldosteronism syndrome is a manifestation of the following
syndrome:

A. Hypercorticism
B. Hypoglycemia
C. Hypocorticism
D. Hyperthyroidism
E. Hypoparathyroidism
58) Under what pathology skin is smooth, warm, tender to the touch with
hyperhidrosis:

A. Hypothyroidism
B. Hyperthyroidism
C. Pituitary Hyperfunction
D. Pituitary hypofunction
E. Hyperglycemia

59) The manifestation of which pathology is mental excitement, imbalance,


quickness of mood changes, constant anxiety:

A. Hypothyroidism
B. Hyperthyroidism
C. Pituitary Hyperfunction
D. Pituitary hypofunction
E. Obesity
60) Patient B., 48 years old. Complaints of swelling of the face, extremities,
chilliness, weakness, memory loss, hoarseness, constipation. Objectively:
height - 170 cm, body weight - 89 years. The skin is dry, pale, flaky. The voice
is low. Pulse - 50 per min., Blood pressure - 110/80 mm Hg. What syndrome
are we talking about?

A. Hyperthyroidism
B. Obesity
C. Hypoglycemia
D. Hypothyroidism
E. Hyperglycemia
61) A young man of 17 years old after severe stress appeared thirst, polyuria.
General weakness, blood sugar level - 20 mmol / l, in urine - 5%, acetone in
urine is positive. Highlight the main syndrome.

A. Hyperglycemia
B. Hyperthyroidism
C. Pituitary Hyperfunction
D. Pituitary hypofunction
E. Chronic adrenal insufficiency

62) Patient 56 years old, height 160 cm, weight 105 kg. No complaints. Fat
distribution is even, especially on the limbs, abdomen, thighs. Borders of the
heart are increased to the left, the deaf tones, systolic murmur at the apex.
Highlight the main syndrome.

A. Hyperglycemia
B. Hypoglycemia
C. Pituitary Hyperfunction
D. pituitary hypofunction
E. Obesity syndrome
63) A patient came to see an endocrinologist. When calculating the BMI was
30.8 kg / m2. Determine the degree of violation of fat metabolism?

A. Overweight
B. Obesity grade II
C. Obesity grade I
D. Grade III obesity
E. Normal body weight
64) Abdominal obesity in women is considered to be a waist circumference
greater than:
A. 76 cm
B. 80 cm
C. 84 cm
D. 88 cm
E. 92 cm
65) Normally, the volume of the thyroid gland (according to ultrasound) in
women is:

A. Up to 16 ml
B. Up to 18 ml
C. More than 20 ml
D. Up to 25 ml
E. Up to 30 ml
66) Normally, the volume of the thyroid gland (according to ultrasound) in
men is:

A. Up to 16 ml
B. Up to 18 ml
C. To 20 ml
D. Up to 25 ml
E. More than 30 ml
67) The most common cause of adrenal insufficiency is:

A. Infections
B. Autoimmune adrenal disease
C. Tuberculosis
D. Syphilis
E. Nephroangiosclerosis
68) What is the characteristic sign of hypercorticismsyndrome:

A. Exophthalmos
B. Matronism
C. Hypogonadism
D. Hypoglycemia
E. Alopecia
69) Patient C., 46 years old, complains of severe weakness, lack of appetite,
weight loss in 6 months per 10 kg, nausea, occasionally loose stools, and at
times decrease in blood pressure. From the anamnesis: ill for six months, the
above symptoms gradually appeared and progressed. Highlight the main
syndrome.
A. Hyperglycemia syndrome
B. Hypoglycemia syndrome
C. Pituitary gland syndrome
D. Syndrome of chronic insufficiency of the adrenal cortex
E. Thyroid hyperfunction syndrome
70) Patient D., 32 years old. Complaints of significant muscle weakness,
dizziness, pain in the heart, emaciation, loss of appetite. History: ill for about
6 months. The disease does not bind to anything. At the age of 26 he suffered
pulmonary tuberculosis. Objectively: height - 176 cm, weight - 60 kg, the skin
on the open parts of the body is pigmented. What syndrome has a patient
developed?

A. hypoglycemia syndrome
B. syndrome of hypofunction of the thyroid gland
C. Hypercortisolism syndrome
D. syndrome hypopituitrism
E. syndrome of chronic insufficiency of the adrenal cortex
71) Patient D., 50 years old, housewife. She complains about weight gain,
flashing of “flies” before her eyes, headaches, weakness. From the anamnesis:
the above complaints began with a change in appearance: the face was
rounded, the belly increased in volume. Objectively: the patient looks much
older than her years. Height-156 cm, weight - 90 kg, on the thighs - wide
purple strii. Determine the pre-syndrome:

A. Hyperglycemia
B. Hypoglycemia
C. Pituitary Hyperfunction
D. Pituitary hypofunction
E. Hypercoticism

72) For what syndrome are typical "moonlike" face, increased hair growth
over the upper lip, in the chin area and on the lateral surfaces of the face. The
skin is dry, with a marble pattern. Hyperpigmentation of the neck. In axillary
areas, on the thighs and lateral surfaces of the abdomen, broad purple-red
bands:

A. Hypothyroidism
B. Hyperthyroidism
C. Pituitary Hyperfunction
D. Hypocorticoidism
E. Hypercortisolism
73) Patient I., 65 years old, complains of chilliness and swelling. Anamnesis is
difficult to collect because of the pronounced loss of memory and inhibition of
the patient. Survey results: T4 free - 0.1 pmol / l; TTG - 69 mMe / l;
Antibodies to TPO - 1000 IU / ml. Highlight the main syndrome.

A. Hypothyroidism
B. Hyperthyroidism
C. Obesity I degree.
D. Hypocorticoidism
E. Hypercortisolism
74) Chilliness, swelling, change in voice, impaired speech, dry skin,
constipation - these complaints are characteristic of the syndrome:

A. Hyperthyroidism
B. Hypocorticoidism
C. Hypercortisolism
D. Hypopituitarism
E. Hypothyroidism
75) What syndrome is manifested by an increase in the size of the thyroid
gland, a puffy, emaciation, tachycardia, an increase in blood pressure?

A. Hypothyroidism
B. Hyperthyroidism
C. Hypocorticoidism
D. Hypercortisolism
E. Hypopituitism
76) Patient Sh., 47 years old, for four years addressed various specialists with
complaints of weakness in the limbs, persistent pain in the calf muscles and
back. Osteoporosis, cysts and pathological fractures were found on
radiographs of the bones. Specify the syndrome.

A. Hypoparathyroidism
B. Thyrotoxicosis
C. Hyperparathyroidism
D. Hypoglycemia
E. Hypothyroidism
77) The patient after strumectomy had convulsions and a positive symptom of
Chvostek. Highlight the main syndrome.
A. Obesity syndrome
B. Thyrotoxicosis syndrome
C. Anorexia syndrome
D. Hypoparathyroidism syndrome
E. Hyperparathyroidism syndrome
78) Thin arms and legs, excessive deposition of fat on the face and torso,
“moon-shaped” hyperemic face occur with the syndrome:

A. Hypocorticoidism
B. Hypercortisolism
C. Hypopituitarism
D. Hypothyroidism
E. Hyperthyroidism
79) A 40-year-old patient complains of chilliness, drowsiness, constipation, dry
skin. An objective study found an enlarged thyroid gland. To confirm this
syndrome the most informative:

A. Determination of the level of Ca2 + in the blood


B. Determination of the level of TSH, sv. T4
C. Ultrasound of the thyroid gland
D. Determination of blood cholesterol
E. Definition of T3
80) Patient Z., turned to the local doctor with complaints of a change in skin
color and severe muscle weakness. From the anamnesis it is known that 6
years ago the patient suffered a tuberculous lesion of the lungs. Objectively:
low nutrition, depigmentation of open areas of the body.

What research is needed?

A. Free thyroxine, thyroid stimulating hormone


B. Cortisol, adrenocorticotropic hormone
C. Fasting plasma glucose, glycohemoglobin
D. Parathyroid hormone, ionized calcium
E. Progesterone, luteinizing hormone
81) Ultrasound of the thyroid gland: right lobe: length - 6.8 cm, width - 3.0
cm, thickness - 1.8 cm; left lobe: length - 6.0 cm, width - 2.0 cm, thickness - 1.9
cm. Determine the volume of the thyroid gland.

A. 14.0 ml
B. 18.0 ml
C. 18.4 ml
D. 28.4 ml
E. 40.0 ml

Urinary System
1. What are the causes of edema in nephrotic syndrome?
A. Kidney ischemia;
B. Oliguria;
C. Decreased blood pressure;
D. Reduction of oncotic pressure;
E. Increased hydrostatic pressure.

2. In the study of blood in patients with nephrotic syndrome revealed:


A. Hyperproteinaemia;
B. Hypoalbuminemia;
C. Hyperalbuminemia;
D. Hypolipidemia;
E. Hyperglycemia.

3. Signs of nephrotic syndrome include:


A. Arterial hypertension;
B. Massive swelling;
C. hematuria;
D. Proteinuria> 1.5 g / day;
E. Leukocyturia.

4. The main causes of massive proteinuria in nephrotic syndrome are:


A. Impaired reabsorption;
B. Loss of negative charge by the membrane and podocytes;
C. The proliferation of mesangium;
D. rupture of the membrane;
E. Proliferation of the membrane.

5. What is not included in the classic triad of symptoms in acute nephritis


syndrome?
A. Edema;
B. Anuria;
C. Arterial hypertension;
D. Hematuria;
E. Piuria.

6. Symptoms of urinary syndrome in acute nephritis syndrome:


A. Leukocyturia;
B. Proteinuria;
C. Bens Jones protein;
D. Bacteriuria;
E. Chiluria.

7. What proteinuria is characteristic of acute-nephritic syndrome?


A. Glomerular;
B. Tubular;
C. Orthostatic;
D. Proteinuria voltage;
E.proteinuria overflow.

8. What test can detect a decrease in glomerular filtration?


A. Sample Nechyporenko;
B. Sample Zimnitsky;
C. Sample Reberga-Tareeva;
D. Addis – Kakowski test;
E. Pasternatsky's test.
9. What signs are typical of Acute-Nephritic Syndrome?
A. Polyuria;
B. Oliguria;
C. Fever;
D. Hematuria;
E. Bacteriuria.

10. Nephrotic syndrome characteristic:


A. Increase in serum VLDL and LDL;
B. Development of hypocoagulation disorders;
C. Hypercalcemia;
D. Convulsive syndrome;
E. Arterial hypertension.

11. Characteristic hematological disorders in chronic renal failure:


A. Anemia;
B. Erythrocytosis;
C. Leukocytosis;
D. Thrombocytosis;
E.Pantsiopeniya.

12. Normal level of urea in blood serum:


A. 150-250 mmol / l;
B. 2.5-3.5 mmol / l;
C. 3.5-8.6 mmol / l;
D. 25-50 mmol / l;
E.40-50 mol / l.

13.Normal glomerular filtration rate:


A. 5-15 ml / min;
B. 50-80 ml / min;
C. 80-120 ml / min;
D. 100-200 ml / min;
E.15-30 ml / min.

14. Hyperkalemia in chronic renal failure occurs when GFR drops below:
A. 80 ml / min;
B. 25 ml / min;
C. 15 ml / min;
D. 60 ml / min;
E.10 ml / min.

15. Anephrotic syndrome characterize:


A. Arterial hypertension, edema, proteinuria, hematuria;
B. Arterial hypertension, hypercholesterolemia;
C. edema, hypo-and dysproteinemia, hypercholesterolemia;
D. massive proteinuria, hypo-and dysproteinemia;
E.Proteinuria, bacteriuria, fever.

16. Daily proteinuria in nephrotic syndrome is:


A. less than 50 mg;
B. 1 g;
C. less than 3.0 g;
D. more than 3.5 g;
E. more than 10 g.

17. The diagnostic criteria for nephrotic syndrome are all. except:
A. proteinuria more than 3.5 grams per day;
B. arterial hypertension;
C. hypoproteinemia, edema;
D. hyperlipidemia;
E. hypercoagulation.
18. The clinical manifestations of faciesnefritica are:
A. pallor and swelling of the face;
B. acrocyanosis;
C. hemorrhagic rash on the face;
D. swelling of the neck veins;
E. Xanthelasma on the face.

19. Which signs carry information about the functional abilities of the kidney?
A. urine protein content
B. isostenuria;
C. glomerular filtration;
D. increased plasma creatinine;
E.High blood pressure.

20. Changes in biochemical parameters in chronic renal failure:


A. hyperalbuminemia;
B. dyslipidemia;
C. hypercreatininemia;
D. urobilinuria;
E. hyperglycemia.

21. What determines the dark and yellowish color of the skin in chronic renal
failure?
A. from increasing direct bilirubin;
B. from increasing indirect bilirubin;
C. from the violation of the allocation of urochromes;
D. for violation of bilirubin secretion;
E.from violation of the synthesis of bilirubin.

22. Nephrotic syndrome is characterized by all symptoms except:


A. hypoalbuminemia;
B. proteinuria;
C. swelling;
D. hypertension;
E. hyperlipidemia.

23. The structural and functional unit of the renal tissue is:
A. calyx;
B. juxtaglomerular apparatus;
C. pyramid;
D. nephron;
E. Loop of Henle.

24. Non-selective proteinurization testifies:


A. about the violation of reabsorption;
B. on the breach of the filtration properties of the glomerulus;
C. about the inflammatory process in interstitial tissue;
D. on dystrophic processes in the tubules;
E. violation of protein metabolism in the body.

25. The following method is used to determine glomerular filtration.:


A. determination of diurnal diuresis;
B. urine study according to Zimnitsky;
C. definition of clearance for endogenous kreatinin;
D. Nechiporenko test;
E. determination of daily proteuria.
26. Nitrogenation of the kidneys can be judged.:
A. according to the plasma protein level;
B. by urine protein level;
C. on the level of urea and plasma creatinine;
D. on the relative density of urine;
E. on the level of blood lipids.

27. The simplest method of studying kidney concentration function:


A.Nechyporenko test;
B. Ambuje test;
C. Zimnitsky test;
D. Reberg-Tareev test;
E. three-glass test.

28. The main mechanisms of development of renal hypertension:


A. loss of plasma albumin;
B. activation of the renin system and enhancement of aldosterone production;
C. impairment of kidney filtration function;
D. impaired renal endocrine function;
E. Impaired renal secretory function;

29. Specify a symptom that is not typical for acute pyelonephritis:


A. Acute onset;
B. Oliguria;
C. Reduced kidney size;
D. Hyperkalemia;
E. pH shift.

30. Which of the proposed criteria is essential for the diagnosis


nephrotic syndrome?
A. Edema;
B. Serum albumin exceeds 30 g / l;
C. Daily proteinuria more than 3.5 g;
D. Hypocholesterolemia;
E. Hyperglycemia.

31. Acute nephritic syndrome:


A. Hematuria, edema, hypertension;
B. Hypercholesterolemia;
C. The development of renal failure;
D. Leukocyturia, bacteriuria;
E. Development of anuria.

32. Urinary syndrome - a clinical laboratory concept that includes:


A. Proteinuria;
B. Bacteriuria;
C. Ciliduria;
D. Leukocyturia;
E. All correct.

33. Criteria for determining chronic renal failure:


A. GFR <100 ml / min / 1.73 m2 for> 3 months;
B. GFR <60 ml / min / 1.73 m2 for> 3 months;
C. GFR <30 ml / min / 1.73 m2 for> 3 months;
D. Kidney damage for> 3 months;
E. Kidney damage for <3 months.

34. Specify an early sign of chronic renal failure:


A. Metabolic acidosis;
B. Hypercalcemia;
C. Nocturia;
D. Reducing the size of the kidneys;
E. Increased kidney size.

35. In the pathogenesis of acute renal damage emit:


A. Ischemic;
B. Tubulo-interstitial;
C. Prerenal, renal, postrenal;
D. Toxic;
E. All right.

36. With non-selective proteinuria, one should think about:


A. About nephrotic syndrome;
B. Focal or diffuse violation of the integrity of the basement membrane;
C. Acute nephrotic syndrome;
D. Impaired tubular reabsorption;
E. On the lesion of interstitial tissue.

37. In the differential diagnosis of ARF and CKD matters:


A. Level of diuresis;
B. X-ray examination of the kidneys;
C. Anamnesis, creatinine level;
D. kidney ultrasound;
E. Patient's age.

38. What signs are typical of Acute-Nephritic Syndrome?


A. Polyuria;
B. Oliguria;
C. Fever;
D.Hematuria;
E. Hypotension.

39. For terminal stage of chronic kidney disease characteristic:


A. Glomerular filtration less than 15 ml / min;
B. Oliguria;
C. Hypokalemia;
D. Hypercalcemia;
E. Increasing the size of the kidneys on ultrasound.

40. Normal levels of potassium in serum:


A. 3.5-5.5 mmol / l;
B. 2.5-7.5 mmol / l;
C. 5.5-8.0 mmol / l;
D. 1.5-3.5 mmol / l;
E. 4.5 - 5.5 mmol / l.

41. Normal serum sodium levels:


A. 5.5-7.5 mmol / l;
B. 135-140 mmol / l;
C. 100-200 mmol / l;
D. 12.5-17.0 mmol / l;
E. 155-160 mmol / l.

42. Normal serum creatinine level:


A. 200-450 μmol / l;
B. 5-15 µmol / L;
C. 44-105 μmol / l;
D. 130-140 μmol / L;
E. 218-618 µmol / l.
43. Normal level of urea in blood serum:
A. 150-250 mmol / l;
B. 2.5-3.5 mmol / l;
C. 3.5-8.6 mmol / l;
D. 25-50 mmol / l;
E. 10-15 mmol / l.

44. Normal glomerular filtration rate:


A. 5-15 ml / min;
B. 50-80 ml / min;
C. 80-120 ml / min;
D. 100-200 ml / min;
E. 60-80 ml / min.

45. The serum creatinine level is 200 µmol / lcorresponds to:


A. Terminal stage of chronic pyronephritis;
B. Reduction of glomerular filtration by 50%;
C. Glomerular filtration 15 ml / min;
D. Glomerular filtration 5 ml / min;
E. Reduction of glomerular filtration by 75%.

46. Not typical for nephrotic syndrome:


A. severe proteinuria;
B. hypoproteinemia;
C. hyperlipidemia;
D. hematuria;
E. hypercoagulation.

47. The following method is used to determine the glomerular filtration rate.:
A. determination of diurnal diuresis;
B. urine study according to Zimnitsky;
C. definition of clearance for endogenous kreatinin;
D. urine test according to Nechyporenko;
E. Use of test strips for microalbuminuria.
48. Nitrogenation of the kidneys can be judged:
A. at the plasma protein level;
B. by urine protein level;
C. at the level of urea and plasma kreatinin;
D. on blood plasma cholesterol levels;
E. according to the level of daily diuresis.

49. Thenormal leukocyte counts in the urine analysis according to Nechyporenko:


A. up to 6000 in 1 ml;
B. up to 2000 in 1 ml;
C. up to 4000 in 1 ml;
D. up to 1000 in 1 ml;
E. up to 5000 in 1 ml.

50. Thenormal erythrocytes counts in urine analysis according to Nechipopenko:


A. 1000 in 1 ml;
B. 2000 in 1 ml;
C. 4000 in 1 ml;
D. 6000 in 1 ml;
E. 10,000 in 1 ml;

51. Most accurately the nature of the morphological changes in the kidneys allows
you to determine:
A. intravenous hepatitis;
B. retrospiography;
C. ultrasound study;
D. renal biopsy;
E. Doppler sonography of the kidneys.

52. What level of proteinuria is moderate?


A. 0.05 - 1.0 g / day;
B. 1.0 –3.5 g / day;
C. 3.5-5 g / day;
D. over 5 g / day;
E. over 10 g / day.

53. What level of proteinuria is pronounced?


A.0.05 - 1.0 g / day;
B.1.0–3.5 g / day;
C. 3.5-5 g / day;
D. more than 5 g / day;
E. Over 10 g / day.

54. What level of proteinuria is nephrotic?


A.0.05 - 1 g / day;
B.1.0 –2.5 g / day;
C.2.5-3.0g / day;
D. 3.5-5 g / day;
E. less than 1 g / day.

55. If there is a number of red blood cells in the general analysis of urine, the fact
of hematuria is established?
A. 0 –1 in sight;
V. 1-2 in sight;
S. 2 - 0 in sight;
D. 3 and more in sight;
E. 3 and less in sight.

56. In the presence of what quantity of leukocytes in the general analysis of urine
is the fact of leukocyturia established?
A. 1–2 in sight
B. 2-3 in sight
Pp. 3-5 in sight
D. 5 –6 in sight
E. 0-1 in sight

57. If there is a number of red blood cells in the general analysis of urine is
established gross hematuria fact?
A. 1–2 in sight;
B. 2-3 in sight;
S. 5 and more in sight;
D. urine "color meat slop";
E. 0-1 in sight.

58. Which of the following pathogens is most often the cause of the development
of acute nephritic syndrome?
A. E. coli;
B. Viruses;
C. B-hemolytic streptococcus group A;
D. Chlamydia and other intermediate forms of bacteria;
E. The simplest.

59. What disease is accompanied by an increase in the kidneys, despite the


deterioration of renal functions?
A. amyloidosis, polycystic;
B. nephrosclerosis;
C. glomerulonephritis;
D. chronic pyelonephritis;
E. Diabetic nephropathy.
Hematopoietic system

1.The normal content of iron in serum is:

A. 2-10 µmol / l

B. 6-15 µmol / l

C. 10-28 μmol / l

D. 12-30 µmol / l

E. 15-40 µmol / l

2.Normal blood levels of Hemoglobin in adults are:

A. 90-140 g / l

B. 100-150 g / l

C. 110-160 g / l

D. 120-170 g / l

E. 130-180 g / l

3.Anemic syndrome manifested:

A. perversion of taste

B. dry skin

C. causeless general weakness

D. hair loss

E. deformation of nails

4.Hyposiderosis is characterized by:


A. perversion of smell

B. dizziness

C. causeless general weakness

D. palpitations

E. shortness of breath

5. In what cases is the development of hyposiderosis possible?

A. hemolysis of red blood cells

B. folic acid deficiency

C. Vitamin B12 Deficiency

D. chronic foci of blood loss

E. Bone marrow aplasia

6.Body containing the main reserves of iron in the body:

A. liver

B. Spleen

C. kidney

D. brain

E. lungs

7. What research is needed to diagnose hypochromic anemia?

A. serum bilirubin

B. serum creatinine

C. definition of revisions

D. serum ferritin
E. determination of blood sugar

8. Iron deficiency is characterized by:

A. megaloblastic type of blood formation

B. hypochromia of erythrocytes

C. normochromia of erythrocytes

D. pancytopenia

E. hyperbilirubinemia

9. The normal content of ferritin in the serum is:

A. 10 - 12 µg / l

B. 80 - 350 µg / l

C. 10 - 30 μg / l

D. 15 - 300 µg / l

E. 5 - 175 µg / l

10.Erythrocyticpoikilocytosis is:

A. change in the average diameter of erythrocytes

B. change in the form of red blood cells

C. red blood cells with basophilic point

D. Jolly red blood cells

E. Erythrocytes with kebot rings

11. Erythrocyticanisocytosis is:

A. red blood cells with pathological inclusions

B. target red blood cells


C. red blood cell hyperchromia

D. oval red blood cells

E. Presence in the blood of erythrocytes of various sizes

12.Organ, containing vitamin B12 in the body:

A. brain

B. adrenal glands

C. liver

D. kidneys

E. spleen

13.The absorption of vitamin B12- and folic acid occurs mainly:

A. in the stomach

B. in the duodenum

C. in the upper small intestine

D. in the colon

E. in the lower small intestine

14.What is the reason for the development of funicular myelosis syndrome?

A. thymidine deficiency

B. Formation of methylmalonic acid

C. folic acid deficiency

D. Ascorbic acid deficiency

E. Uridine deficiency

15.Macrocytosis and hyperchromia of erythrocytes caused by:


A. hyposiderosis

B. folic acid deficiency

C. hemosiderosis

D. red blood cell hemolysis

E. erythrocytosis

16.Vitamin B12 deficiency leads to the formation of which red blood cells?

A. normocytes

B. microcytes

C. macrocyte

D. microspherocytes

E. sickle cell red blood cells

17.Hemolytic syndrome manifests is:

A. pale skin

B. dry skin

C. yellowness of the skin

D. leukemia on the skin

E. trophic ulcers on the skin

18.Erythrocyte hemolysis results in:

A. direct hyperbilirubinemia

B. indirect hyperbilirubinemia

C. hyperbilirubinemia of both fractions

D. Hypobilubinemia

E. abilirubinemia
19.Hemolysis of erythrocytes due to the formation of anti-erythrocyte
antibodies is confirmed by:

A. direct breakdown of Coombs

B. indirect Coombs breakdown

C. direct sucrose breakdown

D. hemosiderin urine

E. hemoglobinuria

20.Dysmetabolic degeneration of the lateral and partially posterior part of the


spinal cord with vitamin B12 deficiency leads to the development of:

A. anemic syndrome

B. neuro-anemic syndrome

C. Sideropenic syndrome

D. hemolytic syndrome

E. neuroleukemia

21.What is the cause of reticulocytosis?

A. tissue hypoxia

B. Hyposiderosis

C. erythrocyte hemolysis

D. hemosiderosis

E. erythrocytosis

22.Areticulocythemia develops when:

A. the presence of antibodies against bone marrow erythrocaryocytes


B. the presence of antibodies against peripheral blood erythrocytes

C. fermentopathy of erythrocytes

D. hemoglobinopathies

E. erythrocyte membranopathyes

23.The cause of red blood cell microcytosis:

A. increased red blood cell sequestration in the spleen

B. excess of “old” red blood cells

C. increasing the number of young red blood cells

D. Reduction of hemoglobin content in the erythrocyte

E. intravascular hemolysis of erythrocytes

24.When examining the patient revealed pancytopenia,Your actions in the


survey plan:

A. determination of ferritin

B. direct Coombs test

C. liver tests

D. Creatinine and other kidney tests

E. bone marrow examination

25.Syndrome of the funicular myelosis appears:

A. violation of tactile and pain sensitivity

B. general weakness

C. hyperplasia of lymphatic tissue

D. perversion of smell

E. dysphagia
26.The normal content of platelets in the blood is:

A. 100-200х109 / l

B. 150-300x109 / l

C. 170-350х109 / l

D. 150-400x109 / l

E. 150-450х109 / l

27.Platelet ancestors are:

A. reticulocyte

B. megakaryocytes

C. myelokaryocytes

D. oxyphilicnormocytes

E. polychromatophilicnormocytes

28.Megakaryocytes are precursors to:

A. reticulocyte

B. neutrophils

C. lymphocyte

D. platelet

E. erythrocyte

29.The life span of platelets is:

A. 12-24 hours
B. 24-48 hours

C. 3-5 days

D. 8-11 days

E. 15-20 days

30.When thrombocytopenia develops:

A. petechial-bruise type of bleeding

B. hematoma type of bleeding

C. vasculitic-purple type of bleeding

D. petechial hematoma type of bleeding

E. angiomatous type of bleeding

31.Choose a type of bleeding with a deficiency of coagulation factors VIII and


IX?

A. petechial-bruising type of bleeding

B. vasculitis-purple type of bleeding

C. hematoma type of bleeding

D. Angiomatous type of bleeding

E. petechial hematoma type of bleeding

32.Vasculitis-purple type of bleeding due to:

A. thrombocytopenia

B. thrombocytopathy

C. pathology of capillary endotheliocytes

D. congenital inferiority of capillary walls

E. deficiency of coagulation factors VIII and IX


33.Specify the type of bleeding with a deficiency of von Willebrand factor?

A. petechial-bruising type of bleeding

B. vasculitis-purple type of bleeding

C. hematoma type of bleeding

D. petechial hematoma type of bleeding

E. Angiomatous type of bleeding

34. Congenital inferiority of the capillary walls with the expansion of the
lumen of blood vessels leads to the development of:

A. vasculitic-purple type of bleeding

B. angiomatous type of bleeding

C. hematoma type of bleeding

D. petechial-Sinyachkov type of bleeding

E. petechial-hematoma type of bleeding

35. Choose a critical thrombocytopenia level?

A. 50x109 / l

V. 40x109 / l

S. 30x109 / l

D. 20x109 / l

E. 10x109 / l

36. Platelets are formed from one megakaryocyte:

A. 1000-2000

B. 2000-4000

C. 3000-6000
D. 4000-8000

E. 5000-10000

37.Hemorrhagic syndrome in aplasia of the blood due to:

A. anomalous structure of vessel walls

B. Increased destruction of platelets in the blood

C. deficiency of blood coagulation factors

D. increased platelet consumption

E. insufficient formation of megakaryocytes

38. Which of the laboratory signs is pathognomic for the deficiency of


coagulation factors?

A. prolonged bleeding time

V. decrease in the number of platelets

C. lengthening of blood clotting time

D. Platelet dysfunction

E. reducing the number of megakaryocytes

39.In what type of bleeding are intermuscular, subcutaneous, retroperitoneal


hematomas observed?

A. vasculitic purple

B. angiomatous

C. hematomic

D. Microcirculatory

E. petechial-bruise
40.Extending of the activated partialthromboplastin time (APTT) is observed
with:

A. petechial-bruising type of bleeding

B. vasculitic-purple type of bleeding

C. hematoma type of bleeding

D. Angiomatous type of bleeding

E. microcirculatory type of bleeding

41. Lengthening the duration of bleeding time by Duke occurs when:

A. thrombocytopenia

B. Intravascular thrombosis

C. congenital coagulopathy

D. Acquired coagulopathy

E. intravascular hemolysis of erythrocytes

42.Platelet dysfunction:

A. thrombocytopenia

B. thrombocytopathy

C. angiopathy

D. coagulopathy

E. Vasopathy

43.The first stage of coagulation hemostasis ends with the formation of:

A. active prothrombinase

B. thrombin
C. plasmin

D. fibrin

E. prothrombin

44.The second stage of coagulation hemostasis ends with the formation of:

A. active prothrombinase

B. prothrombin

S. thrombin

D. fibrin

E. plasmin

45.Physiological anticoagulant is:

A. plasminogen

B. plasmin

C. trombomodulin

D. Kallikrein

E. von Willebrand factor

46.Hyperplastic syndrome:

A. hypoplasia of lymphoid tissue

B. hyperplasia of lymphoid tissue

C. hypoplasia of myeloid tissue

D. hyperplasia of myeloid tissue

E. aplasia of myeloid tissue

47. What is the mechanism of development of hyperplastic syndrome?


A. hypoplasia of lymphoid tissue

B. hypoplasia of myeloid tissue

C. hyperplasia of lymphoid tissue

D. hyperplasia of myeloid tissue

E. aplasia of myeloid tissue

48. What are the precursor cells of plasma cells?

A. Monocytes

B. T-lymphocytes

C. B lymphocytes

D. neutrophils

E. eosinophils

49. The spleen is located at the level of:

A. VIII-X ribs

B. IX-XI ribs

C. X-XII ribs

D. XI-XII ribs

E. VIII-XII ribs

50.Spleen is covered with peritoneum:

A. on the one hand

B. from two sides

C. from three sides

D. all around

E. not covered on one side


51.Bone marrow is located in:

A. in the diaphysis of the bones

B. in bone metaphysis

C. cells of spongy bone

D. in compact bone matter

E. in the periosteum

52. What applies to the central organs of the hematopoietic system?

A. lymph nodes

B. spleen

C. thymus gland

D. liver

E. lymphocytes

53.What are the cells Botkin, basket cells?

A. abnormal monocytes

B. hemolized red blood cells

C. polysegmented neutrophils

D. destroyed lymphocytes

E. activated platelets

54. Select an organ containing red and white pulp?

A. lymph node

B. thymus

C. spleen
D. bone marrow

E. liver

55.Hypersplenism:

A. splenomegaly + cytopenia

B. hepatosplenomegaly

C. splenomegaly + lymphadenopathy

D. Splenomegaly + Erythrocytosis

E. splenomegaly + viral infection

56. Patient I., 34 years old, complaints of general weakness, fatigue, dizziness,
headaches, shortness of breath and palpitations with little exertion, from the
anamnesis: heavy, prolonged menstrual bleeding.
What syndrome can you think of?

A. sideropenic syndrome

B. hemolytic syndrome

C. anemic syndrome

D. hemorrhagic syndrome

E. hyperplastic syndrome

57. Patient S., 28 years old, went to a doctor's office with complaints of taste
perversion, hair loss, brittle nails, difficulty in swallowing food, dry skin.

CBC: HB 128 g / l, erythr. 4,7 h1012 / l, MCV 62 fl, MCH 18 pg.

What syndrome are we talking about?

A. anemic syndrome

B. hemolytic syndrome

C. funicular myelosis
D. Sideropenic syndrome

E. hyperplastic syndrome

58. Patient J., 42 years old, appealed to the outpatient clinic. Complaints when
going to the doctor: yellowness of the skin and visible mucous membranes, change
in color of urine as a strongly brewed tea, feeling of heaviness in hypochondria,
occasionally itchy skin.
Total bilirubin 78.4 µmol / l, mainly due to the indirect fraction.

What syndrome does the patient have?

A. anemic syndrome

B. sideropenic syndrome

C. funicular syndrome

D. hemorrhagic syndrome

E. hemolytic syndrome

59. Patient D., 19 years old, was found to have: HB 58 g / l, eryth.3.7x10 12 / l,


MCV 58 fl, MCH 15 pg, thromb.325.0x109 / l, leuk.4.3h109 / l.

What changes are there in the total blood count?

A. normocytic, normochromic anemia

B. macrocytic, hyperchromic anemia

C. microcytic, hypochromic anemia

D. megalocytic, hyperchromic anemia

E. normocytic, hypochromic anemia

60. Patient S., 70 years old, complains of general weakness, fatigue, shortness of
breath and palpitations at the slightest physical exertion, unsteadiness of gait,
feeling of crawls all over the body, change of color of urine.
CBC: HB 62 g / l, erytr.1.7x1012 / l, MCV 118 fl, MCH 36pg, thromb. 155.0x10 9 /
l, leuk.3.9x109 / l.

What changes are there in the total blood count?

A. normocytic, normochromic anemia

B. macrocytic, hyperchromic anemia

C. microcytic, hypochromic anemia

D. normocytic, hyperchromic anemia

E. normocytic, hypochromic anemia

61. Patient A., 26 years old, was admitted to the hospital with complaints of
marked general weakness, fatigue, headaches, dizziness, blurred vision, shortness
of breath and palpitations at the slightest physical exertion, yellowness of the skin,
feeling of heaviness and pain in the left hypochondrium, change in urine color .

CBC: erythrocytes 1.6x1012 / l, Hb 48 g / l, MCV 92 fl, MCH 31 pg, platelets


187.4x109 / l, leukocytes 8.6x109 / l: fell. 6, segm.72, mon. 6, lymph 16, ESR 32
mm / hour.

What is determined by the general blood test?

A. normocytic, normochromic anemia

B. macrocytic, hyperchromic anemia

C. microcytic, hypochromic anemia

D. normocytic, hyperchromic anemia

E. normocytic, hypochromic anemia

62. Patient I., 16 years old, during the study found: peripheral blood platelet count
20.0x109 / l; the number of bone marrow megakaryocytes in 1 μl of 185 cells.

What type of bleeding can a patient develop?

A. petechial-bruise

B. vasculitic purple
C. hematoma

D. angiomatous

E. petechial hematoma

63. Patient I., 17 years old, was admitted to the clinic with complaints of nasal
bleeding, pain in the right knee joint, aggravated by the slightest movement.
History: ill from early childhood.
CBC: erythrocytes 4,6x1012 / l, Hb -148 g / l, MCV 64 fl, MCH 30pg, platelets
410x109/l, leukocytes 4,6x109 / l: fell. 6, segm.72, mon. 6, lymph 16, ESR 12 mm /
hour.

What type of hemorrhagic syndrome does the patient have?

A. petechial-bruise

B. vasculitic purple

C. hematoma

D. angiomatous

E. petechial hematoma

64. Patient K., 16 years old, was admitted to the hospital with complaints of pain
and a sharp restriction of movement in the right knee joint. Joint pain intensifies
with the slightest movement. Anamnesis: sick from early childhood.

TBC: red blood cells 4.6 × 1012 / l, HB 148 g / l, MCV 64 fl, MCH 30pg, platelets
410 × 109 / l, white blood cells 4.6 × 109 / l: pal. 6, seg.72, mon. 6, lymph.16, ESR
12 mm / hour. What research needs to be done to clarify the type of bleeding?

A. definition of von Willebrand factor

B. determination of coagulation factors VIII, IX

C. platelet aggregogram

D. determination of the titer of antiplatelet antibodies

E. Determination of anti-erythrocyte antibody titer


65. Patient I., 26 years old, came to the clinic. According to the patient, it became
clear that during the last 6 months she began to notice the appearance of petechial
rashes on the extremities, bruises after minor injuries, an increase in the duration of
menstrual bleeding to 10-12 days.

What research is shown?

A. blood coagulation factors VIII, IX

B. APTT

C. platelet count, megakaryocytes

D. Prothrombin time, INR

E. coagulation factors I, VII

66. Patient Z., 26 years old, complaints of an increase in cervical, submandibular,


axillary, inguinal lymph nodes, a feeling of heaviness and pain in the right and left
hypochondria.
CBC: HB 126 g / l, erytr. 3,9x1012 / l, thromb. 176.0 x109 / l,leuk-26.2 x109 / l: neut
- 38, eosin. 2, lymph.- 60, ESR 24 mm / hour.
What is the hematopoietic lesion syndrome in a patient?

A. hemorrhagic

B. hemolytic

C. necrotic ulcer

D. Funicular myeosis

E. hyperplastic

67. Patient K., 35 years old, complaints of a feeling of heaviness and pain in the
left hypochondrium, aggravated by physical activity, after eating, and general
weakness.
Objectively: The general condition is severe. Peripheral lymph nodes are not
palpable. The tongue is coated with white bloom, dry. The abdomen is enlarged,
asymmetric due to the enlarged spleen. The lower edge of the spleen at the
entrance to the pelvis, extends beyond the median line by 3-4 cm, dense, flat
surface, sensitive to palpation. The liver is not palpable.
What syndrome are we talking about?

A. hyperplastic

B. hepatomegaly

C. splenomegaly

D. hepatolienal

E. hepatorenal

68. A patient with lymph proliferative disease has generalized lymphadenopathy.

What blood cells can be involved in this case?

A. neutrophils

B. monocytes

C. lymphocytes

D. eosinophils

E. basophils

69. The patient has generalized lymphadenopathy and splenomegaly. In a general


blood test, there is leukocytosis. What blood cells can be enlarged?

A. stab and segmented neutrophils

B. reticulocytes, red blood cells

C. eosinophils, basophils

D. Monocytes, lymphocytes

E. Platelets and platelet aggregates


70. Patient K., 67 years old, was diagnosed with peripheral blood: HB-138 g / l,
erythrocytes-4,1x1012 / l, MCV 84 fl, MCH 30 pg, platelets-216,8x10 9 / l,
leukocytes-6,4x109 / l: neut 24, lymph. 70, mon. 6, ESR 15 mm / hour.
What changes are there in the total blood count?

A. erythrocytosis

B.thrombocytosis

C. leukocytosis

D. Lymphocytosis

E. monocytosis

71. Patient B., 43 years old, in a general blood test had the following data: HB-148
g / l, red blood cells-4.3 × 1012 / l, MCV 84 fl, MCH 30 pg, platelets-495.0 × 109 /
l, white blood cells-6,4x109 / l: stick 3, segment 54%, lymph 36%, eosin 2,
mon.5%, ESR 12 mm / hour. What changes are available in the general blood test?

A. erythrocytosis

B. thrombocytosis

C. leukocytosis

D. lymphocytosis

E. monocytosis
Skeletal muscle system

1. Diffuse alopecia is:

A. Increased hair breakage

B. Dryness and section of hair

C. Uniform hair loss

D. Local alopecia

E. Excessive hair growth

2. A 56 years old patient.Complainsofpainintheknee, and


anklejoints,worsening when walking. Objectively: right kneejoint swollen,
crepitationisheardwhenmovingthejoints.
Whichindexorscalecanbeusedtoassesstheseverityofpaininthejoints?

A. BASDAI

B. SLICC

C. DAS-28

D. VAS

E. SELENA

3. Theformulaforcalculatingtheactivityindexof DAS28 syndrome in


inflammatoryjointdiseaseis:

A. DAS28 = TJC28 + SJC28 + 0,7 ln (ESR) + 0,014GH

B. DAS28 = √ TJC 28 + √ SJC 28 + 0.7 ln (ESR) + 0.014 GH

C. DAS28 = TJC 28 + SJC 28 + ln (ESR) + GH

D. DAS28 = 0.56√ TJC 28 + 0.28√ SJC 28 + 0.7 ln (ESR) + 0.014 GH

E. DAS28 = 0.56√ TJC 28 + 0.28√ SJC 28 + 0.7√ (ESR + GH)


4. Sjogren syndrome is:

A. "dry" syndrome due to dehydration

B. "dry" syndrome due to lesions of the endocrine glands

C. "dry" syndrome due to lesions of the exocrine glands

D. “dry” syndrome associated with dry skin eczema

E. “dry” syndrome due to photosensitivity

5. Raynaud's syndrome is:

A. Episodes of limb arteriole constriction, manifested by whitening and


cyanosis of the fingers

B. Numbness of fingers

C. Redness of the fingertips after changing the temperature

D. The appearance of ulcers in the periungual bed

E. The appearance of areas of skin atrophy at the tips of the fingers of the hands
and feet

6. DIC syndrome stands for:

A. Degenerative intraarticular syndrome

B. Disseminated intravascular coagulation syndrome

C. Diffuse vegetative-vascular syndrome

D. Diffuse intravascular sclerosis

E. Demyelinating intracranial sclerosing syndrome


7. Patient 75 yearsold, admittedtothedepartmentofrheumatology.
Complainsofpaininthekneejoints, whichintensifyintheevening. Fromthepatient
history: themotherhasdiseasesofthemusculoskeletalsystem. Had
repeatedinjuriesofthekneejoint. Objectively: a patientis ofincreasednutrition,
BMI 35 kg / m2. Locally: ? intheinterphalangealjointsofthehands,
Heberdennodules, varusdeformityofthekneejointsaredetermined. Thereis a
crunchinthekneejoints. CBCC, Urine analysis: nochange. What is the name of
syndrome present in this patient?

A. diffuseconnectivetissuedisease

B. Syndromeofinflammatorymusclechanges

C. degenerativejointdiseasesyndrome

D. Syndromeofsoftperiarticulartissuelesion

E. Syndromeofinflammatoryjointchanges
8. Antiphospholipid syndrome occurs in:

A. Degenerative lesions of the joints

B. Inflammatory lesions of the joints

C. Diffuse lesions of connective tissue syndrome

D. Syndrome of soft tissue lesions around the articular tissues

E. Osteopenic syndrome

9. Viscerites are:

A. Pathognomonic symptomacomplexes, characteristic of lesions within the


same system

B. Concurrent lesions of internal organs

C. Complications of drug therapy for visceral organs

D. Affection of the peritoneum in the syndrome of diffuse lesion of the


connective tissue

E. Syndromes caused by damage to internal organs

10. Viscerite includes:

A. Lymphadenitis

B. Onychodystrophy

C. Cardit

D. Alopecia

E. Arteritis
11. Of the extra-articular manifestations in the syndrome of diffuse lesions of
the connective tissue, the most important is:

A. Lesions of the lymphatic vessels

B. Kidney damage

C. Lesion of the spleen

D. Liver damage

E. Damage of exocrine glands

12. Livedo reticularis is:

A. Varicose saphenous veins

B. Damage to blood vessels with spasm of the capillary bed

C. Resistant bluish color, uneven skin color in the form of tree spots

D. The appearance of bruises of various sizes on the skin of the limbs

E. Palpable nodules along the arteries of medium caliber

13. Osteolysis is:

A. Formation of bone defects in spongy bones

B. Decrease in bone mass per unit volume

C. Bone resorption, often of nail phalanges

D. Thinning of the bones

E. Cystic bone remodeling

14. Aseptic bone necrosis is:

A. Cellular (holes) pattern of bone tissue

B. Complete necrosis of spongy bone tissue and bone marrow

C. Bone resorption, often of nail phalanges

D. Decrease in bone mass per unit volume

E. Formation of bone defects in tubular bones


15. Female, 35 yearsold. Marksseveremyastheniagravisandfever. Objectively:
periorbitaledemawithpurplish-purpleerythemaaroundtheeyes,
tighteningofthemusclesoftheshouldersandhipswith a
painfulreactiontopalpation. Intheblood, anincrease in CRP, ESR.
Whatanalysisshouldbesenttothepatienttoclarifythediagnosis?

A. Creatinephosphokinase;

B. Rheumatoidfactor;

C. Antistreptolysin O;

D. Antibodiesto DNA strands;

E. Bloodcreatinine;

16. 16-years old girl complaints of periodic whitening and cyanosis of the
fingers. Weight loss up to 6 kg for 2 months. Hardening of the skin on the
hands and face. Diffuse alopecia. Antibodies to topoisomerase -1 in blood
serum analysis. Specify the main syndrome:

A. Anemic syndrome

B. Syndrome of diffuse connective tissue damage

C. Astheno-vegetative syndrome

D. Sjogren's syndrome

E. Syndrome of soft tissue damage near the articular tissues

17. The clinical features of autoimmune diseases are:

A. Predominant involvement of males;

B. Full recovery with continuous treatment;

C. Propensity for progression and recurrence;

D. There are no changes in blood tests;

E. Lack of complete remission;

18. The primary role in the development of diffuse diseases of the connective
tissue is:

A. Congenital anomalies of the internal organs;


B. Children's age;

C. A combination of environmental problems with nutritional patterns;

D. A combination of genetic predisposition and immune disorders;

E. Bad habits (smoking);

19. Female 35-years old. The complaints of marked myasthenia (muscle


weakness) and fever. Objectively: periorbital edema with a purple-lilac
erythema around the eyes, thickening of the muscles of the shoulders and hips
with a painful reaction to palpation. In the blood, an increase in CRP, ESR.
What analysis needed to send the patient to clarify the diagnosis?

A. Creatine phosphokinase;

B. Rheumatoid factor;

C. Antistreptolysin O;

D. Antibodies to DNA strands

E. Creatinine blood;

20. The patient has complaints of pain and swelling in the right knee joint.
The ultrasound in the joint cavity shows about 50 ml of fluid. It corresponds
to:

A. A pronounced degree of joint defiguration;

B. Weak degree of joint defiguration;

C. No defiguration;

D. Moderate joint defiguration;

E. Permissible fluid volume;

21. In a patient with pain and limitation of range of motion in the hip joint, an
X-ray examination can determine:

A. Thickness of the synovial membrane;

B. Atrophy of muscles;
C. Viscosity of synovial fluid;

D. The presence of destructive changes in the joint;

E. Volume of synovial fluid;

22. Boutonniere deformation is:

A. Deformation with flexion of the proximal and over-flexion of the distal


interphalangeal joints

B. Deformation, accompanied by subluxation of the distal phalanges of the hands

C. Deviation of the wrist to the radial side

D. Thickening of the distal interphalangeal joints resembling a flower bud

E. Deviation of the hand to the elbow side

23. Ulnar deviation is:

A. Lateral over-extension of the hand

B. Deformation, accompanied by the deviation of the wrist in the radial direction

C. Deformity of the joints of the hands, resembling a snail

D. Deformation, accompanied by a deviation of the hand in the elbow side

E. Deformation, accompanied by subluxations of the distal phalanges of the hands

24. Deformation in the form of mutilating arthritis is characterized by:

A. Abnormal development of the hands and feet due to various genetic mutations

B. Multidirectional curvature and shortening of the phalanges

C. Disruption of the axis of the joints of the hands and feet

D. The fusion of the phalanges between the fingers

E. Deviation of the hand to the radial side

25. Bone erosion detected by X-ray examination is:

A. Marginal bone tissue defects in the form of uzura

B. Bone cysts

C. Subluxations of joints
D. Sharp narrowing of the joint space

E. "punched-out" lesion

26. Monoarticular lesions are:

A. Damage to the joints on one side

B. Damage to one joint

C. A single, non-recurrent joint disorder

D. Monotonic, chronic lesions of the joints

E. Damage of one tendon

27. Visual analog scale (VAS) is:

A. Visual inspection of the joints

B. Evaluation of the function of similar joints compared with the symmetrical side

C. Assessment of the condition of the joint using visualizing methods of


instrumental diagnostics

D. Method of objectification of pain syndrome, assessed by the patient

E. Evaluation of muscle function using visualization methods of instrumental


diagnostics

28. Morning stiffness is:

A. morning stiffness in the whole body

B. Feeling of stiffness in the joints in the morning after waking up

C. Joint pain in the morning

D. Tight swelling of the joints in the morning

E. General weakness in the morning due to intoxication

29. Patient, 28 yearsold, inthedepartmentofcardiorehematology.


Complainsofjointpain. Redness, swelling, localwarmth of the skin,
andpaininthejointareaareobjectivelydetected. Herefusestosteponhissoreleg.
Fromthepatient history: gotsick a weekago, notesafteraninjury. Pre-
exposedsyndromeofdamagetothesoftperiarticulartissues
(bursitisofthekneejointontheleft).
Whatresearchmethodconfirmsyoursyndrome:

   A. Radiographyoftheknee

   B. Ultrasoundoftheknee

   C. Densitometry

   D. Scintigraphy

   E. Cholangiography

30. A 46-year-old
womancametotheclinicwithcomplaintsofachingpainintheshoulderjoint,
aggravatedbypalpationintheaffectedarea. Additional complaints
areimpairedmobility, edemaandhyperemia. Symptoms
appearsafterprolongedphysicalexertion. Heworksas a tenniscoach.
USDwithoutfeatures. Ultrasoundoftheshoulderjoint: supraspinatusmuscle.
Thestructureisheterogeneous, theechogenicityofthetendonisordinary.
Subacromio-subdeltoidbagisnotenlarged. Whatsyndromearewetalkingabout?

   A. ligamentitis

   B. bursitis

   C. tendobursitis

   D. tendonitis

   E. fibrositis

31. In inflammatory lesions of the joint, the ultrasonography can identify the
following characteristic features:

A. Cluster of synovial exudate, proliferation of the synovial membrane

B. The presence of microorganisms that cause joint disease

C. Bone UZURATIONS

D. "punched-out" symptom

E. Regional osteosclerosis

32. Rheumatoid factor is:


A. Risk factor for developing rheumatoid arthritis

B. Factor contributing to the development of deformities in rheumatoid arthritis

C. Serologic marker for rheumatoid arthritis

D. Mandatory criteria for rheumatoid arthritis

E. Factor showing reactive arthritis

33. Myasthenia is:

A. Pain in the muscles of the limbs

B. Muscle hypotrophy

C. Muscle weakness

D. Muscle hyperkinesis

E. Muscle atrophy

34. What is the term for increased sensitivity to ultraviolet irradiation of open
skin surfaces?

A. Photosensitization

B. Dermafotosis

C. Photosynthesis

D. Ringworm

E. Photophobia

35. Patient R., 36 years old, was admitted to the clinic with complaints of pain
in the metacarpophalangeal, wrist, knee joints, swelling of these joints,
restriction of movements in them. In the morning, notes stiffness in the
affected joints until 12 o'clock in the afternoon. Specify the main syndrome:

A. Syndrome of diffuse inflammation of the connective tissue

B. Osteopenic syndrome

C. Syndrome of inflammatory changes of the joints

D. Muscle inflammatory syndrome

E. Syndrome of inflammation of the soft periarticular tissues


36. The most typical sign of deformity of the hand in the syndrome of
degenerative joint damage is:

A. Heberden's nodules

B. Ulnar deviation

C. "swan neck" deformity

D. "buttonhole" deformity

E. "m-shaped deformation"

37. Which joints are often affected in degenerative-dystrophic lesions of the


osteo-articular system:

A. elbow joint

B. knee joints

C. first metacarpophalangeal joint

D. Thoracic spine joints

E. shoulder joints

38. Bouchard nodes appear:

A. in the syndrome of damage to the soft periarticular tissues

B. in the syndrome of inflammatory changes in the joints

C. in the syndrome of degenerative joint damage

D. in diffuse connective tissue syndrome

E. in the syndrome of inflammatory muscle changes

39. The characteristic symptom of degenerative joint disease is:

A. Fever

B. morning stiffness for more than 1 hour

C. myalgia

D. hemorrhagic rash

E. “starting” character of pain


40. Deformation of the joint is due to:

A. Lack of joint pathology

B. Exudative- proliferative phenomena in the joint and periarticular tissues

C. bone ingrowths (fusion)

D. destruction of muscles

E. tight formation, rising above the skin

41. What is the name of the palpation sensation of a crunch or crack in the
joint during movement?

A. Crepitus

B. Soreness

C. Blockade

D. Stiffness

E. Contracture

42. What is the name of a symptom characterized by periodic acute pain in


the joint during movements, falling into the joint space of fragments of
cracking cartilage?

A. “blockade” of the joint

B. “starting pains”

C. "mechanical pain"

D. “night pain”

E. “morning stiffness”

43. How is the starting nature of the pain characterized?

A. Athlete's joint pain during start

B. joint pain at the beginning of movement

C. Morning joint pain after waking up

D. Pain worse at rest and lessened by movement


E. arthralgia more harassing in the first half of the night

44. Crepitation is:

A. limiting the range of motion in a joint in a certain direction

B. The presence of “joint muscle” inside the joint

C. palpation of cracking sound in the joint when moving

D. abnormal movement in the joint

E. fluid accumulation inside the joint

45. The symptom of "blockade" of the joint is:

A. Periodic acute pain in the joint with movements, fragments of cracking


cartilage getting into the joint space

B. inability to move in the joint due to fusion of the articular surfaces

C. increased pain in the joint after intra-articular blockade of the joint

D. morning joint pain upon waking

E. Periodic "stumbling" of the joint while walking

46. The starting nature of the pain is:

A. joint pain in an athlete during the start

B. pain in the joint at the beginning of the movement

C. morning pain in the joint after waking up

D. pain, aggravated at rest and relieved by movement

E. Arthralgia more disturbing in the evening and in the first half of the night.

47. One of the features of pain in a degenerative lesion of the hip joint is:

A. increased pain in the morning

B. irradiation of the pain to the groin area

C. Pain of "lumbago" type

D. burning character of the pain

E. pain worsening at rest


48. What is the name of the syndrome, characterized by a decrease in the
mineral density of bone tissue and a disruption of its microarchitecture with
subsequent transition to osteoporosis?

A. Osteopenia

B. Osteomalacia

C. Osteosclerosis

D. Osteonecrosis

E. Osteochondrosis

49. Patient A., 67-years old. Complaints of pain in the knee, hip joints, when
moving and when going down the stairs. From the anamnesis: pain in the
joints for 5-6 years. The nodes of Heberden and Bouchard are noted on the
hands. CRP - neg. RF - neg. X-ray of the knee joints: narrowing of the
interarticular crevices, subchondral osteosclerosis, osteophytes. What
syndrome are we talking about?

A. diffuse lesion syndrome of connective tissue

B. muscle inflammatory syndrome

C. syndrome of degenerative joint damage

D. syndrome of lesions of the soft periarticular tissues

E. syndrome of inflammatory changes in the joints

50. Lupus cheilitis is:


A. Bubble eruptions on the mucosa of lips
B. Painless ulcers on the gum mucosa
C. Dense dry grayish scales, with inflammation of the red border of the lips
D. "Bits" at the corners of the mouth
E. Aphthous stomatitis on the background of DBST
51. "Dermatomyositis glasses" are:

A. Periorbital edema with purplish-purple erythema

B. Cyanotic eyelid skin staining

C. Dark circles under the eyes

D. Hyperpigmentation of the upper eyelids

E. Eye constriction with erythema of eyelids

52. DIC may be a complication of:

A. Syndrome of diffuse lesions of connective tissue

B. Syndrome of degenerative lesions of the joints

C. Syndrome of soft tissue lesions around the articular tissues

D. Osteopenic syndrome

E. Sjogren's syndrome

53. Patient D., 54 years old, complaints of pain in the knee joints, which are
worse in the evening. Locally: In the area of the interphalangeal joints of the
hands, the Heberden nodules are defined, the knee joints are de-configured
and varus deformity. There is a crunch in the knee joints. UAC, OAM: no
change. What is your intended syndrome?

A. muscle inflammatory syndrome

B. syndrome of degenerative joint damage

C. syndrome of lesions of the soft periarticular tissues

D. syndrome of inflammatory changes in the joints

E. diffuse connective tissue syndrome


54. Osteopenic syndrome is:

A. Vascular pathology characterized by transient ischemia of the fingers, tip of the


nose and / or ears, caused by external factors (for example, cold).

B. It is the presence of an inflammatory process (edema, increased local


temperature, redness, soreness and impairment of function) in the joints, which is
determined by an objective examination.

C. Intermittent muscle weakness in patients with obliterating arteriosclerosis


without marked trophic disturbances.

D. The decrease in bone mineral density and the disruption of its


microarchitecture with the subsequent transition to osteoporosis

E. Aseptic necrosis of metacarpal bones with secondary osteosclerosis and


arthrosis.

55. Which bone cells are responsible for resorption during bone remodeling?

A. Osteoblasts

B. Osteocyte

C. Osteoclasts

D. Osteoid

E. Chondrocytes

56. The most informative instrumental method of diagnosis of osteopenic


syndrome is:

A. X-ray densitometry

B. Conventional radiography

C. Computed tomography

D. Ultrasound of the joints

E. Arthroscopy

57. Densitometry determines:

A. Bone mineral density

B. The ratio of trace elements in bone tissue


C. The presence of synovial fluid

D. Swelling of periarticular soft tissue

E. Cartilage structure

58. The most frequent localization of osteopenic fractures:

A. Neck of the femur, vertebral body, fracture of the radius

B. Ribs, clavicle, shoulder blade,

C. Upper and lower jaw

D. Tibia and ankles

E. Bones of fingers and facial skull

59. Bone Formation Marker

A. Pyridinoline (PD, NTX) and deoxypyridinoline (U-DPD - free)

B. Bone alkaline phosphatase (bone ALP)

C. Hydroxproline (OP)

D. Galactosyloxylysin (GOP)

E. Tartrate-resistant acid phosphatase (t ARF)

60. Age-related changes in bone tissue are related:

A. with increasing bone sensitivity to parathyroid hormone

B. with increasing calcium excretion

C. with calcitonin deficiency

D. with inhibition of the synthesis of vitamin D3

E. with increasing testosterone levels

61. The defiguration is due to:

A. lack of mobility in the joint

B. Exudative- proliferative phenomena in the synovial membrane


C. Loss of joint native form

D. Destruction of articular surfaces

E. Restricted joint mobility

62. Nodules are:

A. A sign of active rheumatic fever, a pale pink, narrow thread that creates a lace
pattern on the skin, quickly appears and disappears

B. Skin infiltrates of red-purple color, clearly elevating above the skin surface, are
located mainly on the front surface of the tibia

C. on the nose and cheeks, such as erythema

D. Multiple aphthous and ulcerative lesions of the mucous membranes

E. dense formation, rising above the skin level, located in the deep layers of
the dermis and subcutaneous tissue

63. A LE cell is:

A. Eosinophils modified

B. Small Platelet Count

C. Discoid erythrocytes

D. Mature neutrophils phagocytizing altered nuclear inclusions

E. Ragocyte

64. When the joint function is changed, it changes:

A. Joint circumference

B. Gait

C. Patient Weight

D. Analysis of synovial fluid

E. Patient growth

65. The method of research that reveals flat feet is:

A. Plantography

B. Futoscopy
C. Stopometry

D. Sledoscopy

E. Tonometry

66. Modifiable risk factors for osteopenia syndrome:

A. Female gender, age over 65

B. Caucasian race, family history

C. Anterior fractures, hypogonadism

D. Insufficient intake of calcium from food and vitamin D deficiency

E. Early menopause in women

67. Measuring the circumference of a joint helps to identify:

A. Swelling of the joint

B. Joint deformity

C. Impaired joint function

D. Hypermobility of the joint

E. Crunch and Crepitations

68. Compensatory fractures of the spine are manifested by:

A. There are no manifestations

B. Swelling of the soft tissue of the back

C. Redness of the skin in the paravertebral zone

D. Acute pain in the lower back arising after exercise

E. Constant pain, aggravated at night

69. A synovial biopsy can be used to identify:

A. Tumors, granulomatous diseases

B. Defibration and necrosis of cartilage

C. Quantities of synovial fluid


D. A large number of ragocytes

E. Viscosity reduction

70. In the analysis of synovial fluid take into account:

A. Cytosis, protein content, the presence and nature of salt crystals

B. pronounced nuclear cell pathology

C. a large number of ragocytes

D. white blood cell count

E. number of uniform elements

71. Tendonitis is:

A. Isolated tendon lesion

B. Inflammation of the joint capsule

C. sprain

D. inflammatory lesion of periarticular tissue

E. lesion located in the tendon-muscular ligament area

72. Patient S., 18 years old, complaints of weakness and muscle pain, fever,
shortness of breath, weight loss of 6 kg. From the patient history: felt sick
after a viral infection, with subsequent pain in the joints and muscles.
Objective: The condition is severe. He can’t raise his head, stand up, eat on
his own. Periorbital edema, purple spots on the thigh skin. Which of the
following is most appropriate for verifying the diagnosis?

A. Definition of creatine phosphokinase

B. Determination of rheumatoid factor

C. Determination of antinuclear factor

D. Determination of antibodies to double-stranded DNA

E. Determination of antibodies to topoisomerase (SCL-70)

73. Myotendinitis is:

A. lesion located in the region of the tendon-muscular ligament


B. Inflammatory lesion of extra-articular ligaments, most often the ligamentous
canal through which the tendon passes

C. lesion of the joint capsule

D. Isolated lesion of the tendon of the corresponding muscle

E. Inflammatory lesion of periarticular lymph nodes

74. Fibrositis is:

A. Isolated lesion of the tendon of the corresponding muscle

B. lesion of the joint capsule

C. Inflammation of the fascia and aponeuroses

D. inflammatory lesion of extra-articular ligaments, most often the ligamentous


canal through which the tendon passes

E. Sprain

75. Patient 64 years old, admitted to the department of rheumatology.


Complaints about the restriction of mobility in the distal interphalangeal
joints of both hands, which arose about 12 years ago and is gradually
progressing. On examination, nodular thickenings in the region of the distal
interphalangeal joints of both hands are noteworthy, the fingers are
deformed, their mobility is limited. From the internal organs pathology is not
detected. Blood and urine tests are normal. What is the syndrome of this
patient?

A. Degenerative-dystrophic joint changes

B. Inflammatory joint changes

C. Lesions of soft periarticular tissue

D. Diffuse inflammation of the connective tissue

E. Osteopenia

76. A patient of 65 years old, admitted to the rheumatology department,


complains of severe pain when walking in the right knee and hip joints,
aggravated by the end of the day, decreased range of motion in the right hip
joint. From the patient history: considers himself ill for 8 years. Objectively:
painful restriction of movement in the right knee joint, crepitus. Labs: within
normal limits. RF - negative, CRP - negative. Pre-exposed syndrome of
degenerative - dystrophic changes in the joints. What symptom confirms?

A. Blockades

B. Thomayer

C. Forestier

D. Schober

E. Ankylosing spondylitis

77. A 36-year-old woman came to the rheumatology department. Complains


of rashes in the area of the cheekbones and cheeks, especially aggravated by
exposure to the sun, constantly cold limbs with whitening of the fingers of the
hands. Weight loss up to 6 kg in one month, hair loss. In blood tests,
antibodies to double-stranded DNA, antibodies to Sm antigen are positive.
What is the syndrome of this patient?

A. degenerative joint disease

B. inflammatory muscles and joints

C. asthenovegetative

D. diffuse damage to the connective tissue

E. Soft lesions near articular tissues.

78. A 36-year-old man admitetd to the rheumatology department with


complaints of pain in the left knee joint, the pain intensifies during pressing
on the area near the joint and during physical exertion, redness, swelling,
muscle weakness and fever above the affected joint, stiffness in the morning .
From the patient history: within 10 days the above complaints appeared after
he fell during football. According to the patient, he has been playing football
for 10 years and denies such complaints before. What is the preliminary
syndrome for this patient?

A. degenerative joint disease

B. inflammatory muscles and joints

C. asthenovegetative
D. diffuse damage to the connective tissue

E. Soft lesions near articular tissues.

79. The formation of the "snapping" finger due to:

A. Severe osteoarthritis of the hand joints

B. mismatch between the diameter of the fibrous ring and the thickness of the
flexor tendon of the finger

C. nodular tendovaginitis of the flexor of the finger

D. arthritis of the interphalangeal joint of the hand

E. arthritis of the shoulder joint

80. Patient S., 18 years old, complaints of weakness and pain in muscles, fever,
shortness of breath, weight loss of 6 kg. From the anamnesis: after a viral
infection pains in the joints and muscles appeared. Objectively: The condition
is serious. He can not raise his head, stand up, eat on their own. Periorbital
edema, purple spots on the skin of the thighs. Which examination of the above
is most appropriate for verification of the diagnosis?

A. Determination of creatine phosphokinase

B. Determination of rheumatoid factor

C. Determination of antinuclear factor

D. Determination of antibodies to double-stranded DNA

E. Detection of antibodies to topoisomerase (SCL-70)

81. Patient J. 38 years. She has admitted withthe complaints of face and arm
skin tightening, weight loss of 12 kg, pain in the joints, difficulty swallowing
dry food, shortness of breath. From the anamnesis: Sick for 5 years, there
were thick edema on the hands and face, arthralgia, and swallowing
disorders. Objective: Low body nutrition. Teleangiectasia on the neck, chest.
Masked face.Restricted mouth. Dummy fingers. The skin in the fold is not
taken by grabbing. What examination is most appropriate for verification of
the diagnosis?

A. Rheumatoid factor
B. Antinuclear antibodies

C. Antibodies to double-stranded DNA

D. Antibodies to topoisomerase (SCL-70)

E. Antibodies to cyclic-citrulinated peptide

82. Patient V., 23 years old. Complaints: pain in the joints, cough, shortness of
breath, rashes on the skin, appearing after exposure to the sun, general
weakness, alopecia, irregular menstruation. From the anamnesis: is ill for the
past six months, started after abortion. Worried about joint pain and
subfebrile. Objectively: General condition of moderate severity. In the area of
the cheekbones and the back of the nose erythema. On the lips - scaly crusts.
What is your presumed syndrome?

A. Syndrome of diffuse lesions of the connective tissue

B. Syndrome of degenerative lesions of the joints

C. Syndrome of soft tissue lesions of the periarticular tissues

D. Osteopenic syndrome

E. Syndrome of inflammatory changes of the joints

83. Joint pain is:

A. Ossalgia

B. Myalgia

C. Arthralgia

D. Tendalgia

E. Odontalgia

84. Dorsalgia is

A. Pain in the joints of the lower limbs

B. Pain in the sacrum

C. Back pain

D. Pain in tailbone

E. Pain in the pelvic region


85. Ossalgia is:

A. Bone pain

B. Muscle pain

C. Pain in the joints

D. Pain in the hands

E. Pain in the whole body

86. The rheumatoid factor is:

A. Subtype of antinuclear antibodies.

B. Antibodies to the Fc fragment of IgG.

C. Antibodies to the components of the synovial membrane of the joints.

D. Immune complex containing IgM class antibodies and IgG Fc fragment.

E. A set of cytokines that play a key role in the pathogenesis of RA.

87. Patient 65 years old, admitted to the on-call clinic. Complains of pain in
the knee joints, which intensify in the evening. From the anamnesis: the
mother has diseases of the musculoskeletal system. Repeatedly received
injuries of the knee joint. Objectively: a patient with increased nutrition, BMI
35 kg / m2. Locally: in the interphalangeal joints of the hands, Heberden
nodules, varus deformity of the knee joints are determined. There is a crunch
in the knee joints. CBCC, Urine analysis: no change. Highlight modifying risk
factors for this patient:

A. Female

B. old age

C. heredity

D. history of injuries

E. overweight

88. Valgus deformity is:

A. O-shaped deformity of the knee joint

B. X-shaped deformity of the knee joint


C. Knee over-bending

D. Z-shaped deformity of the knee joint

E. M-shaped deformity of the knee

89. A 69-year-old patient was admitted to the rheumatology department with


complaints of pain in bones and joints. From the anamnesis, a fracture of the
radial bone in 2012. A preliminary osteopenia syndrome was exposed.
According to densitometry results, what is the expected T-criterion?

A. "-1" and above

B. "-1" - "-2.5"

C. "-2.5" and below

D. “+1” and above

E. "+2.5" and below

90. The manifestation of degenerative changes in which Bouchard's nodules


are found?

A. proximal interphalangeal joints of the hand;

B. Distal interphalangeal joints of the hand;

C. knee joints;

D. first metatarsophalangeal joint;

E. elbow joint.

91. What method is used to evaluate the functions of the wrist?

A. palpation method

B. physical examination

C. Duplex compression method

D. active squeezing fingers

E. uplift

92. A patient is 56 years old. A history of radial bone the distal third fracture
occurred twice a year, when falling from a height of its growth. What
instrumental diagnostic method will be the most informative for confirming
osteopenic syndrome:

A. X-ray densitometry

B. Routine radiography

C. Computed tomography

D. Osteoscintigraphy

E. X-ray angiography

93. The patient is 76 years old. Within 15 months, receives treatment with a
diagnosis of osteoporosis; On re-examination - no complaints. What bone
formation marker can be tested in this patient?

A. Pyridinoline and deoxypyridinoline

B. Bone alkaline phosphatase

C. Lymphotoxin alpha

D. Tumor necrosis factor

E. Tartrate Acid Phosphatase

94. A 36-year-old patient has hyperemia, hyperthermia, and sharp pain in the
first metatarsophalangeal joint of the foot on the left. Uric acid 455 mmol / L,
ESR 20 mm / h. What syndrome are we talking about?

A. degenerative joint disease

B. inflammatory joint damage

C. inflammatory muscle damage

D. diffuse damage to the connective tissue

E. Damage to the soft periarticular tissues

95. A 35 years old woman came to the rheumatology department. Complains


of severe myasthenia gravis and fever. Objectively: periorbital edema with
purplish-purple erythema around the eyes, tightening of the muscles of the
shoulders and hips with a painful reaction to palpation. In the blood, an
increase in CRP, ESR. What analysis is needed to clarify the diagnosis in this
case?

A. Creatine phosphokinase

B. Creatinine

C. Alkaline phosphatase

D. Antistreptolysin O

E. Calcitonin

96. In a 40-year-old patient with inflammatory joint damage syndrome,


radiographs of the hands determine: periarticular osteoporosis, narrowing of
the joint spaces, multiple erosions. What X-ray stage according to Stein-
Broker corresponds to this description?

A. 1

B. 2a

C. 2b

D. 3

E. 4

97. What blood test determines seropositivity in the syndrome of


inflammatory joint damage?

A. Rheumatoid factor

B. Tumor Necrosis Factor

C. C-reactive protein

D. Antistreptolysin O

E. Antinuclear factor
98. A 65 years old patient, amitted to the department of rheumatology.
Complaints of pain in the knee joints, which intensify in the evening. From the
patient history, frequent injuries noted. Mom has osteoarthritis of the knee
joints. Smokes for about ten years. Objectively: BMI 35kg / m2. There is a
crunch in the knee joints. CBCC, Urine analysis: no change. What non-
modifiable risk factors are present in this patient?

A. injuries, smoking, gender

B. old age, gender, heredity

C. smoking, obesity, injury

D. injury, obesity, old age

E. old age, obesity, smoking

99. A 37-year-old patient admitted to the rheumatology department.


Complains of joint pain, fever, weight loss, increased hair loss. Objectively:
Erythema in the region of the upper cheekbones, nose bridge, upper third of
the chest, on the neck. Ulcers on the oral mucosa. In the lungs, breathing is
weakened in the lower sections. Heart tones are muffled, rhythmic. Heart rate
is 81 beats per minute. BP is 100/70 mm Hg. The abdomen is soft, sensitive in
the right hypochondrium. What is the syndrome of this patient?

A. degenerative joint damage

B. damage to the soft periarticular tissues

C. degenerative - dystrophic joint changes

D. diffuse inflammation of the connective tissue

E. inflammatory changes in joints and muscles

100. A 42-year-old patient was admitted to the Department of


Cardiorheumatology, with complaints of joint pain, fever, pain when
swallowing, severe general weakness, shortness of breath at the slightest
exertion, dry cough, sore fingers in the cold. Objectively: face skin is
tightened, "purse-string mouth". In the lungs, breathing is weakened,
crepitus is heard in the basal sections. Heart tones are muffled, arrhythmic.
The doctor put a preliminary syndrome of diffuse inflammation of the
connective tissue. Whattestscanconfirmthissyndrome?

A. c - reactiveprotein
B. Haddelsonreaction
C. Anticentromericantibodies
D. livertests
E. Kazzonireaction
Bonpoc:

Which of the following symptoms are characteristic of lesions of the spinal


cord in the lumbar enlargement ( LI-S2):

1 shortness of breath, hiccups

2 central leg paralysis

3 peripheral hand paralysis

4 peripheral paralysis of the legs

5 true urinary incontinence

Bonpoc: NQ2

A proper evaluation of deep join-position sense can be performed with:

1Using camerton

2Pinprick test

3Using Weber compass

4Making passive movements on phalanges

5Gentle touch with brush or hair

To assess cognitive function, what should be check:

1 Reflexes

2 General urine analysis

3 General blood analysis

4 Attention

5 Liquor

Bonpoc: NQ4

Damage to the posterior horn of the spinal cord leads to:


1 Disorders of sweating at the level of damage

2 Disorders of deep sensitivity

3 Disorders of movement at the level of damage

4 Fascicular twitching of muscle at the level of

damage

5 Loss of superficial sensitivity at the level of

damage

Bonpoc: NQ5

When the corpus callosum is damaged we observe disturbances of:

1 Reciprocal coordination of movements

2 Analytical thinking

3 Hearing and tactile sensitivity

4 Central vision

5 Long-term memory

Bonpoc: NQ6

Periosteal reflexes include:

1 plantar reflex

2 carporadial reflex

3 middle abdominal reflex

4 cremasteric reflex

5 upper abdominal reflex

Bonpoc: NQ7

What structure of the cortex must be damaged to lose stereognosis?

1 Occipital lobe

2 Temporal lobe
3 Parietal lobe

4 Cerebellum

5 Frontal lobe

Bonpoc: NQ9

Morphological asymmetry is:

1 Various content of biologically active

substances

2 Different hemisphere weights

3 Each hemisphere processes information of a

certain type

4 Different blood supply to the hemispheres

5 Different structure of the hemispheres

Bonpoc: NQIO

Gordon's symptom is:

1 reflex flexion of the II - V toes as a result of a short blow to the tips


of the toes with a hammer

2 flexion and rotation of the foot inward, with streak irritation of the
inner edge of the sole

3 reflex extension of the thumb, as a result of compression of the


gastrocnemius muscle

4 abduction of the little finger with dashed stimulation of the outer edge
of the foot

5 reflex flexion of the Il - V toes when tapping on the antero-outer


surface of the rear of the foot
Bonpoc: NQI 1

Functional interhemispheric asymmetry is :

1 Unstable neurophysiological phenomenon

2 This phenomenon has not yet been studied.

3 A stable, persistent neurophysiological

phenomenon.
4 Individual feature

5 Sign of impaired brain function

Bonpoc: WI 2

In case of damage to the caudate nucleus and putamen, the patient develops:

1 alternating syndrome

2 akinet ico- rigid syndrome

3 inner capsule syndrome

4 ataxic syndrome

5 hypotonic hyperkinetic syndrome

Bonpoc: NQI 4

In a 13-year-old patient, a neurological examination revealed right-sided


central hemiparesis with a decrease in muscle strength to 2 points. How
will muscle tone change in this case?

1 Clasp Knife Phenomenon

2 Promotion by plastic type

3 Cog wheel rigidity

4 Decreased tone in the proximal region

5 Decreased distal tone

\
onpoc: NQ22

Mom brought a 13-year-old child to a neurologist with complaints of


choking, pouring liquid food from the nose when swallowing, hoarseness.
Neurological examination revealed dysphagia, dysarthria, dysphonia, and
drooping of the soft palate. What reflex should be investigated to exclude
damage to the corticonuclear pathways in this case?

1 Oppenheim

2 Babinsky

3 Pupillary

4 Distant-Oral

5 Postural
Bonpoc: NQ23

Mother took a 13 year old child to neurologist with


complaints of choking, pouring out liguid food from the
nose while swallowing, hoarseness of the voice. Where
from the following strusctures it is possible to localize
the lesion?

1 nuclei 3,4,6 pair CN


2 basal nuclei
3 cerebellar hemispheres
4 cerebellar worm
5 nuclei pair CN

Bonpoc: NQ24

A mother brought her 13-year-old daughter to the clinic to a


neuropathologist with complaints of a twisted face to the left, inability
to close her right eye, pouring out liquid food from the right corner of
her mouth. When examining the neurological status, Bell's symptoms were
found, the racket on the right. What symptom in this case will speak in
favor of the defeat at the level of the nucleus of the 7th pair of CN in
the pons?

1 Ataxia

2 Xerophthalmia

3 Hyperacusia

4 Lachrymation

5 Hypogeusia

Bonpoc: NQ25

A mother brought her 11-year-old daughter to the clinic to neuropathologist


with complaints of a twisted face to the left, inability to close her right
eye, pouring out liquid food from the right corner of her mouth.
Neurological examination revealed Bell's symptoms, the racket on the right.
The loss of which reflex in this case will in case of 7th pair lesion?

1 Achilles

2 Corneal

3 Rossolimo

4 Knee

5 Zhukovsky
Bonpoc: NQ29

What is the characteristic of the subperiosteal hematoma of the fronto-


parietal-occipital region?

1 freely spreads to the tissue of the face


2 it is difficult to give a clear characteristic
3 has a diffuse character within the fronto-
parieto-occipital region
4 has the shape of a bump
5 spreads within one bone

Bonpoc: NQ30

Thrombosis of which sinus of the dura mater develops in some forms of the
carbuncle of the upper lip, wings of the nose or the superciliary arch:

1 wedge-parietal

2 upper sagittal

3 lower sagittal

4 rectus

5 cavernous

Bonpoc: NQ26

Into what cellular space does the subgaleurotic cellular space of the
temporal region continue downward?

1 periopharyngeal

2 chewing-jaw

3 jaw-pterygoid

4 inter-pterygoid

5 retrtooccipital

Bonpoc: NQ27

Sinus of the dura mater, communicating with the angular vein:

1 transverse

2 inferior petrosal

3 cavernous
4 superior sagittal

5 superior petrosal

Bonpoc: NQ28

The point Of digital pressure of the facial artery is located

1 behind the corner of the lower law

2 at the anterior edge of the masseter muscle on

the lower Jaw

3 1 cm below the ear tragus

4 Q at the base of the temporal arch

5 0.5-1 cm below the middle of the lower edge of

the orbit

Bonpoc: NQ31

What is the first stage in the formation of speech in a child.

1 involuntary humming

2 preparatory

3 imitative

4 Motor speech

5 "sensory speech"

Bonpoc: NQ30

Thrombosis of which sinus of the dura mater develops in some forms of the
carbuncle of the upper lip, wings of the nose or the superciliary arch:

1 wedge-parietal

2 upper sagittal

3 lower sagittal

4 rectus

5 cavernous
Bonpoc: NQ32

The child holds his head from the age of

1) 6 month

2) 4 month

3) 5 month

4) 1 month

5) 2 month

Q. The child is 3 months old. Complaints of anxiety, poor sleep, shudders


during sleep. On examination: physiological reflexes are active, muscle
tone is increased, the gaze is fixed. the size of the head is normal.
Determine which of the symptoms indicates increased tone:

1 Arms out to the sides

2 Clenching the fingers into a fist

3 Fingers and toes are straight

4 Head tilted forward

5 Legs straightened

Bonpoc: NQ33
Baby is 8 month old, On examination: a temperature rise of 39.5 is noted,
vomiting, restlessness, stiff neck muscles, large fontanelle throbbing To
clarify the diagnosis. which reflex should be determined?

1 reflex bauer

2 reflex Galant

3 reflex Babinsky

4 reflex Moro

5 reflex Brudzinsky

Bonpoc: NQ36

Brain imaging technique which is the first choice for premature babies.
1 Skull X-ray

2 Magnetic resonance imaging (MRI)


3 Neurosonography

4 Computed tomography (CT)

5 Positron emission tomography (PET)

Bonpoc: NQ35

On examination, the child sits on his own, turns from stomach to back and
from back to stomach. distinguishes strangers from loved ones, takes a toy
from different positions, crawls, pronounces syllables. Determine the age
of the child,

1) 7 month

2) 10 month

3) 6 month

4) 5 month

5) 12 month

Bonpoc: NQ37

______is performed by introducing a nonionic, water-soluble contrast medium


into the subarachnoid space by spinal puncture.

1 Radiography

2 Magnetic resonance imaging (MRI)

3 Positron emission tomography (PET)

4 Myelography

5 Computed tomography (CT)

Bonpoc: NQ39
An axial view of vertebral body on the CT-scan image demonstrates disk
bulging Which involve entire disc circumference. Which imaging modality is
the best for confirmation this diagnosis and evaluation of soft tissue
involvement?

1 Myelography

2 Radiography

3 Positron emission tomography (PET)

4 Magnetic resonance imaging (MRI)

5 Computed tomography(CT)
Bonpoc: NQ38
Neurosonography of a 3-month-old boy revealed dilated lateral brain
ventricles up to .15 mm. The contents are homogeneous. Which imaging
technique should be done next to make a diagnosis?

1 Computed tomography (CT)

2 Magnetic resonance imaging (MRI)

3 Positron emission tomography (PET)

4 Radiography

5 Myelography

Bonpoc: NQ40
A 72-yeardd man was admitted to the hospital with cornplaints of memory
impairment. The preliminary diagnosis was identified as Alzheimer's
disease. TO confirm the diagnosis, it IS necessary to check blood flow and
oxygen content. Which brain imaging technique is more appropriate•

1 Positron emission tomography (PET)

2 Computed tomography (CT).

3 Functional MRI (fMRl)

4 Electroencephalography (EEG)

5 Magnetic resonance imaging (MRI)

Bonpoc: NQ41

Psychogenic conditions for the development of neurosis.

1 Unfavorable marital status.

2 Psycho-traumatic situations

3 Inconvenient workplace or transport.

4 Features of sex education, etc.

5 Unsatisfactory living conditions.


Bonpoc: NQ42

The central source of pain is

1 Decay products of tissues

2 Inflammatory mediators

3 Generator focus in the brain

4 Regenerating sensory nerves

5 Irritation of tissue receptors

Bonpoc: NQ43

Pain in intemal organs, abdomen. or chest called:

1 Phantom pain

2 Visceral pain

3 Neuropathic pain

4 Deaferentation pain

5 Referred pain

Bonpoc: NQ44

it is a developing trophiC disorder that is caused by loss or alteration of


neural stimuli:

1 Hysterical neurosis

2 Primary pain

3 Neuropathlc pan

4 Secondary pain

5 Neurodystrophic process

Bonpoc: NQ45

How does the sensitivity of denervated structures Change?

1 Biologically active substances

2 Declines

3 Decreases to morphological agents

4 Does not change

5 First decreases then disappears


Bonpoc: NQ46

A patient turned to a doctor complaining that she was constantly haunted by


the thought that she had not turned Off the gas at home and was checking 4-
5 times a day What kind of neurosis can be assumed?

1 The patient has no neurosis

2 Neurasthenia

3 Obsesstve cornpuisive disorder

4 Hysterical neurosis

5 Social phobias

Bonpoc: NQ47

A 36-year-old man complains Of pain in the right side Of the chest, sharply
increasing with movements, coughing, breathing. Moves slowly holds the sore
spot with his hand. An hour ago, slipping, fell, hit his chest about the
edge Of the sidewalk. What kind of pain does this patient have:

1 Phantom pain

2 Epicritical

3 Thalamic pain

4 Visceral pain

5 Causalgia

Bonpoc: NQ48
A 50-yearold man complains of compressive chest pains on the left.
Anamnesis: works as a broker. smokes for 20 years. What type of pain can
this pain be attributed to

1 epicritical

2 Visceral pain

3 Phantom pain

4 Thalamic pain

Bonpoc: NQ49

A 36-year-old patient. a builder. was admitted by an ambulance to the


neuroloqical department. tell from the second floor during work There was a
surgical operation due to a fracture of 2-3 lumbar vertebrae The next day
after regamng consciousness. he cannot move both legs What the patient s
pathology?

1 Paraplegia

2 Tetraplegia

3 Epilepsy

4 Chorea

5 Hemiplegia

Q.The syndrome of decreased mental activity with a preserved level of


wakefulness Includes:

1 Oneroid syndrome

2 Dementia

3 Amentive syndrome

4 Coma

5 Twilight clouding of consciousness

Bonpoc: NQ2

Irritation of the motor projection area of the cortex causes

1 central tetraplegia

2 monoplegia of the upper limbs

3 diplegia on the side of the focus

4 monoparesis on the side of the lesion

5 Jacksonian seizures

Bonpoc: NQ3

Damage to the posterior horn of the spinal cord leads to:

1 Disorders of sweating at the level of damage

2 Disorders of movement at the level of damage

3 Fascicular twitching of muscle at the level of

damage
4 Loss of superficial sensitivity at the level of

damage

5 Disorders of deep sensitivity

Bonpoc: NQ4

Gordon's symptom is:

1 abduction of the little finger with dashed

stimulation of the outer edge of the foot

2 reflex extension of the thumb, as a result of

compression of the gastrocnemius muscle

3 flexion and rotation of the foot inward, with

streak irritation of the inner edge of the sole

4 reflex flexion of the Il -V toes as a result of a

short blow to the tips of the toes with a hammer

5 reflex flexion of the Il -V toes when tapping on

the antero-outer surface of the rear of the foot

Bonpoc: NQ5

A proper evaluation of deep join-position sense can be performed with:

1 Using camerton

2 Making passive movements on phalanges

3 Using Weber compass

4 Gentle touch with brush or hair

5 Pinprick test

Bonpoc: NQ6

When the lateral horns of the spinal cord are affected we observe:

1 Damage to superficial sensitivity

2 Loss of motor function


3 Sweating disorder at the lesion level

4 Fascicular twitching at the level of the lesion

5 The defeat of deep sensitivity at the level of the

defeat

Bonpoc: NQ7

Extramedullary compression lesion syndrome:

1 More often in the thoracic region or in the

cauda equina

2 More often in the area of thickening

3 Descending flow type

4 No block of cerebrospinal fluid

5 Progresses rapidly

Bonpoc: NQ8

The stereognosis is:

1 Recognition of known objects placed on hands


2 An ability to find an irritated point with finger
3 An ability to define numbers, letters written on
skin of hand
4 An ability to define passive joint movements
5 An ability to define two touches at the same
time

Q.Which of the following symptoms are characteristic of lesions of the


spinal cord in the lumbar enlargement (L1-S2)

1 peripheral hand paralysis

2 true urinary incontinence

3 central leg paralysis

4 shortness of breath, hiccups


5 peripheral paralysis of the legs

Bonpoc: NQIO

When the trigeminal (V) nerve is damaged we observe:

1 prosoparesis

2 hyperacusia

3 hearing loss

4 lacrimation and prosoparesis

5 violation of the sensitivity of the facial skin

Bonpoc: NQI 1

Signs of peripheral paralysis include

1 the presence of pathological reflexes

2 synkinesis

3 "Looseness" of the joints

4 increased muscle tone

5 increased normal reflexes

Bonpoc: NQ12

The syndrome with a loss of consciousness:

1 Muscular dystonia

2 Hypothalamic syndrome

3 Coma

4 Meningeal syndrome

5 Hypertensive-hydrocephalic syndrome

Bonpoc: NQ13
In a 6-year-old patient, an examination of the neurological status revealed
lower paraparesis with a decrease in muscle strength to 3 points, as well
as dysfunction of the pelvic organs by the type of urinary retention. What
reflex will indicate the central character of paraparesis in the legs?

1. Marinescu Rodovici

2. Ankle

3. Sucking

4. Gordon

5. Knee

Bonpoc: NQ14

The leading sign that distinguishes sensitive from cerebellar ataxia is:

1. violation of deep sensitivity

2. instability in the Romberg test

3. intentional tremor

4. increased dizziness when turning the head

5. test for adi a dohokinesis

Bonpoc: NQ15

A 13- year-old patient consulted a neuropathologist at the polyclinic with


complaints of weakness in his right arm . Examination of the neurostatus
revealed central paresis of the right hand . What reflex should be
examined to confirm that the patient has just central paresis ?

1. Palmar-chin

2. Sucking

3. Bekhterev

4. Marinescu Rodovici

5. Proboscis

Q.During the survey the patient revealed: unsteadiness when walking,


especially in the dark and with closed eyes, instability in the Romberg
position, decreased muscle tone in both legs, a feeling of crawling in
them. The patient confuses the name of the fingers and the direction
passive movements in them. Name the existing symptom and indicate the
location of the lesion.

1. Cortical ataxia, frontal lobes

2. Sensitive ataxia, posterior cords of the spinalcord

3. Vestibular ataxia, vestibulocochlear nerve

4. Sensitive ataxia, lateral cords of the spinal cord

5. Cerebellar ataxia, cerebellar vermis

Bonpoc: NQ17

A 15-year-old patient with complaints of a feeling of numbness and also


difficult to localize spilled pains of a burning character in the left half
of the body. What type of sensivity should be examined in a patient to
determine the safety of complex types of sensivity?

1. Temperature

2. Pain feeling

3. Two dimensional

4. Articular muscle

5. Vibration

Bonpoc: NQI 8

In a 6-year-old patient, an examination of the neurological status revealed


lower paraparesis with a decrease On muscle strength to 3 points, as well
as dysfunction of the pelvic organs by the type of urinary retention. What
reflex will indicate the central character of paraparesis in the legs?

1. Knee

2. Marinescu-Rodovici

3. Ankle

4. Babinsky

5. Sucking

Bonpoc: NQ20

A 13- year-old patient consulted a neuropathologist at the polyclinic with


complaints of weakness in the right extremities, a feeling of numbness in
the left extremities. Examination of the neurostatus revealed central
right- sided hemiparesis, a decrease in deep sensitivity in the right
extremities, and a decrease in superficial sensitivity in the left half of
the body. Where is the lesion located ?

1. Complete transverse lesion of the upper cervical segments

2. Complete transverse lesion of the thoracic segments

Half lesion of the upper cervical segments

3. Half lesion of the thoracic segments

4. Complete transverse lesion at the level of the cervical thickening

Bonpoc: NQI 9

With degenerative lesions of the pallidary department, the patient may


experience the following symptoms:

1. muscle rigidity

2. athetosis of the hands

3. hemiballism in the legs

4. Brudzinsky symptom

5. facial hemispasm

Bonpoc: NQ22

A mother brought her 1 6-year-old daughter to the clinic to a


neuropathologist with complaints of a twisted face to the left, inability
to close her right eye, pouring out liquid food from the right corner of
her mouth. Neurological examination revealed Bell's symptoms, the racket on
the right. The loss of which reflex in this case will in case of 7th pair
lesion?

1. Achilles

2. Rossolimo

3. Knee

4. Superciliary

5. Zhukovsky
Bonpoc: NQ23

Mother brought a 13 year old child to neurologist with complaints of


choking, pouring out "guid food from the nose while swallowing, hoarseness
of the voice. Where from the following structures it is possible to
localize the lesion?

1. cerebellar worm

2. cervical thickening

3. cerebellar hemispheres

4. medulla

5. basal nuclei

Bonpoc: NQ24

Mother took a 13 year old child to neurologist with complains of choking,


pouring out liguid food from the nose while swallowing, hoarness of the
voice. What syndrome has patient?

1. hyperkinetic

2. Bulbar syndrome

3. pyramid syndrome

4. Parkinson syndrome

5. Ataxia syndrome

Bonpoc: NQ25
Mom brought her 14-year-old daughter to the clinic to a neurologist with
complaints of a twisted face to the left, inability to close her right eye,
pouring out liquid food from the right corner of her mouth. When examining
the neurological status, Bell's symptoms were found, the racket on the
right. What symptom in this case will exclude a lesion at the level of the
motor nucleus of the 7th pair of CN ?

1. Lachrymation

2. Apraxia

3. Hyperacusis

4. Ataxia

5. Xerophthalmia
Bonpoc: NQ26

In what area of the head should craniotomy be performed to ligate the


middle meningeal artery?

1. in the frontal region

2. in the occipital region

3. in the parietal region

4. in the temporal region

5. in the Shipo triangle

Bonpoc: NQ28
The point of digital pressure of the facial artery is Located:

1. 1 cm below the ear tragus

2. at the base of the temporal arch

3. behind the corner of the lower jaw

4. 0.5-1 cm below the middle of the lower edge of the orbit

5. at the anterior edge of the masseter muscle on the lower jaw

Bonpoc: NQ29

What is the characteristic of subgaleal hematoma of the fronto-parieto-


occipital region?

1. moves freely within the fronto-parieto-occipital region

2. it is difficult to give a clear characteristic

3. corresponds to the shape of the underlying bone

4. has the shape of an oval oriented in the longitudinal direction

5. has a pulsating character

Bonpoc•. NQ30
When performing craniotomy, the base of the soft tissue flap should be
turned downward:

1. due to the superficial location of the main arteries and veins

2. due to the fixation of the walls vessels to the connective tissue


bridges

3. due to the ascending course of the neurovascular bundles

4. due to the presence of a rich network of arterial anastomoses

5. in connection with many layers of soft tissues

Bonpoc: NQ31

What is the first stage in the formation of speech in a child:

1. preparatory

2. imitative

3. "sensory speech"

4. involuntary humming

5. Motor speech

Bonpoc: NQ32

The cerebral cortex of a child is similar Iin structure to the cortex of an


adult by:

1) 8 years old

2) 2 years old

3) 10 years old

4) 4 year S Old

5) 6 years old

Bonpoc: NQ33

The child is 3 years old. Diagnosed with meningeal syndrome. Specify what
indicator in the analysis of cerebrospinal fluid will confirm this
diagnosis

1) protein-5g I

2) Chlorides-700mg%
3) Glucose

4) cytosis- neutrophils of 30 cells

5) cytosis - eosinophils of 30 cells

Bonpoc•. NQ34

The child crawls, lies down, recognizes familiar faces, performs "okay",
"give me a pen", "like a dog barks" and so on. Determine the age of the
child:

1) 6 month.

2) 2 month.

3) 8 month.

4) 4 month.

5) 10 month.

Bonpoc: NQ35

Child 5 months old. On examination, convulsions are noted, muscle tone is


increased, reflexes are increased. Diagnosed with Convulsive syndrome. What
instrumental research method should be used to determine the pathological
focus:

1) Doppler

2) Electroencephalography

3) echoencephalography

4) craniography

5) rheoencephalography

Bonpoc: NQ36

Which imaging technique best identifies post-traumatic hematoma of the


brain?

1) Computed tomography (CT)

2) Radiography

3) Positron emission tomography (PET)


4) Myelography

5) Magnetic resonance imaging (MRI)

Bonpoc: NQ37

What is the first line imaging modality for evaluation of congenital


anomaly of brain

1) Neurosonography

2) Magnetic resonance imaging (MRI)

3) Computed tomography (CT)

4) Positron emission tomography (PET)

5) Radiography

Bonpoc: NQ38

An axial view of vertebral body on the CT-scan image demonstrates disk


bulging which involve entire disc circumference. Which imaging modality is
the best for confirmation this diagnosis and evaluation of soft tissue
involvement?

1) Positron emission tomography (PET)

2) Magnetic resonance imaging (MRI)

3) Radiography

4) Myelography

5) Computed tomography (CT)

Bonpoc: NQ39

A 26-year-old guy came to the clinic with complaints of low back pain,
which appeared after playing sports in the gym. Clinical examination
revealed suspicious of disk herniation. Which imaging technique is the best
to clarify this diagnosis?

1) Positron emission tomography (PET)

2) Radiography
3) Myelography

4) Computed tomography (CT)

5) Magnetic resonance imaging (MRI)

Bonpoc: NQ40

A 47-year-old man with complaints of persistent headaches, CT scans


revealed a cerebral artery aneurysm. Condition of patient was identified as
requiring immediate surgical intervention. What imaging
study is necessary to be done before the surgery?

1) Neurosonography

2) Positron emission tomography (PET)

3) Magnetic Resonance angiography

4) Diffusion-Weighted Imaging

5) Transcranial Doppler ultrasound

Bonpoc•. NQ41

Biological conditions for the development of neurosis:

1) Hereditary predisposition

2) Unsatisfactory living conditions

3) Features of sex education, etc

4) Unfavorable marital status

5) Inconvenient workplace or transport

Bonpoc: NQ42
At what cerebral edema is the blood-brain barrier disturbed?

1) Hydrostatic

2) Vasogenic

3) Osmotic

4) Cytotoxic

5) Interstitial
Bonpoc: NQ43
Non-neurological causes of cerebral edema:

1) Brain tumors, trauma

2) Brain hemorrhage

3) Malignant hypertension

4) Brain injuries

5) Ischemic stroke

Bonpoc: NQ44

Impaired mobility of half of the body is indicated as:

1) Tetraplegia

2) Monoplegia

3) Triplegia

4) Athetosis

5) Hemiplegia

Bonpoc: NQ45
Q/The psychological methods of pain relief include:

1) Massage

2) Infiltration anesthesia during surgery

3) Acupuncture

4) Excision of scars and neuromas

5) Hypnosis

Bonpoc: NQ46

A 25-year-old patient turned to a doctor complaining that she was


constantly haunted by the thought that she had not turned off the gas at
home and was checking 4-5 times a day. What kind of neurosis can be
assumed:
1. Hysterical neurosis

2. Obsessive compulsive disorder

3. Social phobias

4. The patient has no neurosis

5. Neurasthenia

Bonpoc: NQ47

A patient was diagnosed with a metastatic tumor of the left frontal lobe of
the cortex, and is being prepared for tumor removal. On examination, the
patient's right arm and right leg do not move. What is the movement
disorder in the patient:

1. Paraplegia

2. Hemiplegia

3. Epilepsy

4. Chorea

5. Tetraplegia

Bonpoc: NQ48

In practice, a medical student mistakenly infused 400 ml of hypotonic


solution intravenously into a patient with hypoglycemia. Towards the end,
the patient lost consciousness, was transferred to the intensive care unit
and was diagnosed with cerebral edema. What brain edema can be assumed:

1. Hydrostatic

2. Interstitial

3. Vasogenic

4. Cytotoxic

5. Osmotic

Bonpoc: NQ49

A 28-year-old patient came to the polyclinic with complaints that after


talking with her friend, undergoing surgery, is haunted by the thought of
breast cancer. The oncologist did not find any pathology. Realizing that
there is no reason for concern, she cannot bring herself not to think about
it. Recently she has lost weight, gets tired quickly, complains of
insomnia, anxiety, irritability, poor appetite. What type of neurosis does
this patient have:

1. Neurasthenia

2. The patient has no neurosis

3. Obsessive compulsive disorder

4. Hysterical neurosis

5. Social phobias

Bonpoc: NQ50

A 47-year-old man, complaints of severe headaches, dizziness, nausea,


vomiting. The diagnosis was intracranial hypertension, impaired outflow of
cerebrospinal fluid. 3 hours later he was transferred to intensive care
with a diagnosis of cerebral edema. What type of cerebral edema did this
patient develop:

1. Vasogenic
2. Osmotic
3. Interstitial
4. Hydrostatic
5. Cytotoxic

Q.When the corpus callosum is damaged we observe disturbances of:

1.
Analytical thinking

2. Reciprocal coordination of movements

3. Hearing and tactile sensitivity

4. Central vision

5. Long-term memory

Q.The syndrome of the lesion of the ante' jot horn of the spinal cord is
characterized by the:

1. Peripheral paralysis in the muscles of this segment

2. Dissociated sensitivity disorder


3. Violation of temperature pain sensitivity, by conductive type

4. Loss of deep sensitivity

5. Sweating disorder

Q. Gait in cerebellar damage?

1. shuffling

2. spastic

3. steppage

4. atactic

5. hemiparetic

Q,Brain asymmetry is caused by ?

1. Different structure of cerebral hemisphere

2. Temporary dominance of one or another hemisphere

3. Various blood supply to the hemisphere

4. Different functions of left and right hemisphere

5. Different weight of each hemisphere

Q,Combination of impaired swallowing, dysphonia, dysarthria, absense of a


pharyngeal reflex indicates a lesion of:

1. midbrain

2. pons

3. Brainstem

4. frontal lobe

5. medulla oblongata

Q.Brown-Séquard syndrome is lesion of:

1. Spinal cord involvement above the cervical thickening

2. spinal cord in the area of the anterior horn

3. spinal cord at the level of the cervical thickening

4. Spinal cord injury at the lumbar level


5. Half lesion of the spinal cord segment

Q,Extramedullary compression lesion syndrome:

1. Progresses rapidly

2. Descending flow type

3. More often in the thoracic region or in the cauda equina

4. More often in the area of thickening

5. No block of cerebrospinal fluid

Q.Which of the following symptoms are characteristic of lesions of the


spinal cord in the cervical enlargement (C5- D 2):

1. peripheral paralysis of the arms, central paralysis of the legs

2. dysfunction of the pelvic organs of the peripheral type

3. central tetraplegia

4. loss of deep sensitivity while the superficial

5. shortness of breath, hiccups

Q.Which cranial nerves typically affected in Fovilleis syndrome:

1. VII , VIII pairs


2. VI Vll , Vlll pairs

3. VI and Vll pairs

4. IX and X pairs

5. III pair

Q.The pathological reflex of the extensor group is the reflex:

1. Palmar - chin

2. Bekhtereva

3. Rossolimo

4. Oppenheim

5. Zhukovsky

Q.Broca’s center is located at:


1. In the left-handed posterior third of the parietal lobe of the brain
on the right

2. In left-handers, in the posterior tÅird of the gyrus of the left


hemisphere

3. In the lower part of the cerebellar hemispheres

4. In right-handers in the posterior third of the occipital lobe of the


brain on the left

5. In right-handers in the posterior third of the gyrug of the lef!


hemisphere

Q. pathogenesis of brain coma related with:

1. Corpus callosum

2. Pituitary

3. Cerebellum

4. Hippocampus

5. Visual crossover

Q. Mom brought her 11 -year-old daughter to a neuropathologist with


complaints of involuntary movements in all extremities, which disappear in
sleep. What syndrome does this patient have?

1. Parkinsonian syndrome

2. Akinetic-rigid syndrome

3. Alternating syndrome

4. Hypertensive-hypokinetic syndrome

5. Hypotonic-hyperkinetic syndrome

Q. The leading sign that distinguishes sensitive from cerebellar ataxia is:

1.
test for adi a dohokinesis

2. violation of deep sensitivity

3. instability in the Romberg test

4. increased dizziness when turning the head

5. intentional tremor
Q.A 15 year old patient with complaints of a feeling of numbness and.also
difficult to localize spilled pains of a burning character in the left half
of the body What type of sensivity should be examined in a patient to
determine the safety of complex types of sensivity?

1. Streognosis

2. Temperature

3. Vibration

4. Painful

5. Articular muscular

Q.17-year-old patient comes to the physician complaining on numbness on his


right side of the body. Physical examination found decreasing deep
sensations on this site. What kind of sensation needed to be check for
evaluation of his superficial sensation?

1. Stereognosis

2. Vibration

3. Pain

4. Pressure

5. Proprioception
Q.During the survey the patient revealed: unsteadiness when walking,
especially in the dark and with closed eyes, instability in the
Romberg position, decreased muscle tone in both legs, a feeling of
crawling in them. The patient confuses the name of the fingers and
the direction of passive movements in them. Name the existing
symptom and indicate the location of the lesion.

I. Vestibular ataxia, vestibulocochlear nerve

II. Sensitive ataxia, lateral cords of the spinal cord

III. Cerebellar ataxia, cerebellar vermis

IV. Cortical ataxia, frontal lobes

V. Sensitive ataxia, posterior cords of the spinal cord

Q.When the inner capsule is damaged on the left, the following symptoms are
revealed:
I. Hemiparesis, hyperreflexia, spastic tone tn the limbs, positive
pathological reflexes on the right

II. Peripheral paresis of the upper limbs, spastic paraparesis of the


lower limbs

III. Peripheral monoparesis on the tight, hyporeflexia, hypotrophy, muscle


hypotonia

IV. Lower "flaccid paraplegia”. hypotrophy, fibrillary twitching

V. Monoparesis on the tight With spastic tone, positive pathological


carpal

Q.Loss of sensitivity in the form of "gloves" and "socks" is characteristic


of sensitivity disorders of the type:

1. Polyneuritic

2. Segmental-dissociated

3. Segmental-radicular

4. Guide spinal

5. Cortical

Q. A 15year old patient complaints of a feeling of numbness and also


difficult to localize spilled pains of a burning character in the left half
of the body. What type of sensivity disorder the patient has?

1. Conductive

2. Segmental

3. Cortical

4. Dissociated

5. Thalamic

Q.When examining the neurological status of the patient, speech impairment


was found in the form of difficulty in choosing words, fuzzy pronunciation.
In this case, the patient understands the speech addressed and performs the
commands. No clinical signs of atactic, bulbar and pseudobulbar syndrome
were found. What symptom is likely to be present in this patient?

I. Motor aphasia
II. Sensory aphasia

III. Amnestic aphasia

IV. Ideatorial aptaxia

V. Semantic aphasia

Q . Mother brought a 13 year old child to neurologist with complaints of


choking, pouring out liguid food from the nose while swallowing, hoarseness
of the voice. Where from the following structures it is possible to
localize the lesion?

I. medulla

II. cerebellar worm

III. cervical thickening

IV. cerebellar hemispheres

V. basal nuclei

Q . A 25-year-old patient turned to a neurologist at the clinic with


complaints of severe, paroxysmal stabbing, shooting pains in the lower jaw
on the right. What reflex should be studied in this patient?

I. palatine

II. corneal

III. conjunctival

IV. Jaw jerk

V. gag

Q . A mother brought her 13-year-old daughter to the clinic to a


neuropathologist with complaints of a twisted face to the left, inability
to close her right eye, pouring out liquid food from the right corner of
her mouth. When examining the neurological status, Bell's symptoms were
found, the racket on the right. What symptom in this case will exclude a
lesion at the level of the motor nucleus of the 7th pair of CN ?

I. Hyperacusis
II. Ataxia

III. Lachrymation

IV. Hypogeusia

V. Apraxia

Q . A 15 year old patient with complaints of a feeling of numbness and also


difficult to localize spilled pains of a burning character in the left half
of the body. What of sensivity should be examined in a patient to determine
the safety of complex types of sensivity?

I. Discriminatory

II. Pain feeling

III. Temperature

IV. Articular muscule

V. Vibration

Q . The angular artery anastomoses with the:

I. dorsal artery of the nose

II. superior labial artery

III. branches of the parotid gland

IV. ophthalmic artery

V. posterior ear artery

Q . Sinus of the dura mater, communicating with the angular vein:

I. cavernous

II. inferior petrosal

III. superior petrosal

IV. superior sagittal

V. transverse

Q . What is the characteristic 01 the subperiosteal hematoma of the fronto-


parietal-occipital region?
I. has a diffuse character within the fronto-parieto-occipital region

II. spreads within one bone

III. it is difficult to give a clear characteristic

IV. has the shape of a bump

V. freely spreads to the tissue of the face

Q. What is the characteristic of the hematoma of the subcutaneous tissue of


the fronto-parieto-occipital region?

I. has the shape of a bump

II. it is difficult to give a certain characteristic

III. freely spreads to the subcutaneous tissue of the temporal region and
face

IV. spreads within one bone

V. has a diffuse character within the fronto-parieto-occipital region

Q . The projection line of the excretory duct of the parotid salivary


gland:

I. from the external auditory canal to the middle of the distance from
the wing of the nose to the corner of the mouth

II. from the base of the ear tragus to the corner of the mouth

III. from the corner of the lower jaw to the corner of the mouth

IV. in the middle of the body of the lower jaw

V. from the base of the tragus to the wing of the nose

Q .Streak irritation of the sole along the outer edge of the foot to
determine the reflex:

I. Galanta

II. Moro

III. Bauer

IV. Babkin

V. Babinsky
Q. Determine specify the features of the composition of the cerebrospinal
fluid of the newborn:

I. transparent, many lymphocytes

II. xantochromic, high in protein

III. opalescent, protein absence

IV. Colorless, protein content is minimal

V. muddy, many neutrophils

Q . The cwell, rolls over from his stomach to his back, reacts to his name
babbles appear, stretches and grabs with his hand, holds toys in his hands
and examines them. Determine the age of the child:

I. 11 month.

II. 2 month.

III. 3 month.

IV. 8 month.

V. 6 month.

Q . The child holds his head well, keeps an object in the field of vision
for a long time, quickly responds with a smile when communicating, performs
"reciprocal buzzing", grabs objects that are put in his hand. Determine the
age of the child:

I. 8 month.

II. 6 month.

III. 3 month.

IV. 4 month.

V. 10 month.

Q . On examination, the child sits on his own, turns from stomach to back
and from back to stomach, distinguishes strangers from loved ones, takes a
toy from different positions, crawls, pronounces syllables. Determine the
age of the child.

I. 6 month
II. 12 month

III. 7 month

IV. 5 month

V. 10 month

Q .Which imaging technique best identifies post-traumatic hematoma of the


brain?

I. Computed tomography (CT)

II. Myelography

III. Radiography

IV. Magnetic resonance imaging (MRI)

V. Positron emission tomography (PET)

Q.On axial view of spine CT-scan image there is disk herniation where the
base is wider then it's dome. Identify the type of herniation

a. Extrusion

b. Protrusion

c. Bulging

d. Sequestration

e. Desiccation

Q . A 32-year-old man was admitted to the hospital, to emergency


department. He got a car accident and was unconscious. Clinical
examinations revealed a spinal fracture. What imaging technique is used as
the first line study to identify the nature of the fractures.

1. Radiography

2. Magnetic resonance imaging (MRI)

3. Positron emission tomography (MAIN TRUNKIMAGWHICH IMAGINF THE )

4. Myelography

5. Computed tomography (CT)

Q . A I-month old baby was brought to the hospital of the neonatology


department. The child received 6 Apgar score at birth. The pregnancy
proceeded with a severe viral infection. At the time of hospitalization,
the baby had a fever about 38 C. During clinical examination seizures were
observed. What imaging study is the first line to clarify the diagnosis of
encephalopathy?

1. Neurosonography (NS)

2. Electroencephalography (EEG)

3. Positron emission tomography (PET)

4. Magnetic resonance imaging (MRI)

5. Computed tomography (CT).

Q . A 67 yr old woman has been diagnosed with breast cancer. She had
surgery of breastectomy 6 years ago. She regularly conducts postoperative
monitoring. Recently she had complaints of headaches, seizures have been
repeated several times. What imaging study should be done to determine the
metastases in the brain?

1. Positron emission tomography (PET)

2. Neurosonography

3. Magnetic resonance imaging (MRI)

4. Computed tomography

5. Transcranial Doppler ultrasound

Q . An increase in the activity of the antinociceptive system is possible


with:

1. Pain receptor blockade

2. The onset of pathologically enhanced excitation in the thalamus

3. Administration of morphine (an analogue of endorphins)

4. Cutting or reducing impulses along C-fibersl

5. Demyelination of A-delta fibers

Q .Impaired mobility of half of the body is indicated as:

1. Athetosis

2. Monoplegia

3. Hemiplegia

4. Tetraplegia
5. Triplegia

Q . The social factors in the development of neuroses include:

1. Information overload

2. Overeating

3. The state of the endocrine organs

4. Past illnesses

5. Types of higher nervous activity

Q .Slow antegrade axonal transport provides:

1. Delivery of trophogens to the neuron

2. Delivery of substances for synaptic activity

3. Delivery of trophogens into tissues

4. Penetration of viruses into the nervous system

5. Penetration of antibodies into the nervous system

Q . This form of cerebral edema occurs 'in obstructive hydrocephalus due to


a rupture of the CSF-brain barrier:

1. Interstitial

2. Vasogenic

3. Cytotoxic

4. Hydrostatic

5. Osmotic

Q .An excellent student of the 5th year applied to the reception,


complaining that she was afraid to read to speak at scientific conferences,
she was afraid to be in a humiliating position. What kind of neurosis does
the student have:

1. Social phobias

2. Obsessive compulsive disorder

3. Neurasthenia

4. Hysterical neurosis

5. The patient has no neurosis


Q . A 50-year-old man complains of compressive chest pains on the left.
Anamnesis: works as a broker, smokes for 20 years. What type of pain can
this pain be attributed to:

1. Phantom pain

2. Epicritical

3. Visceral pain

4. Causalgia

5. Thalamic pain

Q . After a household injury, a 18-year-old patient developed persistent


dizziness, nystagmus, ataxia, asthenia. About dysfunction of which
structures of the brain does this indicate:

1. basal ganglia

2. substantia nigra

3. motor cortex

4. cerebellum

5. vestibular nuclei

Q . A 68-year-old patient suffers from type 1 diabetes mellitus. Due to


chronic renal failure, he receives hemodialysis 2 times a week. Once again,
during hemodialysis, she remembered that she had not given an insulin
injection, and received insulin. After 20 minutes, she lost consciousness,
was transferred to intensive care, where she was diagnosed with cerebral
edema. What cerebral edema can develop in this patient:

1.
Cytotoxic

2. Osmotic

3. Hydrostatic

4. Interstitial

5. Vasogenic

Q . The mother of a 24-year-old girl turned to a psychiatrist with a


complaint that her daughter was afraid for 2 years and did not leave the
house at all. From the anamnesis: stuck in an elevator for 3 hours, after
which she does not leave the house at all. What kind of neurosis can be
assumed:
1. Neurasthenia

2. The patient has no neurosis

3. Hysterical neurosis

4. Agoraphobia

5. Social phobias

Q.Which imaging technique is the first choice for evaluation cerebral


aneurysm?

1. Digital Subtraction Angiography (OSA)

2. Computed Tomography Angiography

3. Magnetic Resonance Angiography

4. Fluoroscopy

5. Dopplet ultrasound

Q.The onset of pain during deafferentation is due to the fact that

1. Reduced impulse on C-fibets

2. Activation Of cells of gelatinous substance is in progress

3. Reduced impulse On A-delta fibers

4. Increased on C-fibers

5. Increased impulse on A-delta fibers

Q.cancer. She had surgery of breastectomy 6 years ago. She regularly


conducts postoperative monitoring. Recently she had complaints of
headaches, seizures have been repeated several times. What imaging study
should be done to determine the metastases in the
brain?

1. Positron emission tomography (PET)

2. Computed tomography

3. Transcranial Doppler ultrasound

4. Neurosonography

5. Magnetic resonance imaging (MRI)


Q.Imaging modality which can evaluate anatomical and functional state of
patients with brain tumor

1. Neurosonography

2. Magnetic resonance imaging (MRI)

3. Computed tomography (CT)

4. Molecular imaging

5. Skull X-ray

Bonpoc: NQ34

The child holds his head well, keeps an object in the field of Vision for a
long time, quickly responds with a smile when communicating, performs
"reciprocal buzzing", grabs objects that are put in his hand. Determine the
age of the child:

1. 10month.

2. 6 month.

3. 8 month.

4. 4 month.

5. 3 month.

Bonpoc: NQ49

Q.A 36-year-old patient, a builder, was admitted by an ambulance to the


neurological department, fell from the second floor during work. There was
a surgical operation due to a fracture of 2-3 lumbar vertebrae. The next
day after regaining consciousness, he cannot move both legs. What is the
patient's pathology?

1. Tetraplegia

2. Chorea

3. Hemiplegia

4. Epilepsy

Q.A 32-year-old man was admitted to the hospital, to emergency department.


He got a car accident and was unconscious. Clinical examinations revealed a
spinal fracture. What imaging technique is used as the first line study to
identify the nature of the fractures.

1. Positron emission tomography (PET)

2. Myelography
3. Computed tomography (CT)

4. Radiography

5. Magnetic resonance imaging (MRI)

Q.A 65-year-old man, very severe, unbearable, burning pains in the right
side of the body, was operated on 2 months ago for lung cancer, metastases
in the thalamus were identified. Pain is not relieved by narcotic
analgesics, the patient is depressed, wants to commit suicide. What kind of
pain does this patient have:

1. Epicritical

2. Thalamic pain

3. Protopathic

4. Visceral pain

5. Causalgia

Bonpoc: NQ2

Brown-Séquard syndrome is:

1. Motor disturbances and disturbances of deep sensitivity on the side


of the lesion, disorder of superficial sensitivity on the opposite
side.

2. Central lower paralysis on the side of the lesion, without sensory


disturbances, fascicular twitching is possible

3. Peripheral paralysis with fascicular twitching


on the side of the lesion without sensitivity disorders

4. Peripheral paralysis with sensory disorders in the innervation zone


of this nerve

5. Loss of function syndrome: central monoparesis on the opposite side

Q.Imaging modality which can evaluate brain tumor changes at cellular level

1. Skull X-ray

2. Magnetic resonance imaging (MRI)

3. Computed tomography (CT)

4. Neurosonography

5. Molecular imaging

Q.When the trigeminal (V) nerve is damaged we observe:

1. lacrimation and prosoparesis


2. prosoparesis

3. violation of the sensitivity of the facial skin

4. hearing loss

5. hyperacusia

Bonpoc: NQ47

In practice, a medical student mistakenly infused 400 ml of hypotonic


solution intravenously into a patient with hypoglycemia. Towards the end,
the patient lost consciousness, was transferred to the intensive care
unit and was diagnosed with cerebral edema. What brain edema can be
assumed:

1. Hydrostatic

2. Osmotic

3. Interstitial

4. CytotOXiC

5. Vasogenic

Q.Periosteal reflexes include:

1. upper abdominal reflex

2. plantar reflex

3. cremasteric reflex

4. middle abdominal reflex

5. carporadial reflex

Q.The third neuron conducting pain is localized in:

1. Cerebral cortex

2. Trigeminal ganglion

3. Spinal cord

4. Spinal nodes

5. Thalamus

Q.Loss of sensitivity is called:

1. Atonya

2. Asthenia
3. Anesthesia

4. Apathy

5. Ataxia

Q.Biological conditions for the development of neurosis:


1.

2. Hereditary predisposition

3. Inconvenient workplace or transport

4. Unfavorable marital status

5. Features of sex education, etc

6. Unsatisfactory living conditions

Bonpoc: NQ6

Combination of impaired swallowing, dysphonia, dysarthria, absense of a


pharyngeal reflex indicates a lesion of:

1. Brainstem

2. pons

3. medulla oblongata

4. midbrain

5. frontal lobe

Q.Morphological asymmetry is:

1. Different hemisphere weights

2. Different blood supply to the hemispheres

3. Different structure of the hemispheres

4. Each hemisphere processes information of a certain type

5. Various content of biologically active substances

Bonpoc: NQ5

Increased finger like impressions on X- rays of skull indicate:

1. Arterial hypertension

2. Intracranial hypertension
3. Intracranial hypotension

4. Arterial hypotension

5. Mental changes

Bonpoc: NQ7

What symptom can rule out meningitis?

1. Brudzinsky symptom

2. Nerry symptom

3. Rigidity of occipital muscles

4. Lesage symptom

5. Kernigach symptom

Q.One of the main changes caused by chronic pain:

1. Productivity

2. Deep dream

3. Laughter

4. Mood boost

5. Depression

Q.A 47-year-old man, complaints of severe headaches,

dizziness, nausea, vomiting. The diagnosis was

intracranial hypertension, impaired outflow of

cerebrospinal fluid. 3 hours later he was transferred to

intensive care with a diagnosis of cerebral edema. What

type of cerebral edema did this patient develop:

1. Interstitial

2. Vasogenic

3. Hydrostatic

4. Osmotic

5. Cytotoxic

Q. ... it is a developing trophic disorder that is caused by loss or


alteration of neural stimuli:
1. Hysterical neurosis

2. Secondary pain

3. Neurodystrophic process

4. Neuropathic pain

5. Primary pain

Q.A 15year old patient complaints of a feeling of numbness and also


difficult to localize spilled pains of a burning character in the left half
of the body. What type of sensivity disorder the patient has?

1. Dissociated

2. Conductive

3. Segmental

4. Thalamic

5. Cortical

Q.Lower flaccid paraparesis with sphincter disorders occurred in case of


lesion:

1. of the spinal cord at the level of the cervical thickening

2. Spinal cord injury at the lumbar level

3. Spinal cord involvement above the cervical thickening

4. of epiconus

5. of the spinal cord at the level of the thoracic region

Q. The child is 3 months old. Complaints of anxiety, poor sleep, shudders


during sleep. On examination: physiological reflexes are active, muscle
tone is increased, the gaze is fixed, the size of the head is normal.
Determine which of the symptoms indicates increased tone:

1. Head tilted forward

2. Clenching the fingers into a fist

3. Arms out to the sides

4. Legs straightened

5. Fingers and toes are straight


Q.The syndrome of decreased mental activity with a preserved level of
wakefulness includes:

1. Amentive syndrome

2. Dementia

3. Twilight clouding of consciousness

4. Coma

5. Oneroid syndrome

Q. Characteristic of lesion of the dorsal horns of the spinal cord:

1. Conductive type of sensitivity disorder

2. Cortical type of sensitivity disorder

3. Polyneuritic type of sensitivity disorder

4. Segmental - the radicular type of sensitivity disorders

5. Segmental-dissociated type of sensory impairment

Q. A 13- year-old patient consulted a neuropathologist at the polyclinic


with complaints of weakness in his right arm. Examination of the
neurostatus revealed central paresis of the right hand. What reflex should
be examined to confirm that the patient has just central paresis?

1. Palmar chin

2. Proboscis

3. Marinescu Rodovici

4. Sucking

5. Bekhterev

Q.In a 9-year-old patient, an examination of the neurological status


revealed lower paraparesis with a decrease in muscle strength to 3 points,
as well as dysfunction of the pelvic organs by the type of urinary
retention. What reflex will indicate the central character
of paraparesis in the legs?

1. Sucking

2. Ankle

3. Knee

4. Rossolimo
5. Marinescu Rodovici

Q.Mom brought her 11-year-old daughter to a neuropathologist with


complaints of involuntary movements in all extremities, which disappear in
sleep. What syndrome does this patient have?

1. Alternating syndrome

2. Akinetjc-rigid syndrome

3. Hypotonic-hyperkinetic syndrome

4. Hypertensive-hypokinetic syndrome

5. Parkinsonian syndrome

Bonpoc: NQ19

The leading sign that distinguishes sensitive from cerebellar ataxia is:

1. increased dizziness when turning the head

2. intentional tremor

3. violation of deep sensitivity

4. test for adi a dohokinesis

Q.Mom brought her 1 2 year old daughter to the clinic to a neuropathologist


with complaints of involuntary , violent, pretentious, worm-like movements
in both hands that disappear in sleep . What syndrome does this patient
have?

1. Akinetic-rigid syndrome

2. Hypertensive-hypokinetic syndrome

3. Parkinsonian syndrome

4. Hypotonic-hyperkinetic syndrome

5. Alternating syndrome

Q. Mother took a 13 year old child to neurologist with complaints of


choking, pouring out liguid food from the nose while swallowing, hoarseness
of the voice. Where from the following strusctures it is possible to
localize the lesion?

1. nuclei 3,4,6 pair CN


2. cerebellar worm

3. basal nuclei

4. nuclei pair CN

5. cerebellar hemispheres

Q.A 15 year old patient with complaints of a feeling of numbness and also
difficult to localize spilled pains of a burning character in the left half
of the body. What types of sensivity should be examined in a patient to
determine the safety of complex types of sensivity?

1. Articular muscule

2. Vibration

3. Discriminatory

4. Temperature

5. Pain feeling

Q.A 29-year-old patient has turned to a neurologist at the clinic with


complaints of severe, paroxysmal pains of a stabbing, shooting character in
the lower jaw on the right. Which pain points should be examined in this
patient?

1. Trigeminal points

2. Erb points

3. Neri points

4. Vale points

5. Gar points

Q.What is the backbone of the Shipo triangle?

1.
the supraspin of the temporal bone

2. the mastoid notch of the temporal bone

3. the groove of the occipital artery of the temporal bone

4. the mastoid crest of the temporal bone

5. the posterior edge of the external auditory canal


Q. Projection of the main trunk of the middle meningeal artery using the
KrenIein-Bryusova scheme:

1. at the intersection of the back vertical and the upper horizontal

2. at the intersection of the middle vertical and the upper horizontal

3. at the intersection of the back vertical and the upper horizontal

4. at the intersection of the front vertical and the lower horizontal

5. at the intersection of the anterior vertical and upper horizontal

Q.What is the characteristic of the subperiosteal hematoma of the fronto-


parietal-occipital region?

1. has a diffuse character within the fronto- parieto-occipital region

2. has the shape of a bump

3. spreads within one bone

4. it is difficult to give a clear characteristic

5. freely spreads to the tissue of the face

Q.Thrombosis of which sinus of the dura mater develops in some forms of the
carbuncle Of the upper lip, wings of the nose or the superciliary arch:

1. cavernous

2. wedge-parietal

3. lower sagittal

4. rectus

5. upper sagittal

Q.The cerebral cortex of a child is similar in structure to the cortex of


an adult by:

1) 6 years old

2) 4 years old

3) 10 years old

4) 2 years old

5) 8 years old

Q.One of the main changes caused by chronic pain:

1) Mood boost

2) Productivity
3) Laughter

4) Depression

5) Deep dReam

Q.Indicate the nasal passage into which the nasolacrimal canal opens

1) Lower

2) Common

3) Middle

4) Posterior

5) upper

Q.Epilepsy is caused by the appearance of a generator pathologically


increased excitement in:

1) Cerebellum

2) Spinal cord

3) Thalamus

4) Cerebral Cortex

5) Brain Stem

Q.The sinus of dura mater. often damaged by injuries of the fornix of the
cerebral section of the head:

1) Upper sagittal

2) Cavernous

3) Lower sagittal

4) Sigmoid

5) Straight

Q. The child is 10 years Old. Based on clinical and laboratory research


methods, the diagnosis was ma cerebrospinal fluid is characteristic of this
syndrome.

1) Glucose- 2.0-3.0 mmol / l

2) protein - 1-2g / l

3) color- colorless
4) cytosis- 15-20cl/ mm3

5) chlorides- 500-550mq%v

Q. the connection of the saphenous veins of the cerebral part of the head
with the sinus of the dura meter is carried out through the:

1) Venous sinus

2) Veins of dura mater

3) Pachyon granules

4) Saphenous veins

5) Diploic vein

Q.Compression by the tumor p-gocess in the parotid salivary gland of which


nerve leads to the

1) glossopharyngeal

2) zygomatic

3) facial

4) trigeminal

5) auricular temporal

Q. What is the characteristic of subgleal hematoma of the fronto-parieto-


occipital region?

1) moves freely within the fronto-parieto-occipital region

2) has the shape of an oval oriented in the longitudinal direction

3) has a pulsating character

4) it is difficult to give a clear characteristic

5) corresponds to the shape of the underlying bone

Q. When a fracture of which bone occurs bleeding and liquorrhea from the
external auditory canal?

1) Temporal

2) Maxillary

3) Occipital

4) Pterygoid

5) Parietal
Q.What kind of reaction to the acute pain?

1) Lowering blood pressure

2) Increased heart rate

3) Decreased breathing

4) Lower blood glucose

5) Decrease in heart rate

Q.Characteristic of segmental dissociated sensory

1) Preservation of pain sensitivity With of temperature sensitivity

2) Loss of pain sensitivity with preservation of temperature sensitivity

3) loss of pain and temperature sensitivity With preservation Of deep


sensitivity

4) of deep sensitivity and preservation of pain sensitivity

5) Loss of deep and superficial sensitivity

Q.15th years old patient complained of numbness and pain on the left side
of the body after analyzing there was left sided .Where is the lesion?

1. Thalamus left

2. Inner capsule left

3. Thalamus right

4. Trunk left

5. inner capsule

Q. Dissociated sensitivity disorder (loss of superficial with preserved


deep) on both sides in the segment area (like a jacket) occurred in case of
lesion of

1. Anterior horn

2. Half segment Lesion

3. Lateral horn lesion

4. lesion of anterior gray commissure

5. Posterior horn

Q. Using which test the light paresis can be detected ?


1. Knocking

2. Kweckenstedt

3. Ashner

4. Romberg

5. Barre

Q. What informative research is used for diagnosing meningitis?

1. Antigraphy

2. CSF research

3. X-ray-diagnostic

4. Electroencephography

5. Electroneurography

Q. Which of the following posture is characterize the damage Of pyramidal


fibers in the inner capsule?

1. supplicant pose

2. Wernicke-Mann pose

3. Romberg pose

4. Point Dog pose

5. Mannequin pose

Q. The localization sensory is:

1. An ability to define numbers, letters written on skin of hand

2. An ability to define two touches at the same time

3. Recognition of known objects placed on hands

4. An ability to find an irritated point with finger

5. An ability to define passive joint movements

Bonpoc: NQ7

in which of the following conditions do pathological reflexes occur?

1. striatum lesion
2. cerebellar lesion

3. peripheral paralysis

4. defeat pallidum

5. central paralysis

Bonpoc:

lower central paraparesis with sphincter disorders occurred in case of


lesion:

1. of the spinal cord at the level of the thoracic region

2. of epiconus

3. Spinal cord involvement above the cervical thickening

4. Spinal cord injury at the lumbar level

5. of the spinal cord at the level of the cervical thickening

Bonpoc: NQI

Q.A discrimination sensitivity is:

1. An ability to define two touches at the same time

2. Recognition of known objects placed on hands

3. An ability to define numbers, letters written on skin of hand

4. An ability to find an irritated point with finger

5. An ability to define passive joint movements

Bonpoc: NQ40

A 46-year-old woman was admitted to the rehabilitation center. She has type
2 diabetes and high blood pressure. She has a history of stroke a month
ago. At the moment the condition has improved. What imaging study should be
used to control treatment and predict further condition.

1. Neurosonography

2. Magnetic resonance imaging (MRI)

3. Computed tomography

4. Positron emission tomography (PET)

5. Transcranial Doppler ultrasound


Bonpoc: NQIO

Q.What type of sensation js related to deep one?

1. Temperature sensitivity

2. Vibratory sensation

3. Localization

4. Stereognosis

5. Pain

Q. which of the following symptoms Are characteristic of spinal cord


lesions in the upper cervical region (C1-C4)?

1. Loss of deep sensitivity while the superficial

2. Central tetraplegia, shortness of breath, hiccups

3. Peripheral paralysis of arms and legs

4. Dysfunction of the pelvic organ of peripheral types

5. Peripheral paralysis, spastic paralysis of the legs

Q. The pathological reflex of the flexor group is the reflex:

1. Oppenheim

2. Rossolimbo

3. Babinsky

4. Gordon

5. Schaeffer

Bonpoc: NQ48

In the training laboratory of pathophysiology, experiments were carried out


on dogs poisoned with isoniazid after death, morphological examination of
the brain tissue showed: the development of intracellular edema of the body
and processes of astrocytes. What type of cerebral edema can be expected?

1. Hydrostatic

2. Cytotoxic

3. Vasogenic
4. Interstitial

5. Osmotic

Bonpoc: NP46

An ambulance delivered the patient after the earthquake from anamnesis


suffers from the syndrome of inappropriate antidiuretic hormone secretion
the doctor of the emergency department diagnosed acute renal failure
cerebral edema. What type of cerebral edema did this patient develop:

1. Osmotic

2. Vasogenic

3. Cytotoxic

4. Hydrostatic

5. Interstitial

Bonpoc: NP45

Violation of higher nervous activity is called:

1. Pain

2. Paresis

3. Paralysis

4. Deafferentation

5. Neurosis

Bonpoc: NQ44

Neurological causes of cerebral edema:

1. Diabetic ketoacidosis

2. Lactic acidotic coma

3. Malignant hypertension

4. Hyponatraemia

5. Brain hemorrhage

Bonpoc: NQ43

Phantom pain is:


1. Burning pain in the area of damaged nerve trunks

2. Pain arising from pathology in internal organs

3. Perception of cold and heat exposure as pain

4. Perception of non-painful exposure as painful

5. Pain in a missing part of the body

Bonpoc: Ng42

A typical variant of neurodystrophic process :

1. Lower blood glucose

2. Pain syndrome

3. Spinal shock

4. Lowering blood pressure

5. Denervation syndrome

Q. What relates to the system of perception and transmission of the pain


signal?

1. Parasympathetic nervous system

2. Autonomic nervous system

3. Sympathetic nervous system

4. Axonal system

5. Nociceptive nervous system

Bonpoc: N937

______ is primary acoustic window uses in neurosonography brain


visualization??

1. Sphenoid fontanelle

2. Anterior fontanelle

3. Posterior fontanelle

4. Posterolateral fontanelle

5. Mastoid fontanelle

Q. which imaging technique shows anatomical structures of brain?

1. Electroencephalography (EEG)

2. Computed tomography (CT)


3. Myelography

4. Positron emission tomography (PET)

5. Functional magnetic resonance imaging (fMRI)

Bonpoc: NQ35

The child is 4 months old. Complaints of anxiety, tremors of the chin and
upper limbs, intermittent short sleep, flinching. She reacts to examination
by crying, increased muscle tone and increased reflexes. The head is
rounded, with a large fontanel, cm. There is a bulging and pronounced
pulsation of the large fontanelle. Determine what syndrome the child has:

1. meningeal syndrome

2. hyperexcitability syndrome

3. hypertensive syndrome

4. hydrocephalic syndrome

5. convulsive syndrome

Bonpoc: NQ34

The child is 6 months old. On examination, anxiety is noted. the skin is


pale, a venous pattern is expressed on the scalp, the head is large, the
size of large fontanel is 4,0x40 cm, and the divergence of the skull bones-
What are you suggesting?

1. Hydrocephalic syndrome

2. Hypertensive syndrome

3. Meningeal syndrome

4. Encephalitic syndrome

5. Convulsive syndrome

Q. Determine the psychomotor skill in child at 9-10 months?

1. Just starting to sit

2. Walks independently

3. Runs independently

4. Gets up and stands holding on to the support

5. Just starts to crawl

Bonpoc: N@27
in which venous Sinus of the hard shell of the brain is hematogenous
transmission of infection possible with furuncle of the nasolabial fold
area?

1. sinus sagittalis interior

2. sinus rectos

3. sinus petrosus major

4. sinus sigmoidius

5. sinus Cavernous

Bonpoc: NQ24

In the study of the neurological status of the patient, speech impairment


was found in the form of difficulty in choosing words, fuzzy pronunciation;
In this case, the patient understands the speech addressed and performs the
commands. No clinical signs of atactic, bulbar and pseudobulbar syndrome
were found. Where is the lesion likely located in this patient ?

1. In the temporal lobe

2. In the parietal lobe

3. In the frontal lobe

4. In the occipital lobe

5. In the basal ganglia

Bonpoc: N02()

Half lesion of the spinal cord diameter characterized by central paralysion


the side of the lesion in combination with:

1. violation of all kinds of sensitivity - to protivopo false from the


lesion side of the body

2. polyneuritic disorder of sensitivity in the distal extremities

3. violation of deep sensitivity on the side of the focus and pain and
temperature sensitivity s on the opposite

4. violation of pain and temperature sensitivity on the side of the


focus

5. violation of all types of sensitivity on the side of the lesion and


central tetraparesis

Bonpoc: NQ16

A 12-year-old patient consulted a neuropathologist at the polyclinic with


complaints of weakness in the left extremities, a feeling of numbness in
the right extremities. And also, badly feels the floor with the left leg.
Examination of the neurostatus revealed central left-sided hemiparesis, a
decrease in deep sensitivity in the left extremities, and a decrease in
superficial sensitivity in the right half of the body. Where is the lesion
located

1. Complete transverse lesion of the upper cervical segments

2. Half lesion of the upper cervical segments

3. Half lesion of the thoracic segments

4. Complete transverse lesion at the level of the cervical thickening

5. Complete transverse lesion of the thoracic segments

Q. Mom brought her 11 years old to a neuropathologist at the clinic with


complaints of involuntary movement in all limbs that disappear in sleep.in
the study of girl’s neurostatus, choreic hyperkinesis was found. How will
muscle tone change?

1. Promotion of plastic type

2. Spasmodic promotion

3. Gear type boost

4. Diffuse decrease of muscle tone

5. Clasp knife promotion

Q. Deep reflexes include:

1. Cremasteric reflex

2. Corneal reflex

3. Abdominal reflex

4. Ankle reflex

Q. Jackson syndrome is characterized

1. Lesions II pairs on the side of the lesion and central hemiparesis on


the opposite side

2. Lesions III pairs on the side of the lesion and central hemiparesis
on the opposite side

3. Lesions VII pairs on the side of the lesion and central hemiparesis
on the opposite side

4. Lesions VIII pairs on the side of the lesion and central hemiparesis
on the opposite side
5. Lesions XII pairs on the side of the lesion and central hemiparesis
on the opposite side

Bonpoc: N24

Intramedullary compression lesion syndrome is:

1. More often in the thoracic region and cauda equina

2. More often in area of thickening

3. Progresses slowly

4. Ascending flow type

5. Block of cerebrospinal fluid

Bonpoc:

Lesion of the trigeminal nerve is characterized by:

1. dysarthria

2. impaired sensitivity

3. paresis of facial muscle

4. swallowing disorder

5. positive bell’s symptom

Bonpoc: N27

Signs of central paralysis include the following symptoms:

1. fasciculations hypotrophy, hyporeflexia. clonuses of the feet

2. fibrillation hypotension hyporeflexia hypotrophy

3. clonus of the feet spastic tone in the limbs positive Pussep symptom

4. hyperreflexia sluggish tone in the limbs positive first in babinski

5. Atony areflexia positive symptom Oppenheim fibrillation

Bonpoc: N28

Damage to the posterior horn of the spinal cord leads to:

1. Disorders of deep sensitivity


2. Fascicular twitching of muscle at the level of damage

3. Disorders of movement at the level of damage

4. Disorders of sweating at the level of damage

5. Less of superficial sensitivity at the level of damage

Bonpoc: N? 10

Hypertensive- hydrocephalic syndrome includes

1. Restless shallow sleep

2. Increase in head size by 1-2 cm

3. Increased tendon reflexes

4. Muscular dystonia

5. Tremor of the limbs and chin

Bonpoc: NE 12

The defeat of the pyramidal fibers in the inner capsule causes

1. central hemiplegia central paresis of the XII and VII FMN on the
opposite side of the body

2. central tetraplegia and central paresis of the XII and VII on their

3. central hemiplegia on its own and central paresis of the XII and VII
FMN on the opposite side of the body

4. central hemiplegia and central paresis of XII and VII FMN on its side
of the body

5. central hemiplegia on the opposite side of the body and central


paresis of XII and VII on its own

Bonpoc: N223

Mother brought year old child to neuropathologist with complaints Of


headache. Vomiting. Ophthalmoscopically found on both sides of the stagnant
discs of the optic nerves. What method can be used to detect an increase in
finger like impressions in this patient?

1. Electroneuronvgraplv;

2. Electroencemiography

3. Lumber puncture
4. skull x-ray

5. MRI of the brain

Bonpoc: 'N226

Name the sinus of the duramater of the brain, which can be damaged by
trepanation of the mastoid process

1. Sigmoid

2. Straight

3. Sagittal

4. Large stony

5. Cavernous

Bonpoc: N227

What is the limitation of the shipo triangle in front?

1. The groove of the occipital artery of the temporal bone

2. The supraspin of the temporal bone

3. The mastoid notch of temporal bone

4. The mastoid crest of temporal bone

5. The posterior edge of the external auditory canal

Q. the sinus of the duramater is often damaged by injuries of the formix of


the cerebral section of the head?

1. Cavernous

2. Upper sagittal

3. Lower sagittal

4. Sigmoid

5. Straight

Bonpoc: N231

What is the presentation of cytosis in cerebrospinal fluid in healthy


children in the field Of vision:

1. basophils 3-4
2. neutrophits 10-12

3. lymhpjcytes 3-5

4. eosinophils 5-6

5. monocytes 1-2

Bonpoc: N232

Indicate symptom, which characteristic of meningeal syndrome:

1. Absence of consciousness

2. Absence of reflexes

3. Stiff neck

4. Limited mobility

5. Decreased muscle tone

Bonpoc: N234

The child is 2 years old. On examination. there is an increase in body


temperature 39, anxiety, crying$ vomiting. stiffness of the muscles of the
occiput. pronounced pulsation of the large fontanel. a positive reflex of
Brudzinsky. What are you suggesting?

1. Damage of digestive system

2. Damage of muscular system

3. Damage of endocrine system

4. Damage to the skeletal system

5. Damage of nervous system

Q. imaging modality which can evaluate functional and anatomical state of


patient of brain tumor?

1. Computed tomography (CT)

2. Neurosonography

3. Skull X-ray

4. Molecular imagining

5. MRI

Bonpoc: NE45

The neurodystrophic process in organs and tissues in the pathology of the


nervous system is caused by a violation of;
1. Trophic function

2. Regulatory function

3. Perception

4. Motor function

5. Sensitive function

Bonpoc: N246

The patient in the afternoon in the presence of relatives after the


conflict fell on the sofa rolled her eyes, She soon developed tonic cramps
in her arms. which lasted 5 minutes and replaced by clonic convulsions of
the whole body. without tongue bite. spontaneous no urination was observed.
At the time the seizure the reaction of the pupils to light was preserved.
The whole seizure lasted 15 minutes. What can be assumed:

1. Neurasthenia

2. Jacksonian Seizure

3. Hysterical neurosis

4. Epilepsy

5. Generalized tonic clonic seizure

Bonpoc: Ne48

A student from KSMA turned to complaining that she washes her hands up to
40 times a day because of fear of contamination. she always thinks that
she has microorganisms on her hands. What can be assumed with this student:

1. The patient has no neurosis

2. Obsessive compulsive disorder

3. Hysterical neurosis

4. Social phobias

5. Neurasthenia

Q. A 13- year-old patient consulted a neuropathologist at the polyclinic


with complaints of weakness in the right limbs. a feeling of numbness in
the left limbs. Examination of the neurostatus revealed central right-
sided hemiparesis. a decrease in deep sensitivity in the right extremities,
and a decrease in superficial sensitivity in the left half of the body.
What syndrome does this patient have?

1. Horner's syndrome

2. Morgan's syndrome
3. Brown-Sequard Syndrome

4. Fauville's syndrome

5. Weber's syndrome

Q. The maximum functional asymmetry or the brain is noted:

1. In childhood

2. During the period or perinatal ontogenesis

3. During puberty

4. In adulthood

5. during the neonatal period

Q. which type of the method the patient is unable to feel weight

1.
using weber compass

2. Gentle touch with brush or hair

3. Making passive movement on phalanges

4. Placing on the extended hand an objects with different weight

5. Pinprick test

Q. indicate Symptom, which characteristic of hydrocephalic syndrome

1. Normal head Size

2. The seams of the head are closed

3. Flattening of the occiput

4. Large fontanelle sinks

5. Dehiscence of the bones of the skull

Q. The middle artery of the dura mater is projected

1. In the fronto-parieto-occipital region

2. in the orbital region

3. in the temporal region


4. the mastoid process

5. in the nasal cavity

Q. Graphesthesia sensitivity is:

1. An ability to find an irritated point with finger

2. An ability to define passive joint movement

3. An ability to define numbers, letter written on the skin of hand

4. An ability to define two sensational factors with the same model

5. Recognition of known objects placed on the hands


1. The most effective method for diagnosing gonococcus is:
A. Linked immunosorbent assay
B. Cultural
C. Bacterioscopic
D. Immunofluorescent
E. Polymerase chain reaction

2. A 23-year-old female patient complains of mucopurulent discharge from the


genital tract, itching at the antenatal clinic. By examination: the mucous of the
cervix is sharply hyperemic, edematous. The discharge is profuse,
mucopurulen. uterus in anteversio-flexio, not enlarged, painless; the
appendages are not defined. What is the most likely diagnosis:
A. Vulvovaginitis
B. Bartholinitis
C. Endometritis
D. Acute salpingo-oophoritis
E. Endocervicitis.

3. For conservative treatment of Abnormal uterine bleeding the 1st line drug is:
A. oxytocin
B. NSAIDs
C. tranexamic acid
D. aminocaproic acid
E. sodium ethamsylate

4. A 16-year-old girl developed spotting from the genital tract, lasting 8 days after
a 2-month delay. The first menstruation appeared 4 months ago for 2 days, after
28 days, moderate, painless. Denies sexuality. The development is correct, well
physically built. When recto-abdominal examination of the pathology is not
detected. Нb-80 g / l. Probable diagnosis:
A. Hormone-producing ovarian tumor
B. Cervical cancer
C. Polyp of the cervix
D. Juvenile uterine bleeding
E. Endometrial polyposis

5. A mother with a 4-year-old girl turned to a pediatric gynecologist, who has


itching and redness in the external genital area, purulent discharge from the
genital tract. These symptoms occur periodically throughout the year. Genital
hygiene is observed. The child is observed by an allergist in connection with
atopic dermatitis (skin rashes on the elbows and on the face).
What is the most likely diagnosis:
A. Salpingo-oophoritis
B. Atopic vulvovaginitis
C. Endometritis
D. Endocervicitis
E. Bartholinitis

6. Patient L., 24 years old, complains of profuse vaginal discharge with an


unpleasant smell of "rotten fish". Gynecological examination: the external
genital organs and vaginal mucosa without signs of inflammation. Vaginal
discharge copious, watery, with an unpleasant odor. Internal genital organs
without pathology. During bacterioscopy of smears from the cervical canal and
urethra, "key cells" were found.What is the most likely diagnosis:

A. Bacterial vaginosis
B. Candidiasis vulvovaginitis
C. Chlamydial cervicitis
D. Atrophic colpitis
E. Bartholinitis

7. Task 2 Patient R., 25 years old, was admitted to the gynecological department
with complaints of pain in the external genital area, awkwardness when
walking, fever. Pain appeared four days ago, after hypothermia. Last period 3
weeks ago. By examination: in the area of the right labia majora, a tumor-like
formation measuring 4x3 cm is determined, the skin above it is hyperemic, hot
to the touch, fluctuation is noted on palpation. Examination of the cervix in the
speculum and bimanual examination revealed no abnormalities.
What is the most probable preliminary diagnosis:
A. Bartholine abscess
B. acute bilateral salpingitis, pelvioperitonitis
C. Torsion of the legs of the ovarian tumor
D. Necrosis of subserous myomatous node
E. Perforation of purulent formation of appendages

8. A 23-year-old female patient came to the antenatal clinic with complaints of


cramps during urination, fever up to 37.8 ° C, purulent discharge from the
genital tract. She has several sexual partners.Examination by speculum: the
mucous membrane of the urethra is hyperemic, the cervix is cylindrical,
hyperemia of the area of the external opening of the cervical canal. Discharge
from the cervical canal is purulent. Define the tactics of the antenatal clinic
doctor:
A. Prescribe an outpatient course of antibiotic therapy
B. Take swabs, culture for gonorrhea
C. Hospitalize the patient in a gynecological hospital
D. Get swabs for chlamydia infection
E. Hospitalization in the dermatovenerologic clinic

9. Patient S., 21 years old, at the reception in the antenatal clinic complains of
mucopurulent discharge from the genital tract, itching. Speculum exam: the
cervix is hyperemic, edematous, with pinpoint hemorrhages. The discharge is
profuse, mucopurulent. Vaginal examination revealed no pathology. Was
diagnosed : Endocervicitis.
Define the tactics of the antenatal clinic doctor:
A To urgently hospitalize the patient for surgical intervention
B. Antibiotic therapy, take culture for microorganisms
C. Healthy lifestyle, diet, phytohomeopathic treatment.
D. Refer the patient to a dermatovenerologic dispensary
E. Recommended a re-examination after the next menstruation

10. Duration of the menstrual cycle:


A. 21-35days
B. 28-29days
C. 25-37days
D. 24-32days
E. 35-39days

11. Preovulatory changes in the hormonal background are characterized by an


increase in the level
A. LH and decreased FSH
B. FSH and decreased LH
C. FSH and LH
D. Prolactin
E. FSH, LH and prolactin

12. The mechanism of action of hormones on the cell is due to the presence of:
A. Prostaglandins
B. Receptors
C. Specific Enzymes
D. Isoenzymes
E. Thromboxant

13. According to the WHO classification (198O), menopause is called


A. Period of stable menstrual function
B. Last menstruation
C. The length of time after the last menstruation
D. Time span after reproductive age
E. The onset of the first menstruation

14. According to the WHO classification (198O), postmenopause is called


A. Last menstruation
B. The period from the onset of menstruation to the last menstruation
C. Period of stable menstrual function
D. Period of first menstruation
E. The length of time after the last menstruation

15. Research methods of the anatomical and functional state of the vagina:
A. Inspection with speculum;
B. Combined vaginal rectal examination;
C. Cytological examination of the contents of the uterine cavity;
D. Puncture of the posterior fornix;
E. Determination of the purity of the vaginal contents.

16. Which test of functional diagnostics indicates the presence of a biphasic


menstrual cycle?
A. Pupil symptom
B. Karyopicnosis Index
C. Basal thermometry
D. Symptom of "fern"
E. Symptom of crystallization

17. The scope of the examination for pathology of the cervix


A. Colposcopy
B. Hysteroscopy
C. Ultrasound
D. Bacterioscopy
E. Laparoscopy

18. Algodismenorea is:


A. Rare and scarce menstruation
B. Painful menstruation
C. Reduced blood loss during menstruation
D. Intermenstrual meager spotting
E. Heavy periods

19. Menorrhagia is:


A. Acyclic uterine bleeding
B. Cyclic uterine bleeding
C. Painful and heavy menstruation
D. Pre- and postmenstrual bleeding
E. Change in the rhythm of menstruation

20. Metrorrhagia is
A. Change in the rhythm of menstruation
B. Increased blood loss during menstruation
C. Increase in the duration of menstruation
D. Acyclic uterine bleeding
E. Reducing blood loss during menstruation

21. Phases of the ovarian cycle:


A. Follicular and luteal
B. Follicular and ovulation
C. Ovulatory and anovulatory
D. Anovulatory and luteal
E. Desquamation and ovulation

22. For anovulatory menstrual cycle with follicular atresia is characteristic:


A. Amenorrhea
B. Algomenorrhea
C. Oligoamenorrhea
D. Polymenorrhea
E. Hypomenorrhea

23. For anovulatory menstrual cycle with follicular atresia is characteristic:


A. Amenorrhea
B. Algomenorrhea
C. Oligoamenorrhea
D. Promenomenorrhea
E. Hypomenorrhea

24. In the anovulatory menstrual cycle, basal temperature is characterized by:


A. Rise In Temperature After Ovulation
B. Lack Of Temperature Rise
C. Rise In Temperature Before Ovulation
D. Rise In Temperature Before Menstruation
E. Rise In Temperature Immediately After Menstruation

25. Anovulatory menstrual cycle is characterized by:


A. Cyclic changes in the body
B. Long-lasting follicle persistence
C. Prevalence of gestagens in phase 2
D. Prevalence of estrogen in phase 1
E. Acyclic change in the body
26. To induce ovulation, use:
A. Progestogens
B. Antiestrogens
C. Natural Estrogens
D. Prostaglandins
E. Antagonists Of Prostaglandins

20. The most effective method of preventing unwanted pregnancy is:


A. Intrauterine device
B. Britm method
C. Spermicides
D. Surgical sterilization
E. Hormonal contraception

21.For planned hormonal contraception use:


A. Pure estrogens
B. Pure gestagens
C. A-gn realizing hormones
D. Prostoglandins
E. Selective inhibitors of progesterone receptors

27. Intrauterine system "Mirena":


A. Contains the progestogen levonorgestrel
B. Introduced for 8 years
C. Has a therapeutic effect in submucous uterine myoma
D. Does not have a contraceptive effect
E. Has a contraceptive effect by suppressing ovulation

28. The most common cause of female infertility is :


A. Psychosexual disorders
B. Tubal-peritoneal factor
C. Endocrine factor
D. Uterine factor
E. Immunological factor

29. During what time does not pregnancy occur in the presence of regular sexual
life, without the use of contraceptives, to consider the marriage infertile:
A. 6 months
B. 1 year
C. 2 years
D. 3 years
E. 5 years

30. Name the composition of injectable contraceptives:


A. Long-acting Progestogens
B. Conjugated estrogens
C. Small doses of a progestogen
D. Antiandrogens
E. Mixed doses of hormones

31. Specify the day of menstrual cycle to start estrogen-progestin preparations for
contraception:
A. 1
B. 5
C. 7
D. 14
E. 28

32. Specify the most serious complication when taking combined oral
contraceptives:
A. Thromboembolic complications
B. Cardiovascular diseases
C. Impaired carbohydrate metabolism
D. Violation of fat metabolism
E. Violation of vitamin metabolism

33. Infection with what microorganisms that cause colpitis requires treatment of
both partners:
A. Trichomonas
B. Candida
C. Streptococci
D. Staphylococci
E. E.Coli

34. In girls at an early age (from 2 to 8 years), the following are more common:
A. Ovarian tumors
B. Dysfunctional bleeding
C. Congenital anomalies of the genital organs
D. Vulvovaginitis
E. Salpingo-oophoritis

35. Classification of pelvioperitonitis by stages, choose the correct answer:


A. Primary, secondary
B. Reactive, terminal, toxic
C. Adhesive, exudative
D. Serous, purulent, fibrinous
E. Adhesive, fibrinous

36. Which of the following inflammatory diseases in the acute stage is


accompanied by symptoms of bryuschina irritation:
A. Endometritis
B. Colpitis
C. Vulvovaginitis
D. Cervicitis
E. Bartholinitis

37. The main clinical symptom of bacterial vaginosis is:


A. Itching of the external genitals and perineum
B. Dyspareunia
C. Profuse leucorrhoea with an unpleasant odor
D. Dysuria
E. Bubble rashes in the genital area

38. Polymenorrhea is:


A. Lean menstruation
B. Short menstruation (1-2 days)
C. Short menstrual cycle
D. Heavy menstruation
E. Painful menstruation

39. Amenorrhea is the absence of menstruation during:


A. 4 months
B. 5 months
C. 6 months
D. 12 months
E. 2 months

40. During the parametrite, a stage is distinguished:


A. Desquamation
B. Infiltration
C. Proliferation
D. Tissue decomposition
E. Edematous

41. The complication that most often arises in the management of the intrauterine
device is:
A. Isthmic-cervical insufficiency
B. Ectopic pregnancy
C. Habitual miscarriage
D. Acute infection
E. Pelvic vein thrombosis

42. The symptom most characteristic of vulvovaginitis:


A. Sharp pains
B. Burning, itching
C. High temperature
D. Ulceration
E. Spotting

43. The most important risk factor for endometritis after childbirth is:
A. Frequent sexual activity
B. Cesarean section
C. Childbirth through the birth canal
D. Previous urinary tract infection
E. Corresponding upper respiratory tract infection

44. What are the diagnostic tests that indicate the presence of bacterial vaginosis:
A. Detection of key cells
B. Lowering the ph of vaginal secretions
C. Negative amino test of whites
D. Detection of abnormal cells
E. Increase in basal temperature

45. The anatomical features of the uterus in a newborn girl include:


A. The body of the uterus is small, the cervix is almost not expressed
B. The length of the cervix 3 times more the length of the uterus
C. The uterus has a bicorn shape
D. The body of the uterus is almost 2 times larger than the cervix
E. Uterus curved posteriorly

46. Small doses of estrogen:


A. Stimulate FSH production
B. Suppress FSH production
C. Strengthen LH production
D. Suppress LH production
E. Stimulates TTG production

47. The vaginal part of the cervix in a woman of reproductive age is normally
covered:
A. Cylindrical epithelium
B. Stratified squamous epithelium
C. Glandular epithelium
D. Stratified squamous non-keratinizing epithelium
E. Cubic epithelium

48. The main estrogen hormone in the body of a woman in the postmenopausal
period is:
A. Estradiol
B. Estron
C. Estriol
D. Estradiol dipropionate
E. Estrogen

49. According to the tests of functional diagnostics (TFD), it is impossible to


determine:
A. Biphasic menstruation
B. The level of estrogenic saturation of the body
C. Ovulation
D. The usefulness of the luteal phase of the cycle
E. Hormone-producing ovarian tumor

50. Cytological cervical screening is performed at the age of:


A. 30 to 69 years old
B. 40 to 69 years old
C. 31 to 69 years old
D. 49 to 69 years old
E. 21 to 69 years old

51. Which of the following examination methods most reliably confirm the
diagnosis of inflammation of the appendages?
A. Quantification of white blood cells;
B. Gram stain of cervical mucus smear;
C. Culdocentesis;
D. Laparoscopy;
E. Ultrasound of the pelvic organs
52. Patient V., 22 years old, consulted a antenatal clinic with complaints of
discomfort in the vagina, burning sensation, itching, leucorrhoea. Considers
himself ill for 5 days. Examination reveald: the mucous membrane of the
vagina and cervix is sharply hyperemic, edematous. Whitish deposits are easily
removed with a gauze ball, a cheesy discharge. Vaginal examination was
normal. Was diagnosed : Vulvovaginal candidiasis .Define the tactics of the
antenatal clinic doctor :
A. Antifungal therapy
B. Hormonal therapy
C. Radiation therapy
D .Surgical therapy
E.Physiotherapy

53. A 20-year-old female patient turned to clinic, complaining of abundant,


purulent discharge from the genital tract, itching, burning and pain in the
vagina. She fell ill acutely, 2 days ago, when the above complaints appeared.
Menstrual function is not impaired. By examination: the vaginal mucosa is
sharply hyperemic, covered with a purulent bloom, bleeds easily when touched.
The cervix is cylindrical in shape, the mucosa is not changed. What is the most
likely diagnosis:
A. Vulvovaginitis
B. Acute colpitis
C. Endometritis
D. Endocervicitis
E. Bartholinitis

54. 5-year-old woman, unmarried, has sex 2-3 times a month, and sometimes less
often. Partners are different. There were no pregnancies. What method of
contraception is appropriate to prescribe:
A. Intrauterine device
B. Condom
C. Combined oral contraceptives
D. Injectable contraceptives
E. Surgical sterilization

55. A young nulliparous woman, 20 years old, has an active sex life outside of
marriage, has several sexual partners. What methods of contraception can be
recommended to the patient:
A. Condom
B. Spermicides
C. Intrauterine device
D. Combined oral contraceptives
E. Surgical sterilization

56. A 25-year-old female patient consulted a gynecologist to receive


recommendations on contraception. FROM ANAMNESIS: menarche from 13
years old, irregular, painful, takes spasmaton to relieve pain. Married.In
obstetric anamnesis was 1birth 3 years ago. She does not plan pregnancy for the
next 2-3 years, she wants reliable contraception. Somatically healthy.
Gynecological examination and ultrasound revealed no pathology. What
method of contraception is recommended for this patient:
A. Surgical sterilization
B. Calendar rhythm method.
C. Intrauterine device with silver
D. Combined oral contraceptives
E. Postcoital contraception (Postinor, Dvella)

57. Patient I., 42 years old, consulted a gynecologist to receive recommendations


on contraception, suffers from hypertension for 5 years. Regular sex life in
marriage. History of 2 births, 2 medical abortions. General condition is
satisfactory, blood pressure is 135/90 mm Hg., height 164 cm, weight 90 kg ,
BMI =35. Recommended method of contraception:
A. Condom
B. Calendar rhythm method
C. Intrauterine device with silver
D. Surgical sterilization
E. Postcoital contraception (Postinor, Dvella)

58. For anovulatory menstrual cycle with short-term persistence of a mature


follicle is not typical:
A. Symptom of the pupil (+++)
B. Single-phase basal temperature
C. High estrogen levels
D. In the scraping of the endometrium in the second phase of the cycle
E. Prolonged spotting discharge

59. Features of the treatment of DUB in menopausal patients are:


A. Conducting a separate diagnostic curettage
B. Ovarian Suppression Therapy
C. Treatment of concomitant somatic pathology
D. Cyclic hormone therapy
E. Hormone treatment

60. The most common mechanism for the development of dysfunctional uterine
bleeding in the juvenile period is:
A. Increased FSH
B. Follicular persistence
C. Follicular atresia
D. Hyperprolactinemi
E. LH increase

61. The conditions for cyclic hormone therapy in patients with menstrual
irregularities are:
A. Preliminary consultation by a neurologist
B. Preliminary consultation of the therapist
C. The use of a minimum amount of estrogen
D. Use of gestagens, ultrasound examination of the uterus
E. Hormone testing

62. Women with AUB are at risk:


A. Miscarriage
B. On the development of placental insufficiency and anomalies of the birth force
C. On the development of genital tumors
D. On the development of breast tumors
E. Pregnancy

63. What is the mechanism of action of the lactational amenorrhea method (LAM):
A. Suppression of ovulation
B. Changes in the structure of the endometrium
C. Violation of implantation
D. Thickening of cervical mucus
E. Change in sperm movement

64. The high efficiency of the method of lactational amenorrhea (LAM), subject to
all the rules, is observed:
A. Within 3 months after childbirth
B. Within 6 months after childbirth
C. Within 9 months after childbirth
D. Within 12 months after childbirth
E. Within 15 months after giving birth

65. What is the mechanism of action of voluntary surgical sterilization:


A. Thickening of cervical mucus
B. Blockage of the fallopian tubes
C. Suppression of ovulation
D. Violation of implantation
E. Thickening of cervical mucus

66. Specify the mechanism of action of spermicides:


A. Reduces the life span of spermatozoa
B. Destroys the membranes of spermatozoa
C. Thickening of cervical mucus
D. Violation of egg implantation
E. Dilution of cervical mucus

67. Before giving a woman an injection of Depo-provera, the health worker must
make sure that she does not have:
A. Vaginal bleeding of unclear etiology
B. Cardiovascular disease
C. History of STIs
D. Thromboembolic disorders
E. High blood pressure

68. Depo-provera is not recommended:


A. Women of any age with more than 3 children
B. Women with migraines
C. Women with breast cancer at this time
D. Unborn women and adolescents
E. Women after abortion

69. Which of the following mechanisms best explains the contraceptive effect
of birth control pills that contain both synthetic estrogen and progestin?
A. Direct inhibition of oocyte maturation
B. Inhibition of ovulation
C. Production of uterine secretions that are toxic to the developing
embryo
D. Impairment of implantation
E. Impairment of sperm transport due to uterotubal obstructio

70. A woman has period every 33 days lasting 8 days with large clots being
passed. She could be classified as having
A. Metrorrhagia
B. Menometrrohagia
C. Dysmenorrhea
D. Menorrhagia
E. Polymenorrhea

71. Of the following, which woman is the best candidate for the female
condom?
A.16 year old high school student who has never had sex, but
wishes to start with her new 18-year-old boyfriend
B.29 year old secretary who has been married for 7 years, and is in
a monogamous relationship with her husband
C.40 year old obese smoker whose partner refuses to wear
condoms
D.34 year old woman who has three children and had a post­
partum tubal ligation

72. For conservative treatment of Abnormal uterine bleeding the 1st line drug is:
F. Oxytocin
G. Nsaids
H. Tranexamic acid
I. Aminocaproic acid
J. Sodium ethamsylate

73. What is the most common presenting complaint of a woman with vaginitis?

A. Dyspareunia
B. Vaginal discharges
C. Pelvic pain
D. Fever
E. Irregular menses
Test TOP. ANATOMY
1. Sperm production begins in the

a. seminiferous tubules

b. epididymis

c. vas deferens

d. ejaculatory duct

2. The structure between the uterus and the vagina is the

a. uterine tube

b. cervix

c. vulva

d. hymen

3. Out of the following, which hormone does not secret from corpus luteum?

a) Estrogen

b) Progesterone

c) Relaxin

d) Testosterone

4. Mark the INCORRECT statement about prostate gland?

a) Located inferior to the urinary bladder

b) Secretion is thin and milky colored

c) Secretion is acidic in nature

d) Function in increasing the mobility of the sperm

5. Which of the following is an energy source for the sperm?

a) Somatostatin

b) Prostaglandin

c) Proteins

d) Fructose
OBS
1. Which one of the following fetal heart rate (FHR) is considered normal:

A. 120-160 per minute

B. 110-150 per minute

C. 100-180 per minute

D. more than 200 per minute

E. 80-100 per minute

2. Pregnancy duration:

A. 36 weeks

B. 40 weeks

C. 42 weeks

D. 38 weeks

E. 37 weeks

3. The length of 1st labor stage (cervical dilatation) in primigravida:

A. 3-6 hours

B. 5-6 hours

C. 10-12 hours

D. 18-20 hours

E. 2-4 hours

4. The length of 1st labor stage (cervical dilatation) of multipara?

A. 3 - 4 hours

B. 5-6 hours

C. 10-12 hours

D. 12-14 hours

5. At what dilatation of cervix rupture of membrane occur normally?

A. 3 - 4 cm

B. 5 - 6 cm C. 7- 8 cm
D. 9-10 cm

E. 11-12 cm

6. What is determined by the 1st maneuvers of Leopold?

A. The position of the fetus

B. Height of the uterine fundus

C. Type of fetus

D. Presenting part

E. Attitude

7. What is determined by the 2nd maneuvers of Leopold?

A. The position of the fetus

B. Height of the uterine fundus

C. Type of fetus

D. Presenting part

E. Attitude

8. What is determined by the 3rd maneuvers of Leopold?

A. The position of the fetus

B. Height of the uterine fundus

C. Type of fetus

D. Presenting part

E. Attitude

9. What is determined by the 4rd maneuvers of Leopold?

A. The position of the fetus

B. Height of the uterine fundus

C. Type of fetus

D. Presenting part

E. Relation of presenting part to pelvic planes


10. Dilatation of cervix is full, amniotic membrane is intact. What phase of 1st

stage?

A. Latent phase

B. Active phase

C. Beginning of 1st stage

D. End of 1nd stage

E. Beginning of 2nd stage

11. What period is, if vaginal examination revealed the cervical dilatation of 5

cm?

A. 1st stage

B. Beginning of 2nd stage

C. End of 2nd stage

D. 3rd stage

E. Preliminary phase

12. Multipara has regular active contractions every 3-4 minutes for 35-40

seconds, which started 6 hours ago? What stage of labor is expected?

A. Latent phase

B. Beginning of labor

C. End of 1st stage

D. 2nd stage

E. 3 rd stage

13. The length of 1st labor stage (cervical dilatation) in primigravida:

A. 3-6 hours

B. 5-6 hours

C. 10-12 hours

D. 18-20 hours

E. 2-4 hours
14. The length of 1st labor stage (cervical dilatation) of multipara?

A. 3 - 4 hours

B. 5-6 hours

C. 10-12 hours

D. 12-14 hours

15. At what dilatation of cervix rupture of membrane occur normally?

A. 3 - 4 cm

B. 5 - 6 cm

C. 7- 8 cm

D. 9-10 cm

E. 11-12 cm

16. During pregnancy, which of the following is not true about maternal

cardiovascular physiology?

A. Heart rate decreases

B. Total peripheral resistance decreases

C. Arterial blood pressure decreases

D. Cardiac output increases

E. New murmurs are common

17. Why do pregnant women fill more easy breathing at the end of the

physiological pregnancy?

A. Absorption of amniotic fluid

B. Descending of the presenting part into the relaxed lower uterine segment

C. Contraction of the uterus

D. Reduction in fetal weight

E. Flexion of the fetal


18. What is determined by the second Leopold’s maneuver?

A. Height of the uterine fundus

B. Presenting part

C. Back and small parts of the fetus

D. Station of fetal head

E. Attitude

19. The smallest anteroposterior diameter of the pelvic inlet is called:

A. Interspinous diameter

B. True conjugate

C. Diagonal conjugate

D. Obstetric conjugate

E. Anatomic conjugate

20. The primigravida women fill the first quickening of fetus in:

A.12 weeks

B.16 weeks

C.18 weeks

D.20 weeks

E.22 weeks

21. The multipara women fill the first quickening of fetus in:

A. 12 weeks

B. 16 weeks

C. 18 weeks

D. 20 weeks

E. 22 weeks

22. The fundal height in 12 weeks of pregnancy is placed on the level of:

A. Umbilicus

B. Pubis symphysis
C. 2 cm above the pelvic inlet

D. Xiphisternum

E. 2 cm above umbilicus

23. Immunological tests depend upon secretion of:

A. Chorionic gonadotropins by trofoblast

B. Progesterone

C. Estrogen

D. Prolactin

E. Lactogen

24. An 22-year-old maltipara with a last menstrual period of May 7 presents for

her first obstetric visit at 10 weeks. What is this patient’s estimated date of

confinement?

A. November 10

B. February 10

C. February 14

D. November 14

E. November 7

25. Which one of the following statements regarding physiological amount of

blood loss in 3 phases of labor is true?

A.100-550 ml

B. 250-500 ml

C. 550-650 ml

D. 500-700 ml

E.10% of placental weight


26. How change the concentrations of hormones during pregnancy?

A. Increase level of progesterone and estrogen

B. Decrease level of progesterone and estrogen

C. Increase level of androgen

D. Increase level of estrogen

E. Decrease level of estrogen

27. All of the following changes are seen in pregnancy except:

A. Increased stroke volume

B. Increased cardiac output

C. Increased intravascular volume

D. Increased peripheral vascular resistance

E. Increased pulse

28. In pregnancy all occurs except:

A. Increased cardiac output

B. Decreased blood volume

C. Increased MCV

D. Increased glomerular flow

E. Increased pulse

29. Which of the following is correct for urinary system changes in pregnancy:

A. Renal blood flow decreased

B. GFR increased

C. Decreased creatinine clearance

D. Kidneys shrink by 1cm

E. Polyuria

30. Cardinal movements of labor are:

A. Engagement → descent → flexion → internal rotation → extension →

restitution → external rotation → expulsion


B. Engagement → flexion → descent → internal rotation → extension →

expulsion

C. Engagement → flexion → descent → external rotation → expulsion

D. Engagement → extension → internal rotation → external rotation → expulsion

31. How does the blood content change during pregnancy?

A. Volume erythrocytes decreases

B.Hemoglobin’s and plasma increases

C.Volume erythrocytes, hemoglobin’s and plasma decreases

D.Volume erythrocytes, hemoglobin’s and plasma increases

E. Hemoglobin, plasma decreases

32. How does the blood and volume of the plasma change during pregnancy?

A. Decreases

B. Increases

C. Volume erythrocytes, hemoglobin’s decreases and plasma increases

D. Volume erythrocytes, hemoglobin’s increases and plasma decreases

E. No changes

33. What is the true sign of pregnancy?

A. Nausea

B. Vomiting

C. Enlargement of the uterus

D. Auscultation of the fetal heartbeat

E. Pigmentation changes

34. Normal diameters of big pelvis?

A. 22-25-28-15 cm

B. 24-26-30-20 cm

C. 26-28-30-20 cm

D. 27-27-30-17 cm

E. 25-27-29-18 cm
35. The smallest diameter of the true pelvis is:

A. Interspinous Diameter

B. Diagonal conjugate

C. True conjugate

D. Intertuberous diameter

E.Lateral conjugate

36. The shortest diameter of fetal head is:

A. Biparietal diameter

B. Suboccipitofrontal diameter

C. Occipitofrontal diameter

D. Bitemporal diameter

C. Verticomental

37. Shortest diameter of pelvis is:

A. Diagonal conjugate

B. Obstetric conjugate

C. True conjugate

D. Lateral conjugate

E. All are equal

38. Longest diameter of pelvic inlet is:

A. Transverse diameter

B. Diagonal conjugate

C. Obstetric conjugate

D. True conjugate

E. All are equal

39.What is common type of presentation?

A. Cephalic

B. Vertex
C. Face

D. Brow

E. Breech

40. What is the submentobregmatic diameter on the fetal head?

This distance extends from:

A. under surface of the occipital bоnе to the chin

B. under surface of the occipital bоnе to the angel of bregma

C. under surface of the occipital bоnе to the line of hair

D. junction of the neck and lower jaw to the center of bregma,

41. What is the suboccipitobregmatic diameter of the fetal head?

A. 10 cm, circumference 33 cm

B. 9.5 cm, circumference 32 cm

C. 12 cm, circumference 34 cm

D. 13.5 cm, circumference of 38 cm

42. Which is not included in active management of III stage of labor?

A. Uterotonic within 1 minute of delivery

B.Immediate clamping, cutting and ligation of cord

C. General massage of uterus

D. Controlled cord traction

43. Cervical dilatation per hour in primapara is:

A. 1.0 cms

B. 1.5 cms

C. 1.7 cms

D. 2 cms

E. 2.5 cms
44. The graph showing relationship between cervical dilatation and duration of

labor is:

A. Partogram

B. Cervicograph

C. Growth curve

D. Dilatation chart

E. Gravidogram

45.What is not characteristic of true labor pain:

A. Absence of ‘bag of waters’

B. Painful uterine contractions

C. Progressive effacement and dilatation of the cervix

D. Pain often felt in front of the abdomen or radiating

towards the thighs

E. Irregular

46. A patient comes to your office with LMP 4 weeks ago. She denies any

symptoms such as nausea, fatigue, urinary frequency, or breast tenderness.

She thinks that she may be pregnant.

Which of the following evaluation methods is most sensitive in diagnosing

pregnancy?

A. No evaluation to determine pregnancy is needed because the patient is

asymptomatic and therefore cannot be pregnant

B. Serum pregnancy test

C. Detection of fetal heart tones by Doppler equipment

D. Abdominal ultrasound

E. Bimanual exam to assess uterine size.


47. A healthy 22-year-old primapara presents for her first OB visit at 1 weeks

gestational age. She denies any significant medical history both personally and in

her family. All of the following tests should be ordered as part of the initial

prenatal care visit except

A. CBC

B. HIV

C. Hepatitis B surface antigen

D. Type and screen

E. One-hour glucose
GYNECOLOGY
1. FSH stimulates:

A. Ovarian follicle growth

B. Corticosteroid production

C. Production of TSH in the thyroid gland

D. Progesterone production

E. Androgen production

1. Polymenorrhea is:

A. Lean menstruation

B. Short menstruation (1-2 days)

C. Short menstrual cycle

D. Heavy menstruation

E. Painful menstruation

2. The treatment of patients with ovarian exhaustion syndrome should be

directed:

A. To stimulate ovarian function

B. On getting a menstrual-like reaction

C. To reduce the severity of vegetovascular disorders

D. On stimulation of ovulation

E. On getting cyclical menstruation

3. The effect of large doses of exogenous estrogen on the ovary:

A. Increased ovarian production of estrogen

B. Increased ovarian production of progesterone

C. Atrophy of the ovarian tissue

D. Reduces ovarian progesterone production

E. Reduces ovarian production of estrogen


4. In a negative test with gonadotropins in patients with amenorrhea, it is

shown:

A. Test with estrogens and gestagens in a cyclic mode

B. Laparoscopy and ovarian biopsy

C. Laparoscopy and gonad removal

D. Progesterone test

E. Functional diagnostic tests

5. In the ovary, androgens serve as a substrate for the formation of:

A. Progesterone

B. Estrogen

C. Progesterone and estrogen

D. Gestagenov

E. Testosterone

6. The following applies to hypergonadotropic amenorrhea:

A. Ovarian Depletion Syndrome

B. Resistant Ovarian Syndrome

C. Ovarian Dysgenesis Syndrome

D. Testicular feminization syndrome

E. Sheehan Syndrome

7. Women with DUB are at risk:

A. Miscarriage

B. On the development of placental insufficiency and anomalies of the birth

force

C. On the development of genital tumors

D. On the development of breast tumors

E. Pregnancy
8. In the diagnosis of amenorrhea associated with acromegaly and gigantism, a

change is significant:

A. the size of the Turkish saddle on the radiograph of the skull

B. visual fields

C. FSH level

D. Excretion of 17-cop

E. Ultrasound diagnosis of the pelvic organs

9. Gestagens are secreted:

A. luteal cells of the corpus luteum

B. cells of the granular layer of the follicle

C. cells of the outer connective tissue of the follicle

D. Ovary cells

E. theca cells

10. Amenorrhea is considered pathological when menstruation is absent

A. before puberty

B. during puberty

C. during lactation

D. in postmenopausal women

E. during pregnancy

11. False amenorrhea can be caused by:

A. atresia of the cervical canal

B. Aplasia of the uterus

C. Dysgenesis of gonads

D. Ovarian exhaustion syndrome

E. Ovarian tumor

12. Ovarian exhaustion syndrome needs to be differentiated

A. with resistant ovary syndrome

B. with gonad dysgenesis syndrome


C. with menopausal syndrome

D. with ovarian tumor

E. with premenstrual syndrome

13. Treatment for DUB does not include:

A. physiotherapy

B. vitamins

C. uterotonics

D. antibiotics

E. hormonal hemostasis

14. The scope of the examination for pathology of the cervix

A. colposcopy

B. hysteroscopy

C. Ultrasound

D. bacterioscopy

E. laparoscopy

15. In what cases, for diagnostic purposes, puncture of the abdominal cavity

through the posterior arch is indicated?

A. suspicion of an ectopic pregnancy;

B. suspected ovarian cancer;

C. dysfunctional uterine bleeding;

D. uterine fibroids;

E. suspected ovarian apoplexy;

16. Research methods of the anatomical and functional state of the vagina:

A. inspection with speculum;

B. combined vaginal rectal examination;

C. cytological examination of the contents of the uterine cavity;

D. puncture of the posterior fornix;

E. determination of the purity of the vaginal contents.


17. The main method for the diagnosis of dysplasia and pre-invasive cancer of

the vulva is:

A. Biopsy followed by histological examination

B. Vulvoscopy

C. Radioisotope study

D. Cytological examination of smear prints

E. Colposcopy

18. The most informative method for the diagnosis of genital prolapse is:

A. ultrasound;

B. gynecological examination;

C. sigmoidoscopy;

D. cystoscopy;

E. hysteroscopy.

19. Algodismenorea is:

A. Rare and scarce menstruation

B. painful menstruation

C. Reduced blood loss during menstruation

D. intermenstrual meager spotting

E. heavy periods

20. Menorrhagia is:

A. Acyclic uterine bleeding

B. cyclic uterine bleeding

C. Painful and heavy menstruation

D. pre- and postmenstrual bleeding

E. Change in the rhythm of menstruation


21. Dysfunctional uterine bleeding with follicular atresia occurs against

A. High estrogen saturation

B. low estrogen saturation

C. High gestagenic saturation

D. Hyperandrogenemia

E. Prolactin overproduction

22. Ovulatory acyclic bleeding characteristic

A. with short-term rhythmic persistence of the follicle

B. with prolonged persistence of the follicle

C. with follicular atresia

D. with persistence of the corpus luteum

E. in case of corpus luteum insufficiency

23. Anovulatory cyclic bleeding characteristic

A. with prolonged follicular persistence

B. with short-term persistence of follicles

C. with persistence of the corpus luteum

D. in case of corpus luteum insufficiency

E. with follicular atresia

24. A test with clomiphene is performed if:

A. Uterine fibroids

B. Anovulation

C. Adenomyosis

D. Endometritis

E. Hyperprolactinemia
25. The diagnosis of impaired tubal pregnancy is an indication

A. for emergency laparotomy

B. for laparoscopy

C. for ultrasound examination

D. for special studies

E. for puncture of the back arch

26. Currently, age is considered normal for menopause.

A. 45 years

B. 5O years

C. 4O years

D. 55 years

E. 43 years

27. The autonomic-nervous manifestations of menopausal syndrome include

A. Irritability

B. Sweating

C. Sleep Disorders

D. Decrease in memory

E. Decreased libido

28. Metrorrhagia is

A. Change in the rhythm of menstruation

B. increased blood loss during menstruation

C. increase in the duration of menstruation

D. Acyclic uterine bleeding

E. Reducing blood loss during menstruation


29. The most informative in the differential diagnosis between uterine myoma

and ovarian tumor:

A. two-handed vaginal examination

B. hysteroscopy

C. ultrasound;

D. laparoscopy;

E. Probing of the uterine cavity.

30. Hysteroscopy allows to diagnose:

a) subserous uterine fibroids;

b) submucous uterine fibroids;

c) tubal-peritoneal infertility;

d) Stein-Levinthal syndrome;

e) tubal pregnancy.

31. The screening method to detect cervical pathology in modern conditions is:

A. visual inspection;

B. smear cytology;

C. vaginal smear on the flora

D. radionuclide method

E. PCR for HPV

32. The presence of ovulation can be judged by the results of all the studies listed

below:

A. Definitions of hCG

B. Ultrasound monitoring of the development of a dominant follicle

C. Histological examination of endometrial scraping

D. Determination of the concentration of sex steroid hormones in the blood on

days 12-14 of the menstrual cycle

E. Follicle persistence
33. Screening - a method for detecting cervical pathology is:

A. Visual inspection

B. Colposcopy

C. Radionuclide method

D. Cytological examination of smears

E. Laparoscopy

34. In girls at an early age (from 2 to 8 years), the following are more common:

A. ovarian tumors;

B. dysfunctional bleeding;

C. congenital anomalies of the genital organs;

D. vulvovaginitis;

E. salpingo-oophoritis.

35. Classification of pelvioperitonitis by stages, choose the correct answer:

A. primary, secondary

B. reactive, terminal, toxic

C. adhesive, exudative

D. serous, purulent, fibrinous;

E. adhesive, fibrinous

36. Which of the following inflammatory diseases in the acute stage is

accompanied by symptoms of bryuschina irritation?

A. Endometritis.

V. Kolpit

C. vulvovaginitis

D. Cervicitis

E. Bartholinitis
37. Which of the following inflammatory diseases is an indication for

laparotomy in case of suppuration?

A. Parameter

V. Kolpit

C. vulvovaginitis

D. Cervicitis

E. Bartholinitis

38. What are the diagnostic tests that indicate the presence of bacterial vaginosis:

A. detection of key cells;

B. lowering the pH of vaginal secretions;

C. negative amino test of whites;

D. detection of abnormal cells

E. increase in basal temperature.

39. During the parametrite, a stage is distinguished:

A. desquamation

B. infiltration

C. proliferation

D. tissue decomposition

E. edematous

40. The complication that most often arises in the management of the IUD is:

A. isthmic-cervical insufficiency;

B. ectopic pregnancy;

C. habitual miscarriage;

D. acute infection;

E. pelvic vein thrombosis.


41. The symptom most characteristic of vulvovaginitis:

A. sharp pains;

B. burning, itching

C. high temperature;

D. ulceration;

E. spotting.

42. The most important risk factor for endometritis after childbirth is:

A. frequent sexual activity;

B. cesarean section;

C. childbirth through the birth canal;

D. previous urinary tract infection;

E. corresponding upper respiratory tract infection

43. The main clinical symptom of tuberculous lesions of the uterine appendages:

A. chronic pelvic pain;

B. amenorrhea;

C. menometrorrhagia;

D. primary infertility;

E. secondary infertility.

44. Which of the following symptoms is the main clinical one for bacterial

vaginosis?

A. itching of the external genitalia and perineum;

B. dyspareunia;

C. profuse leucorrhoea with an unpleasant odor;

D. dysuria;

E. pelvic pain.
45. Specify the lack of an intrauterine device:

A. increases the risk of inflammatory diseases

B. contraindications for lactation

C. short term

D. a large number of side effect

E. effect on lactation

46. Specify the time during which the sperm retains the ability to fertilize during:

A. 6 hours

B. 12 hours

C. 24 hours

D. 48 hours

E. 72 hours

47. The most informative way to assess the functional state of the ovaries is:

A. basal temperature measurement

B. symptom of cervical mucus tension

C. suction curettage

D. laparoscopy

F. Hysteroscopy

48. Algodismenorea is:

A. Rare and scarce menstruation

B. painful menstruation

C. Reduced blood loss during menstruation

D. intermenstrual meager spotting

E. heavy periods
49. Basal temperature is measured:

A. In the morning

B. In the evening;

C. 2 times a day;

D. After 3 hours.

E. After 5 hours

50. Parametritis is:

A. inflammation of the ovary;

B. inflammation of the cecum;

C. inflammation of the fallopian tube;

D. inflammation of the peritoneal tissue;

E. inflammation of the omentum.


Training test questionnaire for Reproductive system

1. Which imaging modality is used as breast diagnostic screening?

A. Mammography
B. Chest X-ray
C. Ultrasound
D. CT-scan
E. MRI

2. Identify the view on the mammogram

A. Mediolateral oblique (MLO)


B. Bilateral craniocaudal (CC)
C. Lateromedial (LM)
D. Lateromedial oblique (LMO)
E. Mediolateral (ML)

3. Simple cyst was detected during ultrasound examination. Which BI-RADS classification it
belongs to?

A. BI-RADS 0
B. BI-RADS 1
C. BI-RADS 2
D. BI-RADS 3
E. BI-RADS 4

4. Which imaging modality is the best for evaluation of breast cyst?

A. Mammography
B. Chest X-ray
C. MRI
D. CT-scan
E. Ultrasonography
5. Which imaging examination can be performed to study fallopian tubes with iodine containing
contrasting agent?

A. Hysterosalpingography
B. Hysteroscopy
C. Intravenous pyelography
D. Ultrasonography
E. Cystoscopy

6. A 48 year- old female came to mammologist, she complained to breast lump. Clinical
examination revealed palpable mass of breast. What is first choice imaging modality in this
case?

A. Mammography
B. Chest X-ray
C. MRI
D. CT-scan
E. Ultrasonography

7. A 50-year-old women admitted to oncology center. She has complains of pain and last week
abnormal bleeding. Hb level 70 . A 2 year ago she had diagnosed with cervical polyp (15
mm) Which imaging study is most suitable for clarifying the diagnosis?

A. Ultrasound
B. CT-scan
C. Hysteroscopy
D. MRI
E. X-ray

8. A 32-year female was consulted to the doctor with complain to heavy and painful periods.
Doctor suggested pelvic endometriosis. Which imaging modality is the first choice in this
case?

A. Ultrasound
B. CT-scan
C. Hysteroscopy
D. MRI
E. X-ray
PATHOPHYSIOLOGY
Training Tests
1) The hypothalamus produces the following hormones:
A) Gonadotropins
b) Estrogen
b) Gestagens
d) Leasing – factors

2) FSH stimulate:
A) Growth of follicles in the ovaries
b) Products of corticosteroid
(b) TTG products in the thyroid gland
D) All of the above

3) Progestin:
A) Reduce blood cholesterol
b) Determine the content of primary and secondary sexual characteristics
b) All of the above
D) None of the above

4) The group of risk of inflammatory after birth diseases include women:


A) With a burdened obstetric and gynecological history
b) With chronic inflammatory process
c) With Extra Energy inflammatory diseases (tonsillitis, pyelonephritis)
D) All of the above

5) in inflammatory diseases of female genital organs is noted:


(a) Prevalence of microbial associations
b) Increase the number of anaerobic and virus
(b) increased permeability and exhaustion
d) All answers correct

6) The causes of infertility of women in marriage are:


(a) Inflammatory diseases of the genital organs
b) Infantialism and hypoplasia of genital organs
(b) General debilitating diseases and intoxication
d) All answers correct

7) Features of normal menstrual cycle are:


A) Ovulation
b) Formation of yellow body in the ovary
b) Prevalence of progesterone in the second phase of the cycle
D) All of the above

8) Estrogen:
A) Promote peristalsis of the uterus and tubes
b) Strengthen the processes of ossification
b) All of the above
D) None of the above

9) Androgens are formed:


A) In the ovaries (interstitial cells, Strome, inner Take)
b) In the net zone of the adrenal cortex
b) All of the above
D) None of the above
10) In its action on the body of estrogen:
A) Block receptors to oxytocin
b) Cease (weaken) proliferative processes in the endometrium
b) Cause secretory transformations in the endometrium
D) All of the above
e) None of the above

11) Increase of rectal temperature in the second phase of menstrual menstrual


cycle
due to:
A) The action of the ovarian progesterone
b) The action of progesterone, which reduces the heat return
(b) Intensification of biochemical processes in the uterus
D) All of the above
e) None of the above

12) The full value of the luteal phase of the menstrual cycle is evidenced by:
A) Increase in basal temperature in the first phase of the cycle
b) Proliferative processes in the endometrium in the second phase of the cycle
c) All of the above
D) None of the above

13) Amenorrhea is the absence of menstruation during:


A) 4 months
b) 5 months
c) 6 months
d) All of the above
e) None of the above
14) In the interaction of the hypothalamus, pituitary gland and ovaries in the
regulation of the menstrual cycle:
A) One link (overlying) stimulates the function of another link
b) The underlying link brakes or regulates the function of the overlying
b) The function of the links is synchronous
D) The shutdown of one link violates the whole system
e) All of the above

15) After ovulation, the egg retains the ability to fertilize during:
A) 6 hours:
b) 12-24 hours;
in) 3-5 days;
d). 10 days

16) During puberty, the following major changes occur in the body:
(a) Suppression of the Gonadotropic function of the pituitary gland;
b) Activation of hormonal function of ovaries;
c) The rhythm of the FSH allocation is not established;
D) Regular &quot;peaks&quot; of the LG excretion are established;
e) None of the above.

17) Body mass deficiency is usually one of the reasons:


a) Delay in the onset of the first menstruation;
b) Long-term formation of menstrual function;
b) The development or aggravation of menstrual dysfunction;
d) All listed;
e) None of the above

18) Mechanism of action of hormones on a cage is caused by presence:


a) Prostaglandins;
b) Receptors;
b) Specific enzymes;
d) All listed;
e) None of the above.

19) Small doses of estrogen:


a) Stimulate FSH products;
b)Ssuppress FSH products;
b) Strengthen the production of LG;
D) Suppress the production of LTG.

20) Desquamation of the functional layer of endometrium occurs due to:


a) Pathway; &quot;Peak&quot; reset luther
b) Reducing the level of estrogen and progesterone in the blood;
b) Reducing the level of prolactin in the blood;
d) Increase the level of estradiol in the blood;
e) &quot;Peak&quot; release of Follitropin.
21) Physiological Amenorrhea is the absence of menstruation:
a) Girls under 10-12 years of age;
b) During pregnancy;
b) During lactation;
d) In old age;
e) All of the above are true.
22) The reason of false amenorrhea in girls of Puberty Nogo age can be all
listed,
except:
a) Hymen atresia;
b) Atrezia Vagina;
b) Uterine hyperplasia;
d) Atrezia Cervical canal.

23) False amenorrhea may be caused by:


a) Cervical canal atresia;
b) Aplasia of the body of the uterus;
b) The dysgenesis of the Gonnad;
d) All the diseases listed above.

24) Lack of sexual development is due to:


a) Transferred infectious diseases in early childhood;
b) Undergone inflammatory processes of pelvic organs of viral etiology;
b) Chromosomal anomalies;
d) All listed;
e) None of the above.

25) Delay of sexual development is absence:


a)Secondary sexual characteristics by 14 years and menstruation by 16 years;
b) Secondary sexual characteristics by 12 years, and menstruation by 16 years;
b) Secondary sexual characteristics by 14 years, and menstruation by 18 years;
d) Secondary sexual characteristics to 12 years, and menstruation to 18 years.
Tests for the propedeutics of childhood diseases.
1. Delayed sexual development in girls consider the absenceof the secondary sexual
characteristics at the age?
A. 10-13
B. 11-14
C. 12-16
D. 13-17
E. 14-18
2. What age corresponds to the degree of puberty (years) Ма3P2 Ax2Мео?
A. 16-17
B. 14-15
C. 12-13
D. 10-11
E. 8-9
3. One of the main reasons for delayed sexual development, it is believed
A. Birth injury
B. Intrauterine infection
C. Chromosomal abnormality
D. Infectious diseases
E. Stressful situation
4. The girl has 3 days of life, there is an engorgement of the mammary glands, bloody
discharge from the genital tract, hyperemia of the skin. Determine what condition you are
talking about?
A. sclerema
B. jaundice of newborn
C. sexual crisis
D. hermaphroditism
E. hypothyroidism

5. When examined by a pediatrician, a 2-month old boy, there is a change in the urine
stream, during urination, in the form of leakage into the perineum. Objectively: in the area of
the penis there is a splitting of the lower wall of the urethra. What pathology of the genital
organs is mentioned in the task?
A. epispadias
B. hypospadias
C. cryptorchidism
D. anarchism
E. micropenis
6.A boy of 8 years old is being examined by a pediatrician. Objectively: a change in the
timbre of the voice, an increase in the width of the shoulder girdle, hair growth in the armpits,
pubic region, enlargement of the penis and testicles. What state does the objective status
correspond to?
A. The child is healty
B. hypopituitarism
C. False male hepmaphroditism
D. Premature sexual development
E. Delayed sexual development
7. On a routine examination, the girl 14 years old. Objectively: the mammary glands are
round in shape, the nipples rise above the areola. Pubic hair in a triangle, long, curly, thick. In
the hair is thick, curly over the entire surface. Menses are regular. Estimate the sexual
development of a girl according to Tanner?
A. Мао РоАхоМео
B. Ма2Р1 АхоМео
C. Ма3 Р3 Ах1Ме1
D. Ма3 Р2Ах2 Мео
E. Ма3 Р3 Ах3 Ме3
8.A 2-year-old girl is being examined by a pediatrician Objectively: physical development
corresponds to age, there is an increase in mammary glands. What changes do you see in the
patient?
A. Isolated thelarche
B. Isolated adrenarche
C. Sexual crisis
D. Premature pubarche
E. Premature sexual development

9.At the reception at FMC №3, a mother and her 8-year-old daughter addressed.Objectively:
the mammary glands are significantly enlarged, hair growth in the armpits, pubic region, the
appearance of the first menarche. What state is described in the task?
A. sexual crisis
B. premature sexual development
C. delayed sexual development
D. hypothyroidism
E. The child is healty
Urology Test for students: REDRODUCTIVE SYSTEM

1). Marriage should be considered infertile if pregnancy is not comes?


1. within 6 months

2. within 1 year

3. within 2 years

4. within 3 years

5. More than 3 years

2). The lower limit of normal sperm count in 1 ml should be (according


to WHO criteria):

1. 20 million

2. 40 million

3. 60 million

4. 80 million

5. 100 million

3). Azoospermia - is

1. The absence of the cells of spermatogenesis in the ejaculate and


spermatozoas

2. The presence of single cells of spermatogenesis in the ejaculate

3. The presence of only dead sperm in the ejaculate

4. The lack of allocation of ejaculate

5. the absence of ejaculate

4). Aspermia - is

1. The absence of sperm in the ejaculate

2. The absence of ejaculate

3. The lack of allocation of ejaculate

4. The small amount of sperm in the ejaculation

5. The absence of motile sperm in the ejaculate


5). Male sex cells (sperm) are formed in:

1. Ovaries

2. Gonad - testis

3. Epididymis

4. Prostate

5. Seminal vesicles

6). Male sex hormones are produced in:

1. Ovarian Theca cells

2. Prostate

3. Testis Leydig cells

4. Seminal vesicles

5. Sertoli cells

7). Reproductive function is:

1. The regulation of sexual activity

2. The fusion of two cells

3. Regulation of oogenesis and spermatogenesis

4. Reproductive function

5. Sexual arousal and attraction

8). Functions of male sex hormones:

1. The regulation of oogenesis

2. The development of secondary sexual characteristics

3. Ensuring the development of mammary glands

4. Ensure the development of the genital organ


5. Regulation of water-salt metabolism

9). What is oligozoospermia?

1. The absence of ejaculate

2. The absence of sperm in the ejaculate

3. The absence of mature sperm in the ejaculate

4. The small amount of sperm in the ejaculate

5. The absence of sperm in the ejaculate

10). How much the normal amount of single ejaculate in an adult?

1. 0.5-1.5 ml

2. depends on body weight men

3. 2.0-5.0 ml

4. 6-7 ml

5. 8-10 ml

11). What is aspermia?

1. The absence of ejaculate

2. The absence of sperm in the ejaculate

3. The absence of motile sperm in the ejaculate 4. The absence of mature


sperm in the ejaculate 5. The small number of sperm in the ejaculate

12). What is hemospermia?

1. The presence of blood in urine

2. The total hematuria

3. The discharge of blood after urination

4. The presence of blood in the ejaculate


5. The discharge of blood from the urethra at the beginning
of urination

13). What is necrospermia?

1. The absence of ejaculate

2. The absence of sperm in the ejaculate

3. The absence of mature

sperm in the ejaculate

4. The small amount of

sperm in the ejaculate

5. The absence of motile sperm in the ejaculate

14). What is prostatorrhea?

1. The allocation of prostate fluid during erection

2. The allocation of prostate fluid in the end of the act of

defecation or urination 3. Premature ejaculation

4. The allocation of semen without erection

5. Prostate secretions during intercourse.

15). Oligozoospermia may be the result of: 1. After


severe infections

2. Diabetes

3. Nephroptosis

4. Urolithiasis

5. Genitourinary fistula
16). Where occurs maturation of majority of the
sperm?

1. in the testis

2. in the epididymis

3. in the vas deferens


4. in the seminal vesicles
5. throughout ejaculatory tract

17). Insufficient numb


er of sperm with normal morphology is
1. Asthenozoospermia

2.Oligozoospermia

3. Necrospermia

4. Hemospermia

5. Theratozoospermia

8). Causes of male infertility

1. Pyelonephritis

2. Urolithiasis

3. Acute respiratory viral

infections

4. Varicocele

5. Nephroptosis

20). Testicular inflammation is

1. Prostatitis

2. Epididymitis

3. Orchitis

4. Cystitis

5. Vesiculitis
Reproduc.sys. (maam discussion)
1. Duration of the menstrual cycle:
A. 21-35days
B. 28-29days
C. 25-37days
D. 24-32days
E. 35-39days
2. Phases of the ovarian cycle:
A. Follicular and luteal
B. Follicular and ovulation
C. Ovulatory and anovulatory
D. Anovulatory and luteal
E. Desquamation and ovulatioon

3. Preovulatory changes in the hormonal


background are characterized by an
increase in the level
A. LH and decreased FSH
B. FSH and decreased LH
C. FSH and LH
D. Prolactin
E. FSH, LH and prolactin
4. The mechanism of action of hormones on
the cell is due to the presence of.
A. Prostaglandins
B. Receptors
C. Specific Enzymes
D. Isoenzymes
E. Thromboxant
5. According to the WHO classification (1980),
menopause is called
A. Period of stable menstrual function
B. Last menstruation
C. The length of time after the last
menstruation
D. Time span after reproductive age
E. The onset of the first menstruation

6. According to the WHO classification (1980),


postmenopause is called
A. Last menstruation
B. The period from the onset of menstruation
to the last menstruation
C. Period of stable menstrual function
D. Period of first menstruation
E. The length of time after the last
menstruation

7.Research methods of the anatomical and


functional state of the vagina:
A. Inspection with speculum;
B. Combined vaginal rectal examination
C. Cytological examination of the contents of
the uterine cavity;
D. Puncture of the posterior fornix;
E. Determination of the purity of the vaginal
contents.
8. Which test of functional diagnostics
indicates the presence of a biphasic
menstrual cycle?
A. Pupil symptom
B. Karyopicnosis Index
C. Basal thermometry
D. Symptom of "fern"
E. Symptom of crystallization

9. The scope of the examination for pathology


of the cervix
A. Colposcopy
B. Hysteroscopy
C. Ultrasound
D. Bacterioscopy
E. LaparOScopy

10. Algodismenorea is:


A. Rare and scarce menstruation
B. Painful menstruation
C. Reduced blood loss during menstruation
D. Intemenstrual meager spotting
E. Heavy periods

11. Menorhagia is:


A. Acyclic uterine bleeding
B. Cyclic uterine bleeding
C. Painful and heavy menstruation
D. Pre- and postmenstrual bleeding
E. Change in the rhythm ofmenstruation
12. Metrorrhagia is
A. Change in the rhythm of menstruation
B. Increased blood loss during menstruation
C. Increase in the duration of menstruation
D. Acyclic uterine bleeding
E. Reducing blood loss during menstruation

13. For anovulatory menstrual cycle with


follicular atresia is characteristic
A. Amenorrhea
B. Algomenorrhea
C. Oligoamenorrhea
D. Polymenomhea
E. Hypomenorrhea

14. Polymenorrhea is
A. Lean menstruation
B. Short menstruation (1-2 days)
C. Short menstrual cycle
D. Heavy menstruation
E. Painful menstruation

15. Amenorrhea is the absence of


menstruation during:
A. 4 months
B.5 months
C. 6 months
D. 12 months
E. 2 months

16. For anovulatory menstrual cycle with


follicular atresia is characteristic:
A. Amenorrhea
B. Algomenomhea
C. Oligoamenorrhea
D. Promenomenorrhea
E. Hypomenorrheea

17. In the anovulatory menstrual cycle, basal


temperature is characterized by:
A. Rise In Temperature After Ovulation
B. Lack of Temperature Rise
C. Rise In Temperature Before Ovulation
D. Rise In Temperature Before Menstruation
E. Rise In Temperature Immediately After
Menstruation

18. Anovulatory menstrual cycle is


characterized by:
A. Cyclic changes in the body
B. Long-lasting follicle persistence
C. Prevalence of gestagens in phase 2
D. Prevalence of estrogen in phase 1
E. Acyclic change in the body
19. To induce ovulation, use:
A. Progestogens
B. Antiestrogens
C. Natural Estrogens
D. Prostaglandins
E. Antagonists OfProstaglandins

20. The most effective method of preventing


unwanted pregnancy is:
A. Intrauterine device
B. Britm method
C. Spermicides
D. Surgical sterilization
E. Hormonal contraception

21.For planned hormonal contraception use:


A. Pure estrogens
B. Pure gestagens
C. A-gn realizing hormones
D. Prostoglandins
E. Selective inhibitors ofprogesterone
receptors

22. Intrauterine system "Mirena":


A. Contains the progestogen levonorgestrel
B. Introduced for 8 years
C. Has a therapeutic effect in submucous
uterine myoma
D. Does not have a contraceptive effect
E. Has a contraceptive effect by suppressing
ovulation

23. The most common cause of female


infertility is:
A. Psychosexual disorders
B. Tubal-peritoneal factor
C. Endocrine factor
D. Uterine factor
E. Immunological factor

24. During what time does not pregnancy


occur in the presence of regular sexual life,
without the use of contraceptives, to consider
the marriage infertile:
A. 6 months
B. 1 year
C. 2 years
D. 3 years
E. 5 years
25. Name the composition of injectable
contraceptives:
A. Long-acting Progestogens
B. Conjugated estrogens
C. Small doses of a progestogen
D. Antiandrogens
E. Mixed doses of hormones

26. Specify the day of menstrual cycle to start


estrogen-progestin preparations for
contraception:
A. 1
B. 5
C.7
D. 14
E. 28

27. Specify the most serious complication


when taking combined oral
contraceptives:
A. Thromboembolic complications
B. Cardiovascular diseases
C. Impaired carbohydrate metabolism
D. Violation of fat metabolism
E. Violation of vitamin metabolism
28. Infection with what microorganisms that
cause colpitis requires treatment of both
partners:
A. Trichomonas
B. Candida
C. Streptococci
D. Staphylococci
E. E.Coli
29. In girls at an early age (from 2 to 8 years),
the following are more common:
A. Ovarian tumors
B. Dysfunctional bleeding
C. Congenital anomalies of the genital organs
D. Vulvovaginitis
E. Salpingo-oophoritis
30. Classification of pelvioperitonitis by stages,
choose the correct answer
A. Primary, secondary
B. Reactive, terminal, toxic
C. Adhesive, exudative
D. Serous, purulent, fibrinous

31. Which of the following inflammatory


diseases in the acute stage is
accompanied by symptoms of bryuschina
irritation:
A. Endometritis
B. Colpitis
C. Vulvovaginitis
D. Cervicitis
E. Bartholinitis

32. The main clinical symptom of bacterial


vaginosis is:
A. Itching of the external genitals and
perineum
B.Dyspareunia
C. Profuse leucorrhoea with an unpleasant
odor
D. Dysuria
E. Bubble rashes in the genital area

33. During the parametrite, a stage is


distinguished:
A. Desquamation
B. Infiltration
C. Proliferation
D. Tissue decomposition
E. Edematous

34. The complication that most often arises in


the management of the intrauterine device is:
A. Isthmic-cervical insufficiency
B. Ectopic pregnancy
C. Habitual miscarriage
D. Acute infection
E. Pelvic vein thrombosis

35. The symptom most characteristic of


vulvovaginitis:
A. Sharp pains
B. Burning, itching
C. High temperature
D. Ulceration
E. Spotting
36. The most important risk factor for
endometritis after childbirth is:
A. Frequent sexual activity
B. Cesarean section
C. Childbirth through the birth canal

37. What are the diagnostic tests that indicate


the presence of bacterial vaginosis
A. Detection of key cells
B. Lowering the ph of vaginal secretions
C. Negative amino test of whites
D. Detection ofabnormal cells
E. Increase in basal temperature

38. The anatomical features of the uterus in a


newborn girl include:
A. The body of the uternus is small, the cervix
is almost not expressed
B. The length of the cervix 3 times more the
length of the uterus
C. The uterus has a bicom shape
D. The body of the uterus is almost 2 times
larger than the cervix
E. Uterus curved posteriorly

39. Small doses of estrogen:


A. Stimulate FSH production
B. Suppress FSH production
C. Strengthen LH production
D. Suppress LH production
E. Stimulates TTG production

40. The vaginal part of the cervix in a woman


of reproductive age is normally
covered:
A. Cylindrical epithelium
B. Stratified squamous epithelium
C. Glandular epithelium
D. Stratilied squamous non-keratinizing
epithelium
E. Cubic epithelium

41. The main estrogen hormone in the body


ofawoman in the postmenopausal
period is:
A. Estradiol
B. Estron
C. Estriol
D. Estradiol dipropionate
E. Estrogen

42. According to the tests of functional


diagnostics (TFD), it is impossible to
determine:
A. Biphasic menstuation
B. The level of estrogenic saturation ofthebody
C. Ovulation
D. The usefulness of the luteal phase ofthe
cycle
E. Hormone-producing ovarian tumor
43. Cytological cervical screening is performed
at the age of
A. 30 to 69 years old
B. 40 to 69 years old
C. 31 to 69 years old
D. 49 to 69 years old
E. 21 to 69 years old

44. Which of the following examination


methods most reliably confirm the
diagnosis of inflammation of the appendages?
A. Quantification of white blood cells;
B. Gram stain of cervical mucus smear,
C. Culdocentesis;
D.Laparoscopy:
E.Ultrasound of the pelvic organ
45. For anovulatory menstrual cycle with
short-term persistence of a mature
follicle is not typical:
A. Symptom of the pupil (+++)
B. Single-phase basal temperature
C. High estrogen levels
D. In the scraping of the endometrium in the
second phase of the cycle
E. Prolonged spotting discharge

46. Features of the treatment of DUB in


menopausal patients are:
A. Conducting a separate diagnostic curettage
B. Ovarian Suppression Therapy
C. Treatment of concomitant somatic
pathology
D. Cyclic hormone therapy
E. Hormone treatment

47. The most common mechanism for the


development of dysfunctional uterine
bleeding in the juvenile period is:
A. Increased FSH
B. Follicular persistence
C. Follicular atresia
D. Hyperprolactinemi
E. LH increase

48. The conditions for cyclic hormone therapy


in patients with menstrual
irregularities are:
A. Preliminary consultation by a neurologist
B. Preliminary consultation of the therapist
C. The use of a minimum amount of estrogen
D. Use of gestagens, ultrasound examination
of the uterus
E. Hormone testing
49. Women with AUB are at risk:
A. Miscarriage
B. On the development of placental
insufficiency and anomalies of the birth force
C. On the development of genital tumors
D. On the development of breast tumors
E. Pregnancy

50. What is the mechanism of action of the


lactational amenorrhea method (LAM):
A. Suppression of ovulation
B. Changes in the structure of the
endometrium
C. Violation of implantation
D. Thickening ofcervical mucus
E. Change in sperm movement

51. The high eficiency of the method of


lactational amenorrhea (LAM), subject to all
the nules, is observed:
A. Within 3 months after childbirth
B. Within 6 months after childbirth
C. Within 9 months after childbirth
D. Within 12 months after childbirth
E. Within 15 months after giving birth

52. What is the mechanism of action of


voluntary surgical sterilization:
A. Thickening ofcervical mucus
B. Blockage of the fallopian tubes
C. Suppression of ovulation
D. Violation of implantation
E. Thickening ofcervical mucus

53. Specify the mechanism of action of


spermicides:
A. Reduces the life span of spermatozoa
B.Destroys the membranes of spermatozoa
C. Thickening of cervical mucus
D. Violation of egg implantation
E. Dilution of cervical mucus

54. Before giving a woman an injection of


Depo-provera, the health worker must make
sure that she does not have:
A. Vaginal bleedling of unclear etiology
B. Cardiovascular disease
C. History of STls
D. Thromboembolic disorders
E. High blood pressure

55. Depo-provera is not recommended:


A. Women of any age with more than 3
children
B. Women with migraines I
C. Women with breast cancer at this time
D. Unbom women and adolescents
E. Women after abortion

56. Which of the following mechanisms best


explains the contraceptive effect
of birth control pills that contain both
synthetic estrogen and progestin?
A. Direct inhibition of oocyte maturation
B. Inhibition of ovulation
C. Production ofuterine secretions thal are
toxic to the developing
embryo
D. Impairment of implantation
E. Impairment of spernm transport due to
uterotubal obstruction

57. A woman has period every 33 days lasting


8 days with large clots being
passed. She could be classified as having
A. Metrorhagia
B. Menometrrohagia
C. Dysmenorrhea
D. Menorrhagia
E.Polymenorrhea

58.Of the following,which woman is the best


candidate for the female condom?
A.16 year old high school student who has
never had sex,but wishes to start with her new
18-year old boyfriend
B.29 year old secretary who has been married
for 7 years,and is in a monogamous
relationship with her husband
C.40 year old obese smoker whose partner
refuses to wear condoms
D.34 year old woman who has three children
and had a post-partum tubal ligation
59. For conservative treatment of Abnomal
uterine bleeding the 1st line drug is:
A. Oxytocin
B. Nsaids
C. Tranexamic acid
D. Aminocaproic acid
E. Sodium ethamsylate

60. What is the most common presenting


complaint of a woman with vaginitis?
A. Dyspareunia
B. Vaginal discharges
C. Pelvic pain
D. Fever
E. Irregular menses
61. The most effective method for diagnosing
gonococcus is:
A. Linked immunosorbent assay
B. Cultural
C. Bacterioscopic
D. Immunofluorescent
62. A 23-year-old female patient complains of
mucopurulent discharge from the genital tract,
itching at the antenatal clinic. By examination:
the mucous of the cervix is sharply hyperemic,
edematous. The discharge is profuse,
mucopurulen. uterus in anteversio-flexio, not
enlarged, painless; the appendages are not
defined. What is the most likely diagnosis:
A. Vulvovaginitis
B. Bartholinitis
C. Endometritis
D. Acute salpingo-0ophoritis
E. Endocervicitis.
63. For conservative treatment of Abnormal
uterine bleeding the Ist line drug is:
F. Oxytocin
G. NSAIDs
H. tranexamic acid
I.aminocaproic acid
J. sodium ethamsylate
64. A 16-year-old girl developed spotting from
the genital tract, lasting 8 days after a 2-month
delay. The first menstruation appeared 4
months ago for 2 days. after 28 days.
moderate. painless. Denies sexuality. The
development is correct, well physically built.
When recto-abdominal examination of the
pathology is not detected. Hb-80 g/l. Probable
diagnosis:
A. Homonc-producing ovarian tumor
B. Cervical cancer
C. Polyp of the cervix
D. Juvenile uterine bleeding
E. Endometrial polyposis

65. A mother with a 4-year-old girl turned to a


pediatric gynecologist, who has itching and
redness in the extermal genital area,purnlent
discharge from the genital tract. These
symptoms occur periodically throughout the
year. Genital hygiene is observed. The child is
observed by an allergist in connection with
atopic dermatitis (skin rashes on the elbows
and on the face).What is the most likely
diagnosis
A. Salpingo-oophoritis
B. Atopic vulvovaginitis
C. Endometritis
D. Endocervicitis
E. Bartholinitis

66. Patient L., 24 years old, complains of


profuse vaginal discharge with an unpleasant
smell of "rotten fish". Gynecological
examination: the external genital organs and
vaginal mucosa without signs of inflammation.
Vaginal discharge copious, watery, with an
unpleasant odor. Internal genital organs
without pathology. During bacterioscopy of
smears from the cervical canal and urethra,
"key cells" were found.What is the most likely
diagnosis:
A. Bacterial vaginosis
B. Candidiasis vulvovaginitis
C. Chlamydial cervicitis
D. Atrophic colpitis
E. Bartholinitis

67. Task 2 Patient R., 25 years old, was


admitted to the gynecological department
with complaints of pain in the external genital
area, awkwardness when walking, fever. Pain
appeared four days ago, after hypothermia.
Last period 3 weeks ago. By examination: in
the area of the right labia majora, a tumor-like
formation measuring 4x3 cm is determined,
the skin above it is hyperemic, hot to the
touch, fluctuation is noted on palpation.
Examination of the cervix in the speculum and
bimanual examination revealed no
abnormalities. What is the most probable
preliminary diagnosis:
A. Bartholine abscess
B. acute bilateral salpingitis, pelvioperitonitis
C. Torsion ofthe legs of the ovarian tumor
D. Necrosis of subserous myomatous node
E. Perforation of purulent formation of
appendages

68. A 23-year-old female patient came to the


antenatal clinic with complaints of cramps
during urination, fever up to 37.8 ° C, purulent
discharge from the genital tract. She has
several sexual partners.Examination by
speculum: the mucous membrane of the
urethra is hyperemic, the cervix is cylindrical,
hyperemia of the area of the external opening
of the cervical canal. Discharge from the
cervical canal is purulent. Define the tactics of
the antenatal clinic doctor:
A. Prescribe an outpatient course of antibiotic
therapy
B. Take swabs, culture for gonorrhea
C. Hospitalize the patient in a gynecological
hospital
D. Get swabs for chlamydia infection
E. Hospitalization in the dermatovenerologic
clinic

69. Patient S., 21 years old, at the reception in


the antenatal clinic complains of mucopurulent
discharge from the genital tract, itching.
Speculum exam: the cervix is hyperemic,
edematous, with pinpoint hemorrhages. The
discharge is profuse, mucopurulent. Vaginal
examination revealed no pathology. Was
diagnosed : Endocervicitis.
Define the tactics of the antenatal clinic
doctor:
A To urgently hospitalize the patient for
surgical intervention
B. Antibiotic therapy, take culture for
microorganisns
C. Healthy lifestyle, diet. phytohomeopathic
treatment.
D. Refer the patient to a dematovenerologic
dispensary
E. Recommended a re-examination after the
next mensthuation

70. Patient V., 22 years old, consulted a


antenatal clinic with complaints of discomfort
in the vagina, burning sensation, itching,
leucorrhoea. Considers himself ill for 5 days.
Examination reveald: the mucous membrane
of the vagina and cervix is sharply hyperemic,
edematous. Whitish deposits are easily
removed with a gauze ball, a cheesy discharge.
Vaginal examination was normal. Was
diagnosed : Vulvovaginal candidiasis. Define
the tactics of the antenatal clinic doctor:
A. Antifungal therapy
B. Hormonal therapy
C. Radiation therapy
D.Surgical therapy
E.Physiotherapy

71. A 20-year-old female patient turned to


clinic, complaining of abundant,purulent
discharge from the genital tract, itching,
burning and pain in the vagina.She fell ill
acutely, 2 days ago, when the above
complaints appeared. Menstrual function is
not impaired. By examination: the vaginal
mucosa is sharply hyperemic, covered with a
purulent bloom, bleeds easily when touched.
The cervix is cylindrical in shape, the mucosa is
not changed. What is the most likely diagnosis:
A. Vulvovaginitis
B. Acute colpitis
C. Endometritis
D. Endocervicitis
E. Bartholinitis
72. 5-year-old woman, unmaried, has sex 2-3
times a month, and sometimes less often.
Partners are different. There were no
pregnancies. What method of contraception is
appropriate to prescribe:
A. Intrauterine device
B. Condom
C. Combined oral contraceptives
D. Injectable contraceptives
E. Surgical sterilization

73. A young nulliparous woman, 20 years old,


has an active sex life outside of
marriage, has several sexual partners. What
methods of contraception can be
recommended to the patient:
A. Condom
B. Spermicides
C. Intrauterine device
D. Combined oral contraceptives
E. Surgical sterilization

74. A 25-year-old female patient consulted a


gynecologist to receive recommendations on
contraception. FROM ANAMNESIS: menarche
from 13 years old, iregular, painful, takes
spasmaton to relieve pain. Maried.In obstetric
anamnesis was 1birth 3 years ago. She does
not plan pregnancy for the next 2-3 years, she
wants reliable contraception. somatically
healthy. Gynecological examination and
ultrasound revealed no pathology. What
method of contraception is recommended for
this patient:
A. Surgical sterilization
B. Calendar rhythm method.
C. Intrauterine device with silver
D. Combined oral contraceptives
E. Postcoital contraception (Postinor, Dvella)

75. Patient I., 42 years old, consulted a


gynecologist to receive recommendations on
contraception, suffers from hypertension for 5
years. Regular sex life in marriage. History of 2
births, 2 medical abortions. General condition
is satisfactory, blood pressure is 135/90 mm
Hg., height 164 cm, weight 90 kg, BMI=35.
Recommended method of contraception:
A. Condom
B. Calendar rhythm method
C. Intrauterine device with silver
D. Surgical sterilization
E. Postcoital contraception (Postinor, Dvella)
Additional questions(pediatric)
1. The timing of the first signs of puberty in boys
(years):

A. 7-9
B. 10-11

C. 13-14

D. 15-16

E. 14-15

2. The timing of the first signs of puberty in girls


(years):

A. 5-6

B. 7-8

C. 9-10

D. 13-14

E. 14-15

3. The first signs of puberty for boys:

A. Pubic hair distribution

B. Start of growth of the testicles and penis

C. Axillary hair distribution

D. voice change

E. Armpit hair growth, further voice change, facial hair


appearance, pigmentation

4. The first signs of puberty for girls:

A. The growth of the pelvic bones, the rounding of the


buttocks, a slight

elevation of the mammary glands

B. Dome-shaped lifting of the mammary gland (“bud”


stage), appearance of pubic

hair.

C. The increase in external genital organs, change

D. Development of the glandular tissue of the


mammary glands and areas adjacent

to the areola

E. underarm hair growth, not regular

5. At premature puberty, bone age:

A. corresponds to the passport age;

B. Ahead of passport age;

C. lags behind passport age.

D. 3-4 years behind

E. Corresponds to 5-7 years

6. The onset of puberty in boys begins with:

A. Enlarged penis length

B. pigmentation of the skin of the scrotum;

C. enlarged testicles;

D. the appearance of initial pubic hair and in the


axillary region;

E. voice mutations.

7. The form of violation of sexual differentiation,


related to the false female

hermaphroditism:

A. Klinefelter syndrome;

B. congenital dysfunction of the adrenal cortex;

C. testicular feminization syndrome;

D. mixed testicular dysgenesis.

8. Growth inhibition due to cerebral-pituitary nanism,


most often diagnosed:

A. at birth;

B. in the first year of life;


C. 2-4 years;

D. in puberty

E. 5-7 years old

9. For the primary forms of hypogonadism are


characteristic:

A. high level of gonadotropic hormones and low level


of sex hormones;

B. low level of sex hormones and low level of


gonadotropic hormones;

C. low level of gonadotropins and sharply positive test


of chorionic gonadotropin

D. high levels of sex hormones and high levels of


gonadotropic hormones;

E. low level of sex hormones and high level of


gonadotropic hormones;

10. The lack of function of the sexual glands is


evidenced by the absence of

secondary sexual characteristics in boys older:

A. 11 years;

B. 13.5 years;

C. 15 years.

D. 10 years

E. 18 years

11. The following symptoms are characteristic of true


precocious puberty:

A. acceleration of bone age, pubertal values of levels of


luteinizing and follicle

stimulating hormones in the blood;

B. a sharp increase in the levels of luteinizing and


follicle-stimulating hormones in
the blood;

C. hyperplasia of one or two adrenal glands;

D. increased blood levels of 17-hydroxyprogesterone;

E. Changing the set of sex chromosomes according to


type 46 XX / XO
PREVIOUS YEAR PAPER
GYNECOLOGY TESTS (500) FOR THE 5th YEAR
STUDENTS OF THE GENERAL MEDICINE.
1) A positive test (small test) with dexamethasone indicates
that:
A. The source of hyperandrogenism is the ovaries
B. The source of hyperandrogenism is the adrenal gland
C. Hyperandrogenism associated with pituitary adenoma
D. Hyperandrogenism due to adrenal corticosteroma
E. Hyperandrogenism associated with an ovarian tumor

2) The therapeutic and diagnostic effect of dexamethasone


in hyperandrogenism is due to:
A. Inhibition of ovarian function
B. Inhibition of adrenal function
C. Inhibition of ACTH
D. Acceleration of androgen inactivation
E. Inhibition of pituitary function

3) To restore generative function during gonad dysgenesis, it


is necessary:
A. Long-term cyclic therapy with sex hormones
B. Stimulation of ovulation
C. Functional diagnostic tests
D. Sphenoid ovarian resection
E. Restoring generative function is futile.

4) Dysfunctional uterine bleeding is called:


A. bleeding due to changes in the uterus
B. bleeding in inflammatory diseases of the uterus
C. due to impaired rhythmic secretion of ovarian
hormones
D. Genital bleeding in Werlhof disease
E. Bleeding due to miscarriage

5) The most common mechanism for the development of


dysfunctional uterine bleeding in the juvenile period is:
A. hypoluteinism
B. follicle persistence
C. atresia of the follicle
D. hyperprolactinemia
E. disorders in the blood coagulation system

6) The most informative way to assess the functional state of


the ovaries is:
A. basal temperature measurement
B. symptom of cervical mucus tension
C. suction curettage
D. laparoscopy
E. Hysteroscopy

7) According to the tests of functional diagnostics (TFD), it


is impossible to determine:
A. biphasic menstruation
B. The level of estrogenic saturation of the body
C. Ovulation
D. The usefulness of the luteal phase of the cycle
E. Hormone-producing ovarian tumor
8) For anovulatory menstrual cycle with follicular atresia is
characteristic:
A. amenorrhea
B. Algomenorrhea
C. oligoamenorrhea
D. promenomenorrhea
E. hypomenorrhea

9) The conditions for cyclic hormone therapy in patients


with menstrual irregularities are:
A. Preliminary consultation by a neurologist
B. Preliminary consultation of the therapist
C. The use of a minimum amount of estrogen,
taking into account the age of a woman, monitoring tests
of functional diagnostics
D. use of gestagens, ultrasound examination of the
uterus
E. hormone testing

10) A 16-year-old girl developed spotting from the genital


tract, lasting 8 days after a 2-month delay. The first
menstruation appeared 4 months ago for 2 days, after 28
days, moderate, painless. Denies sexuality. The
development is correct, well physically built. When recto-
abdominal examination of the pathology is not detected. Нb-
80 g / l. Probable diagnosis:
A. hormone-producing ovarian tumor
B. cervical cancer
C. Polyp of the cervix
D. juvenile uterine bleeding
E. endometrial polyposis
11) Pathogenetic therapy of endometrial hyperplastic
processes in women of reproductive age consists in the use
of:
A. estrogen-progestogen drugs or progestogens
B. dexamethasone
C. androgen
D. Thyroidin
E. estrogen

12) In the anovulatory menstrual cycle, basal temperature is


characterized by:
A. rise in temperature after ovulation
B. lack of temperature rise
C. rise in temperature before ovulation
D. rise in temperature before menstruation
E. Rise in temperature immediately after menstruation

13) Most often, women aged 40-45 years complain of:


A. Algodismenorea
B. heavy periods
C. irregular menstruation
D. premenstrual tension
E. painful menstruation

14) Anovulatory menstrual cycle is characterized by:


A. cyclic changes in the body
B. Long-lasting follicle persistence
C. Prevalence of gestagens in phase 2
D. prevalence of estrogen in phase 1
E. Acyclic change in the body

13) The hypothalamus produces the following hormones:


A. Gonadotropins
B. Estrogens
C. gestagens
D. Releasing factors
E. prolactin

14) Releasing factors carry out:


A. Transmission of nerve impulses to the nervous system
B. Production of gonadotropins
C. Estrogen Production
D. Production of FSH and LH
E. ACTH production

15) In the glomerular zone of the adrenal cortex are formed:


A. Glucocorticoids
B. Aldosterone
C. Norepinephrine
D. Androgens
E. Estrogens

16) FSH stimulates:


A. Ovarian follicle growth
B. Corticosteroid production
C. Production of TSH in the thyroid gland
D. Progesterone production
E. Androgen production

17) Small doses of estrogen:


A. Stimulate FSH production
B. Suppress FSH production
C. Strengthen LH production
D. Suppress LH production
E. Stimulates TTG production

18) Polymenorrhea is:


A. Lean menstruation
B. Short menstruation (1-2 days)
C. Short menstrual cycle
D. Heavy menstruation
E. Painful menstruation

19) For anovulatory menstrual cycle with short-term


persistence of a mature follicle is not typical:
A. Symptom of the pupil (+++)
B. Single-phase basal temperature
C. High estrogen levels
D. In the scraping of the endometrium in the second phase
of the cycle - the late phase of proliferation
E. Prolonged spotting discharge

20) The treatment of patients with ovarian exhaustion


syndrome should be directed:
A. To stimulate ovarian function
B. On getting a menstrual-like reaction
C. To reduce the severity of vegetovascular disorders
D. On stimulation of ovulation
E. On getting cyclical menstruation

21) The mechanism of action of hormones on the cell is due


to the presence of:
A. Prostaglandins
B. Receptors
C. Specific Enzymes
D. Isoenzymes
E. Thromboxant

22) The effect of large doses of exogenous estrogen on the


ovary:
A. Increased ovarian production of estrogen
B. Increased ovarian production of progesterone
C. Atrophy of the ovarian tissue
D. Reduces ovarian progesterone production
E. Reduces ovarian production of estrogen

23) Amenorrhea is the absence of menstruation during:


A. 4 months
B. 5 months
C. 6 months
D. 1 year
E. 2 months
24) Clinical manifestations, a typical form of gonadal
dyskinesia karyotype (45XO / 46XY) and no menstruation:
A. Sheehan Syndrome
B. Swyer Syndrome
C. Bisexual Gonad Syndrome
D. Symmonds disease
E. Shereshevsky-Turner

25) In patients with amenorrhea with Symmonds disease,


the following clinical manifestation is not observed:
A. Severe metabolic endocrine disorders
B. Premature aging
C. Decreased appetite
D. Sudden exhaustion
E. Massive bleeding

26) Amenorrhea associated with gigantism:


A. With hyperproduction of ACTH until puberty
B. With hyperproduction of GH, until puberty
C. With hyperproduction of ACTH after puberty
D. With hyperproduction of GH after puberty
E. With hyperproduction of ACTH during puberty

27) When treating a patient with any form of gonadal


dysgenesis, recovery, as a rule, is excluded:
A. Menstrual function
B. Sexual function
C. Generative function
D. Proliferative function
E. Ovulation

28) Shereshevsky-Turner syndrome is:


A. A “pure” form of gonadal dysgenesis
B. A typical form of gonadal dysgenesis
C. "Mixed" form of gonadal dysgenesis
D. False male hermaphroditism
E. The pathological process associated with massive
bleeding in childbirth

29) In a negative test with gonadotropins in patients with


amenorrhea, it is shown:
A. Test with estrogens and gestagens in a cyclic mode
B. Laparoscopy and ovarian biopsy
C. Laparoscopy and gonad removal
D. Progesterone test
E. Functional diagnostic tests

30) In the ovary, androgens serve as a substrate for the


formation of:
A. Progesterone
B. Estrogen
C. Progesterone and estrogen
D. Gestagenov
E. Testosterone

31) The main estrogen hormone in the body of a woman in


the postmenopausal period is:
A. Estradiol
B. Estron
C. Estriol
D. Estradiol dipropionate
E. estrogen

32) The following applies to hypergonadotropic


amenorrhea:
A. Ovarian Depletion Syndrome
B. Resistant Ovarian Syndrome
C. Ovarian Dysgenesis Syndrome
D. Testicular feminization syndrome
E. Sheehan Syndrome

33) Features of the treatment of DUB in menopausal


patients are:
A. Conducting a separate diagnostic diagnostic curettage
to verify the diagnosis
B. Ovarian Suppression Therapy
C. Treatment of concomitant somatic pathology
D. Cyclic hormone therapy
E. Hormone treatment

34) Women with DUB are at risk:


A. Miscarriage
B. On the development of placental insufficiency and
anomalies of the birth force
C. On the development of genital tumors
D. On the development of breast tumors
E. Pregnancy
38) In the diagnosis of amenorrhea associated with
acromegaly and gigantism, a change is significant:
A. the size of the Turkish saddle on the radiograph of the
skull
B. visual fields
C. FSH level
D. Excretion of 17-cop
E. Ultrasound diagnosis of the pelvic organs

39) Gestagens are secreted:


A. luteal cells of the corpus luteum
B. cells of the granular layer of the follicle
C. cells of the outer connective tissue of the follicle
D. Ovary cells
E. theca cells

40) Amenorrhea is considered pathological when


menstruation is absent
A. before puberty
B. during puberty
C. during lactation
D. in postmenopausal women
E. during pregnancy

41) False amenorrhea can be caused by:


A. atresia of the cervical canal
B. Aplasia of the uterus
C. Dysgenesis of gonads
D. Ovarian exhaustion syndrome
E. Ovarian tumor

42) Ovarian exhaustion syndrome needs to be differentiated


A. with resistant ovary syndrome
B. with gonad dysgenesis syndrome
C. with menopausal syndrome
D. with ovarian tumor
E. with premenstrual syndrome

43). Treatment for DUB does not include:


A. physiotherapy
B. vitamins
C. uterotonics
D. antibiotics
E. hormonal hemostasis

44). Which test of functional diagnostics indicates the


presence of a biphasic menstrual cycle?
A. Pupil symptom
B. Karyopicnosis Index
C. Basal thermometry
D. Symptom of "fern"
E. Symptom of crystallization

45) Algodismenorea is:


A. Rare and scarce menstruation
B. painful menstruation
C. Reduced blood loss during menstruation
D. intermenstrual meager spotting
E. heavy periods

46). Menorrhagia is:


A. Acyclic uterine bleeding
B. cyclic uterine bleeding
C. Painful and heavy menstruation
D. pre- and postmenstrual bleeding
E. Change in the rhythm of menstruation

47) Metrorrhagia is
A. Change in the rhythm of menstruation
B. increased blood loss during menstruation
C. increase in the duration of menstruation
D. Acyclic uterine bleeding
E. Reducing blood loss during menstruation

48) The most common mechanism for the development of


dysfunctional uterine bleeding in the juvenile period is
A. increased FSH
B. follicular persistence
C. follicular atresia
D. hyperprolactinemia
E. LH increase
49) The main method for stopping dysfunctional uterine
bleeding in the premenopausal period is:
A. use of synthetic estrogen-progestogen drugs
B. The introduction of hemostatic and uterine-contracting
agents
C. Use of 17-hydroxyprogesterone capronate (17-OPK)
continuously
D. Separate diagnostic curettage of the mucous
membrane of the uterine cavity and cervical canal.
E. use of progestogens

50) The composition of injectable contraceptives includes:


A. prolonged progestogens
B. conjugated estrogens
C. Microdoses of progestogens
D. Antiandrogens
E. estrogens

51) False amenorrhea can be caused by:


A. atresia of the cervical canal
B. Aplasia of the uterus
C. Dysgenesis of gonads
D. follicular atresia
E. Severe Infectious Diseases

52) False amenorrhea may be the result of:


A. hypothyroidism
B. neurogenic anorexia
C. testicular feminization syndrome
D. hyperthyroidism
E. Atresia hymen

53) What additional research methods are not used to clarify


the diagnosis of amenorrhea:
A. Ultrasound examination of the internal genital organs
B. Functional diagnostic test examination
C. Craniography
D. Separate curettage
E. Determination of hormone levels

54) What form of amenorrhea indicates a negative result of


a functional test with combined estrogen-progestogen
drugs?
A. Hypothalamic
B. Pituitary
C. Ovary
D. Uterine
E. Central

55) When a combination of uterine fibroids and internal


uterine endometriosis in a patient of reproductive age with
hyperpolymenorrhea and secondary anemia is shown:
A. Extirpation of the uterus with appendages
B. Subvaginal uterine amputation without appendages
C. Subvaginal amputation of the uterus with fallopian
tubes, with excision of the mucous membrane of the
cervical canal
D. Subvaginal amputation of the uterus with tubes
E. Uterine amputation with appendages
56) The term "Adenomyosis" applies:
A. In all cases of detection of endometriosis, regardless of
location
B. Only with focal growths of endometrioid tissue in the
muscle layer of the uterus
C. With endometriosis, which is accompanied by the
formation of cysts
D. Only in cases when the germination of the myometrium
is accompanied by the presence of myomatous nodes
E. With endometriosis of the cervix

57) For internal endometriosis of the uterus of the 3rd stage


on the eve of menstruation, a bimanual examination is not
characteristic of:
A. Uterine compaction
B. Uterine enlargement
C. Uterine softening
D. Severe soreness
E. Uterus reduction

58) A 38-year-old patient complained of abdominal pain.


The pain appeared today 3 hours ago, weakly positive with \
m Shchetkina. T-38.2 C, leukocytosis. With a gynecological
examination, the uterus is enlarged to 8 weeks, nodular.
Diagnosis:
A. Inflammation of the appendages
B. Charioamnionitis
C. Endometritis
D. Necrosis of myoma nodes
E. Salpingitis

59) A 38-year-old woman complained of very painful


menstruation during the last 6 years, especially in the first 2
days. A history of 2 births and 2 medical abortions without
complications, the last year ago. The menstrual cycle is not
broken. The last menstruation ended 5 days ago. She was
protected from pregnancy by interrupted sexual intercourse.
On examination, the abdomen without painful, the cervix
and vagina without pathologies, the body of the uterus is
slightly more than normal, dense, the appendages are not
palpable. Probable diagnosis:
A. Uterine fibroids
B. endometriosis
C. Uterine pregnancy
D. Endometrial polyposis
E. Adenomyosis

60) The main symptom of submucous uterine fibroids:


A. Chronic pelvic pain
B. Algodismenorea
C. Menorrhagia
D. Acute pain
E. Chills

61) The most informative method for diagnosing a born


myomatous node:
A. Transvaginal ultrasound
B. Inspection by the speculum
C. Hysteroscopy
D. Laparoscopy
E. Bimanual research

62) An informative method for the diagnosis of submucous


myomatous node:
A. Inspection by the speculum
B. Laparoscopy
C. Hysteroscopy
D. Colposcopy
E. Sonography

63) The method of drug treatment of uterine fibroids in


reproductive age:
A. gestagens
B. Estrogens
C. Androgens
D. Progestins
E. Vitamin Therapy

64. What are the leading clinical symptoms of dysplasia and


in situ cervical cancer:
A. pelvic pain;
B. mucopurulent leucorrhoea;
C. contact bleeding;
D. Acyclic uterine bleeding;
E. infertility.

65) The optimal amount of surgical treatment in the


presence of uterine fibroids with the location of the node in
the cervix:
A. Subvaginal uterine amputation
B. Conservative myomectomy using vaginal access
C. Extirpation of the uterus
D. Surgical treatment of uterine fibroids does not result in
such localization
E. Subvaginal uterine amputation with appendages

66) Which of the following symptoms are not related to


uterine fibroids?
A. Palpable palpable tuberous formation in the small pelvis
associated with the cervix
B. Painful menstruation
C. Heavy menstruation
D. Dysfunction of the bladder and rectum
E. Rare menstruation

67) What complications are characteristic for the subserous


form of uterine fibroids?
A. Malignant degeneration of the tumor
B. Twisting the legs of the tumor
C. Uterus inversion
D. Posthemorrhagic anemia
E. Bladder dysfunction

68) To prevent the development of cervical endometriosis,


organ cryodestruction is performed on the following days of
the menstrual cycle:
A. 1-2 days before your period
B. Immediately after menstruation
C. On day 12-14
D. On day 16-18
E. On day 16-25

69) Basal temperature is measured:


A. In the morning
B. In the evening;
C. 2 times a day;
D. After 3 hours.
E. After 5 hours

70. The ovary is maintained in the abdominal cavity due to:


A. round ligament;
B. cardinal ligament;
C. funnel-pelvic ligament;
D. Sacro-uterine ligament
E. Broad ligament

71. The microorganisms that are most often the causative


agents of inflammatory diseases of the female genital organs
of a non-specific etiology do not include:
1. staphylococci
2. streptococci
3. gonococci
4. gardnerella
5. anaerobes
72. Blood supply to the ovaries is carried out:
A. uterine artery;
B. ovarian artery;
C. ileo-lumbar artery;
D. Internal genital and ovarian arteries;
E. Uterine and ovarian arteries.

73. Ovarian cysts include:


A. dermoid cyst
B. follicular cyst
C. corpus luteum cysts
D. piovar
E. Theca-luteal cyst

74. With a slight bleeding from the ovary, detected by


laparoscopy, is performed:
A. suturing the ovary;
B. diathermocoagulation of the ovary under the control
of laparoscopy;
C. Ovarian resection
D. laparotomy and removal of the uterine appendages one
hundred

75. What is the most common complication in benign


ovarian tumors?
A. hemorrhage into the tumor cavity;
B. capsule rupture;
C. torsion of the legs of the tumor;
D. suppuration of contents;
E. Compression of adjacent organs.

76. The most informative in the differential diagnosis


between uterine myoma and ovarian tumor:
A. two-handed vaginal examination
B. hysteroscopy
C. ultrasound;
D. laparoscopy;
E. Probing of the uterine cavity.

77. With genital tuberculosis, the primary focus is most


often localized:
A. In the lungs
B. In bone
C. In the urinary system
D. In the lymph nodes
E. On the peritoneum

78.. What parts of the female reproductive system are most


often affected by tuberculosis?
A. fallopian tubes
B. Ovaries
C. Uterus
D. external genitalia
E. vagina

79. In what age period is tuberculosis of the internal genital


organs most often detected?
A. in childhood
B. puberty
C. in the reproductive period
D. in the premenopausal period
E. with the same frequency in any of the above periods
80. The main clinical symptom of tuberculosis of the uterus
appendages?
A. chronic pelvic pain
B. amenorrhea
C. Menometerorrhagia
D. infertility
E. DUB

81. During an operation for a paraovarial cyst:


A. cyst husking;
B. removal of appendages on the affected side;
C. removal of the ovary on the affected side;
D. Ovarian resection on the affected side;
E. Removal of the uterus with appendages

82. Blood test CA-125 for endometrioid ovarian cysts:


A. elevated
B. lowered
C. normal
D. negative
E. doesn't matter

83. What is the main clinical symptom of bacterial


vaginosis?
A. Itching of the external genitalia and perineum
B. dyspareunia
C. Profuse unpleasant whitewash
D. dysuria
E. pain in the lower abdomen
84. A characteristic feature of ovarian cysts:
A. increase due to the accumulation of liquid contents;
B. do not have capsules;
C. relate to malignant tumors of the female genital organs;
D. have invasive growth.
E. have fusion with neighboring organs

85. The following diagnostic method is used to recognize


ovarian tumors:
A. cytological
B. endoscopic
C. ultrasound
D. histological
E. bacterioscopic

86. For menopausal manifestations of a typical form of


menopause syndrome is most characteristic:
A. Dry mucous membranes
B. Pain in the heart
C. Osteoporosis
D. Laryngitis
E. Anemia

87. Specify precancerous changes in the vaginal part of the


cervix:
A. recurrent cervical canal polyp
B. true cervical erosion
C. Cervical dysplasia
D. cervical ectropion
E. Cervical ectopy

88. The main route of transmission of HPV:


A. lymphogenous
B. airborne
C. sexual
D. hematogenous
E. Contact Household

89. Itching of the vulva as a consequence of neuroendocrine


disorders is characteristic of:
A. candidiasis
B. trichomoniasis
C. krauroza
D. leukoplakia
E. erythroplakia
90. The screening method to detect cervical pathology in
modern conditions is:
A. visual inspection;
B. smear cytology;
C. vaginal smear on the flora
D. radionuclide method
E. PCR for HPV
91.. The main etiological factor for dysplasia and cervical
cancer is:
A. herpes simplex virus type 2;
B. human papillomavirus;
C. hyperestrogenia;
D. violation of the pH of the vaginal secretion;
E. immune and metabolic disorders in the body.

92. When conducting extended colposcopy after treatment


with a 3% solution of cervical acetic acid, the following
epithelial reaction is normal:
A. does not change;
B. turns pale;
C. evenly stained dark brown;
D. coated with white coating;
E. becomes embossed, papillae are visible in the form of
“grapes”.

93. The treatment of acute endometritis does not include:


A. detoxification and infusion therapy
B. glucocorticoids
C. antibiotics
D. desensitizing agents
E. Vitamins

94. A qualitative reaction (Schiller test) is due to the


interaction of iodine with cervical epithelium contained in
the multilayer:
A. glycogen;
B. proteins;
C. fats;
D. Ca salts;
E. immunoglobulins.

95. The differential diagnosis of acute salpingo-oophoritis


does not include:
A. acute appendicitis
B. tube abortion
C. colpitis
D. Ovarian apoplexy
E. Torsion of cyst legs

96. The main cause of adrenogenital syndrome (congenital


dysfunction of the adrenal cortex - CDAC) is:
A. Chronic inflammatory diseases of the ovaries;
B. Adrenal gland tumor;
C. Inherited deficiency of C21 - hydroxylase;
D. Decreased secretion of THG;
E. Hyperproduction of ACTH.

97. For bleeding from the genital tract in girls under the age
of 9, it is necessary:
A. Hormonal hemostasis
B. Observation
C. Appointment of hemostatic and uterine-contracting
agents
D. Exclusion of a local “organic” cause of bleeding
E. Hormone therapy

98. Specify the main criteria for PCOS:


A. weight loss;
B. Hyperandrogenism;
C. Normal ovulation;
D. Echographic signs in the uterus;
E. Polymenorrhea.

99. Indicate the universal diagnostic ultrasound criterion for


PCOS:
A. Increasing the thickness of the M-echo;
B. Increase in ovarian volume ≥ 7 cm2;
C. Increase in ovarian volume ≥ 5 cm3;
D. The presence of hyperplastic stroma;
E. At least 5 follicles along the periphery of the ovary d =
10mm.

100. Indicate typical clinical diagnostic signs of PCOS:


A. Polymenorrhea;
B. AMK;
C. Anorexia
D. Dysmenorrhea;
E. Infertility, chronic anovulation.

101. Hirsutism is ...


A. Excessive hair growth
B. Excessive male terminal hair growth
C. Excessive growth of nail plates
D. Overweight
E. Weight loss

102. Hormonal changes in PCOS are characterized by the


presence of:
A. A sharp increase in the secretion of FSH, LH;
B. Increases in TSH levels;
C. Decreased levels of testosterone, estrogen;
D. Increased testosterone levels, 17-OP;
E. Decreased prolactin levels.

103. Treatment of CDAC (congenital dysfunction of the


adrenal cortex) is carried out:
A. Regulon;
B. Femoston 1/5;
C. Dexamethasone;
D. clomiphene;
E. metformin

104. Polycystic ovary syndrome (PCOS) is characterized by:


A. Bilateral reduction of the ovaries;
B. Hyperandrogenism;
C. The ratio of LH / FSH is less than 2.5;
D. Habitual miscarriage;
E. AMK.
105. When identifying adrenogenital syndrome (AGS),
treatment must begin:
A. Since diagnosis
B. After the establishment of menstrual function
C. After marriage (depending on the time of the planned
pregnancy)
D. Only after childbirth
E. After menopause
106. Among various forms of ectopic pregnancy, tubal
pregnancy is
A. about 28%
B. about 48%
C. about 68%
D. about 88%
E. about 98%

107. In the pathogenesis of PCOS, there are:


A. Hypoprolactinemia;
B. Pituitary dysfunction;
C. Violation of the menstrual cycle;
D. Ovarian hyperandrogenism and decreased synthesis
of estradiol;
E. AMK.
108. Symptoms are observed in the clinical presentation
of PCOS:
A. Tachycardia;
B. Anorexia;
C. Polymenorrhea;
D. hirsutism;
E. Infertility.

109. For the diagnosis of PCOS, it is necessary to carry out:


A. Ultrasound, hormonal examination, biochemical
analysis of blood
B. MRI of the brain, mammography
C. Culdocentesis;
D. X-ray of OGK;
E. Hysteroscopy.
110. At the first stage of treatment for PCOS, it is
prescribed:
A. Low-calorie diet, metformin, lipase inhibitors;
B. Laparoscopic resection of 2/3 of the ovaries;
C. Clomiphene ovulation stimulator;
D. Monophasic COCs;
E. Progestogens in the second phase of the menstrual cycle.

111. CDAC (congenital dysfunction of the adrenal cortex) is


...
A. Hormone-active pituitary adenoma (corticotropinoma);
B. Genetically determined deficiency of 21-hydroxylase
involved in the synthesis of cortisol;
C. Androgen-producing adrenal gland tumor;
D. Adrenal hypoandrogenia;
E. Acromegaly.

112. The clinical forms of CDAC (congenital dysfunction of


the adrenal cortex) are:
A. Salting;
B. Vascular;
C. Asthenovegetative;
D. Edematous;
E. Total.

113. Hormonal changes in CDAC (congenital dysfunction


of the adrenal cortex)
A. Increased cortisol levels;
B. Increased levels of LH and FSH, the ratio of LH / FSH
more than 2.5;
C. Increased levels of 17-SNP, DHEA-S;
D. Decrease in testosterone and estradiol;
E. Decreased ACTH.
114. For the differential diagnosis of viril syndromes
(PCOS, CDAC) carry out:
A. ultrasound of internal organs;
B. Biochemical studies;
C. CT, MRI of the pituitary, adrenal gland;
D. A test with estrogen;
E. Hysterosalpingography.

115. Progressive tubal pregnancy can be reliably diagnosed


with
A. Bimanual research
B. Ultrasound Research
C. Curettage of the uterus
D. Puncture of the posterior arch
E. Serological Pregnancy Response
116. Adrenogenital syndrome
A. Has a synonym for adrenal hyperandrogenism
B. Manifested by hypermenstrual syndrome
C. May cause infertility
D. It is necessary to differentiate with menopausal syndrome
E. Treated HRT

117. To confirm adrenogenital syndrome (AGS) use


A. Determination of blood testosterone and 17
ketosteroids in urine
B. Probing of the uterus
C. Hysterosalpingography
D. Laparoscopy
E. Pelvic Scan
118. Pathogenetic treatment for adrenogenital syndrome
(AGS) is
A. Therapy aimed at normalizing body weight
B. Improving cerebral hemodynamics
C. Glucocorticoid therapy
D. Use of vegetotropic drugs (Belloid)
E. Use of small doses of thyroidin

119. In the development of polycystic ovary syndrome, the


following are essential:
A. Increased ovarian secretion of estrogen
B. Increased ovarian secretion of progesterone
C. Increased secretion compared to androgen
D. Increased secretion of prolactin
E. Decreased TSH secretion

120. Sterilization is carried out by:


A. Pipe compression;
B. Hysterosalpingography;
C. Tube ligation;
D. Hysteroscopy
E. Pipe removal.

121. For the diagnosis of polycystic ovaries, the following is


used:
A. General clinical laboratory tests
B. Functional tests with progesterone and
dexamethasone
C. Adrenal ultrasound
D. Laparotomy
E. Functional tests with estrogen

122. Which of the tests of functional diagnostics indicates


the presence of a two-phase menstrual cycle?
A. Pupil symptom
B. Karyopicnosis Index
C. Basal thermometry
D. Symptom of "fern"
E. Cervical index
123. A negative dexamethasone test (a slight decrease in the
excretion of 17-ACS and 17-KS) indicates the presence of:
A. Tumors of the adrenal cortex;
B. Adrenogenital Syndrome (AGS);
C. Neuro-exchange-endocrine syndrome;
D. Sclerocystic ovary syndrome.
E. Pituitary tumors

124. The main clinical signs in true premature puberty of the


central genesis
A. neurological symptoms,
B. intracranial hypertension
C. emotional disturbances
D. lack of menstruation until 15-16 years
E. weight gain

125. The full form of premature puberty is characterized by:


A. varying degrees of development of secondary sexual
characteristics and abcense of menstruation
B. bone age corresponds to the calendar
C. Body weight not elevated
D. lack of neurological symptoms
E. emotional disturbances
126. Features of the physique of girls with congenital
adrenogenital syndrome
A. narrow shoulders and wide pelvis
B. long limbs
C. tall
D. overweight
E. neurological symptoms

127. Causes of the viril syndrome in a girl


A. dysfunction of the adrenal cortex
B. masculinizing tumors of the adrenal cortex
C. Congenital androgenital syndrome
D. chromosome abnormalities
E. pituitary adenoma

128. During puberty, the following major changes occur in


the body:
A. Suppression of the pituitary gonadotropic function;
B. activation of hormonal function of the ovaries;
C. The rhythm of FSH release is not established;
D. regular “peaks” of LH excretion are established;
E. elevated prolactin levels

129. Nipple pigmentation and breast enlargement usually


occur:
A. at 8-9 years old;
B. at 10-11 years old;
C. at 12-13 years old;
D. at the age of 14-15;
E. at the age of 16-18.

130. The anatomical features of the uterus in a newborn girl


include:
A. The body of the uterus is small, the cervix is almost not
expressed;
B. the uterus is small, the length of the cervix is almost 3
times the length of the uterus;
C. The uterus has a bicorn shape;
D. The body of the uterus is almost 2 times larger than the
cervix;
E. Uterus curved posteriorly

131. The diagnosis of absolute female infertility can be


made:
A. in the absence of one fallopian tube
B. in the absence of the uterus
C. in PCA syndrome
D. during anovulatory cycles
E. with AGS syndrome

132. To exclude cervical factor of infertility use:


A. Shuvarsky-Huner test
B. Hysterosalpingography
C. Study of sex chromatin
D. Chromosomal analysis
E. Determination of Antisperm antibodies in the blood

133. The course of hydrotubations is shown:


A. with partial obstruction of the fallopian tubes
B. With obstruction of the tubes in the ampullar sections
C. With severe adhesions in the pelvis.
D. With an abundance of peritubular adhesions
E. In Asherman's Syndrome

134. Good patency of the fallopian tubes can be judged by


the data of chromohydrotubation in case urine
A. blue after one hour
B. Green in one hour
C. Green in two hours
D. After one hour, colorless
E. remains colorless after 24 hours

135. The frequency of male infertility is


A. 10-15%
B. 15-20%
C. 20-30%
D. 40-50%
E. 60-70%

136. Microsurgical operations on the fallopian tubes are


performed
A. with tube occlusion in various departments
B. In bilateral tubo-ovarian formations
C. With severe adhesions in the pelvis
D. With a duration of infertility of more than 10 years
E. In patients older than 35 years
137. Artificial insemination with donor sperm is used
A. with Rokytansky-Mayer-Küster syndrome
B. in women with anovulatory cycle
C. in women with Asherman syndrome
D. for male infertility
E. tubal infertility

138. Secondary infertility is called

A. Two-year infertility
B. Infertility in women with a history of pregnancy
C. Infertility due to endocrine pathology
D. Infertility due to inflammatory genesis
E. Husband Disease

139. What percentage of abnormal sperm is permissible in a


normal spermogram?
A. 1-2%
B. 6-8%
C. 10-15%
D. 20-40%
E. over 50%

140. What percentage of motile sperm is acceptable in a


normal spermogram?
A. 10%
B. 30%
C. 50-60%
D. 80%
E. 90%

141. The total number of sperm in a normal male ejaculate:


A. 20 - 40 million
B. 100 thousand - 1 million
C. 60 - 80 million
D. 10 - 20 million
E. 150 - 200 million

142. Reducing the number of antibodies to sperm in cervical


mucus can be achieved
A. Using oral contraceptives
B. taking antihistamines
C. Having more frequent coituses
D. Using condoms
E. Irrigating the cervix

143. The modern method of treating tubal infertility is:


A. Artificial insemination using sperm from a donor;
B. psychotherapy;
C. insemination;
D. In Vitro Fertilization
E. Stimulation of ovulation

144. When examining a sterile couple, it is primarily shown:


A. Hysterosalpingography
B. Cytology of the vaginal smear
C. Determination of basal temperature
D. Sperm examination
E. ECHO-GHA

145. The term "primary infertility" means that:


A. there was no pregnancy
B. no live births were born
C. All pregnancies are interrupted for medical reasons
D. a woman has no internal genital organs
E. All pregnancies are interrupted spontaneously

146. The synthesis of gonadoliberin is carried out in:


A. Anterior pituitary gland
B. The posterior pituitary gland
C. The nuclei of the hypothalamus
D. Neurons of the cerebral cortex
E. Middle lobe of the pituitary gland

376. 147. The following does not apply to endoscopic


research methods in gynecology:
A. Hysteroscopy;
B. Colposcopy;
C. Culdocentesis
D. Laparoscopy;
E. Culdoscopy.
148. With tubal-peritoneal infertility, the most effective
treatment is:
A. therapeutic laparoscopy;
B. laparotomy, microsurgical intervention on the pipes
(resection of the pipe section, the imposition of
anastomoses);
C. course of antibacterial and anti-inflammatory therapy;
D. enzyme preparations;
E. physiotherapeutic treatments.

149. The presence of ovulation does not determine:


A. functional diagnostic tests;
B. tests with LH;
C. ultrasound monitoring;
D. determination of progesterone levels in phase 2 of the
menstrual cycle;
E. FSH indicators

150. As a treatment method for infertility on the background


of intrauterine synechia does not include:
A. laparotomy, metroplasty;
B. hysteroresectoscopy;
C. hormonal treatment with estrogen-progestogen drugs;
D. antibacterial and anti-inflammatory treatment;
E. Stimulation of ovulation

151. At the first stage, in IVF programs:


A. embryo transfer;
B. transvaginal ovarian puncture;
C. maintaining the luteal phase;
D. stimulation of superovulation;
E. In vitro fertilization.
152. Sperm in the crypts of the cervical canal can retain the
ability to move (deadline) for
A. 6-12 hours
B. 24-48 hours
C. 3-5 days
D. 10 days
E. 1-2 hours

153. Sperm after penetration into the uterus and tubes


retain fertility for
A. 6-12 hours
B. 24-48 hours
C. 3-5 days
D. 10 days
E. 1-2 hours

154. After ovulation the egg remains fertile for


A. 6 hours
B. 12-24 hours
C. 3-5 days
D. 10 days
E. 1-2 hours

155. Positive test (small test) with dexamethasone


indicates that
A. Ovarian source of hyperandrogenism
B. Adrenal gland source of hyperandrogenism
C. Hyperandrogenism associated with pituitary adenoma
D. Hyperandrogenism due to adrenal corticosteroma
E. Hyperandrogenism due to ovarian cystoma

156. The therapeutic and diagnostic effect of dexamethasone


in hyperandrogenism is due to:
A. Inhibition of ovarian function
B. Inhibition of adrenal function
C. Inhibition of ACTH
D. Acceleration of androgen inactivation
E. Increased prolactin

157. The cause of infertility in gonadal dysgenesis is the


absence of
A. Ovarian tissue or its sharp underdevelopment
B. Uterus or its severe underdevelopment
C. gonadotropin production
D. Sensitivity of the normally formed ovary receptor
apparatus to gonadotropins
E. vaginal atresia

158. Ovarian hyperstimulation may occur.


A. When using clomiphene (clostilbegit)
B. When using pergonal
C. With prolonged use of combined estrogen-progestogen
drugs
D. When using radon baths
E. When using progestogens.
159. A woman asked you to pick a contraceptive. Single.
She lives sexually 2-3 times a month, and sometimes less
often. The partners are different. There were no pregnancies.
Your recommendations:
A. Intrauterine contraceptive
B. Condom
C. Emergency contraception
D. Oral contraceptives
E. Surgical sterilization

160. A married woman of 28 years, having one sexual


partner, suffering from chronic thrombophlebitis of the
veins of the lower extremities, the mother of one child,
needs:
A. Oral contraceptives
B. Surgical sterilization
C. Intrauterine contraception
D. Mechanical contraception
E. Barrier contraceptives

161. The composition of injectable contraceptives includes:


A. Long-acting progestogens
B. Conjugated Estrogens
C. Microdoses of progestogens
D. Antiandrogens
E. Mixed doses of hormones

162. For the purpose of contraception, the use of combined


estrogen-progestogen drugs begins:
A. During ovulation
B. On the eve of menstruation
C. From the 1st day of the menstrual cycle
D. Regardless of the day of the menstrual cycle
E. On the 7th day of the menstrual cycle

163. Combined estrogen-progestogen drugs are


contraindicated in:
A. At the age of 18 to 28 years
B. For the purpose of contraception
C. During breastfeeding and during pregnancy
D. For the purpose of treatment
E. At the age of 28-38 years

164. The Pearl Index is:


A. Percentage of contraceptive failure when using the
method during the year;
B. The ratio of heart rate and systolic blood pressure;
C. The ratio of heart rate and diastolic blood pressure.
D. The percentage of contraceptive failures when using the
method for 6 months.
E. The percentage of contraceptive failures when using the
method for 3 months.

165. What method of contraception refers to the natural


method of family planning:
A. Calendar;
B. Barrier;
C. Surgical;
D. Chemical;
E. Hormonal.

166. The optimal time for the introduction of the intrauterine


device (IUD):
A. 4-6 day of the menstrual cycle;
B. 14-16 day of the menstrual cycle;
C. During menstrual bleeding.
D. Before menstruation
E. 18-20 day of the menstrual cycle.

167. The composition of injectable contraceptives includes


A. Long-acting progestogens
B. Conjugated Estrogens
C. Microdoses of progestogens
D. Antiandrogens
E. Antigonadotropins

168. In the differential diagnosis of tubal pregnancy


A. History is usually not significant
B. Detection by histological examination of the
endometrium
The Arias-Stella phenomenon is indisputable evidence
C. A positive serological response to pregnancy is a reliable
sign
D. Leading role belongs to laparoscopy and ultrasound
E. Under any conditions, puncture of the posterior arch is
crucial
169. What are the frequent complications of taking
progestogens?
A. Allergic reactions
B. Intermenstrual spotting
C. Nausea, vomiting
D. Weight gain
E. Oligoamenorrhea

170. Contraindications to taking combined oral


contraceptives
A. Hypertension III stage
B. Women with miscarriage
C. Women with a history of ectopic pregnancy
D. Women with mild anemia
E. Algodismenorea

171. The diagnosis of impaired tubal pregnancy is an


indication
A. for emergency laparotomy
B. For laparoscopy
C. For ultrasound examination
D. For special studies
E. For puncture of the back arch

172. Voluntary sterilization - a mechanism of action.


A. thickening of cervical mucus
B. Blocking the patency of the fallopian tubes
C. Suppression of ovulation
D. Decreased uterine tone
E. Promotes adhesions in the pelvis.

173. Advantages of tubal occlusion


A. Irreversibility of the method
B. Protection against STIs
C. Regulation of the menstrual cycle
D. Increased libido
E. Thickening of cervical mucus

174. Mechanism of action for vasectomy


A. Blocking the patency of the vas deferens
B. Change in hormone production by the ovaries
C. Spermicidal effect
D. Decreased libido
E. Decreased vascular tone

175. Benefits of Condoms


A. Protection against STIs
B. High Efficiency
C. Influencing sexual feelings
D. Spermicidal effect
E. Contributes to thickening

176. Disadvantages of condoms


A. Protection against STIs
B. Lack of systemic side effects
C. Wide availability
D. Contraceptive effectiveness depends on the
willingness of the couple to follow the instructions.
E. High price

177. The mechanism of action of spermicides


A. Suppression of ovulation
B. Destruction of sperm membranes, which reduces their
motility and ability to fertilize the egg
C. Thickening of cervical mucus
D. Disruption of egg implantation
E. Toning the fallopian tubes

178. The mechanism of action of the method of lactational


amenorrhea (MDA)
A. Suppression of ovulation
B. Change in the structure of the endometrium
C. Implant Impaired
D. Thickening of cervical mucus
E. Endometrial hyperplasia

179. The high efficiency of the method of lactational


amenorrhea (MDA) with all the rules observed
A. Within 3 months after delivery
B. Within 6 months after delivery
C. Within 9 months after delivery
D. Within 12 months after delivery
E. Within 15 months after delivery

180. Contraception contraceptive against STI


A. COC
B. Navy
C. Barrier
D. Calendar method
E. Emergency contraception

181. The goals of family planning are:


A. Birth of wanted children
B. Reducing the prevalence of sexually transmitted
infections
C. Correction of menstrual irregularities.
D. Overcoming infertility
E. Decrease in maternal and perinatal mortality

182. Blastocyst implantation disorders:


A. Changes in the enzymatic system of the endometrium
B. Activation of peristalsis of the fallopian tubes
C. Lack of ovulation
D. Decrease in the tone of the fallopian tubes
E. Late ovulation

183. Disadvantages of the intrauterine device


A. Increased risk of inflammatory diseases
B. Effect on lactation
C. Short validity
D. A large number of side effects
E. Systemic effects on the body

184. Indications for use of the intrauterine device:


A. The need for long-term contraception and the
presence of one sexual partner
B. menstrual irregularities
C. Genital tract infections
D. Genital abnormalities
E. Adolescence

185. The complication most often arising on 3 5 days


after the introduction of the intrauterine device is:
A. Cervical insufficiency
B. Ectopic pregnancy
C. Habitual miscarriage
D. Uterine inflammation
E. Pelvic vein thrombosis

186. What are the most common complications in women


who have been using the intrauterine device for a long time
as a contraceptive?
A. Thrombophlebitis of the pelvic veins
B. Adhesive process in the pelvis
C. Inflammatory diseases of the internal genital organs
D. Cervical insufficiency
E. Amenorrhea
187. Visualization of an intrauterine contraceptive in the
cervical canal indicates:
A. Normal position of the intrauterine device
B. Low position of the intrauterine device
C. Uterine perforation
D. Intrauterine device expulsion
E. Pregnancy

188. The ovary is maintained in the abdominal cavity due to:


A. round ligament;
B. cardinal ligament;
C. funnel-pelvic ligament;
D. Sacro-uterine ligament
E. Broad ligament
189. Blood supply to the ovaries is carried out:
A. uterine artery;
B. ovarian artery;
C. ileo-lumbar artery;
D. Internal genital and ovarian arteries;
E. Uterine and ovarian arteries.

190. Ovarian cysts include:


A. dermoid cyst
B. follicular cyst
C. corpus luteum cysts
D. piovar
E. Theca-luteal cyst
191. Ovarian tumors that do not have hormonal activity
include:
A. granulosa cell tumor
B. dysgerminomas
C. Theca cell tumor
D. androblastoma
E. arrhenoblastoma

192. With a small bleeding from the ovary, detected by


laparoscopy, it is performed:
A. suturing the ovary;
B. diathermocoagulation of the ovary under the control
of a laparoscope;
C. Ovarian resection
D. Laparotomy and removal of the uterus on the affected
side
E. Hemostatic infusion therapy

193. The tumorous formations of the ovaries do not include:


A. dermoid cyst;
B. follicular cyst;
C. corpus luteum cysts;
D. pyovar;
E. Theca-luteal cysts.

194. What complication is most common with benign


ovarian tumors?
A. hemorrhage into the tumor cavity;
B. capsule rupture;
C. torsion of the legs of the tumor;
D. suppuration of contents;
E. Compression of adjacent organs.

195. The most informative in the differential diagnosis


between uterine myoma and ovarian tumor:
A. two-handed vaginal examination
B. hysteroscopy
C. ultrasound;
D. laparoscopy;
E. Probing of the uterine cavity.

196. The presence of ovulation can be judged by the results


of all the studies listed below:
A. Definitions of hCG
B. Ultrasound monitoring of the development of a
dominant follicle
C. Histological examination of endometrial scraping
D. Determination of the concentration of sex steroid
hormones in the blood on days 12-14 of the menstrual cycle
E. Follicle persistence

197. During surgery for a paraovarial cyst:


A. cyst husking;
B. removal of appendages on the affected side;
C. removal of the ovary on the affected side;
D. Ovarian resection on the affected side;
E. Removal of the uterus with appendages
198. Treatment of benign tumors of the ovary in childhood
and puberty is:
A. in the course of chemotherapy;
B. in the appointment of hormone therapy;
C. in bilateral removal of appendages;
D. in resection of the affected ovary;
E. in the supravaginal amputation of the uterus with
appendages;

199. An examination for an ovarian tumor may include:


A. X-ray of the gastrointestinal tract;
B. sigmoidoscopy;
C. hormonal colpocytology;
D. X-ray of the Turkish saddle;
E. phlebography;

200. The symptom complex characteristic of a granulosa


cell tumor of the ovary in menopausal women includes:
A. galactorrhea in menopause;
B. menopausal masculinization;
C. change in voice in menopause;
D. bleeding in menopause;
E. decreased libido;

201. Blood test CA-125 for endometrioid ovarian cysts:


A. elevated
B. lowered
C. normal
D. negative
E. doesn't matter

202. Prognosis for life in borderline ovarian tumors


A. Doubtful
B. adverse
C. poorly understood
D. relatively favorable
E. unknown

203. A characteristic feature of ovarian cysts:


A. increase due to the accumulation of liquid contents;
B. do not have capsules;
C. relate to malignant tumors of the female genital organs;
D. have invasive growth.
E. have fusion with neighboring organs

204. Paraovarial cysts are formed from residues:


A. omphalomesenteric duct;
B. paramesonephral duct;
C. Wolf duct
D. Muller duct;
E. allantois.

205. Each woman with a detected ovarian tumor should:


A. hospitalize urgently;
B. direct to separate diagnostic curettage;
C. put on dispensary records;
D. conduct hormone therapy;
E. carry out anti-inflammatory therapy.

206. Cervical pregnancy


A. Usually interrupted for 4-5 weeks
B. Interruption usually accompanied by internal bleeding
C. Diagnosed only during curettage of the uterine cavity
D. Can be diagnosed by the location of the external
pharynx of the cervical canal
E. In most cases, it is treated with vacuum aspiration of the
ovum.

207. Diagnostic methods most often used in the initial


detection of benign genital tumors in women
A. Cytological examination
B. gynecological examination, ultrasound
C. Ultrasound
D. pneumopelviography
E. Tumor puncture

208. Follicular cysts are most common:


A. in reproductive age;
B. in the neonatal period;
C. postmenopausal women;
D. in the juvenile period;
E. in the perimenopausal period.

209. A sign of the transition of the physiological process of


follicular maturation into a pathological follicular cyst is the
diameter of the fluid formation more:
A. 15 mm;
B. 20 mm;
C. 30 mm;
D. 35 mm;
E. 40 mm.

210. For tumor-like formations of the ovaries, it is


characteristic:
A. proliferation of the epithelium of the cellular elements of
the wall
B. lack of proliferation of the epithelium of the cellular
elements of the wall
C. consist of parenchyma and stroma
D. formed from germ cells
E. susceptible to metaplasia and paraplasia.

211. The most common ovarian tumors are:


A. cystadenomas;
B. endometrioid tumors;
C. Brenner's tumor;
D. granulosa cell tumor;
E. Mature teratoma.

212. Treatment of mucinous cystadenoma in reproductive


age:
A. course of anti-inflammatory therapy;
B. ovarian resection;
C. adnexectomy on the side of the affected ovary;
D. hysterectomy with affected appendages;
E. Combined oral contraceptives for 3–6 months.
213. The contents of the dermoid cyst:
A. hair, fat, cartilage;
B. mucus-like secret;
C. liquid, clear contents;
D. connective tissue;
E. blood.
214. Masculinizing tumor:
A. tecom;
B. androblastoma;
C. serous cystoma;
D. papillary.
E. fibroma
215. Endometriosis of the vaginal part of the cervix
A. refers to internal endometriosis
B. manifested by intense pain before and during
menstruation
C. rarely manifested as a violation of the nature of
menstruation
D. diagnosed by colposcopy
E. Good for conservative therapy

216. The vaginal part of the cervix in a woman of


reproductive age is normally covered:
A. cylindrical epithelium
B. stratified squamous epithelium
C. glandular epithelium
D. stratified squamous non-keratinizing epithelium
E. cubic epithelium
217. In the puberty, the vaginal part of the cervix has, as a
rule, the following features of the epithelial cover
A. covered with cylindrical epithelium
B. the junction of the stratified squamous and cylindrical
epithelium is located on the surface of the ectocervix, is
covered by a stratified squamous epithelium
C. the junction of the stratified squamous and cylindrical
epithelium is located on the surface of the endocervix, is
covered with stratified squamous epithelium
D. covered with stratified squamous epithelium
E. the junction of the stratified squamous and cylindrical
epithelium is located on the surface of the ectocervix,
covered with a cylindrical epithelium

218. Clinical manifestations of endometrial hyperplasia:


abnormal uterine bleeding;
A. asthenovegetative syndrome;
B. amenorrhea;
C. pain in the lower abdomen;
D. intoxication syndrome.
E. NMC

219. Differential diagnosis of endometrial polyp must be


performed with:
A. submucous uterine myoma;
B. subserous uterine myoma;
C. interstitial uterine myoma;
D. adenomyosis;
E. chorionic carcinoma.
220.. Side effects of agonists of gonadotropic releasing
hormones that limit the duration of use in the reproductive
period are:
A. osteoporosis;
B. impaired metabolism of fats and carbohydrates;
C. hyperthyroidism;
D. dyspepsia;
E. diarrhea.

221. The scope of the examination for pathology of the


cervix
A. colposcopy
B. hysteroscopy
C. Ultrasound
D. bacterioscopy
E. laparoscopy

222. Kraurosis and vulvar leukoplakia occur in women aged


A. 31–40 years old
B. 41–50 years old
C. 51-60 years
D. 61–70 years old
E. regardless of age

223. Uterine prolapse:


A. the bottom of the uterus is at the level of the plane of
entry into the pelvis;
B. the body of the uterus outside the genital gap, cysto and
rectocele;
C. The internal uterine pharynx is below the interspinal
line, the prolapse of the walls of the vagina;
D. the cervix is elongated, determined outside the genital
gap, cysto and rectocele;
E. The internal uterine pharynx is located above or at the
level of the interspinal line, the prolapse of the walls of the
vagina I degree.
224. Morphological changes with vulvar kraurosis are more
pronounced
A. in the epithelium
B. in the vessels of the vulva
C. Throughout the vulva
D. in the connective tissue of the vulva
E. in the basement membrane
225. For kraurosis of the vulva is not characteristic:
A. Puckering of the labia minora and labia minora
clitoral itching
B. dryness of the mucous membranes of the vulva
C. narrowing of the vaginal opening
D. vulvar edema

226. Hyperkeratosis of the epithelium of the vaginal part of


the cervix is ...
A. erythroplakia;
B. leukoplakia;
C. ectropion;
D. pseudo-erosion;
E. intraepithelial neoplasia.

227. The precancerous conditions of the cervix include:


A. erythroplakia;
B. leukoplakia without atypia;
C. ectropion;
D. pseudo-erosion;
E. Dysplasia of the stratified squamous epithelium.

228. What corrective hormone therapy is carried out with


DMK of the reproductive period:
A. Estrogens in the 1st phase of chicoa
B. Estrogen-progestogen drugs in a contraceptive
regimen;
C. Estrogen-progestogen drugs in the 1st phase of the cycle;
D. Estrogens in phase 2;
E. Gestagens continuously.

229. The main etiological factor for dysplasia and cervical


cancer is:
A. herpes simplex virus type 2;
B. human papillomavirus;
C. hyperestrogenia;
D. violation of the pH of the vaginal secretion;
E. immune and metabolic disorders in the body.

230. When conducting extended colposcopy after treatment


with a 3% solution of cervical acetic acid, the following
epithelial reaction is normal:
A. does not change;
B. turns pale;
C. evenly stained dark brown;
D. coated with white coating;
E. becomes embossed, papillae are visible in the form of
“grapes”.

231. A qualitative reaction (Schiller test) is due to the


interaction of iodine with cervical epithelium contained in
the multilayer:
A. glycogen;
B. proteins;
C. fats;
D. Ca salts;
E. immunoglobulins.

232. Incomplete prolapse of the uterus:


A. the bottom of the uterus is at the level of the plane of
entry into the pelvis;
B. the body of the uterus outside the genital gap, cysto and
rectocele;
C. The internal uterine pharynx is below the interspinal line,
the prolapse of the walls of the vagina;
D. the cervix is elongated, determined outside the genital
gap, cysto and rectocele;
E. The internal uterine pharynx is located above or at the
level of the interspinal line, the prolapse of the walls of the
vagina I degree.

233. Complete prolapse of the uterus:


A. the bottom of the uterus is at the level of the plane of
entry into the pelvis;
B. the body of the uterus outside the genital gap, cysto
and rectocele;
C. The internal uterine pharynx is below the interspinal line,
the prolapse of the walls of the vagina;
D. the cervix is elongated, determined outside the genital
gap, cysto and rectocele;
E. The internal uterine pharynx is located above or at the
level of the interspinal line, the prolapse of the walls of the
vagina I degree.

234. What are the leading clinical symptoms of dysplasia


and cancer of the cervical insitu:
A. pelvic pain;
B. mucopurulent leucorrhoea;
C. contact bleeding;
D. Acyclic uterine bleeding;
E. infertility.

235. In the etiology of pelvic organ prolapse, it does not


matter: Answer options:
A. traumatic birth;
B. estrogen deficiency;
C. the presence of tumors of the pelvic organs;
D. connective tissue dysplasia;
E. heavy physical labor.

236. For the treatment of cervical dysplasia of moderate and


severe degree do not apply:
A. local destruction of the cervical epithelium with acid
solutions;
B. radiosurgical excision;
C. electroconization;
D. Cone amputation of the cervix according to Sturmdorf;
E. Uterus extirpation.

237. Indicate the precancerous changes in the vaginal part of


the cervix:
A. recurrent cervical canal polyp
B. true cervical erosion
C. Cervical dysplasia
D. cervical ectropion
E. Cervical ectopy

238. To prevent the development of cervical endometriosis,


organ cryodestruction is performed on the following days of
the menstrual cycle
A. 1-2 days before the start of menstruation
B. immediately after the end of menstruation
C. on day 12-14
D. on day 16-18
E. on day 20-24

239. Itching of the vulva as a result of neuroendocrine


disorders is characteristic of:
A. candidiasis
B. trichomoniasis
C. krauroza
D. leukoplakia
E. erythroplakia
240. In the surgical treatment of a patient with kraurosis and
vulvar leukoplakia, the following is performed:
A. ovariectomy;
B. vulvectomy;
C. extended vulvectomy;
D. lymphadenectomy;
E. Uterus

241. Women with DMK are at risk:


A. Miscarriage
B. On the development of placental insufficiency
C. Development of genital tumors
D. On the development of anomalies of labor
E. Pregnancy

242. Hyperprolactinemia is:


A. Increase in blood gonadotropins
B. Increase in hemoglobin
C. Increase in BCC
D. Increased blood prolactin
E. Decreased blood prolactin levels

243. Cytological cervical screening is performed at the age


of:
A. 30 to 69 years old
B. 40 to 69 years old
C. 21 to 69 years old
D. from the age of menarche to 69 years
E. from the onset of sexual activity (after 21 years) to 69
years

244.. The earliest symptom of genital prolapse is:


A. stress urinary incontinence
B. recurrent vaginitis;
C. gaping genital fissure;
D. frequent urination;
E. foreign body sensation in the perineum.

245. To improve the effect of surgical treatment of genital


prolapse in elderly patients, the preoperative preparation
includes:
A. course of antibiotic therapy;
B. indirect anticoagulants;
C. topical estrogen preparations
D. a-GnRH for 3 months;
E. immunomodulators.

246. Indicate the most effective method for the early


diagnosis of postmenopausal osteoporosis:
A. Radiography of the lumbosacral spine
B. Mono- and biphoton absorptiometry
C. Computed tomography
D. Radiography of cysts
E. Joint ultrasound
247. Indications for the use of sounding of the uterine
cavity:
A. A suspected uterine perforation during curettage;
B. B acute endometritis;
C. C suspected presence of submucous uterine fibroids;
D. D cervical cancer;
E. E ectopic pregnancy;
248. In what cases, for diagnostic purposes, puncture of the
abdominal cavity through the posterior arch is indicated?
A suspicion of an ectopic pregnancy;
B suspected ovarian cancer;
C dysfunctional uterine bleeding;
D uterine fibroids;
E suspected ovarian apoplexy;

249. Research methods of the anatomical and functional


state of the vagina:
A inspection in the mirrors;
B combined vaginal rectal examination;
C cytological examination of the contents of the uterine
cavity;
D puncture of the posterior fornix;
E determination of the purity of the vaginal contents.

250. The most effective treatment for postmenopausal


osteoporosis is:
A. Diet Therapy
B. Physiotherapy and exercise therapy
C. Hormone therapy
D. Vitamin Therapy
E. Antibiotic therapy

251. The treatment of bartholinitis is:


A. Physiotherapy
B. Radio waves
C. Hormones
D. Vitamin Therapy
E. Antibiotic therapy

252. Treatment of false (ovarian) premature sexual


development:
A. Vitamin Therapy
B. Surgical
C. Hormonal
D. Antibiotic therapy
E. Physiotherapy
253. Uterine prolapse:
A. the bottom of the uterus is at the level of the plane of
entry into the pelvis;
B. the body of the uterus outside the genital gap, cysto and
rectocele;
C. The internal uterine pharynx is below the interspinal
line, the prolapse of the walls of the vagina;
D. the cervix is elongated, determined outside the genital
gap, cysto and rectocele;
E. The internal uterine pharynx is located above or at the
level of the interspinal line, the prolapse of the walls of the
vagina I degree.

254. Incomplete prolapse of the uterus:


A. the bottom of the uterus is at the level of the plane of
entry into the pelvis;
B. the body of the uterus outside the genital gap, cysto and
rectocele;
C. The internal uterine pharynx is below the interspinal line,
the prolapse of the walls of the vagina;
D. the cervix is elongated, determined outside the genital
gap, cysto and rectocele;
E. The internal uterine pharynx is located above or at the
level of the interspinal line, the prolapse of the walls of the
vagina I degree.

255. Complete prolapse of the uterus:


A. the bottom of the uterus is at the level of the plane of
entry into the pelvis;
B. the body of the uterus outside the genital gap, cysto
and rectocele;
C. The internal uterine pharynx is below the interspinal line,
the prolapse of the walls of the vagina;
D. the cervix is elongated, determined outside the genital
gap, cysto and rectocele;
E. The internal uterine pharynx is located above or at the
level of the interspinal line, the prolapse of the walls of the
vagina I degree.

256. The horizontal displacement of the uterus in the pelvic


cavity does not apply:
A. anterior displacement;
B. posterior displacement;
C. Turn of the uterus;
D. offsets to the left;
E. Offset to the right.
257. The prolapse of the walls of the vagina is accompanied
by the prolapse of the bladder and the anterior wall of the
rectum. What is the clinical degree of prolapse of the walls
of the vagina, uterus and their prolapse?
A. I;
B. II;
C. III;
D. IV;
E. V.
258. The genital gap is gaping, the anterior and posterior
walls of the vagina are slightly lowered. What is the clinical
degree of prolapse of the walls of the vagina, uterus and
their prolapse?
A. I;
B. II;
C. III;
D. IV;
E. V.

259. What complaint does the patient not have when


lowering the internal genital organs:
A. difficulty in emptying the bladder;
B. stress urinary incontinence;
C. difficulty with bowel movements;
D. a feeling that something is falling out;
E. on the mucous membranes from the vagina.

260. Squeezing the entrance to the vagina carries out:


A. transverse superficial muscle of the perineum;
B. muscle lifting the anus;
C. sciatic-cavernous muscle;
D. bulbous-cavernous muscle.
E. transverse deep muscle of the perineum

261. Polycystic ovary syndrome is:


A. Androgenic disease
B. Profuse uterine bleeding
C. Heterogeneous disease
D. Cyclic uterine bleeding
E. Lack of menarche

262. In the etiology of pelvic organ prolapse do not matter:


A. traumatic birth;
B. estrogen deficiency;
C. the presence of tumors of the pelvic organs;
D. connective tissue dysplasia;
E. heavy physical labor.
263. The earliest symptom of genital prolapse is:
A. stress urinary incontinence;
B. recurrent vaginitis;
C. gaping genital fissure;
D. frequent urination;
E. foreign body sensation in the perineum.

264. The most common cause of bleeding from the genital


tract in postmenopausal women is:
A. Cervical Cancer
B. Endometrial cancer
C. Submucous uterine fibroids
D. Ovarian Cancer
E. Cervical dysplasia

265. The main method for the diagnosis of dysplasia and


pre-invasive cancer of the vulva is:
A. Biopsy followed by histological examination
B. Vulvoscopy
C. Radioisotope study
D. Cytological examination of smear prints
E. Colposcopy
266. The most informative method for the diagnosis of
genital prolapse is:
A. ultrasound;
B. gynecological examination;
C. sigmoidoscopy;
D. cystoscopy;
E. hysteroscopy.
267. For the surgical treatment of genital prolapse, do not
use:
A. Manchester operation;
B. vaginal extirpation of the uterus;
C. use of mesh implants;
D. supravaginal amputation of the uterus;
E. shortening and strengthening of the ligamentous
apparatus of the uterus.
268. The choice of method for the correction of genital
prolapse does not depend on:
A. age of the patient;
B. severity of prolapse;
C. history of childbirth;
D. reproductive plans of the patient;
E. sexual activity.
269. Correction of asymptomatic genital prolapse of 1–2
degrees in a 38-year-old patient is carried out:
A. Pelvic floor muscle training
B. surgical method - vaginal extirpation of the uterus;
C. staging of the mesh implant;
D. staging of the urogynecological pessary;
E. Surgical method - over vaginal amputation of the uterus;

270. The main treatment for cancer and vulvar melanoma:


A. Remote radiation therapy
B. Chemotherapy
C. Hormone therapy
D. Surgical
E. Radio waves
271. The most effective screening test for early diagnosis of
cervical cancer:
A. Simple colposcopy
B. Bimanual rectovaginal examination
C. Cytological examination of smears from the surface of
the cervix and cervical canal
D. Vacuum curettage of the cervical canal
E. Pelvic ultrasound

272. The most informative method for the diagnosis of


cervical dysplasia:
A. Advanced colposcopy
B. Histological examination of the cervical biopsy
C. Cytological examination of smears from the surface of
the vaginal part of the cervix
D. Vacuum curettage of the cervical canal
E. Pelvic ultrasound
272. Reconstructive plastic surgery using a mesh implant for
pelvic organ prolapse is not performed when:
A. senile age of the patient;
B. recurrence of prolapse;
C. post hysterectomy prolapse (enterocele);
D. severe connective tissue dysplasia;
E. Hereditary factor of genital prolapse.

273. The most informative method for the diagnosis of


cervical dysplasia:
A. Advanced colposcopy
B. Histological examination of the cervical biopsy
C. Cytological examination of smears from the surface of
the vaginal part of the cervix
D. Vacuum curettage of the cervical canal
E. Pelvic ultrasound

274. To improve the effect of surgical treatment of genital


prolapse in elderly patients, the preoperative preparation
includes:
A. course of antibiotic therapy;
B. indirect anticoagulants;
C. topical estrogen preparations
D. a-GnRH for 3 months;
E. immunomodulators.
275. Before giving a woman an injection of Depo-Provera,
the health care provider must make sure that she does not
have:
A. Vaginal bleeding of unknown etiology
B. Cardiovascular Disease
C. History of STIs
D. Thromboembolic disorders
E. High blood pressure
276. What contraceptive methods are not recommended for
breastfeeding women:
A. Combined oral contraceptives
B. Condoms and spermicides
C. Navy
D. MLA
E. Purely progestin contraceptives

277. What method is not recommended for a 40-year-old


smoking woman:
A. Pure-progestin oral contraceptives
B. Depot-Provera
C. Combined oral contraceptives
D. Voluntary surgical sterilization
E. Navy

278. What should be done when spotting spotting occurs


during the installation of a copper-containing IUD:
A. Carry out a diagnostic curettage of the uterine cavity
B. Prescribe haemostatic and contractions
C. Remove IUD
D. Reassure a woman and prescribe a short course of
non-steroidal anti-inflammatory drugs
E. Refer to hospital

279. If a woman uses the method of lactational amenorrhea,


when any signals appear she needs to see a doctor to receive
recommendations on the use of another contraceptive:
A. The child is 3 months old
B. Maternal period resumes
C. Baby exclusive breastfeeding
D. Baby sleeps all night without feeding
E. 10 days of the postpartum period

280. The most common complication of taking progestin


drugs is:
A. Menstrual irregularities
B. Strokes, heart attacks
C. Weight gain
D. Violation of the blood coagulation system
E. Nausea, vomiting
281. The cytological equivalent of cervical dysplasia is:
A. Acanthosis
B. Discariosis
C. Hyperkeratosis
D. Cariolysis
E. Hypokeratosis
282. Characteristic features of uterine fibroids:
A. It is a benign, hormone-dependent tumor
B. Comes from the striated muscle of the tissue
C. Most common in puberty and women of early
reproductive age
D. Prone to malignancy
E. Malignant tumor

283. What method of contraception has the effect of double


protection
A. COC
B. Navy
C. Implants
D. Condoms
E. Depot-Provera

284. Depo-Provera is not recommended:


A. Women of all ages with more than 3 children
B. Women with migraine
C. Women with breast cancer at this time
D. Nulliparous women and adolescents
E. Women after abortion

285. Reliable information about contraceptives is provided:


A. From the personal experience of a specialist
B. Pharmaceutical companies
C. From evidence-based material
D. From the personal experience of the consumer
E. From the Internet
286. What can be done if a woman missed the next COC?
A. As soon as possible, take the missed active (hormonal)
tablet, then continue taking the tablets as usual
B. Do not refrain from sexual intercourse and do not use
additional contraceptives (condom) for the next 7 days
C. Do not resort to emergency contraception
D. Cancel birth control pills
E. Start taking pills from the next pack

287. A high risk of HIV infection or the presence of HIV


infection / AIDS is a contraindication for use:
A. Spermicides
B. Surgical sterilization methods
C. Navy
D. Combined oral contraceptives
E. Progestin-only contraceptives

288. Women with a history of deep vein thrombosis are not


recommended to use:
A. COC
B. Navy
C. Implants
D. Condoms
E. Depot-Provera

289. What qualities are not acceptable for a good consultant:


A. Competence and professionalism
B. Respect for Confidentiality
C. Use of personal preferences when choosing a method
of contraception
D. Friendliness and attentiveness
E. Listening skills
290. What method of contraception is least effective:
A. COC
B. Navy
C. Natural family planning methods
D. Condoms
E. Depot-Provera

291. The duration of the menstrual cycle when taking most


COCs is
A. 21 days
B. 28 days
C. 35 days
D. 32 days
E. 40 days

292. The most informative method for the diagnosis of


intermuscular uterine fibroids:
A. Vaginal examination
B. Ultrasound examination
C. Hysterosalpingography
D. Hysteroscopy
E. Colposcopy

293. One of the disadvantages of spermicides is:


A. Possible burning or itching in the vagina when used
B. The presence of estrogen-related side effects
C. Effect on breastfeeding
D. Need for pelvic examination before use
E. Progesterone-related side effects

294. The introduction of IUD as a postcoital method is


effective if, after unprotected intercourse, it has passed to:
A. 2days
B. 5days
C. 1week
D. 2 weeks
E. 1 month

295. Using Postinor as a post-coital method is effective if,


after unprotected intercourse, it has passed to:
A. 24 hours
B. 72 -120 hours
C. 10 days
D. 15 days
E. 1 month
296. Interrupted intercourse:
A. Highly effective contraceptive method
B. Ineffective contraception
C. STI Prevention Method
D. HIV Prevention Method
E. Ideal for continuous contraception
297. What conditions of the endometrium are classified as
precancerous?
A. Glandular cystic hyperplasia
B. Glandular endometrial polyp
C. Atrophy of the endometrium
D. Atypical hyperplasia
E. Fibrous endometrial polyp

298. Characteristic features of ovarian cysts:


A. These are retention formations
B. Increase in size due to cell proliferation
C. Do not have capsules
D. May be malignant
E. Multi-chamber

299. Which of the ovarian tumors most frequently


undergoes malignancy?
A. Fibroma
B. Mucinous cystadenoma
C. Serous cystadenoma
D. Tekoma
E. Sarcoma

300. What method of contraception is irreversible?


A. DMPA
B. Navy
C. Implants
D. COC
E. DHS
301. What can be used as emergency contraception:
A. Combined oral contraceptives
B. Interrupted intercourse
C. Depot Check
D. Voluntary surgical sterilization
E. Emergency contraception

302. The most appropriate time for a contraceptive after an


abortion:
A. Immediately after the procedure
B. The next menstrual cycle
C. 3 months after abortion
D. 6 months after abortion
E. Do not prescribe at all

303. The absolute indication for surgery in gynecology is


not:
A. intra-abdominal bleeding;
B. peritonitis;
C. gonorrheal pelvioperitonitis;
D. threat of perforation of the tubo-ovarian abscess.
E. Beginning miscarriage

304. An ectopic pregnancy can be localized in all of the


organs listed below, except:
A. vagina;
B. cervix;
C. rudimentary uterine horn;
D. ovary;
E. abdominal cavity.

305. The methods for diagnosing endometrial cancer are as


follows, except:
A. metrosalpingography;
B. separate diagnostic curettage of the uterine mucosa and
cervix;
C. functional diagnostic tests;
D. ultrasound examination of the pelvic organs;
E. hysteroscopy.
307. What is the most common morphological characteristic
of the endometrium prior to adenocarcinoma?
A. secretory transformation;
B. adenomatosis;
C. hyperplasia;
D. proliferation;
E. resting endometrium.

308. The main symptoms of adenocarcinoma, except:


A. uterine bleeding in postmenopausal women;
B. pain in the lower abdomen and in the lumbar region;
C. infiltration of the pelvic tissue;
D. decrease in the size of the body of the uterus;
E. enlargement and compaction of regional lymph nodes.

309. Additional methods for diagnosing the degree of spread


of adenocarcinoma, except:
A. cystoscopy;
B. bacteriological examination of uterine aspirate;
C. rectoscopy;
D. radiopaque lymphography;
E. Radioisotope lymphography.

310. The age period during which in situ carcinoma is most


often detected?
A. 45-55 years old;
B. 7-17 years old;
C. 30-40 years;
D. older than 60 -70 years;
E. older than 70 years.

311. What is the main treatment for chorionepithelioma?


A. antibiotic therapy;
B. immunostimulating therapy;
C. hysterectomy with appendages;
D. cytostatic therapy;
E. physiotherapeutic treatment.

312. Methods for diagnosing cervical pathology are used,


except:
A. Pap smear;
B. colposcopy;
C. cervicogysterosalpingography;
D. measuring rectal temperature;
E. Curettage of the cervical canal.

313. Factors contributing to the development of cervical


cancer, except:
A. hormonal contraception;
B. cicatricial deformity of the cervix after childbirth;
C. ectropion;
D. long-existing cervical erosion;
E. Cervical leukoplakia.

314. Regional metastasis of cervical cancer to the lymph


nodes, except:
A. external iliac;
B. common iliac;
C. Presacral;
D. paraaortic;
E. inguinal.

315. Treatment of preinvasive carcinoma of the cervix in a


young woman, except:
A. laser conization of the cervix;
B. hysterectomy with appendages;
C. intracavitary gamma therapy;
D. electroconization of the cervix;
E. knife conical excision of the cervix.
316. What research allows to establish the diagnosis of
invasive carcinoma of the cervix?
A. Pap smear;
B. aspiration of cervical mucus;
C. targeted biopsy of the neck with histological
examination;
D. flushing from the vagina;
E. colposcopy.
317. The most common cause of endometrial hyperplasia in
reproductive age?
A. inflammatory diseases of the genitals;
B. prolonged hyperestrogenism with anovulation;
C. hyperestrogenia in case of insufficiency of the luteal
phase of the menstrual cycle;
D. estrogen producing ovarian tumor;
E. Long-term use of estrogen.

318. Endometrial hyperplasia develops as a result of:


A. hyperprogesteronemia;
B. hyperestrogenemia;
C. hyperprolactinemia;
D. use of combined estrogen-progestogen drugs;
E. Genetically determined proliferation of basal endometrial
cells.

320. The histological differential diagnosis between in situ


carcinoma and invasive carcinoma is based on the following
symptoms:
A. damage to the basement membrane by atypical cells;
B. atypical cells are detected in a smear for oncocytology;
C. detection of cells with squamous metaplasia;
D. the entire layer of stratified squamous epithelium is
replaced by atypical cells;
E. pronounced proliferation of basal cells of the stratified
squamous epithelium.

321. With endometrial cancer, the lymph nodes are


primarily affected:
A. paracervical;
B. obstructive;
C. inguinal;
D. sacred;
E. hypogastric.

322. Treatment for stage II endometrial cancer includes:


A. only exposure;
B. extirpation of the uterus with appendages;
C. a combination of irradiation and hysterectomy with
appendages;
D. radical hysterectomy with resection of the greater
omentum;
E. only symptomatic therapy.

323. Uterine sarcoma can occur from the following tissues,


except:
A. myometrium;
B. nerve fibers;
C. endometrium;
D. blood vessels;
E. fibrous node.
324. The primary treatment for stage II ovarian cancer is:
A. immunotherapy;
B. remote exposure;
C. chemotherapy;
D. hormone therapy;
E. surgical method.
325. In a barren woman of reproductive age, histological
examination of the endometrium revealed adenomatosis.
Which drug will be more effective?
A. clostilbegit;
B. non-ovlon;
C. norkolut;
D. Pergon;
E. Chorionic gonadotropin.

328. The most appropriate sequence of measures in the


diagnosis of impaired ectopic pregnancy:
A. surgery, blood transfusion;
B. consultation of the therapist, anesthetist, surgery;
C. ultrasound, blood transfusion, surgery;
D. blood transfusion, surgery;
E. The use of hemostatic therapy, blood transfusion,
surgery.

330. When a vaginal examination in a patient with suspected


ectopic pregnancy revealed: external pharynx ajar; scarlet
spotting from the cervical canal; the uterus is enlarged to 8
weeks of pregnancy; appendages are not determined; the
vaginal arches are free. Diagnosis:
A. tube abortion;
B. impaired uterine pregnancy;
C. ovarian apoplexy;
D. inflammatory process of the uterus;
E. endometrial hyperplasia

331. Ovarian apoplexy often occurs:


A. during ovulation;
B. into the stage of vascularization of the corpus luteum;
C. during the maturation of the graaf follicle;
D. during atresia of the follicles.
E. During menstruation

332. With significant bleeding into the abdominal cavity in a


patient with ovarian apoplexy, it is shown:
A. laparotomy, ovarian resection;
B. gluttony, removal of the ovary;
C. dynamic observation of the doctor on duty, according to
indications - blood transfusion;
D. conservative therapy: rest, cold on the lower abdomen,
restorative therapy.
E. Hormone therapy;

333. In a patient with a clinical diagnosis of ovarian


apoplexy, the indication for surgery is:
A. history of appendage inflammation;
B. history of ovarian dysfunction;
C. pain syndrome;
D. intra-abdominal bleeding.
E. amenorrhea in the anamnesis.

335. Termination of tubal pregnancy by type of tubal


abortion occurs more often during pregnancy:
A. 11-12 weeks;
B. 9-10 weeks;
C. 7-8 weeks;
D. 4-6 weeks;
E. 16-17 weeks.
342. Risk factor for ectopic pregnancy?
A. uterine hypoplasia;
B. oral contraception;
C. transferred inflammatory diseases of the genitals;
D. a history of cesarean section;
E. Insufficiency of the luteal phase of the menstrual cycle.

343. What is the most common implantation of a fetal egg


during an ectopic pregnancy?
A. in the ampulla of the fallopian tube;
B. on the peritoneum;
C. on the ovary;
D. in the isthmic section of the fallopian tube;
E. in the interstitial section of the fallopian tube.

344. The least informative sign for the differentiation of


uterine and tubal pregnancy?
A. ultrasound examination of the pelvic organs;
B. level of chorionic gonadotropin in the blood;
C. bimanual examination of the pelvic organs;
D. smears for colpocytology;
E. Uterine curettage.

347. The main cause of death during ectopic pregnancy:


A. embolism of the lungs with trophoblast elements;
B. intestinal obstruction;
C. acute renal failure;
D. intra-abdominal bleeding;
E. peritonitis.

348. The diagnosis of ectopic pregnancy denies:


A. decidual endometrial reaction;
B. absence of fetal sac on echoscopy;
C. lack of a characteristic clinical picture;
D. negative test for chorionic gonadotropin;
E. Negative puncture of the abdominal cavity through the
posterior vaginal fornix.

349. The system of specialized gynecological care for


infertility includes the steps of:
A examination of women in antenatal clinics;
B examination and treatment of a married couple;
C in-patient examination and treatment;
D periodic medical examinations.
E women counseling

351. Which of the factors does not increase the risk of


developing inflammatory diseases of the genital organs?
A. the beginning of sexual activity at the age of 15;
B. medical abortion;
C. use of oral contraceptives;
D. hysterosalpingography;
E. Use of the IUD.

352. Choriocarcinoma - malignant neoplasm:


A. Decidual sheath
B. Myometrium
C. trophoblast
D. Theca tissue
E. Endometrium
353. Complaint, not typical for inflammatory diseases of the
genital organs:
A. pain in the lower abdomen;
B. fever;
C. fetid vaginal discharge;
D. high bilirubin in the blood;
E. Accelerate ESR and increase white blood cells.

354. What microorganisms causing colpitis require


treatment for both partners?
A. Trichomonas;
B. Candida;
C. streptococci;
D. staphylococci;
E. Escherichia coli.

355. Which of the following examination methods most


reliably confirm the diagnosis of inflammation of the
appendages?
A. quantification of white blood cells;
B. Gram stain of cervical mucus smear;
C. culdocentesis;
D. laparoscopy;
E. Ultrasound of the pelvic organs.
356. What is unnecessary in establishing a diagnosis of
inflammatory disease of the pelvic organs:
A. laparoscopy;
B. ultrasound;
C. culdocentesis;
D. urinalysis according to Zimnitsky;
E. rectal studies.

357. It is not a complication of inflammatory diseases of the


pelvic organs:
A. endometriosis;
B. ectopic pregnancy;
C. adhesions in the pelvic area;
D. dyspareunia;
E. hydrosalpinx.

358. In girls at an early age (from 2 to 8 years) are more


common:
A. ovarian tumors;
B. dysfunctional bleeding;
C. Congenital malformations of the genitals
D. vulvovaginitis;
E. salpingoophoritis.

359. The complication most often arising in the conduct of


the IUD is:
A. isthmic-cervical insufficiency;
B. ectopic pregnancy;
C. habitual miscarriage;
D. acute infection;
E. Pelvic vein thrombosis.

360. Pathological changes in cervical mucus can be the


result of all of the following conditions, except:
A. infection of the cervix by cytotoxic microorganisms;
B. posterior uterine displacement;
C. chronic inflammatory process in the cervix;
D. previous electrocoagulation of any formations of the
cervix;
E. vaginal inflammation.
361. Endometritis is:
A. inflammation of the fallopian tube;
B. uterine muscle inflammation;
C. inflammation of the peritoneum;
D. inflammation of the peritoneal tissue;
E. inflammation of the uterine mucosa.

362. Treatment of acute endometritis includes:


A. physiotherapy;
B. surgical treatment;
C. antibacterial agents;
D. diuretics;
E. antispasmodics.

363. Parametritis is:


A. inflammation of the ovary;
B. inflammation of the cecum;
C. inflammation of the fallopian tube;
D. inflammation of the peritoneal tissue;
E. inflammation of the omentum.

364. What treatment is not used for acute inflammation of


the uterine appendages of a non-specific etiology?
A. cold to the lower abdomen;
B. antibiotic therapy;
C. mud therapy;
D. vitamin therapy;
E. detoxification therapy.

365. What is the most common cause of atrophic colpitis?


A. oral contraception with progestins;
B. drug amenorrhea in the treatment of uterine fibroids or
endometriosis;
C. menopause;
D. oral contraception with progestogens;
E. Uterine fibroids

366. Factors of resistance of the vaginal mucosa to


infection?
A. high levels of androgens;
B. low estrogen levels;
C. acidic environment;
D. lack of Dederlein bacteria;
E. High progesterone.

367. Treatment of acute endometritis, except:


A. desensitizing agents;
B. surgical treatment;
C. antibacterial agents
D. antioxidant complex of vitamins;
E. Antispasmodics.

368. What disease is not differentiated in acute


inflammation of the pelvic organs?
A. acute appendicitis;
B. uterine fibroids;
C. acute urinary tract infection;
D. Lower lobar pneumonia
E. Torsion of the legs of the tumor.
369. The main diagnostic method for evaluating the
effectiveness of treatment of trophoblastic disease?
A. Dynamic transvaginal ultrasound
B. Computed tomography
C. Determination of the titer of chorionic gonadotropin
in blood serum and urine in dynamics
D. Hysteroscopy with separate diagnostic curettage
E. Determination of hemoglobin level

370. What least contributes to the development of the


inflammatory process in the pelvis?
A. curettage of the uterine cavity;
B. menstruation;
C. sperm;
D. endocervicitis;
E. fibrotic changes.
371. Atrophic colpitis may develop in the following
situations, with the exception of:
A. postmenopause;
B. premature depletion of ovarian function;
C. use of oral contraceptives;
D. pituitary necrosis;
E. Surgical castration at a young age.

572. The following complaint is characteristic of the onset


of acute inflammation of the uterus:
A. fever;
B. the appearance of rashes;
C. dyspeptic disorders;
D. polyphagy;
E. vomiting.

373. An indication for surgical treatment in inflammatory


processes of the uterine appendages is:
A. pyosalpinx;
B. perforation of purulent tubo-ovarian formation;
C. frequent exacerbations of the chronic inflammatory
process of the uterus;
D. endometriosis;
E. acute salpingitis.

374. The symptom most characteristic of vulvovaginitis:


A. sharp pains;
B. burning, itching
C. high temperature;
D. ulceration;
E. spotting.

375. Select the symptoms characteristic of pelvic peritonitis:


A. the appearance of rashes;
B. intestinal paresis;
C. a positive symptom of Shchetkin in the hypogastric
region;
D. abdominal distension;
E. enterocolitis.

376. Which statement of relative anaerobic sepsis is not


true:
A. more common with criminal termination of pregnancy;
B. may result from a hospital-acquired clostridial
infection;
C. accompanied by hemolysis of red blood cells;
D. early development of oligo- and anuria due to
hemoglobinuria;
E. Blood levels of total and unconjugated bilirubin are
elevated

377. For a clinic of septic shock are uncharacteristic:


A. drop in blood pressure;
B. oligo- and anuria;
C. icteric color of the skin precedes a drop in blood
pressure;
D. hyperthermia is replaced by hypothermia;
E. progressive DIC.

378. For emergency care in septic shock, it is not applicable:


A. paracetamol;
B. corticosteroids;
C. dopamine;
D. fresh frozen plasma;
E. Broad-spectrum antibiotics.

379. The most important risk factor for endometritis after


childbirth is:
A. frequent sex life;
B. caesarean section;
C. childbirth through the natural birth canal;
D. previous urinary tract infection;
E. associated upper respiratory tract infection.

380. Select characteristic complaints for acute endometritis:


A. pain throughout the abdomen;
B. pain radiating to the lower extremities;
C. fever;
D. nausea, vomiting;
E. dyspeptic disorders;
381. Parametritis occurs more often after:
A. childbirth, abortion
B. hypothermia;
C. accidental sexual intercourse;
D. ARVI
E. otitis media.

382. Select the research methods necessary for the diagnosis


of acute salpingo-oophoritis:
A. chest x-ray;
B. fluoroscopy of the abdominal organs;
C. urinalysis;
D. hysteroscopy;
E. laparoscopy.

383. Indicate an unfavorable period for surgery in case of


chronic inflammatory processes of the uterine appendages:
A. without exacerbation;
B. during an exacerbation;
C. fever is low-grade;
D. ESR more than 20 mm / hour;
E. The temperature is normal;

384. What parts of the female reproductive system are most


often affected by tuberculosis?
A. fallopian tubes;
B. ovaries;
C. uterus;
D. external genitalia;
E. vagina.

385. The main clinical symptom of tuberculosis of the


uterus appendages:
A. chronic pelvic pain;
B. amenorrhea;
C. menometerorrhagia;
D. primary infertility;
E. secondary infertility.
386. Rising gonorrhea is a defeat:
A. cervical canal;
B. fallopian tubes;
C. paraurethral glands;
D. urethra;
E. All of the above organs.

387. Clinical data characteristic of candidiasis:


A. Profuse curdled discharge from the genital tract,
itching and burning in the genital area;
B. Vaginal discharge with an unpleasant odor, plentiful,
gray-green, yellow, foamy; dysuria, itching, burning;
hyperemia, swelling, "raspberry neck", involvement in the
process of the vulva;
C. Mucous or mucopurulent discharge from the cervical
canal, often asymptomatic treatment;
D. Vaginal discharge is liquid, with an unpleasant "fishy"
smell, homogeneous adhering to the walls of the vagina, the
absence of an inflammatory reaction, itching and burning;
E. Drawing pains in the lower abdomen and lower back.

388. The diagnosis of gonorrhea can be established based


on:
A. detection in smears of microorganisms separated from
the genital tract located in pairs in the form of diplococci;
B. Gram negative cocci;
C. the location of bacteria inside the cell;
D. detection of any of the listed symptoms;
E. When establishing a diagnosis, a combination of all
the listed symptoms is necessary.
389. What is the main clinical symptom of bacterial
vaginosis:
A. itching of the external genitalia and perineum;
B. dyspareunia;
C. profuse leucorrhoea;
D. dysuria;
E. Pelvic pain.

390. The most common cause of ectopic pregnancy is


A. Genital infantilism
B. External genital endometriosis
C. Submucous uterine fibroids
D. Chronic salpingitis
E. Adenomyosis
391. The patient has been taking antibiotics for acute
pyelonephritis for a long time. She had a burning sensation
in the vagina, itching, copious discharge. What complication
arose?
A. acute endometritis;
B. ectopic pregnancy;
C. inflammation of the uterus;
D. candidal colpitis;
E. Cervical erosion.

392. An 18-year-old woman with a delay of menstruation


for 10 days had acute pain in the lower abdomen, fever up to
37.4 ° C, leukocytosis 12.4 109 / L. On palpation, the pain
spreads to the upper abdomen on the right. Differential
diagnosis is carried out with all of the following diseases,
except:
A. ectopic pregnancy;
B. appendicitis;
C. acute salpingitis;
D. colpitis;
E. Torsion of the legs of the ovarian tumor.

393. The following do not predispose to the development of


candidal vaginosis:
A. oral contraceptives;
B. pregnancy and diabetes;
C. taking antidepressants;
D. antihypertensive drugs
E. diuretics

394. What disease should be suspected if vaginal yeast


infection often recurs?
A. anemia;
B. diabetes mellitus;
C. systemic lupus erythematosus;
D. genital endometriosis;
E. Congenital adrenal hyperplasia.

395. Bacterial vaginosis is not characterized by the


following manifestation:
A. pH 5.0;
B. key cells;
C. severe inflammatory reaction;
D. an increase in the number of gardnerella;
E. Good effect from metronidazole treatment.
396. Ectopic pregnancy should not be differentiated:
A. with salpingitis;
B. with abortion;
C. with torsion of the cystoma legs;
D. with hemorrhage in the corpus luteum;
E. with endometritis;

397. The main clinical symptoms of pipe rupture do not


include:
A. sharp paroxysmal pain in the lower abdomen;
B. short-term loss of consciousness;
C. positive phrenicus symptom;
D. pallor of the skin, cold sweat.
E. Increase in body temperature;

398. Treatment measures for abortion should include:


A. observation against antibiotic therapy;
B. laparotomy after the development of a clinic of intra-
abdominal bleeding;
C. the appointment of drugs that enhance blood
coagulability;
D. immediate laparotomy and removal of the tube;
E. hormonal hemostasis.

399. The risk factors for ectopic pregnancy are not:


A. inflammatory diseases of the pelvic organs;
B. surgical interventions on the pelvic organs;
C. Tubal ligation
D. spontaneous abortion;
E. Herpes simplex virus infections.
401. At what daily dose is heparin used to prevent
thromboembolic complications?
A. 10 thousand units;
B. 20 thousand units;
C. 30 thousand units;
D. 15 thousand units.;
E. 5 thousand units.;

402. Preovulatory changes in the hormonal background are


characterized by an increase in the level
A. LH and decreased FSH
B. FSH and decreased LH
C. FSH and LH
D. prolactin
E. FSH, LH and prolactin

403. In the first phase of the menstrual cycle


A. LH secretion increases gradually, reaching maximum
values
B. endometrial glands become sawtooth
C. decreases the number of estradiol-binding receptors
D. there is an overgrowth of granulosa cells of the follicle
E. pupil symptom becomes "negative"

405. In the second phase of the menstrual cycle


A. There is a second peak in estrogen production
associated with the flowering of the corpus luteum
B. There is a pronounced proliferation of follicle granulosis
C. Intensive proliferation of the endometrial functional layer
occurs.
D. mucus crystallization symptom reaches maximum
severity
E. Basal temperature rises by O.2 degrees

406. The maturity of the follicle can be determined


A. blood FSH level
B. blood progesterone level
C. urinary pregnanediol level
D. ultrasound examination
E. basal temperature

407. Cervical index


A. allows you to diagnose the moment of ovulation
B. is rated on a 3-point system
C. determined during colposcitological examination
D. takes into account the shape of the vaginal cervix
E. allows you to assess the level of estrogen saturation

408. Dysfunctional uterine bleeding in the reproductive


period due to
A. impaired ectopic pregnancy
B. endometriosis
C. polycystic ovary syndrome
D. inflammatory process of the endometrium
E. Disorder of the mechanism of regulation of the
menstrual cycle
409. Dysfunctional uterine bleeding with persistence of the
follicle proceeds against the background
A. High estrogen saturation
B. low estrogen saturation
C. High gestagenic saturation
D. androgenic saturation
E. glucocorticoid deficiency

410. Dysfunctional uterine bleeding in menopause due to


A. Disorder of the circhoral production of gonadotropin
releasing hormone (GnRH)
B. adenomyosis
C. adenomatosis
D. Atrophic processes in the endometrium
E. Immunodeficiency

411. Dysfunctional uterine bleeding in the juvenile period


includes bleeding
A. for blood diseases
B. with cardiovascular disease
C. with thyroid disease
D. for adrenal disease
E. Disorder of the formation of circhoral production of
gonadotropin releasing hormone

412. Dysfunctional uterine bleeding with follicular atresia


occurs against
A. High estrogen saturation
B. low estrogen saturation
C. High gestagenic saturation
D. Hyperandrogenemia
E. Prolactin overproduction

413. Ovulatory acyclic bleeding characteristic


A. with short-term rhythmic persistence of the follicle
B. with prolonged persistence of the follicle
C. with follicular atresia
D. with persistence of the corpus luteum
E. in case of corpus luteum insufficiency

414. Anovulatory cyclic bleeding characteristic


A. with prolonged follicular persistence
B. with short-term persistence of follicles
C. with persistence of the corpus luteum
D. in case of corpus luteum insufficiency
E. with follicular atresia

415. Hypoplastic endometrium is characteristic


A. for ovarian hypofunction
B. for dysfunctional uterine bleeding in menopause
C. For ectopic pregnancy
D. for endometrial precancer
E. for tecom and granulosa cell tumors of the ovary

416. The condition of the endometrium with follicular


atresia is characterized by
A. glandular hyperplasia
B. atypical glandular hyperplasia (adenomatosis)
C. High glycogen
D. Decidual changes
E. The emergence of large Arias-Stella cells

417. The condition of the endometrium with persistence of


the follicle is characterized by the presence of
A. the light glands of Overbeck
B. adenocontoms
C. adenomatosis
D. adenomyosis
E. glandular cystic hyperplasia

418. For the treatment of bleeding with persistence of the


corpus luteum, apply
A. curettage of the uterus
B. cervical electrical stimulation
C. gestagens
D. danazol
E. Parlodel

419. Indications for hysteroscopy:


A. Pregnancy
B. Suspicion of submucous uterine fibroids
C. Heavy uterine bleeding
D. Cervical stenosis
E. Common Cervical Cancer

420. The diagnosis of torsion of the cystoma legs confirms:


A. sudden abrupt onset of pain;
B. tension of the anterior abdominal wall;
C. positive symptom of Shchetkin-Blumberg;
D. detection of a sharply painful tumor in the pelvis;
E. leukocytosis, accelerated ESR.

421. Acute inflammation of the uterine appendages of


gonorrheal etiology
A. usually develops within the first week after infection
B. often complicated by pelvioperitonitis
C. is an indication for surgical treatment
D. diagnosed with hysterosalpingography
E. is an indication for the administration of ampicillin in a
course dose of 3.5 g

422. The diagnosis of gonorrhea can be made.


A. with a positive Borde-Zhangu reaction
B. if bilateral inflammation of the fallopian tubes is detected
C. In cases of concomitant inflammation of the urethra and
cervical canal
D. In cases of gonococcus detection
E. in cases of fever up to 38 degrees in response to
administration of gonovaccine

423. Endometritis is:


A. Inflammation of the vulvar mucosa
B. Inflammation of the uterine lining
C. Cervical inflammation
D. Inflammation of the fallopian tubes
E. Ovarian inflammation
424. A feature of the course of gonorrhea in women is
A. the presence of pronounced clinical symptoms with
lesions of the urethra and cervical canal
B. the presence of foamy discharge
C. bilateral lesion of the uterus
D. the lack of connection between certain stages of the
development of the disease and menstruation, childbirth,
abortion
E. Frequent development of parametritis

425. The criterion for cured gonorrhea in women is the


absence of gonococci in smears taken
A. during follow-up examinations using physiological
and combined provocations for three months
B. after three monthly nutritional provocations
C. after three series of monthly physical provocations
D. after a series of intramuscular injections of increasing
doses of gonovaccine
E. after completion of treatment

426. The rapid increase in the size of fibroids.


A. may be due to malignant degeneration of the node
B. usually associated with necrosis of the node
C. may be a symptom of endometrial cancer
D. is an indication for more active conservative therapy
E. always accompanied by pain

427. Conservative treatment of uterine fibroids


A. includes the use of diathermy
B. based on long-term use of androgens in a continuous
mode
C. can be carried out by preparations of a norsteroid
row
D. Based on long-term use of vitamin B12
E. includes chemotherapy

428. Common forms of ectopic pregnancy include


A. tubal pregnancy
B. Ovarian pregnancy
C. abdominal pregnancy
D. Interconnected pregnancy
E. Pregnancy in the rudimentary horn

429. Progressive tubal pregnancy can be reliably diagnosed


with
A. bimanual research
B. ultrasound examination
C. Curettage of the uterus
D. puncture of the posterior fornix
E. serological pregnancy response

430. In the differential diagnosis of tubal pregnancy:


A. History data are usually not significant
B. The detection of an Arias-Stella phenomenon in
histological examination of the endometrium is indisputable
evidence.
C. A positive serological response to pregnancy is a reliable
sign
D. Leading role belongs to laparoscopy and ultrasound
E. Under any conditions, puncture of the posterior fornix is
crucial
431. Inflammation of the perinatal fiber is called:
A. Peritonitis
B. Endometritis
C. Vaginitis
D. Parametritis
E. Vulvit

432. A test with clomiphene is performed if:


A. Uterine fibroids
B. Anovulation
C. Adenomyosis
D. Endometritis
E. Hyperprolactinemia

433. The diagnosis of impaired tubal pregnancy is an


indication
A. for emergency laparotomy
B. for laparoscopy
C. for ultrasound examination
D. for special studies
E. for puncture of the back arch

435. The pelvic floor is:


A. Hymen;
B. vagina;
C. Muscles and fascia of the perineum;
D. The vestibule of the vagina.
E. Douglas space
436 Cervical pregnancy
A. usually interrupted for 4-5 weeks
B. interruption usually accompanied by internal bleeding
C. Diagnosed only during curettage of the uterus
D. can be diagnosed by the location of the external
pharynx of the cervical canal
E. In most cases, can be treated with vacuum aspiration of
the ovum.

437. Cervical cancer


A. diagnosed with a specific Schiller test
B. more common in nulliparous women
C. found only in exophytic form
D. in the early stages manifested by acyclic bleeding
E. in the pre-invasive stage may be an indication for
cervical electroexcision

438. The acidic environment of the vagina is provided by


the presence of:
A. Epithelium of the vagina;
B. white blood cells;
C. The wand of Dederlein;
D. Gonococcus.
E. Kokkov

439. If pathological changes in the cervix are detected, it is


necessary:
A. Take a smear from the changed area for cytological
examination;
B. Treat the neck with a disinfectant solution;
C. Surveillance with periodic examinations.
D. Take a smear on the flora
E. PCR analysis

440. Dysplasia
A. This is a pathology of the integumentary cervical
epithelium, in the entire thickness of which there are
histological signs of cancer, but there is no invasion of the
underlying stroma
B. does not apply to background diseases of the cervix
C. may be detected by a special study of a visually
unchanged cervix
D. is an indication for hysterectomy at any age
E. is usually treated with electrocoagulation of the affected
areas of the cervix

441. Endometriosis of the vaginal part of the cervix


A. refers to internal endometriosis
B. manifested by intense pain before and during
menstruation
C. rarely manifested as a violation of the nature of
menstruation
D. diagnosed by colposcopy
E. Good for conservative therapy

442. Internal endometriosis


A. diagnosed by colposcopy
B. rare
C. develops from the basal layer of the endometrium
D. Well amenable to hormone therapy
E. not detected by hysteroscopy

443. Relates to internal genital endometriosis.


A. endometriosis of the vaginal cervix
B. peritoneal endometriosis
C. Ovarian endometriosis
D. endometriosis of the interstitial uterine tubes
E. retrocervical endometriosis

444. Retrocervical endometriosis


A. may be manifested by pain, flatulence, delayed stool
on the eve and during menstruation
B. Impossible to diagnose with bimanual examination
C. diagnosed with hysterography
D. diagnosed with pneumopelviography
E. is treated only by surgery

445. The differential criterion that determines stage II of


cancer

A. ovary (FIGO classification, 1976), is


B. ascites
C. capsule rupture
D. hydrothorax
E. damage to the second ovary
F. Damage to the fallopian tube and uterus
446. A sign defining the III stage of ovarian cancer (FIGO
classification, 1976) is
A. ascites
B. capsule rupture
C. damage to the second ovary
D. damage to the uterus
E. the presence of metastases in the omentum

447. The most important additional method of preoperative


diagnosis of the nature of the tumor process in the ovary is
A. ultrasound examination
B. Bi-Contrast Pelviography
C. pelviotomography
D. lymphography
E. Cytological examination of punctate from the
abdominal cavity

448. The treatment of benign tumors of the ovary in


childhood and puberty is
A. in the course of chemotherapy
B. in the appointment of hormone therapy
C. Bilateral removal of appendages
D. in resection of the affected ovary
E. in the supravaginal amputation of the uterus with
appendages

449. The <choice operation> in benign ovarian tumor in pre-


and postmenopausal women is
A. Ovarian resection
B. removal of the appendages from the lesion
C. Bilateral removal of appendages
D. supravaginal amputation of the uterus with
appendages
E. Subvaginal uterine amputation with appendages +
omentectomy

450. The <operation of choice> in ovarian cancer is


Wertheim operation
A. expanded hysterectomy with removal of paraaortic
lymph nodes
B. Extirpation or supravaginal amputation of the uterus
with appendages and omentectomy
C. hysterectomy with appendages
D. Removal of affected appendages
E. Subvaginal uterine amputation with appendages +
omentectomy

451. Risk factor for ectopic pregnancy?


A. uterine hypoplasia;
B. oral contraception;
C. Pelvic inflammatory disease
D. a history of cesarean section;
E. Insufficiency of the luteal phase of the menstrual cycle.

452. The leading method of complex treatment of malignant


ovarian cancer is
A. chemotherapy and surgical treatment
B. surgical treatment + radiotherapy
C. X-ray and hormone therapy
D. hormone therapy and surgical treatment
E. chemo-hormone therapy
453. Most malignant tumors of the ovary are
A. serous tumors
B. Mucinous tumors
C. Tumor stroma tumors
D. teratomas
E. metastatic tumors

454. Secondary infertility is called


A. Two-year infertility
B. Infertility in women with a history of pregnancy
C. Infertility due to endocrine pathology
D. Infertility due to inflammatory genesis
E. Husband Disease

455. The good patency of the fallopian tubes can be judged


by the data of chromohydrotubation in case urine
A. Blue in one hour
B. Green in one hour
C. Green in two hours
D. After one hour, colorless
E. Remains colorless after one hour and after 24 hours

456. According to the WHO classification (198O),


premenopause is called the period
A. From the end of reproductive age to the onset of
menopause
B. The length of time after the last menstruation
C. Last menstruation
D. Period of menstrual dysfunction until the last
menstruation
E. The Period Before the Menarche

457. According to the WHO classification (198O),


menopause is called
A. Period of stable menstrual function
B. Last menstruation
C. The length of time after the last menstruation
D. Time span after reproductive age
E. The onset of the first menstruation

458. According to the WHO classification (198O),


postmenopause is called
A. Last menstruation
B. The period from the onset of menstruation to the last
menstruation
C. Period of stable menstrual function
D. Period of first menstruation
E. The length of time after the last menstruation

459. Currently, age is considered normal for menopause.


A. 45 years
B. 5O years
C. 4O years
D. 55 years
E. 43 years
460. The autonomic-nervous manifestations of menopausal
syndrome include
A. Irritability
B. Sweating
C. Sleep Disorders
D. Decrease in memory
E. Decreased libido

461. The pathogenetic treatment of adrenogenital syndrome


(AGS) is
A. Therapy aimed at normalizing body weight
B. Improving cerebral hemodynamics
C. Glucocorticoid Therapy
D. Use of vegetotropic drugs (Belloid)
E. Use of small doses of thyroidin

462. To confirm Shereshevsky-Turner syndrome


A. Transfenoidal Arteriography
B. Determination of gonadotropin hormones
C. Ultrasound scan of the pelvic organs or
pneumopelviography
D. Hysterography
E. Karyotype study

463. Rokytansky-Mayer-Küster syndrome is an indication


A. For long-term parlodel therapy
B. For demedulation of the ovaries
C. For plastic surgery on the uterus
D. For colpopoiesis
E. For surgery on the pituitary gland

464. Sheehan syndrome


A. Develops with pituitary tumors
B. May be due to massive blood loss during childbirth
C. Characterized by hyperprolactinemia
D. Accompanied by hyperthyroidism
E. Accompanied by hypercorticism

465. When hyperprolactinemia is used


A. Diphenin
B. Pergonal
C. Clomiphene
D. Parlodel
E. Danazole

466. The most important role in the formation of prolapse


and prolapse of the walls of the vagina and uterus belongs to
traumatic injuries.
A. vaginal mucosa
B. wide uterine ligaments
C. Sacro-uterine ligaments
D. round uterine ligaments
E. Pelvic floor muscles

467. Uterine prolapse


A. not found in nulliparous women
B. may be associated with a sedentary lifestyle
C. It is considered complete if the cervix extends beyond the
genital gap
D. may be complicated by the development of decubital
ulcers
E. is an indication for Emmett's operation

468. The uterine suspension does not include


A. Own ligaments of the ovaries
B. round uterine ligaments
C. wide uterine ligaments
D. Suspending ovarian ligaments
E. Sacro-uterine ligaments

469. The most common chorionic carcinoma occurs after:


A. abortion
B. cystic drift
C. Normal birth
D. premature birth
E. inflammatory processes
470. Management tactics for patients with DMC of the
juvenile period:
A. limit yourself to symptomatic hemostatic and antianemic
therapy
B. carry out hormonal hemostasis with progesterone
C. Treatment and diagnostic curettage of the endometrium
and endocervix
D. complex therapy, including hemostatic, antianemic,
uterotonic therapy, with inefficiency - hormonal
hemostasis
E. hysteroscopy
471. The main clinical symptom of submucous uterine
fibroids:
A. chronic pelvic pain;
B. Algodismenorea
C. menorrhagia
D. leucorrhoea
E. secondary infertility

472. Conservative treatment of uterine fibroids


A. Includes diathermy
B. Based on long-term use of androgens in a continuous
mode
C. can be carried out by preparations of a norsteroid
row
D. Based on long-term use of Vitamin B12
E. includes chemotherapy

473. For anovulatory menstrual cycle is characteristic


A. cyclic changes in the body
B. prolonged persistence of the follicle
C. Prevalence of gestagens in the second phase of the cycle
D. prevalence of gestagens in the first phase of the cycle
E. estrogen deficiency

474. The following does not apply to endoscopic research


methods in gynecology:
A. hysteroscopy
B. colposcopy
C. culdocentesis
D. laparoscopy
E. Culdoscopy

475. Obstetric peritonitis most often occurs after:


A. childbirth
B. early spontaneous miscarriage
C. Cesarean section
D. Artificial abortion
E. Late spontaneous miscarriage

476. The hypothalamus produces the following hormones


A. gonadotropins
B. estrogens
C. gestagens
D. releasing factors
E. glucocorticoids

477. With exacerbation of chronic salpingoophoritis by the


type of pelvic nerve neuralgia, the effect does not have:
A. antibiotic therapy
B. electrophoresis of amidopyrine
C. Diadynamic currents
D. ultraviolet erythema
E. amplipulse therapy

478. Low doses of estrogen


A. stimulate FSH production
B. inhibit FSH production
C. enhance LH production
D. inhibit LH production
E. do not affect FSH production

479. Hysterosalpingography in the diagnosis of internal


uterine endometriosis is the most informative:
A. 1-2 days before the start of menstruation
B. immediately after menstruation
C. on day 12-14
D. on day 16-18
E. on day 20-22

480. The complication most often arising with the


introduction of the IUD is:
A. ischemic cervical insufficiency
B. ectopic pregnancy
C. perforation
D. acute infection
E. Pelvic vein thrombosis

481. In the differential diagnosis between uterine myoma


and ovarian tumor, the most informative:
A. two-handed vaginal examination
B. Ultrasound
C. Puncture of the posterior fornix.
D. laparoscopy
E. Probing of the uterine cavity

482. When examining a sterile couple, it is primarily shown:


A. hysterosalpinography
B. vaginal smear cytology
C. determination of basal temperature
D. endometrial biopsy
E. sperm examination

483. If you suspect a malignant lesion of the ovary in a


patient 55 years old, it is shown:
A. Removal of the uterus on the affected side
B. supravaginal amputation of the uterus with
appendages and resection of the greater omentum
C. hysterectomy with appendages
D. Removal of the uterus with appendages on both sides
E. Subvaginal uterine amputation with appendages

484. Tactics at the clinic of “acute abdomen” at the


prehospital stage:
A. Anesthesia
B. Cold on the stomach;
C. Cleansing enema;
D. Urgent hospitalization
E. Observation

485. When acyclic spotting appears:


A. hysterosalpingography
B. definition of PH
C. Ultrasound
D. definition of HCG
E. Diagnostic curettage

486. Amenorrhea is the absence of menstruation during


A.4 months
B.5 months
C.6 months
D.2 months
E.3 months

487. The main method for stopping abnormal uterine


bleeding in the premenopausal period is:
A. use of synthetic estrogen-progestogen drugs
B. The introduction of hemostatic and uterine-contracting
agents
C. Use of androgens
D. continuous use of 17-hydroxyprogesterone capronate
(17-OPK)
E. Separate diagnostic curettage of the mucous
membrane of the uterine cavity and cervical canal

488. Which parts of the female reproductive system are


most often affected by tuberculosis:
A. fallopian tubes
B. Ovaries
C. Uterus
D. external genitalia
E. vagina

489. The main clinical symptom of tuberculosis of the


uterus appendages:
A. chronic pelvic pain
B. amenorrhea
C. Menometerorrhagia
D. infertility
E. NMC
490. What is the main clinical symptom of bacterial
vaginosis:
A. Itching of the external genitalia and perineum
B. dyspareunia
C. Profuse discharge whitewash
D. dysuria
E. Pelvic pain

491. The main clinical symptom of submucous uterine


fibroids:
A. chronic pelvic pain
B. Algodismenorea
C. Uterine bleeding
D. secondary infertility
E. iron deficiency anemia

492. In the chain of steroid biosynthesis, the first


biologically active
hormone is
A. androstenedione
B. estradiol
C. estriol
D. testosterone
E. progesterone
493. The main estrogen hormone in a woman’s body
postmenopausal period is
A. estradiol
B. estrone
C. estriol
D. Estradiol dipropionate
E. progesterone

494. Gonadotropins, which play a role in the pathogenesis of


hyperplastic processes and endometrial cancer, are secreted:
A. adrenal glands
B. hypothalamus
C. anterior pituitary
D. posterior pituitary gland
E. Ovaries

495. At what daily dose is heparin used to prevent


thromboembolic complications?
A. 10 thousand units
B. 20 thousand units
C. 30 thousand units
D. 25 thousand units
E. 15 thousand units

496. Delayed sexual development is lack


A. secondary sexual characteristics by 14 g., and
menstruation by 16 g.
B. secondary sexual characteristics by 12 g., and
menstruation by 16 g.
C. secondary sexual characteristics by 14 g., and
menstruation by 18 g.
D. secondary sexual characteristics by 12 g., and
menstruation by 18 g.
E. secondary sexual characteristics by 13 g., and
menstruation by 15 g.

497. The most appropriate sequence of measures in the


diagnosis of impaired ectopic pregnancy:
A. surgery, blood transfusion
B. consultation of the therapist, anesthetist, surgery
C. Ultrasound, blood transfusion, surgery
D. blood transfusion, surgery
E. application of hemostatic therapy, blood transfusion,
surgery

498. In patients with amenorrhea with Itsenko-Cushing's


disease, there is a hyperproduction of the hormone
adenohypophysis
A. somatotropic
B. thyrotropic
C. adrenocorticotropic
D. follicle-stimulating
E. Luteinizing

499. The criterion of gonorrhea cured in patients is


established after treatment during
A.1 month
B.2 months
C.3 months
D.4 months
E. 5 months

500. The transferred inflammatory process of the pelvic


organs cannot be the reason:
A. tubal pregnancy
B. endometriosis
C. Adhesive process in the pelvis
D. painful intercourse
E. hydrosalpinx

END
1. Duration of menstrual cycle

A. 21-35 days

2. Phases of ovarian cycle


A. Follicular and luteal

3. preovulatory changes in the hormonal background are characterized by an:

C. FSH & LH

4. The mechanism of action of hormones on the cell is due to the presence


of:

A. Prostaglandins

B. Receptors

C. Specific Enzymes

D. isoenzymes

E. Thromboxant

5. According to the WHO classification (1980), menopause is called

A. Period Of stable menstrual function

B. Last menstruation

C. The length of after the last menstruation

Time span after reproductive age

E. The onset of the first menstruation

6. According to the WHO classification (1980), is called

A. Last menstruation

B. The period from the onset of menstruation to the last menstruation

C. Period of stable menstrual function

D. Period of first menstruation

E. The length of time after the last menstruation

10.Algodismenorea is:

A. Rare and scarce menstruation

B. Painful menstruation

C, Reduced blood loss during menstruation


D. meager spotting

E. Heavy periods

ll. Menorrhagia is:

A. Acyclic uterine bleeding

B. Cyclic uterine bleeding

C. Painful and heavy menstruation

D. pre- and postmenstrual bleeding

E. Change in the rhythm Of menstruation

12. Metrorrhagia is

A. Change in the rhythm of menstruation

B. Increased blood loss during menstruation

C. Increase in the duration Of menstruation

D. Acyclic uterine bleeding

E. Reducing blood loss during menstruation

13. For anovulatory menstrual cycle With follicular atresia is


characteristic:

A. Amenorrhea

B. Algomenorrhea

C. Oligoamenorrhea

D. polymenorrhea

E. Hypomenorrhea

14. Polymenorrhea is:

A, Lean menstmatlon

B. Short menstruation (1-2 days)

C. Short menstrual cycle

D. Heavy menstruation

E. Pmnful menstruation

15. Amenorrhea is the absence of menstruation during:

A. 4 months
B. 5months

C. 6 months

D. 12 months

E. 2 months

16. For menstrual cycle with follicular atresia is characteristic:

A. Amenorrhea

B. Algomenorrhgo

C. Oligoamenorrhea

D. Premenomenorrhea

E. Hypomenorrhea

17. In the anovulatory menstrual cycle, basal temperature is characterized


by:

A. Rise In Temperature After Ovulation

B. Lack Of Temperature Rise

C. Rise In Temperature Before Ovulation

D. Rise In Temperature Before Menstruation

E, Rise In Temperature Immediately After Menstruation

18. menstrual cycle is characterized by:

A. Cyclic changes in the body

B. Long-lasting follicle persistence

C. Prevalence Of in phase 2

D. Prevalence of estrogen in phase I

E, Acyclic change in the body

19. To induce ovulation, use:

A. Progestogens

B. Antiestrogens

C. Natural Estrogens

D Prostaglandins

E. Antagonists Of Prostaglandins

20. most effective method of preventing unwanted pregnancy is:


A. Intrauterine device

B. Britm method

C. Spermicides

D. Surgical sterilization

E. Hormonal contraception

21 For planned hormonal contraception use:

A. Pure estrogens

B. Pure gestagens

C. A-gn realizing hormones

D. Prostoglandins

E. Selective Inhibitors Of progesterone receptors

22. Intrauterine system "Mirena"

A. Contains the progestogen levonorgestrel

B. Introduced for 8 years

C. Has a therapeutic effect in submucous uterine myoma

D Does not have a contraceptive effect

E. Has a contraceptive effect by suppressing ovulation

23. the most common cause of female infertility is :

A. Psychosexual disorders

B. Tubal-peritoneal factor

C. Endocrine

D. Uterine factor

24. During what time does not pregnancy occur in the presence of regular
sexual life, without the use of contraceptives, to consider the marriage
infertile:

A. 6 months

B. 1 year

C. 2 years

D 3 years

E. 5 years

25. Name the composition of injectable contraceptives:


A. Long-acting Progestogens

B. Conjugated estrogens

C. Small doses of a progestogen

D. Antiandrogens

E. Mixed doses of hormones

26. Specify the day of menstrual cycle to start estrogen-progestin


preparations for contraception:

A. 1

B. 5

C. 7

D. 14

E. 28

27. Specify the most serious complication when taking combined oral
contraceptives:

A. Thromboembolic complications

B. Cardiovascular diseases

C. Impaired carbohydrate metabolism

D. Violation of fat metabolism

E. Violation ofvltamrn metabolism

28. Infection with what microorganisms that cau$ requires treatment of

both partners:

A. Trichomonas

B. Candida

C. Streptococci

D. Staphylococci

E. E-co1i

29. In girls at an early age (from 2 to 8 years). the following are more
common:

A. Ovarian tumors

B. Dysfunctional bleeding

C. Congenital anomalies of the genital organs

D. Vulvovaginitis
E. salpingo-oophoritis

30. Classification of pelvioperitonitis by stages, choose the correct


answer:

A. Primary, secondary

B. Reactive, terminal, toxic

C. Adhesive. exudative

D. serous purulent fibrinous

E. Adhesive. fibrinous

31. Which of the following inflammatory diseases in the acute stage is


accompanied by symptoms of irritation:

A. Endometritis

B. Colpitis

C. vulvovaginitis

D. Cervicitis

E. Bartholinitis

32. The main clinical symptom or bacterial is:

A. Itching of the external genitals and perineum

B. Dyspareunia

C. Profuse leucorrhea with an unpleasant odor

E. Bubble rashes in the genital area

33. Dunne the a stage is distinguished.

A. Desquamation

B. Infiltration

C. Proliferation

D. Tissue decomposition

E. Edematous

34. The complication that most often arises in the management of the
intrauterine device is:

A. Isthmic-cervical insufficiency

B. Ectopic pregnancy

C. Habitual miscarriage
D. Acute infection

E. Pelvic vein thrombosis

35. The symptom most characteristic of

A. Sharp pains

B. Burning, itching

C. High temperature

D. Ulceration

E. Spotting

36. The most important risk factor for endometritis after childbirth is:

A. Frequent sexual activity

B. Cesarean section

C. Childbirth through the birth canal

37. What are the diagnostic tests that indicate the presence of bacterial
vaginosis:

A. Detection of key cells

B. Lowering the ph. of vaginal secretions

C. Negative amino test of whites

D. Detection of abnormal cells

E. Increase in basal temperature

38. The anatomical features of the uterus in a newborn girl include:

A. The body of the uterus is small, the cervix is almost not expressed

B. The length or the cervix 3 times more the length of the uterus

C. The uterus has a bicorn shape

D. The body of the uterus is almost 2 times larger than the cervix

E. Uterus curved posteriorly

39. Small doses of estrogen'

A Stimulate FSH production

B. Suppress FSH production

C. Strengthen LH production

D, Suppress I.H production


E. Stimulates TTG production

40. The vaginal part of the cervix In a woman of reproductive age is


normally

A Cylindrical epithelium

B. Stratified squamous epithelium

C. Glandular epithelium

D. Stratified squamous non-keratinizing epithelium

E. Cubic epithelium

41. The main estrogen hormone in the body of a woman in the postmenopausal

period is:

A. Estradiol

B. Estron

C. Estriol

D. Estradiol dipropionate

E. Estrogen

A. priliminary consultation by a neurologist

B. Preliminary consultation of the therapist

C. The use of a minimum amount of estrogen

D. Use of ultrasound examination of the uterus

E. Hormone testing

49. Women with AUB are at risk:

A. Miscarriage

B. On the development of placental insufficiency and anomalies of the birth


force

C. On the development of genital tumors

D. On the development of breast tumors

E. Pregnancy

50. What is the mechanism of action of the amenorrhea method (LAM)

A. Suppression of ovulation
B. Changes In the structure of the endometrium

C. Violation of implantation

D. Thickening of ggrvieal mucus

E. Change movement

51. The high efficiency of the method amenorrhea (LAM), subject to all the
rules, is observed:

A_ Within 3 months after childbirth

B. Within 6 months after childbirth

C. u•ithin 9 months after childbirth

D. Within 12 months after childbirth

E. Within 15 months after giving birth

52. What is the mechanism Of action of voluntary surgical sterilization:

A. Thickening of cervical mucus

B. Blockage of the fallopian tubes

C. Suppression of ovulation

D. Violation of implantation

E. dilution of cervical mucus

53. Specify the mechanism of action of spermicides:

A. Reduces the life span of spermatozoa

B. Destroy the membranes or spermatozoa

C. thickening Of cervical mucus

D. Violation of egg Implantation

E. Dilution of cervical mucus

54. Before giving a woman an injection of Depo-provera, the health worker


must make sure that she does not have:

A. Vaginal bleeding of unclear etiology

B. Cardiovascular disease

C. History of STIs

D. Thromboembolic disorders

E. High blood pressure

55. Depo-Provera is not recommended:


A Women of any age with more than 3 children

B. Women with migraines

C. Women with breast cancer at this time

D. Unborn women and adolescents

E. Women after abortion

56. Which of the following mechanisms best explains the contraceptive


effect Of birth control pills that contain both synthetic estrogen and
progestin?

A. Direct inhibition of oocyte maturation

B. Inhibition or ovulation

C. Production of uterine secretions that are toxic to the developing embryo

D. Impairment of implantation

E. Impairment of sperm transport due to uterotubal obstruction

57. A woman has period every 33 days lasting 8 days with large clots being
passed. She could be classified as having

A. Metrorrhaggia

B. Menometrrohagia

C. Dysmenorhea

D. Menorrhagia

E. Polymenorrhea

58. Of the following. which woman is the best candidate for the female

condom"

A. 16 year Old high school student who has never had but wishes to start
with her new Is-year-old boyfriend

B. 29 year old secretary who has been married for 7 years, and is in
monogamous relationship with her husband

C. 40 year old obese smoker whose partner refuses to wear condoms

D. 34 year old woman who has three children and had a post-

partum tubal ligation

59. For conservative treatment of Abnormal uterine bleeding the 1st line
drug is:

A Oxytocin

B Nsaids
C. Tranexamic acid

D. Aminocaproic acid

E. Sodium ethamsylate

60. What is the most common presenting complaint ofa woman with vaginitis?

A Dyspareunia

B. Vaginal discharges

C. Pelvic pam

D Fever

E. Irregular menses

61 The most effective method for diagnosing gonococcus is:

A Linked immunosorbent

B. Cultural

C. Bacterioscopic

D. Immunofluorescent

62. A 23-year-old female patient complains of mucopurulent discharge from


the genital tract, itching at the antenatal clinic. By examination: the
mucous of the cervix is sharply hyperemic, edematous. the discharge is
profuse, mucopurulen. uterus In anteversion-flexio_ not enlarged, painless;
the appendages are not defined_ is the most likely diagnosis:

A Vulvovaginitis

B. Bartholinitis

C Endometritis

D. Acute salpingo-oophoritis

E. Endocervicitis.

63. For conservative treatment of Abnormal uterine bleeding the 1st line
drug is:

A. oxytocin

B. NSATDs

C. tranexamic acid

D. aminocaproic acid

E. sodium
65. A mother with a 4-year-old girl turned to a pediatric gynecologist. who
has itching and redness in the external genital area, purulent discharge
from the genital tract. These symptoms occur periodically throughout the
year. Genital hygiene IS observed. The child is observed by an allergist in
connection with atopic dermatitis (skin rashes on the elbows and on the
face). What is the most likely diagnosis:

a. salpingo-oophoritis

b. Atopic vulvovaginitis

c. Atopic

d. Endometntls

e. Endocervlcitls

66. Patient L. 24 years old. complains of profuse vaginal discharge with an


unpleasant smell of "rotten fish". Gynecological examination: the external
genital organs and vaginal mucosa without signs of inflammation. Vaginal
discharge copious. watery. with an unpleasant odor. Internal genital organs
without pathology. During of smears from the cervical canal and urethra,
"key cells" were found. What is the most likely diagnosis

A. Bacterial vaginosis

B. Candidiasis vulvovaginitis

C. Chlamydia] cervicitis

D. Atrophic colpitis

E. Bartholmltis

67. Task 2 Patient R.. 25 years old. was admitted to the gynecological
department With complaints of pain in the external genital area.
awkwardness when walking, fever. Pain appeared four days ago, after
hypothermia. Last period 3 weeks ago. By examination: in the area of the
right labia majora. a tumor-like formation measuring 4x3 cm is determined,
the skin above it is hyperemic, hot to the touch. fluctuation IS noted on
palpation. Examination of the cervix In the speculum and bimanual
examination revealed no abnormalities. What is the most probable
preliminary

A. Bartholine abscess

B. acute bilateral salpingitis, pelvioperitonitis

C. Torsion of the legs of the ovarian tumor

D. Necrosis of subserous myomatatus node

E. Perforation of purulent formation of appendages

68. A 23-year-old female patient came to the antenatal clinic with


complaints of cramps during urination, fever up to 37. S 0 C, purulent
discharge from the genital tract. She has several sexual partners.
Examination by speculum: the mucous membrane of the urethra is hyperemic
the cervix is cylindrical, hyperemia of the area of the external opening of
the cervical canal. Discharge from the cervical canal is purulent. Define
the tactics of the antenatal clinic doctor:

A. Prescribe an outpatient course of antibiotic therapy

B. Take swabs, culture for gonorrhea

C. Hospitalize the patient in a gynecological hospital

D. Get swabs for chlamydia infection

E. Hospitalization In the dermatovenerologic clinic

69, Patient S. 21 years Old. at the reception in the antenatal clinic


complains of mucopurulent discharge from the genital tract. itching.
Speculum exam: the cervix is hyperemic. edematous, with pinpoint
hemorrhages_ The discharge is profuse, mucopurulent. Vaginal examination
revealed no pathology. Was diagnosed Endocervitis

Define the tactics of the antenatal clinic doctor.

A To urgently hospitalize the patient for surgical intervention

B. Antibiotic therapy, take culture for microorganisms

C. Healthy lifestyle. diet. phytohomeopatholic treatment.

D. Refer the patient to a dermatovenerologic dispensary

E. Recommended a re-examination after the next menstruation

70. Patient V. 22 years old. consulted a antenatal clinic with complaints


of discomfort in the vagina, burning sensation. itching. leucorrhea.
Considers himself ill for S days. Examination revealed: the mucous membrane
of the vagina and cervix is sharply hyperemic, edematous. deposits easily
removed with a gauze ball, a cheesy discharge. examination normal.
diagnosed: Vulvovaginal candidiasis. Define the tactics of the antenatal
clinic doctor

A. Antifungal therapy

B. Hormonal therapy

C. Radiation therapy

D. Surgica1 therapy

E. Physiotherapy

71. A 20-year-old female patient turned to clinic, complaining of abundant.


purulent discharge from the genital tract, itching, burning and pam m the
vagina. She fell ill acutely, 2 days ago, when the above complaints
appeared. Menstrual function is not impaired. By examination: the vaginal
mucosa is sharply hyperemic, covered with a purulent bloom, bleeds easily
when touched. The cervix is cylindrical in shape, the mucosa is not
changed. is the most likely acutely. 2 days ago. when the above complaints
appeared. Menstrual function is not impaired. By examination: the vaginal
mucosa is sharply hyperemic, covered with a purulent bloom. bleeds easily
when touched. The cervix is cylindrical in shape, the mucosa is not
changed. What is the most likely diagnosis
A. Vulvovaginitis

B. Acute colpitis

C. Endometritis

D. Endocervicitis

E. Bartholinltis

72. 5-year-old woman. unmarried. has sex 2-3 times a month, and sometimes
less often. Partners are different. There were no pregnancies. What method
of contraception is appropriate to prescribe?

A. Intrauterine deuce

B. Condom

C. Combined oral contraceptives

D. Injectable contraceptives

E. Surgical sterilization

73. A young nulliparous woman, 20 years old. has an active sex life outside
of manage. has several sexual partners. What methods Of contraception can
be recommended to the patient?

A. Condom

B. Spermicides

C. Intrauterine device

D. Combined oral contraceptives

E. Surgical sterilization

74. A 25-year-old female patient consulted a gynecologist to receive


recommendations on contraception. FROM ANAMNESIS: menarche from 1 3 years
old, irregular, painful, takes to relieve Bain. Married.In obstetric
anamnesis was I birth 3 years ago. She does not plan pregnancy for the next
2-3 years, she Wants reliable contraception. Somatically healthy,
Gynecological examination and ultrasound revealed no pathology. What method
of contraception is recommended for this patient?

A. Surgical sterilization

B. Calendar rhythm meth04.

C. Intrauterine device with silver

D. Combined oral contraceptives

E. postcoital contraception

75. Patient L, 42 years old, consulted a gynecologist to receive


recommendations on contraception. suffers from hypertension for years.
Regular sex life in marriage. History of 2 births. 2 medical abortions,
General condition is satisfactory, blood pressure is 135/90 mm Hg., height
164 cm, weight 90 kg, BMI —35. Recommended method of contraception:

A. Condom

B. Calendar rhythm method

C. Intrauterine device with silver

D. Surgical sterilization

E. Postcoital contraception
Test TOP. ANATOMY
1. Sperm production begins in the

a. seminiferous tubules

b. epididymis

c. vas deferens

d. ejaculatory duct

2. The structure between the uterus and the vagina is the

a. uterine tube

b. cervix

c. vulva

d. hymen

3. Out of the following, which hormone does not secret from corpus luteum?

a) Estrogen

b) Progesterone

c) Relaxin

d) Testosterone

4. Mark the INCORRECT statement about prostate gland?

a) Located inferior to the urinary bladder

b) Secretion is thin and milky colored

c) Secretion is acidic in nature

d) Function in increasing the mobility of the sperm

5. Which of the following is an energy source for the sperm?

a) Somatostatin

b) Prostaglandin

c) Proteins

d) Fructose
OBS
1. Which one of the following fetal heart rate (FHR) is considered normal:

A. 120-160 per minute

B. 110-150 per minute

C. 100-180 per minute

D. more than 200 per minute

E. 80-100 per minute

2. Pregnancy duration:

A. 36 weeks

B. 40 weeks

C. 42 weeks

D. 38 weeks

E. 37 weeks

3. The length of 1st labor stage (cervical dilatation) in primigravida:

A. 3-6 hours

B. 5-6 hours

C. 10-12 hours

D. 18-20 hours

E. 2-4 hours

4. The length of 1st labor stage (cervical dilatation) of multipara?

A. 3 - 4 hours

B. 5-6 hours

C. 10-12 hours

D. 12-14 hours

5. At what dilatation of cervix rupture of membrane occur normally?

A. 3 - 4 cm

B. 5 - 6 cm C. 7- 8 cm
D. 9-10 cm

E. 11-12 cm

6. What is determined by the 1st maneuvers of Leopold?

A. The position of the fetus

B. Height of the uterine fundus

C. Type of fetus

D. Presenting part

E. Attitude

7. What is determined by the 2nd maneuvers of Leopold?

A. The position of the fetus

B. Height of the uterine fundus

C. Type of fetus

D. Presenting part

E. Attitude

8. What is determined by the 3rd maneuvers of Leopold?

A. The position of the fetus

B. Height of the uterine fundus

C. Type of fetus

D. Presenting part

E. Attitude

9. What is determined by the 4rd maneuvers of Leopold?

A. The position of the fetus

B. Height of the uterine fundus

C. Type of fetus

D. Presenting part

E. Relation of presenting part to pelvic planes


10. Dilatation of cervix is full, amniotic membrane is intact. What phase of 1st

stage?

A. Latent phase

B. Active phase

C. Beginning of 1st stage

D. End of 1nd stage

E. Beginning of 2nd stage

11. What period is, if vaginal examination revealed the cervical dilatation of 5

cm?

A. 1st stage

B. Beginning of 2nd stage

C. End of 2nd stage

D. 3rd stage

E. Preliminary phase

12. Multipara has regular active contractions every 3-4 minutes for 35-40

seconds, which started 6 hours ago? What stage of labor is expected?

A. Latent phase

B. Beginning of labor

C. End of 1st stage

D. 2nd stage

E. 3 rd stage

13. The length of 1st labor stage (cervical dilatation) in primigravida:

A. 3-6 hours

B. 5-6 hours

C. 10-12 hours

D. 18-20 hours

E. 2-4 hours
14. The length of 1st labor stage (cervical dilatation) of multipara?

A. 3 - 4 hours

B. 5-6 hours

C. 10-12 hours

D. 12-14 hours

15. At what dilatation of cervix rupture of membrane occur normally?

A. 3 - 4 cm

B. 5 - 6 cm

C. 7- 8 cm

D. 9-10 cm

E. 11-12 cm

16. During pregnancy, which of the following is not true about maternal

cardiovascular physiology?

A. Heart rate decreases

B. Total peripheral resistance decreases

C. Arterial blood pressure decreases

D. Cardiac output increases

E. New murmurs are common

17. Why do pregnant women fill more easy breathing at the end of the

physiological pregnancy?

A. Absorption of amniotic fluid

B. Descending of the presenting part into the relaxed lower uterine segment

C. Contraction of the uterus

D. Reduction in fetal weight

E. Flexion of the fetal


18. What is determined by the second Leopold’s maneuver?

A. Height of the uterine fundus

B. Presenting part

C. Back and small parts of the fetus

D. Station of fetal head

E. Attitude

19. The smallest anteroposterior diameter of the pelvic inlet is called:

A. Interspinous diameter

B. True conjugate

C. Diagonal conjugate

D. Obstetric conjugate

E. Anatomic conjugate

20. The primigravida women fill the first quickening of fetus in:

A.12 weeks

B.16 weeks

C.18 weeks

D.20 weeks

E.22 weeks

21. The multipara women fill the first quickening of fetus in:

A. 12 weeks

B. 16 weeks

C. 18 weeks

D. 20 weeks

E. 22 weeks

22. The fundal height in 12 weeks of pregnancy is placed on the level of:

A. Umbilicus

B. Pubis symphysis
C. 2 cm above the pelvic inlet

D. Xiphisternum

E. 2 cm above umbilicus

23. Immunological tests depend upon secretion of:

A. Chorionic gonadotropins by trofoblast

B. Progesterone

C. Estrogen

D. Prolactin

E. Lactogen

24. An 22-year-old maltipara with a last menstrual period of May 7 presents for

her first obstetric visit at 10 weeks. What is this patient’s estimated date of

confinement?

A. November 10

B. February 10

C. February 14

D. November 14

E. November 7

25. Which one of the following statements regarding physiological amount of

blood loss in 3 phases of labor is true?

A.100-550 ml

B. 250-500 ml

C. 550-650 ml

D. 500-700 ml

E.10% of placental weight


26. How change the concentrations of hormones during pregnancy?

A. Increase level of progesterone and estrogen

B. Decrease level of progesterone and estrogen

C. Increase level of androgen

D. Increase level of estrogen

E. Decrease level of estrogen

27. All of the following changes are seen in pregnancy except:

A. Increased stroke volume

B. Increased cardiac output

C. Increased intravascular volume

D. Increased peripheral vascular resistance

E. Increased pulse

28. In pregnancy all occurs except:

A. Increased cardiac output

B. Decreased blood volume

C. Increased MCV

D. Increased glomerular flow

E. Increased pulse

29. Which of the following is correct for urinary system changes in pregnancy:

A. Renal blood flow decreased

B. GFR increased

C. Decreased creatinine clearance

D. Kidneys shrink by 1cm

E. Polyuria

30. Cardinal movements of labor are:

A. Engagement → descent → flexion → internal rotation → extension →

restitution → external rotation → expulsion


B. Engagement → flexion → descent → internal rotation → extension →

expulsion

C. Engagement → flexion → descent → external rotation → expulsion

D. Engagement → extension → internal rotation → external rotation → expulsion

31. How does the blood content change during pregnancy?

A. Volume erythrocytes decreases

B.Hemoglobin’s and plasma increases

C.Volume erythrocytes, hemoglobin’s and plasma decreases

D.Volume erythrocytes, hemoglobin’s and plasma increases

E. Hemoglobin, plasma decreases

32. How does the blood and volume of the plasma change during pregnancy?

A. Decreases

B. Increases

C. Volume erythrocytes, hemoglobin’s decreases and plasma increases

D. Volume erythrocytes, hemoglobin’s increases and plasma decreases

E. No changes

33. What is the true sign of pregnancy?

A. Nausea

B. Vomiting

C. Enlargement of the uterus

D. Auscultation of the fetal heartbeat

E. Pigmentation changes

34. Normal diameters of big pelvis?

A. 22-25-28-15 cm

B. 24-26-30-20 cm

C. 26-28-30-20 cm

D. 27-27-30-17 cm

E. 25-27-29-18 cm
35. The smallest diameter of the true pelvis is:

A. Interspinous Diameter

B. Diagonal conjugate

C. True conjugate

D. Intertuberous diameter

E.Lateral conjugate

36. The shortest diameter of fetal head is:

A. Biparietal diameter

B. Suboccipitofrontal diameter

C. Occipitofrontal diameter

D. Bitemporal diameter

C. Verticomental

37. Shortest diameter of pelvis is:

A. Diagonal conjugate

B. Obstetric conjugate

C. True conjugate

D. Lateral conjugate

E. All are equal

38. Longest diameter of pelvic inlet is:

A. Transverse diameter

B. Diagonal conjugate

C. Obstetric conjugate

D. True conjugate

E. All are equal

39.What is common type of presentation?

A. Cephalic

B. Vertex
C. Face

D. Brow

E. Breech

40. What is the submentobregmatic diameter on the fetal head?

This distance extends from:

A. under surface of the occipital bоnе to the chin

B. under surface of the occipital bоnе to the angel of bregma

C. under surface of the occipital bоnе to the line of hair

D. junction of the neck and lower jaw to the center of bregma,

41. What is the suboccipitobregmatic diameter of the fetal head?

A. 10 cm, circumference 33 cm

B. 9.5 cm, circumference 32 cm

C. 12 cm, circumference 34 cm

D. 13.5 cm, circumference of 38 cm

42. Which is not included in active management of III stage of labor?

A. Uterotonic within 1 minute of delivery

B.Immediate clamping, cutting and ligation of cord

C. General massage of uterus

D. Controlled cord traction

43. Cervical dilatation per hour in primapara is:

A. 1.0 cms

B. 1.5 cms

C. 1.7 cms

D. 2 cms

E. 2.5 cms
44. The graph showing relationship between cervical dilatation and duration of

labor is:

A. Partogram

B. Cervicograph

C. Growth curve

D. Dilatation chart

E. Gravidogram

45.What is not characteristic of true labor pain:

A. Absence of ‘bag of waters’

B. Painful uterine contractions

C. Progressive effacement and dilatation of the cervix

D. Pain often felt in front of the abdomen or radiating

towards the thighs

E. Irregular

46. A patient comes to your office with LMP 4 weeks ago. She denies any

symptoms such as nausea, fatigue, urinary frequency, or breast tenderness.

She thinks that she may be pregnant.

Which of the following evaluation methods is most sensitive in diagnosing

pregnancy?

A. No evaluation to determine pregnancy is needed because the patient is

asymptomatic and therefore cannot be pregnant

B. Serum pregnancy test

C. Detection of fetal heart tones by Doppler equipment

D. Abdominal ultrasound

E. Bimanual exam to assess uterine size.


47. A healthy 22-year-old primapara presents for her first OB visit at 1 weeks

gestational age. She denies any significant medical history both personally and in

her family. All of the following tests should be ordered as part of the initial

prenatal care visit except

A. CBC

B. HIV

C. Hepatitis B surface antigen

D. Type and screen

E. One-hour glucose
GYNECOLOGY
1. FSH stimulates:

A. Ovarian follicle growth

B. Corticosteroid production

C. Production of TSH in the thyroid gland

D. Progesterone production

E. Androgen production

1. Polymenorrhea is:

A. Lean menstruation

B. Short menstruation (1-2 days)

C. Short menstrual cycle

D. Heavy menstruation

E. Painful menstruation

2. The treatment of patients with ovarian exhaustion syndrome should be

directed:

A. To stimulate ovarian function

B. On getting a menstrual-like reaction

C. To reduce the severity of vegetovascular disorders

D. On stimulation of ovulation

E. On getting cyclical menstruation

3. The effect of large doses of exogenous estrogen on the ovary:

A. Increased ovarian production of estrogen

B. Increased ovarian production of progesterone

C. Atrophy of the ovarian tissue

D. Reduces ovarian progesterone production

E. Reduces ovarian production of estrogen


4. In a negative test with gonadotropins in patients with amenorrhea, it is

shown:

A. Test with estrogens and gestagens in a cyclic mode

B. Laparoscopy and ovarian biopsy

C. Laparoscopy and gonad removal

D. Progesterone test

E. Functional diagnostic tests

5. In the ovary, androgens serve as a substrate for the formation of:

A. Progesterone

B. Estrogen

C. Progesterone and estrogen

D. Gestagenov

E. Testosterone

6. The following applies to hypergonadotropic amenorrhea:

A. Ovarian Depletion Syndrome

B. Resistant Ovarian Syndrome

C. Ovarian Dysgenesis Syndrome

D. Testicular feminization syndrome

E. Sheehan Syndrome

7. Women with DUB are at risk:

A. Miscarriage

B. On the development of placental insufficiency and anomalies of the birth

force

C. On the development of genital tumors

D. On the development of breast tumors

E. Pregnancy
8. In the diagnosis of amenorrhea associated with acromegaly and gigantism, a

change is significant:

A. the size of the Turkish saddle on the radiograph of the skull

B. visual fields

C. FSH level

D. Excretion of 17-cop

E. Ultrasound diagnosis of the pelvic organs

9. Gestagens are secreted:

A. luteal cells of the corpus luteum

B. cells of the granular layer of the follicle

C. cells of the outer connective tissue of the follicle

D. Ovary cells

E. theca cells

10. Amenorrhea is considered pathological when menstruation is absent

A. before puberty

B. during puberty

C. during lactation

D. in postmenopausal women

E. during pregnancy

11. False amenorrhea can be caused by:

A. atresia of the cervical canal

B. Aplasia of the uterus

C. Dysgenesis of gonads

D. Ovarian exhaustion syndrome

E. Ovarian tumor

12. Ovarian exhaustion syndrome needs to be differentiated

A. with resistant ovary syndrome

B. with gonad dysgenesis syndrome


C. with menopausal syndrome

D. with ovarian tumor

E. with premenstrual syndrome

13. Treatment for DUB does not include:

A. physiotherapy

B. vitamins

C. uterotonics

D. antibiotics

E. hormonal hemostasis

14. The scope of the examination for pathology of the cervix

A. colposcopy

B. hysteroscopy

C. Ultrasound

D. bacterioscopy

E. laparoscopy

15. In what cases, for diagnostic purposes, puncture of the abdominal cavity

through the posterior arch is indicated?

A. suspicion of an ectopic pregnancy;

B. suspected ovarian cancer;

C. dysfunctional uterine bleeding;

D. uterine fibroids;

E. suspected ovarian apoplexy;

16. Research methods of the anatomical and functional state of the vagina:

A. inspection with speculum;

B. combined vaginal rectal examination;

C. cytological examination of the contents of the uterine cavity;

D. puncture of the posterior fornix;

E. determination of the purity of the vaginal contents.


17. The main method for the diagnosis of dysplasia and pre-invasive cancer of

the vulva is:

A. Biopsy followed by histological examination

B. Vulvoscopy

C. Radioisotope study

D. Cytological examination of smear prints

E. Colposcopy

18. The most informative method for the diagnosis of genital prolapse is:

A. ultrasound;

B. gynecological examination;

C. sigmoidoscopy;

D. cystoscopy;

E. hysteroscopy.

19. Algodismenorea is:

A. Rare and scarce menstruation

B. painful menstruation

C. Reduced blood loss during menstruation

D. intermenstrual meager spotting

E. heavy periods

20. Menorrhagia is:

A. Acyclic uterine bleeding

B. cyclic uterine bleeding

C. Painful and heavy menstruation

D. pre- and postmenstrual bleeding

E. Change in the rhythm of menstruation


21. Dysfunctional uterine bleeding with follicular atresia occurs against

A. High estrogen saturation

B. low estrogen saturation

C. High gestagenic saturation

D. Hyperandrogenemia

E. Prolactin overproduction

22. Ovulatory acyclic bleeding characteristic

A. with short-term rhythmic persistence of the follicle

B. with prolonged persistence of the follicle

C. with follicular atresia

D. with persistence of the corpus luteum

E. in case of corpus luteum insufficiency

23. Anovulatory cyclic bleeding characteristic

A. with prolonged follicular persistence

B. with short-term persistence of follicles

C. with persistence of the corpus luteum

D. in case of corpus luteum insufficiency

E. with follicular atresia

24. A test with clomiphene is performed if:

A. Uterine fibroids

B. Anovulation

C. Adenomyosis

D. Endometritis

E. Hyperprolactinemia
25. The diagnosis of impaired tubal pregnancy is an indication

A. for emergency laparotomy

B. for laparoscopy

C. for ultrasound examination

D. for special studies

E. for puncture of the back arch

26. Currently, age is considered normal for menopause.

A. 45 years

B. 5O years

C. 4O years

D. 55 years

E. 43 years

27. The autonomic-nervous manifestations of menopausal syndrome include

A. Irritability

B. Sweating

C. Sleep Disorders

D. Decrease in memory

E. Decreased libido

28. Metrorrhagia is

A. Change in the rhythm of menstruation

B. increased blood loss during menstruation

C. increase in the duration of menstruation

D. Acyclic uterine bleeding

E. Reducing blood loss during menstruation


29. The most informative in the differential diagnosis between uterine myoma

and ovarian tumor:

A. two-handed vaginal examination

B. hysteroscopy

C. ultrasound;

D. laparoscopy;

E. Probing of the uterine cavity.

30. Hysteroscopy allows to diagnose:

a) subserous uterine fibroids;

b) submucous uterine fibroids;

c) tubal-peritoneal infertility;

d) Stein-Levinthal syndrome;

e) tubal pregnancy.

31. The screening method to detect cervical pathology in modern conditions is:

A. visual inspection;

B. smear cytology;

C. vaginal smear on the flora

D. radionuclide method

E. PCR for HPV

32. The presence of ovulation can be judged by the results of all the studies listed

below:

A. Definitions of hCG

B. Ultrasound monitoring of the development of a dominant follicle

C. Histological examination of endometrial scraping

D. Determination of the concentration of sex steroid hormones in the blood on

days 12-14 of the menstrual cycle

E. Follicle persistence
33. Screening - a method for detecting cervical pathology is:

A. Visual inspection

B. Colposcopy

C. Radionuclide method

D. Cytological examination of smears

E. Laparoscopy

34. In girls at an early age (from 2 to 8 years), the following are more common:

A. ovarian tumors;

B. dysfunctional bleeding;

C. congenital anomalies of the genital organs;

D. vulvovaginitis;

E. salpingo-oophoritis.

35. Classification of pelvioperitonitis by stages, choose the correct answer:

A. primary, secondary

B. reactive, terminal, toxic

C. adhesive, exudative

D. serous, purulent, fibrinous;

E. adhesive, fibrinous

36. Which of the following inflammatory diseases in the acute stage is

accompanied by symptoms of bryuschina irritation?

A. Endometritis.

V. Kolpit

C. vulvovaginitis

D. Cervicitis

E. Bartholinitis
37. Which of the following inflammatory diseases is an indication for

laparotomy in case of suppuration?

A. Parameter

V. Kolpit

C. vulvovaginitis

D. Cervicitis

E. Bartholinitis

38. What are the diagnostic tests that indicate the presence of bacterial vaginosis:

A. detection of key cells;

B. lowering the pH of vaginal secretions;

C. negative amino test of whites;

D. detection of abnormal cells

E. increase in basal temperature.

39. During the parametrite, a stage is distinguished:

A. desquamation

B. infiltration

C. proliferation

D. tissue decomposition

E. edematous

40. The complication that most often arises in the management of the IUD is:

A. isthmic-cervical insufficiency;

B. ectopic pregnancy;

C. habitual miscarriage;

D. acute infection;

E. pelvic vein thrombosis.


41. The symptom most characteristic of vulvovaginitis:

A. sharp pains;

B. burning, itching

C. high temperature;

D. ulceration;

E. spotting.

42. The most important risk factor for endometritis after childbirth is:

A. frequent sexual activity;

B. cesarean section;

C. childbirth through the birth canal;

D. previous urinary tract infection;

E. corresponding upper respiratory tract infection

43. The main clinical symptom of tuberculous lesions of the uterine appendages:

A. chronic pelvic pain;

B. amenorrhea;

C. menometrorrhagia;

D. primary infertility;

E. secondary infertility.

44. Which of the following symptoms is the main clinical one for bacterial

vaginosis?

A. itching of the external genitalia and perineum;

B. dyspareunia;

C. profuse leucorrhoea with an unpleasant odor;

D. dysuria;

E. pelvic pain.
45. Specify the lack of an intrauterine device:

A. increases the risk of inflammatory diseases

B. contraindications for lactation

C. short term

D. a large number of side effect

E. effect on lactation

46. Specify the time during which the sperm retains the ability to fertilize during:

A. 6 hours

B. 12 hours

C. 24 hours

D. 48 hours

E. 72 hours

47. The most informative way to assess the functional state of the ovaries is:

A. basal temperature measurement

B. symptom of cervical mucus tension

C. suction curettage

D. laparoscopy

F. Hysteroscopy

48. Algodismenorea is:

A. Rare and scarce menstruation

B. painful menstruation

C. Reduced blood loss during menstruation

D. intermenstrual meager spotting

E. heavy periods
49. Basal temperature is measured:

A. In the morning

B. In the evening;

C. 2 times a day;

D. After 3 hours.

E. After 5 hours

50. Parametritis is:

A. inflammation of the ovary;

B. inflammation of the cecum;

C. inflammation of the fallopian tube;

D. inflammation of the peritoneal tissue;

E. inflammation of the omentum.


Training test questionnaire for Reproductive system

1. Which imaging modality is used as breast diagnostic screening?

A. Mammography
B. Chest X-ray
C. Ultrasound
D. CT-scan
E. MRI

2. Identify the view on the mammogram

A. Mediolateral oblique (MLO)


B. Bilateral craniocaudal (CC)
C. Lateromedial (LM)
D. Lateromedial oblique (LMO)
E. Mediolateral (ML)

3. Simple cyst was detected during ultrasound examination. Which BI-RADS classification it
belongs to?

A. BI-RADS 0
B. BI-RADS 1
C. BI-RADS 2
D. BI-RADS 3
E. BI-RADS 4

4. Which imaging modality is the best for evaluation of breast cyst?

A. Mammography
B. Chest X-ray
C. MRI
D. CT-scan
E. Ultrasonography
5. Which imaging examination can be performed to study fallopian tubes with iodine containing
contrasting agent?

A. Hysterosalpingography
B. Hysteroscopy
C. Intravenous pyelography
D. Ultrasonography
E. Cystoscopy

6. A 48 year- old female came to mammologist, she complained to breast lump. Clinical
examination revealed palpable mass of breast. What is first choice imaging modality in this
case?

A. Mammography
B. Chest X-ray
C. MRI
D. CT-scan
E. Ultrasonography

7. A 50-year-old women admitted to oncology center. She has complains of pain and last week
abnormal bleeding. Hb level 70 . A 2 year ago she had diagnosed with cervical polyp (15
mm) Which imaging study is most suitable for clarifying the diagnosis?

A. Ultrasound
B. CT-scan
C. Hysteroscopy
D. MRI
E. X-ray

8. A 32-year female was consulted to the doctor with complain to heavy and painful periods.
Doctor suggested pelvic endometriosis. Which imaging modality is the first choice in this
case?

A. Ultrasound
B. CT-scan
C. Hysteroscopy
D. MRI
E. X-ray
PATHOPHYSIOLOGY
Training Tests
1) The hypothalamus produces the following hormones:
A) Gonadotropins
b) Estrogen
b) Gestagens
d) Leasing – factors

2) FSH stimulate:
A) Growth of follicles in the ovaries
b) Products of corticosteroid
(b) TTG products in the thyroid gland
D) All of the above

3) Progestin:
A) Reduce blood cholesterol
b) Determine the content of primary and secondary sexual characteristics
b) All of the above
D) None of the above

4) The group of risk of inflammatory after birth diseases include women:


A) With a burdened obstetric and gynecological history
b) With chronic inflammatory process
c) With Extra Energy inflammatory diseases (tonsillitis, pyelonephritis)
D) All of the above

5) in inflammatory diseases of female genital organs is noted:


(a) Prevalence of microbial associations
b) Increase the number of anaerobic and virus
(b) increased permeability and exhaustion
d) All answers correct

6) The causes of infertility of women in marriage are:


(a) Inflammatory diseases of the genital organs
b) Infantialism and hypoplasia of genital organs
(b) General debilitating diseases and intoxication
d) All answers correct

7) Features of normal menstrual cycle are:


A) Ovulation
b) Formation of yellow body in the ovary
b) Prevalence of progesterone in the second phase of the cycle
D) All of the above

8) Estrogen:
A) Promote peristalsis of the uterus and tubes
b) Strengthen the processes of ossification
b) All of the above
D) None of the above

9) Androgens are formed:


A) In the ovaries (interstitial cells, Strome, inner Take)
b) In the net zone of the adrenal cortex
b) All of the above
D) None of the above
10) In its action on the body of estrogen:
A) Block receptors to oxytocin
b) Cease (weaken) proliferative processes in the endometrium
b) Cause secretory transformations in the endometrium
D) All of the above
e) None of the above

11) Increase of rectal temperature in the second phase of menstrual menstrual


cycle
due to:
A) The action of the ovarian progesterone
b) The action of progesterone, which reduces the heat return
(b) Intensification of biochemical processes in the uterus
D) All of the above
e) None of the above

12) The full value of the luteal phase of the menstrual cycle is evidenced by:
A) Increase in basal temperature in the first phase of the cycle
b) Proliferative processes in the endometrium in the second phase of the cycle
c) All of the above
D) None of the above

13) Amenorrhea is the absence of menstruation during:


A) 4 months
b) 5 months
c) 6 months
d) All of the above
e) None of the above
14) In the interaction of the hypothalamus, pituitary gland and ovaries in the
regulation of the menstrual cycle:
A) One link (overlying) stimulates the function of another link
b) The underlying link brakes or regulates the function of the overlying
b) The function of the links is synchronous
D) The shutdown of one link violates the whole system
e) All of the above

15) After ovulation, the egg retains the ability to fertilize during:
A) 6 hours:
b) 12-24 hours;
in) 3-5 days;
d). 10 days

16) During puberty, the following major changes occur in the body:
(a) Suppression of the Gonadotropic function of the pituitary gland;
b) Activation of hormonal function of ovaries;
c) The rhythm of the FSH allocation is not established;
D) Regular &quot;peaks&quot; of the LG excretion are established;
e) None of the above.

17) Body mass deficiency is usually one of the reasons:


a) Delay in the onset of the first menstruation;
b) Long-term formation of menstrual function;
b) The development or aggravation of menstrual dysfunction;
d) All listed;
e) None of the above

18) Mechanism of action of hormones on a cage is caused by presence:


a) Prostaglandins;
b) Receptors;
b) Specific enzymes;
d) All listed;
e) None of the above.

19) Small doses of estrogen:


a) Stimulate FSH products;
b)Ssuppress FSH products;
b) Strengthen the production of LG;
D) Suppress the production of LTG.

20) Desquamation of the functional layer of endometrium occurs due to:


a) Pathway; &quot;Peak&quot; reset luther
b) Reducing the level of estrogen and progesterone in the blood;
b) Reducing the level of prolactin in the blood;
d) Increase the level of estradiol in the blood;
e) &quot;Peak&quot; release of Follitropin.
21) Physiological Amenorrhea is the absence of menstruation:
a) Girls under 10-12 years of age;
b) During pregnancy;
b) During lactation;
d) In old age;
e) All of the above are true.
22) The reason of false amenorrhea in girls of Puberty Nogo age can be all
listed,
except:
a) Hymen atresia;
b) Atrezia Vagina;
b) Uterine hyperplasia;
d) Atrezia Cervical canal.

23) False amenorrhea may be caused by:


a) Cervical canal atresia;
b) Aplasia of the body of the uterus;
b) The dysgenesis of the Gonnad;
d) All the diseases listed above.

24) Lack of sexual development is due to:


a) Transferred infectious diseases in early childhood;
b) Undergone inflammatory processes of pelvic organs of viral etiology;
b) Chromosomal anomalies;
d) All listed;
e) None of the above.

25) Delay of sexual development is absence:


a)Secondary sexual characteristics by 14 years and menstruation by 16 years;
b) Secondary sexual characteristics by 12 years, and menstruation by 16 years;
b) Secondary sexual characteristics by 14 years, and menstruation by 18 years;
d) Secondary sexual characteristics to 12 years, and menstruation to 18 years.
Tests for the propedeutics of childhood diseases.
1. Delayed sexual development in girls consider the absenceof the secondary sexual
characteristics at the age?
A. 10-13
B. 11-14
C. 12-16
D. 13-17
E. 14-18
2. What age corresponds to the degree of puberty (years) Ма3P2 Ax2Мео?
A. 16-17
B. 14-15
C. 12-13
D. 10-11
E. 8-9
3. One of the main reasons for delayed sexual development, it is believed
A. Birth injury
B. Intrauterine infection
C. Chromosomal abnormality
D. Infectious diseases
E. Stressful situation
4. The girl has 3 days of life, there is an engorgement of the mammary glands, bloody
discharge from the genital tract, hyperemia of the skin. Determine what condition you are
talking about?
A. sclerema
B. jaundice of newborn
C. sexual crisis
D. hermaphroditism
E. hypothyroidism

5. When examined by a pediatrician, a 2-month old boy, there is a change in the urine
stream, during urination, in the form of leakage into the perineum. Objectively: in the area of
the penis there is a splitting of the lower wall of the urethra. What pathology of the genital
organs is mentioned in the task?
A. epispadias
B. hypospadias
C. cryptorchidism
D. anarchism
E. micropenis
6.A boy of 8 years old is being examined by a pediatrician. Objectively: a change in the
timbre of the voice, an increase in the width of the shoulder girdle, hair growth in the armpits,
pubic region, enlargement of the penis and testicles. What state does the objective status
correspond to?
A. The child is healty
B. hypopituitarism
C. False male hepmaphroditism
D. Premature sexual development
E. Delayed sexual development
7. On a routine examination, the girl 14 years old. Objectively: the mammary glands are
round in shape, the nipples rise above the areola. Pubic hair in a triangle, long, curly, thick. In
the hair is thick, curly over the entire surface. Menses are regular. Estimate the sexual
development of a girl according to Tanner?
A. Мао РоАхоМео
B. Ма2Р1 АхоМео
C. Ма3 Р3 Ах1Ме1
D. Ма3 Р2Ах2 Мео
E. Ма3 Р3 Ах3 Ме3
8.A 2-year-old girl is being examined by a pediatrician Objectively: physical development
corresponds to age, there is an increase in mammary glands. What changes do you see in the
patient?
A. Isolated thelarche
B. Isolated adrenarche
C. Sexual crisis
D. Premature pubarche
E. Premature sexual development

9.At the reception at FMC №3, a mother and her 8-year-old daughter addressed.Objectively:
the mammary glands are significantly enlarged, hair growth in the armpits, pubic region, the
appearance of the first menarche. What state is described in the task?
A. sexual crisis
B. premature sexual development
C. delayed sexual development
D. hypothyroidism
E. The child is healty
Urology Test for students: REDRODUCTIVE SYSTEM

1). Marriage should be considered infertile if pregnancy is not comes?


1. within 6 months

2. within 1 year

3. within 2 years

4. within 3 years

5. More than 3 years

2). The lower limit of normal sperm count in 1 ml should be (according


to WHO criteria):

1. 20 million

2. 40 million

3. 60 million

4. 80 million

5. 100 million

3). Azoospermia - is

1. The absence of the cells of spermatogenesis in the ejaculate and


spermatozoas

2. The presence of single cells of spermatogenesis in the ejaculate

3. The presence of only dead sperm in the ejaculate

4. The lack of allocation of ejaculate

5. the absence of ejaculate

4). Aspermia - is

1. The absence of sperm in the ejaculate

2. The absence of ejaculate

3. The lack of allocation of ejaculate

4. The small amount of sperm in the ejaculation

5. The absence of motile sperm in the ejaculate


5). Male sex cells (sperm) are formed in:

1. Ovaries

2. Gonad - testis

3. Epididymis

4. Prostate

5. Seminal vesicles

6). Male sex hormones are produced in:

1. Ovarian Theca cells

2. Prostate

3. Testis Leydig cells

4. Seminal vesicles

5. Sertoli cells

7). Reproductive function is:

1. The regulation of sexual activity

2. The fusion of two cells

3. Regulation of oogenesis and spermatogenesis

4. Reproductive function

5. Sexual arousal and attraction

8). Functions of male sex hormones:

1. The regulation of oogenesis

2. The development of secondary sexual characteristics

3. Ensuring the development of mammary glands

4. Ensure the development of the genital organ


5. Regulation of water-salt metabolism

9). What is oligozoospermia?

1. The absence of ejaculate

2. The absence of sperm in the ejaculate

3. The absence of mature sperm in the ejaculate

4. The small amount of sperm in the ejaculate

5. The absence of sperm in the ejaculate

10). How much the normal amount of single ejaculate in an adult?

1. 0.5-1.5 ml

2. depends on body weight men

3. 2.0-5.0 ml

4. 6-7 ml

5. 8-10 ml

11). What is aspermia?

1. The absence of ejaculate

2. The absence of sperm in the ejaculate

3. The absence of motile sperm in the ejaculate 4. The absence of mature


sperm in the ejaculate 5. The small number of sperm in the ejaculate

12). What is hemospermia?

1. The presence of blood in urine

2. The total hematuria

3. The discharge of blood after urination

4. The presence of blood in the ejaculate


5. The discharge of blood from the urethra at the beginning
of urination

13). What is necrospermia?

1. The absence of ejaculate

2. The absence of sperm in the ejaculate

3. The absence of mature

sperm in the ejaculate

4. The small amount of

sperm in the ejaculate

5. The absence of motile sperm in the ejaculate

14). What is prostatorrhea?

1. The allocation of prostate fluid during erection

2. The allocation of prostate fluid in the end of the act of

defecation or urination 3. Premature ejaculation

4. The allocation of semen without erection

5. Prostate secretions during intercourse.

15). Oligozoospermia may be the result of: 1. After


severe infections

2. Diabetes

3. Nephroptosis

4. Urolithiasis

5. Genitourinary fistula
16). Where occurs maturation of majority of the
sperm?

1. in the testis

2. in the epididymis

3. in the vas deferens


4. in the seminal vesicles
5. throughout ejaculatory tract

17). Insufficient numb


er of sperm with normal morphology is
1. Asthenozoospermia

2.Oligozoospermia

3. Necrospermia

4. Hemospermia

5. Theratozoospermia

8). Causes of male infertility

1. Pyelonephritis

2. Urolithiasis

3. Acute respiratory viral

infections

4. Varicocele

5. Nephroptosis

20). Testicular inflammation is

1. Prostatitis

2. Epididymitis

3. Orchitis

4. Cystitis

5. Vesiculitis
PREVIOUS YEAR PAPER
GYNECOLOGY TESTS (500) FOR THE 5th YEAR
STUDENTS OF THE GENERAL MEDICINE.
1) A positive test (small test) with dexamethasone indicates
that:
A. The source of hyperandrogenism is the ovaries
B. The source of hyperandrogenism is the adrenal gland
C. Hyperandrogenism associated with pituitary adenoma
D. Hyperandrogenism due to adrenal corticosteroma
E. Hyperandrogenism associated with an ovarian tumor

2) The therapeutic and diagnostic effect of dexamethasone


in hyperandrogenism is due to:
A. Inhibition of ovarian function
B. Inhibition of adrenal function
C. Inhibition of ACTH
D. Acceleration of androgen inactivation
E. Inhibition of pituitary function

3) To restore generative function during gonad dysgenesis, it


is necessary:
A. Long-term cyclic therapy with sex hormones
B. Stimulation of ovulation
C. Functional diagnostic tests
D. Sphenoid ovarian resection
E. Restoring generative function is futile.

4) Dysfunctional uterine bleeding is called:


A. bleeding due to changes in the uterus
B. bleeding in inflammatory diseases of the uterus
C. due to impaired rhythmic secretion of ovarian
hormones
D. Genital bleeding in Werlhof disease
E. Bleeding due to miscarriage

5) The most common mechanism for the development of


dysfunctional uterine bleeding in the juvenile period is:
A. hypoluteinism
B. follicle persistence
C. atresia of the follicle
D. hyperprolactinemia
E. disorders in the blood coagulation system

6) The most informative way to assess the functional state of


the ovaries is:
A. basal temperature measurement
B. symptom of cervical mucus tension
C. suction curettage
D. laparoscopy
E. Hysteroscopy

7) According to the tests of functional diagnostics (TFD), it


is impossible to determine:
A. biphasic menstruation
B. The level of estrogenic saturation of the body
C. Ovulation
D. The usefulness of the luteal phase of the cycle
E. Hormone-producing ovarian tumor
8) For anovulatory menstrual cycle with follicular atresia is
characteristic:
A. amenorrhea
B. Algomenorrhea
C. oligoamenorrhea
D. promenomenorrhea
E. hypomenorrhea

9) The conditions for cyclic hormone therapy in patients


with menstrual irregularities are:
A. Preliminary consultation by a neurologist
B. Preliminary consultation of the therapist
C. The use of a minimum amount of estrogen,
taking into account the age of a woman, monitoring tests
of functional diagnostics
D. use of gestagens, ultrasound examination of the
uterus
E. hormone testing

10) A 16-year-old girl developed spotting from the genital


tract, lasting 8 days after a 2-month delay. The first
menstruation appeared 4 months ago for 2 days, after 28
days, moderate, painless. Denies sexuality. The
development is correct, well physically built. When recto-
abdominal examination of the pathology is not detected. Нb-
80 g / l. Probable diagnosis:
A. hormone-producing ovarian tumor
B. cervical cancer
C. Polyp of the cervix
D. juvenile uterine bleeding
E. endometrial polyposis
11) Pathogenetic therapy of endometrial hyperplastic
processes in women of reproductive age consists in the use
of:
A. estrogen-progestogen drugs or progestogens
B. dexamethasone
C. androgen
D. Thyroidin
E. estrogen

12) In the anovulatory menstrual cycle, basal temperature is


characterized by:
A. rise in temperature after ovulation
B. lack of temperature rise
C. rise in temperature before ovulation
D. rise in temperature before menstruation
E. Rise in temperature immediately after menstruation

13) Most often, women aged 40-45 years complain of:


A. Algodismenorea
B. heavy periods
C. irregular menstruation
D. premenstrual tension
E. painful menstruation

14) Anovulatory menstrual cycle is characterized by:


A. cyclic changes in the body
B. Long-lasting follicle persistence
C. Prevalence of gestagens in phase 2
D. prevalence of estrogen in phase 1
E. Acyclic change in the body

13) The hypothalamus produces the following hormones:


A. Gonadotropins
B. Estrogens
C. gestagens
D. Releasing factors
E. prolactin

14) Releasing factors carry out:


A. Transmission of nerve impulses to the nervous system
B. Production of gonadotropins
C. Estrogen Production
D. Production of FSH and LH
E. ACTH production

15) In the glomerular zone of the adrenal cortex are formed:


A. Glucocorticoids
B. Aldosterone
C. Norepinephrine
D. Androgens
E. Estrogens

16) FSH stimulates:


A. Ovarian follicle growth
B. Corticosteroid production
C. Production of TSH in the thyroid gland
D. Progesterone production
E. Androgen production

17) Small doses of estrogen:


A. Stimulate FSH production
B. Suppress FSH production
C. Strengthen LH production
D. Suppress LH production
E. Stimulates TTG production

18) Polymenorrhea is:


A. Lean menstruation
B. Short menstruation (1-2 days)
C. Short menstrual cycle
D. Heavy menstruation
E. Painful menstruation

19) For anovulatory menstrual cycle with short-term


persistence of a mature follicle is not typical:
A. Symptom of the pupil (+++)
B. Single-phase basal temperature
C. High estrogen levels
D. In the scraping of the endometrium in the second phase
of the cycle - the late phase of proliferation
E. Prolonged spotting discharge

20) The treatment of patients with ovarian exhaustion


syndrome should be directed:
A. To stimulate ovarian function
B. On getting a menstrual-like reaction
C. To reduce the severity of vegetovascular disorders
D. On stimulation of ovulation
E. On getting cyclical menstruation

21) The mechanism of action of hormones on the cell is due


to the presence of:
A. Prostaglandins
B. Receptors
C. Specific Enzymes
D. Isoenzymes
E. Thromboxant

22) The effect of large doses of exogenous estrogen on the


ovary:
A. Increased ovarian production of estrogen
B. Increased ovarian production of progesterone
C. Atrophy of the ovarian tissue
D. Reduces ovarian progesterone production
E. Reduces ovarian production of estrogen

23) Amenorrhea is the absence of menstruation during:


A. 4 months
B. 5 months
C. 6 months
D. 1 year
E. 2 months
24) Clinical manifestations, a typical form of gonadal
dyskinesia karyotype (45XO / 46XY) and no menstruation:
A. Sheehan Syndrome
B. Swyer Syndrome
C. Bisexual Gonad Syndrome
D. Symmonds disease
E. Shereshevsky-Turner

25) In patients with amenorrhea with Symmonds disease,


the following clinical manifestation is not observed:
A. Severe metabolic endocrine disorders
B. Premature aging
C. Decreased appetite
D. Sudden exhaustion
E. Massive bleeding

26) Amenorrhea associated with gigantism:


A. With hyperproduction of ACTH until puberty
B. With hyperproduction of GH, until puberty
C. With hyperproduction of ACTH after puberty
D. With hyperproduction of GH after puberty
E. With hyperproduction of ACTH during puberty

27) When treating a patient with any form of gonadal


dysgenesis, recovery, as a rule, is excluded:
A. Menstrual function
B. Sexual function
C. Generative function
D. Proliferative function
E. Ovulation

28) Shereshevsky-Turner syndrome is:


A. A “pure” form of gonadal dysgenesis
B. A typical form of gonadal dysgenesis
C. "Mixed" form of gonadal dysgenesis
D. False male hermaphroditism
E. The pathological process associated with massive
bleeding in childbirth

29) In a negative test with gonadotropins in patients with


amenorrhea, it is shown:
A. Test with estrogens and gestagens in a cyclic mode
B. Laparoscopy and ovarian biopsy
C. Laparoscopy and gonad removal
D. Progesterone test
E. Functional diagnostic tests

30) In the ovary, androgens serve as a substrate for the


formation of:
A. Progesterone
B. Estrogen
C. Progesterone and estrogen
D. Gestagenov
E. Testosterone

31) The main estrogen hormone in the body of a woman in


the postmenopausal period is:
A. Estradiol
B. Estron
C. Estriol
D. Estradiol dipropionate
E. estrogen

32) The following applies to hypergonadotropic


amenorrhea:
A. Ovarian Depletion Syndrome
B. Resistant Ovarian Syndrome
C. Ovarian Dysgenesis Syndrome
D. Testicular feminization syndrome
E. Sheehan Syndrome

33) Features of the treatment of DUB in menopausal


patients are:
A. Conducting a separate diagnostic diagnostic curettage
to verify the diagnosis
B. Ovarian Suppression Therapy
C. Treatment of concomitant somatic pathology
D. Cyclic hormone therapy
E. Hormone treatment

34) Women with DUB are at risk:


A. Miscarriage
B. On the development of placental insufficiency and
anomalies of the birth force
C. On the development of genital tumors
D. On the development of breast tumors
E. Pregnancy
38) In the diagnosis of amenorrhea associated with
acromegaly and gigantism, a change is significant:
A. the size of the Turkish saddle on the radiograph of the
skull
B. visual fields
C. FSH level
D. Excretion of 17-cop
E. Ultrasound diagnosis of the pelvic organs

39) Gestagens are secreted:


A. luteal cells of the corpus luteum
B. cells of the granular layer of the follicle
C. cells of the outer connective tissue of the follicle
D. Ovary cells
E. theca cells

40) Amenorrhea is considered pathological when


menstruation is absent
A. before puberty
B. during puberty
C. during lactation
D. in postmenopausal women
E. during pregnancy

41) False amenorrhea can be caused by:


A. atresia of the cervical canal
B. Aplasia of the uterus
C. Dysgenesis of gonads
D. Ovarian exhaustion syndrome
E. Ovarian tumor

42) Ovarian exhaustion syndrome needs to be differentiated


A. with resistant ovary syndrome
B. with gonad dysgenesis syndrome
C. with menopausal syndrome
D. with ovarian tumor
E. with premenstrual syndrome

43). Treatment for DUB does not include:


A. physiotherapy
B. vitamins
C. uterotonics
D. antibiotics
E. hormonal hemostasis

44). Which test of functional diagnostics indicates the


presence of a biphasic menstrual cycle?
A. Pupil symptom
B. Karyopicnosis Index
C. Basal thermometry
D. Symptom of "fern"
E. Symptom of crystallization

45) Algodismenorea is:


A. Rare and scarce menstruation
B. painful menstruation
C. Reduced blood loss during menstruation
D. intermenstrual meager spotting
E. heavy periods

46). Menorrhagia is:


A. Acyclic uterine bleeding
B. cyclic uterine bleeding
C. Painful and heavy menstruation
D. pre- and postmenstrual bleeding
E. Change in the rhythm of menstruation

47) Metrorrhagia is
A. Change in the rhythm of menstruation
B. increased blood loss during menstruation
C. increase in the duration of menstruation
D. Acyclic uterine bleeding
E. Reducing blood loss during menstruation

48) The most common mechanism for the development of


dysfunctional uterine bleeding in the juvenile period is
A. increased FSH
B. follicular persistence
C. follicular atresia
D. hyperprolactinemia
E. LH increase
49) The main method for stopping dysfunctional uterine
bleeding in the premenopausal period is:
A. use of synthetic estrogen-progestogen drugs
B. The introduction of hemostatic and uterine-contracting
agents
C. Use of 17-hydroxyprogesterone capronate (17-OPK)
continuously
D. Separate diagnostic curettage of the mucous
membrane of the uterine cavity and cervical canal.
E. use of progestogens

50) The composition of injectable contraceptives includes:


A. prolonged progestogens
B. conjugated estrogens
C. Microdoses of progestogens
D. Antiandrogens
E. estrogens

51) False amenorrhea can be caused by:


A. atresia of the cervical canal
B. Aplasia of the uterus
C. Dysgenesis of gonads
D. follicular atresia
E. Severe Infectious Diseases

52) False amenorrhea may be the result of:


A. hypothyroidism
B. neurogenic anorexia
C. testicular feminization syndrome
D. hyperthyroidism
E. Atresia hymen

53) What additional research methods are not used to clarify


the diagnosis of amenorrhea:
A. Ultrasound examination of the internal genital organs
B. Functional diagnostic test examination
C. Craniography
D. Separate curettage
E. Determination of hormone levels

54) What form of amenorrhea indicates a negative result of


a functional test with combined estrogen-progestogen
drugs?
A. Hypothalamic
B. Pituitary
C. Ovary
D. Uterine
E. Central

55) When a combination of uterine fibroids and internal


uterine endometriosis in a patient of reproductive age with
hyperpolymenorrhea and secondary anemia is shown:
A. Extirpation of the uterus with appendages
B. Subvaginal uterine amputation without appendages
C. Subvaginal amputation of the uterus with fallopian
tubes, with excision of the mucous membrane of the
cervical canal
D. Subvaginal amputation of the uterus with tubes
E. Uterine amputation with appendages
56) The term "Adenomyosis" applies:
A. In all cases of detection of endometriosis, regardless of
location
B. Only with focal growths of endometrioid tissue in the
muscle layer of the uterus
C. With endometriosis, which is accompanied by the
formation of cysts
D. Only in cases when the germination of the myometrium
is accompanied by the presence of myomatous nodes
E. With endometriosis of the cervix

57) For internal endometriosis of the uterus of the 3rd stage


on the eve of menstruation, a bimanual examination is not
characteristic of:
A. Uterine compaction
B. Uterine enlargement
C. Uterine softening
D. Severe soreness
E. Uterus reduction

58) A 38-year-old patient complained of abdominal pain.


The pain appeared today 3 hours ago, weakly positive with \
m Shchetkina. T-38.2 C, leukocytosis. With a gynecological
examination, the uterus is enlarged to 8 weeks, nodular.
Diagnosis:
A. Inflammation of the appendages
B. Charioamnionitis
C. Endometritis
D. Necrosis of myoma nodes
E. Salpingitis

59) A 38-year-old woman complained of very painful


menstruation during the last 6 years, especially in the first 2
days. A history of 2 births and 2 medical abortions without
complications, the last year ago. The menstrual cycle is not
broken. The last menstruation ended 5 days ago. She was
protected from pregnancy by interrupted sexual intercourse.
On examination, the abdomen without painful, the cervix
and vagina without pathologies, the body of the uterus is
slightly more than normal, dense, the appendages are not
palpable. Probable diagnosis:
A. Uterine fibroids
B. endometriosis
C. Uterine pregnancy
D. Endometrial polyposis
E. Adenomyosis

60) The main symptom of submucous uterine fibroids:


A. Chronic pelvic pain
B. Algodismenorea
C. Menorrhagia
D. Acute pain
E. Chills

61) The most informative method for diagnosing a born


myomatous node:
A. Transvaginal ultrasound
B. Inspection by the speculum
C. Hysteroscopy
D. Laparoscopy
E. Bimanual research

62) An informative method for the diagnosis of submucous


myomatous node:
A. Inspection by the speculum
B. Laparoscopy
C. Hysteroscopy
D. Colposcopy
E. Sonography

63) The method of drug treatment of uterine fibroids in


reproductive age:
A. gestagens
B. Estrogens
C. Androgens
D. Progestins
E. Vitamin Therapy

64. What are the leading clinical symptoms of dysplasia and


in situ cervical cancer:
A. pelvic pain;
B. mucopurulent leucorrhoea;
C. contact bleeding;
D. Acyclic uterine bleeding;
E. infertility.

65) The optimal amount of surgical treatment in the


presence of uterine fibroids with the location of the node in
the cervix:
A. Subvaginal uterine amputation
B. Conservative myomectomy using vaginal access
C. Extirpation of the uterus
D. Surgical treatment of uterine fibroids does not result in
such localization
E. Subvaginal uterine amputation with appendages

66) Which of the following symptoms are not related to


uterine fibroids?
A. Palpable palpable tuberous formation in the small pelvis
associated with the cervix
B. Painful menstruation
C. Heavy menstruation
D. Dysfunction of the bladder and rectum
E. Rare menstruation

67) What complications are characteristic for the subserous


form of uterine fibroids?
A. Malignant degeneration of the tumor
B. Twisting the legs of the tumor
C. Uterus inversion
D. Posthemorrhagic anemia
E. Bladder dysfunction

68) To prevent the development of cervical endometriosis,


organ cryodestruction is performed on the following days of
the menstrual cycle:
A. 1-2 days before your period
B. Immediately after menstruation
C. On day 12-14
D. On day 16-18
E. On day 16-25

69) Basal temperature is measured:


A. In the morning
B. In the evening;
C. 2 times a day;
D. After 3 hours.
E. After 5 hours

70. The ovary is maintained in the abdominal cavity due to:


A. round ligament;
B. cardinal ligament;
C. funnel-pelvic ligament;
D. Sacro-uterine ligament
E. Broad ligament

71. The microorganisms that are most often the causative


agents of inflammatory diseases of the female genital organs
of a non-specific etiology do not include:
1. staphylococci
2. streptococci
3. gonococci
4. gardnerella
5. anaerobes
72. Blood supply to the ovaries is carried out:
A. uterine artery;
B. ovarian artery;
C. ileo-lumbar artery;
D. Internal genital and ovarian arteries;
E. Uterine and ovarian arteries.

73. Ovarian cysts include:


A. dermoid cyst
B. follicular cyst
C. corpus luteum cysts
D. piovar
E. Theca-luteal cyst

74. With a slight bleeding from the ovary, detected by


laparoscopy, is performed:
A. suturing the ovary;
B. diathermocoagulation of the ovary under the control
of laparoscopy;
C. Ovarian resection
D. laparotomy and removal of the uterine appendages one
hundred

75. What is the most common complication in benign


ovarian tumors?
A. hemorrhage into the tumor cavity;
B. capsule rupture;
C. torsion of the legs of the tumor;
D. suppuration of contents;
E. Compression of adjacent organs.

76. The most informative in the differential diagnosis


between uterine myoma and ovarian tumor:
A. two-handed vaginal examination
B. hysteroscopy
C. ultrasound;
D. laparoscopy;
E. Probing of the uterine cavity.

77. With genital tuberculosis, the primary focus is most


often localized:
A. In the lungs
B. In bone
C. In the urinary system
D. In the lymph nodes
E. On the peritoneum

78.. What parts of the female reproductive system are most


often affected by tuberculosis?
A. fallopian tubes
B. Ovaries
C. Uterus
D. external genitalia
E. vagina

79. In what age period is tuberculosis of the internal genital


organs most often detected?
A. in childhood
B. puberty
C. in the reproductive period
D. in the premenopausal period
E. with the same frequency in any of the above periods
80. The main clinical symptom of tuberculosis of the uterus
appendages?
A. chronic pelvic pain
B. amenorrhea
C. Menometerorrhagia
D. infertility
E. DUB

81. During an operation for a paraovarial cyst:


A. cyst husking;
B. removal of appendages on the affected side;
C. removal of the ovary on the affected side;
D. Ovarian resection on the affected side;
E. Removal of the uterus with appendages

82. Blood test CA-125 for endometrioid ovarian cysts:


A. elevated
B. lowered
C. normal
D. negative
E. doesn't matter

83. What is the main clinical symptom of bacterial


vaginosis?
A. Itching of the external genitalia and perineum
B. dyspareunia
C. Profuse unpleasant whitewash
D. dysuria
E. pain in the lower abdomen
84. A characteristic feature of ovarian cysts:
A. increase due to the accumulation of liquid contents;
B. do not have capsules;
C. relate to malignant tumors of the female genital organs;
D. have invasive growth.
E. have fusion with neighboring organs

85. The following diagnostic method is used to recognize


ovarian tumors:
A. cytological
B. endoscopic
C. ultrasound
D. histological
E. bacterioscopic

86. For menopausal manifestations of a typical form of


menopause syndrome is most characteristic:
A. Dry mucous membranes
B. Pain in the heart
C. Osteoporosis
D. Laryngitis
E. Anemia

87. Specify precancerous changes in the vaginal part of the


cervix:
A. recurrent cervical canal polyp
B. true cervical erosion
C. Cervical dysplasia
D. cervical ectropion
E. Cervical ectopy

88. The main route of transmission of HPV:


A. lymphogenous
B. airborne
C. sexual
D. hematogenous
E. Contact Household

89. Itching of the vulva as a consequence of neuroendocrine


disorders is characteristic of:
A. candidiasis
B. trichomoniasis
C. krauroza
D. leukoplakia
E. erythroplakia
90. The screening method to detect cervical pathology in
modern conditions is:
A. visual inspection;
B. smear cytology;
C. vaginal smear on the flora
D. radionuclide method
E. PCR for HPV
91.. The main etiological factor for dysplasia and cervical
cancer is:
A. herpes simplex virus type 2;
B. human papillomavirus;
C. hyperestrogenia;
D. violation of the pH of the vaginal secretion;
E. immune and metabolic disorders in the body.

92. When conducting extended colposcopy after treatment


with a 3% solution of cervical acetic acid, the following
epithelial reaction is normal:
A. does not change;
B. turns pale;
C. evenly stained dark brown;
D. coated with white coating;
E. becomes embossed, papillae are visible in the form of
“grapes”.

93. The treatment of acute endometritis does not include:


A. detoxification and infusion therapy
B. glucocorticoids
C. antibiotics
D. desensitizing agents
E. Vitamins

94. A qualitative reaction (Schiller test) is due to the


interaction of iodine with cervical epithelium contained in
the multilayer:
A. glycogen;
B. proteins;
C. fats;
D. Ca salts;
E. immunoglobulins.

95. The differential diagnosis of acute salpingo-oophoritis


does not include:
A. acute appendicitis
B. tube abortion
C. colpitis
D. Ovarian apoplexy
E. Torsion of cyst legs

96. The main cause of adrenogenital syndrome (congenital


dysfunction of the adrenal cortex - CDAC) is:
A. Chronic inflammatory diseases of the ovaries;
B. Adrenal gland tumor;
C. Inherited deficiency of C21 - hydroxylase;
D. Decreased secretion of THG;
E. Hyperproduction of ACTH.

97. For bleeding from the genital tract in girls under the age
of 9, it is necessary:
A. Hormonal hemostasis
B. Observation
C. Appointment of hemostatic and uterine-contracting
agents
D. Exclusion of a local “organic” cause of bleeding
E. Hormone therapy

98. Specify the main criteria for PCOS:


A. weight loss;
B. Hyperandrogenism;
C. Normal ovulation;
D. Echographic signs in the uterus;
E. Polymenorrhea.

99. Indicate the universal diagnostic ultrasound criterion for


PCOS:
A. Increasing the thickness of the M-echo;
B. Increase in ovarian volume ≥ 7 cm2;
C. Increase in ovarian volume ≥ 5 cm3;
D. The presence of hyperplastic stroma;
E. At least 5 follicles along the periphery of the ovary d =
10mm.

100. Indicate typical clinical diagnostic signs of PCOS:


A. Polymenorrhea;
B. AMK;
C. Anorexia
D. Dysmenorrhea;
E. Infertility, chronic anovulation.

101. Hirsutism is ...


A. Excessive hair growth
B. Excessive male terminal hair growth
C. Excessive growth of nail plates
D. Overweight
E. Weight loss

102. Hormonal changes in PCOS are characterized by the


presence of:
A. A sharp increase in the secretion of FSH, LH;
B. Increases in TSH levels;
C. Decreased levels of testosterone, estrogen;
D. Increased testosterone levels, 17-OP;
E. Decreased prolactin levels.

103. Treatment of CDAC (congenital dysfunction of the


adrenal cortex) is carried out:
A. Regulon;
B. Femoston 1/5;
C. Dexamethasone;
D. clomiphene;
E. metformin

104. Polycystic ovary syndrome (PCOS) is characterized by:


A. Bilateral reduction of the ovaries;
B. Hyperandrogenism;
C. The ratio of LH / FSH is less than 2.5;
D. Habitual miscarriage;
E. AMK.
105. When identifying adrenogenital syndrome (AGS),
treatment must begin:
A. Since diagnosis
B. After the establishment of menstrual function
C. After marriage (depending on the time of the planned
pregnancy)
D. Only after childbirth
E. After menopause
106. Among various forms of ectopic pregnancy, tubal
pregnancy is
A. about 28%
B. about 48%
C. about 68%
D. about 88%
E. about 98%

107. In the pathogenesis of PCOS, there are:


A. Hypoprolactinemia;
B. Pituitary dysfunction;
C. Violation of the menstrual cycle;
D. Ovarian hyperandrogenism and decreased synthesis
of estradiol;
E. AMK.
108. Symptoms are observed in the clinical presentation
of PCOS:
A. Tachycardia;
B. Anorexia;
C. Polymenorrhea;
D. hirsutism;
E. Infertility.

109. For the diagnosis of PCOS, it is necessary to carry out:


A. Ultrasound, hormonal examination, biochemical
analysis of blood
B. MRI of the brain, mammography
C. Culdocentesis;
D. X-ray of OGK;
E. Hysteroscopy.
110. At the first stage of treatment for PCOS, it is
prescribed:
A. Low-calorie diet, metformin, lipase inhibitors;
B. Laparoscopic resection of 2/3 of the ovaries;
C. Clomiphene ovulation stimulator;
D. Monophasic COCs;
E. Progestogens in the second phase of the menstrual cycle.

111. CDAC (congenital dysfunction of the adrenal cortex) is


...
A. Hormone-active pituitary adenoma (corticotropinoma);
B. Genetically determined deficiency of 21-hydroxylase
involved in the synthesis of cortisol;
C. Androgen-producing adrenal gland tumor;
D. Adrenal hypoandrogenia;
E. Acromegaly.

112. The clinical forms of CDAC (congenital dysfunction of


the adrenal cortex) are:
A. Salting;
B. Vascular;
C. Asthenovegetative;
D. Edematous;
E. Total.

113. Hormonal changes in CDAC (congenital dysfunction


of the adrenal cortex)
A. Increased cortisol levels;
B. Increased levels of LH and FSH, the ratio of LH / FSH
more than 2.5;
C. Increased levels of 17-SNP, DHEA-S;
D. Decrease in testosterone and estradiol;
E. Decreased ACTH.
114. For the differential diagnosis of viril syndromes
(PCOS, CDAC) carry out:
A. ultrasound of internal organs;
B. Biochemical studies;
C. CT, MRI of the pituitary, adrenal gland;
D. A test with estrogen;
E. Hysterosalpingography.

115. Progressive tubal pregnancy can be reliably diagnosed


with
A. Bimanual research
B. Ultrasound Research
C. Curettage of the uterus
D. Puncture of the posterior arch
E. Serological Pregnancy Response
116. Adrenogenital syndrome
A. Has a synonym for adrenal hyperandrogenism
B. Manifested by hypermenstrual syndrome
C. May cause infertility
D. It is necessary to differentiate with menopausal syndrome
E. Treated HRT

117. To confirm adrenogenital syndrome (AGS) use


A. Determination of blood testosterone and 17
ketosteroids in urine
B. Probing of the uterus
C. Hysterosalpingography
D. Laparoscopy
E. Pelvic Scan
118. Pathogenetic treatment for adrenogenital syndrome
(AGS) is
A. Therapy aimed at normalizing body weight
B. Improving cerebral hemodynamics
C. Glucocorticoid therapy
D. Use of vegetotropic drugs (Belloid)
E. Use of small doses of thyroidin

119. In the development of polycystic ovary syndrome, the


following are essential:
A. Increased ovarian secretion of estrogen
B. Increased ovarian secretion of progesterone
C. Increased secretion compared to androgen
D. Increased secretion of prolactin
E. Decreased TSH secretion

120. Sterilization is carried out by:


A. Pipe compression;
B. Hysterosalpingography;
C. Tube ligation;
D. Hysteroscopy
E. Pipe removal.

121. For the diagnosis of polycystic ovaries, the following is


used:
A. General clinical laboratory tests
B. Functional tests with progesterone and
dexamethasone
C. Adrenal ultrasound
D. Laparotomy
E. Functional tests with estrogen

122. Which of the tests of functional diagnostics indicates


the presence of a two-phase menstrual cycle?
A. Pupil symptom
B. Karyopicnosis Index
C. Basal thermometry
D. Symptom of "fern"
E. Cervical index
123. A negative dexamethasone test (a slight decrease in the
excretion of 17-ACS and 17-KS) indicates the presence of:
A. Tumors of the adrenal cortex;
B. Adrenogenital Syndrome (AGS);
C. Neuro-exchange-endocrine syndrome;
D. Sclerocystic ovary syndrome.
E. Pituitary tumors

124. The main clinical signs in true premature puberty of the


central genesis
A. neurological symptoms,
B. intracranial hypertension
C. emotional disturbances
D. lack of menstruation until 15-16 years
E. weight gain

125. The full form of premature puberty is characterized by:


A. varying degrees of development of secondary sexual
characteristics and abcense of menstruation
B. bone age corresponds to the calendar
C. Body weight not elevated
D. lack of neurological symptoms
E. emotional disturbances
126. Features of the physique of girls with congenital
adrenogenital syndrome
A. narrow shoulders and wide pelvis
B. long limbs
C. tall
D. overweight
E. neurological symptoms

127. Causes of the viril syndrome in a girl


A. dysfunction of the adrenal cortex
B. masculinizing tumors of the adrenal cortex
C. Congenital androgenital syndrome
D. chromosome abnormalities
E. pituitary adenoma

128. During puberty, the following major changes occur in


the body:
A. Suppression of the pituitary gonadotropic function;
B. activation of hormonal function of the ovaries;
C. The rhythm of FSH release is not established;
D. regular “peaks” of LH excretion are established;
E. elevated prolactin levels

129. Nipple pigmentation and breast enlargement usually


occur:
A. at 8-9 years old;
B. at 10-11 years old;
C. at 12-13 years old;
D. at the age of 14-15;
E. at the age of 16-18.

130. The anatomical features of the uterus in a newborn girl


include:
A. The body of the uterus is small, the cervix is almost not
expressed;
B. the uterus is small, the length of the cervix is almost 3
times the length of the uterus;
C. The uterus has a bicorn shape;
D. The body of the uterus is almost 2 times larger than the
cervix;
E. Uterus curved posteriorly

131. The diagnosis of absolute female infertility can be


made:
A. in the absence of one fallopian tube
B. in the absence of the uterus
C. in PCA syndrome
D. during anovulatory cycles
E. with AGS syndrome

132. To exclude cervical factor of infertility use:


A. Shuvarsky-Huner test
B. Hysterosalpingography
C. Study of sex chromatin
D. Chromosomal analysis
E. Determination of Antisperm antibodies in the blood

133. The course of hydrotubations is shown:


A. with partial obstruction of the fallopian tubes
B. With obstruction of the tubes in the ampullar sections
C. With severe adhesions in the pelvis.
D. With an abundance of peritubular adhesions
E. In Asherman's Syndrome

134. Good patency of the fallopian tubes can be judged by


the data of chromohydrotubation in case urine
A. blue after one hour
B. Green in one hour
C. Green in two hours
D. After one hour, colorless
E. remains colorless after 24 hours

135. The frequency of male infertility is


A. 10-15%
B. 15-20%
C. 20-30%
D. 40-50%
E. 60-70%

136. Microsurgical operations on the fallopian tubes are


performed
A. with tube occlusion in various departments
B. In bilateral tubo-ovarian formations
C. With severe adhesions in the pelvis
D. With a duration of infertility of more than 10 years
E. In patients older than 35 years
137. Artificial insemination with donor sperm is used
A. with Rokytansky-Mayer-Küster syndrome
B. in women with anovulatory cycle
C. in women with Asherman syndrome
D. for male infertility
E. tubal infertility

138. Secondary infertility is called

A. Two-year infertility
B. Infertility in women with a history of pregnancy
C. Infertility due to endocrine pathology
D. Infertility due to inflammatory genesis
E. Husband Disease

139. What percentage of abnormal sperm is permissible in a


normal spermogram?
A. 1-2%
B. 6-8%
C. 10-15%
D. 20-40%
E. over 50%

140. What percentage of motile sperm is acceptable in a


normal spermogram?
A. 10%
B. 30%
C. 50-60%
D. 80%
E. 90%

141. The total number of sperm in a normal male ejaculate:


A. 20 - 40 million
B. 100 thousand - 1 million
C. 60 - 80 million
D. 10 - 20 million
E. 150 - 200 million

142. Reducing the number of antibodies to sperm in cervical


mucus can be achieved
A. Using oral contraceptives
B. taking antihistamines
C. Having more frequent coituses
D. Using condoms
E. Irrigating the cervix

143. The modern method of treating tubal infertility is:


A. Artificial insemination using sperm from a donor;
B. psychotherapy;
C. insemination;
D. In Vitro Fertilization
E. Stimulation of ovulation

144. When examining a sterile couple, it is primarily shown:


A. Hysterosalpingography
B. Cytology of the vaginal smear
C. Determination of basal temperature
D. Sperm examination
E. ECHO-GHA

145. The term "primary infertility" means that:


A. there was no pregnancy
B. no live births were born
C. All pregnancies are interrupted for medical reasons
D. a woman has no internal genital organs
E. All pregnancies are interrupted spontaneously

146. The synthesis of gonadoliberin is carried out in:


A. Anterior pituitary gland
B. The posterior pituitary gland
C. The nuclei of the hypothalamus
D. Neurons of the cerebral cortex
E. Middle lobe of the pituitary gland

376. 147. The following does not apply to endoscopic


research methods in gynecology:
A. Hysteroscopy;
B. Colposcopy;
C. Culdocentesis
D. Laparoscopy;
E. Culdoscopy.
148. With tubal-peritoneal infertility, the most effective
treatment is:
A. therapeutic laparoscopy;
B. laparotomy, microsurgical intervention on the pipes
(resection of the pipe section, the imposition of
anastomoses);
C. course of antibacterial and anti-inflammatory therapy;
D. enzyme preparations;
E. physiotherapeutic treatments.

149. The presence of ovulation does not determine:


A. functional diagnostic tests;
B. tests with LH;
C. ultrasound monitoring;
D. determination of progesterone levels in phase 2 of the
menstrual cycle;
E. FSH indicators

150. As a treatment method for infertility on the background


of intrauterine synechia does not include:
A. laparotomy, metroplasty;
B. hysteroresectoscopy;
C. hormonal treatment with estrogen-progestogen drugs;
D. antibacterial and anti-inflammatory treatment;
E. Stimulation of ovulation

151. At the first stage, in IVF programs:


A. embryo transfer;
B. transvaginal ovarian puncture;
C. maintaining the luteal phase;
D. stimulation of superovulation;
E. In vitro fertilization.
152. Sperm in the crypts of the cervical canal can retain the
ability to move (deadline) for
A. 6-12 hours
B. 24-48 hours
C. 3-5 days
D. 10 days
E. 1-2 hours

153. Sperm after penetration into the uterus and tubes


retain fertility for
A. 6-12 hours
B. 24-48 hours
C. 3-5 days
D. 10 days
E. 1-2 hours

154. After ovulation the egg remains fertile for


A. 6 hours
B. 12-24 hours
C. 3-5 days
D. 10 days
E. 1-2 hours

155. Positive test (small test) with dexamethasone


indicates that
A. Ovarian source of hyperandrogenism
B. Adrenal gland source of hyperandrogenism
C. Hyperandrogenism associated with pituitary adenoma
D. Hyperandrogenism due to adrenal corticosteroma
E. Hyperandrogenism due to ovarian cystoma

156. The therapeutic and diagnostic effect of dexamethasone


in hyperandrogenism is due to:
A. Inhibition of ovarian function
B. Inhibition of adrenal function
C. Inhibition of ACTH
D. Acceleration of androgen inactivation
E. Increased prolactin

157. The cause of infertility in gonadal dysgenesis is the


absence of
A. Ovarian tissue or its sharp underdevelopment
B. Uterus or its severe underdevelopment
C. gonadotropin production
D. Sensitivity of the normally formed ovary receptor
apparatus to gonadotropins
E. vaginal atresia

158. Ovarian hyperstimulation may occur.


A. When using clomiphene (clostilbegit)
B. When using pergonal
C. With prolonged use of combined estrogen-progestogen
drugs
D. When using radon baths
E. When using progestogens.
159. A woman asked you to pick a contraceptive. Single.
She lives sexually 2-3 times a month, and sometimes less
often. The partners are different. There were no pregnancies.
Your recommendations:
A. Intrauterine contraceptive
B. Condom
C. Emergency contraception
D. Oral contraceptives
E. Surgical sterilization

160. A married woman of 28 years, having one sexual


partner, suffering from chronic thrombophlebitis of the
veins of the lower extremities, the mother of one child,
needs:
A. Oral contraceptives
B. Surgical sterilization
C. Intrauterine contraception
D. Mechanical contraception
E. Barrier contraceptives

161. The composition of injectable contraceptives includes:


A. Long-acting progestogens
B. Conjugated Estrogens
C. Microdoses of progestogens
D. Antiandrogens
E. Mixed doses of hormones

162. For the purpose of contraception, the use of combined


estrogen-progestogen drugs begins:
A. During ovulation
B. On the eve of menstruation
C. From the 1st day of the menstrual cycle
D. Regardless of the day of the menstrual cycle
E. On the 7th day of the menstrual cycle

163. Combined estrogen-progestogen drugs are


contraindicated in:
A. At the age of 18 to 28 years
B. For the purpose of contraception
C. During breastfeeding and during pregnancy
D. For the purpose of treatment
E. At the age of 28-38 years

164. The Pearl Index is:


A. Percentage of contraceptive failure when using the
method during the year;
B. The ratio of heart rate and systolic blood pressure;
C. The ratio of heart rate and diastolic blood pressure.
D. The percentage of contraceptive failures when using the
method for 6 months.
E. The percentage of contraceptive failures when using the
method for 3 months.

165. What method of contraception refers to the natural


method of family planning:
A. Calendar;
B. Barrier;
C. Surgical;
D. Chemical;
E. Hormonal.

166. The optimal time for the introduction of the intrauterine


device (IUD):
A. 4-6 day of the menstrual cycle;
B. 14-16 day of the menstrual cycle;
C. During menstrual bleeding.
D. Before menstruation
E. 18-20 day of the menstrual cycle.

167. The composition of injectable contraceptives includes


A. Long-acting progestogens
B. Conjugated Estrogens
C. Microdoses of progestogens
D. Antiandrogens
E. Antigonadotropins

168. In the differential diagnosis of tubal pregnancy


A. History is usually not significant
B. Detection by histological examination of the
endometrium
The Arias-Stella phenomenon is indisputable evidence
C. A positive serological response to pregnancy is a reliable
sign
D. Leading role belongs to laparoscopy and ultrasound
E. Under any conditions, puncture of the posterior arch is
crucial
169. What are the frequent complications of taking
progestogens?
A. Allergic reactions
B. Intermenstrual spotting
C. Nausea, vomiting
D. Weight gain
E. Oligoamenorrhea

170. Contraindications to taking combined oral


contraceptives
A. Hypertension III stage
B. Women with miscarriage
C. Women with a history of ectopic pregnancy
D. Women with mild anemia
E. Algodismenorea

171. The diagnosis of impaired tubal pregnancy is an


indication
A. for emergency laparotomy
B. For laparoscopy
C. For ultrasound examination
D. For special studies
E. For puncture of the back arch

172. Voluntary sterilization - a mechanism of action.


A. thickening of cervical mucus
B. Blocking the patency of the fallopian tubes
C. Suppression of ovulation
D. Decreased uterine tone
E. Promotes adhesions in the pelvis.

173. Advantages of tubal occlusion


A. Irreversibility of the method
B. Protection against STIs
C. Regulation of the menstrual cycle
D. Increased libido
E. Thickening of cervical mucus

174. Mechanism of action for vasectomy


A. Blocking the patency of the vas deferens
B. Change in hormone production by the ovaries
C. Spermicidal effect
D. Decreased libido
E. Decreased vascular tone

175. Benefits of Condoms


A. Protection against STIs
B. High Efficiency
C. Influencing sexual feelings
D. Spermicidal effect
E. Contributes to thickening

176. Disadvantages of condoms


A. Protection against STIs
B. Lack of systemic side effects
C. Wide availability
D. Contraceptive effectiveness depends on the
willingness of the couple to follow the instructions.
E. High price

177. The mechanism of action of spermicides


A. Suppression of ovulation
B. Destruction of sperm membranes, which reduces their
motility and ability to fertilize the egg
C. Thickening of cervical mucus
D. Disruption of egg implantation
E. Toning the fallopian tubes

178. The mechanism of action of the method of lactational


amenorrhea (MDA)
A. Suppression of ovulation
B. Change in the structure of the endometrium
C. Implant Impaired
D. Thickening of cervical mucus
E. Endometrial hyperplasia

179. The high efficiency of the method of lactational


amenorrhea (MDA) with all the rules observed
A. Within 3 months after delivery
B. Within 6 months after delivery
C. Within 9 months after delivery
D. Within 12 months after delivery
E. Within 15 months after delivery

180. Contraception contraceptive against STI


A. COC
B. Navy
C. Barrier
D. Calendar method
E. Emergency contraception

181. The goals of family planning are:


A. Birth of wanted children
B. Reducing the prevalence of sexually transmitted
infections
C. Correction of menstrual irregularities.
D. Overcoming infertility
E. Decrease in maternal and perinatal mortality

182. Blastocyst implantation disorders:


A. Changes in the enzymatic system of the endometrium
B. Activation of peristalsis of the fallopian tubes
C. Lack of ovulation
D. Decrease in the tone of the fallopian tubes
E. Late ovulation

183. Disadvantages of the intrauterine device


A. Increased risk of inflammatory diseases
B. Effect on lactation
C. Short validity
D. A large number of side effects
E. Systemic effects on the body

184. Indications for use of the intrauterine device:


A. The need for long-term contraception and the
presence of one sexual partner
B. menstrual irregularities
C. Genital tract infections
D. Genital abnormalities
E. Adolescence

185. The complication most often arising on 3 5 days


after the introduction of the intrauterine device is:
A. Cervical insufficiency
B. Ectopic pregnancy
C. Habitual miscarriage
D. Uterine inflammation
E. Pelvic vein thrombosis

186. What are the most common complications in women


who have been using the intrauterine device for a long time
as a contraceptive?
A. Thrombophlebitis of the pelvic veins
B. Adhesive process in the pelvis
C. Inflammatory diseases of the internal genital organs
D. Cervical insufficiency
E. Amenorrhea
187. Visualization of an intrauterine contraceptive in the
cervical canal indicates:
A. Normal position of the intrauterine device
B. Low position of the intrauterine device
C. Uterine perforation
D. Intrauterine device expulsion
E. Pregnancy

188. The ovary is maintained in the abdominal cavity due to:


A. round ligament;
B. cardinal ligament;
C. funnel-pelvic ligament;
D. Sacro-uterine ligament
E. Broad ligament
189. Blood supply to the ovaries is carried out:
A. uterine artery;
B. ovarian artery;
C. ileo-lumbar artery;
D. Internal genital and ovarian arteries;
E. Uterine and ovarian arteries.

190. Ovarian cysts include:


A. dermoid cyst
B. follicular cyst
C. corpus luteum cysts
D. piovar
E. Theca-luteal cyst
191. Ovarian tumors that do not have hormonal activity
include:
A. granulosa cell tumor
B. dysgerminomas
C. Theca cell tumor
D. androblastoma
E. arrhenoblastoma

192. With a small bleeding from the ovary, detected by


laparoscopy, it is performed:
A. suturing the ovary;
B. diathermocoagulation of the ovary under the control
of a laparoscope;
C. Ovarian resection
D. Laparotomy and removal of the uterus on the affected
side
E. Hemostatic infusion therapy

193. The tumorous formations of the ovaries do not include:


A. dermoid cyst;
B. follicular cyst;
C. corpus luteum cysts;
D. pyovar;
E. Theca-luteal cysts.

194. What complication is most common with benign


ovarian tumors?
A. hemorrhage into the tumor cavity;
B. capsule rupture;
C. torsion of the legs of the tumor;
D. suppuration of contents;
E. Compression of adjacent organs.

195. The most informative in the differential diagnosis


between uterine myoma and ovarian tumor:
A. two-handed vaginal examination
B. hysteroscopy
C. ultrasound;
D. laparoscopy;
E. Probing of the uterine cavity.

196. The presence of ovulation can be judged by the results


of all the studies listed below:
A. Definitions of hCG
B. Ultrasound monitoring of the development of a
dominant follicle
C. Histological examination of endometrial scraping
D. Determination of the concentration of sex steroid
hormones in the blood on days 12-14 of the menstrual cycle
E. Follicle persistence

197. During surgery for a paraovarial cyst:


A. cyst husking;
B. removal of appendages on the affected side;
C. removal of the ovary on the affected side;
D. Ovarian resection on the affected side;
E. Removal of the uterus with appendages
198. Treatment of benign tumors of the ovary in childhood
and puberty is:
A. in the course of chemotherapy;
B. in the appointment of hormone therapy;
C. in bilateral removal of appendages;
D. in resection of the affected ovary;
E. in the supravaginal amputation of the uterus with
appendages;

199. An examination for an ovarian tumor may include:


A. X-ray of the gastrointestinal tract;
B. sigmoidoscopy;
C. hormonal colpocytology;
D. X-ray of the Turkish saddle;
E. phlebography;

200. The symptom complex characteristic of a granulosa


cell tumor of the ovary in menopausal women includes:
A. galactorrhea in menopause;
B. menopausal masculinization;
C. change in voice in menopause;
D. bleeding in menopause;
E. decreased libido;

201. Blood test CA-125 for endometrioid ovarian cysts:


A. elevated
B. lowered
C. normal
D. negative
E. doesn't matter

202. Prognosis for life in borderline ovarian tumors


A. Doubtful
B. adverse
C. poorly understood
D. relatively favorable
E. unknown

203. A characteristic feature of ovarian cysts:


A. increase due to the accumulation of liquid contents;
B. do not have capsules;
C. relate to malignant tumors of the female genital organs;
D. have invasive growth.
E. have fusion with neighboring organs

204. Paraovarial cysts are formed from residues:


A. omphalomesenteric duct;
B. paramesonephral duct;
C. Wolf duct
D. Muller duct;
E. allantois.

205. Each woman with a detected ovarian tumor should:


A. hospitalize urgently;
B. direct to separate diagnostic curettage;
C. put on dispensary records;
D. conduct hormone therapy;
E. carry out anti-inflammatory therapy.

206. Cervical pregnancy


A. Usually interrupted for 4-5 weeks
B. Interruption usually accompanied by internal bleeding
C. Diagnosed only during curettage of the uterine cavity
D. Can be diagnosed by the location of the external
pharynx of the cervical canal
E. In most cases, it is treated with vacuum aspiration of the
ovum.

207. Diagnostic methods most often used in the initial


detection of benign genital tumors in women
A. Cytological examination
B. gynecological examination, ultrasound
C. Ultrasound
D. pneumopelviography
E. Tumor puncture

208. Follicular cysts are most common:


A. in reproductive age;
B. in the neonatal period;
C. postmenopausal women;
D. in the juvenile period;
E. in the perimenopausal period.

209. A sign of the transition of the physiological process of


follicular maturation into a pathological follicular cyst is the
diameter of the fluid formation more:
A. 15 mm;
B. 20 mm;
C. 30 mm;
D. 35 mm;
E. 40 mm.

210. For tumor-like formations of the ovaries, it is


characteristic:
A. proliferation of the epithelium of the cellular elements of
the wall
B. lack of proliferation of the epithelium of the cellular
elements of the wall
C. consist of parenchyma and stroma
D. formed from germ cells
E. susceptible to metaplasia and paraplasia.

211. The most common ovarian tumors are:


A. cystadenomas;
B. endometrioid tumors;
C. Brenner's tumor;
D. granulosa cell tumor;
E. Mature teratoma.

212. Treatment of mucinous cystadenoma in reproductive


age:
A. course of anti-inflammatory therapy;
B. ovarian resection;
C. adnexectomy on the side of the affected ovary;
D. hysterectomy with affected appendages;
E. Combined oral contraceptives for 3–6 months.
213. The contents of the dermoid cyst:
A. hair, fat, cartilage;
B. mucus-like secret;
C. liquid, clear contents;
D. connective tissue;
E. blood.
214. Masculinizing tumor:
A. tecom;
B. androblastoma;
C. serous cystoma;
D. papillary.
E. fibroma
215. Endometriosis of the vaginal part of the cervix
A. refers to internal endometriosis
B. manifested by intense pain before and during
menstruation
C. rarely manifested as a violation of the nature of
menstruation
D. diagnosed by colposcopy
E. Good for conservative therapy

216. The vaginal part of the cervix in a woman of


reproductive age is normally covered:
A. cylindrical epithelium
B. stratified squamous epithelium
C. glandular epithelium
D. stratified squamous non-keratinizing epithelium
E. cubic epithelium
217. In the puberty, the vaginal part of the cervix has, as a
rule, the following features of the epithelial cover
A. covered with cylindrical epithelium
B. the junction of the stratified squamous and cylindrical
epithelium is located on the surface of the ectocervix, is
covered by a stratified squamous epithelium
C. the junction of the stratified squamous and cylindrical
epithelium is located on the surface of the endocervix, is
covered with stratified squamous epithelium
D. covered with stratified squamous epithelium
E. the junction of the stratified squamous and cylindrical
epithelium is located on the surface of the ectocervix,
covered with a cylindrical epithelium

218. Clinical manifestations of endometrial hyperplasia:


abnormal uterine bleeding;
A. asthenovegetative syndrome;
B. amenorrhea;
C. pain in the lower abdomen;
D. intoxication syndrome.
E. NMC

219. Differential diagnosis of endometrial polyp must be


performed with:
A. submucous uterine myoma;
B. subserous uterine myoma;
C. interstitial uterine myoma;
D. adenomyosis;
E. chorionic carcinoma.
220.. Side effects of agonists of gonadotropic releasing
hormones that limit the duration of use in the reproductive
period are:
A. osteoporosis;
B. impaired metabolism of fats and carbohydrates;
C. hyperthyroidism;
D. dyspepsia;
E. diarrhea.

221. The scope of the examination for pathology of the


cervix
A. colposcopy
B. hysteroscopy
C. Ultrasound
D. bacterioscopy
E. laparoscopy

222. Kraurosis and vulvar leukoplakia occur in women aged


A. 31–40 years old
B. 41–50 years old
C. 51-60 years
D. 61–70 years old
E. regardless of age

223. Uterine prolapse:


A. the bottom of the uterus is at the level of the plane of
entry into the pelvis;
B. the body of the uterus outside the genital gap, cysto and
rectocele;
C. The internal uterine pharynx is below the interspinal
line, the prolapse of the walls of the vagina;
D. the cervix is elongated, determined outside the genital
gap, cysto and rectocele;
E. The internal uterine pharynx is located above or at the
level of the interspinal line, the prolapse of the walls of the
vagina I degree.
224. Morphological changes with vulvar kraurosis are more
pronounced
A. in the epithelium
B. in the vessels of the vulva
C. Throughout the vulva
D. in the connective tissue of the vulva
E. in the basement membrane
225. For kraurosis of the vulva is not characteristic:
A. Puckering of the labia minora and labia minora
clitoral itching
B. dryness of the mucous membranes of the vulva
C. narrowing of the vaginal opening
D. vulvar edema

226. Hyperkeratosis of the epithelium of the vaginal part of


the cervix is ...
A. erythroplakia;
B. leukoplakia;
C. ectropion;
D. pseudo-erosion;
E. intraepithelial neoplasia.

227. The precancerous conditions of the cervix include:


A. erythroplakia;
B. leukoplakia without atypia;
C. ectropion;
D. pseudo-erosion;
E. Dysplasia of the stratified squamous epithelium.

228. What corrective hormone therapy is carried out with


DMK of the reproductive period:
A. Estrogens in the 1st phase of chicoa
B. Estrogen-progestogen drugs in a contraceptive
regimen;
C. Estrogen-progestogen drugs in the 1st phase of the cycle;
D. Estrogens in phase 2;
E. Gestagens continuously.

229. The main etiological factor for dysplasia and cervical


cancer is:
A. herpes simplex virus type 2;
B. human papillomavirus;
C. hyperestrogenia;
D. violation of the pH of the vaginal secretion;
E. immune and metabolic disorders in the body.

230. When conducting extended colposcopy after treatment


with a 3% solution of cervical acetic acid, the following
epithelial reaction is normal:
A. does not change;
B. turns pale;
C. evenly stained dark brown;
D. coated with white coating;
E. becomes embossed, papillae are visible in the form of
“grapes”.

231. A qualitative reaction (Schiller test) is due to the


interaction of iodine with cervical epithelium contained in
the multilayer:
A. glycogen;
B. proteins;
C. fats;
D. Ca salts;
E. immunoglobulins.

232. Incomplete prolapse of the uterus:


A. the bottom of the uterus is at the level of the plane of
entry into the pelvis;
B. the body of the uterus outside the genital gap, cysto and
rectocele;
C. The internal uterine pharynx is below the interspinal line,
the prolapse of the walls of the vagina;
D. the cervix is elongated, determined outside the genital
gap, cysto and rectocele;
E. The internal uterine pharynx is located above or at the
level of the interspinal line, the prolapse of the walls of the
vagina I degree.

233. Complete prolapse of the uterus:


A. the bottom of the uterus is at the level of the plane of
entry into the pelvis;
B. the body of the uterus outside the genital gap, cysto
and rectocele;
C. The internal uterine pharynx is below the interspinal line,
the prolapse of the walls of the vagina;
D. the cervix is elongated, determined outside the genital
gap, cysto and rectocele;
E. The internal uterine pharynx is located above or at the
level of the interspinal line, the prolapse of the walls of the
vagina I degree.

234. What are the leading clinical symptoms of dysplasia


and cancer of the cervical insitu:
A. pelvic pain;
B. mucopurulent leucorrhoea;
C. contact bleeding;
D. Acyclic uterine bleeding;
E. infertility.

235. In the etiology of pelvic organ prolapse, it does not


matter: Answer options:
A. traumatic birth;
B. estrogen deficiency;
C. the presence of tumors of the pelvic organs;
D. connective tissue dysplasia;
E. heavy physical labor.

236. For the treatment of cervical dysplasia of moderate and


severe degree do not apply:
A. local destruction of the cervical epithelium with acid
solutions;
B. radiosurgical excision;
C. electroconization;
D. Cone amputation of the cervix according to Sturmdorf;
E. Uterus extirpation.

237. Indicate the precancerous changes in the vaginal part of


the cervix:
A. recurrent cervical canal polyp
B. true cervical erosion
C. Cervical dysplasia
D. cervical ectropion
E. Cervical ectopy

238. To prevent the development of cervical endometriosis,


organ cryodestruction is performed on the following days of
the menstrual cycle
A. 1-2 days before the start of menstruation
B. immediately after the end of menstruation
C. on day 12-14
D. on day 16-18
E. on day 20-24

239. Itching of the vulva as a result of neuroendocrine


disorders is characteristic of:
A. candidiasis
B. trichomoniasis
C. krauroza
D. leukoplakia
E. erythroplakia
240. In the surgical treatment of a patient with kraurosis and
vulvar leukoplakia, the following is performed:
A. ovariectomy;
B. vulvectomy;
C. extended vulvectomy;
D. lymphadenectomy;
E. Uterus

241. Women with DMK are at risk:


A. Miscarriage
B. On the development of placental insufficiency
C. Development of genital tumors
D. On the development of anomalies of labor
E. Pregnancy

242. Hyperprolactinemia is:


A. Increase in blood gonadotropins
B. Increase in hemoglobin
C. Increase in BCC
D. Increased blood prolactin
E. Decreased blood prolactin levels

243. Cytological cervical screening is performed at the age


of:
A. 30 to 69 years old
B. 40 to 69 years old
C. 21 to 69 years old
D. from the age of menarche to 69 years
E. from the onset of sexual activity (after 21 years) to 69
years

244.. The earliest symptom of genital prolapse is:


A. stress urinary incontinence
B. recurrent vaginitis;
C. gaping genital fissure;
D. frequent urination;
E. foreign body sensation in the perineum.

245. To improve the effect of surgical treatment of genital


prolapse in elderly patients, the preoperative preparation
includes:
A. course of antibiotic therapy;
B. indirect anticoagulants;
C. topical estrogen preparations
D. a-GnRH for 3 months;
E. immunomodulators.

246. Indicate the most effective method for the early


diagnosis of postmenopausal osteoporosis:
A. Radiography of the lumbosacral spine
B. Mono- and biphoton absorptiometry
C. Computed tomography
D. Radiography of cysts
E. Joint ultrasound
247. Indications for the use of sounding of the uterine
cavity:
A. A suspected uterine perforation during curettage;
B. B acute endometritis;
C. C suspected presence of submucous uterine fibroids;
D. D cervical cancer;
E. E ectopic pregnancy;
248. In what cases, for diagnostic purposes, puncture of the
abdominal cavity through the posterior arch is indicated?
A suspicion of an ectopic pregnancy;
B suspected ovarian cancer;
C dysfunctional uterine bleeding;
D uterine fibroids;
E suspected ovarian apoplexy;

249. Research methods of the anatomical and functional


state of the vagina:
A inspection in the mirrors;
B combined vaginal rectal examination;
C cytological examination of the contents of the uterine
cavity;
D puncture of the posterior fornix;
E determination of the purity of the vaginal contents.

250. The most effective treatment for postmenopausal


osteoporosis is:
A. Diet Therapy
B. Physiotherapy and exercise therapy
C. Hormone therapy
D. Vitamin Therapy
E. Antibiotic therapy

251. The treatment of bartholinitis is:


A. Physiotherapy
B. Radio waves
C. Hormones
D. Vitamin Therapy
E. Antibiotic therapy

252. Treatment of false (ovarian) premature sexual


development:
A. Vitamin Therapy
B. Surgical
C. Hormonal
D. Antibiotic therapy
E. Physiotherapy
253. Uterine prolapse:
A. the bottom of the uterus is at the level of the plane of
entry into the pelvis;
B. the body of the uterus outside the genital gap, cysto and
rectocele;
C. The internal uterine pharynx is below the interspinal
line, the prolapse of the walls of the vagina;
D. the cervix is elongated, determined outside the genital
gap, cysto and rectocele;
E. The internal uterine pharynx is located above or at the
level of the interspinal line, the prolapse of the walls of the
vagina I degree.

254. Incomplete prolapse of the uterus:


A. the bottom of the uterus is at the level of the plane of
entry into the pelvis;
B. the body of the uterus outside the genital gap, cysto and
rectocele;
C. The internal uterine pharynx is below the interspinal line,
the prolapse of the walls of the vagina;
D. the cervix is elongated, determined outside the genital
gap, cysto and rectocele;
E. The internal uterine pharynx is located above or at the
level of the interspinal line, the prolapse of the walls of the
vagina I degree.

255. Complete prolapse of the uterus:


A. the bottom of the uterus is at the level of the plane of
entry into the pelvis;
B. the body of the uterus outside the genital gap, cysto
and rectocele;
C. The internal uterine pharynx is below the interspinal line,
the prolapse of the walls of the vagina;
D. the cervix is elongated, determined outside the genital
gap, cysto and rectocele;
E. The internal uterine pharynx is located above or at the
level of the interspinal line, the prolapse of the walls of the
vagina I degree.

256. The horizontal displacement of the uterus in the pelvic


cavity does not apply:
A. anterior displacement;
B. posterior displacement;
C. Turn of the uterus;
D. offsets to the left;
E. Offset to the right.
257. The prolapse of the walls of the vagina is accompanied
by the prolapse of the bladder and the anterior wall of the
rectum. What is the clinical degree of prolapse of the walls
of the vagina, uterus and their prolapse?
A. I;
B. II;
C. III;
D. IV;
E. V.
258. The genital gap is gaping, the anterior and posterior
walls of the vagina are slightly lowered. What is the clinical
degree of prolapse of the walls of the vagina, uterus and
their prolapse?
A. I;
B. II;
C. III;
D. IV;
E. V.

259. What complaint does the patient not have when


lowering the internal genital organs:
A. difficulty in emptying the bladder;
B. stress urinary incontinence;
C. difficulty with bowel movements;
D. a feeling that something is falling out;
E. on the mucous membranes from the vagina.

260. Squeezing the entrance to the vagina carries out:


A. transverse superficial muscle of the perineum;
B. muscle lifting the anus;
C. sciatic-cavernous muscle;
D. bulbous-cavernous muscle.
E. transverse deep muscle of the perineum

261. Polycystic ovary syndrome is:


A. Androgenic disease
B. Profuse uterine bleeding
C. Heterogeneous disease
D. Cyclic uterine bleeding
E. Lack of menarche

262. In the etiology of pelvic organ prolapse do not matter:


A. traumatic birth;
B. estrogen deficiency;
C. the presence of tumors of the pelvic organs;
D. connective tissue dysplasia;
E. heavy physical labor.
263. The earliest symptom of genital prolapse is:
A. stress urinary incontinence;
B. recurrent vaginitis;
C. gaping genital fissure;
D. frequent urination;
E. foreign body sensation in the perineum.

264. The most common cause of bleeding from the genital


tract in postmenopausal women is:
A. Cervical Cancer
B. Endometrial cancer
C. Submucous uterine fibroids
D. Ovarian Cancer
E. Cervical dysplasia

265. The main method for the diagnosis of dysplasia and


pre-invasive cancer of the vulva is:
A. Biopsy followed by histological examination
B. Vulvoscopy
C. Radioisotope study
D. Cytological examination of smear prints
E. Colposcopy
266. The most informative method for the diagnosis of
genital prolapse is:
A. ultrasound;
B. gynecological examination;
C. sigmoidoscopy;
D. cystoscopy;
E. hysteroscopy.
267. For the surgical treatment of genital prolapse, do not
use:
A. Manchester operation;
B. vaginal extirpation of the uterus;
C. use of mesh implants;
D. supravaginal amputation of the uterus;
E. shortening and strengthening of the ligamentous
apparatus of the uterus.
268. The choice of method for the correction of genital
prolapse does not depend on:
A. age of the patient;
B. severity of prolapse;
C. history of childbirth;
D. reproductive plans of the patient;
E. sexual activity.
269. Correction of asymptomatic genital prolapse of 1–2
degrees in a 38-year-old patient is carried out:
A. Pelvic floor muscle training
B. surgical method - vaginal extirpation of the uterus;
C. staging of the mesh implant;
D. staging of the urogynecological pessary;
E. Surgical method - over vaginal amputation of the uterus;

270. The main treatment for cancer and vulvar melanoma:


A. Remote radiation therapy
B. Chemotherapy
C. Hormone therapy
D. Surgical
E. Radio waves
271. The most effective screening test for early diagnosis of
cervical cancer:
A. Simple colposcopy
B. Bimanual rectovaginal examination
C. Cytological examination of smears from the surface of
the cervix and cervical canal
D. Vacuum curettage of the cervical canal
E. Pelvic ultrasound

272. The most informative method for the diagnosis of


cervical dysplasia:
A. Advanced colposcopy
B. Histological examination of the cervical biopsy
C. Cytological examination of smears from the surface of
the vaginal part of the cervix
D. Vacuum curettage of the cervical canal
E. Pelvic ultrasound
272. Reconstructive plastic surgery using a mesh implant for
pelvic organ prolapse is not performed when:
A. senile age of the patient;
B. recurrence of prolapse;
C. post hysterectomy prolapse (enterocele);
D. severe connective tissue dysplasia;
E. Hereditary factor of genital prolapse.

273. The most informative method for the diagnosis of


cervical dysplasia:
A. Advanced colposcopy
B. Histological examination of the cervical biopsy
C. Cytological examination of smears from the surface of
the vaginal part of the cervix
D. Vacuum curettage of the cervical canal
E. Pelvic ultrasound

274. To improve the effect of surgical treatment of genital


prolapse in elderly patients, the preoperative preparation
includes:
A. course of antibiotic therapy;
B. indirect anticoagulants;
C. topical estrogen preparations
D. a-GnRH for 3 months;
E. immunomodulators.
275. Before giving a woman an injection of Depo-Provera,
the health care provider must make sure that she does not
have:
A. Vaginal bleeding of unknown etiology
B. Cardiovascular Disease
C. History of STIs
D. Thromboembolic disorders
E. High blood pressure
276. What contraceptive methods are not recommended for
breastfeeding women:
A. Combined oral contraceptives
B. Condoms and spermicides
C. Navy
D. MLA
E. Purely progestin contraceptives

277. What method is not recommended for a 40-year-old


smoking woman:
A. Pure-progestin oral contraceptives
B. Depot-Provera
C. Combined oral contraceptives
D. Voluntary surgical sterilization
E. Navy

278. What should be done when spotting spotting occurs


during the installation of a copper-containing IUD:
A. Carry out a diagnostic curettage of the uterine cavity
B. Prescribe haemostatic and contractions
C. Remove IUD
D. Reassure a woman and prescribe a short course of
non-steroidal anti-inflammatory drugs
E. Refer to hospital

279. If a woman uses the method of lactational amenorrhea,


when any signals appear she needs to see a doctor to receive
recommendations on the use of another contraceptive:
A. The child is 3 months old
B. Maternal period resumes
C. Baby exclusive breastfeeding
D. Baby sleeps all night without feeding
E. 10 days of the postpartum period

280. The most common complication of taking progestin


drugs is:
A. Menstrual irregularities
B. Strokes, heart attacks
C. Weight gain
D. Violation of the blood coagulation system
E. Nausea, vomiting
281. The cytological equivalent of cervical dysplasia is:
A. Acanthosis
B. Discariosis
C. Hyperkeratosis
D. Cariolysis
E. Hypokeratosis
282. Characteristic features of uterine fibroids:
A. It is a benign, hormone-dependent tumor
B. Comes from the striated muscle of the tissue
C. Most common in puberty and women of early
reproductive age
D. Prone to malignancy
E. Malignant tumor

283. What method of contraception has the effect of double


protection
A. COC
B. Navy
C. Implants
D. Condoms
E. Depot-Provera

284. Depo-Provera is not recommended:


A. Women of all ages with more than 3 children
B. Women with migraine
C. Women with breast cancer at this time
D. Nulliparous women and adolescents
E. Women after abortion

285. Reliable information about contraceptives is provided:


A. From the personal experience of a specialist
B. Pharmaceutical companies
C. From evidence-based material
D. From the personal experience of the consumer
E. From the Internet
286. What can be done if a woman missed the next COC?
A. As soon as possible, take the missed active (hormonal)
tablet, then continue taking the tablets as usual
B. Do not refrain from sexual intercourse and do not use
additional contraceptives (condom) for the next 7 days
C. Do not resort to emergency contraception
D. Cancel birth control pills
E. Start taking pills from the next pack

287. A high risk of HIV infection or the presence of HIV


infection / AIDS is a contraindication for use:
A. Spermicides
B. Surgical sterilization methods
C. Navy
D. Combined oral contraceptives
E. Progestin-only contraceptives

288. Women with a history of deep vein thrombosis are not


recommended to use:
A. COC
B. Navy
C. Implants
D. Condoms
E. Depot-Provera

289. What qualities are not acceptable for a good consultant:


A. Competence and professionalism
B. Respect for Confidentiality
C. Use of personal preferences when choosing a method
of contraception
D. Friendliness and attentiveness
E. Listening skills
290. What method of contraception is least effective:
A. COC
B. Navy
C. Natural family planning methods
D. Condoms
E. Depot-Provera

291. The duration of the menstrual cycle when taking most


COCs is
A. 21 days
B. 28 days
C. 35 days
D. 32 days
E. 40 days

292. The most informative method for the diagnosis of


intermuscular uterine fibroids:
A. Vaginal examination
B. Ultrasound examination
C. Hysterosalpingography
D. Hysteroscopy
E. Colposcopy

293. One of the disadvantages of spermicides is:


A. Possible burning or itching in the vagina when used
B. The presence of estrogen-related side effects
C. Effect on breastfeeding
D. Need for pelvic examination before use
E. Progesterone-related side effects

294. The introduction of IUD as a postcoital method is


effective if, after unprotected intercourse, it has passed to:
A. 2days
B. 5days
C. 1week
D. 2 weeks
E. 1 month

295. Using Postinor as a post-coital method is effective if,


after unprotected intercourse, it has passed to:
A. 24 hours
B. 72 -120 hours
C. 10 days
D. 15 days
E. 1 month
296. Interrupted intercourse:
A. Highly effective contraceptive method
B. Ineffective contraception
C. STI Prevention Method
D. HIV Prevention Method
E. Ideal for continuous contraception
297. What conditions of the endometrium are classified as
precancerous?
A. Glandular cystic hyperplasia
B. Glandular endometrial polyp
C. Atrophy of the endometrium
D. Atypical hyperplasia
E. Fibrous endometrial polyp

298. Characteristic features of ovarian cysts:


A. These are retention formations
B. Increase in size due to cell proliferation
C. Do not have capsules
D. May be malignant
E. Multi-chamber

299. Which of the ovarian tumors most frequently


undergoes malignancy?
A. Fibroma
B. Mucinous cystadenoma
C. Serous cystadenoma
D. Tekoma
E. Sarcoma

300. What method of contraception is irreversible?


A. DMPA
B. Navy
C. Implants
D. COC
E. DHS
301. What can be used as emergency contraception:
A. Combined oral contraceptives
B. Interrupted intercourse
C. Depot Check
D. Voluntary surgical sterilization
E. Emergency contraception

302. The most appropriate time for a contraceptive after an


abortion:
A. Immediately after the procedure
B. The next menstrual cycle
C. 3 months after abortion
D. 6 months after abortion
E. Do not prescribe at all

303. The absolute indication for surgery in gynecology is


not:
A. intra-abdominal bleeding;
B. peritonitis;
C. gonorrheal pelvioperitonitis;
D. threat of perforation of the tubo-ovarian abscess.
E. Beginning miscarriage

304. An ectopic pregnancy can be localized in all of the


organs listed below, except:
A. vagina;
B. cervix;
C. rudimentary uterine horn;
D. ovary;
E. abdominal cavity.

305. The methods for diagnosing endometrial cancer are as


follows, except:
A. metrosalpingography;
B. separate diagnostic curettage of the uterine mucosa and
cervix;
C. functional diagnostic tests;
D. ultrasound examination of the pelvic organs;
E. hysteroscopy.
307. What is the most common morphological characteristic
of the endometrium prior to adenocarcinoma?
A. secretory transformation;
B. adenomatosis;
C. hyperplasia;
D. proliferation;
E. resting endometrium.

308. The main symptoms of adenocarcinoma, except:


A. uterine bleeding in postmenopausal women;
B. pain in the lower abdomen and in the lumbar region;
C. infiltration of the pelvic tissue;
D. decrease in the size of the body of the uterus;
E. enlargement and compaction of regional lymph nodes.

309. Additional methods for diagnosing the degree of spread


of adenocarcinoma, except:
A. cystoscopy;
B. bacteriological examination of uterine aspirate;
C. rectoscopy;
D. radiopaque lymphography;
E. Radioisotope lymphography.

310. The age period during which in situ carcinoma is most


often detected?
A. 45-55 years old;
B. 7-17 years old;
C. 30-40 years;
D. older than 60 -70 years;
E. older than 70 years.

311. What is the main treatment for chorionepithelioma?


A. antibiotic therapy;
B. immunostimulating therapy;
C. hysterectomy with appendages;
D. cytostatic therapy;
E. physiotherapeutic treatment.

312. Methods for diagnosing cervical pathology are used,


except:
A. Pap smear;
B. colposcopy;
C. cervicogysterosalpingography;
D. measuring rectal temperature;
E. Curettage of the cervical canal.

313. Factors contributing to the development of cervical


cancer, except:
A. hormonal contraception;
B. cicatricial deformity of the cervix after childbirth;
C. ectropion;
D. long-existing cervical erosion;
E. Cervical leukoplakia.

314. Regional metastasis of cervical cancer to the lymph


nodes, except:
A. external iliac;
B. common iliac;
C. Presacral;
D. paraaortic;
E. inguinal.

315. Treatment of preinvasive carcinoma of the cervix in a


young woman, except:
A. laser conization of the cervix;
B. hysterectomy with appendages;
C. intracavitary gamma therapy;
D. electroconization of the cervix;
E. knife conical excision of the cervix.
316. What research allows to establish the diagnosis of
invasive carcinoma of the cervix?
A. Pap smear;
B. aspiration of cervical mucus;
C. targeted biopsy of the neck with histological
examination;
D. flushing from the vagina;
E. colposcopy.
317. The most common cause of endometrial hyperplasia in
reproductive age?
A. inflammatory diseases of the genitals;
B. prolonged hyperestrogenism with anovulation;
C. hyperestrogenia in case of insufficiency of the luteal
phase of the menstrual cycle;
D. estrogen producing ovarian tumor;
E. Long-term use of estrogen.

318. Endometrial hyperplasia develops as a result of:


A. hyperprogesteronemia;
B. hyperestrogenemia;
C. hyperprolactinemia;
D. use of combined estrogen-progestogen drugs;
E. Genetically determined proliferation of basal endometrial
cells.

320. The histological differential diagnosis between in situ


carcinoma and invasive carcinoma is based on the following
symptoms:
A. damage to the basement membrane by atypical cells;
B. atypical cells are detected in a smear for oncocytology;
C. detection of cells with squamous metaplasia;
D. the entire layer of stratified squamous epithelium is
replaced by atypical cells;
E. pronounced proliferation of basal cells of the stratified
squamous epithelium.

321. With endometrial cancer, the lymph nodes are


primarily affected:
A. paracervical;
B. obstructive;
C. inguinal;
D. sacred;
E. hypogastric.

322. Treatment for stage II endometrial cancer includes:


A. only exposure;
B. extirpation of the uterus with appendages;
C. a combination of irradiation and hysterectomy with
appendages;
D. radical hysterectomy with resection of the greater
omentum;
E. only symptomatic therapy.

323. Uterine sarcoma can occur from the following tissues,


except:
A. myometrium;
B. nerve fibers;
C. endometrium;
D. blood vessels;
E. fibrous node.
324. The primary treatment for stage II ovarian cancer is:
A. immunotherapy;
B. remote exposure;
C. chemotherapy;
D. hormone therapy;
E. surgical method.
325. In a barren woman of reproductive age, histological
examination of the endometrium revealed adenomatosis.
Which drug will be more effective?
A. clostilbegit;
B. non-ovlon;
C. norkolut;
D. Pergon;
E. Chorionic gonadotropin.

328. The most appropriate sequence of measures in the


diagnosis of impaired ectopic pregnancy:
A. surgery, blood transfusion;
B. consultation of the therapist, anesthetist, surgery;
C. ultrasound, blood transfusion, surgery;
D. blood transfusion, surgery;
E. The use of hemostatic therapy, blood transfusion,
surgery.

330. When a vaginal examination in a patient with suspected


ectopic pregnancy revealed: external pharynx ajar; scarlet
spotting from the cervical canal; the uterus is enlarged to 8
weeks of pregnancy; appendages are not determined; the
vaginal arches are free. Diagnosis:
A. tube abortion;
B. impaired uterine pregnancy;
C. ovarian apoplexy;
D. inflammatory process of the uterus;
E. endometrial hyperplasia

331. Ovarian apoplexy often occurs:


A. during ovulation;
B. into the stage of vascularization of the corpus luteum;
C. during the maturation of the graaf follicle;
D. during atresia of the follicles.
E. During menstruation

332. With significant bleeding into the abdominal cavity in a


patient with ovarian apoplexy, it is shown:
A. laparotomy, ovarian resection;
B. gluttony, removal of the ovary;
C. dynamic observation of the doctor on duty, according to
indications - blood transfusion;
D. conservative therapy: rest, cold on the lower abdomen,
restorative therapy.
E. Hormone therapy;

333. In a patient with a clinical diagnosis of ovarian


apoplexy, the indication for surgery is:
A. history of appendage inflammation;
B. history of ovarian dysfunction;
C. pain syndrome;
D. intra-abdominal bleeding.
E. amenorrhea in the anamnesis.

335. Termination of tubal pregnancy by type of tubal


abortion occurs more often during pregnancy:
A. 11-12 weeks;
B. 9-10 weeks;
C. 7-8 weeks;
D. 4-6 weeks;
E. 16-17 weeks.
342. Risk factor for ectopic pregnancy?
A. uterine hypoplasia;
B. oral contraception;
C. transferred inflammatory diseases of the genitals;
D. a history of cesarean section;
E. Insufficiency of the luteal phase of the menstrual cycle.

343. What is the most common implantation of a fetal egg


during an ectopic pregnancy?
A. in the ampulla of the fallopian tube;
B. on the peritoneum;
C. on the ovary;
D. in the isthmic section of the fallopian tube;
E. in the interstitial section of the fallopian tube.

344. The least informative sign for the differentiation of


uterine and tubal pregnancy?
A. ultrasound examination of the pelvic organs;
B. level of chorionic gonadotropin in the blood;
C. bimanual examination of the pelvic organs;
D. smears for colpocytology;
E. Uterine curettage.

347. The main cause of death during ectopic pregnancy:


A. embolism of the lungs with trophoblast elements;
B. intestinal obstruction;
C. acute renal failure;
D. intra-abdominal bleeding;
E. peritonitis.

348. The diagnosis of ectopic pregnancy denies:


A. decidual endometrial reaction;
B. absence of fetal sac on echoscopy;
C. lack of a characteristic clinical picture;
D. negative test for chorionic gonadotropin;
E. Negative puncture of the abdominal cavity through the
posterior vaginal fornix.

349. The system of specialized gynecological care for


infertility includes the steps of:
A examination of women in antenatal clinics;
B examination and treatment of a married couple;
C in-patient examination and treatment;
D periodic medical examinations.
E women counseling

351. Which of the factors does not increase the risk of


developing inflammatory diseases of the genital organs?
A. the beginning of sexual activity at the age of 15;
B. medical abortion;
C. use of oral contraceptives;
D. hysterosalpingography;
E. Use of the IUD.

352. Choriocarcinoma - malignant neoplasm:


A. Decidual sheath
B. Myometrium
C. trophoblast
D. Theca tissue
E. Endometrium
353. Complaint, not typical for inflammatory diseases of the
genital organs:
A. pain in the lower abdomen;
B. fever;
C. fetid vaginal discharge;
D. high bilirubin in the blood;
E. Accelerate ESR and increase white blood cells.

354. What microorganisms causing colpitis require


treatment for both partners?
A. Trichomonas;
B. Candida;
C. streptococci;
D. staphylococci;
E. Escherichia coli.

355. Which of the following examination methods most


reliably confirm the diagnosis of inflammation of the
appendages?
A. quantification of white blood cells;
B. Gram stain of cervical mucus smear;
C. culdocentesis;
D. laparoscopy;
E. Ultrasound of the pelvic organs.
356. What is unnecessary in establishing a diagnosis of
inflammatory disease of the pelvic organs:
A. laparoscopy;
B. ultrasound;
C. culdocentesis;
D. urinalysis according to Zimnitsky;
E. rectal studies.

357. It is not a complication of inflammatory diseases of the


pelvic organs:
A. endometriosis;
B. ectopic pregnancy;
C. adhesions in the pelvic area;
D. dyspareunia;
E. hydrosalpinx.

358. In girls at an early age (from 2 to 8 years) are more


common:
A. ovarian tumors;
B. dysfunctional bleeding;
C. Congenital malformations of the genitals
D. vulvovaginitis;
E. salpingoophoritis.

359. The complication most often arising in the conduct of


the IUD is:
A. isthmic-cervical insufficiency;
B. ectopic pregnancy;
C. habitual miscarriage;
D. acute infection;
E. Pelvic vein thrombosis.

360. Pathological changes in cervical mucus can be the


result of all of the following conditions, except:
A. infection of the cervix by cytotoxic microorganisms;
B. posterior uterine displacement;
C. chronic inflammatory process in the cervix;
D. previous electrocoagulation of any formations of the
cervix;
E. vaginal inflammation.
361. Endometritis is:
A. inflammation of the fallopian tube;
B. uterine muscle inflammation;
C. inflammation of the peritoneum;
D. inflammation of the peritoneal tissue;
E. inflammation of the uterine mucosa.

362. Treatment of acute endometritis includes:


A. physiotherapy;
B. surgical treatment;
C. antibacterial agents;
D. diuretics;
E. antispasmodics.

363. Parametritis is:


A. inflammation of the ovary;
B. inflammation of the cecum;
C. inflammation of the fallopian tube;
D. inflammation of the peritoneal tissue;
E. inflammation of the omentum.

364. What treatment is not used for acute inflammation of


the uterine appendages of a non-specific etiology?
A. cold to the lower abdomen;
B. antibiotic therapy;
C. mud therapy;
D. vitamin therapy;
E. detoxification therapy.

365. What is the most common cause of atrophic colpitis?


A. oral contraception with progestins;
B. drug amenorrhea in the treatment of uterine fibroids or
endometriosis;
C. menopause;
D. oral contraception with progestogens;
E. Uterine fibroids

366. Factors of resistance of the vaginal mucosa to


infection?
A. high levels of androgens;
B. low estrogen levels;
C. acidic environment;
D. lack of Dederlein bacteria;
E. High progesterone.

367. Treatment of acute endometritis, except:


A. desensitizing agents;
B. surgical treatment;
C. antibacterial agents
D. antioxidant complex of vitamins;
E. Antispasmodics.

368. What disease is not differentiated in acute


inflammation of the pelvic organs?
A. acute appendicitis;
B. uterine fibroids;
C. acute urinary tract infection;
D. Lower lobar pneumonia
E. Torsion of the legs of the tumor.
369. The main diagnostic method for evaluating the
effectiveness of treatment of trophoblastic disease?
A. Dynamic transvaginal ultrasound
B. Computed tomography
C. Determination of the titer of chorionic gonadotropin
in blood serum and urine in dynamics
D. Hysteroscopy with separate diagnostic curettage
E. Determination of hemoglobin level

370. What least contributes to the development of the


inflammatory process in the pelvis?
A. curettage of the uterine cavity;
B. menstruation;
C. sperm;
D. endocervicitis;
E. fibrotic changes.
371. Atrophic colpitis may develop in the following
situations, with the exception of:
A. postmenopause;
B. premature depletion of ovarian function;
C. use of oral contraceptives;
D. pituitary necrosis;
E. Surgical castration at a young age.

572. The following complaint is characteristic of the onset


of acute inflammation of the uterus:
A. fever;
B. the appearance of rashes;
C. dyspeptic disorders;
D. polyphagy;
E. vomiting.

373. An indication for surgical treatment in inflammatory


processes of the uterine appendages is:
A. pyosalpinx;
B. perforation of purulent tubo-ovarian formation;
C. frequent exacerbations of the chronic inflammatory
process of the uterus;
D. endometriosis;
E. acute salpingitis.

374. The symptom most characteristic of vulvovaginitis:


A. sharp pains;
B. burning, itching
C. high temperature;
D. ulceration;
E. spotting.

375. Select the symptoms characteristic of pelvic peritonitis:


A. the appearance of rashes;
B. intestinal paresis;
C. a positive symptom of Shchetkin in the hypogastric
region;
D. abdominal distension;
E. enterocolitis.

376. Which statement of relative anaerobic sepsis is not


true:
A. more common with criminal termination of pregnancy;
B. may result from a hospital-acquired clostridial
infection;
C. accompanied by hemolysis of red blood cells;
D. early development of oligo- and anuria due to
hemoglobinuria;
E. Blood levels of total and unconjugated bilirubin are
elevated

377. For a clinic of septic shock are uncharacteristic:


A. drop in blood pressure;
B. oligo- and anuria;
C. icteric color of the skin precedes a drop in blood
pressure;
D. hyperthermia is replaced by hypothermia;
E. progressive DIC.

378. For emergency care in septic shock, it is not applicable:


A. paracetamol;
B. corticosteroids;
C. dopamine;
D. fresh frozen plasma;
E. Broad-spectrum antibiotics.

379. The most important risk factor for endometritis after


childbirth is:
A. frequent sex life;
B. caesarean section;
C. childbirth through the natural birth canal;
D. previous urinary tract infection;
E. associated upper respiratory tract infection.

380. Select characteristic complaints for acute endometritis:


A. pain throughout the abdomen;
B. pain radiating to the lower extremities;
C. fever;
D. nausea, vomiting;
E. dyspeptic disorders;
381. Parametritis occurs more often after:
A. childbirth, abortion
B. hypothermia;
C. accidental sexual intercourse;
D. ARVI
E. otitis media.

382. Select the research methods necessary for the diagnosis


of acute salpingo-oophoritis:
A. chest x-ray;
B. fluoroscopy of the abdominal organs;
C. urinalysis;
D. hysteroscopy;
E. laparoscopy.

383. Indicate an unfavorable period for surgery in case of


chronic inflammatory processes of the uterine appendages:
A. without exacerbation;
B. during an exacerbation;
C. fever is low-grade;
D. ESR more than 20 mm / hour;
E. The temperature is normal;

384. What parts of the female reproductive system are most


often affected by tuberculosis?
A. fallopian tubes;
B. ovaries;
C. uterus;
D. external genitalia;
E. vagina.

385. The main clinical symptom of tuberculosis of the


uterus appendages:
A. chronic pelvic pain;
B. amenorrhea;
C. menometerorrhagia;
D. primary infertility;
E. secondary infertility.
386. Rising gonorrhea is a defeat:
A. cervical canal;
B. fallopian tubes;
C. paraurethral glands;
D. urethra;
E. All of the above organs.

387. Clinical data characteristic of candidiasis:


A. Profuse curdled discharge from the genital tract,
itching and burning in the genital area;
B. Vaginal discharge with an unpleasant odor, plentiful,
gray-green, yellow, foamy; dysuria, itching, burning;
hyperemia, swelling, "raspberry neck", involvement in the
process of the vulva;
C. Mucous or mucopurulent discharge from the cervical
canal, often asymptomatic treatment;
D. Vaginal discharge is liquid, with an unpleasant "fishy"
smell, homogeneous adhering to the walls of the vagina, the
absence of an inflammatory reaction, itching and burning;
E. Drawing pains in the lower abdomen and lower back.

388. The diagnosis of gonorrhea can be established based


on:
A. detection in smears of microorganisms separated from
the genital tract located in pairs in the form of diplococci;
B. Gram negative cocci;
C. the location of bacteria inside the cell;
D. detection of any of the listed symptoms;
E. When establishing a diagnosis, a combination of all
the listed symptoms is necessary.
389. What is the main clinical symptom of bacterial
vaginosis:
A. itching of the external genitalia and perineum;
B. dyspareunia;
C. profuse leucorrhoea;
D. dysuria;
E. Pelvic pain.

390. The most common cause of ectopic pregnancy is


A. Genital infantilism
B. External genital endometriosis
C. Submucous uterine fibroids
D. Chronic salpingitis
E. Adenomyosis
391. The patient has been taking antibiotics for acute
pyelonephritis for a long time. She had a burning sensation
in the vagina, itching, copious discharge. What complication
arose?
A. acute endometritis;
B. ectopic pregnancy;
C. inflammation of the uterus;
D. candidal colpitis;
E. Cervical erosion.

392. An 18-year-old woman with a delay of menstruation


for 10 days had acute pain in the lower abdomen, fever up to
37.4 ° C, leukocytosis 12.4 109 / L. On palpation, the pain
spreads to the upper abdomen on the right. Differential
diagnosis is carried out with all of the following diseases,
except:
A. ectopic pregnancy;
B. appendicitis;
C. acute salpingitis;
D. colpitis;
E. Torsion of the legs of the ovarian tumor.

393. The following do not predispose to the development of


candidal vaginosis:
A. oral contraceptives;
B. pregnancy and diabetes;
C. taking antidepressants;
D. antihypertensive drugs
E. diuretics

394. What disease should be suspected if vaginal yeast


infection often recurs?
A. anemia;
B. diabetes mellitus;
C. systemic lupus erythematosus;
D. genital endometriosis;
E. Congenital adrenal hyperplasia.

395. Bacterial vaginosis is not characterized by the


following manifestation:
A. pH 5.0;
B. key cells;
C. severe inflammatory reaction;
D. an increase in the number of gardnerella;
E. Good effect from metronidazole treatment.
396. Ectopic pregnancy should not be differentiated:
A. with salpingitis;
B. with abortion;
C. with torsion of the cystoma legs;
D. with hemorrhage in the corpus luteum;
E. with endometritis;

397. The main clinical symptoms of pipe rupture do not


include:
A. sharp paroxysmal pain in the lower abdomen;
B. short-term loss of consciousness;
C. positive phrenicus symptom;
D. pallor of the skin, cold sweat.
E. Increase in body temperature;

398. Treatment measures for abortion should include:


A. observation against antibiotic therapy;
B. laparotomy after the development of a clinic of intra-
abdominal bleeding;
C. the appointment of drugs that enhance blood
coagulability;
D. immediate laparotomy and removal of the tube;
E. hormonal hemostasis.

399. The risk factors for ectopic pregnancy are not:


A. inflammatory diseases of the pelvic organs;
B. surgical interventions on the pelvic organs;
C. Tubal ligation
D. spontaneous abortion;
E. Herpes simplex virus infections.
401. At what daily dose is heparin used to prevent
thromboembolic complications?
A. 10 thousand units;
B. 20 thousand units;
C. 30 thousand units;
D. 15 thousand units.;
E. 5 thousand units.;

402. Preovulatory changes in the hormonal background are


characterized by an increase in the level
A. LH and decreased FSH
B. FSH and decreased LH
C. FSH and LH
D. prolactin
E. FSH, LH and prolactin

403. In the first phase of the menstrual cycle


A. LH secretion increases gradually, reaching maximum
values
B. endometrial glands become sawtooth
C. decreases the number of estradiol-binding receptors
D. there is an overgrowth of granulosa cells of the follicle
E. pupil symptom becomes "negative"

405. In the second phase of the menstrual cycle


A. There is a second peak in estrogen production
associated with the flowering of the corpus luteum
B. There is a pronounced proliferation of follicle granulosis
C. Intensive proliferation of the endometrial functional layer
occurs.
D. mucus crystallization symptom reaches maximum
severity
E. Basal temperature rises by O.2 degrees

406. The maturity of the follicle can be determined


A. blood FSH level
B. blood progesterone level
C. urinary pregnanediol level
D. ultrasound examination
E. basal temperature

407. Cervical index


A. allows you to diagnose the moment of ovulation
B. is rated on a 3-point system
C. determined during colposcitological examination
D. takes into account the shape of the vaginal cervix
E. allows you to assess the level of estrogen saturation

408. Dysfunctional uterine bleeding in the reproductive


period due to
A. impaired ectopic pregnancy
B. endometriosis
C. polycystic ovary syndrome
D. inflammatory process of the endometrium
E. Disorder of the mechanism of regulation of the
menstrual cycle
409. Dysfunctional uterine bleeding with persistence of the
follicle proceeds against the background
A. High estrogen saturation
B. low estrogen saturation
C. High gestagenic saturation
D. androgenic saturation
E. glucocorticoid deficiency

410. Dysfunctional uterine bleeding in menopause due to


A. Disorder of the circhoral production of gonadotropin
releasing hormone (GnRH)
B. adenomyosis
C. adenomatosis
D. Atrophic processes in the endometrium
E. Immunodeficiency

411. Dysfunctional uterine bleeding in the juvenile period


includes bleeding
A. for blood diseases
B. with cardiovascular disease
C. with thyroid disease
D. for adrenal disease
E. Disorder of the formation of circhoral production of
gonadotropin releasing hormone

412. Dysfunctional uterine bleeding with follicular atresia


occurs against
A. High estrogen saturation
B. low estrogen saturation
C. High gestagenic saturation
D. Hyperandrogenemia
E. Prolactin overproduction

413. Ovulatory acyclic bleeding characteristic


A. with short-term rhythmic persistence of the follicle
B. with prolonged persistence of the follicle
C. with follicular atresia
D. with persistence of the corpus luteum
E. in case of corpus luteum insufficiency

414. Anovulatory cyclic bleeding characteristic


A. with prolonged follicular persistence
B. with short-term persistence of follicles
C. with persistence of the corpus luteum
D. in case of corpus luteum insufficiency
E. with follicular atresia

415. Hypoplastic endometrium is characteristic


A. for ovarian hypofunction
B. for dysfunctional uterine bleeding in menopause
C. For ectopic pregnancy
D. for endometrial precancer
E. for tecom and granulosa cell tumors of the ovary

416. The condition of the endometrium with follicular


atresia is characterized by
A. glandular hyperplasia
B. atypical glandular hyperplasia (adenomatosis)
C. High glycogen
D. Decidual changes
E. The emergence of large Arias-Stella cells

417. The condition of the endometrium with persistence of


the follicle is characterized by the presence of
A. the light glands of Overbeck
B. adenocontoms
C. adenomatosis
D. adenomyosis
E. glandular cystic hyperplasia

418. For the treatment of bleeding with persistence of the


corpus luteum, apply
A. curettage of the uterus
B. cervical electrical stimulation
C. gestagens
D. danazol
E. Parlodel

419. Indications for hysteroscopy:


A. Pregnancy
B. Suspicion of submucous uterine fibroids
C. Heavy uterine bleeding
D. Cervical stenosis
E. Common Cervical Cancer

420. The diagnosis of torsion of the cystoma legs confirms:


A. sudden abrupt onset of pain;
B. tension of the anterior abdominal wall;
C. positive symptom of Shchetkin-Blumberg;
D. detection of a sharply painful tumor in the pelvis;
E. leukocytosis, accelerated ESR.

421. Acute inflammation of the uterine appendages of


gonorrheal etiology
A. usually develops within the first week after infection
B. often complicated by pelvioperitonitis
C. is an indication for surgical treatment
D. diagnosed with hysterosalpingography
E. is an indication for the administration of ampicillin in a
course dose of 3.5 g

422. The diagnosis of gonorrhea can be made.


A. with a positive Borde-Zhangu reaction
B. if bilateral inflammation of the fallopian tubes is detected
C. In cases of concomitant inflammation of the urethra and
cervical canal
D. In cases of gonococcus detection
E. in cases of fever up to 38 degrees in response to
administration of gonovaccine

423. Endometritis is:


A. Inflammation of the vulvar mucosa
B. Inflammation of the uterine lining
C. Cervical inflammation
D. Inflammation of the fallopian tubes
E. Ovarian inflammation
424. A feature of the course of gonorrhea in women is
A. the presence of pronounced clinical symptoms with
lesions of the urethra and cervical canal
B. the presence of foamy discharge
C. bilateral lesion of the uterus
D. the lack of connection between certain stages of the
development of the disease and menstruation, childbirth,
abortion
E. Frequent development of parametritis

425. The criterion for cured gonorrhea in women is the


absence of gonococci in smears taken
A. during follow-up examinations using physiological
and combined provocations for three months
B. after three monthly nutritional provocations
C. after three series of monthly physical provocations
D. after a series of intramuscular injections of increasing
doses of gonovaccine
E. after completion of treatment

426. The rapid increase in the size of fibroids.


A. may be due to malignant degeneration of the node
B. usually associated with necrosis of the node
C. may be a symptom of endometrial cancer
D. is an indication for more active conservative therapy
E. always accompanied by pain

427. Conservative treatment of uterine fibroids


A. includes the use of diathermy
B. based on long-term use of androgens in a continuous
mode
C. can be carried out by preparations of a norsteroid
row
D. Based on long-term use of vitamin B12
E. includes chemotherapy

428. Common forms of ectopic pregnancy include


A. tubal pregnancy
B. Ovarian pregnancy
C. abdominal pregnancy
D. Interconnected pregnancy
E. Pregnancy in the rudimentary horn

429. Progressive tubal pregnancy can be reliably diagnosed


with
A. bimanual research
B. ultrasound examination
C. Curettage of the uterus
D. puncture of the posterior fornix
E. serological pregnancy response

430. In the differential diagnosis of tubal pregnancy:


A. History data are usually not significant
B. The detection of an Arias-Stella phenomenon in
histological examination of the endometrium is indisputable
evidence.
C. A positive serological response to pregnancy is a reliable
sign
D. Leading role belongs to laparoscopy and ultrasound
E. Under any conditions, puncture of the posterior fornix is
crucial
431. Inflammation of the perinatal fiber is called:
A. Peritonitis
B. Endometritis
C. Vaginitis
D. Parametritis
E. Vulvit

432. A test with clomiphene is performed if:


A. Uterine fibroids
B. Anovulation
C. Adenomyosis
D. Endometritis
E. Hyperprolactinemia

433. The diagnosis of impaired tubal pregnancy is an


indication
A. for emergency laparotomy
B. for laparoscopy
C. for ultrasound examination
D. for special studies
E. for puncture of the back arch

435. The pelvic floor is:


A. Hymen;
B. vagina;
C. Muscles and fascia of the perineum;
D. The vestibule of the vagina.
E. Douglas space
436 Cervical pregnancy
A. usually interrupted for 4-5 weeks
B. interruption usually accompanied by internal bleeding
C. Diagnosed only during curettage of the uterus
D. can be diagnosed by the location of the external
pharynx of the cervical canal
E. In most cases, can be treated with vacuum aspiration of
the ovum.

437. Cervical cancer


A. diagnosed with a specific Schiller test
B. more common in nulliparous women
C. found only in exophytic form
D. in the early stages manifested by acyclic bleeding
E. in the pre-invasive stage may be an indication for
cervical electroexcision

438. The acidic environment of the vagina is provided by


the presence of:
A. Epithelium of the vagina;
B. white blood cells;
C. The wand of Dederlein;
D. Gonococcus.
E. Kokkov

439. If pathological changes in the cervix are detected, it is


necessary:
A. Take a smear from the changed area for cytological
examination;
B. Treat the neck with a disinfectant solution;
C. Surveillance with periodic examinations.
D. Take a smear on the flora
E. PCR analysis

440. Dysplasia
A. This is a pathology of the integumentary cervical
epithelium, in the entire thickness of which there are
histological signs of cancer, but there is no invasion of the
underlying stroma
B. does not apply to background diseases of the cervix
C. may be detected by a special study of a visually
unchanged cervix
D. is an indication for hysterectomy at any age
E. is usually treated with electrocoagulation of the affected
areas of the cervix

441. Endometriosis of the vaginal part of the cervix


A. refers to internal endometriosis
B. manifested by intense pain before and during
menstruation
C. rarely manifested as a violation of the nature of
menstruation
D. diagnosed by colposcopy
E. Good for conservative therapy

442. Internal endometriosis


A. diagnosed by colposcopy
B. rare
C. develops from the basal layer of the endometrium
D. Well amenable to hormone therapy
E. not detected by hysteroscopy

443. Relates to internal genital endometriosis.


A. endometriosis of the vaginal cervix
B. peritoneal endometriosis
C. Ovarian endometriosis
D. endometriosis of the interstitial uterine tubes
E. retrocervical endometriosis

444. Retrocervical endometriosis


A. may be manifested by pain, flatulence, delayed stool
on the eve and during menstruation
B. Impossible to diagnose with bimanual examination
C. diagnosed with hysterography
D. diagnosed with pneumopelviography
E. is treated only by surgery

445. The differential criterion that determines stage II of


cancer

A. ovary (FIGO classification, 1976), is


B. ascites
C. capsule rupture
D. hydrothorax
E. damage to the second ovary
F. Damage to the fallopian tube and uterus
446. A sign defining the III stage of ovarian cancer (FIGO
classification, 1976) is
A. ascites
B. capsule rupture
C. damage to the second ovary
D. damage to the uterus
E. the presence of metastases in the omentum

447. The most important additional method of preoperative


diagnosis of the nature of the tumor process in the ovary is
A. ultrasound examination
B. Bi-Contrast Pelviography
C. pelviotomography
D. lymphography
E. Cytological examination of punctate from the
abdominal cavity

448. The treatment of benign tumors of the ovary in


childhood and puberty is
A. in the course of chemotherapy
B. in the appointment of hormone therapy
C. Bilateral removal of appendages
D. in resection of the affected ovary
E. in the supravaginal amputation of the uterus with
appendages

449. The <choice operation> in benign ovarian tumor in pre-


and postmenopausal women is
A. Ovarian resection
B. removal of the appendages from the lesion
C. Bilateral removal of appendages
D. supravaginal amputation of the uterus with
appendages
E. Subvaginal uterine amputation with appendages +
omentectomy

450. The <operation of choice> in ovarian cancer is


Wertheim operation
A. expanded hysterectomy with removal of paraaortic
lymph nodes
B. Extirpation or supravaginal amputation of the uterus
with appendages and omentectomy
C. hysterectomy with appendages
D. Removal of affected appendages
E. Subvaginal uterine amputation with appendages +
omentectomy

451. Risk factor for ectopic pregnancy?


A. uterine hypoplasia;
B. oral contraception;
C. Pelvic inflammatory disease
D. a history of cesarean section;
E. Insufficiency of the luteal phase of the menstrual cycle.

452. The leading method of complex treatment of malignant


ovarian cancer is
A. chemotherapy and surgical treatment
B. surgical treatment + radiotherapy
C. X-ray and hormone therapy
D. hormone therapy and surgical treatment
E. chemo-hormone therapy
453. Most malignant tumors of the ovary are
A. serous tumors
B. Mucinous tumors
C. Tumor stroma tumors
D. teratomas
E. metastatic tumors

454. Secondary infertility is called


A. Two-year infertility
B. Infertility in women with a history of pregnancy
C. Infertility due to endocrine pathology
D. Infertility due to inflammatory genesis
E. Husband Disease

455. The good patency of the fallopian tubes can be judged


by the data of chromohydrotubation in case urine
A. Blue in one hour
B. Green in one hour
C. Green in two hours
D. After one hour, colorless
E. Remains colorless after one hour and after 24 hours

456. According to the WHO classification (198O),


premenopause is called the period
A. From the end of reproductive age to the onset of
menopause
B. The length of time after the last menstruation
C. Last menstruation
D. Period of menstrual dysfunction until the last
menstruation
E. The Period Before the Menarche

457. According to the WHO classification (198O),


menopause is called
A. Period of stable menstrual function
B. Last menstruation
C. The length of time after the last menstruation
D. Time span after reproductive age
E. The onset of the first menstruation

458. According to the WHO classification (198O),


postmenopause is called
A. Last menstruation
B. The period from the onset of menstruation to the last
menstruation
C. Period of stable menstrual function
D. Period of first menstruation
E. The length of time after the last menstruation

459. Currently, age is considered normal for menopause.


A. 45 years
B. 5O years
C. 4O years
D. 55 years
E. 43 years
460. The autonomic-nervous manifestations of menopausal
syndrome include
A. Irritability
B. Sweating
C. Sleep Disorders
D. Decrease in memory
E. Decreased libido

461. The pathogenetic treatment of adrenogenital syndrome


(AGS) is
A. Therapy aimed at normalizing body weight
B. Improving cerebral hemodynamics
C. Glucocorticoid Therapy
D. Use of vegetotropic drugs (Belloid)
E. Use of small doses of thyroidin

462. To confirm Shereshevsky-Turner syndrome


A. Transfenoidal Arteriography
B. Determination of gonadotropin hormones
C. Ultrasound scan of the pelvic organs or
pneumopelviography
D. Hysterography
E. Karyotype study

463. Rokytansky-Mayer-Küster syndrome is an indication


A. For long-term parlodel therapy
B. For demedulation of the ovaries
C. For plastic surgery on the uterus
D. For colpopoiesis
E. For surgery on the pituitary gland

464. Sheehan syndrome


A. Develops with pituitary tumors
B. May be due to massive blood loss during childbirth
C. Characterized by hyperprolactinemia
D. Accompanied by hyperthyroidism
E. Accompanied by hypercorticism

465. When hyperprolactinemia is used


A. Diphenin
B. Pergonal
C. Clomiphene
D. Parlodel
E. Danazole

466. The most important role in the formation of prolapse


and prolapse of the walls of the vagina and uterus belongs to
traumatic injuries.
A. vaginal mucosa
B. wide uterine ligaments
C. Sacro-uterine ligaments
D. round uterine ligaments
E. Pelvic floor muscles

467. Uterine prolapse


A. not found in nulliparous women
B. may be associated with a sedentary lifestyle
C. It is considered complete if the cervix extends beyond the
genital gap
D. may be complicated by the development of decubital
ulcers
E. is an indication for Emmett's operation

468. The uterine suspension does not include


A. Own ligaments of the ovaries
B. round uterine ligaments
C. wide uterine ligaments
D. Suspending ovarian ligaments
E. Sacro-uterine ligaments

469. The most common chorionic carcinoma occurs after:


A. abortion
B. cystic drift
C. Normal birth
D. premature birth
E. inflammatory processes
470. Management tactics for patients with DMC of the
juvenile period:
A. limit yourself to symptomatic hemostatic and antianemic
therapy
B. carry out hormonal hemostasis with progesterone
C. Treatment and diagnostic curettage of the endometrium
and endocervix
D. complex therapy, including hemostatic, antianemic,
uterotonic therapy, with inefficiency - hormonal
hemostasis
E. hysteroscopy
471. The main clinical symptom of submucous uterine
fibroids:
A. chronic pelvic pain;
B. Algodismenorea
C. menorrhagia
D. leucorrhoea
E. secondary infertility

472. Conservative treatment of uterine fibroids


A. Includes diathermy
B. Based on long-term use of androgens in a continuous
mode
C. can be carried out by preparations of a norsteroid
row
D. Based on long-term use of Vitamin B12
E. includes chemotherapy

473. For anovulatory menstrual cycle is characteristic


A. cyclic changes in the body
B. prolonged persistence of the follicle
C. Prevalence of gestagens in the second phase of the cycle
D. prevalence of gestagens in the first phase of the cycle
E. estrogen deficiency

474. The following does not apply to endoscopic research


methods in gynecology:
A. hysteroscopy
B. colposcopy
C. culdocentesis
D. laparoscopy
E. Culdoscopy

475. Obstetric peritonitis most often occurs after:


A. childbirth
B. early spontaneous miscarriage
C. Cesarean section
D. Artificial abortion
E. Late spontaneous miscarriage

476. The hypothalamus produces the following hormones


A. gonadotropins
B. estrogens
C. gestagens
D. releasing factors
E. glucocorticoids

477. With exacerbation of chronic salpingoophoritis by the


type of pelvic nerve neuralgia, the effect does not have:
A. antibiotic therapy
B. electrophoresis of amidopyrine
C. Diadynamic currents
D. ultraviolet erythema
E. amplipulse therapy

478. Low doses of estrogen


A. stimulate FSH production
B. inhibit FSH production
C. enhance LH production
D. inhibit LH production
E. do not affect FSH production

479. Hysterosalpingography in the diagnosis of internal


uterine endometriosis is the most informative:
A. 1-2 days before the start of menstruation
B. immediately after menstruation
C. on day 12-14
D. on day 16-18
E. on day 20-22

480. The complication most often arising with the


introduction of the IUD is:
A. ischemic cervical insufficiency
B. ectopic pregnancy
C. perforation
D. acute infection
E. Pelvic vein thrombosis

481. In the differential diagnosis between uterine myoma


and ovarian tumor, the most informative:
A. two-handed vaginal examination
B. Ultrasound
C. Puncture of the posterior fornix.
D. laparoscopy
E. Probing of the uterine cavity

482. When examining a sterile couple, it is primarily shown:


A. hysterosalpinography
B. vaginal smear cytology
C. determination of basal temperature
D. endometrial biopsy
E. sperm examination

483. If you suspect a malignant lesion of the ovary in a


patient 55 years old, it is shown:
A. Removal of the uterus on the affected side
B. supravaginal amputation of the uterus with
appendages and resection of the greater omentum
C. hysterectomy with appendages
D. Removal of the uterus with appendages on both sides
E. Subvaginal uterine amputation with appendages

484. Tactics at the clinic of “acute abdomen” at the


prehospital stage:
A. Anesthesia
B. Cold on the stomach;
C. Cleansing enema;
D. Urgent hospitalization
E. Observation

485. When acyclic spotting appears:


A. hysterosalpingography
B. definition of PH
C. Ultrasound
D. definition of HCG
E. Diagnostic curettage

486. Amenorrhea is the absence of menstruation during


A.4 months
B.5 months
C.6 months
D.2 months
E.3 months

487. The main method for stopping abnormal uterine


bleeding in the premenopausal period is:
A. use of synthetic estrogen-progestogen drugs
B. The introduction of hemostatic and uterine-contracting
agents
C. Use of androgens
D. continuous use of 17-hydroxyprogesterone capronate
(17-OPK)
E. Separate diagnostic curettage of the mucous
membrane of the uterine cavity and cervical canal

488. Which parts of the female reproductive system are


most often affected by tuberculosis:
A. fallopian tubes
B. Ovaries
C. Uterus
D. external genitalia
E. vagina

489. The main clinical symptom of tuberculosis of the


uterus appendages:
A. chronic pelvic pain
B. amenorrhea
C. Menometerorrhagia
D. infertility
E. NMC
490. What is the main clinical symptom of bacterial
vaginosis:
A. Itching of the external genitalia and perineum
B. dyspareunia
C. Profuse discharge whitewash
D. dysuria
E. Pelvic pain

491. The main clinical symptom of submucous uterine


fibroids:
A. chronic pelvic pain
B. Algodismenorea
C. Uterine bleeding
D. secondary infertility
E. iron deficiency anemia

492. In the chain of steroid biosynthesis, the first


biologically active
hormone is
A. androstenedione
B. estradiol
C. estriol
D. testosterone
E. progesterone
493. The main estrogen hormone in a woman’s body
postmenopausal period is
A. estradiol
B. estrone
C. estriol
D. Estradiol dipropionate
E. progesterone

494. Gonadotropins, which play a role in the pathogenesis of


hyperplastic processes and endometrial cancer, are secreted:
A. adrenal glands
B. hypothalamus
C. anterior pituitary
D. posterior pituitary gland
E. Ovaries

495. At what daily dose is heparin used to prevent


thromboembolic complications?
A. 10 thousand units
B. 20 thousand units
C. 30 thousand units
D. 25 thousand units
E. 15 thousand units

496. Delayed sexual development is lack


A. secondary sexual characteristics by 14 g., and
menstruation by 16 g.
B. secondary sexual characteristics by 12 g., and
menstruation by 16 g.
C. secondary sexual characteristics by 14 g., and
menstruation by 18 g.
D. secondary sexual characteristics by 12 g., and
menstruation by 18 g.
E. secondary sexual characteristics by 13 g., and
menstruation by 15 g.

497. The most appropriate sequence of measures in the


diagnosis of impaired ectopic pregnancy:
A. surgery, blood transfusion
B. consultation of the therapist, anesthetist, surgery
C. Ultrasound, blood transfusion, surgery
D. blood transfusion, surgery
E. application of hemostatic therapy, blood transfusion,
surgery

498. In patients with amenorrhea with Itsenko-Cushing's


disease, there is a hyperproduction of the hormone
adenohypophysis
A. somatotropic
B. thyrotropic
C. adrenocorticotropic
D. follicle-stimulating
E. Luteinizing

499. The criterion of gonorrhea cured in patients is


established after treatment during
A.1 month
B.2 months
C.3 months
D.4 months
E. 5 months

500. The transferred inflammatory process of the pelvic


organs cannot be the reason:
A. tubal pregnancy
B. endometriosis
C. Adhesive process in the pelvis
D. painful intercourse
E. hydrosalpinx

END
Gprinciple of antise X Be6 TecTvpoea
Gcoronary heart dis X

avn.kgma.kg/webtest/testing
C
CHHrx MaHMaipman

Bonpoc: N°23
Why is expected change in the hematologic system during the 2nd trimester of pregnancy?

OTBeTbI(oAMH OTBeT)

1 Decreses in protein
2 Decrease in sedimentation rate

3 Increase in blood volume

4 Decrease in WBC's

5 Increase inhematocrit

19 20 21 22 23 24 25

50

1
2 3 4567 8 9 10
11 12 13 14 15 16 17 18 19 20
21 22 23 24 25 26 27 28 29 30
31 32 33 34 35 36 37 38 39 400

41 42 43 44 45 46 47 48 49 50

oTBe4eHHlM Bonpo
niponyueiHHbi Bonpoc
TexyuwA Bonpoc

Ao aabepiioHHA TecTa oCranoo

1:11:51
coronary heart d x G principle of antisex Be6TecTypoBa
G

O avn.kgma.kg/webtest/testing
C
CHHrx KaHManpma Kg Ru

Bonpoc: N°22
to reduced incidence of neural tube
defects:
Periconceptional use of the following agent leads

OTBeTbi(oAMH OTBeT)

1 Magnesium

2 Calcium
3 Vitamin A

4 Iron

5( Folic acid

18 19 20 21 22 23 24 25 26

50

89 10
1
2 3 4 567
11 12 13 14 15 16 17 18 19 20
21 22 23 24 25 26 27 28 29 30
31 32 33 34 35 36 37 38 39 40
41 42 43 44 45 46 47 48 49 50

OTBeueHHbi Bonpoc
nponyueHHbil Bonpoc
Tekyuwn Bonpoc

Ao 3abepuieHw TecTa ocTanoc

1:12:01
Gcoronary heart di X principle of antise x Be6 TectwpoBaH X

C Oavn.kgma.kg/webtest/testing9

CHHTx MaHMaRAMan

Bonpoc: N 26
Why does anemia of pregnancy occur?

OTBeTbi(oqMH OTBeT)
1 Levels of erythropoietin decrease

2 None of the above

3 Plasma volume increases

4 More Fe is available for fetal erythropoesis

5/ Red cellmassdecreases

22 23 24 25 26 27

50

1 2 3 4 567 8 9 10
11 12 13 1415 16 17 18 19 20
2122 23 24 25 26 27 28 29 30
31 32 33 34 35 36 37 38 39 40
41 42 43 44 45 46 47 48 49 50
OTBeHeHHbl Bonpoc
ponyujenin Bonpoc
Texyunn Bonpoc

o sabepuienw Tedra ocTanocb

1:11:31
G coronary heart dis X Gprinciple of antise X 6 TecTpo

C Oavn.kgma.kg/webtest/testing

CHHx KaHMaip

Bonpoc: N°21
The duration of the second stage of labor should not be more:

OTBeTbi(oAMH OTBeT)
1
20 min
2 30 min
3 2 hour
4 3 hour

5(1 hour

17 18 19 20
50

12 3 45 6 7 B 10 9
11 12 13 14 15 16 17 18 19 20

21 22 23 24 25 26 27 28 29 30
31 32 33 34 35 36 37 38 39 40
41 42 43 44 45 46 47 48 49 50
OTBe4eHHl Bonpoc
nponyujewHbl BOnpoc
TeKyuiw Bonpoc
X Gprinciple of antis Be6TecTHpoBa
Gcoronary heart dis

O avn.kgma.kg/webtest/testing
C Ka Ru
CHHrx KaHMangwa

Bonpoc: N 24
three-analyte combination for Down's syndrome screening?
What is considered the best

OTBeTbI(oqMH OTBeT)

1 AFP hCG, estriol


2 AFP hCG, PAPP-A
3 AFP estriol, inhibin
4 AFP estriol, PAPP-A

AFPhCG, inhibin

20 2122 23 24 25 26 27

50

1 2 3 4 6 7 8 9 10
5
11 12 13 14 15 16 17 18 19 20

2122 23 24 25 26 27 28 29 30
31 32 33 34 35 36 37 38 39 40
41 42 43 44 45 46 47 48 49 50
oTBeHeHHbi BOnpoc
nponyujeHHLIM BOnpoc
Texyiunn Bonpoc

-La o Banepüieiwa TecTa oTanoch

1:11:44
61% 7:33AM
Gcoronary heart dis X G principle of antise x Be6 TecTpoBar

C O avn.kgma.kg/webtest/testing

CHHrx KaHMaaman Ks Ru

Bonpoc: N°25
During physiologically developing pregnancy the
following changes in the haemostatic system
help to prevent postpartum bleeding:

OTBETbI(oAMH OTBeT)
1 Hypercoagulation
2 Activation of only the plasma link

3 Consumption coagulopathy

Activation of only the vascular-platelet link

5 Hypocoagulation

21 22 23 24 25 26

50

1
2 3 4 5 6 789 10
31 1213 14 15 16 17 18 19 20
2122 23 24 25 26 27 28 29 30
31 32 33 34 35 36 37 38 39 40
41 42 43 44 45 46 47 48 49 50

oTBeHeHHI BOnpo0c
nponyuetHbi BOnpoc
TeKyunn Bonpoc

Ao BaBepuieuwa Tecra ocranocu:

1:11:37
O O
61% 7:33

G coronary heart dis x G principle of antise X Be6 TecTpoBa

O COavn.kgma.kg/webtest/testing
CHHrx Mawmanama

Bonpoc: N 27
You have just examined a 28 year-old primagravida in spontaneous labor at 38 weeks
Examination findings are cephalic presentation, cervix is 7 cm dilated, posterior fontanelle
palpable anteriorly under symphisis pubis. What is presenting diameter of the fetus?

OTBeTbI(OAMH OTBET)

1 Suboccipitobregmatic

Verticomental

3 Suboccipitjfrontal

Submentobregmatic

5 Occipitofrontal

23 24 25 26 27
50

1 2 3 4 56789 10
11 12 13 14 15 16 17 18 19 20
21 22 23 24 25 26 27 28 29 30
31 32 33 34 35 36 37 38 39 40
41 42 43 44 45 46 47 48 49 50

OTBe4eHHii Bonpoc
iponyuueHHil Bonp0c
wsssl unnnn
27
G twenty-seven

4 O
6% 7:33 AM
G coronary heart di: X Gprinciple of antise x Be6TeCTHpOBaH X

f C O avn.kgma.kg/webtest/testing

CHHrx aHMa@pma K3 Ru
Bonpoc: N°28
Pregnant woman at term of pregnancy complains
of a feeling of heaviness and pressure in the
lower abdomen, sometimes a little pain?
From the vagina there were scanty mucous
What is your diagnosis? discharges?

OTBeTbI(oAMH OTBeT)
1
Beginning of labor

2 Premature labor
3 Active phase

4 Latent phase
5 Preliminary phase

50
24 25 26 27
28 20

2 345 6 7 8 9 10
11 12 13 14 15 16 77 18 19 20
21 22 23 24 25 26 27 28 29 30
31 32 33 34 35 36 37 38 39 40
41 42 43 44 45 46 47 48 49 50

OTBeHeHHbih Bonpoe
nponyujeHHbi Bonpoc
Tekyuini Bornpoc

Ao sanepuieina Tecra o0Tanoch

Multi-User Lenovo
PS HD Dual Front 86 ( Kid's Ac
Speakers 9 Accounts
5 Samsung India X LMS KTMA Be6TecrwpoBal

C A avn.kgma.kg/webtest/testing

rapr AMKua

Bonpoc: N°1
What frequency range of transducers should be used for transabdorminal examination of the
pelvic organs?

OTBETbi(oAMH OTBeT)

1 3.5-5.0 MHz
2 10.0-15.0 MHz

3 7.5-10.0 MHz
4 15.0-20.0 MHz
5 5.0-7.5 MHz

23 4 5 6 7 50

2 3 4 5 678 10 9
11 12 13 14 15 16 17 18 19 20
21 22 23 24 25 26 27 28 29 30
31 32 33 34 35 36 37 38 39 40
41 42 43 44 45 46 47 48 49 50

OTBeveHHEIM BONpoc
nporyujeHHbIM BOnpoc
Tekyuan Bonpoc

Ao saBepiueHnA TeCTa oCTanoCb

1:12:56
3aBepuwTb TeCT
SAMGUNG

al 76% m07 41
Samsung India LMS KrMA X
Be6TecTwpoBaX
C A avn.kgma.kg/webtest/testing

Fapr Auxua

Bonpoc:Ne6
Specity the age at which girls are determined by the stage of butane breast and nipple
"

pigmentation.:

OTBeTbi(0gMH OTBeT)
1 8-9
2 16-17

3 14-15

4 12-13
5 10-11

23 4 7 8 9 10 50

7 212 3 4 57 8 9 10
11 13 14 15 16 17 18 19 20
21 22 23 24 25 26 27 28 29 30
31 32 33 34 35 36 37 38 39 40
41 42 43 44 45 46 47 48 49 50
OTBeYeHHbl BOnpoC
riponyueHHbIM Bonpoc
Texyuna Bonpoc

Ao 3aBepweHwa TecTa ocTanocb

1:06:43
3anepuWTb TeCT
SAMSUB
Al 76% m0743

B Samsung IndiaX LMS KTMA Be6TecTMpOBR

C A avn.kgma.kg/webtest/testing

FaprAKua

Bonpoc: N°7
Determine at what age the absence of secondary sexual characteristics is considered a sign of
delayed sexual development in giris

OTBeTbi(OAMH OTBeT)

1 14-18
2 10-13
3 11-14

4 13-17

5 12-16

3 4 5 6 7 8 9 10 11 50

123 4 5 678 9 10

11 12 13 14 15 16 17 18 19 20
21 22 23 24 25 26 27 28 29 30
31 32 33 34 35 36 37 38 39 40
41 42 43 44 45 46 47 48 49 50

OTBeHeHHblM Bonpoc
rponyiueHHbi Bonpoc
Teixyuumt sonpoc

Ao sanepiuieHR TecTa oCTanoch

1:03:51
3asepuwTb TeCT
A avn.kgma.kg/webtest/1
TpMnaTxW AeBAyTT Kg Ru

Bonpoc: N°1

A 56-year-old man was admitted to the oncology clinic


with complaints of pain in the pelvic bones. He has
frequent urination. The urine, sometimes with blood
impurities. Digital rectal exam revealed an abnormal
solid mass in the prostate. What imaging study is the
first choice to clarify the diagnosis ?

OTBeTbi(oAWH OTBET)
1 X-ray

2 Transabdominal Ultrasound

3 Transrectal Ultrasound

4 MRI

5 CT-scan

1
23 4 5 6 1
50
SAMSUNG

S Samsung india | al 75%m07 48


X LMS KTMA
Be6TecTApoBBX
A avn.kgma.kg/webtest/testing

rapr Awxwa

Bonpoc: N9
At the pediatricians examination,
the girl is 3 months old. Objectively: hypertrophy
aplasia of the labia, and hymen. Guess ofthe clitoris
what the girl's preliminary diagnosis is?

OTBETbI(ogMH OTBeT)
1 synechia of the labia

2 Shereshevsky-Turner

3 hyperpitultarism
hermaphroditism

5 hypopituitarism

7 9 10 11 12 13 50

1 2 3 45 5 7 8910
11 12 13 14 15 16 17 18 19 20
21 22 23 24 25 26 27 28 29 30
31 32 33 34 35 36 37 38 39 40
41 42 43 44 45 46 47 48 49 50
OTBeveHHbiM Bonpoc
nponyujeHHbl Boipoc
TexyuuA BOnpoc

AO 3anepuenwR TecTa oCTanoch

0:59:23
3anepunTb TecT
Aepa Ke

Bonpoc: N926
Periconceptional use
of the following
agent leads to reduced incidence
of neuraltube defects:

OTBeTbi(oAMH oTBeT)
1 Calcium

2 Vitamin A

3 Magnesium
Iron

5 Folic acid

22
22 23 24 25
2627 28 29 30 So
10
20
30

40
Bonpoc: N941
Parametritis is inflammation
50 of the:

OTBeTbl(oAMH oTBeT)
1OParauterinefiber
1anocb 2 Fallopian tube

3 Cecum

4 Gland
5 Ovary

37 38 39 40 42 43 44 45
SAMSUNG

S SamsungIndia x
,

l 73% m08 05
LMS KTMA
f

Be6TecTposa X

C A avn.kgma.kg/webtest/testing

TaprAMxua R
Bonpoc: N 24
What causes flexion of the
head during labor?

OTBeTbI(oAMH OTBeT)
1
Resistance of pelvic floor

2 Rotation of head

3 Resistance of pelvic bones


4 Decent of head

5 Engagement of head

20 21 22 23
24 25 26 27 28

50

1 2 3 4567 8 9 10
11 12 13 14 15 16 17 18 19 20
21 22 232 25 26 27 28 29 30
31 32 33 34 35 36 37 38 39 40
41 42 43 44 45 46 47 48 49 50

OTBeveHHbIM Bonpoc
nporyuyetHbl Bonpo0C
Texyumh Bonpoc

A0 30BepuieHus TecTa ocTanocb

0:42:33
3aeepuwh TeCT
SAMSUNG

Al 73%m08.08
S Samsung India| x LMS KrMA X Be6TeCTHpOBa x

C A avn.kgma.kg/webtest/testing

Fapr AMKwa

Bonpoc: N°27
A 25 year old, pregnant presents to the 0B clinic. She is 30 weeks pregnant by
last menstrual
period dating. With an external obstetric examination of a pregnant woman in the fundus of
the
uterus determined a round, dense, balloted part of fetus. Which of the following Leopold's
maneuvers determines this?

OTBeTbi(oAMH OTBeT)
1 Second maneuvers
2 Third maneuvers
3 Fourth maneuvers
4 Firstmaneuvers
5 Fifth maneuvers

23 24 25 26 27 28 29 30 31

50

1 2 3 4567 89.10
11 12 13 14 15 16 17 18 19 20
21 22 23 24 25 2627 28 29 30
31 32 33 34 35 36 37 38 39 40
41 42 43 44 45 46 47 48 49 50
OTBeeHHiR BOnpoe
iponyLujeuel BOnpoc
Tekyun sunpod

Ao sasepuwewn recTa ocTaIOC

0:39:27
SAMSUNO
73% mOh 10
0 Samng inte LMS KTMA
Be6 Tectpoe

C A avn.kgma,kg/webtest/testin

r'apr axu

Bonpoc: N°31
Algodismenorea is

OTBETbi(oAMH OTBeT)

1 Painful menstruation
2 Reduced blood loss during menstniation

Intermenstrual meager spotting

4 Rare and scarce menstruation

Heavy periods

27 28 29 30 31 32 33 34 35

50

10
123 13 144 1556 16 177 a9
18 19 20
11 12
21 22 23 24 25 26 27 28 29 30
32 33 34 35 36 37 38 39 40
41 42 43 44 45 46 47 48 49 50
OTBeveHHblt BOnpec
npanyuieHibt Bonpoc
TERyiuw Boripoc

Ao 3aBepweHwa TeCTa oCTanoCb

0:36:53
3anepuwTb TeCT
M
al 72%
mO8 13
6 Samsung India |
x MLMSKTMA Be6TecrwpoBaX
C A avn.kgma.kg/webtest/testing

apr AMia

Bonpoc: N°32
Name the composition of injectable contraceptives:

OTBeTbI(OAMH OTBeT)

1 Mixed doses of hormones

2 Small doses of a progestogen

3 Long-acting Progestogens

Conjugated estrogens

5 Antiandrogens

28 29 30 31 32 33 34 35 36

50

1 23 456 7 B
9 10
11 12 13 14 15 16 17/18 19 20
21 22 23 24 25 26 27 28 29 30
31 32 33 34 35 36 37 38 39 40
41 42 43 44 45 46 47 48 49 50
OTBeyeHHblM Bonpoc
nponyueHHbi Bonpoc
TexyLn eonpoc

Ao 38BepuieHMA TecTa ocTanocb

0:34:24
SaBepuWTb TeCT
61% 7:34 A
Gcoronary heart dis Xx
G principle of antise x Be6 TecTpoBaH

C Oavn.kgma.kg/webtest/testing

CWHx KaHManpa Kg

Bonpoc: N°29
In the anovulatory menstrual cycle, basal temperature is characterized by:

OTBeTbi(oqMH OTBeT)
1
Rise In Temperature Before Ovulation

2 Lack Of Temperature Rise

3 Rise In Temperature Before Menstruation

Rise In Temperature After Ovulation

5 Rise In Temperature Immediately After Menstruation

25 26 27 28 29 30

50

12
11
3 4 5 6 7 89 10
12 13 14 15 16 17 18 19 20
21 22 23 24 25 26 27 28 29 30
31 32 33 34 35 36 37 38 39 40
41 42 43 44 45 46 47 48 49 50

OTBeveHHbiM BOnp0c
nponyuyeHHbiM Bonpoc
Tekyuinn Bonpoc

29
G twenty-nine
61 61% 7

Gcoronary heart dis X Gprinciple of antise X Be6TeCTpoBaHx

C O avn.kgma.kg/webtest/testing

CMHrx MaHMaigma

Bonpoc: N930
What are the diagnostic tests that indicate the presence of bacterial vaginosis

OTBeTb (oAMH OTBeT)


1 Lowering the ph of vaginal secretions

2 Increase in basal temperature

Detection of abnormal cells

4 Detection of key cells

5 Negative amino test of whites

26 27 28 29 31 34

50

1 23 4 5 67 8 9 10
11 12 13 14 15 16 17 18 19 20
21 22 23 24 25 26 27 28 29 30
31 32 33 34 35 36 37 38 39 40
41 42 43 44 45 46 47 48 49 50
OTBeHeHHbiM Bopoc

nporiyueHHbIM BOnpoc
TeKyuiwn Bonpoc

Ao a6eLAIe recra ograngcb

1:10:57
O
A55% 8:1

CMHrx KaHManpan K

Bonpoc: N°1
In which of the following conditions is recommended to perform Hysterosalpingography?

OTBETbl(oAWH OTBeT)
1 Pregnancy
2 Infertility

3 Bloody discharge

4 Pelvicinfection
5 Heavy periods

23 50

211 12 133 14456 177 89 10


15 16 18 19
20
2122 23 2425 2627 28 29 30
3132 33 34 35 36 37 38 39 40
41 42 43 44 45 46 47 A8 49
50

oTBeeHHbi Bonpo
niponyujeHHi Bonpog
TEKyuinn Bonpoc

Ao aanepuenwa Tecra
ograioch

0:27:24
3aBepuwTb TeCT
72% m0814
x LMS KIMA X Be6 TecTMpoBaX
Samsung India |

A avn.kgma.kg/webtest/testing
Of C
raprAwKua

Bonpoc: N 33
For planned hormonal contraception use:

OTBeTbi(oAMH OTBET)

1
Pure gestagens
2 A-gn realizing hormones

Pure estrogens

Selective inhibitors of progesterone


receptors
4

5 Prostoglandins

35 36 37
30 31 32 33 34
29

50

10
2 3 45 6 7
1 9 8
16 17 18 19 20
11 12 13 14 15
29 30
21 22 23 24 25 26 27 28
39 400
31 32 3334 35 36 37 38 50
48 49
41 42 43 44 45 46 47
OTBeyeHHbIMBOpoc
iponyujeHHbM BOnpoc
Texyunn Bonpoc
ocTanoch
Ao 3asepuueHwa Tecra

0:32:52
3aBepuWTh TeCT
SAMSUNG

al 72% 08 18
0 SamsungIndia x LMS KTMA
Be6 TecTWpOB

C A avn.kgma.kg/webtest/testing
rapr Awxu1a

Bonpoc: Ne39
A 20-year-old female patient turned
to clinic, complaining of abundant, purulent
the genital tract, itching, burning and discharge from
pain in the vagina. She fell ll acutely, 2 days ago,
above complaints appeared. Menstrual function not when the
is impaired. By examination:
mucosa is sharply hyperemic, covered the vaginal
with a purulent bloom, bleeds easily
cervix is cylindrical in shape, the mucosa when touched. The
is not changed. What is the most likely diagnosis

OTBeTbI(OAMH OTBeT)
1 Acute colpitis

2 Vulvovaginitis

3 Bartholinitis

4 Endocervicitis
5 Endometritiss

35 36 37 38 39 40 41 42 43

50

123 4 5 6 7 8 9 10
11 12 13 14 15 16 17 18, 19 20
21 22 23 24 25 26 27 28 29 30
31.32 33 34 35 36 37 38 39 40
41 42 43 44 45 46 47 48 49 50
OTBe4eHHbi BOnpoc
FiponyuyeHHb1A BonpoC
TekyLLn Bonpoc

Ao 3aBepuieHwa TecTa ocTanoch


Tagwepa Keean NpaeHH6

Bonpoc: N942
Which factors not increase
the risk of developing inflammatory
diseases sexual organs:

OTBeTbi(oAUH OTBeT)
1 Hysterosalpingography

2 Oral contraceptives

3 IUCD
4 Medical abortion

5 Sexual initiation at age 15

38 39 40 41 47 43 44 45 46 50
45 8:18 AM
CHHrx MaHMaipan Ka Rtu

Bonpoc: N°4
Ultrasound examination revealed hyperechoic finding on the endometrium. Most probable
diagnosis is endometrium polyp. What is the best imaging study to do next to clarify the
diagnosis 7

OTBeTbI(oAMH OTBeT)

Pelvic Xray

2 Hysterosalpingography

3 MRI

CTscan

5 Hysteroscopy

12 3 5 6 7 50

111 122 133 144 515 166 717 18B919 10


20
2122 23 2425 2627 28 29 30
31 32 33 34 35 36 37 38 39 40
41 42 43 44 45 46 47 48 49 50

OTBeHeHHbIM BOnpoc
nporyiuewHbl BOnpoc
Teryunn sonpoc

Ao 3asepuenas TecTa ocTanoc

0:27:06
3anepuMTb TecT
4
Gfour

IPS HD dDual Frant Muit-User enovo


55% 8:17 AM
CHHrx HaHManpwa Kg
Ru
Bonpoc: Ne3
Which medical imaging is used to obtain an image of the mucous surface of the uterine cavity?

OTBETbli(OAMH OTBeT)

Hysteroscopy

2 MRI
Ct-scan

4 Ultrasound

5 Sonohysterography

1 4 5 6 7 50

1212 133 414 155 166717 188 91020


11 19
21 22 24 25 26 27 28 29
23 30

31 32 33 34 35 36 37 38 39 40
41 42 43 44 45 46 47 48 49 50

OTBeeHHbIM BOnpoc
nponyujeHHbi Bonpoc
Tekyuinn BOnpoc

Ao 3asepueHwa Tecta ocTanocb

0:27:18
3aBepunTb Tecr

G 3
three

A O O
Kgma.kg/webtest/testing

Taqepa Keean MpaeWHOxan

Patient L, 42 years old, consulted


Bonpoc: N939
a gynecologist to receive
marriage. History of 2 births, recommendations on contraception,
2 medical abortions. General suffers from hypertension
35. Recommended method of contraception: condition is satisfactory, for 5 years. Regular sexlifein
blood pressure is 135/90mm Hg,
height 164 cm, weight 90
kg. BMI

OTBeTbI(oAnH OTBeT)
Postcoital contraception (Postinor, Dvella)

OIntrauterine device with silver


3 Surgical sterilization

4 Condom

5 Calendar chythm method

35 36 37 38 39 40 41 42 43 S0
SAMSUNG

71%m0823
Samsung India x LMS KIMA Be6TecrpoBa
C A avn.kgma.kg/wehbtest/testin9

rapr Awxu

Bonpoc: N939
A 20-year-old female patient turned
to clinic, complaining of abundant, purulent
the genital tract, itching, burning and pain in the vagina. She fell ill acutely,2 discharge fron
above complaints appeared. Menstrual function days ago, when the
is not impaired. By examination: the vagnal
mucosa is sharply hyperemic,
covered witha purülent bloom, bleeds easily
cervix is cylindrical in shape, the mucosa is not when touched, The
changed. What is the most likely diagnósi

OTBeTbI(oaMH OTBeT)
1 Acute colpitis
2 VulvovaginitisS
Bartholinitis

Endocervicitis

5 Endometritis

35 36 37
38 3940 41 42 43

50

1 23 45 6 78 9 10
11 1213 1718
14 15 16 19 20

21 22 23 24 25 26 27 28 29 30
31 32 3334 35 36 37 38 39 40
41 42 43 44 45 46 47 48 49 50

OTBe4eHHbi BOnpoc
nponyuteHHbiM Boripoc
Tekyiuw Boripoc

Mo 3asepwenwn TecTa OCTanoC


SAMSUNG

8 Samsung India 71% m 0823


X LMS KTMA X Ee6TecTHpoBa X

C A avn.kgma.kg/webtest/testing

rapr Auxuwa

Bonpoc: NP42
Pathological amenorhea does not include:

OTBeTbi(oAMH OTBeT)
Amenorrhea In Kallmann syndrome

2 Ameniorthea in Sheehan's syndrome

3 Amenorrhea with gonadal dysgenesis

4 Psychogenic amenorrhea
5 Amenorrhea of childhood

38 39 40 41 42 43 44 45 46

50

12 3 4 567 8 9 10
11 12 13 14 15 16 17 18 19 20
21 22 23 24 25 26 27 28 29 30
31 32 33 34 35 36 37 38 39 40
41 62 43 44 45 46 47 48 49 50

OTBeeHHi BOnpoc
riponyieuik sonpoe
Texyun Bonpoe

o saBepuuena TeCTa DCTanOC

0:24:08
308epuinTb TeCT
a6 Tecrvposo
C kgma.kg/webtest/tenting

apr Au

Parametritis-is inflammation
Bonpoc: N943
ofthe:

OTBeTbi(opMH
1
OTBeT)
Cecum

2 Gland
3 Parauterinefiber
4 Ovary

5 Fallopiantube

39 40 41 42 43 44 45 46 47
50

1 2 3 4 56 78 9 10
11 12 13 14 15 16
17 18 19 20
21 22 23 24 25 26 27 28 29 30
31 32 33 34 35 36 37 38 39 40
41 42 43 44 45 46 47 48 49 50
OTBeHeHHbiM BOnpoc
npornyujeHHbih BOnpoe
TeKyuw Bonpoc

Ao 3aBepweHwA TecTa ocTanocb

0:24:43
3aBepwwTb TeCT
SAMSUNG

S Samsung India X l 70%m08 28


LMS KrMA
Be6TecTwpoB
C A avn.kgma.kg/webtest/testing

Tapr AMiKua

Bonpoc: NP48
Through the lesser
sciatic foramen passes
the following structures

OTBeTbi(oAMH OTBeT)
1
the pudendal neurovascular
bundle, internus obturator
muscle
2 the external obturator artery, external
cutaneus nerve of thigh
3 the inferior vesical artery, internal pudendal
artery and vein
4 the posterior cutaneus nerve of thigh, internus
obturator muscle
5 the inferior vesical artery, posterior cutaneus nerve
of thigh

43 44 45 46 47
48 49 50

1 2 3 4 56 7 89 10
11 12 13 14 15 16 17 18 19 20

21 22 23 24 25 26 27 28 29 30
31 32 33 34 35 36 37 38 39 40
41 42 43 44 45 46 47 48 49 50

OTBeHeHHbiM BOnpoc
riponyujeHHbl Bonpoc
TEKyuyin BOnpoc

Ao 3aBepuweHMA TecTa oCTanocb

0:19:29
O Sammungtndia
i MS KTMA
Bef Tec
O C A avn kgma.kg/webtest/testing

Tayr Fo

Bonpoc: N947
The suspensory system of the uterus includes

OTBeTbi(oAMH OTBeT)
cardinal ligaments of uterus

2 ig propria of ovaries

suspensory ligaments of ovaries

rectouterine ligaments

5 broad ligaments of uterus

1 43 44 45 46 47 48 49

1 2 3 45 6 7 89 10
11 12 13 14 15 16 17 18 19 20
21 22 23 24 25 26 27 28 29 30
31 32 33 34 35 36 37 38 39 40
41 42 43 44 45 4647 48 49 50

OTBeveHHbl BOnp0c
nporyueHHblR Bonpoc
Texyuna Bonpoc

Ao sanepuieHwa TecTa oCTanocb

0:18:02
3aBepuiwrb TeCT
ShMSUNG

S SamsingIndiox LMSKTMA l 70m8 29


Be6Tecrvpoes
CA C A avn.kgma.kg/webtest/testing
1
Tapr Aptu

Bonpoc: N948
Through the lesser sciatic foramen
passes the following structures:

OTBETbI(ogMH OTBeT)
the pudendal neurovascular bundle, internus
obturator muscle
2 the external obturator artery, external cutaneus nerve
of thigh
3 the inferior vesical artery, internal pudendal artery and vein

4 the posterior cutaneus nerve of thigh, internus obturator muscle

5 the inferior vesical artery, posterior cutaneus nerve of thigh

43 44 45 46 17 48 49 50

1 2 3 4 5 6 7 B9 10
11 12 13 14 15 16 17 18 19 20
26 29 30
21 22 23 24 25 26 27
31 32 33 34 35 36 37 38 39 40
41 42 43 44 45 46 47 48 49 50

oTBevewHbit Bonpoc
riponyugeubia Bonpoc
Texyunn ponpoc

o sanepuienws recTa oCTanocb

0:17:54
3anepuwTb TeCT
avn.k na.kg/webt
btest/testing
s
rapr
e
Bonpoc: N949
Benign hyperplasia
(excessive growth
would be most likely of cells) of which part
to interfere with the of
passage of urine? the male1eproductive system

OTBeTbi(OAMH
OTBeT)
Peripheral Zone of
the Prostate
2 Ejaculatory Duct

3 Seminal Vesicle
4 Central Zone of the Prostate

5 Periurethral Zone of the Prostate

43 44 45 46 47 48 49 S0

1 2 3 4 56 7 8 9
10
11 12 1314 15 16 17 18 19 20
21 22 23 24 25 26 27 28 29 30
31 32 33 34 35 36 37 38 39 40
41 42 43 44 45 46 47 48 49 50
OTBeveHHbl Bonpoc
nponyueHHbIR Bonpoc
TeKyuiwn Bonpoc

Ao 3aBepuweHHA TeCta oCTanocb

0:17:21
3aBepuwTb TeCT
SAMSUNG

al 69% m08.36
S Samsung Indla x LMS KTMA
Be6 Tecrpoea

C A avn.kgma.kg/webtest/testing

rapr Awku

Bonpoc: N°18
Which of the following statements is the most accurate
regarding the normal changes in the
cardiovascular system at term? Total volume

OTBeTbi(opMH OTBeT)
1
Increases by 20%, plasma volume increase exceeding RBC mass increase

2 Increases by 20%, RBC mass increase exceeding plasma volume increase

3 Increases by 40%, RBC mass increase exceeding plasma volume increase

4 Increases by 40%, volume increase exceeding RBC mass increase

5 No change in the total blood volume

14 15 16 17 18 19 20 21 22

50

2 3 4 56 7 89 10
1
11 12 13 14 15 16 17 18 19 20
21 22 23 24 25 26 27 28 29 30
39 40
31 32 33 34 35 36 37 38
41 42 43 44 45 46 47 48 49 50

oTBeHeHHbIM BOpoc
iponyuueHHbl Bonpoc
TeKyuun Bonpoc

AO 3asepuieHa TeCTa oCTan0cb

0:10:53
3aBepuiHTb TeCT
SAMSUNG

S SamsungIndiax LMSKIMA 69%m08 39


Be6TectwpoBa
O C A avn.kgma.kg/webtest/testing

Tapr cua
Bonpoc: N949
Benign hyperplasia (excessive
growth of cells) of which part
would be most likely to interfere of the malereproductive systerm
with the passage of urine?

OTBeTbI(ogMH OTBeT)
1
Peripheral Zone of the Prostate

2 Ejaculatory Duct
3 Seminal Vesicle
4 Central Zone of the Prostate

5 Periurethral Zone of the Prostate

A3 44 45 46 47 48 49 50

1 2 34 5 6 7 8 9 10
11 12 13 14 15 16 17 18 19 20
21 22 23 24 25 26 27 28 29 30
31 32 33 34 35 36 37 38 39 40
41 42 43 44 45 46 47 48 49 50

oTBeHeHHbl BOnpoc
nporyuueHHbIR Bonpoc
Texyuiwn BOnpoc

ocCTanocb
Ao 3aBepuweHwa TecTa

0:08:27
3aBepuiwTb TeCT
l 98% 8.55 am
S samsung | GLOBAL x Be6 TecTpoBaHMe

C A avn.kgma.kg/webtest/testing

CWHrx KyHBap MpaTan


gRu

Bonpoc: Ne9
The14-year-old girl complained of low growth (on
the scale of height-age", red zone), there are
no secondary sexual characteristics. From
the family history, it is known that the parents are of
average height. What is your presumed diagnosis?

OTBeTbI(ogMH OTBeT)
1 chronic eating disorder

2 Shereshevsky-Turner Syndrome

3 congenital hypothyroidism

constitutional growth retardation

5 family short stature

56 7
89 10 11 1213 50

1
2 3 45 67 89 10
11 12 13 14 15 16 17 18 19 20
21 22 23 24 25 26 27 28 29 30
31 32 33 34 35 36 37 38 39 40
41 42 43 44 45 46 47 48 49 50
OTBeyeHHbIM BOnpoc
nponyuyeHHblM BOnpoc
TeKyunn BOnpoc

Ao 3aBepuweHMA TeCTa oCTanocb.

1:06:23
l 98% B8.56 am
S samsung| GLOBAL X Be6 TecTwpoBaHMe
C A avn.kgma.kg/webtest/testing

CWHrx KyHBap lpaTan

Bonpoc: Ne10
The 12-year-old boy has an appointment with a pediatrician. Objectively: the body weight is 45 kg,
thePFA is excessively developed. There is a symmetrical increase in the mammary glands,
moderate pain on palpation. Choose which clinical syndrome reflects the above complaints?

OTBeTbi(oAMH OTBeT

gynecomastia

hypogonadism

3 hypopituitarism

4 hyperpituitarism

5 Shereshevsky Turner Syndrome

6 7 8 9
1011 12 13 14

50

1 23 4 5 6 7
89 10
11 12 13 14 15 16 17 18 19 20

21 22 23 24 25 26 27 28 29 30

31 32 33 34 35 36 37 38 39 40
41 42 43 44 45 46 47 48 49 50

OTBeyeHHbIM BOnpoc
nponyuyeHHbln Bonpoc
TeKyun BOnpoc

Ao 3aBepueHw TeCTa ocTanocb

1:05:16
98%B9 58 arm
S samsung GLOBAL
Be6 Tectposaee

CA avn.kgma.kg/webtest/testing
O
CHHx KyHBap Tparan

Bonpoc: N°11
Examination of the boy's external genitalia revealed that the external
opening of the urethra is
located in the area of the penoscrotal angle. What is the type of anomaly in a
child?

OTBETbI(oAMH OTBeT)

1 Epispadias
2 Phimosis
3 Paraphimosis

4 Hypospadias
5 Bladder exstrophy

8 10 11 12 13 14 15

50

1 2 3 4 567 8 9 10
31 12 1314 15 16 17 18 19 20
2122 23 24 25 26 27 28 29 30
3132 33 34 35 36 37 38 39 40
41 42 43 44 45 46 47 48 49 50

OTBeeHulh 8onpoe
nponyuyewa Bonpoe
TexyuM BOnpoc

Ao sabepuienin Tecra 0Cranoch

1:04:06
3aepuTb TeCT
S samsung | GLOBAL l 96% 9 19 arm
Be6 TecTMpoBaHMe

A avn.kgma.kg/webtest/testing

CWHrx KyHBap
Nparan gRu
Bonpoc: N°24
Early breastfeeding and
manual stimulation of the breast nipple
to decrease postpartum bleeding. The is a technique that can be
mechanism by which this maneuver works
used
is stimulation

OTBeTbli(OAMH OTBeT)
Sexual stimulation causing vaginal
contractions
2 Stimulation of opiate production

3 Stimulation of prolactin production

Stimulation of prostaglandins

5 Of oxytocin production causing uterine contraction

20 21 22 23 24 25 26 27 28

50

1 2 3 4 5 6 7 8919 10
11 12 13 14 15 16 17 18 20

21 22 23 24 25 26 27 28 29 30
31 32 33 34 35 36 37 38 39 40
41 42 43 44 45 46 47 48 49 50
OTBeveHHbl BonpoOC
nponyueHHbiM Bonpoc
Texyuwn Bonpoc

Ao 3aBepweHwa TeCTa oCTanocb

0:42:21
3aBepwwTb TeCT
95% 927 arm

S samsung | GLOBAL X Be6 TecTpoBaHne


A
C avn.kgma.kg/webtest/testing

CWHrx KyHBap ipaTan Ru

Bonpoc: N°30
To induce ovulation, use:

OTBeTbl(oAMH OTBeT)

Antiestrogens

2 Prostaglandins

3 Antagonists Of Prostaglandins

4 Progestogens

5 Natural Estrogens

29 30 31 32 33 34
26 27 28

50

10
1 2 3 4 5
717 89
18 19 20
11 12 13 14 15 16

21 22 23 24 25 26 27 28 29 30
31 32 33 34 35 36 37 38 39 40
50
41 42 43 44 45 46 47 48 49
OTBeyeHHbl BOnpoc
nponyuyeHHbln Bonpoc
TekyuiuM Bonpoc

TeCTa oCTanocb
Ao 3aBepuweHwA
l 94%929 am
S samsung | GLOBAL
Be6 TecTvpoBaHne

C A avn.kgma.kg/webtest/testing

CWHrx KyHBap lpaTan Kg Ru

Bonpoc: N 33
Research methods of the anatomical and functional state
of the vagina:

OTBETbi(ogMH OTBeT)
1 Inspection with speculum;

2 Puncture of the posterior fornix,

3 Combined
vaginal rectal examination

4 Determination of the purity of the vaginal contents.

5 Cytological examination of the contents of the uterine cavity

29 30 31 32 33 34 35 36 37

50

1 2 3 4 5 6 7 89 10
11 12 13 14 15 16 17 18 19 20
21 22 23 24 25 26 27 28 29 30
31 32 33 34 35 36 37 38 39 40
41 42 43 44 45 46 47 48 49 50
OTBe4eHHBIM BOnpoc
nponyueHHEIM BOnpoc
Texyuwh eonpoc

Ao 3aBepweHwA TeCTa oCTanocb

0:32:36
3aBepuWTb TeCT
l 94%929 am
S samsung | GLOBAL
X Be6 TecTwpoBanne

C A avn.kgma.kg/webtest/testing
0
CWHrx KyHBap npaTan

Bonpoc: N°33
Research methods of the anatomical and functional state of the
vagina

OTBeTbl(OAMH OTBeT)
1
Inspection with speculum

2 Puncture of the posterior fornix

3 Combined vaginal rectal examination,

4 Determination of the purity of the vaginal contents.

5 Cytological examination of the contents of the uterine cavity

29 30 31 32 33 34 35 36 37

50

1 2 3 4 5 6 7 8
9 10
11 12 13 14 15 16 17 18 19 20
21 22 23 24 25 26 27 28 29 30

31 32 33 34 35 36 37 38 39 40
41 42 43 44 45 46 47 48 49 50

OTBeyeHHblM Bonpoc
nponyueHHblM BOnpoc
TeKyuju Bonpoc

Ao 388epuwewwa TeCTa ocTanocb

0:32:27
3aBepuHTb TeCT
94% 9.33 am
S samsung | GLOBAL X Be6 TeCTMpOBaHMe

C A avn.kgma.kg/webtest/testing
o
CWHTx KyHBap npaTan Kg Ru
Bonpoc: N°39
A 23-year-old female patient came to the antenatal clinic with complaints of cramps during
urination, fever up to 37.8 C, purulent discharge from the genital tract. She has several sexual
partners.Examination by speculum: the mucous membrane of the urethra is hyperemic, the cervix
is cylindrical, hyperemia of the area of the external opening of the cervical canal. Discharge from
the cervical canal is purulent. Define the tactics of the antenatal clinic doctor:

OTBeTbi(oAMH OTBeT)
1 Prescribe an outpatient course of antibiotic therapy

2 Get swabs for chlamydia infection

3 Hospitalize the patient in a gynecological hospital

4 Hospitalization in the dermatovenerologic clinic

5 Take swabs, culture for gonorrhea

1 35 36 37 38 39 40 41 42 43

50

1 2 3 4 5 6 7 8 9 10
11 12 13 14 15 16 17 18 19 20
21 22 23 24 25 26 27 28 29 30
31 32 33 34 35 36 37 38 39 40
41 42 43 44 45 46 47 48 49 50

OTBeyeHHblM Bonpoc
nponyujeHHbiM BOnpoc
TekyuwM BOnpocC

oCTanocb
Ao 3aBepuueHwa TeCTa
94% 1935am
S samsung | GLOBAL X Be6 TeCTMpoBaHme

C A avn.kgma.kg/webtest/testing

Bonpoc: N940
Task 2 Patient R., 25 years old, was admitted to the gynecological department with complaints of
pain in the external genital area, awkwardness when walking, fever. Pain appeared four days ago
after hypothermia. Last period 3 weeks ago. By examination: in the area of the right labia majora,
a tumor-like formation measuring 4x3 cm is determined, the skin above it is hyperemic, hot to the
touch, fluctuation is noted on palpation. Examination of the cervix in the speculum and bimanual
examination revealed no abnormalities. What is the most probable preliminary diagnosis:

OTBeTbl (OAMH OTBeT)


1 OPerforation of purulent form ofappendages
2 Torsion of the legs of the ovarian tumor

3 acutebilateral salpingitis, pelvioperitonitis


4 Necrosis of subserous myomatous node

5 Bartholine abscess

36 37 38 39 40 41 42 43 44

50

1 23 4 5 67 89 10
11 12 13 14 15 16 17 18 19 20
21 22 23 24 25 26 27 28 29 30
31 32 33 34 35 36 37 38 39 40
41 42 43 44 45 46 47 48 49 50
OTBeueHHbIMBonpoc
iponyujeHHblM Bonpoc
Tekyuinn BOnpoc

o 3aBepweHwa TeCTa oCTanocb

0:27:05
Al 93 % 19.:36 arm

S samsung | GLOBAL Be6 TeCTMpOBaHMe

C A avn.kgma.kg/webtest/testing

CMHx KyHBap TpaTan KgRu

Bonpoc: Ne41
Which factors not increase the risk of developing inflammatory diseases sexual organs:

OTBeTbI(ogMH OTBeT)
1 Oral contraceptives

2 Medical abortion

3 Sexual initiation at age 15

4 IUCD
5 Hysterosalpingography

37 38 39 40 41 42 43 44 45

50

1 2 3 4 5 67 89 10
11 12 13 14 15 16 17 18 19 20
21 22 23 24 25 26 27 28 29 30
31 32 33 34 35 36 37 38 39 40
41 42 43 44 45 46 47 48 49 50
OTBeyeHHbiM BOnpoc
nponyujeHHbIM BOnpoc
Tekyunn BOnpoc

Ao 3aBepweHwA TeCTa oCTanocb

0:25:40
3aBepuvTb TeCT
al 93% 9.38 anm
TecTMpoBaHme
S samsung | GLOBAL Be6

C A avn.kgma.kg/webtest/testing
CHHrx KyHBap nparan ORu

Bonpoc: N942
central genesis include everything besides:
The causes of secondary amenorrhea of

OTBeTbi(oAMH OTBeT)

Psychogenic amenorrhea

2 Syndrome of hyperprolactinemia

Sheehan syndrome
3
Amenorrhea due to weight loss

5 Kallmann Syndrome

43 44 45 46
40 41 42
38 39

50

3 4 5 67 89 10
12
18 19 20
11 12 13 14 15 16 17
28 29 30
21 22 23 24 25 26 27
38 39 40
31 32 33 34 35 36 37
50
41 42 43 44 45 46 47 48 49
OTBeveHHblMBonpoc
nponyueHHbiM Bonpoc
TeKyunn Bonpoc
TecTa oCTanoCb
Ao saBepweHna
93% 940 am
S samsung | GLOBAL X Be6 TeCTwpoBaHe

C A avn.kgma.kg/webtest/testing

Bonpoc: N944
Patient with amenorrhea has a negative result (absence of menstrual reaction) after the
hormonal test with estrogens and progesterone, it means:

OTBeTbi(ogMH OTBeT)
1 Presence uterine form of amenorrhea

2 Presence ovarian form of amenorrhea

3 Presence central form of amenorrhea

4 Presence polycystic ovary syndrome

5 Presence amenorrhea of central genesisS

40 41 42 43 44 45 46 47 48

50

123 4 5 6 7 8 9 10
1112 13 14 15 16 17 18 19 20
21 22 23 24 25 26 27 28 29 30

31 32 33 34 35 36 37 38 39 40
41 42 43 44 45 46 47 48 49 50

OTBeyeHHblM BOnpoc
nponyujeHHblM Bonpoc
TeKyuni Bonpoc

Ao 3aBepweHna TeCTa OCTanocb

0:21:33
3aBepuinrb TeCT
92%1 94
Ssamsung | GLOBAL X Be6 TeCTMpOBaHve
+

O F C A avn.kgma.kg/webtest/testing

CWHx KyHBap NpaTan

Bonpoc: N°46
The suspensory system of the uterus includes:

OTBETbi(ogVH OTBET)

suspensory ligaments of ovaries

2 rectouterine ligaments

3 lig. propria of ovaries

4 broad ligaments of uterus

5 cardinal ligaments of uterus

42 A3 44 45 46 47 48 49 50

7 89 10
3 4 5 6

11 12 13 14 15 16 17 18 19 20
21 22 23 24 25 26 27 28 29 30
31 32 33 34 35 36 37 38 39 40
41 42 4344 45 46 47 48 49 50
OTBeyeHHblM BOnpoc
nponyueHHbiM BOnpoc
Tekyuwn Bonpoc

Ao 3aBepujeHwa TeCTa ocTanocb

0:18:46
l 92%1945
S samsung | GLOBAL Be6 TecTWpoBaHMe

O F C Oavn.kgma.kg/webtest/testing

CWHTx KyHBap paTan

Bonpoc: Ne47
hich structure is not included in the deep perineal pouch of female?

OTBETbi(oqMH OTBeT)
OCowper's glands

OInferior perineal fascia

Superior perineal fascia

M.transversus perinei profundi


External urethral sphincter

46
1 43 44 45 48 49 50

1 3 4 5 67 89
2 10

1 12 13 14 15 16 17 18 19 20
1 22 23 24 25 26 27 28 29 30
31 32 33 34 35 36 37 38 39 40
1
42 43 44 45 46 47 48 49 50
OTBeyeHHbiM Bonpoc
nponyueHHblM Bonpoc
TekyuMM BOnpoC

o 3aBepweHnI TeCTa OCTanocb.

0:16:49
nonuu1ATh TACT
91% 9.47 am

Text se.. cOPY SHARE SELECT ALL QWEB SEARCH

Oavn.kgma.kg/webtest/testing
f C

Bonpoc: N°48
Innervation of superficial perineal pouch formed by?

OTBETbl(OAMH OTBET)

1 Genitofemoral nerve

2 Perineal nerve

3 Superficial pudendal nerve

4 Internal pudendal nerve

5 Superficial perineal nerve

43 44 4546 47 48 49 50

12 3 4 5 6 7 89 10
11 12 13 14 15 16 17 18 19 20

21 22 23 24 25 26 27 28 29 30
31 32 33 34 35 36 37 38 39 40
41 42 43 44 45 46 47 48 49 50

OTBeyeHHb1M BOnpoc
nponyujeHHbl BOnpoc
TeKyuinn Bonpoc

ocTanoch
Ao 3aBepuieHwa TeCTa

0:14:22
3aBepunTb TeCT

C 4344454647484950
A91%1 948 ar
S samsung | GLOBAL x Be6 TecTupoBaHne

C O avn.kgma.kg/webtest/testing
CMHrx KyHBap lpaTan

Bonpoc: N949
part of the scrotum. Two
During a vasectomy, the ductus deferens is ligated in the superior
months following this sterilization procedure, the subsequent ejaculate contains

OTBeTbi(oqMH OTBET)

Sperm and seminal fluid

2 Seminal fluid and prostatic fluid

3 Sperm only

only
4 Prostatic fluid
seminal fluid, and prostatic fluid
5 Sperm,
46 48 49 50
43 4445 47

1 2 3 4 567 8 9 10
11 12 13 14 15 16 17 18 19 20

21 22 23 24 25 26 27 28 29 30
31 32 33 34 35 36 37 38 39 40
41 42 43 44 45 46 47 48 49 50

OTBeyeHHbI BOnpoc
nponyujeHHbIM BOnpoc
Tekyunn BOnpoc

Ao 3aBepueHwA TeCTa ocTanocb

0:13:38
S samsung | GLOBAL
91% 948 am
X Be6 TeCTupOBaHMe
X

C avn.kgma.kg/webtest/testing

CHHTx KyHBap IpaTan Kg Ru


Bonpoc: N°50
Which structure should be taken
into account not to damage while ligating the uterine artery
during hysterectomy (amputation of uterus)?

OTBETbI(oqMH OTBeT)
1 Urethra

2 Pelvic splanchnic nerves

3 Pudendal nerve

4 Ovarian artery

5 Ureter

43 44 45 46 47 48 49 50

1 2 3 45 6 7 8 9 10
11 12 13 14 15 16 17 18 19 20
21 22 23 24 25 26 27 28 29 30
31 32 33 34 35 36 37 38 39 40
41 42 43 44 45 46 47 48 49 50
OTBeyeHHblM BOnpoc
nponyuyeHHbIM BOnpoc
TeKyuinn BOnpoc

40 3aBepweHuA TeCTa oCTanocb

0:13:29
3aBepuinTb TeCT
The duration of the second stage of labor should not be
more:

OTBeTbli(oqMH OTBeT)
1 30 min

2 20 min

3 3 hour

4 1 hour

5 2 hour

Scanned by TapScanner
t what age are boys determined by the appearance d

acial hair, pigmentation of the scrotum, the first


jaculation?:

OTBETbI(oAMH OTBeT)

14-15

8-9 8-9

10-11

1 12-13

16-17

Scanned by TapScanner
Ultrasound examination using saline solution to
visualize uterus is:

OTBETbi(oaMH OTBeT)
1 Ultrasonography

2 Cystoscopy

3 Hysteroscopy

4 Sonohysterography

5 Hysterosalpingography

Scanned by TapScanner
Biparietal diameter of the fetal head

OTBETbl(OAMH OTBeT)

1
11.5cm

2 8.5 cm

3 12.5cm

A
4 9.5

5 10.5

Scanned by TapScanner
At what age are boys determined by the appearance of
facial hair, pigmentation of the scrotum, the first
ejaculation?:

OTBeTbli(oqMH OTBET
1 14-15

2 8-9

3 10-11

4 12-13

5 16-17

Scanned by TapScanner
Which of the following is not true (positive) sign of
pregnancy?

OTBETbi(oAMH OTBeeT)

1 Audible fetal heart

2 Ultrasound evidence of a fetus

3 Amenorrhea

4 Palpation of fetal parts

55 Fetal movements

Scanned by TapScanner
Male, 30 years old, married for 4 years. Have no children.
A history of trauma to both testicles at the age of 20, for
which an organ-preserving operation was performed
(which one the patient does not know exactly) on both
testicles. On examination: both testicles are reduced in
size, of dense consistency, the vas deferens are
unremarkable. The prostate gland is unchanged.
Spermogram. No spermatozoa or spermatogenesis cells
were found in the ejaculate. Total testosterone in the
blood is 4 nmol/I (the norm is 12-35 nmol/ 1). What is
your diagnosis?

OTBeTbi(oqMH OTBeT)
1
Testicular hypogonadism, azoospermia.

2 Cryptorchidism

33 Testicular cancer

4 Congenital atrophy of testicles

5 Late onset hypogonadism

Scanned by TapScanner
The onset of labor should be considered by the following
criteria:

OTBeTbi(oAMH OTBeT)
1 Regular contractions

2 Decent of fetal head

3 True labor pain

A Rupture of membrane

5 Engagement of fetal head

Scanned by TapScanner
Specify what age corresponds to the degree of puberty
(years) MA 3 2x2 My:

OTBETbI(oAMH OTBeT)

1
12-13

2 14-15

33 8-9

4 10-11

16-17

Scanned by TapScanner
The lower limit of the norm for the number of sperm in
the ejaculate is (according to WHO criteria)

OTBeTbi(oAuH OTBeT)
1 80 million

22 60 million

3 20 million

A 100 million

5 40 million

Scanned by TapScanner
The mass of the uterus after childbirth is on average:

OTBETbi(onMH OTBeT)
1
150-200 g

2 2000 g

33 300-500g

A
4 1000 g

5 50-80 g

Scanned by TapScanner
At the reception, the boy is 2 years old. Objectively:
physical development corresponds to age, testicles with
a volume of 1 ml, palpated on both sides, the scrotum is
split, urethral hypospadias are noted. Determine which
diagnosis corresponds to the objective data?

OTBeTbi(oqMH OTBeT)
1
delayed sexual development

2 false hermaphroditism

3 true hermaphroditism

A
4 a healthy child

55 premature sexual development

Scanned by TapScanner
Specify the age at which girls are determined by the
stage of" butane breast and nipple pigmentation.
"

OTBeTbI(oAMH OTBeT)
1
14-15

2 8-9

3 12-13

4 10-11

5 16-17

Scanned by TapScanner
The labor has the following stages

OTBETbl(oguH OTBeT)
1 Preparation, dilation, expulsion

2 Effacement of the cervix, the expulsion, delivery


of placenta

3 Preparation, dilation, expulsion, early


postpartum

A Preliminary, dilatation, expulsion, postpartum

5 Dilation, expulsion, delivery of placenta, early


postpartum

Scanned by TapScanner
At the reception, the girl is 8 years old. Objectively: the
mammary glands are significantly enlarged, hair in the
armpits, pubic area, the appearance of the first
menarche. Find out from the description what kind of
preliminary diagnosis can be made for a girl?

OTBeTbI(oAWH OTBeT)
1
premature sexual development

2 healthy child

3 hypothyroidism

4 delayed sexual development

5 sexual crisis

Scanned by TapScanner
At which gestational age detecting pregnancy using
transvaginal ultrasonography?

OTBeTbi(oAMH OTBET)
1 5-6 weeks

2 2-3 weeks

3 1-2 weeks

4 4-5 weeks

5 3-4 weeks

Scanned by TapScanner
The 17-year-old boy was examined. Objectively: men's
tone of voice, the protrusion (Adamis Apple) thyroid
cartilage. On the pubis, thick, curly hair that spreads on
the inner surface of the thigh and is directed to the navel,
in the armpit, hair that curls all over the hollow. Evaluate
the boy's sexual development according to Tanner.

OTBeTbI(oquH OTBeT)
1 V1 P1 LO Ax0 FO

2 V1 P4 L1 Ax0 FO

3 V2 P3 L1 Ax2 FO

4 V2 P5 L2 Ax4 F3

5 V2 P5 L2 Ax4 F3

Scanned by TapScanner
A woman was admitted to the oncology department with
complaints of pain in the lower abdomen and bloody
discharge. 2 years ago, she was diagnosed with a
cervical polyp (11 mm in size). Which imaging study is
most suitable for clarifying the diagnosis?

OTBeTbi(oqMH OTBeT)
1 MRI

2 X-ray

33 Hysteroscopy

4 Ultrasound

55 CT-scan

Scanned by TapScanner
Which of the following imaging technique in which
iodine containing contrast agent is inserted into the
uterus to obtain an image?

OTBETbl(oAMH OTBeT)
1
Hysteroscopy

2 Ultrasound

3 Hysterosalpingography

4 Ct-scan

5 Sonohysterography

Scanned by TapScanner
A 20-year-old patient was kicked in the crotch while
playing football. Urethroragia appeared. He notes the
urge to urinate, but cannot urinate on his own. Above the
pubis, a bulging is determined, over which a dull sound is
revealed during percussion. There is a perineal
hematoma. Blood is released from the urethra. What
kind of damage do you assume?

OTBeTbl(oguH OTBET)
1 Ruptured bladder

2 Balanoposthitis

3 Damage to the prostate

4 Kidney rupture

5 Urethral ruptured

Scanned by TapScanner
What happens to the kidneys during pregnancy?

OTBeTbi(oAMH OTBeT)
1
Glucosuria increases

2 Creatinine fall during pregnancy

3 GFR falls during pregnancy

Renal function remains unchanged

5 Proteinuria increases

Scanned by TapScanner
Gprinciple of antise X Be6 TecTvpoea
Gcoronary heart dis X

avn.kgma.kg/webtest/testing
C
CHHrx MaHMaipman

Bonpoc: N°23
Why is expected change in the hematologic system during the 2nd trimester of pregnancy?

OTBeTbI(oAMH OTBeT)

1 Decreses in protein
2 Decrease in sedimentation rate

3 Increase in blood volume

4 Decrease in WBC's

5 Increase inhematocrit

19 20 21 22 23 24 25

50

1
2 3 4567 8 9 10
11 12 13 14 15 16 17 18 19 20
21 22 23 24 25 26 27 28 29 30
31 32 33 34 35 36 37 38 39 400

41 42 43 44 45 46 47 48 49 50

oTBe4eHHlM Bonpo
niponyueiHHbi Bonpoc
TexyuwA Bonpoc

Ao aabepiioHHA TecTa oCranoo

1:11:51
coronary heart d x G principle of antisex Be6TecTypoBa
G

O avn.kgma.kg/webtest/testing
C
CHHrx KaHManpma Kg Ru

Bonpoc: N°22
to reduced incidence of neural tube
defects:
Periconceptional use of the following agent leads

OTBeTbi(oAMH OTBeT)

1 Magnesium

2 Calcium
3 Vitamin A

4 Iron

5( Folic acid

18 19 20 21 22 23 24 25 26

50

89 10
1
2 3 4 567
11 12 13 14 15 16 17 18 19 20
21 22 23 24 25 26 27 28 29 30
31 32 33 34 35 36 37 38 39 40
41 42 43 44 45 46 47 48 49 50

OTBeueHHbi Bonpoc
nponyueHHbil Bonpoc
Tekyuwn Bonpoc

Ao 3abepuieHw TecTa ocTanoc

1:12:01
Gcoronary heart di X principle of antise x Be6 TectwpoBaH X

C Oavn.kgma.kg/webtest/testing9

CHHTx MaHMaRAMan

Bonpoc: N 26
Why does anemia of pregnancy occur?

OTBeTbi(oqMH OTBeT)
1 Levels of erythropoietin decrease

2 None of the above

3 Plasma volume increases

4 More Fe is available for fetal erythropoesis

5/ Red cellmassdecreases

22 23 24 25 26 27

50

1 2 3 4 567 8 9 10
11 12 13 1415 16 17 18 19 20
2122 23 24 25 26 27 28 29 30
31 32 33 34 35 36 37 38 39 40
41 42 43 44 45 46 47 48 49 50
OTBeHeHHbl Bonpoc
ponyujenin Bonpoc
Texyunn Bonpoc

o sabepuienw Tedra ocTanocb

1:11:31
G coronary heart dis X Gprinciple of antise X 6 TecTpo

C Oavn.kgma.kg/webtest/testing

CHHx KaHMaip

Bonpoc: N°21
The duration of the second stage of labor should not be more:

OTBeTbi(oAMH OTBeT)
1
20 min
2 30 min
3 2 hour
4 3 hour

5(1 hour

17 18 19 20
50

12 3 45 6 7 B 10 9
11 12 13 14 15 16 17 18 19 20

21 22 23 24 25 26 27 28 29 30
31 32 33 34 35 36 37 38 39 40
41 42 43 44 45 46 47 48 49 50
OTBe4eHHl Bonpoc
nponyujewHbl BOnpoc
TeKyuiw Bonpoc
X Gprinciple of antis Be6TecTHpoBa
Gcoronary heart dis

O avn.kgma.kg/webtest/testing
C Ka Ru
CHHrx KaHMangwa

Bonpoc: N 24
three-analyte combination for Down's syndrome screening?
What is considered the best

OTBeTbI(oqMH OTBeT)

1 AFP hCG, estriol


2 AFP hCG, PAPP-A
3 AFP estriol, inhibin
4 AFP estriol, PAPP-A

AFPhCG, inhibin

20 2122 23 24 25 26 27

50

1 2 3 4 6 7 8 9 10
5
11 12 13 14 15 16 17 18 19 20

2122 23 24 25 26 27 28 29 30
31 32 33 34 35 36 37 38 39 40
41 42 43 44 45 46 47 48 49 50
oTBeHeHHbi BOnpoc
nponyujeHHLIM BOnpoc
Texyiunn Bonpoc

-La o Banepüieiwa TecTa oTanoch

1:11:44
61% 7:33AM
Gcoronary heart dis X G principle of antise x Be6 TecTpoBar

C O avn.kgma.kg/webtest/testing

CHHrx KaHMaaman Ks Ru

Bonpoc: N°25
During physiologically developing pregnancy the
following changes in the haemostatic system
help to prevent postpartum bleeding:

OTBETbI(oAMH OTBeT)
1 Hypercoagulation
2 Activation of only the plasma link

3 Consumption coagulopathy

Activation of only the vascular-platelet link

5 Hypocoagulation

21 22 23 24 25 26

50

1
2 3 4 5 6 789 10
31 1213 14 15 16 17 18 19 20
2122 23 24 25 26 27 28 29 30
31 32 33 34 35 36 37 38 39 40
41 42 43 44 45 46 47 48 49 50

oTBeHeHHI BOnpo0c
nponyuetHbi BOnpoc
TeKyunn Bonpoc

Ao BaBepuieuwa Tecra ocranocu:

1:11:37
O O
61% 7:33

G coronary heart dis x G principle of antise X Be6 TecTpoBa

O COavn.kgma.kg/webtest/testing
CHHrx Mawmanama

Bonpoc: N 27
You have just examined a 28 year-old primagravida in spontaneous labor at 38 weeks
Examination findings are cephalic presentation, cervix is 7 cm dilated, posterior fontanelle
palpable anteriorly under symphisis pubis. What is presenting diameter of the fetus?

OTBeTbI(OAMH OTBET)

1 Suboccipitobregmatic

Verticomental

3 Suboccipitjfrontal

Submentobregmatic

5 Occipitofrontal

23 24 25 26 27
50

1 2 3 4 56789 10
11 12 13 14 15 16 17 18 19 20
21 22 23 24 25 26 27 28 29 30
31 32 33 34 35 36 37 38 39 40
41 42 43 44 45 46 47 48 49 50

OTBe4eHHii Bonpoc
iponyuueHHil Bonp0c
wsssl unnnn
27
G twenty-seven

4 O
6% 7:33 AM
G coronary heart di: X Gprinciple of antise x Be6TeCTHpOBaH X

f C O avn.kgma.kg/webtest/testing

CHHrx aHMa@pma K3 Ru
Bonpoc: N°28
Pregnant woman at term of pregnancy complains
of a feeling of heaviness and pressure in the
lower abdomen, sometimes a little pain?
From the vagina there were scanty mucous
What is your diagnosis? discharges?

OTBeTbI(oAMH OTBeT)
1
Beginning of labor

2 Premature labor
3 Active phase

4 Latent phase
5 Preliminary phase

50
24 25 26 27
28 20

2 345 6 7 8 9 10
11 12 13 14 15 16 77 18 19 20
21 22 23 24 25 26 27 28 29 30
31 32 33 34 35 36 37 38 39 40
41 42 43 44 45 46 47 48 49 50

OTBeHeHHbih Bonpoe
nponyujeHHbi Bonpoc
Tekyuini Bornpoc

Ao sanepuieina Tecra o0Tanoch

Multi-User Lenovo
PS HD Dual Front 86 ( Kid's Ac
Speakers 9 Accounts
5 Samsung India X LMS KTMA Be6TecrwpoBal

C A avn.kgma.kg/webtest/testing

rapr AMKua

Bonpoc: N°1
What frequency range of transducers should be used for transabdorminal examination of the
pelvic organs?

OTBETbi(oAMH OTBeT)

1 3.5-5.0 MHz
2 10.0-15.0 MHz

3 7.5-10.0 MHz
4 15.0-20.0 MHz
5 5.0-7.5 MHz

23 4 5 6 7 50

2 3 4 5 678 10 9
11 12 13 14 15 16 17 18 19 20
21 22 23 24 25 26 27 28 29 30
31 32 33 34 35 36 37 38 39 40
41 42 43 44 45 46 47 48 49 50

OTBeveHHEIM BONpoc
nporyujeHHbIM BOnpoc
Tekyuan Bonpoc

Ao saBepiueHnA TeCTa oCTanoCb

1:12:56
3aBepuwTb TeCT
SAMGUNG

al 76% m07 41
Samsung India LMS KrMA X
Be6TecTwpoBaX
C A avn.kgma.kg/webtest/testing

Fapr Auxua

Bonpoc:Ne6
Specity the age at which girls are determined by the stage of butane breast and nipple
"

pigmentation.:

OTBeTbi(0gMH OTBeT)
1 8-9
2 16-17

3 14-15

4 12-13
5 10-11

23 4 7 8 9 10 50

7 212 3 4 57 8 9 10
11 13 14 15 16 17 18 19 20
21 22 23 24 25 26 27 28 29 30
31 32 33 34 35 36 37 38 39 40
41 42 43 44 45 46 47 48 49 50
OTBeYeHHbl BOnpoC
riponyueHHbIM Bonpoc
Texyuna Bonpoc

Ao 3aBepweHwa TecTa ocTanocb

1:06:43
3anepuWTb TeCT
SAMSUB
Al 76% m0743

B Samsung IndiaX LMS KTMA Be6TecTMpOBR

C A avn.kgma.kg/webtest/testing

FaprAKua

Bonpoc: N°7
Determine at what age the absence of secondary sexual characteristics is considered a sign of
delayed sexual development in giris

OTBeTbi(OAMH OTBeT)

1 14-18
2 10-13
3 11-14

4 13-17

5 12-16

3 4 5 6 7 8 9 10 11 50

123 4 5 678 9 10

11 12 13 14 15 16 17 18 19 20
21 22 23 24 25 26 27 28 29 30
31 32 33 34 35 36 37 38 39 40
41 42 43 44 45 46 47 48 49 50

OTBeHeHHblM Bonpoc
rponyiueHHbi Bonpoc
Teixyuumt sonpoc

Ao sanepiuieHR TecTa oCTanoch

1:03:51
3asepuwTb TeCT
A avn.kgma.kg/webtest/1
TpMnaTxW AeBAyTT Kg Ru

Bonpoc: N°1

A 56-year-old man was admitted to the oncology clinic


with complaints of pain in the pelvic bones. He has
frequent urination. The urine, sometimes with blood
impurities. Digital rectal exam revealed an abnormal
solid mass in the prostate. What imaging study is the
first choice to clarify the diagnosis ?

OTBeTbi(oAWH OTBET)
1 X-ray

2 Transabdominal Ultrasound

3 Transrectal Ultrasound

4 MRI

5 CT-scan

1
23 4 5 6 1
50
SAMSUNG

S Samsung india | al 75%m07 48


X LMS KTMA
Be6TecTApoBBX
A avn.kgma.kg/webtest/testing

rapr Awxwa

Bonpoc: N9
At the pediatricians examination,
the girl is 3 months old. Objectively: hypertrophy
aplasia of the labia, and hymen. Guess ofthe clitoris
what the girl's preliminary diagnosis is?

OTBETbI(ogMH OTBeT)
1 synechia of the labia

2 Shereshevsky-Turner

3 hyperpitultarism
hermaphroditism

5 hypopituitarism

7 9 10 11 12 13 50

1 2 3 45 5 7 8910
11 12 13 14 15 16 17 18 19 20
21 22 23 24 25 26 27 28 29 30
31 32 33 34 35 36 37 38 39 40
41 42 43 44 45 46 47 48 49 50
OTBeveHHbiM Bonpoc
nponyujeHHbl Boipoc
TexyuuA BOnpoc

AO 3anepuenwR TecTa oCTanoch

0:59:23
3anepunTb TecT
Aepa Ke

Bonpoc: N926
Periconceptional use
of the following
agent leads to reduced incidence
of neuraltube defects:

OTBeTbi(oAMH oTBeT)
1 Calcium

2 Vitamin A

3 Magnesium
Iron

5 Folic acid

22
22 23 24 25
2627 28 29 30 So
10
20
30

40
Bonpoc: N941
Parametritis is inflammation
50 of the:

OTBeTbl(oAMH oTBeT)
1OParauterinefiber
1anocb 2 Fallopian tube

3 Cecum

4 Gland
5 Ovary

37 38 39 40 42 43 44 45
SAMSUNG

S SamsungIndia x
,

l 73% m08 05
LMS KTMA
f

Be6TecTposa X

C A avn.kgma.kg/webtest/testing

TaprAMxua R
Bonpoc: N 24
What causes flexion of the
head during labor?

OTBeTbI(oAMH OTBeT)
1
Resistance of pelvic floor

2 Rotation of head

3 Resistance of pelvic bones


4 Decent of head

5 Engagement of head

20 21 22 23
24 25 26 27 28

50

1 2 3 4567 8 9 10
11 12 13 14 15 16 17 18 19 20
21 22 232 25 26 27 28 29 30
31 32 33 34 35 36 37 38 39 40
41 42 43 44 45 46 47 48 49 50

OTBeveHHbIM Bonpoc
nporyuyetHbl Bonpo0C
Texyumh Bonpoc

A0 30BepuieHus TecTa ocTanocb

0:42:33
3aeepuwh TeCT
SAMSUNG

Al 73%m08.08
S Samsung India| x LMS KrMA X Be6TeCTHpOBa x

C A avn.kgma.kg/webtest/testing

Fapr AMKwa

Bonpoc: N°27
A 25 year old, pregnant presents to the 0B clinic. She is 30 weeks pregnant by
last menstrual
period dating. With an external obstetric examination of a pregnant woman in the fundus of
the
uterus determined a round, dense, balloted part of fetus. Which of the following Leopold's
maneuvers determines this?

OTBeTbi(oAMH OTBeT)
1 Second maneuvers
2 Third maneuvers
3 Fourth maneuvers
4 Firstmaneuvers
5 Fifth maneuvers

23 24 25 26 27 28 29 30 31

50

1 2 3 4567 89.10
11 12 13 14 15 16 17 18 19 20
21 22 23 24 25 2627 28 29 30
31 32 33 34 35 36 37 38 39 40
41 42 43 44 45 46 47 48 49 50
OTBeeHHiR BOnpoe
iponyLujeuel BOnpoc
Tekyun sunpod

Ao sasepuwewn recTa ocTaIOC

0:39:27
SAMSUNO
73% mOh 10
0 Samng inte LMS KTMA
Be6 Tectpoe

C A avn.kgma,kg/webtest/testin

r'apr axu

Bonpoc: N°31
Algodismenorea is

OTBETbi(oAMH OTBeT)

1 Painful menstruation
2 Reduced blood loss during menstniation

Intermenstrual meager spotting

4 Rare and scarce menstruation

Heavy periods

27 28 29 30 31 32 33 34 35

50

10
123 13 144 1556 16 177 a9
18 19 20
11 12
21 22 23 24 25 26 27 28 29 30
32 33 34 35 36 37 38 39 40
41 42 43 44 45 46 47 48 49 50
OTBeveHHblt BOnpec
npanyuieHibt Bonpoc
TERyiuw Boripoc

Ao 3aBepweHwa TeCTa oCTanoCb

0:36:53
3anepuwTb TeCT
M
al 72%
mO8 13
6 Samsung India |
x MLMSKTMA Be6TecrwpoBaX
C A avn.kgma.kg/webtest/testing

apr AMia

Bonpoc: N°32
Name the composition of injectable contraceptives:

OTBeTbI(OAMH OTBeT)

1 Mixed doses of hormones

2 Small doses of a progestogen

3 Long-acting Progestogens

Conjugated estrogens

5 Antiandrogens

28 29 30 31 32 33 34 35 36

50

1 23 456 7 B
9 10
11 12 13 14 15 16 17/18 19 20
21 22 23 24 25 26 27 28 29 30
31 32 33 34 35 36 37 38 39 40
41 42 43 44 45 46 47 48 49 50
OTBeyeHHblM Bonpoc
nponyueHHbi Bonpoc
TexyLn eonpoc

Ao 38BepuieHMA TecTa ocTanocb

0:34:24
SaBepuWTb TeCT
61% 7:34 A
Gcoronary heart dis Xx
G principle of antise x Be6 TecTpoBaH

C Oavn.kgma.kg/webtest/testing

CWHx KaHManpa Kg

Bonpoc: N°29
In the anovulatory menstrual cycle, basal temperature is characterized by:

OTBeTbi(oqMH OTBeT)
1
Rise In Temperature Before Ovulation

2 Lack Of Temperature Rise

3 Rise In Temperature Before Menstruation

Rise In Temperature After Ovulation

5 Rise In Temperature Immediately After Menstruation

25 26 27 28 29 30

50

12
11
3 4 5 6 7 89 10
12 13 14 15 16 17 18 19 20
21 22 23 24 25 26 27 28 29 30
31 32 33 34 35 36 37 38 39 40
41 42 43 44 45 46 47 48 49 50

OTBeveHHbiM BOnp0c
nponyuyeHHbiM Bonpoc
Tekyuinn Bonpoc

29
G twenty-nine
61 61% 7

Gcoronary heart dis X Gprinciple of antise X Be6TeCTpoBaHx

C O avn.kgma.kg/webtest/testing

CMHrx MaHMaigma

Bonpoc: N930
What are the diagnostic tests that indicate the presence of bacterial vaginosis

OTBeTb (oAMH OTBeT)


1 Lowering the ph of vaginal secretions

2 Increase in basal temperature

Detection of abnormal cells

4 Detection of key cells

5 Negative amino test of whites

26 27 28 29 31 34

50

1 23 4 5 67 8 9 10
11 12 13 14 15 16 17 18 19 20
21 22 23 24 25 26 27 28 29 30
31 32 33 34 35 36 37 38 39 40
41 42 43 44 45 46 47 48 49 50
OTBeHeHHbiM Bopoc

nporiyueHHbIM BOnpoc
TeKyuiwn Bonpoc

Ao a6eLAIe recra ograngcb

1:10:57
O
A55% 8:1

CMHrx KaHManpan K

Bonpoc: N°1
In which of the following conditions is recommended to perform Hysterosalpingography?

OTBETbl(oAWH OTBeT)
1 Pregnancy
2 Infertility

3 Bloody discharge

4 Pelvicinfection
5 Heavy periods

23 50

211 12 133 14456 177 89 10


15 16 18 19
20
2122 23 2425 2627 28 29 30
3132 33 34 35 36 37 38 39 40
41 42 43 44 45 46 47 A8 49
50

oTBeeHHbi Bonpo
niponyujeHHi Bonpog
TEKyuinn Bonpoc

Ao aanepuenwa Tecra
ograioch

0:27:24
3aBepuwTb TeCT
72% m0814
x LMS KIMA X Be6 TecTMpoBaX
Samsung India |

A avn.kgma.kg/webtest/testing
Of C
raprAwKua

Bonpoc: N 33
For planned hormonal contraception use:

OTBeTbi(oAMH OTBET)

1
Pure gestagens
2 A-gn realizing hormones

Pure estrogens

Selective inhibitors of progesterone


receptors
4

5 Prostoglandins

35 36 37
30 31 32 33 34
29

50

10
2 3 45 6 7
1 9 8
16 17 18 19 20
11 12 13 14 15
29 30
21 22 23 24 25 26 27 28
39 400
31 32 3334 35 36 37 38 50
48 49
41 42 43 44 45 46 47
OTBeyeHHbIMBOpoc
iponyujeHHbM BOnpoc
Texyunn Bonpoc
ocTanoch
Ao 3asepuueHwa Tecra

0:32:52
3aBepuWTh TeCT
SAMSUNG

al 72% 08 18
0 SamsungIndia x LMS KTMA
Be6 TecTWpOB

C A avn.kgma.kg/webtest/testing
rapr Awxu1a

Bonpoc: Ne39
A 20-year-old female patient turned
to clinic, complaining of abundant, purulent
the genital tract, itching, burning and discharge from
pain in the vagina. She fell ll acutely, 2 days ago,
above complaints appeared. Menstrual function not when the
is impaired. By examination:
mucosa is sharply hyperemic, covered the vaginal
with a purulent bloom, bleeds easily
cervix is cylindrical in shape, the mucosa when touched. The
is not changed. What is the most likely diagnosis

OTBeTbI(OAMH OTBeT)
1 Acute colpitis

2 Vulvovaginitis

3 Bartholinitis

4 Endocervicitis
5 Endometritiss

35 36 37 38 39 40 41 42 43

50

123 4 5 6 7 8 9 10
11 12 13 14 15 16 17 18, 19 20
21 22 23 24 25 26 27 28 29 30
31.32 33 34 35 36 37 38 39 40
41 42 43 44 45 46 47 48 49 50
OTBe4eHHbi BOnpoc
FiponyuyeHHb1A BonpoC
TekyLLn Bonpoc

Ao 3aBepuieHwa TecTa ocTanoch


Tagwepa Keean NpaeHH6

Bonpoc: N942
Which factors not increase
the risk of developing inflammatory
diseases sexual organs:

OTBeTbi(oAUH OTBeT)
1 Hysterosalpingography

2 Oral contraceptives

3 IUCD
4 Medical abortion

5 Sexual initiation at age 15

38 39 40 41 47 43 44 45 46 50
45 8:18 AM
CHHrx MaHMaipan Ka Rtu

Bonpoc: N°4
Ultrasound examination revealed hyperechoic finding on the endometrium. Most probable
diagnosis is endometrium polyp. What is the best imaging study to do next to clarify the
diagnosis 7

OTBeTbI(oAMH OTBeT)

Pelvic Xray

2 Hysterosalpingography

3 MRI

CTscan

5 Hysteroscopy

12 3 5 6 7 50

111 122 133 144 515 166 717 18B919 10


20
2122 23 2425 2627 28 29 30
31 32 33 34 35 36 37 38 39 40
41 42 43 44 45 46 47 48 49 50

OTBeHeHHbIM BOnpoc
nporyiuewHbl BOnpoc
Teryunn sonpoc

Ao 3asepuenas TecTa ocTanoc

0:27:06
3anepuMTb TecT
4
Gfour

IPS HD dDual Frant Muit-User enovo


55% 8:17 AM
CHHrx HaHManpwa Kg
Ru
Bonpoc: Ne3
Which medical imaging is used to obtain an image of the mucous surface of the uterine cavity?

OTBETbli(OAMH OTBeT)

Hysteroscopy

2 MRI
Ct-scan

4 Ultrasound

5 Sonohysterography

1 4 5 6 7 50

1212 133 414 155 166717 188 91020


11 19
21 22 24 25 26 27 28 29
23 30

31 32 33 34 35 36 37 38 39 40
41 42 43 44 45 46 47 48 49 50

OTBeeHHbIM BOnpoc
nponyujeHHbi Bonpoc
Tekyuinn BOnpoc

Ao 3asepueHwa Tecta ocTanocb

0:27:18
3aBepunTb Tecr

G 3
three

A O O
Kgma.kg/webtest/testing

Taqepa Keean MpaeWHOxan

Patient L, 42 years old, consulted


Bonpoc: N939
a gynecologist to receive
marriage. History of 2 births, recommendations on contraception,
2 medical abortions. General suffers from hypertension
35. Recommended method of contraception: condition is satisfactory, for 5 years. Regular sexlifein
blood pressure is 135/90mm Hg,
height 164 cm, weight 90
kg. BMI

OTBeTbI(oAnH OTBeT)
Postcoital contraception (Postinor, Dvella)

OIntrauterine device with silver


3 Surgical sterilization

4 Condom

5 Calendar chythm method

35 36 37 38 39 40 41 42 43 S0
SAMSUNG

71%m0823
Samsung India x LMS KIMA Be6TecrpoBa
C A avn.kgma.kg/wehbtest/testin9

rapr Awxu

Bonpoc: N939
A 20-year-old female patient turned
to clinic, complaining of abundant, purulent
the genital tract, itching, burning and pain in the vagina. She fell ill acutely,2 discharge fron
above complaints appeared. Menstrual function days ago, when the
is not impaired. By examination: the vagnal
mucosa is sharply hyperemic,
covered witha purülent bloom, bleeds easily
cervix is cylindrical in shape, the mucosa is not when touched, The
changed. What is the most likely diagnósi

OTBeTbI(oaMH OTBeT)
1 Acute colpitis
2 VulvovaginitisS
Bartholinitis

Endocervicitis

5 Endometritis

35 36 37
38 3940 41 42 43

50

1 23 45 6 78 9 10
11 1213 1718
14 15 16 19 20

21 22 23 24 25 26 27 28 29 30
31 32 3334 35 36 37 38 39 40
41 42 43 44 45 46 47 48 49 50

OTBe4eHHbi BOnpoc
nponyuteHHbiM Boripoc
Tekyiuw Boripoc

Mo 3asepwenwn TecTa OCTanoC


SAMSUNG

8 Samsung India 71% m 0823


X LMS KTMA X Ee6TecTHpoBa X

C A avn.kgma.kg/webtest/testing

rapr Auxuwa

Bonpoc: NP42
Pathological amenorhea does not include:

OTBeTbi(oAMH OTBeT)
Amenorrhea In Kallmann syndrome

2 Ameniorthea in Sheehan's syndrome

3 Amenorrhea with gonadal dysgenesis

4 Psychogenic amenorrhea
5 Amenorrhea of childhood

38 39 40 41 42 43 44 45 46

50

12 3 4 567 8 9 10
11 12 13 14 15 16 17 18 19 20
21 22 23 24 25 26 27 28 29 30
31 32 33 34 35 36 37 38 39 40
41 62 43 44 45 46 47 48 49 50

OTBeeHHi BOnpoc
riponyieuik sonpoe
Texyun Bonpoe

o saBepuuena TeCTa DCTanOC

0:24:08
308epuinTb TeCT
a6 Tecrvposo
C kgma.kg/webtest/tenting

apr Au

Parametritis-is inflammation
Bonpoc: N943
ofthe:

OTBeTbi(opMH
1
OTBeT)
Cecum

2 Gland
3 Parauterinefiber
4 Ovary

5 Fallopiantube

39 40 41 42 43 44 45 46 47
50

1 2 3 4 56 78 9 10
11 12 13 14 15 16
17 18 19 20
21 22 23 24 25 26 27 28 29 30
31 32 33 34 35 36 37 38 39 40
41 42 43 44 45 46 47 48 49 50
OTBeHeHHbiM BOnpoc
npornyujeHHbih BOnpoe
TeKyuw Bonpoc

Ao 3aBepweHwA TecTa ocTanocb

0:24:43
3aBepwwTb TeCT
SAMSUNG

S Samsung India X l 70%m08 28


LMS KrMA
Be6TecTwpoB
C A avn.kgma.kg/webtest/testing

Tapr AMiKua

Bonpoc: NP48
Through the lesser
sciatic foramen passes
the following structures

OTBeTbi(oAMH OTBeT)
1
the pudendal neurovascular
bundle, internus obturator
muscle
2 the external obturator artery, external
cutaneus nerve of thigh
3 the inferior vesical artery, internal pudendal
artery and vein
4 the posterior cutaneus nerve of thigh, internus
obturator muscle
5 the inferior vesical artery, posterior cutaneus nerve
of thigh

43 44 45 46 47
48 49 50

1 2 3 4 56 7 89 10
11 12 13 14 15 16 17 18 19 20

21 22 23 24 25 26 27 28 29 30
31 32 33 34 35 36 37 38 39 40
41 42 43 44 45 46 47 48 49 50

OTBeHeHHbiM BOnpoc
riponyujeHHbl Bonpoc
TEKyuyin BOnpoc

Ao 3aBepuweHMA TecTa oCTanocb

0:19:29
O Sammungtndia
i MS KTMA
Bef Tec
O C A avn kgma.kg/webtest/testing

Tayr Fo

Bonpoc: N947
The suspensory system of the uterus includes

OTBeTbi(oAMH OTBeT)
cardinal ligaments of uterus

2 ig propria of ovaries

suspensory ligaments of ovaries

rectouterine ligaments

5 broad ligaments of uterus

1 43 44 45 46 47 48 49

1 2 3 45 6 7 89 10
11 12 13 14 15 16 17 18 19 20
21 22 23 24 25 26 27 28 29 30
31 32 33 34 35 36 37 38 39 40
41 42 43 44 45 4647 48 49 50

OTBeveHHbl BOnp0c
nporyueHHblR Bonpoc
Texyuna Bonpoc

Ao sanepuieHwa TecTa oCTanocb

0:18:02
3aBepuiwrb TeCT
ShMSUNG

S SamsingIndiox LMSKTMA l 70m8 29


Be6Tecrvpoes
CA C A avn.kgma.kg/webtest/testing
1
Tapr Aptu

Bonpoc: N948
Through the lesser sciatic foramen
passes the following structures:

OTBETbI(ogMH OTBeT)
the pudendal neurovascular bundle, internus
obturator muscle
2 the external obturator artery, external cutaneus nerve
of thigh
3 the inferior vesical artery, internal pudendal artery and vein

4 the posterior cutaneus nerve of thigh, internus obturator muscle

5 the inferior vesical artery, posterior cutaneus nerve of thigh

43 44 45 46 17 48 49 50

1 2 3 4 5 6 7 B9 10
11 12 13 14 15 16 17 18 19 20
26 29 30
21 22 23 24 25 26 27
31 32 33 34 35 36 37 38 39 40
41 42 43 44 45 46 47 48 49 50

oTBevewHbit Bonpoc
riponyugeubia Bonpoc
Texyunn ponpoc

o sanepuienws recTa oCTanocb

0:17:54
3anepuwTb TeCT
avn.k na.kg/webt
btest/testing
s
rapr
e
Bonpoc: N949
Benign hyperplasia
(excessive growth
would be most likely of cells) of which part
to interfere with the of
passage of urine? the male1eproductive system

OTBeTbi(OAMH
OTBeT)
Peripheral Zone of
the Prostate
2 Ejaculatory Duct

3 Seminal Vesicle
4 Central Zone of the Prostate

5 Periurethral Zone of the Prostate

43 44 45 46 47 48 49 S0

1 2 3 4 56 7 8 9
10
11 12 1314 15 16 17 18 19 20
21 22 23 24 25 26 27 28 29 30
31 32 33 34 35 36 37 38 39 40
41 42 43 44 45 46 47 48 49 50
OTBeveHHbl Bonpoc
nponyueHHbIR Bonpoc
TeKyuiwn Bonpoc

Ao 3aBepuweHHA TeCta oCTanocb

0:17:21
3aBepuwTb TeCT
SAMSUNG

al 69% m08.36
S Samsung Indla x LMS KTMA
Be6 Tecrpoea

C A avn.kgma.kg/webtest/testing

rapr Awku

Bonpoc: N°18
Which of the following statements is the most accurate
regarding the normal changes in the
cardiovascular system at term? Total volume

OTBeTbi(opMH OTBeT)
1
Increases by 20%, plasma volume increase exceeding RBC mass increase

2 Increases by 20%, RBC mass increase exceeding plasma volume increase

3 Increases by 40%, RBC mass increase exceeding plasma volume increase

4 Increases by 40%, volume increase exceeding RBC mass increase

5 No change in the total blood volume

14 15 16 17 18 19 20 21 22

50

2 3 4 56 7 89 10
1
11 12 13 14 15 16 17 18 19 20
21 22 23 24 25 26 27 28 29 30
39 40
31 32 33 34 35 36 37 38
41 42 43 44 45 46 47 48 49 50

oTBeHeHHbIM BOpoc
iponyuueHHbl Bonpoc
TeKyuun Bonpoc

AO 3asepuieHa TeCTa oCTan0cb

0:10:53
3aBepuiHTb TeCT
SAMSUNG

S SamsungIndiax LMSKIMA 69%m08 39


Be6TectwpoBa
O C A avn.kgma.kg/webtest/testing

Tapr cua
Bonpoc: N949
Benign hyperplasia (excessive
growth of cells) of which part
would be most likely to interfere of the malereproductive systerm
with the passage of urine?

OTBeTbI(ogMH OTBeT)
1
Peripheral Zone of the Prostate

2 Ejaculatory Duct
3 Seminal Vesicle
4 Central Zone of the Prostate

5 Periurethral Zone of the Prostate

A3 44 45 46 47 48 49 50

1 2 34 5 6 7 8 9 10
11 12 13 14 15 16 17 18 19 20
21 22 23 24 25 26 27 28 29 30
31 32 33 34 35 36 37 38 39 40
41 42 43 44 45 46 47 48 49 50

oTBeHeHHbl BOnpoc
nporyuueHHbIR Bonpoc
Texyuiwn BOnpoc

ocCTanocb
Ao 3aBepuweHwa TecTa

0:08:27
3aBepuiwTb TeCT
l 98% 8.55 am
S samsung | GLOBAL x Be6 TecTpoBaHMe

C A avn.kgma.kg/webtest/testing

CWHrx KyHBap MpaTan


gRu

Bonpoc: Ne9
The14-year-old girl complained of low growth (on
the scale of height-age", red zone), there are
no secondary sexual characteristics. From
the family history, it is known that the parents are of
average height. What is your presumed diagnosis?

OTBeTbI(ogMH OTBeT)
1 chronic eating disorder

2 Shereshevsky-Turner Syndrome

3 congenital hypothyroidism

constitutional growth retardation

5 family short stature

56 7
89 10 11 1213 50

1
2 3 45 67 89 10
11 12 13 14 15 16 17 18 19 20
21 22 23 24 25 26 27 28 29 30
31 32 33 34 35 36 37 38 39 40
41 42 43 44 45 46 47 48 49 50
OTBeyeHHbIM BOnpoc
nponyuyeHHblM BOnpoc
TeKyunn BOnpoc

Ao 3aBepuweHMA TeCTa oCTanocb.

1:06:23
l 98% B8.56 am
S samsung| GLOBAL X Be6 TecTwpoBaHMe
C A avn.kgma.kg/webtest/testing

CWHrx KyHBap lpaTan

Bonpoc: Ne10
The 12-year-old boy has an appointment with a pediatrician. Objectively: the body weight is 45 kg,
thePFA is excessively developed. There is a symmetrical increase in the mammary glands,
moderate pain on palpation. Choose which clinical syndrome reflects the above complaints?

OTBeTbi(oAMH OTBeT

gynecomastia

hypogonadism

3 hypopituitarism

4 hyperpituitarism

5 Shereshevsky Turner Syndrome

6 7 8 9
1011 12 13 14

50

1 23 4 5 6 7
89 10
11 12 13 14 15 16 17 18 19 20

21 22 23 24 25 26 27 28 29 30

31 32 33 34 35 36 37 38 39 40
41 42 43 44 45 46 47 48 49 50

OTBeyeHHbIM BOnpoc
nponyuyeHHbln Bonpoc
TeKyun BOnpoc

Ao 3aBepueHw TeCTa ocTanocb

1:05:16
98%B9 58 arm
S samsung GLOBAL
Be6 Tectposaee

CA avn.kgma.kg/webtest/testing
O
CHHx KyHBap Tparan

Bonpoc: N°11
Examination of the boy's external genitalia revealed that the external
opening of the urethra is
located in the area of the penoscrotal angle. What is the type of anomaly in a
child?

OTBETbI(oAMH OTBeT)

1 Epispadias
2 Phimosis
3 Paraphimosis

4 Hypospadias
5 Bladder exstrophy

8 10 11 12 13 14 15

50

1 2 3 4 567 8 9 10
31 12 1314 15 16 17 18 19 20
2122 23 24 25 26 27 28 29 30
3132 33 34 35 36 37 38 39 40
41 42 43 44 45 46 47 48 49 50

OTBeeHulh 8onpoe
nponyuyewa Bonpoe
TexyuM BOnpoc

Ao sabepuienin Tecra 0Cranoch

1:04:06
3aepuTb TeCT
S samsung | GLOBAL l 96% 9 19 arm
Be6 TecTMpoBaHMe

A avn.kgma.kg/webtest/testing

CWHrx KyHBap
Nparan gRu
Bonpoc: N°24
Early breastfeeding and
manual stimulation of the breast nipple
to decrease postpartum bleeding. The is a technique that can be
mechanism by which this maneuver works
used
is stimulation

OTBeTbli(OAMH OTBeT)
Sexual stimulation causing vaginal
contractions
2 Stimulation of opiate production

3 Stimulation of prolactin production

Stimulation of prostaglandins

5 Of oxytocin production causing uterine contraction

20 21 22 23 24 25 26 27 28

50

1 2 3 4 5 6 7 8919 10
11 12 13 14 15 16 17 18 20

21 22 23 24 25 26 27 28 29 30
31 32 33 34 35 36 37 38 39 40
41 42 43 44 45 46 47 48 49 50
OTBeveHHbl BonpoOC
nponyueHHbiM Bonpoc
Texyuwn Bonpoc

Ao 3aBepweHwa TeCTa oCTanocb

0:42:21
3aBepwwTb TeCT
95% 927 arm

S samsung | GLOBAL X Be6 TecTpoBaHne


A
C avn.kgma.kg/webtest/testing

CWHrx KyHBap ipaTan Ru

Bonpoc: N°30
To induce ovulation, use:

OTBeTbl(oAMH OTBeT)

Antiestrogens

2 Prostaglandins

3 Antagonists Of Prostaglandins

4 Progestogens

5 Natural Estrogens

29 30 31 32 33 34
26 27 28

50

10
1 2 3 4 5
717 89
18 19 20
11 12 13 14 15 16

21 22 23 24 25 26 27 28 29 30
31 32 33 34 35 36 37 38 39 40
50
41 42 43 44 45 46 47 48 49
OTBeyeHHbl BOnpoc
nponyuyeHHbln Bonpoc
TekyuiuM Bonpoc

TeCTa oCTanocb
Ao 3aBepuweHwA
l 94%929 am
S samsung | GLOBAL
Be6 TecTvpoBaHne

C A avn.kgma.kg/webtest/testing

CWHrx KyHBap lpaTan Kg Ru

Bonpoc: N 33
Research methods of the anatomical and functional state
of the vagina:

OTBETbi(ogMH OTBeT)
1 Inspection with speculum;

2 Puncture of the posterior fornix,

3 Combined
vaginal rectal examination

4 Determination of the purity of the vaginal contents.

5 Cytological examination of the contents of the uterine cavity

29 30 31 32 33 34 35 36 37

50

1 2 3 4 5 6 7 89 10
11 12 13 14 15 16 17 18 19 20
21 22 23 24 25 26 27 28 29 30
31 32 33 34 35 36 37 38 39 40
41 42 43 44 45 46 47 48 49 50
OTBe4eHHBIM BOnpoc
nponyueHHEIM BOnpoc
Texyuwh eonpoc

Ao 3aBepweHwA TeCTa oCTanocb

0:32:36
3aBepuWTb TeCT
l 94%929 am
S samsung | GLOBAL
X Be6 TecTwpoBanne

C A avn.kgma.kg/webtest/testing
0
CWHrx KyHBap npaTan

Bonpoc: N°33
Research methods of the anatomical and functional state of the
vagina

OTBeTbl(OAMH OTBeT)
1
Inspection with speculum

2 Puncture of the posterior fornix

3 Combined vaginal rectal examination,

4 Determination of the purity of the vaginal contents.

5 Cytological examination of the contents of the uterine cavity

29 30 31 32 33 34 35 36 37

50

1 2 3 4 5 6 7 8
9 10
11 12 13 14 15 16 17 18 19 20
21 22 23 24 25 26 27 28 29 30

31 32 33 34 35 36 37 38 39 40
41 42 43 44 45 46 47 48 49 50

OTBeyeHHblM Bonpoc
nponyueHHblM BOnpoc
TeKyuju Bonpoc

Ao 388epuwewwa TeCTa ocTanocb

0:32:27
3aBepuHTb TeCT
94% 9.33 am
S samsung | GLOBAL X Be6 TeCTMpOBaHMe

C A avn.kgma.kg/webtest/testing
o
CWHTx KyHBap npaTan Kg Ru
Bonpoc: N°39
A 23-year-old female patient came to the antenatal clinic with complaints of cramps during
urination, fever up to 37.8 C, purulent discharge from the genital tract. She has several sexual
partners.Examination by speculum: the mucous membrane of the urethra is hyperemic, the cervix
is cylindrical, hyperemia of the area of the external opening of the cervical canal. Discharge from
the cervical canal is purulent. Define the tactics of the antenatal clinic doctor:

OTBeTbi(oAMH OTBeT)
1 Prescribe an outpatient course of antibiotic therapy

2 Get swabs for chlamydia infection

3 Hospitalize the patient in a gynecological hospital

4 Hospitalization in the dermatovenerologic clinic

5 Take swabs, culture for gonorrhea

1 35 36 37 38 39 40 41 42 43

50

1 2 3 4 5 6 7 8 9 10
11 12 13 14 15 16 17 18 19 20
21 22 23 24 25 26 27 28 29 30
31 32 33 34 35 36 37 38 39 40
41 42 43 44 45 46 47 48 49 50

OTBeyeHHblM Bonpoc
nponyujeHHbiM BOnpoc
TekyuwM BOnpocC

oCTanocb
Ao 3aBepuueHwa TeCTa
94% 1935am
S samsung | GLOBAL X Be6 TeCTMpoBaHme

C A avn.kgma.kg/webtest/testing

Bonpoc: N940
Task 2 Patient R., 25 years old, was admitted to the gynecological department with complaints of
pain in the external genital area, awkwardness when walking, fever. Pain appeared four days ago
after hypothermia. Last period 3 weeks ago. By examination: in the area of the right labia majora,
a tumor-like formation measuring 4x3 cm is determined, the skin above it is hyperemic, hot to the
touch, fluctuation is noted on palpation. Examination of the cervix in the speculum and bimanual
examination revealed no abnormalities. What is the most probable preliminary diagnosis:

OTBeTbl (OAMH OTBeT)


1 OPerforation of purulent form ofappendages
2 Torsion of the legs of the ovarian tumor

3 acutebilateral salpingitis, pelvioperitonitis


4 Necrosis of subserous myomatous node

5 Bartholine abscess

36 37 38 39 40 41 42 43 44

50

1 23 4 5 67 89 10
11 12 13 14 15 16 17 18 19 20
21 22 23 24 25 26 27 28 29 30
31 32 33 34 35 36 37 38 39 40
41 42 43 44 45 46 47 48 49 50
OTBeueHHbIMBonpoc
iponyujeHHblM Bonpoc
Tekyuinn BOnpoc

o 3aBepweHwa TeCTa oCTanocb

0:27:05
Al 93 % 19.:36 arm

S samsung | GLOBAL Be6 TeCTMpOBaHMe

C A avn.kgma.kg/webtest/testing

CMHx KyHBap TpaTan KgRu

Bonpoc: Ne41
Which factors not increase the risk of developing inflammatory diseases sexual organs:

OTBeTbI(ogMH OTBeT)
1 Oral contraceptives

2 Medical abortion

3 Sexual initiation at age 15

4 IUCD
5 Hysterosalpingography

37 38 39 40 41 42 43 44 45

50

1 2 3 4 5 67 89 10
11 12 13 14 15 16 17 18 19 20
21 22 23 24 25 26 27 28 29 30
31 32 33 34 35 36 37 38 39 40
41 42 43 44 45 46 47 48 49 50
OTBeyeHHbiM BOnpoc
nponyujeHHbIM BOnpoc
Tekyunn BOnpoc

Ao 3aBepweHwA TeCTa oCTanocb

0:25:40
3aBepuvTb TeCT
al 93% 9.38 anm
TecTMpoBaHme
S samsung | GLOBAL Be6

C A avn.kgma.kg/webtest/testing
CHHrx KyHBap nparan ORu

Bonpoc: N942
central genesis include everything besides:
The causes of secondary amenorrhea of

OTBeTbi(oAMH OTBeT)

Psychogenic amenorrhea

2 Syndrome of hyperprolactinemia

Sheehan syndrome
3
Amenorrhea due to weight loss

5 Kallmann Syndrome

43 44 45 46
40 41 42
38 39

50

3 4 5 67 89 10
12
18 19 20
11 12 13 14 15 16 17
28 29 30
21 22 23 24 25 26 27
38 39 40
31 32 33 34 35 36 37
50
41 42 43 44 45 46 47 48 49
OTBeveHHblMBonpoc
nponyueHHbiM Bonpoc
TeKyunn Bonpoc
TecTa oCTanoCb
Ao saBepweHna
93% 940 am
S samsung | GLOBAL X Be6 TeCTwpoBaHe

C A avn.kgma.kg/webtest/testing

Bonpoc: N944
Patient with amenorrhea has a negative result (absence of menstrual reaction) after the
hormonal test with estrogens and progesterone, it means:

OTBeTbi(ogMH OTBeT)
1 Presence uterine form of amenorrhea

2 Presence ovarian form of amenorrhea

3 Presence central form of amenorrhea

4 Presence polycystic ovary syndrome

5 Presence amenorrhea of central genesisS

40 41 42 43 44 45 46 47 48

50

123 4 5 6 7 8 9 10
1112 13 14 15 16 17 18 19 20
21 22 23 24 25 26 27 28 29 30

31 32 33 34 35 36 37 38 39 40
41 42 43 44 45 46 47 48 49 50

OTBeyeHHblM BOnpoc
nponyujeHHblM Bonpoc
TeKyuni Bonpoc

Ao 3aBepweHna TeCTa OCTanocb

0:21:33
3aBepuinrb TeCT
92%1 94
Ssamsung | GLOBAL X Be6 TeCTMpOBaHve
+

O F C A avn.kgma.kg/webtest/testing

CWHx KyHBap NpaTan

Bonpoc: N°46
The suspensory system of the uterus includes:

OTBETbi(ogVH OTBET)

suspensory ligaments of ovaries

2 rectouterine ligaments

3 lig. propria of ovaries

4 broad ligaments of uterus

5 cardinal ligaments of uterus

42 A3 44 45 46 47 48 49 50

7 89 10
3 4 5 6

11 12 13 14 15 16 17 18 19 20
21 22 23 24 25 26 27 28 29 30
31 32 33 34 35 36 37 38 39 40
41 42 4344 45 46 47 48 49 50
OTBeyeHHblM BOnpoc
nponyueHHbiM BOnpoc
Tekyuwn Bonpoc

Ao 3aBepujeHwa TeCTa ocTanocb

0:18:46
l 92%1945
S samsung | GLOBAL Be6 TecTWpoBaHMe

O F C Oavn.kgma.kg/webtest/testing

CWHTx KyHBap paTan

Bonpoc: Ne47
hich structure is not included in the deep perineal pouch of female?

OTBETbi(oqMH OTBeT)
OCowper's glands

OInferior perineal fascia

Superior perineal fascia

M.transversus perinei profundi


External urethral sphincter

46
1 43 44 45 48 49 50

1 3 4 5 67 89
2 10

1 12 13 14 15 16 17 18 19 20
1 22 23 24 25 26 27 28 29 30
31 32 33 34 35 36 37 38 39 40
1
42 43 44 45 46 47 48 49 50
OTBeyeHHbiM Bonpoc
nponyueHHblM Bonpoc
TekyuMM BOnpoC

o 3aBepweHnI TeCTa OCTanocb.

0:16:49
nonuu1ATh TACT
91% 9.47 am

Text se.. cOPY SHARE SELECT ALL QWEB SEARCH

Oavn.kgma.kg/webtest/testing
f C

Bonpoc: N°48
Innervation of superficial perineal pouch formed by?

OTBETbl(OAMH OTBET)

1 Genitofemoral nerve

2 Perineal nerve

3 Superficial pudendal nerve

4 Internal pudendal nerve

5 Superficial perineal nerve

43 44 4546 47 48 49 50

12 3 4 5 6 7 89 10
11 12 13 14 15 16 17 18 19 20

21 22 23 24 25 26 27 28 29 30
31 32 33 34 35 36 37 38 39 40
41 42 43 44 45 46 47 48 49 50

OTBeyeHHb1M BOnpoc
nponyujeHHbl BOnpoc
TeKyuinn Bonpoc

ocTanoch
Ao 3aBepuieHwa TeCTa

0:14:22
3aBepunTb TeCT

C 4344454647484950
A91%1 948 ar
S samsung | GLOBAL x Be6 TecTupoBaHne

C O avn.kgma.kg/webtest/testing
CMHrx KyHBap lpaTan

Bonpoc: N949
part of the scrotum. Two
During a vasectomy, the ductus deferens is ligated in the superior
months following this sterilization procedure, the subsequent ejaculate contains

OTBeTbi(oqMH OTBET)

Sperm and seminal fluid

2 Seminal fluid and prostatic fluid

3 Sperm only

only
4 Prostatic fluid
seminal fluid, and prostatic fluid
5 Sperm,
46 48 49 50
43 4445 47

1 2 3 4 567 8 9 10
11 12 13 14 15 16 17 18 19 20

21 22 23 24 25 26 27 28 29 30
31 32 33 34 35 36 37 38 39 40
41 42 43 44 45 46 47 48 49 50

OTBeyeHHbI BOnpoc
nponyujeHHbIM BOnpoc
Tekyunn BOnpoc

Ao 3aBepueHwA TeCTa ocTanocb

0:13:38
S samsung | GLOBAL
91% 948 am
X Be6 TeCTupOBaHMe
X

C avn.kgma.kg/webtest/testing

CHHTx KyHBap IpaTan Kg Ru


Bonpoc: N°50
Which structure should be taken
into account not to damage while ligating the uterine artery
during hysterectomy (amputation of uterus)?

OTBETbI(oqMH OTBeT)
1 Urethra

2 Pelvic splanchnic nerves

3 Pudendal nerve

4 Ovarian artery

5 Ureter

43 44 45 46 47 48 49 50

1 2 3 45 6 7 8 9 10
11 12 13 14 15 16 17 18 19 20
21 22 23 24 25 26 27 28 29 30
31 32 33 34 35 36 37 38 39 40
41 42 43 44 45 46 47 48 49 50
OTBeyeHHblM BOnpoc
nponyuyeHHbIM BOnpoc
TeKyuinn BOnpoc

40 3aBepweHuA TeCTa oCTanocb

0:13:29
3aBepuinTb TeCT
Bonpoc: N°46
The uterine artery is a branch of:

OTBeTbI(OAMH OTBeT)

thecommon iliac artery


the inferior epigastric artery

theobturator artery
theexternal iliac artery
the internal iliac artery

42 43 44 45 46 47 48 49 50
Bonpoc: Ne48
The urinary bladder in relation to prostate gland is situated:

OTBETbI(oaMH OTBET)
1 posteriorly
22 superiorly

3 postero-inferiorly
4 anteriorly

5 inferiorly

1 43 44 45 46 47 48 49 50
Bonpoc: N913
Male, 30 years old, maried for 4 years. Have no children A history of trauma to both testicle
the age of 20, for which an organ-preserving operation was performed (which one the patien
does not know exactly) on both testicles. On examination: both testicles are reduced in size,
dense consistency, the vas deferens are unremarkable. The prostate gland is unchanged
Spermogram. No spermatozoa or spermatogenesis cells were found in the ejaculate. Total
testosterone intheblood is 4 nmol /I (the norm is 12-35 nmol/). What is your diagnosis?

OTBETbI(oaMH OTBeT)
1 Cryptorchidism
2 Late onset hypogonadism

3 Testicular hypogonadism, azoospermia.

4 Testicular cancer

5 Congenital atrophy of testicles

9 10 11 12 14 15 16 17

50
Bonpoc: N 47
The peritoneum covers the following areas of anterior surface of uterus

OTBeTbI(oquH OTBeT)
1 not covered at all

2 body, fundus of uterus

3 the body and supravaginal part of uterus

4 the body of uterus, supravaginal part of cervix, anterior vaginal fornix

5 the body and entire cervix of uterus

43 44 45 46 47 48 49 50
Bonpoc: Ne45
Which structure from following not included in perineal body containment?

OTBeTbi(oqMH OTBeT)

1 the superficialtransverse perineim

2 therectovaginal fascia (Denonvillier's)


3 the m. ischiocavernosus

4 the m. bulbospongiosus

55 the deep transverse perinei muscle and its fascia

41 42 43 44 45 46 47 48 49
Bonpoc: Ne49
Which structure is outlined with contrast on a CT using intraperitoneal contrast material?

OTBeTbl(oAMH OTBET)
1 Ovary

2 Rectum

3 Prostate
4 Vagina

5 Seminal Vesicles

1
* 43 44 45 46 47 48 49 50
Bonpoc: N°41
Pathological amenorrhea does not include

OTBETbI(oAUH OTBeT)
1
0Amenorrhea in Kallmann syndrome
2 O Psychogenic amenorrhea

OAmenorrhea in Sheehan's syndrome

O Amenorrhea of childhood

O Amenorrhea with gonadal dysgenesis

37 38 39 40 41 42 43 44
Bonpoc: N942
Pathogenetic therapy of amenorrhea in Kallmann syndrome:

OTBeTbi(oAuH OTBeT)
1 Multivitamins

2OPhystotherapy
3 Diuretics and antidepressants

4 Antidepressants and multivitamins

5 OHormone replacementtherapy

1 38 39 40 41 42 43 44 45 46

50
Bonpoc: N 43
latrogenic amenorrhea can be caused by all except:

OTBeTbi(oqnH OTBeT)
1 0 Taking GnRH agonists

2 O cauterization of the ovaries

3 O Bilateral oophorectomy

4 Taking MAO inhibitors

5 Total hysterectomy

1 39 40 41 42 43 44 45 46 A

50
Bonpoc: N°38
The conditions for cyclic horrnone therapy In pallentn with nenstrual rretjularition afe

OTBeTbI(ognH OTBeT)
1
Hormone testlng
2 The use of a minimum arnount of estrogen

3 Preliminary consultation by a neurologist


4 Use of gestagens, ultrasound exarnination of the uterus
5 Preliminary consultation of the therapist

1 34 35 36 37 38 39 40 41 42

50
Bonpoc: Ne44
Patient A 28 years old, was admitted to the gynecological department with complaints
spotting for two weeks. Ultrasound scan was revealed persistence of the follicles. Wha
hormonal changes can you expect?

OTBeTbl(oAMH OTBeT)

1 Hyperandrogenemia

2 Hyper progesteronemia

3 Hypoestrogen saturation

4 Hyperestrogenemia

5 Failure of glucocorticoids

40 41 42 43 44 45 46 47 48

50
Bonpoc: N 39
For conservative treatment of Abnormal uterine bleeding the 1st line drug is:

OTBeTbi(ogMH OTBeT)
1 sodium ethamsylate

2 oxytocin

3 tranexamic acid

4 0aminocaproic acid

5 O NSAIDS

1 35 36 37 38 39 40 41 42 43

50
Bonpoc: N°40
A mother with a 4-year-old girl turned to a pediatric gynecologist, who has itchlng and redness
the external genital area, purulent discharge from the genital tract. These symptoms occur
periodically throughout the year. Genital hygiene is observed. The child is observed by an alle.
In connection with atopic dermatitis (skin rashes on the elbows and on the face). What is the
most ikely diagnosis

OTBETbI(oquH OTBeT)
1 Bartholinitis

2 Salpingo-oophoritis
3 O Atopic vulvovaginitis

4 Endocervicitis
5 Endometritis

36 37 38 39 40 41 42 43 44

50
Bonpoc:N937
What is the mechanism of action of voluntary surgical sterilization

OTBeTbI(oAMH OTBeT)
1
Suppression of ovulation

2 O Thickening of cervical mucus

3 O Violatlon of implantation

4 OBlockage of the falloplan tubes

5 O Thickening of cervical mucus

1 33 34 35 36 37 38 39 40 41

50
Bonpoc: N 31
The most common cause of female infertility is:

OTBeTbI(oqMH OTBeT)
1 Endocrinefactor

2 O Immunological factor

3 O Tubal-peritoneal factor

4 OUterinefactor
5 Psychosexual disorders

1 27 28 29 30 31 32 33 34 35

50
Bonpoc: N®35
What Is the most cormmon presenting cornplaint of a wornan ith vaylnitis?

OTBeTbI(ouH OTBeT)
1 Dyspareunia

2 Pelvic paln
3
Vaginal discharges

Irregular menses

O Fever

1 ** 31 32 33 34 35 36 37 38 39
Bonpoc: Ne36
The high efficiency of the method of lactational amenorrhea (LAM), subject to all the rules, is
observed:

OTBeTbI(oAMH OTBeT)

10Within 12 months after childbirth


2 O Within 9 months after childbirth

3 Within 6 months after childbirth

4 Within 15 months after giving birth

50 Within 3 months after childbirth

32 33 34 35 36 37 38 39 40

50
Bonpoc: Ne34
The mechanism of action of hormones on the cell is due to the presence of

OTBeTbI(oguH OTBeT)
1
OThromboxant
2 O Isoenzymes

3 OProstaglandins

4 OReceptors
5O Specific Enzymes

30 31 32 33 34 35 36 37 38

50
Bonpoc: N932
Name the composition of injectable contraceptives:

OTBeTbI(oAMH OTBer)
1 Long-acting Progestogens

2 Mixed doses of hormones

3 Small doses of a progestogen

4 Antiandrogens

5 Conjugated estrogens

1 28 29 30 31 32 33 34 35

50
Bonpoc: N30
which af the folliowing infiammatary diseasesin e acute sage is zccompariedby smptom
af bryuschina iritation

OTBETBI(ogMH OTBET)

1 Vulvovagintis

2 Bartholintis

3 Coipitis
Endometritis

5 Cervictis

26 2

50
Bonpoc: N°29
Duration of the menstrual cycle:

OTBeTbi(oqMH OTBET)
1 24-32days

2 21-35days

3 O25-37days
4 0 28-29days

5 35-39days

25 26 27 28 29 30 31 32

50
Bonpoc: N 33
In the anovulatory menstrual cycle, basal temperature is characterized by:

OTBeTbI(oAMH OTBeT)
1 Rise In Temperature Before Menstruation

2 Rise In Temperature Immediately After Menstruation

3 Rise In Temperature Before Ovulation

4 O Rise In Temperature After Ovulation

5 OLack Of Temperature Rise

1 29 30 31 32 33 34 35 36 37

50
Bonpoc: N°27
A 19 year-old primigravlda is expecling her first-child, she is 12 weeks pregnant by last menstrua
period daling. By external examination the uterline fundus is at the level of pubis. Determine the
expected date of pregnancy?

OTBeTbi(oquH OTBer)
1
O12 weeks

2 6 weeks

3 10 weeks

4 O16 weeks
5 8 weeks
1 23 24 25 26
27 28 29 30 31

50
Bonpoc: N°28
pregnant presents to the 0B clinic. She is 30 weeks pregnant by last menstrual
A 25 year old,
pregnant woman in the fundus of the
period dating. With an external obstetric examination of a
Leopold's
uterus determined a round, dense, balloted part of fetus. Which of the following
maneuvers determines this?

OTBeTbI(oAuH OTBeT)

1
First maneuvers
2 Third maneuvers
3 Fourth maneuvers

4 Second maneuvers

5 Fifth maneuvers

24 25 26 27 28 29 30 31 32

50

1 23 45 6 7 89 10
11 12 13 14 15 16 17 18 19 20

23 24 25 26 27 28
2122 29 30
31 32 33 34 35 36 37 38 39 40
41 42 43 44 45 46 47 48 49 50

OTBeeHHbIM BONpoc
nponyueHHBIM BOnpoc
Tekynn Bonpoc

Ao 3aBepuweHwa TecTa oCTanocb

1:07:45
Bonpoc: N 21
After the first four months of pregnancy, the chief source of estrogen and progesterone is t

OTBeTbI(0gMH OTBeT)
1 0 Corpus luteum

2 O Anterior hypophysis

3 Placenta
4 chorion
5 0 Adrenal cortex

17 18 19 20 21 22 23 24 25

50
Bonpoc: N°25
What is the role of digital exam in the evaluation of normal labor?

OTBeTbI(oAMH OTBeT)
1
0Assessment of fetal well-being
2 O Fetal movements

3 O Time of labor

4 O Degree of dilatations of the cervix

5 Frequency and duration of contractions

1 21 22 23 24 25 26 27 28 29

50
Bonpoc: Ne24
To calculate the expected date of dellvery (EDD) in primaparouse women to the date of the fr
fetal movements should be added:

OTBeTbI(oaMH OTBeT)
1
18 weeks

2 22 weeks

3 25 weeks

4 20 weeks

5 16 weeks

20 21 22 23
24 25 26 27 28

50
Bonpoc: N 26
How is the first labor characterlzed? Labor begins with:

OTBeTbi(oAuH OTBeT)

10Regular uterine contractions and ends with engagement of the head

2 0 Regular uterine contractions and ends with full dilatation of cervix

3 O Regular uterine contractions and ends with effacement of cervix

4O Regular uterine contractions and ends with decent of the head

5 Rupture of membrane and ends with full dilatation of cervix

22 23 24 25 26 27 28 29 30

50
Bonpoc: N 22
A baby Is born In the following conditlon at 1 minute. She is pink with acrocyanosis, a heart ra
of 120 bpm. She is crying, sneeze. Her limbs are flexion. What is her Apgar score?

OTBeTbI(oquH OTBeT)

109
207
3 8
4 O6

1 18 19 20
2122 23 24 25 26

50
Bonpoc: N°50
Female 23 years old came to gynecological department with swelled and enlarged labia majora
on one slde. Examination revealed inflammation of Bartholin's gland cyst. Determine the locatic
of Bartholin's glands according to topography In anatomy?

OTBeTbI(oAuH OTBeT)
1 Superficial perineal pouch

2 O Labia majora

3 Deep perineal pouch

4 Labla minora

5 Vaginal vestibule

1 43 44 A5 46 47 48 49 50
Bonpoc: N944
Pallent A, 28 years old, was admitted to the gynecological department with complaints of
spotting for two weeks. Ultrasound scan was revealed persistence of the follicles. What
hormonal changes can you expect?

OTBeTbi(oAWH OTBeT)
1 Hyperandrogenemla
2 Hyper progesteronemia

3 Hypoestrogen saturation

4 Hyperestrogenemia
5 Failure of glucocorticoids

1 40 41 42 43 44 45 46 47 48

50
Bonpoc: N933
In the anovulatory menstrual cycle, basal temperature is characterized by:

OTBETbI(oqMH OTBeT)
1 Rise In Temperature Before Menstruation

2 Rise In Temperature Immediately After Menstruation

3 Rise In Temperature Before Ovulation

4 Rise In Temperature After Ovulation

5 Lack Of Temperature Rise

29 30 31 32 33 34 35 36 37

50

2 3 4 5 678 910
1

11 12 13 14 15 16 17 18 19 20
21 22 23 24 25 26 27 28 2930
31 3233 34 35 36 37 38 39 40
41 42 43 44 45 46 47 48 49 50
OTBeueHHbIM Bonpoc
iponyueHHbl Bonpoc
TexyuuA Bonpoc
Bonpoc: N940
A mother with a 4-year-old girl turned to a pediatric gynecologist, who has itching and redness
the external genital area, purulent discharge from the genital tract. These symptoms occur
perlodically throughout the year. Geital hygiene is observed. The child is observed by an allerg
in connection with atopic dermatitis (skin rashes on the elbows and on the face). What is the
most likely diagnosis:

OTBeTbi(oquH OTBeT)
1 Bartholinitis

2 Salpingo-oophoritis

3 O Atopic vulvovaginitis

4 Endocervicitls
5 Endometritis

37 38 39 41 42 43 44
1 36

50
Bonpoc: N°30
Which of the following inflammatory diseases in the acute stage is accompanied by symptoms
of bryuschina irritation:

OTBeTbI(oquH OTBeT)
1 OVulvovaginitis

2 O Batholinitis

3 O Colpitis

4 0Endometritis
5 Cervicitis

1 26 27 28 29 30 31 32 33 34

50 P
Bonpoc: N 37
What is the mechanism of action of voluntary surgical sterilization:

OTBeTbi(oAuH OTBeT)
1
Suppression of ovulation

2 O Thickening of cervical mucus

3OViolationof implantation
4 O Blockage of the fallopian tubes

5 OThickening of cervical mucus

1 33 34 35 36 37 38 39 40 41

50
Bonpoc: N 28
25 year old,pregnant presents to the OB clinic. She is 30 weeks pregnant by last menstrual
A
period dating. With an external obstetric examination of a pregnant woman in the fundus of the
uterus determined a round, dense, balloted part of fetus. Which of the following Leopold's
maneuvers determines this?

OTBETbI(oqMH OTBeT)

1 First maneuvers
2 Third maneuvers
3 Fourth maneuvers

4 Second maneuvers
5 Fifth maneuvers

24 25 26 27 28 29 30 31 32

50
Biparietal diameter of the fetal head:

OTBeTbI(oAMH OTBeT)

1 9.5

2 10.5

3 12.5cm
4 8.5 cm
5 11.5cm
Bonpoc: N°18
The labor has the following stages

OTBeTbI(oAMH OTBeT)

1 Preparation, dilation, expulsion, early postpartum

2 Preliminary, dilatation, expulsion, postpartum

3 Preparation, dilation, expulsion

Effacement of the cervix, the expulsion, delivery of placenta

5 Dilation, expulsion, delivery of placenta, early postpartum

14 15 16 17 18 19 20 21 22
50
Bonpoc: N°19
Which of the following investigations are performed at the antenatal visitat 30 weeks of
pregnancy?

OTBeTbI(opMH OTBeT)

1 Rubella
2 cytomegalovirus (CMV)

3 Toxoplasmosis
4 Syphills
5 Hepatitis A

15 16 17 18 19 20 21 22
50
Bonpoc:N°17
Which of the following hormones is not synthesized by the placenta?

OTBETbi(oAMH OTBeT)

1 Prolactin

2 Progesterone

3 Estriol
4 Human chorionic gonadotropin,
5 Human placental lactogen

13 14 15 16 17 18 19 20 21

50
Bonpoc:N°16
A sign of placental separation from the uterus is

OTBETbI(oaMH oTBeT)

1 Contraction of the uterus on palpation

2 Softening of the uterus on palpation

3 The umbilicalcord shortened


4 The umbilical cord lengthens
5 Lack of bloody discharge

12 13 14 15 16 17 18 19 20

50
Bonpoc: N°10
The17-year-old boy was examined. Objectively: men's tone of voice, the protrusion (Adams
Apple)thyroid cartilage. On the pubis, thick, curly hair that spreads on the inner surface of the
thighand is directed to the navel, in the armpit, hair that curls all over the hollow. Evaluatethe
boy's sexual development according to Tanner.

OTBeTbI(oAuH OTBeT)

1 V1 P1 LO AxO FO

2 v2 P5 L2 Ax4 F3

3 v2 P5 L2 Ax4 F3

4 1 P4 L1 AxO FO0

5 V2 P3 L1 Ax2 FO

7 8 10 11 12 13 14

50
Bonpoc: N°19
Which of the following investigations are performed at the antenatal visit at 30 weeks of
pregnancy?

OTBeTbi(oqMH OTBeT)
1 Rubella
2 Cytomegalovirus (CMV)

3 Toxoplasmosis
4 Syphilis
5 Hepatitis A

15 16 17 18 19 20 21 22 23
50
Bonpoc: N°2
Which medical imaging is used to obtain an image of the mucous surface of the uterine cavity?

OTBeTbi(oaH OTBeT)
1 Hysteroscopy

2 Ultrasound

3 Sonohysterography

4 Ct-scan
5 MRI

3 5 7 50

23 4 567 8 9 10
11 12 13 14 15 16 17 18 19 20
21 22 23 24 25 26 27 28 29 30
31 32 33 34 35 36 37 38 39 40
41 42 43 44 45 46 47 48 49 50

oTBeveHHbi BOnpoc
nponyueHHbIA BOnpoc
texyunA Bonp0c
Bonpoc: N°7
Identify the main clinical signs of delayed sexual development in girls

OTBeTbI(oAMH OTBeT)
1 amenorrhea, the mammary glands are poorly developed

2 the presence of menstruation and developed mammary glands

3 amenorrhea, developed mammary glands

4 amenorrhea, mammary glands do not exist

5 the presence of menstruation, in the absence of mammary glands

3 4 6 7 8 9 10 11 5
Bonpoc: Ne6
Specify the reasons for delayed sexual development

OTBeTbI(oAMH OTBer)

1 intrauterine infection

2 infectious diseases

3 chromosomal anomaly
birth trauma

5 stressful situation

2 3 4 5 6 7 8 9 10 50
Bonpoc: N°8
A 9-year-old boy was examined. Objectively: a change in the timbre of the voice, an increasein
thewidth ofthe shoulder girdle, hair in the armpits, pubic area, an increase in the penis and
testicles.Forthis objective status, specify the presumed diagnosis?

OTBeTbi(oquH OTBET)
1 premature sexual development

2 the child is healthy

3 hypopituitarism

4 false male hermaphroditism

5 delayed sexual development

5 6 10 11 12
Bonpoc: N°5
Determine at what age the absence of secondary sexual characteristics is considered a sign of
delayed sexual development in girls.

OTBeTbi(oAMH OTBeT)

1 13-17

2 10-13

12-16

4 14-18

5 11-14

2 3 5
7 9 50
Bonpoc: N94
finding on the endometrium. Most probable
trasound examination revealed hyperechoic
study to do next to clarify the
agnosis is endometrium polyp. What is the best imaging
agnosis?

OTBeTbi(oAnH OTBeT)

Pelvic X-ray

2 Hysterosalpingography

Hysteroscopy

4 MR

s CTscan
Bonpoc: N°11
Thelower limit of the norm for the number of sperm in the ejaculate is (according to WHO
criteria)

OTBeTbi(oAMH OTBeT)

1 40 million
2 100milion
3 80 million
4 60 million

5 20 million

8 9 10
11 12 13 14 15

50
Bonpoc: N°11
Thelowerlimit of the norm for the number of sperm in the ejaculate is (according to WH0
criteria)

OTBeTbi(oAuH OTBeT)

1 40 million

2 100 million

3 million

4 60 million

5 20 million
Bonpoc: N°12
Synorchidism is:

OTBeTb(oaMH OTBeT)
1 Testicular cross-dystopia

2 Intraperitoneal fusion of the testicless

3 Testicularhypotrophy

4 Congenital absence of testicles

5 Having one testicle

9 10 11 12 13 14 15 16

50
Bonpoc: N°12
Synorchidism is:

OTBeTb(oaMH OTBeT)
1 Testicular cross-dystopia

2 Intraperitoneal fusion of the testicless

3 Testicularhypotrophy

4 Congenital absence of testicles

5 Having one testicle

9 10 11 12 13 14 15 16

50
Bonpoc: N 3
Inwhich of the following conditions isrecommended to perform Hysterosalpingography?

OTBeTbI(oAMH OTBeT)
1
Heavy periods

2 infertility

3 Pelvic infection
4 Bloody discharge

Pregnancy

50
6 Samsung India | Mobile |TVI X Be6 TecTHpoBaHve

CAavn.kgma.kg/webtest/testing

KyMap IpaBHH

Bonpoc: N®1

Ultrasound examination using saline solution to visualize uterus is:

OTBeTbi(oAMH OTBeT)
CystoscopPY

2 Sonohysterography

Ultrasonography

Hysteroscopy

5 Hysterosalpingography

4 5
6 50

Scanned with CamScanner


KyMap npaBMH

Bonpoc: N°2
Which medical imaging is used to obtain an image of the mucous surface of the uterine cavity?

OTBeTbI(oAMH OTBeT)
Ultrasound

MRI

3 Ct-scan

Hysteroscopy

5 Sonohysterography

23 4 5 6 50

Scanned with CamScanner


S Samsung India | Mobile TV x Be6 TecTpoBaHne

C A avn.kgma.kg/webtest/testing

KyMap npasHH Kg Ru

Bonpoc: N 3
A 32-year female was consulted to the doctor with complain to heavy and painful periods. Doctor suggested pelvic endometriosis. Which imaging modality is
the first choice?

OTBETbi(oqMH OTBeT)
Ultrasonography

MRI

Hysteroscopy

CT-scan

5 CystoscopPy

50

Scanned with CamScanner


SSamsung India | Mobile | TV|X Be TecTwpOBaHMe

C A avn.kgma.kg/webtest/testing

KyMap 1paBwH KgRu

Bonpoc: N°4
A 43-year-old man came to the clinic with complaints of acute pain in the groin and frequent painful urination. He has a fever and weakness for the second
day.Ultrasound examination of the prostate revealed hypoechoic areas in the periphery. He was diagnosed with a suspected prostate abscess. What imaging
studyis preferred to clarify this diagnosis?

OTBeTbI(ogMH OTBeT)
CT-scan

Nuclear imaging

Xray

Pelvic MRi

Fluoroscopy

2
34 6

Scanned with CamScanner


TecTHpoBaHne
Mobile | TV| X Be6
S Samsung India |
avn.kgma.kg/webtest/testing Kg Ru
C A KyMap paBMH

Bonpoc: N°5
of premature puberty:
Recognize the main clinical signs

OTBeTbI(ognH OTBeeT)

changes of the fundus


1 congestive
sexual characteristics at the age of less
than 6 years.
2 the appearance of repetitive
emotional stability
older than 10 yearsS
theappearance of secondary sexual characteristics
5 delaying intellectual development

2 3 4 567 8

Scanned with CamScanner


TeCTHpOBaHMe
Samsung India | Mobile
|
TV|X Be6
6
Aavn.kgma.kg/webtest/testing
C
KyMap lpaBHH Kg Ru

Bonpoc: N°6
development in boys:
Identify the main clinical signs of delayed sexual

OTBeTbI(oAMH OTBeT)

1 eunuchoid physique

2 the rapid growth of the testes, but the lack of body hair on the male type

3 no testicular growth, but the beginning of voice changes

4 rapid testicular growth, and voice changes

notesticular growth, but hair growth in the armpit

23 4 5
7 10 50

Scanned with CamScanner


TecTMpOBaHe
Samsung India Mobile TV| Be6
|
X

CA avn.kgma.kg/webtest/testing

KyMap npasMH Kg Ru

Bonpoc: N°7
At what age are boys determined by the appearance of facial hair, pigmentation of the scrotum, the first ejaculation?

OTBeTbi(oguH OTBeT
12-13

14-15

10-11

4 8-9

5 16-17

3 4 5 89 10 11

Scanned with CamScanner


TecTwpoBaHne
Samsung India | Mobile| TV|
X Be6
6
avn.kgma.kg/webtest/testing
C A
KyMap npaBMH
Kg Ru

Bonpoc: N°8
area, the appearance of the first
enlarged, hair in the armpits, pubic
old. Objectively: the mammary glands are significantly
8 years can be made for a girl?
At the reception, the
girl is
description what kind of preliminary diagnosis
menarche. Find out from the

OTBeTbI(oquH OTBeT)

healthy child

2 premature sexual development

sexual crisis

delayed sexual development

5 hypothyroidism

10 11 2

Scanned with CamScanner


6 Samsung India | Mobile| TV X Be6 TecTupoBaHWe
+

CA avn.kgma.kg/webtest/testing

Kymap paBMH Kg Ru

Bonpoc: N°9
The14-year-old girl complained of low growth (on the scale of "height-age", red zone),
there are no secondary sexual characteristics. From the family history
itis known that the parents are of average height. What is your presumed diagnosis?

OTBeTbi(oAMH OTBeT)

1 constitutional growth retardation


congenital hypothyroidism

3 family short stature

4 Shereshevsky-Turner Syndrome

5 chronic eating disorder

5 7 8 10 1112 13

Scanned with CamScanner


S Samsung India | Mobile| TV X Be6TecTpOBaHMe +

CA avn.kgma.kg/webtest/testing

KyMap npaenH Kg Ru

Bonpoc: N°10
On inspection at the walls of 11 years old. Objectively: the mammary glands are significantly protruding along with the nipple, have the shape of a cone.
There is no hair on the pubis or under the armpit. There is no menu. Evaluate the girls sexual development according to Tanner

OTBeTbi(oAVH OTBeT)
Mao Po Axo Me0

Ma2 P1 Axo Meo

Ma3 P3 Ax3 Me

Mas P2 Ax2 Meo

Ma3 P3 Ax1 Me1

6 7 8 10 11 1213 14

Scanned with CamScanner


TecTWpOBaHMe
TV X Be6
Samsung India | Mobile |
S
avn.kgma.kg/webtest/testing
Kg Ru
C A KyMap pasH

Bonpoc: N°11
criteria)
ejaculate is (according to WHO
lower limit of the norm for the number of sperm in the
The

OTBeTbI(oAMH OTBeT)

40 million

2 80 million

3 100 million

60 million

5 20 million

7
910 11 12 13 14 15 50

Scanned with CamScanner


TeCTMpOBaHMe
Samsung India | Mobile |
TV X Be6
S
Aavn.kgma.kg/webtest/testing Kg Ru
C KyMap npasMH

Bonpoc: N°12
occurs after
Dilution of the ejaculate normaly

OTBeTbI(oqMH OTBeT)

30-50 min.

2 50-60 min

8 min

4 10-30 min.

5 >1 hour

B 9 10 1112 13 14 15 16 50

Scanned with CamScanner


TeCTHpOBaHne
Samsung India | Mobile TV|
| Xx Be6
S
avn.kgma.kg/webtest/testing Ru
C A KyMap npaBMH Kg

Bonpoc: N°13
operation was
which an organ-preserving
testicles at the age of 20, for vas
years. Have no children. A history of trauma to both
testicles are reduced in size of dense consistency. the
Male, 30 years old, married for 4 examination: both ejaculate. otal
on both testicles. On
performed (which one the patient
does not know exactly) No spermatozoa or
spermatogenesis cells were found in the
unchanged. Spermogram.
prostate gland is
deferens are unremarkable. The 71). VWhat is your diagnosis?
blood is 4 nmol/1 (the norm is 12:35 nmol
testosterone in the

OTBeTbi(oaMH OTBeT

Congenital atrophy of testicles

Testicular hypogonadism, azoospermia

3 Cryptorchidism

4 Testicular cancer

5 Late onset hypogonadism

Scanned with CamScanner


6 Samsung India Mobile TV
| |
X Be6TecTWpOBaHe

C A avn.kgma.kg/webtest/testing

Bonpoc: N914
What is the occipitofrontal diameter of fetal head?

OTBETbI(ogMH OTBeT)

9.5 cm, circumference 32 cm

8.0 cm, circumference 30 cm

3 10 cm, circumference 33 cm
4 12 cm, circumference 34 cm

cm, circumference 38 cm
135
10 11 12 13 14 5 16 17 18

23 456 78 9 10

Scanned with CamScanner


Samsung India | Mobile TVX Be6 TecTMpoBaHHe

CAavn.kgma.kg/webtest/testing

Bonpoc: N 15
Which of the following statements is the most accurate regarding the normal changes in the cardiovascular system at term? Total volume:

OTBETbi(oAMH OTBeT)

1 Increases by 40%, RBC mass increase exceeding plasma volume increase


2 Increases by 20%, RBC mass increase exceeding plasma volume increase

Increases by 40%, volume increase exceeding RBC mass increase

Increases by 20%, plasma volume increase exceeding RBC mass increase

5 No change in the total blood volume

50
11 12 13 14 15 16 17 18 19

Scanned with CamScanner


Bonpoc: N°16
Name the 5th movement of the mechanism of labor in the anterior occipital presentation:

OTBeTbi(oqWH OTBeT)

Sacral rotation

Extension of the head

Flexion of the headE

internal head rotation

External head rotation

15 16 18 19 20 50
12 1314

Scanned with CamScanner


TecTHpoBaHMe
Samsung India | Mobile| TV|
X Be6
S
avn.kgma.kg/webtest/testing
C A

Bonpoc: Ne17
not synthesized by the placenta?
Which of the following hormones is

OTBeTbI(oqMH OTBeT)

Human chorionic gonadotropin,

2 Human placental lactogen,


Estriol

Prolactin

Progesterone,

13 14 15 16 17 18 19 20 21 50

9 10

Scanned with CamScanner


Bonpoc: N18
The labor has the following stages

OTBeTbI(oqMH OTBeT)
Preliminary, dilatation, expulsion, postpartum

2 Effacement of the cervix, the expulsion, delivery of placenta

Preparation, dilation, expulsion, early postpartum

Dilation, expulsion, delivery of placenta, early postpartum

Preparation, dilation, expulsion

14 15 16 17
18 19 20 21 22 50

2 3 4 5678 9 10

Scanned with CamScanner


TecTupOBaHHe
Samsung India | Mobile|
TV X Be6
S
avn.kgma.kg/webtest/testing
C A

Bonpoc: N°19
estrogen and progesterone is the:
pregnancy, the chief source of
After the first four months of

OTBeTbI(oqMH OTBeT)

Anterior hypophysis

2 Chorion

Adrenal cortex

Corpusluteum

5 Placenta

15 16 17 18 19 20 21 22 23 50

2 3 4 5 67 89 10

Scanned with CamScanner


TecTMpOBaHne
X Be6
Samsung India | Mobile | TV|
avn.kgma.kg/webtest/testing
C A

Bonpoc: N°20
physiology?
the following is not true
about maternal cardiovascular
During pregnancy, which of

OTBeTbi(oAuH OTBeT)

1 Extracellular fluld volume decreases

2 Total peripheral resistance decreases

3 Heart rate increases from an average of 70->85

4 Blood flow to brain increases 150-250 ml/min

5 Uterine blood flow increases by 500 ml/min


50
16 17 18 19 20 2122 23 24

2 3 4 56 7 8

Scanned with CamScanner


TecCTHpOBaHMe
Samsung India Mobile | TV X Be6

C A avn.kgma.kg/webtest/testing

Bonpoc: N°21
A sign of placental separation from the uterus is

OTBeTbI(ogMH OTBeT)

Lackofbloody discharge
Contraction of the uterus on palpation

The umbilical cord shortened

4 Softening of the uterus on palpation

umbilicalcord fengthens
The
17 18 19 20
21 22 23 24 25

89 10

Scanned with CamScanner


S Samsung India | Mobile| TV| X Be6 TecTHpoBaHne

CA avn.kgma.kg/webtest/testing

Bonpoc: N°22
Which of the following conditions is common in pregnant women in the 2nd trimester of pregnancy?

OTBETbi(ogMH OTBeT)

10 Respiratory acidosis

Physiologic anemia

3 Metabolic alkalosis
4 Disuria

5 Mastitis

18 19 20 21 23 24 25 26 50

12 3 45 6 7 89 1

Scanned with CamScanner


mpee: NP23

Oraer(e erser)

Scanned with CamScanner


TecTHpoBaHMe
Samsung India | Mobile | TVX
Be6
6 avn.kgma.kg/webtest/testing
C A
Bonpoc: N°23 mechanism by which
The
that can be used to
decrease postpartum bleeding.
nipple is a technique
manual stimulation of the breast
Early breastfeeding and
maneuver works is stimulation:
this

OTBeTbi(oAuH OTBeT)

stimulation of opiate production


1
2 Sexual stimulation causing vaginal contractions

3 Stimulation of prolactin production

4 Stimulation of prostaglandins

uterine contraction
5 Of oxytocin production causing
19 20 21 22 23 24 25 26 27 50

2 3 4 5 6 78 9 10

Scanned with CamScanner


TecTHpoBaHMe
India | Mobile|TV|
X Be6
Samsung
avn.kgma.kg/webtest/testing
A
C
Bonpoc: N°24 presence of a
physician has documented the
pregnancy test is positive. The
women who has just been told that a
the history of
A doctor is reviewing
Goodell's sign. This sign indicates

OTBeTbI(oqMH OTBeT)

Bluishcoloration of the vagina

pulse during palpation of the uterus


OA
A softening of the cervix

The presence of fetal movement

Thepresence of hCG in the urine


20 21 22 23 24 25 2 27 28 50

2 3 4 5 6 73

Scanned with CamScanner


TeCTMpOBaHMe
India Mobile |
TV XK
Be6
Samsung
A
avn.kgma.kg/webtest/testing
C

Bonpoc: N 25
labor?
exam in the evaluation of normal
What is the role of digital

OTBeTbi(oAWH OTBeT)

Time of labor

2 Fetal movements
Frequency and duration of contractions

4 Assessment of fetal well-being

5 Degree of dilatations of the cervix

21 22 23 24 25 26 21 28 29 50

34 5 67 8 9 10

Scanned with CamScanner


ll 89o8.b0 am
Gcongenital acyar h 6a3 papM3
Be6TecTMpOBaX
f C A avn.kgma.kg/webtest/testing

KyMap LUlawneu Kg Ru

Bonpoc: N91
Which of the following imaging technique in which iodine containing
cotrast agent is inserted
into the uterus to obtain an image?

OTBeTbli(OgMH OTBeT)

Ct-scan
2 Hysterosalpingography

3 Hysteroscopy
4 Ultrasound

5 Sonohysterography

2 3 4 5 6 7 50

2 3 4 5 6 7 8 9 10
11 12 13 14 15 16 17 18 19 20
21 22 23 24 25 26 27 28 29 30
31 32 33 34 35 36 37 38 39 40
41 42 43 44 45 46 47 48 49 50

OTBeveHHbi BOnpoc
nponyujeHHbI BOnpoc
Tekyujnn Bonpoc

Ho 3aBepuieHwA TeCTa oCTanocb

1:15:01
3aBepuwTb TeCT

Scanned with CamScanner


89% I 8:50 am

Gcongenital acyan h Ba3 bapM3 X


Be6 TecTWpOBaX

C A avn.kgma.kg/webtest/testing

KyMap lawneu Kg Ru

Bonpoc: N°2
What medical imaging is used to control diagnosed ovarian cancer?

OTBeTbI(opMH OTBeT)
1 Sonohysterography
2 Ct-scan
3 Ultrasound

MRI

5 Hysteroscopy

1 2 3 4 56 50

12 3 4 5 6 7 89 10
11 12 13 14 15 16 17 18 19 20
21 22 23 24 25 26 27 28 29 30
31 32 33 34 35 36 37 38 39 40
41 42 43 44 45 46 47 48 49 50
OTBeYeHHbIM BOnpoc
nponyueHHblM BOnpoc
TeKyuwn Bonpoc

Ao saBepueHwa TeCTa oCTanocb

1:14:55
3aBepwTb TeCT

Scanned with CamScanner


G congenital acyan i Ba3 cþapM3 Be6 TeCTMpoBaHX

C A avn.kgma.kg/webtest/testing
KyMap llannew Kg Ru

Bonpoc: N93
Ultrasound examination using saline solution to visualize uterus is:

OTBeTbli(oAMH OTBeT)
1 Uitrasonography
2 cystoscopy
3 Hysterosalpingography
4 Sonohysterography
5 HysteroscopY

1 2 3 4 5 6 7 50

1 2 345 6 7 89 10
11 12 13 14 15 16 17 18 19 20
21 22 23 24 25 26 27 28 29 30
31 32 33 34 35 36 37 38 39 40
41 42 43 44 45 46 47 48 49 50
OTBeYeHHbI BOnpoc
nponyuyeHHblM BOnpoc
Tekyun Bonpoc

Ao 3aBepweHwA TeCTa oCTanocb

1:14:52
3aBepunTb TeCT

Scanned with CamScanner


89% 8:51 am
Gcongenital acyar iBas dapM3 X Be6TeCTMpOBa X +
C A avn.kgma.kg/webtest/testing
O >

Kymap llanew Kg Ru

Bonpoc: N°4
A 43-year-old man came to the clinic with complaints of acute pain in the groin and frequent
painful urination. He has a fever and weakness for the second day. Ultrasound examination of the
prostaterevealed hypoecholic areas in the periphery. He was diagnosed with a suspected
prostate abscess. What imaging study is preferred to clarify this diagnosis ?

OTBeTbi(ogMH OTBET)
Nuclear imaging

2 Xay
3 Pelvic MRI

4 OFluoroscopy

5 CT-scan

12 3 5 6 7 8 50

123 4567 89 10
11 12 13 14 15 16 17 18 19 20
21 22 23 24 25 26 27 28 29 30
31 32 33 34 35 36 37 38 39 40
41 42 43 44 45 46 47 48 49 50
OTBeYeHHblM BOnpoC
nponyueHHLIM BOnpoc
TEKyuwn BOnpoc

Scannead with CamScanner


(020ecc0.2

89% 18:51 am
Gcongenital acyar iBa3 papM3 Be6TeCTWpoBa
C A avn.kgma.kg/webtest/testing

KyMap launeu Kg
Ru

Bonpoc: Ne5
Recognize the main clinical signs of premature puberty:

OTBETbI(oqMH OTBeT)

the appearance of secondary sexual characteristics older than 10 years.

2 the appearance of repetitive sexual characteristics at the age of less than 6 years.
congestive changes of the fundus

4 emotional stability
5 delaying intellectual development

22 33
456 7 8 9 50

1 234567 89 10
11 12 13 14 15 16 17 18 19 20
21 22 23 24 25 26 27 28 29 30
31 32 33 34 35 36 37 38 39 40
41 42 43 44 45 46 47 48 49 50
OTBe4eHHLIMBonpoc
nponyuyeHHLIA BOnpoc
Texyujnn Bonpoc

Ao 3aBepuiewwa TecTa ocTanocb

1:14:45
3aBepuiwTb TeCT

Scanned with CamScanner


89% 8:51 am
G congenital acyar i Eaa papM3 Be6TecTwpoea X

C A avn.kgma.kg/webtest/testing
KyMap lawneu Kg Ru
Bonpoc: N°6
Determine at what age girls have breast enlargement and nipple pigmentation.:

OTBeTbi(ogMH OTBET)
10-11

2 12-13

3 14-15
B-9

5 16-18

2 3 5 8 9 10 *** 50

12 12 34 56 7 B 9 10
11 13 14 15 16 17 18 19 20
21 22 23 24 25 26 27 28 29 30
31 32 33 34 35 36 37 38 39 40
41 42 43 44 45 46 47 48 49 50

OTBe4eHHbi BOnpoc
nponyuyenHblM BOnpoc
Tekyujnn Bonpoc

Ao 3aBepuieHnA TecTa ocTanocb

1:14:42
3aBepuuwTb TeCT

Scanned with CamScanner


89% 8:51 am
G congenital acyar Ba3 dapM3 X Be6TeCTupoBaX

C A avn.kgma.kg/webtest/testing

KyMap auneu Kg Ru

Bonpoc: N°7
Determine at what age the absence of secondary sexual characteristics is considereda sign of
delayed sexual development in boys

OTBETbl (OAMH OTBeT)

secondary sexual characteristics by 14 years, and sperm maturation by 17 years


by 16 years
2 secondary sexual characteristics by 13 years, and sperm maturation

3 secondary sexual characteristics by 12 years, and sperm maturation by 15 years

secondary sexual characteristics by 11 years, and sperm maturation by 14 years

by 13 years
5 secondary sexual characteristics by 10 years, and sperm maturation

9 10 11 50
4 5 6 **

10
2 3 45 67 89
12 13 14 15 16 17 18 19 20
11
21 22 23 24 25 26 27 28 29 30
31 32 33 34 35 36 37 38 39 40
41 42 43 44 45 46 47 48 49 50

OTBeveHHbiM BOnpoc
nponyujeHHbI BOnpoc
TeKyuynn Bonpoc

TECTa oCTanocb
Ao 3aBepweHnA

1:14:39
Scanned with CamScanner
89% 8:51 am
G congenital acyar Baa cpapM3 Be6TeCTwpoBaX

C A avn.kgma.kg/webtest/testing

KyMap Llanneu Kg Ru

Bonpoc: N 8
At the pediatrician's examination,the girl is 3 months old. Objectively: hypertrophy of the clitoris,
aplasia of the labia, and hymen. Guess what the girl's preliminary diagnosis is?

OTBETbl(OAMH OTBeT)

hermaphroditism

synechia of the labia

3 hyperpituitarism

4 Shereshevsky-Turner

5 hypopituitarism

5 6 7 9 10 11 12 50

123 4 5679 10
11 12 13 14 15 16 17 18 19 20
21 22 23 24 25 26 27 28 29 30
31 32 33 34 35 36 37 38 39 40
41 42 43 44 45 46 47 48 49 50

OTBeeHHbM BOnpoC
nponyujeHHbi BOnpoc
TeKyuwn BOnpoc

Scanned with CamScanner


89%89%8:51 am
Gcongenital acyar hBas dpapM3 Be6TecTwpoBa

C A avn.kgma.kg/webtest/testing

KyMap lanneu
Kg

Ru
Bonpoc: N99

At the reception, the girl is 8 years old. Objectively: the mammary glands are significantly
enlarged, hair in the armpits, pubic area, the appearance of the first menarche. Find out from the
description what kind of preliminary diagnosis can be made for a girl?

OTBETbl(OAMH OTBET)

1 healthy child
hypothyroidism

3 premature sexual development

4 delayed sexual development

5 sexual crisis

5 6 7 10 11 12 13 50

123 4 56 7 B9 10
11 12 13 14 15 16 17 18 19 20
21 22 23 24 25 26 27 28 29 30
31 32 33 34 35 36 37 38 39 40
41 42 43 44 45 46 47 48 49 50

OTBeYeHHblM BOnpoc
nponyujeHHbI BOrnpoc
TekyulM BOnpoc

o 3aBepuweHMA TeCTa oCTanocb

1:14:33 Scanned with CamScanner


A 89% 8:51 am
Gcongenital acyan ibaa papM3 Be6TecTpoBaX +

C A avn.kgma.kg/webtest/testing

KyMap llawneu Kg Ru

Bonpoc: N°10
On inspection at the girl of 15 years old. Objectively: rounded mammary glands, the nipples are
raised above the pre-pedigree angular. Pubic hair in the form of a triangile, long, curly, thick. In the
ampits of the head thick, curly on all surfaces. Manager of regular services. Evaluate the girls
sexual development according to Tanner.

OTBeTbi(oqMH OTBeT)
1 Ma3 P2 Ax2 Meo

Ma3 P3 Ax1 Me1

3 Ma2 P1 Axo Meo

4 Mao Po Axo Meo

5 Ma3 P3 Ax3 Me3

1
6 7 89 10 11 12 13 4

59

12 3 4 5678 9 10
11 12 13 14 15 16 17 18 19 20
21 22 23 24 25 26 27 28 29 30
31 32 33 34 35 36 37 38 39 40
41 42 43 44 45 46 47 48 49 50
OTBeeHHbI BOnpoc
nponyuyeHHblM Bonpoc
Tekyuni Bonpoc

Ao 3aBepuueHnA TeCTa oCTanocb

1:14:2O
Scanned with CamScanner
89%89%8:51 am
congenital acyan i Baa hapM3 Be6TecrupoBaX

C A avn.kgma.kg/webtest/testing 2
KyMap lanneu Kg Ru
Bonpoc: Ne11
Examination of the boy's external genitalia revealed that the external opening of the urethra is
located in the area of the penoscrotal angle. What is the type of anomaly in a child?

OTBETbl(oAMH OTBeT)

Phimosis

2 Hypospadias

3 Paraphimosis
4 Bladder exstrophy

5 Epispadias

1 8 10 1112 13 14 15

50

12 3 4567 8 9 10
11 12 13 14 15 16 17 18 19 20
21 22 23 24 25 26 27 28 29 30
31 32 33 34 35 36 37 38 39 40
41 42 43 44 45 46 47 48 49 50
OTBe4eHHbiM Bonpo0c
nponyueHHbIM BOnpoc
TeKyunn Bonpoc

go 3aBepweHnA TeCTa oCTa/nocb:

1:14:26
Scanned with CamScanner
89% 8:51 am
congenital acya Ea3 papM3 Be6TeCTMpoBaX

C A avn.kgma.kg/webtest/testing
KyMap Wanneuw Kg Ru
Bonpoc: N°12
Necrospermia is:

OTBETbI(oqMH OTBeT)
1 Presence of only dead sperms in the ejaculate

Presence of only sedentary sperm in the ejaculate

Presence of only spermatogenesis cells in the ejaculate

4 Absence of sperm in the ejaculate

5 Absence of elements ofspermatogenesis in the ejaculate

9 10 11
12 13 14 15 16

50

1 23 4 56 7 8
9
10
11 12 13 14 15 16 17 18 19 20
21 22 23 24 25 26 27 28 29 30
31 32 33 34 35 36 37 38 39 40
41 42 43 44 45 46 47 48 49 50

OTBeYeHHbI BOnpoc
nponyueHHbIMBOnpoc
Tekyuunn Bonpoc

o 3aBepuweHwA TeCTa oCTanocb

1:14:23
3aBepunTb TeCT

Scanned with CamScanner


22eeeoecco

8:51 am
89%
Gcongenital acyan h baa dapM3 Be6TeCTMpOBaX
avn.kgma.kg/webtest/testing
C A
Kg Ru
KyMap Llawneu

Bonpoc: N 13
at
A history of trauma to both testicles
4 years. Have no children.
Male, 30 years old, married for performed (which one the patient
for which an organ-preserving operation was
the age of 20,
On examination: both testicles
are reduced in size, of
exactly) on both testicles.
does not know unremarkable. The prostate gland is
unchanged.
deferens are
dense consistency, the vas ejaculate. Total
No spermatozoa or spermatogenesis cells were found in the
Spermogram. your diagnosis?
(the norm is 12-35 nmol/ What is
1).

testosterone in the blood is 4 nmol/1

OTBeTbi(OAMH OTBeT)

1 Testicular hypogonadism, azoospermia.

2 Cryptorchidism

3 Testicular cancer

4 Late onset hypogonadism

5 Congenital atrophy of testicles

14 15 16 17
10 11 12 13

50

1 2 3 4 5 6 7
89 10
11 12 13 14 15 16 17 18 19 20

21 22 23 24 25 26 27 28 29 30
31 32 33 34 35 36 37 38 39 40
41 42 43 44 45 46 47 48 49 50

OTBeYeHHbl BOnpoc
ponyujeHHblM Bonpoc
Tekyuwn Bonpoc

oCTanocb
Ao3aBepweHns TeCTa

Scanned with CamScanner


ese

89% 89%8:51 am

Gcongenital acyar h Ba3 papM3 Be6TecTnpoBaX

O C A avn.kgma.kg/webtest/testing

KyMap LLanneu Kg
Ru

Bonpoc: N°14
Which of the following does not apply to instrumental research methods?

OTBeTbI(opMH OTBeT)

Amniocentesis

2 USE with dopplerometry


3 Cordocentesis
Gravidogramm

5 Cardiotocography

13 14 15 16 17 18
10 11 12

50

2 3 4 5 678 10 9
11 12 13 14 15 16 17 18 19 20
21 22 23 24 25 26 27 28 29 30
31 32 33 34 35 36 37 38 39 40
41 42 43 44 45 46 47 48 49 50

OTBeYeHHbiMBOnpoc
nponyujeHHbiM Bonpoc
TeKyuiwn Bonpoc

Ao 3aBepweHnA TeCTa ocCTanocb:

1:14:15
3aBepunTb TeCT

Scanned with CamScanner


89% 1 8:51 am
congenital acyan iBa3 dapM3 Be6TeCTpoBa X +

C A avn.kgma.kg/webtest/testing

KyMap lannew Kg Ru

Bonpoc: Ne15
Active management of the third stage of labor includes intramuscular injection (1M) of

OTBETbl(OAMH OTBeT)

1 10 W of oxytocin

2 5IU of oxytocin

3 1 IU ofoxytocin
4 20 1U of oxytocin

5 30 IU of oxytocin

11 12 13 14 15 16 17 18 19

50

1 2 3 456 789 10
11 12 13 14 15 16 1718 19 20
21 22 23 24 25 26 27 28 29 30
31 32 33 34 35 36 37 38 39 40
41 42 43 44 45 46 47 48 49 50
OTBeueHHbl Bonpoc
nponyujeHHbi Bonpoc
TeKyunn Bonpoc

Ao 3aBepweHna TecTa oCTanoCb

1:14:11
3aBepuWTb TeCT
Scanned with CamScanner
8:51 am
89%
TecTupoBa
+
Be6
i ba3 hapM3
congenital acyar avn.kgma.kg/webtest/testing
G
Ru
CA A
KyMap
Ulanneus Kg

Bonpoc: N°16
pregnancy
during
1000 times
which hormone increases
The synthesis of

OTBeTbi(OqMH OTBeT)

Progesterone

2 Estriol
3 Cortisol
4 Thyroxine
5 Estradiol
17 18 19 20
14 15 16
12 13

50

123 567 89 10
A

1112 13 14 15 16 17 18 19 20
21 22 23 24 25 26 27 28 29 30
31 32 33 34 35 36 37 38 39 40
41 42 43 44 45 46 47 48 49 50
OTBe4eHHbinBonpoc
rnponyueHHbl Bonpoc
Texyuwn Bonpoc

4o 3aBepweHMA TeCTa oCTanocb

1:14:08B
3aBepuwTb TeCT

Scanned with CamScanner


m8:51
89% am

G congenital acya h Ea3 bapM3 Be6TeCTupoBa X

C avn.kgma.kg/webtest/testing

KyMap Llanneu Kg Ru
Bonpoc: N°17
The labor has the following stages:

OTBeTbi(oguH OTBeT)
Preparation, dilation, expulsion

2 Preparation, dilation, expulsion, early postpartum

3 Dilation, expulsion, delivery of placenta, early postpartum

Effacement of the cervix, the expulsion, delivery of placenta

5 Preliminary, dilatation, expulsion, postpartum

13 14 15 16 17 18 19 20 21

50

1 5678
2 3 4
9 10
11 12 13 14 15 16 17 18 19 20
21 22 23 24 25 26 27 28 29 30
31 32 33 34 35 36 37 38 39 40
41 42 43 44 45 46 47 48 49 50

OTBeveHHLIM BOnIpoc
nponyuueHHbIM Bonpoc
TeKyuyn Bonpoc

o 3aBepweHwa TeCTa oCTanoCb

1:14:04
3aBepunTb TeCT

Scanned with Camscanner


89% 8:51 am
G congenital acyar hBa3papM3 Be6 TecTupoBaX +

C A avn.kgma.kg/webtest/testing

KyMap Llanneuu Kg Ru
Bonpoc: N°18
A sign of placental separation from the uterus is:

OTBETbI(oquH OTBeT)
Contraction of the uterus on palpation

Lack of bloody discharge

3 The umbilical cord lengthens

Softening of the uterus on palpation

5 The umbilical cord shortened

14 15 16 17 18 19 20 21 22

50

1
23 4567 10 89
11 12 13 14 15 16 17 18 19 20
21 22 23 24 25 26 27 28 29 30
31 32 33 34 35 36 37 38 39 40
41 42 43 44 45 46 47 48 49 50

oTBe4eHHbi BOnpoc
nponyujeHHEIM BONIpoc
Texyunn BOnpoc

Ao 3aBepuweHwa TecTa oCTanocb

1:14:01
3aBepuWTb TeCT

Scanned with Camscanner


89% 89%8:51 am
congenital acyar iEBaa chapM3 Be6TecTwpoBarX

O C A avn.kgma.kg/webtest/testing

Ru
Kg

KyMap lWawneu

Bonpoc: N919
Adaptation of pregnancy is an increased blood supply to the pelvic region that results in a
purplish discoloration of the vaginal mucosa, which is known as:

OTBETbi(oqMH OTBeT)
Ladin's sign

2 Chadwick's sign

Palmer's sign

4 Hegar's sign
5 Goodell's sign

15 16
16 17 18 1920 21 22 23

50

123 4567 89 10
11 12 13 14 15 16 17 18 19 20
21 22 23 24 25 26 27 28 29 30
31 32 33 34 35 36 37 38 39 40
41 42 43 44 45 46 47 48 49 50

OTBeueHHbiM BOnp0c
nponyueHHbIM BOnpoc
Tekyunn BOnpoc

Ao 3aBepweHna TeCTa oCTanocb

1:13:57
3aBepuWTb TecT
Scanned with Camscanner
ll 89%8:51 am
G congenital acyar :Eaa dapM3 Be6TecTupoBa X

C A avn.kgma.kg/webtest/testing

KyMap lanneu Kg Ru
Bonpoc: N°21
Biparietal diameter of the fetal head:

OTBETbI(ogMH OTBeT)

1 12.5cm

2 11.5cm

9.5
4 10.5
5
8.5cm
17 18 19 20 21 22 23 24 25

50

1
2 3 4567 89 10
11 12 13 14 15 16 17 18 19 20
22 23 24 25
26 27 28 29 30
31 32 33 34 35 36 37 38 39 40
41 42 43 44 45 46 47 48 49 50
OTBe4eHHblM BOnpoc
nponyuyeHHLIM BOnpoc
Teryuywn BOnpoc

g0 3aBepuwenns TeCTa oCTanocb

1:13:52
3aBepunTb TeCT

Scanned with CamScanner


89% M8:52 am

G congenital acyan Ba3 dapM3 Be6TeCTwpoBa X

C A avn.kgma.kg/webtest/testing

KyMap awnew Kg Ru
Bonpoc: Ne20
The third stage of labor continues from:

OTBETbl(ogMH OTBeT)

Cervical dilatation to birth of membranes

Birth of fetus to birth of placenta

3 Dilatation of the cervix to the birth of the fetus

4 Separation of the placenta to the birth of the placenta

5 Birth of fetus to birth of membranes

16 17 18 19 20 21 22 23 24 ***

50

12 3 4 567 6 10 9
11 12 13 14 15 16 17 18 19 20
21 22 23 24 25 26 27 28 29 30
31 32 33 34 35 36 37 38 39 40
41 42 43 44 45 46 47 48 49 50
OTBeYEHHbl BOnpoc
nponyuieHHLA BOrIpoc
Tekyunn Bonpoc

Mo 3aBepuueHWA TeCTa oCTanocb

1:13:48
3aBepuiWTb TeCT

Scanned with CamScanner


89% 8:52 am
Gcongenital acyar Eaa hapM3 Be6TecTpoBaX

C avn.kgma.kg/webtest/testing

KyMap lawneuu Kg Ru

Bonpoc: N 22
Which of the following conditions is common in pregnant women in the 2nd trimester of
pregnancy?

OTBeTbi(oqMH OTBeT)

1 Disuria
Metabolic alkalosis

3 Physiologicanemia

4 Respiratory acidosis

Mastitis

18 19 20 21 22 23
24 25 26

50

1 2 34 567 89 10
11 12 13 14 15 16 17 18 19 20
21 22 23 24 25 26 27 28 29 30
31 32 33 34 35 36 37 38 39 40
41 42 43 44 45 46 47 48 49 50
OTBeHeHHbIM BOpoc
nponyuyeHbiM BOnpoc
TeKylnn BOnpoc

Ao 3aBepuweHna TeCTa oCTanocb

1:13:45
3aBepuwTb TeCT

Scanned With CamScanner


l 89% 8:52 am
G congenital acyar Ba3 dapM3 Be6TeCTMpoBa x +

C A avn.kgma.kg/webtest/testing

Kymap LWanneu Ru
Kg

Bonpoc: N°23
How is the first labor characterized ? Labor begins with:

OTBeTbl(oAMH OTBeT)

1 Regular uterine contractions and ends with decent of the head

2 Regular uterine contractions and ends with engagement of the head

3 Regular uterine contractions and ends with full dilatation of cervix

Rupture of membrane and ends with full dilatation of cervix

5 Regular uterine contractions and ends with effacement of cervix

19 20 21 22 23 24 25 26 27

50

123 45 67 89 10
11 12 13 14 15 16 17 18 19 20
21 22 23 24 25 26 27 28 29 30
31 32 33 34 35 36 37 38 39 40
41 42 43 44 45 46 47 48 49 50
OTBeyeHHbI Bonpoc
nponyuyeHHbI BOnpoc
TeKyuinn Boripoc

Ao 3aBepweHna TeCTa oCTa/iocb

1:13:42
3aBepuwTb TeCT

Scanned with CamScanner


89% 8:52 am
congenital acyar h ba3 dapM3 Be6 TecTupoBa X

avn.kgma.kg/webtest/testing

KyMap anne Kg Ru

Bonpoc: N 24
After the delivery of the newborn the doctor manage 3 stage of labor. Which observation would
indicate that the placenta has separated from uterine wall and is ready for delivery?

OTBETbl(OqMH OTBeT)

1 Changes in the shape of the uterus

2 Soft and boggy uterus

3 Maternal complaints of severe uterine cramping

4 The umbilical cord shortens in length and changes in color

Hemorrhage

1 20 21 22 23 24 25 26 27 28

50

12 3 456 78910
11 12 13 14 15 16 17 18 19 20
21 22 23 2425 26 27 28 29 30
31 32 33 34 35 36 37 38 39 40
41 42 4344 45 46 47 48 49 50

TBeyeHHbiM BOnpoc
nponyueHHbiM Bonpoc
rexyunn Bonpoc

Ao 3aBepuweHna TeCTa oCTanocb

1:13:38
3aBepunTb TeCT

Scanned with Camscanner


O C A avn.kgma.kg/webtest/testing

KyMaplawnew Kg Ru

Bonpoc: N°25
Which of the following is true about the uterine contractility?

OTBETbI(ogMH OTBeT)

Aspirin or indomethacin may delay the onset of labor

2 The number of gap junctions decreases prior to labor

3 Progesterone levels increases prior to labor

Estrogen has an inhibitory effect

5 The uterus contracts segmentally via nervous connections

21 22 23 2425 26 27 2829 **

50

12
11
3 4 56789 10
12 13 14 15 16 17 18 19 20
21 22 23 24 25 26 27 28 29 30
31 32 33 34 35 36 37 38 39 40
41 42 43 44 45 46 47 48 49 50
OTBeueHHbI BOpoc
nponyuyeHHIM BOnpoc
TeKyunn BOnpoc

Ao 3aBepuweHna TeCTa ocTanocb

1:13:35
3aBepuwwTb TeCT

Scanned with CamScanner


89% 8:52 am
G congenital acyar i 6a3 cpapM3 Be6TecTMpoBaX

C A avn.kgma.kg/webtest/testing

KyMap lWanneu Kg Ru

Bonpoc: N°26
The true conjugate is the distance between:

OTBETbi(OAMH OTBeT)

Uppersurface of pubic arch to sacral promontory

2 Upper edge of the symphysis pubis and to promontory

Inner surface of pubis and sacral promontory

ishial tuberosities

5 Undersurface of pubic arch to sacral promontory

22 23 24 25 26 27 28 29 30

50

1 234 56 7 9 10
11 12 13 14 15 16 1718 19 20
21 22 24 25 26 27 28 29 30
23
31 32 33 34 35 36 37 38 39 40
41 42 43 44 45 46 47 48 49 50
OTBeueHHblM Bonipoc
nponyueHHbi Bonpoc
TeKyuni Bonpoc

Ao 3aBepweHWA TeCTa ocTanocb

1:13:32
3aBepuwTb TeCT

Scanned with CamScanner


Bonpoc: N°27
A doctor determines that the woman is in the beginning 2nd stage of labor when which of the
following assess ments is noted?

OTBETbl(OAMH OTBET)

1 The cervix is dilated completely

2 The membrane have ruptured

3 Begins expel clear vaginal fluid

4 Thecontractions are regular


5 The face of women becomes red

23 24 25 26 27 28 29 30 31

50

1212 1334 5678 9 10


11 14 15 16 17 18 19 20
21 22 23 24 25 26 27 28 29 30
31 32 33 34 35 36 37 38 39 40
41 42 43 44 45 46 47 48 49 50

OTBeyeHHbiM BOnpoc
nponyueHHbIM BOnpoc
TeKyuinn Bonpoc

o 3aBepweHna TeCTa oCTanocb

1:13:30
3aBepuTb TeCT

Scanned with CamScanner


O C Aavn.kgma.kg/webtest/testing

KyMap Llawneu Kg Ru

Bonpoc: N°28
weeks' pregnant is diagnosed with hyperemesis gravidarum. This
A 21-year old client, 6
excessive vomiting during pregnancy will often result in which of the following conditions?

OTBeTbi(oAMH OTBET)
1
Pregnancy induced hypertension
2 Bowel perforation

3 Toxemia

4 Electrolyte imbalance

5 Miscarriage

24 25 26 27 28 29 30 31 32

50

1 2 3 456789 10
11 12 13 14 15 16 17 18 19 20
21 22 23 24 25 26 27 28 29 30
31 32 33 34 35 36 37 38 39 40
41 42 43 44 45 46 47 48 49 50

OTBeveHHbl BOnpoC
nponyujeHHbl Bonpoc
Tekyunn BOnpoc

Ao saBepueHnA TeCTa oCTanocb:

1:13:27
3aBepuiwTb TeCT

Scanned with CamScanner


89%8:52 am
G congenital acyar 6a3 dapM3 Be6TeCTupoBa X +

O C A avn.kgma.kg/webtest/testing

Kymap Lawnew Kg Ru

Bonpoc: N°29
Amenorrhea is the absence of menstruation during:

OTBETbI(oqMH OTBeT)

5months
2 months

O4 months

12months
5 6 months

25 26 27 28 29
3031 32 33

50

1
2 3 4 567 8910
11 12 13 14 15 16 17 18 19 20
21 22 23 24 25 26 27 28 29 30
31 32 33 34 35 36 37 38 39 40
41 42 43 44 45 46 47 48 49 50
OTBeHeHHbl Bonp0c
nponyujeHHbiM BOipoc
TeKyuinn BOnpoc

Ao 3aBepuweHwa TeCTa oCTanocb:

1:13:23
3aBepuiwTb TeCT

Scanned with CamScanner


l89%8:52 am
Gcongenital acyar i Baa dapM3 Be6TeCTwpoBa X +

C A avn.kgma.kg/webtest/testing
KyMap Llawneu
Kg

Ru
Bonpoc: Ne30
Preovulatory changes in the hormonal background are characterized by an increase in the level

OTBETbI(ogMH OTBeT)

1 FSH and decreased LH

Prolactin

LH and decrea sed FSH

4 FSH and LH

5 FSH, LH and prolactin

26 27
27 28 29 30 31 3233 34

50

1 2 3 456 7 8 9 10
11 12 13 14 15 16 17 18 19 20
21 22 23 24 25 26 27 28 29 30
31 32 33 34 35 36 37 38 39 40
41 42 4344 45 46 47 48 49 50
OTBeHeHHbIM BOnp0c
nponyuueHHbiM BOnpoc
Tekyujnn Bonpoc

lo 3aBepuweHna TeCTa oCTanocb

1:13:20
3aBepunTb TeCT

Scanned with CamScanner


Bonpoc: N31
Phases of the ovarian cycle:

OTBETbl(ogMH OTBeT)

1 Desquamation and ovulation

2 Follicularand luteal

3 Anovulatory and luteal

4 Ovulatory and anovulatory

5 Folicular and ovulation

29 30 31 32 33 34 35
27 28

50

12 34 5678 9 10
11 12 13 14 15 16 17 18 19 20
21 22 23 24 25 26 27 28 29 30
31 32 33 34 35 36 37 38 39 40
41 42 43 44 45 46 47 48 49 50
OTBeYeHHbIM BOnpoc
npornyuyeHHbIM BOIpoc
TeKyunn BOnpoc

Ao 3aBepweHwa TeCTa oCTanOCb

1:13:17
3aBepwWTb TeCT

Scanned with CamScanner


SA
l 89% 8:52 am
congenital acyar i 6a3 dapM3 Be6TeCTHpOBa X

CA avn.kgma.kg/webtest/testing

KyMap Llauneu Kg Ru

Bonpoc: N°32
in giris at an early age (from 2 to 8 years), the following are more common:

OTBETbl(ogMH OTBeT)

Ovarian tumors

2 Dysfunctional bleeding

Salpingo-oophoritis

Vulvovaginitis

Congenital anomalies of the genital organs

28 29 30 31 32 33 34 35 36

50

1 23 4 567 89 10
11 12 13 14 15 16 17 18 19 20
21 22 23 24 25 26 27 28 29 30
31 32 33 34 35 36 37 38 39 40
41 42 43 44 45 46 47 48 49 50

OTBeHEHHLIM BOnpoc
nponyuenHbi Bonpoc
Tekyunn Bonpoc

o 3aBepuieHnA TeCTa ocTanocb

1:13:14
3aBepuTb TeCT

Scanned with Camscanner


Bonpoc: N°33
The complication that most often arises in the management of the intrauterine device is

OTBETbi(OAMH OTBeT)

OPelvic vein thrombosis


2 Habitual miscarriage

3 Acuteinfection
4 isthmic-cervical insufficiency

5 Ectopic pregnancy

29 30 31 32 333334 35 36 37

50

B
12 3 4 567 9 10
11 12 13 14 15 16 17 18 19 20
21 22 23 24 25 26 27 28 29 30
31 32 33 34 35 36 37 38 39 40
41 42 43 44 45 46 47 48 49 50

OTBeHeHHbi BOnpoc
nponyuyeHHbi BOnIpoc
Texyu BOnpoc

o 3aBepuweHnA TecTa oCTanocb

1:13:10
3aBepwnTb TeCT

Scanned with CamScanner


89% 8:52 am
congenital acyar ba3 dapM3 Be6TeCTwpoBaX
C Aavn.kgma.kg/webtest/testing

KyMap lawnew Kg Ru

Bonpoc: N°34
For anovulatory menstrual cycle with follicular atresia is characteristic:

OTBETbi(oqMH OTBeT)

1 Aigomenorrhea

2 Oligoamenorthea
3 Hypomenorrhea

Amenorrhea

5 Polymenorrhea

30 31 32 33 34 35 36 37 38

50

123 4567 B9 10
11 12 13 14 15 16 17 18 19 20
21 22 23 24 25 26 27 28 29 30
31 32 33 34 35 36 37 38 39 A0
41 42 43 44 45 46 47 48 49 50

OTBeHEHHbl BOnpooc
riponyuieHHbIM BOnpoc
TeKyuynA BOnpoc

o 3aBepuieHnA TeCTa oCTanoCb

1:13:07
3aBepwwTb TeCT

Scanned with CamScanner


l 89% 8:52 am
Gcongenital acyar i Ba3 dhapM3 Be6TeCTMpOBaX

C A avn.kgma.kg/webtest/testing

KyMap LWanneu Kg Ru

Bonpoc: N935
The anatomical features of the uterus in a newborn girl include:

OTBETbl(ogMH OTBeT)

Uterus curved posteriorly

2 The body of the uterus is smal, the cervix is almost not expressed

3 The uterus has a bicorn shape

The length of the cervix 3 times more the length of the uterus

The body of the uterus is almost 2 times larger than the cervix

31 32 33 34 35 36 37 38 39

50

1 23 4 5 6 7 8 9 10
11 12 13 14 15
16 17 18 19 20
21 22 23 24 25 26 27 28 29 30
31 32 33 34 35 36 37 38 39 40
41 42 43 44 45 46 47 48 49 50

OTBeHeHHblM Bonpoc
nponyujeHHbi BOIpoc
TeKyuinn Bonpoc

Mo 3aBepuieHna TeCTa oCTanocb

1:13:04
3aBepwWTb TeCT

Scanned wíth CamScanneer


l89% M 8:52 am

Gcongenital acyar hBaa dapM3 Be6TecTMpOBaX


avn.kgma.kg/webtest/testing

KyMap lanneu Kg Ru

Bonpoc: N936
Women with AUB are at risk:

OTBETbi(oaMH OTBeT)
On the development of breast tumors

2 On the development of placental insufficiency and anomalies of the birth force

3 Pregnancy

4 Miscarriage

5 0n the development of genital tumors

32 33 34 35 36 37 38 39 40 ***

50

1 2 3 4 5 6 78910
11 12 13 14 15 16 17 18 19 20
21 22 23 24 25 26 27 28 29 30

31 32 33 34 35 36 37 38 39 40
41 42 43 44 45 46 47 48 49 50

OTBe4eHHBIM BOnpoc
nponyujeHHbl Bonpoc
Tekyuwn Bonpoc

Ao 3aBepuweHna TecTa oCTanocb

1:13:01
3aBepuwTb TeCT

Scanned with CamScanner


l88%8:52 am
congenital acyar i
Ba3 cpapM3 Be6 TecTMpoBa X

C A avn.kgma.kg/webtest/testing

KyMap Llawneu Kg Ru

Bonpoc: N937
What is the most common presenting complaint of a woman with vaginitis?

OTBETbi(oAMH OTBeT)

1 Pelvic pain

2 Fever

3 Vaginal discharges

4 Dyspareuniaa

5 irregular menses

33 34 35 36 37 38 39 40 41

50

1
23 4 56 78 10 9
11 12 13 14 15 16 17 18 19 20
21 22 23 24 25 26 27 28 29 30
31 32 33 34 35 36 37 38 39 40

41 42 43 44 45 46 47 48 49 50

OTBe4EHHbiM BOnpoc
nporiyuieHHblM Bonpoc
TeKyuwn Bonpoc

Ao 3aBepweHnA TecTa oCTanocb

1:12:58
3aBepuWTb TeCT

Scanned with CamScanner


Bonpoc: N°38
Which of the following mechanisms best explains the contraceptive effect of birth control pills
that contain both synthetic estrogen and progestin?

OTBETbI(oqMH OTBeeT)
1 Production of uterine secretions that are toxic to the developing embryo

2 Direct inhibition of oocyte maturation

Impairment of implantation

4 Inhibition of ovulation

Impairment of sperm transport due to uterotubal obstructio

34 35 36 37 38 39 40 41 42

50

12 3 4 56789 10
11 12 13 14 15 16 17 18 19 20
21 22 23 24 25 26 27 28 29 30
31 32 33 34 35 36 37 38 39 40
41 42 43 44 45 46 47 48 49 50
OTBe4eHHblM BOnpoc
nponyuyeHHbM Bonpoc
TeKyuinn Bonpoc

Ao 3aBepuweHwa TeCra oCTanocb

1:12:55
3aBepuMTb TeCT

Scanned with CamScanner


Bonpoc: N°39
receive recommendations on contraception,
gynecologist to
consulted a births, 2 medical
Patient 1, 42 years oid, Regular sex life in marriage. History of 2
for 5 years. mm Hg., height 164 cm,
suffers from hypertension pressure is 135/90
condition is satisfactory, blood
abortions. General contraception
BMI =35. Recommended method of
weight 90 kg,

OTBeTbi(OAMH OTBeT)

Calendar rhythm method

2 Intrauterine device with silver

3 Surgical sterilization

4 Condom

5 Postcoital contraception (Postinor, Dvella)

39 40 41 42 43
35 36 37 38

50

7 89 10
12 3 4 56
11 12 13 14 15 16 17 18 19 20
21 22 23 24 25 26 27 28 29 30
31 32 33 34 35 36 37 38 39 40
41 42 43 44 45 46 47 48 49 50
OTBeYeHHblM BOnpoc
npornyueHHblM BOnpoc
TeKyunn BOnpoc

Ao 3aBepuweHwa TeCTa oCTanoCb

1:12:51
Scanned with CamScanner
ss

88% 8:53 am
congenital acyar Bas dapM3 Be6TeCTpOBa X +

A avn.kgma.kg/webtest/testing

KyMap Llanneu Kg Ru

Bonpoc: N940
A 16-year-oldgirl developed spotting from the genital tract, lasting 8 days after a 2-month delay.
The first menstruation appeared 4 months ago for 2 days, after 28 days, moderate, painless.
Denies sexuality. The development is correct, well physically built. When recto-abdominal
examination of the pathology is not detected. Hb-80 g/1. Probable diagnosis:

OTBeTbi(ogMH OTBeT)

Juvenile uterine bleeding

2 Polyp of the cervix

Hormone-producing ovarian tumor

Cervical cancer

Endometrial polyposis

36 37 38 39 40 41 42 43 44 ***

50

12 3 4 5678 910
11 12 13 14 15 16 17 18 19 20
21 22 23 24 25 26 27 28 29 30
31 32 33 34 35 36 37 38 39 40
41 42 4344 45 46 47 48 49 50
OTBeeHHBIM BOnpoc
nponyuyeHHbIM BOnpoc
TeKyuMn BOnpoc

Ao 3aBepuweHna TeCTa oCTanocb

1:12:48 Scanned wíth Camscanner


88% 8:53 am
congenital acyar 6a3 hapM3 Be6TeCTMpOBaX

C A avn.kgma.kg/webtest/testing

Kymap lawneu Kg Ru

Bonpoc: Ne41
Infiammation of the female genital organs.

OTBETbi(oqMH OTBeT)

Tend to increase incidence

No differences in acute and chronic forms

Often develops with one pathogen

in 60% of cases proceeds as an acute process

Have a relationship with contraception

37 38 39 40
41 42 43 44 45

50

1
23 4567 89 10
11 12 13 14 15 16 17 18 19 20
21 22 23 24 25 26 27 28 29 30
31 32 33 34 35 36 37 38 39 40

41 42 43 44 45 46 47 48 49 50

oTBeHeHHIM BOnpoc
nponyuewwui BOIpoc
TeKyunn BOnpoc

Ao 3aBepwenwa TecTa oCTanocb

1:12:46
3aBepuwTb TeCT

Scanned with CamScanner


88% 8:53 am
congenital acyar i Baa dapM3 Be6TecTWpoBa X

C Aavn.kgma.kg/webtest/testing

KyMap lawneu Kg Ru
Bonpoc: N942
The optimum tempereture for sperm activity:

OTBeTbl(OAUH OTBeT)

40 C

From 38 to 39

30 C below body t0

37 C abovebodyt0

42C

38 39 40
4142 43 44 45 46

50

1 2 3 456 789 10
11 12 13 14 15 16 17 18 19 20
21 22 23 24 25 26 27 28 29 30
31 32 33 34 35 36 37 38 39 40

41 42 43 44 45 46 47 48 49 50

OTBeHeHHLIM BOnpoc

nponyueHHbiM BOnpoc
TeKyuinn Bonpoc

Ho 3aBepujenmA TeCTa ocTanocb

1:12:43
3aBepuWTb TeCT

Scanned with CamScanner


88% 8:53 am
ngenital acya Esas hapM3 Be6TecTpoBaX

C A avn.kgma.kg/webtest/testing

KyMap Lllawneu Kg Ru

Bonpoc: N943
The causes of secondary amenorrhea of central genesis include everything besides:

OTBETbi(opuH OTBeT)
Psychogenicamenorrhea

Sheehan syndrome

Kalimann Syndrome

Syndrcome of hyperprolactinemia

Amenorhea due to weight loss

39 A0 41 42 43 44 45 46 47

50

1234
11
S67B910
12 13 14 15 16 17 18 19 20
21 22 23 24 25 26 27 28 29 30
31 32 33 34 35 36 37
38 39 40
41 42 43 44 45 46 47 48 49 50
OTBeHeHHulh Bonpoc
nponyueHH BOnpoc
Texyuawn BOnpoc

ho 3asepujewna TecTa oCTanoCb

1:12:39
3aBepunTb TeCT

Scanned with CamScanner


88% 8:53 am
congenital acyar 5a3 papM3 x Be6TecTMpOBai X +

C A avn.kgma.kg/webtest/testing

Kymap Llaune Kg Ru

Bonpoc: Ne44
The ultrasound examination revealed the persistence of the follicles in the patient. What phase of
the menstrual cycle does not occur with follicle persistence:

OTBeTbi(oqMH OTBeT)
1 Desquamation

2 Regeneration

3 Proliferation
4 Secretion
5 Luteal

40 41 42 43 44 4546 47 48

50

123 4 5 6 7 89 10
11 12 13 14
15 16 17 18 19 20
21 22 23 24 25 26 27 28 29 30

31 32 33 34 35 36 37 38 39 40
41 42 43 44 45 46 47 48 49 50
OTBeHeHHbiM BOnpoc
nponyueHHbi Bonpoc
Tekyulni Bonpoc

Ao 3aBepuieHWA TecTa oCTanocb

1:12:36
3aBepuWTb TeCT

Scanned with CamScanner


C A avn.kgma.kg/webtest/testing

KyMap LLlanne Kg Ru

Bonpoc: N945
The second cross of ureter of the uterine artery at the level of

OTBeTbi(ogMH OTBeT)
laterally 1.5-2 cm to upper part of vagina

2 laterally 1.5-2cm to ovaries

3 laterally 1.5-2 cm to cervix

all are incorrect

laterally 1.5-2 cm to corpus uteri

42 43 4445 46 47 48 49 50

1 2 3 4 5 6 7 89 10
11 12 13 14 15 16 17 18 19 20
21 22 23 24 25 26 27 28 29 30
31 32 33 34 35 36 37 38 39 40
41 42 43 44 45 46 47 48 49 50
OTBeHEHHLi BOnpo0
nponyueHHLI BOnpoc
Texyuinn Bonipoc

Ao 3aBepueHna TeCTa ocTanocb

1:12:33
3aBepwWTb TeCT

Scanned with Camscanner


88% 88% 8:53 am
congenital acyar iBa3 dapM3 Be6TecTpoBaX
C A avn.kgma.kg/webtest/testing

Kymap Llawneu Kg Ru

Bonpoc: N946
The uterine artery is a branch of

OTBETbi(OAMH OTBET)

the external iliac artery

the common iliac artery

the internal iliac artery

4 the inferior epigastric artery


5 the obturator artery

43 44 45 46 47 48 49 50

1 2 3 45 6 7 89 10D

11 12 13 14 15 16 17 18 19 20
21 22 23 24 25 26 27 28 29 30
31 32 33 34 35 36 37 38 39 40
41 42 43 44 45 46 47 48 49 50
OTBe4eHHbiM BOnpoc
nponyuyeHHLIM BOnpoc
Tekyuinn Bonpoc

o 3aBepuienwa TecTa oCTanocb

1:12:30
3aBepuwTb TeCT

Scanned with CamScanner


88% am
I 8:53
Gcongenital acyar hBaa papM3 Be6TecTMpOBaX
C A avn.kgma.kg/webtest/testing

KyMap lannew Kg
Ru
Bonpoc: N947
Pelvic diaphragm refers to:

OTBETbi(oquH OTBeT)
Perin eal membrane with levator ani m.

2 Perineal membrane with coccygeus m.

3 Coccygeus m. with urogenital diaphragm

4 Urogenital diaphragm with Levator ani m.

5 Coccygeus m. with levator ani m.

43 44 45 46 47 48 49 50

1 2 3 45 67 89 10
11 12 13 14 15 16 17 18 19 20
21 22 23 24 25 26 27 28 29 30
31 32 33 34 35 36 37 38 39 40
41 42 43 44 45 46 47 48 49 50

OTBeHeHHbi BOnpoc
nponyuieHHLI BOnpoc
Texyuinn Bonpoc

o 3aBepujeHna TeCTa oCTanocb

1:12:27
3aBepuWTb TeCT

Scanned with CamScanner


88% 18:53 am
G congenital acya h Ba3 hapM3 Be6TeCTWpoBaH X

A avn.kgma.kg/webtest/testing

KyMap lanneu Kg
Ru
Bonpoc: Ne48
Anatomic structures passing through the suprapiriform foramen:

OTBETbi(ogMH OTBeT)
1
theilioinguinal nerve
2 the pudendal neurovascular bundle

the sciatic nerve

4 the superior gluteal neurovascular bundle

5 the inferior epigastric artery

43 44 45 46 47 48 49 50

12 3 4 5 6 7 89
10
11 12 13 14 15 16 17 18 19 20
21 22 23 24 25 26 27 28 29 30
31 32 33 34 35 36 37 38 39 40
41 42 43 44 45 46 47 48 49 500

OTBeHeHHbIM BOnpoc
nponyujeHHBI BOnpoc
TeKyujnn Bonpoc

o 3aBepuweHna TeCTa oCTanocb

1:12:24
3aBepwWTb TeCT

Scanned with Camscanner


SA
88% 8:53 am
congenital acyar h baa dapM3 X Be6TeCTMpOBa X +

C A avn.kgma.kg/webtest/testing

Ru
Kg

KyMap Lllawneuu

Bonpoc: N949
During a prostatectomy, the surgeon attempts to protect the prostatic plexus of nerves which
contains nerve fibers that innervate penile tissue to cause erection. From which nerves do these
fibers originate?

OTBETbl(oqUH OTBeT)
Deepperineal

2 Pudendal

3 Pelvic splanchnics
Genitofemoral

5 Dorsal nerve of the penis

43 44 45 4647 48 49 50

12 3 45678910
11 12 13 14 15 16 17 18 19 20
21 22 23 24 25 26 27 28 29 30
31 32 33 34 35 36 37 36 39 40
41 42 43 44 45 46 47 48 49 50

OTBeeHHLI BOonpoc
nponyuyewHbi Bonpoc
TeKyujun Bonp0C

o 3aBepuennA TeCTa ocTanocb

1:12:20
3aBepunTb TeCT

Scanned with CamScanner


Be6TeCTpu
papM3
acya h 6a3
congenital avn.kgma.kg/webtest/testing

C KyMap LLlanneu
Kg Ru

Bonpoc: NO50
reproductive system
which part of the male
cells) of
(excessive growth of
hyperplasia passage of urine?
Benign
likely to interfere with the
would be most

OTBETbl(oAMH OTBeT)

Prostate
1 Periurethral Zone ofthe
Prostate
Peripheral Zone of the

Central Zone ofthe


Prostate
3

4
Ejaculatory Duct

5 Seminal Vesicle
48 49 50
44 45 46 47
43

1 2 3 456 17789 10
18 19 200
11 12 13 14 15 16
21 22 23 24 25 26 27 28 29 30
400
31 32 33 34 35 36 37 38 39
41 42 43 44 45 46 47 48 49 50

OTBeYEHHLIM BOnpoc
nponyuyeHHbIM BOnpoc
Tekyuiwn BOnpoc

o 3aBepuweHna TeCTa oCTanocb:

1:12:17
3aBepwwTb TeCT

Scanned with CamScanner


What is not typical for the corpus luteum of pregnancy?
1
Secretes progesterone
2 Develops from the corpus luteum of the ovary
3 Actively functions up to 3 months of pregnancy
4 Actively functions throughout pregnancy
5 HCG stimulates the development of the corpus luteum

Determine at what age the absence of secondary sexual characteristics is


considered a sign of delayed sexual development in boys
1 secondary sexual characteristics by 10 years, and sperm maturation by 13
years
2 secondary sexual characteristics by 11 years, and sperm maturation by 14
years
3 secondary sexual characteristics by 14 years, and sperm maturation by 17
years
secondary sexual characteristics by 12 years, and sperm maturation by 15
years
5 secondary sexual characteristics by 13 years, and sperm maturation by 16
years

What frequency range of transducers should be used for transabdominal


examination of the pelvic organs?
1
10.0-15.0 MHz
2 15.0-20.0 MHz
3 7.5-10.0MHz
4 5.0-7.5 MHz
5 3.5 5.0 MHz
-

A 19 year old primagravida presents to the OB clinic. She is 36 weeks


pregnant by last menstrual period dating. By pelvimetry pelvis sizes:
Distancia Spinarum 25cm, - Distancia Cristarum-28 cm,- Disatancia
Trochanterica 30cm, - Conjugata Externa 20 cm. Determine the true
conjugate by the external conjugate.
1
12 cm
211 cm
3 10 cm
4 8 cm
59 cm
Why desquamation of the functional layer of the endometrium occurs
1
Increase in estradiol levels in the blood
2 Hypersecretion of FSH
3 Decrease the prolactin level in the blood
4 Hypersecretion of LH
5 Due to low estrogen and progesterone

During pregnancy, which of the following is not true about the respiratory
system response to pregnancy?
1
There is a respiratory alkalosis
2 The thoracic cage is pushed up and widened
3 The vital capacity is decreased
4 The diaphragm is elevated
5 The residual volume decreases

Patient with amenorrhea has a negative result (absence of menstrual


reaction) after the hormonal test with estrogens and progesterone, it
means
1
Presence central form of amenorrhea
2 Presence uterine form of amenorrhea
3 Presence ovarian form of amenorrhea
4 Presence polycystic ovary syndrome
5 Presence amenorrhea of central genesis

What is the role of digital exam in the evaluation of normal labor?


1
Assessment of fetal well-being
2 Frequency and duration of contractions
3 Fetal movements
4 Time of labor
5 Degree of dilatations of the cervix

The peritoneum covers the following areas of anterior surface of uterus


1
not covered at all
2 the body and entire cervix of uterus
3 the body and supravaginal part of uterus
4 body, fundus of uterus
5 the body of uterus, supravaginal part of cervix, anterior vaginal fornix

Which structure from following not included in perineal body containment


?
1
the superficial transverse perinei m.
2 the m. bulbospongiosus
3 the rectovaginal fascia (Denonvillier's)
4 the m. ischiocavernosus
5 the deep transverse perinei muscle and its fascia

A 21 year old primigravida is expecting her first-child, She is 16 weeks


pregnant by last menstrual period dating She is upset by the appearance of
spots on her face Which of the following terms applies to the dark blotchy,
brown, blanched, patches of skin found on the forehead, upper lip. nose,
and cheeks during pregnancy?
1
Epulis
2 Telangiectasia's
3 Striae gravidarumm
4 Linea nigra
5 Melasma

Gravidogramm is graphic recordings:

Number of deliveries
Symphisis fundal height
Number of pregnancies
Date of pregnancy
Circumference of abdomen

Which of the following would be most likely to be damaged by a stab


wound into the ischiorectal (ischioanal) fossa 2 cm lateral to the anal
canal?
Crus of the Penis with cavernous tissue
Vesicular Bulb
Pudendal Neurovascular bundle
Inferior Rectal Artery, vein
Perineal Body

The third stage of labor continues from:


Cervical dilatation to birth of membranes
Birth of fetus to birth of placenta
Dilatation of the cervix to the birth of the fetus
Separation of the placenta to the birth of the placenta
Birth of fetus to birth of membranes

The onset of labor should be considered by the following criteria:

Regular contractions
Rupture of membrane
True labor pain
Engagement of fetal head
Decent of fetal head

In girls at an early age (from 2 to 8 years), the following are more common:

Salpingo-oophoritis
Ovarian tumors
Congenital anomalies of the genital organs
Vulvovaginitis
Dysfunctional bleeding

Multigravidae woman in postpartum period developed septic pelvic


thrombophlebitis, after inadequately treated endometritis. Where will be the
possibility of coursing the thrombus if the left ovarian vein involved in the
process?

IVC
Portal vein
Azygos vein
Left lumbar vein
Left renal vein

Determine at what age the absence of secondary sexual characteristics is


considered a sign of delayed sexual development in girls.

12-16
14-18
11-14
10-13
13-17

Name the conjugate, which is the determining factor for the outcome of labor

Anatomical conjugate
Diagonal conjugate
Lateral conjugate
External conjugate
True conjugate

What signs characterize the state of readiness of the organism of a pregnant


woman for
delivery

True labor pain


Braxton-Hicks contractions
Cervical ripening
Cervical immaturity
Date of pregnancy

Which of the following is not true (positive) sign of pregnancy?

Amenorrhea
Palpation of fetal parts
Audible fetal heart
Fetal movements
Ultrasound evidence of a fetus

What is the suboccipitobregmatic diameter of the fetal head?

12 cm, circumference 34 cm
13.5 cm, circumference 38 cm
8.0 cm, circumference 30 cm
9.5 cm, circumference 32 cm
10 cm, circumference 33 cm

Specify the age at which girls are determined by the stage of butane breast and
nipple
pigmentation

16-17
10-11
12 13
14-15
8-9

On which of the postpartum days can the women expect lochia serosa

Days 8 to 14
Days 3 and 4
Days 15 to 30
Days 16 to 42
Days 5 to 7

Specify what age corresponds to the degree of puberty (years) MA 3 2x2 My.

8-9
12-13
14-15
10-11
16-17

Male, 30 years old, married for 4 years. Have no children. A history of trauma to
both testicles at the age of 20 for which an organ-preserving operation was
pertormed (which one the patient does not know exactly) on both testicles. On
examination: both testicles are reduced in size, of dense consistency, the vas
deferens are unremarkable The prostate gland is unchanged. Spermogram No
spermatozoa or spermatogenesis cells were found in the ejaculate. Total
testosterone in the blood is 4 nmol/l (the norm is 12-35 nmol ). What is your
diagnosis?

Testicular cancer
Late onset hypogonadism
Congenital atrophy of testicles
Cryptorchidism
Testicular hypogonadism, azoospermia.

For what purpose do we measure the size of the false pelvis? To determine the

Shape of pelvis
Degree of contracted of the pelvis
Size of the small pelvis
Diagonal conjugates
Thickness of the pelvic bones
Which of the following conditions is common in pregnant women in the 2nd
trimester of pregnancy?

Disuria
Mastitis
Respiratory acidosis
Physiologic anemia
Metabolic alkalosis

The ovarian artery is a branch of:

Internal iliac artery


Uterine artery
Inferior vesical artery
Common iliac artery
Abdominal aorta

pregnant is diagnosed with hyperemesis


A 21-year old client, 6 weeks'
gravidarum. This excessive vomiting during pregnancy will often result in which
of the following conditions

Bowel perforation
Miscarriage
Electrolyte imbalance
Toxemia
Pregnancy induced hypertension

Which structure should be taken into account not to damage while ligating the
uterine artery during hysterectomy (amputation of uterus)

Ovarian artery
Ureter
Pelvic splanchnic nerves
Pudendal nerve
Urethra

Pathological amenorrhea does not include:

Amenorrhea of childhood
Amenorrhea in Sheehan's syndrome
Amenorrhea with gonadal dysgenesis
Psychogenic amenorrhea
Amenorrhea in Kallmann syndrome

What is the least contributes to the appearance of infection in the pelvis:

Endometritis
Curettage of the uterus
The semen
Fibrotic changes
Menstruation

During pregnancy, which of the following is not true about the respiratory system
response to pregnancy?

There is a respiratory alkalosis


The thoracic cage is pushed up and widened
The vital capacity is decreased
The diaphragm is elevated
The residual volume decreases
A 14-year-old boy came to the clinic complaining of acute pain in the scrotum
Clinical examination revealed edematous scrotum on both sides the skin is
hyperemic. What imaging study should be done first?

MRI
Hysteroscopy
CT-scan
X-ray
Ultrasound

What is the reason for a slight decrease in blood pressure in the 1st trimester of
pregnancy?

Decreases total peripheral resistance


Increased stroke volume of the heart
Physiological left ventricular hypotension
Increased cardiac output
Increased heart rate

Canal formed by the fascia of m. obturator internus?

Femoral canal
Perineal canal
Pudendal canal
Obturator canal
Inguinal canal

Ultrasound examination using saline solution to visualize uterus is:

Ultrasonography
Hysteroscopy
Cystoscopy
Hysterosalpingograpny
Sonohysterography

Pus leakage from the periuterine space into the inguinal canal along

the cardinal lig of uterus


the suspensory lig of ovary
the inferior epigastric artery
the round lig, of uterus
the broad lig of uterus

The most effective method for diagnosing gonococCus is:

Polymerase chain reaction


Cultural
Linked immunosorbent assay
Bacterioscopic
Immunofluorescent

Why desquamation of the functional layer of the endometrium occurs

Increase in estradiol levels in the blood


Hypersecretion of FSH
Decrease the prolactin level in the blood
Hypersecretion of LH
Due to low estrogen and progesterone

is expected change in the hematologic system during the 2nd trimester of


pregnan

Decreses in protein
Decrease in WBC's
Decrease in sedimentation rate
Increase in blood volume
Increase in hematocrit

Among the following, specify the therapeutic manipulations that are not used for
priapism relief

Novocaine blockade of the spermatic cord


Intracavernous administration of epinephrine solution
Cavernous-spongiose bypass surgery
Puncture of the corpora cavernosa with blood aspiration
Perfusion of the corpora cavemosa with beparin solution

Perineal membrane, superior perineal fascias together with structures of deep


perineal pouch known in clinical anatomy as:

Pelvic floor
Urogenital triangle
Urogenital diaphragm
None is correct
Pelvic diaphragm

What signs characterize the state of readiness of the organsm of a pregnant


woman for delivery

True labor pain


Braxton-Hicks contractions
Cervical ripening
Cervical immaturity
Date of pregnancy

W ich of the following hormones is used as a marker a normally developing


pregnancy in the First trimester?

Estnol
Chorionic gonadotropin
Prolactin
Pituitary gonadotropin
Progesterone
A 26-year old multigravida is 14 weeks' pregnant and is scheduled for an alpha-
fetoprotein test. She asks "What does the alpha-fetoprotein test indicate?" The
bases a response on the knowledge that this test can detect:

Cardiac defects
Kidney defects
Urinary tract defect
Neural tube defects
Heart defect

Enter to what age corresponds to the degree of sexual development (years) Mao
Horomia

14-15
16-17
10-11
12-13
8-9

Which of the following conditions is common in pregnant women in the 2nd


trimester of
pregnancy?

Disuria
Mastitis
Respiratory acidosis
Physiologic anemia
Metabolic alkalosis

Which structure should be taken into account not to damage while ligating the
uterine artery
during hysterectomy (amputation of uterus)?

Ovarian artery
Ureter
Pelvic splanchnic nerves
Pudendal nerve
Urethra

Ovaries are fixed by the following ligaments:

Suspensory ovarian and round ligament of uterus


Suspensory ovarian and proper ovarian
Round lig. and proper ovarian
Cardinal lig. and proper ovarian lig.
Cardinal lig., and rOund lig.

years old, her physical and neuropsychic state


At the reception, the girl is 7
corresponds to he age. There is pubic hair on the female type from 4 years, the
mammary glands are not develop menarche is absent. What is your presumed
diagnosis?

premature puberty
isolated adrenarche
sexual crisis
isolated telarche
hyperpituitarism

Factors that not play a role in the pathogenesis of inflammatory diseases of the
internal genital organs:

Age
Uterine fibroids
Sexual intercourse
Menstruation
IUCD

What is the suboccipitobregmatic diameter of the fetal head?

12 cm, circumference 34 cm
13.5 cm, circumference 38 cm
8.0 cm, circumference 30 cm
9.5 cm, circumference 32 cm
10 cm, circumference 33 cm

Fetal presentation refers to which of the following descriptions:

Fetal head to the uterine fundus


A classification according to the fetal part
Relationship of the presenting part to the maternal pelvis
Relationship of fetal parts to each other
Relationship of the long axis of the fetus to the long axis of the mother
At the reception,a 12-year-old girl, complaints about the appearance of hair on
her fa armpits, mammary glands are not developed, there are no monthly
periods, body we bone age is ahead of the passport. Assume which syndrome
the above data relate to

delayed puberty
false male hermaphroditism
true hermaphroditismn
congenital adrenogenital syndrome
congenital hypothyroidism

After the end of labor, the postpartum woman should be monitored in the
maternity ward for.

30 min
60 min
120 min
180 min
90 min

Which structure is not included in the deep perineal pouch of female?

Superior perineal fascia


M. transversus perinei profundi
External urethral sphincter
Inferior perineal fascia
Cowpers glands
Bonpoc: N91
In which of the following conditions is recommended to perform Hysterosalpingography?

OTBeTbi(oAMH OTBET)
1 Pregnancy
2 O Infertility

3 Pelvic infection

4 OBloody discharge
5 OHeavy periods

2 B 4 5 6 7 50
Bonpoc: N93
Which medical imaging is used to obtain an image of the mucous surface of the uterine cavity?

OTBeTbl(oqMH OTBeT)
1OHysteroscopy
2 O Sonohysterography

3 OUltrasound
4 O MRI

5 OCt-scan

34 5 67 50
Bonpoc: N94
A 43-year-old man came to the clinic with complaints of acute pain in the groin and frequent painful urination. He has a fever and weakness for the second day.
Ultrasound examination of the prostate revealed hypoechoic areas in the periphery. He was diagnosed with a suspected prostate abscess. What imaging study is
preferred to clarify this diagnosis?

OTBeTbI(oAuH OTBeT)
1OX-ray

2O CT-scan

3 OFluoroscopy

4 OPelvic MRI

5 Nuclear imaging

6 7 8 S0

07:39
OEi O ) ENG
31-12-2020 L
Bonpoc: N96
Determine at what age girls have breast enlargement and nipple pigmentation

OTBeTbi(OAMH OTBeT)
1O10-11

2 016-18
3 O 12-13

4 O 14-15

5 8-9

1 2 5
67 8 10 50
Bonpoc: N95
At what age are boys determined by the appearance of facial hair, pigmentation of the scrotum, the first ejaculation?:

OTBeTbi(oaMH OTBET)
1O14-15
20 12-13

3 8-9

4 10-11
5 16-17

1
2 A 5 6 7 8 9 50
BonpoC: N97
to the degree of sexual development (years) Mao Horomia:
Enter to what age corresponds

OTBeTbi(OAMH OTBeT

1 8-9

20 12-13

3 010-11
4 O 14-15

5 0 16-17

67 8 50
Bonpoc: N98
on the female type from years, the
girl is 7 years old, her physical and neuropsychic state corresponds to her age. There is pubic hair
Atthe reception, the 4
mammary glands are not developed, menarche is absent. What is your presumed diagnosis?

OTBeTbi(o4MH oTBeT)
1 O isolated adrenarche

2 Opremature puberty
3 O sexual crisis

4 O isolated telarche

5 Ohyperpituitarism

4 10 1 12 50

07:46
O D )ENG 31-12-2020
Bonpoc: N°9
Objectively physical development corresponds to age. testicles with a volume of 1 m, palpated on both sides, the scrotum is
At the reception, the boy is 2 years ald.
split, urethral hypospadias are noted. Determine which diagnosis corresponds to the objective data?

OTBeTb1(o4uH OTBeT)
1O a healthy child

2 O false hermaphroditism

3 Otrue hermaphroditism
4 premature sexual development
5 delayed sexual development

1
e112 13 50

0747
O d) ENG
31-12-2020
Bonpoc: N910
A 15-year-old boy was examined. Objectively: mutation (breaking) of the voice, noticeable protrusion of the thyroid catilage. Thick, curly pubic hair in the form of a
triangle. There is no hair in the armpit or on the face. Evaluate the boy's sexual development according to Tanner

OTBeTbi(oAnH OTBET)
1 OV1 P4 L1 Ax0 FO

2 OV1 P1 LO AxO FO

3 Ov2 P5 L2 Ax4 F3

4 V2 P3 L1 Ax2 FO

F
5 Ov2 P5 L2 Ax4

6 78910 1 12 13 14 **
S0

07:47
n 9) ENO 31-12-2020 L
Bonpoc: Ne12
the>
He notes the urge to urinate, but
cannot urinate on his own. Above
while playing football. Urethroragia appeared. kind
the crotch
was kicked in from the urethra. What
There is a perineal hematoma. Blood released
A 20-year-old patient is
a dull sound is revealed during percussion.
pubis, a bulging is determined, over which
of damage do you assume?

OTBETbi(oqMH OTBET)

1 OKidney rupture
2 ODamage to the prostate

3OUrethralruptured
4ORuptured bladder

5 Balanoposthitis

8 9 10 11 12 13 14 15 16 S0
Bonpoc: Ne13
Male, 30 years old, married for 4 years. Have no children. A history of trauma to both testicles at the age of 20, for which an organ-preserving operation was
performed (which one the patient does not know exactly) on both testicles. On examination: both testicles are reduced in size, of dense consistency, the vas deferens
are unremarkable. The prostate gland is unchanged. Spermogram. No spermatozoa or spermatogenesis cells were found in the ejaculate. Total testosterone in the
blood is 4 nmol/I (the norm is 12-35 nmol / 1). What is your diagnosis?

OTBeTbl(OAMH OTBeT)

1OTesticular hypogonadism, azoospermia


2 O Congenital atrophy of testicles

3 OTesticular cancer
4 Late onset hypogonadism
5 Cryptorchidism

9 12 1415M1617 S0

07:51
D la) ENG
31-12-2020
Bonpoc: N914
What signs characterize the state of readiness of the organism of a pregnant woman for delivery?

OTBeTbi(OAMH OTBET)
1 True labor pain

2 O Date of pregnancy

3O Braxton Hicks contractions

4 Cervical immaturity

5 Cervical ripening

10 11 13
14 15 16 17 18 50
Bonpoc: N916
Which symptom refers to the probable signs of pregnancy?

OTBeTbI(oqnH OTBeT)

10Nervous irritation
2 Pigmentation changes
3 Appetite changes

4 Nausea and vomiting

5 Uterine enlargement

1
12 13 14 1516
15 17 18 19 20 50
Bonpoc: Ne17
The labor has the following stages

OTBETbi(OAMH OTBET)
1 Dilation, expulsion, delivery of placenta, early postpartum

2 0 Preliminary, dilatation, expulsion, postpartum

3 O Effacement of the cervix, the expulsion, delivery of placenta

4 Preparation, dilation, expulsion, early postpartum

5 Preparation, dilation, expulsion

1 ** 13 14 15 16 17 18y 19 20 21 ** 50
Bonpoc: Ne19
What is fetal lie? This is relation of:

OTBeTbi(oAMH OTBET)
1 Fetal parts to each other

2 Fetal head to the fundus

3 Fetal long axis to the long axis of uterus

4 O Fetal head to the pelvic inlet

5 Fetal back to the left or right sides of uterus

15 16 17 18 19 20 21 22 23 50
Bonpoc: N920
Which of the following hormones is secreted during pregnancy the anterior lobe of the pituitary gland ofthe mother and fetus?

OTBeTbi (oAMH OTBeT)


1 OProlactin

2 O Human chorionic gonadotropin

3 O Estriol

4 O Human placental lactogen

5 Progesterone

16 17 18 19
208 22 23 24 50
Bonpoc: N921
In the first day of the postpartum period, the fundus of the uterus Is

OTBeTbi(oqMH oTBeT)
1 3 transverse fingers below the navel

2 O At the level of the navel

3 O Midway between the navel and the symphysIS

4 2 transverse fingers above the navel


5 O 3 transverse fingers above the navel

22 23
17 18 19 20
21 24 25 50
Bonpoc: Ne23
Indicate purpose of external conjugate measurement. o determine:

OTBeTbl(oAnH oTBeT)
1 Diagonal conjugates

2 O Thickness of the pelvic bones

3 Lateral conjugates

Shape of pelvis

5 True conjugate

1 19 20 21 22 23 24 25 26 27 50
Bonpoc: N25
Active management of the third stage of labor includes:

OTBeTbi(oAuH OTBET)
1 Supervision of the woman in labor

2 Manual separation of the placenta


3 Instrumental separation of the placenta
4 O Controlled cord traction

5 Ice on the lower abdomen

21 22 23 24 2526 27 28 29 50
Bonpoc: N°27
When involved in prenatal teaching, the doctor should advise the clients that an increase in vaginal secretions during pregnancy is called leukorrhea and is caused by
increased:

OTBeTbi(oquH OTBeT)
1OSupply of sodium chloride to the vagina

2 O Functioning of the Bartholin glands

3 OIncrease blood supply


4 O Metabolic rates

5 O Production of estrogen

23 24 25 26 27 28 29 30 31 50

08:12
D( ) ENG 31-12-2020
Bonpoc: N 26
What causes low blood pressure during first half of pregnancy?

OTBeTbi(oqnH OTBET)
1 Uterine blood flow increases

2 Progesterone effect

3 Blood flow to skin increases

4 OBlood flow increase

5 Blood flow to brain increases

22 23 24
25 26 2 28 29 30 50
Bonpoc: Ne28
A 25 year-old woman has vaginal delivery of a baby in her first pregnancy After delivery newborn was assessed by neonatologist. Which of the following is a no
sign of fetus born at term:

OTBeTbi(OAMH OTBeT)
1O Good newborn reflex

2 VWeight >2500 gr., length 47 cm.

3 O Testes within scrotum

4 Length of hair > 2 cm

5OWeight <2500 gr., length 45 cm.

24
25 28
26 27 29 30 31 32 50
Bonpoc: N932
According to the WHO classification (1980), postmenopause is called

OTBeTbi(O4MH OTBeT)
1O The period from the onset of menstruation to the last menstruation

2 Period of first menstruation

3 O Period of stable menstrual function

4 Last menstruation

5 The length of time after the last menstruation

30 31 32 33 34 35 36 50
Bonpoc: N935
Women with AUB are at risk:

OTBeTbi(OAMH OTBET)
1 On the development of placental insufficiency and anomalies of the birth force
2 On the development of genital tumors
3 O Miscarriage

4 On the development of breast tumors


5 Pregnancy

31 32 33 34 35 36 37 38 39 50
Bonpoc: N936
womanbperiod esery 33 days lasting 8days with iarge ciots being passed. She could be dassifed as having

OTBETDI(oAMH oTBeT)
Meortaga

2 Menanagia

ysamenonea
agja

34 39 40
avn.kgma.kg/webtest/testing
Maps KSMA Sub entre Session.

Kancapn NpTM

Bonpoc: N936
as
A woman has period every 33 days lasting 8 days with large clots being passed. She could be classified having

OTBeTbi(OAUH OTBeT)

O Metrorrhagia

OMenorrhagia

Dysmenorrhea
Menometrrohagia

5 OPolymenorrhea

38 39 40 50
32 33 34 35 36 87

08:22
) ENG 31-12-2020
Bonpoc: N937
For anovulatory menstrual cycle with short term persistence of a mature follicle is not typical:

OTBeTbi(oAMH OTBET)

1O Single-phase basal temperature

2 Prolonged spotting discharge

3 O High estrogen levels

4 Symptom of the pupil (+++)

5 In the scraping of the endometrium in the second phase of the cycle

33 34353637 38 39 40 41 50
Bonpoc: N939
A 23-year-old female patient complains of mucopurulent discharge from the genital tract, itching at the antenatal clinic. By examination: the mucous of the cervix is
sharply hyperemic, edematous. The discharge is profuse, mucopurulen. uterus in anteversio-flexio, not enlarged, painless, the appendages are not defined. What is
the mostlikely diagnosis:

OTBeTbl(oanH OTBeT)
1 OEndocervicitis.

2 O Endometritis

3 O Vulvovaginitis

4 OBartholinitis
5 OAcute salpingo-oophoritis

35 40 41 42 43 50

08:25
D )EO
O C 31-12-2020
kgma.kg/webtest/testing
s KSMA E Sub Centre Session..

Kancap npwrHu

Bonpoc: N940
Patient L, 24 years old, complains of profuse vaginal discharge with an unpleasant smell of "rotten fish". Gynecological examination: the external genital organs and
vaginal mucosa without signs of inflammation. Vaginal discharge copious, watery, with an unpleasant odor. Internal genital organs without pathology. During
bacterioscopy of smears from the cervical canal and urethra, "key cells" were found.What is the most likely diagnosis:

OTBeTbl(OAMH OTBeT)
1 OBartholinitis

2 OBacterial vaginosis
3 OChlamydial cervicitis
OCandidiasis vulvovaginitis

5 O Atrophic colpitis

36
36 37 38 39 40 41 42 43 44 50

08:26
) ENG31-12-2020
Bonpoc: N944
The patient after childbirth was diagnosed with endometritis. Describe the possible complications of acute inflammation of the endometrium:

OTBeTbi(oAuH oTBeT)
1 tubal infertility
2OOvarian cyst
3O Becomes chronic

4 O Hyperestrogenemia

5 PCOS

40 41 42 43 44 45 46 47 48 50
Bonpoc: N9465
Innervation of superficial perineal pouch formed by?

OTBeTb(oquH OTBeT)
1 OPerineal nerve
20 Superficial perineal nerve

3 Superficial pudendal nerve

Genitofemoral nerve

5 Internal pudendal nerve

42 43 45 46 47 48 49 50
Bonpoc: N947
Which level doesn't exist in the pelvic cavity division?

OTBETbi(o4MH OTBeT)
1
Peritoneale
2 O Subperitoneal

3 Subcutaneus
4 All correct
5 Superficial

1 43 44 45 46 48 49 50
SAMSUNG
93% 0900am
BeTecsee Bes p Be6 Teerpom

O C Aavn kgma kg/webtest/testing

KyMap faneneu Ru

Bonpoc: N°1
This condition is a common cardiac dysrhythmia characterized by prermature heartbeats
ornginating in the atria. While the sinoatrial node typicaly regulates the heartbeat during normal
ainus thythm, this condition occurs when another region of the atrla depolarizes before the
sinoatral node and thus triggers a premature heartbeat. Select correct answer

OTBETbI(oAMH OTBeT)

Supraventricular tachycardia

2 Ventricular premature beats

Torsade's de pointes

Ventricular tachycardia

Supraventricular extrasystole

5
6 50

2 34 5 678 9 10
11 12 13 14 15 16 17 18 19 20
21 22 23 24 25 26 27 28 29 30
31 32 33 34 35 36 37 38 39 40
41 42 43 44 45 46 47 48 49 50

oTBeeHHLah BOnpoc
nponyuewHui Bonpoo
Texyunn Bonpoc

Ao saBepueHwa TecTa oCTanocb:

1:14:58
BapenILIA
Scanned wíth Camscanner
SAMSUNG

93% 9.00 am
Be6TecTHpOBO X h BaadapM3 Be6TeCTupoBa X

C A avn.kgma.kg/webtest/testing

KyMap Wawnew Kg Ru

Bonpoc: N 2
What is the location of the auscultation point of mitral valve?

OTBETbI(ogMH OTBeT)

2nd intercostal space, left border of sternum

4th intercostal space, midclavicular line

3 2nd intercostal space, right border of sternum

2nd intercostal space, midclavicular line

5 5th intercostal space, left midclavicular line

4 5 67 50

3 4 5 6 7 8 9 10
11 12 13 14 15 16 17 18 19 20
21 22 23 24 25 26 27 28 29 30
31 32 33 34 35 36 37 38 39 40
41 42 43 44 45 46 47 48 49 50

OTBeYeHHbi BOnpoc
nponyujeHHbiM Bonpoc
Tekyunn Bonpoc

Ao 3aBepweHwA TecCTa oCTanoCb

1:14:55
3aBepwwTb TeCT

Scanned with Camscanner


Be6TeCTMpOBaHX h Baa papm3 Be6TecTupoBa

O C A avn.kgma.kg/webtest/testing

Kymap Wawneu Kg Ru

Bonpoc: N93
Single ventricular impulses caused by reentry mechanism within the ventricle or abnormal
automaticity of ventricular cells. They may present in both healthy patients and patients with a
heart disorder, it may be asymptomatic or cause palpitations. Diagnosed by electrocardiography
What is it?

OTBETbI(oaMH OTBeT)
1 Ventriculartachycardia

2 Ventricular extrasystoles

3 Ventricularfibrillation

4 Partial atrioventricular block

5 Full atrioventricular block

1 2 3
4 5 6 7 50

1 23 4 5 6 7 89 10
11 12 13 14 15 16 17 18 19 20
21 22 23 24 25 26 27 28 29 30
31 32 33 34 35 36 37 38 39
41 42 43 44 45 46 47 48 49 50

OTBeveHHbIM BOnpoc
nponyuyeHHbl Bonpoc
TeKyunn Bonpoc

Ao saBepuweHMA TeCTa oCTanocb

1:14:52
SaBepuIMTh TecCT
Scanned with CamScanner
93%l93% 9:00 am
Be6 TecTMpOBa X h Ba3 dapM3 Be6 TeCTpOBa X

C A avn.kgma.kg/webtest/testing

KyMap LlWanneu Kg Ru

Bonpoc: N94
Which BP level is considered as arterial hypertension? (ESC classification)

OTBETbi(oAMH OTBeT)

1 2140/90 mm Hg
2130/90mm Hg
3 2130/80 mmHg
4 140/100 mm Hg
5 2120/80mm Hg

2 67 8 50

1 234 5 6 7 8 9 10
11 12 13 14 15 16 17 18 19 20
21 22 23 24 25 26 27 28 29 30
31 32 33 34 35 36 37 38 39 40
41 42 43 44 45 46 47 48 49 50

OTBeYeHHbli Bonpoc
nponyueHHbl BOnpoc
TeKyunn Bonpoc

Ao saBepueHWR TecTa oCTanoCb:

1:14:49
3aBepuiTb TeCT

Scanned with CamScanner


Be6TecTHpOBaH h baa dapM3 Be6TecTupoBaX

O C avn.kgma.kg/webtest/testingg

KyMap lawnew Kg
Ru
Bonpoc: N°5
Which of the following numbers is optimal blood pressure level? (ESC classification)

OTBeTbli(ogMH OTBET)

1 119/80 mm Hg

180/100 mm Hg

3 160/70 mm Hg

4 140/92 mm Hg

5 135/90 mm Hg

2 34 6 8 9 50

123 4 56 7 8 9 10
11 12 13 14 15 16 17 18 19 20
21 22 23 24 25 26 27 28 29 30
31 32 33 34 35 36 37 38 39 40
41 42 43 44 45 46 47 48 49 50

OTBeeHHbI BOnpoc
nponyujeHHbin BOnpoc
TeKyuinn BOripoc

Ao 3aBepweHna TeCTa oCTanoCb

1:14:46
3aBepuTb TeCT

Scanned with CamScanner


SAMSUNG
93% 09.00 am
Be6 TecTpOBa X h Baa þapM3 Be6TecTwponaX

C A avn.kgma.kg/webtest/testing

KyMap llawneus Kg Ru
Bonpoc: NP6
What is dyspnea?

OTBeTbi(oaMH OTBeT)
fatigue
dizziness

shortness of breath

fainting

sensation of heartbeat

1 2 3 456 8 910 50

2 3 4567 89 10
11 12 13 14 15 16 17 18 19 20
21 22 23 24 25 26 27 28 29 30
31 32 33 34 35 36 37 38 39 40
41 42 43 44 45 46 47 48 49 50

OTBeeHHbIMBOnpoc
nponyuyeHHbi BOnpoc
Tekyunn Bonpoc

Ao 3aBepuueHna TecTa oCTanocb

1:14:43
3aBepunTb TeCT

Scanned with CamScanner


SAMSUNG
93% 0 9.00 am
Be6TecTpoBa X h Baa chapM3 Be6TeCTMpoBa

C A avn.kgma.kg/webtest/testing
KyMap lawneus Kg Ru

Bonpoc: N°7
Q wave at ECG during syndrome of acute coronary
What reflects appearance of pathological
insufficiency?

OTBeTbi(OAMH OTBeT)

Myocardial Injury

Subepicardial Myocardial Ischemia


2
3 Transient Myocardial Ischemia

4 Subendocardial Myocardial Ischemia

5 Myocardial Necrosis

11 50
3 5
6 8 9 10

3 45 67 10
12 89

11 12 13 14 15 16 17 18 19 20
21 22 23 24 25 26 27 28 29 30
31 32 33 34 35 36 37 38 39 40
41 42 43 44 45 46 47 48 49 50
OTBeYeHHbIMBOnpoc
nponyujeHHblM Bonpoc
TeKyujni Bonpoc

Jo 3aBepweHnA TeCTa ocTanocb


1:14:37

Scanned with CamScanner


SAMSUNG

93%l93%9.00 am
Be6TecTHpOBa X h baa papM3 Be6TeCTMpoBa X

C A avn.kgma.kg/webtest/testing

KyMap lanneu Kg Ru
Bonpoc: Ne8
This arrhythmia is presented as a fast, abnormal heart rate. It starts in hear's ventricles, and is
defined as 3 or more heartbeats in a row, at a rate of more than 100 beats per minute. How is it
called?

OTBeTbl(OAMH OTBeT)

1 Atrialfibrillation

2 Atrial flutter

3 Ventricular tachycardia

4 Sinus bradycardia

5 Supraventricular tachycardia

5 6 7 9 10 1112 50

12 3 4 56 7 89 10
11 12 13 14 15 16 17 18 19 20
21 22 23 24 25 26 27 28 29 30
31 32 33 34 35 36 37 38 39 40
41 42 43 44 45 46 47 48 49 50

OTBeeHHblM BOnpoc
nponyujeHHbiM BOnpoc
TeKyunn BOnpoc

Scanned with Camscanner


SAMSUNG

94% 9.00 am
Be6TecTHpOBa Baa dapM3 Be6TecTwpoBa
C Aavn.kgma.kg/webtest/testing

KyMap lanneu Kg Ru

Bonpoc: N°9
Which valvular disease is typical for the development of De Musset's sign (rhythmic nodding of
the head)?

OTBeTbi(oAMH OTBeT)

1 Mitralregurgitation
2 Aortic stenosis

3 Aorticregurgitation

4 Mitral stenosis
5 Tricuspid regurgitation

6 7 89 10 11 12 13 50

Scanned with CamScanner


SAMSUNG

94% 9:00am
Be6TecTpOBa X i Baa dapM3 Be6 TecTMpoBa

CA CA avn.kgma.kg/webtest/testing

KyMaplawneu Kg Ru

Bonpoc: N°10
This arhythmia occurs when a "reentrant" circuit is present, causing a repeated loop of electrical
activity to depolarize the atriums at a rate of about 250 to 350 beats per minute. This produces a
characteristic "sawtooth pattern of the P waves. Which condition do you think about?

OTBETbi(oqMH OTBeT)

1 Atrialflutter

2 Ventricular tachycardia

Atrial premature beats

4 Ventricular fibrillation

5 Atrial fibrillation

6 7 8 10 11 1213 14

50

1 2 3 4 567 8 9 10
11 12 13 14 15 16 17 18 19 20
21 22 23 24 25 26 27 28 29 30
31 32 33 34 35 36 37 38 39 40
41 42 43 44 45 46 47 48 49 50
OTBeyeHHblM BOnpoc
nponyuyeHHblM BOnpoc
Tekyunn Bonpoc

Scanned with CamScanner


SAMSUNG

94% 9.00 am
Be6 TecTpoBaX h Ba3 dapM3 Be6TecTMpoBa X +

CA C A avn.kgma.kg/webtest/testing

KyMap lanneuw Kg Ru

Bonpoc: N°11
This condition can be recognized as a consequence of using of drugs (digoxin, beta blockers,
calcium channel blockers), increased vagal tone, sinoatrial conduction disease, right coronary
infarction, at ECC: progressive lengthening of the PR interval until a beat is dropped. Select
answe

OTBeTbi(oAMH OTBeT)

Atrioventricular block, first-degree block

2 Right bundle branch block

3 Atrioventricular block, third-degree block

AVblock, Mobitz type l1/Wenckebach

5 AV block, Mobitz type 1/Wenckebach

7 9
1011 12 13 14 15

50

2 3 4 56 789 10
11 12 13 14 15 16 17 18 19 20
21 22 23 24 25 26 27 28 29 30
31 32 33 34 35 36 37 38 39 40
41 42 43 44 45 46 47 48 49 50
OTBeeHHblM BOnpoc
nponyueHHbIM BOnpoc
TeKyuwn BOnpoc

Scannedwith CanmScanner
SAMSUNG

94% 09.00 am
Be6 TecTpoBa X h Baa cþapM3 Be6TecTMpoBa +

C A avn.kgma.kg/webtest/testing

KyMap Waunew Ru

Bonpoc: Ne12
Which of the listed mechanisms of myocardial hypoxia occur in the chronic coronary
insufficiency syndrome?

OTBETbi(oqMH OTBeT)

1 reduced oxygen utilization

2 insufficient oxygen supply

3 circulatory disorders

4 conduction disturbance

5 valvular dysfunction

9 10 11 12
1314 15 16

50

Scanned with CamScanner


SAMSUNG
l94% 09:00 am
BeóTecTHpoBa X i 6aa dhapm3 Be6TeCTMpOBa X

C A avn.kgma.kg/webtest/testing

KyMap lannew Kg Ru
Bonpoc: Ne13
Which of the followings isolated hypertension?

OTBeTbi(ogMH OTBeT)
180/100 mm Hg
160/110 mm Hg

140/90 mm Hg

140/80 mm Hg

150/90 mm Hg

9 1011 12 13 14 15 16 17

50

23 4 567 89 10
11 13 14 15 16
12 17 18 19 20
21 22 23 24 25 26 27 28 29 30
31 32 33 34 35 36 37 38 39 40
41 42 43 44 45 46 47 48 49 50

OTBeyeHHbi BOnpoOc
nponyujeHHbIM BOnpoc
TeKyuw Bonpoc

Ho 3aBepuweHna TeCTa oCTanocb

1:14:15
3aBepuwwTb TeCT

Scanned with CamScanner


SAMSUNG

94% 94%99.00 am

Be6TecTHpOBa X h Baa hapM3 Be6TecTpoBa

O C A avn.kgma.kg/webtest/testing

KyMap Wannew Kg Ru

Bonpoc: Ne14
What characterizes syndrome of endocardium inflammation?

OTBETbi(ogMH OTBeT

Normal body temperature

Dizziness
3 Burning anginal pain

Hemorrhage in the skin and mucous membranes

5 Epigastric pain

10 11 2 13 14 1516 17 18

50

1 23 4 5 678 9 10
11 12 13 1415 16 17 18 19 20
21 22 23 24 25 26 27 28 29 30
31 32 33 34 35 36 37 38 39 40
41 42 43 44 45 46 47 48 49 50
OTBeYeHHbIM BOnpoc
nponyueHHbI Bonpoc
Texyuwn Bonpoc

o 3aBepueHna TeCTa oCTanoCb

1:14:12
3aBepuiwTb TeCT

Scanned with CamScanner


SAMSUNG

94% 0 9:00 am
BeóTeCTHpOBaN X h Ba3 papM3 Be6 TecTMpoBa X

C A avn.kgma.kg/webtest/testing

KyMap Llannew K Ru

Bonpoc: N915
Endomyocardial biopsy is gold standard in diagnostics of?

OTBETbI(oqMH OTBeT)

coronary insufficiency syndrome

2
arterial hypertension syndrome

3
heart failure syndrome

syndrome of myocardium inflamation

5 syndrome of pericardium inflamation

11 12 13
1415 16 17 18 19

50

12 3 4 5 6 7 8 9 10
11 12 13 14 15 16 17 18 19 20
21 22 23 24 25 26 27 28 29 30
31 32 33 34 35 36 37 38 39 40
41 42 43 44 45 46 47 48 49 50
OTBeYeHHbiM BOnpoc
nponyueHHbiM BOnpoc
TeKyun Bonpoc

Ao 3aBepweHna TecTa oCTanocb

1:14:08
3aBepuiwTb TeCT

Scanned with CamScanner


Be6TectpoBar X
ibaa dapM3 Be6TeCTpoBaX

O C A
avn.kgma.kg/webtest/testing

KyMap Llawneus Kg Ru

Bonpoc: N°16
Patient with complaints like dyspnea at the rest, orthopnea and paroxysmal nocturnal dyspnea,
palpitations, chest pain admitted to hospital. The symptoms are worsening over the years. In
anamnesis he had an Coronary heart disease. Echocardiography showed ejection fraction 20%
What is your diagnosis?

OTBeTbi(oguH OTBeT)

1 Chronic Heart failure Class l

2 Chronic Heart failure Class ll

3 Chronic Heart failure Class

Chronic Heartfailure Class IV


5 Acute Heart failure

14
12 13 15
16 17 18 19 20
50

12 3 4
567 89 10
11 12 13 14 15 16 17 18 19 20
21 22 23 24 25 26 27 28 29 30
31 32 33 34 35 36 37 38 39 40
41 42 43 44 45 46 47 48 49 50

OTBeveHHbIM BOnpoc
nponyujeHHbiM BOnpoc
Tekyuwn Bonpoc

Ao 3aBepuieHWA TeCTa oCTanocb

1:14:05
Scanned with CamScanner
SAMSUNG

94% 09:00am
Be6TeCTpOBar X o Baa hapM3 Be6TecTpOBal +

C A avn.kgma.kg/webtest/testing

KyMap Wawneu Kg Ru
Bonpoc: Ne18
A 53-year-old man admitted to the hospital with complains on burning pain behind the sternum,
radiating to left hand, accompanied by cold sweat, weakness. Pain has developed at rest and
lasts for 3 hours. The last few days patient has been drinking heavily and smoking a lot. What is
he most reliable laboratory diagnostic method to clarify the diagnosis?

OTBeTbI(oquH OTBeT)

1 Tropinins
2 CRRP

3 Creatinine
4 CBC
5 AST

14 15 16 17 18 19 20 21 22

50

1 2 3 4 5 67 89 10
11 12 13 14 15 16 17 18 19 20

21 22 23 24 25 26 27 28 29 30
31 32 33 34 35 36 37 38 39 40
41 42 43 44 45 46 47 48 49 50
OTBeyeHHblM BOnpoc
nponyueHHbI BOnpoc
TekyuMM BONpoc

Ao 3aBepuweHna TeCTa oCTanocb

1:14:00
Scanned With Camscanner
9:01 am
94%
Be6TeCTHpOBaH X h bas cpapM3 Be6TecTMpOBaX
C Aavn.kgma.kg/webtest/testing

KyMap llawnew Kg Ru

Bonpoc: N°17
According to NYHA classification of heart failure, which of the following is associated with no
limitation of physical activity?

OTBeTbi(oguH OTBeT)
1 Heart
failure Class IV

2 Heart failure Class ll with preserved ejection fraction

Heart failure Class l

Heartfailure Class

5 Heart failure Class II with reduced ejection fraction

13 14 1516 17 18 19 20 21

50

1 2 34 567 89 10
11 12 13 14 15 16 17 18 19 20
21 22 23 24 25 26 27 28 29 30
31 32 33 34 35 36 37 38 39 40
41 42 43 44 45 46 47 48 49 50

OTBeYeHHbI BOnpoc
nponyueHHbIM BOnpoc
T TeKyunn Bonpoc

go 3aBepweHnA TeCTa ocTanocb

1:13:56
3aBepuWTb TecT
Scanned with CamScanner
SAMSUNG

94% 09:01 am
Be6TecTHpOBO X h Ba3 chapM3 Be6TecTpoBa X

C A avn.kgma.kg/webtest/testing

KyMap Wawneuu Kg Ru

Bonpoc: N°19
Patient is describing his complaints as: "my heart flip-flops, skips beats, especially if I carry stuff
up his stairs or when I'm bending down.". ECG showed no visible P waves and an irregular narrow
QRS Complexes. What do you suspect?

OTBETbl(oguH OTBeT)
1 atrialfibrillation
2 atrial flutter

3 ventricular tachycardia

4 supraventricular tachycardia

ventricular fibrillation

15 16 17 18 19 20 21 22 23
50

12 3 4 5 67 89 10
11 12 13 14 15 16 17 18 19 20
21 22 23 24 25 26 27 28 29 30
31 32 33 34 35 36 37 38 39 40
41 42 43 44 45 46 47 48 49 50
OTBeYeHHbiM Bonpoc
nponyuyeHHbiM BOnpoc
TeKyujnn BOnpoc

Ao 3aBepuweHwa TeCTa oCTanocb

1:13:52
Scanned with CamScanner
Be6 TecTpoBa X h Baa dapM3 Be6 TecTpoBa

C A avn.kgma.kg/webtest/testing

KyMap Llauneu Kg Ru

Bonpoc: N°20
54 year old male admitted to physician with complaints for an chest pain, nocturnal dyspnea,
fatigue. On auscultation: loud S1 and opening snap after s2. Which of the following diagnostic
tools can confim diagnosis?

OTBeTbi(oAMH OTBeT)
1 Chest x-ray

2 CT/MRI

3 ECG
Spirometry

5 Echocardiography

16 17 18 19 20 21 22 23 24

50

12 3 4 5678 9 10
11 12 13 14 15 16 17 18 19 20
21 22 23 24 25 26 27 28 29 30
31 32 33 34 35 36 37 38 39 40
41 42 43 44 45 46 47 48 49 50
OTBeYeHHblM BOnpoc
nponyuyeHHblM BOnpoc
Texyuw Bonpoc

Ao 3aBepweHnA TeCTa oCTanocb

1:13:49
Scanned with Camscanner
SAMSUNG

94% 9:01 am

Be6TeCTHpOBa X h 5a3 (hapM3 Be6TecTMpoBa X

C A avn.kgma.kg/webtest/testing

KyMap lanneu Kg Ru

Bonpoc: Ne21
A 45 year old male admitted to the hospital with complaints of fever for 2 weeks up to 39
degrees, severe weakness. A history of congenital heart disease since childhood bicuspid
aorticvalve. RR-20 per minute. Auscultation-diastolic murmur in the aorta region. BP
40 mm Hg. CBC: increased ESR. Echocardiography mobile vegetation of large sizes on the
-tic valve. Blood culture results: Staphylococcus aureus. What is your preliminary diagnosis?

OTBeTbi(OAMH OTBeT)

Syndrome of inflammation pericardium

2 Syndrome of acute coronary insufficiency

3 Syndrome of inflammation endocardium

4 Syndrome of inflammation myocardium

5 Syndrome of mitral valvular diseases

17 18 19 20 21 2223 24 25

50

1 2 3 4 5 6 78 9 10
11 12 13 14 15 16 17 18 19 20
21 22 23 24 25 26 27 28 29 30
31 32 33 34 35 36 37 38 39 40
41 42 43 44 45 46 47 48 49 50
OTBeHeHHbIM BOnpoc
nponyuyeHHblM Bonpoc
Tekyuwi Bonpoc

Ao saBepuweHwa TeCTa oCTanocb.

Scanned with CamScanner


Be6TecTHpoBa h Eaa dapM3 Be6TecTMpoBa
C
f A avn.kgma.kg/webtest/testing

KyMap llanneu Kg Ru

Bonpoc: N°22
A 63-year-old woman complains on the chest pain developed at rest and lasted for 1.5 hours
accompanied by cold sweat, weakness. Nitroglycerin didn't help to relief the pain. Patient history
retrostermal chest pain worries her for about a year, which usually happens during moderate
physical exertion and lasts for about 5 minutes. What is your preliminary diagnosis?

OTBeTbl(oAMH OTBeT)

Stable angina FCIV


2 New-onset angina pectoris

3 Unstable angina

4 Vasospastic angina

5 Acute myocardial infarction

18 19 20 21 22 23 24 25 26

50

12 3 456 7 89 1
11 12 13 14 15 16 17 18 19 20
21 22 23 24 25 26 27 28 29 30
31 32 33 34 35 36 37 38 39 40
41 42 43 44 45 46 47 48 49 50

OTBeYeHHblM BOnpoc
nponyuyeHHbl BOnpoc
TeKyuin Bonpoc

Ao 3aBepuweHna TeCTa oCTanoCb

1:13:40
Scanned with Camscanner
Be6 TeCTHpOB X h Bas dapm3 Be6TecTpoBa +
C A avn.kgma.kg/webtest/testing

Kymap Wawneu Kg Ru

Bonpoc: N°23
40 years old female without admitted to physician with complaints for an feeling of irregular
heartbeat, such as: pounding or jumping, missed beats. Doctor performed an auscultation and
revealed rare iregular heartbeats. Which diagnostic test should be recommended in this case?

OTBeTbI(oAMH OTBET)

1 Selective coronaryangiography

2 RestingECG

3 Transthoracic echocardiography

Transesophageal echocardiography

5 24 ECG (Holter) monitoring

19 20
2122 2324 25 26 27

50

1
2 3 4 5 6789 10
11 12 13 14 15 16 17 18 19 20
21 22 23 24 25 26 27 28 29 30
31 32 33 34 35 36 37 38 39 40
41 42 43 44 45 46 47 48 49 50

OTBeveHHblM BOnpoc
nponyueHHbiM BOnpoc
rexyuwi Bonpoc

Ao 3aBepuweHwA TeCTa oCTanoCb

1:13:36
Scanned with CamScanner
Bonpoe 24

m eg

Oreerog oTRer)

1:13:32
Scanned wíth CamScanner
Bonpoc: 25
e gs e

of he dgot te

OTBeT(og OTBeT)

Scanned with CamScanner


Bonpoc: N°25
52years old female admitted to the family doctor with the complaints on intense headaches,
dizziness, noise in ears and weakness at night. Heavy smoker. Examination: no signs of heart
failure. Heart rate 68 per minute, blood pressure is 135/85 mm Hg. Heart sounds are clear. ECG:
sinus rhythm, regular, 68 in 1 min. Echocardiography: normal. Which of the diagnostic test you
are going to administer to this patient?

OTBeTbI(oAMH OTBeT)

1 24-hourblood pressure monitor

2 Stresstest
3 Chest Xray

4 Coronary angiography

5 Echocardiography

21 22 23 24 25 26 27 28 29

50

1 2 3 4 5 67 89 10
11 12 13 14 15 16 17 18 19 20
21 22 23 24 25 26 27 28 29 30
31 32 33 34 35 36 37 38 39 40
41 42 43 44 45 46 47 48 49 50

OTBeyeHHblM BOnpoc
nponyujeHHblM BOnpoc
Tekyuni Bonpoc

Ao saBepweHwA TeCTa oCTanocb:

Scanned with CamScanner


SAMSUNG

94% 9.01 am
Be6TecrpoBa X hBa3 chapM3 Be6TecTMpOBa

C A avn.kgma.kg/webtest/testing

Kymap lanneu Kg Ru

Bonpoc: N 26
A 40 years old male applied to physician with complaints for an pain in chest, orthopnea, fatigue.
Doctor did auscultation and the first heart sound is loud and palpable at the apex. After $2 the
doctor auscultated an opening snap sound. Which of the following results of Echo will be seen in
this disease

OTBETbi(ogMH OTBeT)

1 Concentric hypertrophy

2 Narrowing of aortic valve

3 Narrowing of mitral valve

Calcification of aortic valvea

5 Dilated aorta

22 23 24 25 26 27 28 29 30

50

1
2 3 4 5 6 7 89 10
11 12 13 14 15 16 17 18 19 20

21 22 23 24 25 26 27 28 29 30
31 32 33 34 35 36 37 38 39 40
41 42 43 44 45 46 47 48 49 50

OTBeHeHHblM BOnpocC

nponyueHHblM BOnpoc
Tekyuwn Bonpoc

Ao sabepweHWA TeCTa oCTanocb.

1:13:18
Scanned with CamScanner
Be6 TectwpOBa X h Ba3 hapM3 Be6TeCTpOBal X +

CA avn.kgma.kg/welbtest/testing

KyMap anneu Kg Ru

Bonpoc: N 27
62years old male complains of retrosternal chest pain, radiating to the left arm, jaw. Pain
develops after 200-300 meters of walking or at rest and lasts 3-5 minutes. BP -130/70 mm Hg.
HR-82 beats per min. Which diagnostic method is necessary to confirm diagnosis?

OTBeTbi(oAWH OTBeT)

1 24-hourblood pressure monitor

2 24-hour ECG monitor

3 Stress ECGtest

4 Chest X-ray

5 Coronary angiography

23 24 25
2627 28 29 30 31

50

1
2 3 4 567 8 109
11 12 13 14 15 16 17 18 19 20
21 22 23 24 25 26 27 28 29 30
31 32 33 34 35 36 37 38 39 40
41 42 43 44 45 46 47 48 49 50

OTBeYeHHbIM BOnpoc
nponyujeHHblM BOnpoc
TekyuMn Bonpoc

Ao 3aBepweHna TeCTa oCTanocb

1:13:14
Scanned with CamScanner
SAMSUNG

94% 0 9.01 am
Be6TecTIHpoBa X i Bas papM3 Be6TeCTMpOBaX

f C A avn.kgma.kg/webtest/testing

KyMap Wawneu K Ru

Bonpoc: N 28
Select, whereby enhanced apical impulse in children:

OTBETbi(ogMH OTBeT)

hypertrophy ofright ventricle

leftatrial hypertrophy
3 hypertrophy of the right atrium

4 heartfailure
5 hypertrophy of the left ventricle

24 2526 27 28 29 30 31 32 **

50

1 23 4 5 67 89 10
11 12 13 14 15 16 17 18 19 20
21 22 23 24 25 26 27 28 29 30

31 32 33 34 35 36 37 38 39 40
41 42 43 44 45 46 47 48 49 50

OTBeYeHHbIM BOnpoc
ponyuyeHHbI BOnpoc
Tekyuw BOnpoc

Ao 3aBepweHwa TecTa oCTanocb

1:13:06
3aBepwwTb TeCT

Scanned with CamScanner


SAMSUNG

94% 94% 9:01 am

Be6TecTHpOBa X h Ba3 dapM3 Be6TecTMpOBaX

C A avn.kgma.kg/webtest/testing

KyMap Llawneus Kg Ru

Bonpoc: N°29
Expansion of the left border of the heart is observed in:

OTBeTbI(oAMH OTBeT)

right ventricular hypertrophy

2 right atrial hypertrophy

3 left atrial hypertrophy


stenosis of the mouth of the pulmonary artery

5 left ventricular hypertrophy

25 26 27 28 29 30 31 32 33 ***

50

1 2 3 4 5 6 7 89 10
11 12 13 14 15 16 17 18 19 20
21 22 23 24 25 26 27 28 29 30

31 32 33 34 35 36 37 38 39 40
41 42 43 44 45 46 47 48 49 50

OTBeveHHblM BOnpoc
nponyueHHbl Bonpoc
Tekyuwn Bonpoc

Ao 3aBepueHWA TecTa oCTanoCb

1:13:01
3aBepwwTb TeCT

Scanned with CamScanner


Be6TecrpoBa X baa dapM3 Be6TeCTMpoBa X

C A
avn.kgma.kg/webtest/testing

KyMap lWawneu Kg Ru

Bonpoc: N 30
The Shalkov Test is considered positive (adequate) when:

OTBETbi(oAMH OTBeT)

all indicators are restored in 5 minutes

all indicators are restored in 10 minutes

3 all indicators are restored in 20 minutes

4 all indicators are restored in 15 minutes

5 all indicators are restored in 3 minutes

26 2728 29 30 3132 33 34

50

1
23 456 7
89 10
11 12 13 14 15 16 17 18 19 20
21 22 23 24 25 26 27 28 29 30
31 32 33 34 35 36 37 38 39 40
41 42 43 44 45 46 47 48 49 50
OTBeHeHHbiMBOnpoc
nponyuyeHHbIM BOnpoc
Tekyuwn BOnpoc

o 3aBepweHnA TeCTa oCTanocb


1:12:57
3aBepuWTb TeCT

Scanned with Camscanner


Bonpoc: Ne31
What is the difference between "pericardial friction noise" and endocardial noise in children:

OTBETbI(oAMH OTBeT

1 coincides with heart tones

radiation in the back and side is not characteristic

increases when the stethoscope is pressed

4 does not change when the patient changes position

5 disappears whenthe stethoscope is pressed

27 28 29 30 31 32 33 34 35

50

1
2 3 45678 9 10
11 12 13 14 15 16 17 18 19 20
21 22 23 24 25 26 27 28 29 30

31 32 33 34 35 36 37 38 39 40
41 42 43 44 45 46 47 48 49 50

OTBeeHHbiM BOnpoc
nponyujeHHbIM BOnpoc
Texyuni Bonpoc

Ao saBepweHnA TeCTa oCTanocb

1:12:53
3aBepuuWTb TeCT

Scanned with CamScanner


Be6 TeCTHpOBa X h baa dapM3 Be6TeCTMpOBa

A avn.kgma.kg/webtest/testing9
A

f C
KyMap Wannew Kg Ru

Bonpoc: N932
What is the formation of the apical shock in a newborn

OTBeTbi(ogMH OTBeT)

1 right ventricle
2 left atrium
3 left and right ventricles

4 left ventricle
5 rightatrium

28 29 30 31 32 33 34 35 36

50

1
2 3 A
567 89 10
11 12 13 14 15 16 17 18 19 20
21 22 23 24 25 26 27 28 29 30

31 32 33 34 35 36 37 38 39 40
41 42 43 44 45 46 47 48 49 50

OTBeYeHHbl BOnpOC
nponyueHHbl Bonpoc
TeKyun Bonpoc

Ao 3aBepweHnA TecTa oCTanoCb

1:12:46
3aBepuwTb TeCT

Scanned with Camscanner


Bes TecTpoBa X bas dapM3 Be6TeCTMpOBa

A avn.kgma.kg/webtest/testing

Kymap Wawnew Ru

Bonpoc: N33
Describe the pulse tension in congenital aortic stenosis noted

OTBeTbi(oAMH OTBeT)

1 thread-like pulse

hard pulse

large pulse

4 paradoxical pulse

5 alternating pulse

29 30 31 32 33 34 35 36 37

50

1 23 4 5678 9 10
11 12 13 14 15 16 17 18 19 20
21 22 23 24 25 26 27 28 29 30
31 32 33 34 35 36 37 38 39 40
41 42 43 44 45 46 47 48 49 50

OTBeYeHHbI BOnpOc
niponyueHHbIM BOnpoc
Tekyuwn BOnpoc

Ao 3aBepweHWA TeCTa oCTanocb

1:12:41
3aBepuiTb TeCT

Scanned with CamScanner


SAMSUNG

94% 09:02 am
Be6TectTpoBa X h Bas tpapM3 Be6TecTMpOBa X

C Aavn.kgma.kg/webtest/testing

KyMap Wawneu Kg Ru
Bonpoc: N934
Edema of cardiac origin in children initially appears on:

OTBeTbi(oAMH OTBET)
1
feet
abdomen

face

anterior surface of the shins

chest
30 31
32 3334 35 36 3738

50

12
11
3 4 5 67 89 10
12 13 14 15 16 17 18 19 20
21 22 23 24 25 26 27 28 29 30

31 32 33 34 35 36 37 38 39 40
41 42 43 44 45 46 47 48 49 50

OTBeyeHHblM BOnpoc
nponyuyeHHbIM BOnpoc
TekyuMM BOnpoc

Ao saBepweHwA TeCTa oCTanocb

1:12:37
3aBepuuWTb TeCT

Scanned with CamScanner


Be6 TecTpoBa X iBaadhapM3 Be6TeCTMpoBa
A avn.kgma.kg/webtest/testing

KyMap Wawnew Ru

Bonpoc: N°35
Evaluate the pulse in aortic stenosis:

OTBeTbi(oguH OTBeT)
1 high fast pulse

small, weak filling and tension

3 positive venous pulse


4 it is not palpable on the back of the foot

5 the radial artery is tense

31 3233 34 35 36 37 38 39

50

1
2 3 456789 10
11 12 13 14 15 16 17 18 19 20
21 22 23 24 25 26 27 28 29 30
31 32 33 3435 36 37 38 39 40
41 42 43 44 45 46 47 48 49 50

OTBeeHHbiM BOnpoc
nponyujeHHbiM BOnpoc
Texyuwn Bonpoc

Ao 3aBepweHwA TeCTa oCTanocb

1:12:32
3aBepunTb TeCT

Scanned wíth CamScanner


SAMSUNG

94% 09.02 am
Be6TecvpoBar X baa dapM3 Be6TecTmpona

A avn.kgma.kg/webtest/testing

KyMap Wawneu Kg Ru

Bonpoc: N936
The Child is 11 months behind in physical development, there is cyanosis of the nasolabial
triangle and shortness of breath. A heart hump formed. With percussion, the borders of the heart
areexpanded across. Auscultative listening to systolic-diastolic noise 'machine type". According
to the patient's description, what kind of congenital heart disease do you assume?

OTBeTbI(ogMH OTBeT)

atrial septal defect

2 pulmonary artery stenosis

3 open ductus arteriosus

4 aorticstenosis
5 Vventricular septal defect

32 33 34 35 36 37 38
38 39
39 40
A0

50

1
2 3 45 67 8 9.10
11 12 13 14 15 16 17 18 19 20
21 22 23 24 25 26 27 28 29 30
3132 33 34 35 36 37 38 39 40
41 42 43 44 45 46 47 48 49 50

OTBeYeHHblM BOnpoc
nponyueHHbiM BOnpoc
TeKyuni Bonpoc

Ao 3aBepweHWA TecTa oCTanocCb.

1:12:27
Scannead with CamScanner
Be6 TeCTHpOBa h Bas papM3
X Be6TeCTMpoBar +
O C A avn.kgma.kg/webtest/testing

KyMap Wawneu Kg Ru

Bonpoc: N 37
The Child is 1 year old. Complaints from the mother: weakness, fatigue. Objectively:
Physical
development lag. The skin is pale, cyanosis of the nasolabial triangle. During auscultation,
abnormal heart murmurs are heard on the right side of the sternum. What additional
instrumental
method should be prescribed to clarify the diagnosis?

OTBeTbl(OAMH OTBeT

MRI

2 EC

3 ECH0-CG

4 ECHO-EG

5 CT

33 34 35 36 37 38 39 40 41

50

12 3 4 5 6 7 8
9 10
11 12 13 14 15 16 17 18 19
20
21 22 23 24 25 26 27 28 29 30
31 32 33 34 35 36 37 38 39 40
41 42 43 44 45 46 47 48 49 50

OTBeyeHHblM BOnpoc
nponyueHHbi BOnpoc
Tekyuw Bonpoc

Ao 3aBepweHna TeCTa oCTanocb

1:12:24
Scanned with CamScanner
SAMSUNG

l 94%09:02 am
Be6TecTpOBar X Saa dapM3 Be6Tecrmposa +

C A avn.kgma.kg/webtest/testing

KyMap lllannew Kg Ru
Bonpoc: N938
What is the main source of energy in the heart

OTBETbi(oAMH OTBeT)

Sucrose

Fatty acid

Ketone
bodies

Aminoacids

5 Glucose

34 35 36 37 38 39 40 41 42

50

12 3 4 56 7
89 10
11 12 13 14 15 16 17 18 19 20
21 22 23 24 25 26 27 28 29 30
31 32 33 34 35 36 37 38 39 40

41 42 43 44 45 46 47 48 49 50

OTBeYeHHbiM BOnpOC
nponyueHHbiM BOnpoc
TeKyynn Bonpoc

Ao 3aBepuweHna TeCTa oCTanocb

1:12:19
3aBepuTb TeCT

Scanned with CamScanner


SAMSUNG

94%0902 am

Be6TecTpOBO X hBaa dapM3 Be6TecTpoBa X

C A avn.kgma.kg/webtest/testing

KyMap llawneus Kg Ru

Bonpoc: N°39
The etiological factors of primary arterial hypertension can be:

OTBETbl(oAMH OTBeT)

Renal artery atherosclerosis

Hyperplasia of the adrenal cortex

Frequent psycho-emotional overstrain

Insufficient production of glucocorticoids

5 Decreased blood flow to the kidneys

35 36 37 38
39 40 41 42 43

50

1 2 3 4 5 67 8 9 10
11 12 13 14 15 16 17 18 19 20
21 22 23 24 25 26 27 28 29 30
31 32 33 34 35 36 37 38 39 40

41 42 43 44 45 46 47 48 49 50

OTBeeHHbIM BOnpoc
nponyueHHblM BOnpoc
TeKyuni Bonpoc

Ao 3aBepweHwa TeCTa oCTanocb

1:12:16
3aBepunTb TeCT

Scanned with CamScanner


SAMSUNG

94% 09.02 am
Be6TecTvpoeaX Baa dapm3 Be6TecTmpoRa

C A avn.kgma.kg/webtest/testing

KyMap Wawnew Kg Ru

Bonpoc: Ne40
What blood pressure values indicate the presence of hypertension

OTBeTbi(oAMH OTBeT

1 170/110

100/60

120/80

105/60

110/70
1 36 37 38 39 40 41 42 43 44

50

12 3 4 567 89 10
11 12 13 14 15 16 17 18 19 20
21 22 23 24 25 26 27 28 29 30
31 32 33 34 35 36 37 38 39 40
41 42 43 44 45 46 47 48 49 50

OTBeYeHHblM BONnpoc
nponyujewHbI BOnpoc
TeKyuw BOnpoc

Ao 3aBepweHMA TecTa ocTanocb

1:12:11
3aBepuiwTb TeCT

Scanned with CamScanner


SAMSUNG

94% 09.02 am
Be6 TecrpoBar X h Baa dapM3 Be6TecTpoBa X +

A avn.kgma.kg/webtest/testing
C
KyMap lawnew Kg Ru

Bonpoc: N941
with left ventricular heart failure, there is

OTBETbI(oAH OTBeT)

Neckvein pulsation
Swelling in the lower extremities

3 Pulmonary edema

Ascites

5 Cirhosis of the liver

37 38 39 40 41 42 43 44 45

50

12 3 4 5 678 9 10
11 12 13 14 15 16 17 18 19 20

21 22 23 24 25 26 27 28 29 30
31 32 33 34 35 36 37 38 39 40
42 43 44 45 46 47 48 49 50
-OTBeYeHHbin Bonpoc
nponyueHHblM BOnpoc
TekyunM BOnpoc

Ao 3aBepweHwa TeCTa oCTanocb

1:12:08
3aBepuiWTb TeCT

Scanned with CamScanner


SAMSUNG

94% 9:02 am
Be6TecTvpoBar: X i Baa hapM3 Be6TecTpoBa

O C A avn.kgma.kg/webtest/testing

KyMap LWawnew Kg Ru
Bonpoc: N 42
The re-entry mechanism is

OTBeTbI(oAMH OTBeT)

Re-entry of the impulse to the place of its origin

An increase in the number of mitochondria in cardiomyocytes

Overload of cardiomyocytes with calcium

Excess energy and substrates in cardiomyocytes

5 Ativation of antioxidant systems in cardiomyocytes

38 3940 41 42 43 44 45 46

50

1 2 3 4 5 67 8 9 10
11 12 13 14 15 16 17 18 19 20
21 22 23 24 25 26 27 28 29 30
31 32 33 34 35 36 37 38 39 40
41 42 43 44 45 46 47 48 49 50
OTBeYeHHbI BOnpoc
nponyueHHb1 BOnpoc
Tekyun BOnpoc

Ao 3asepweHua TeCTa oCTanocb

1:12:04
3aBepwWTb TeCT

Scanned with CamScanner


Be6TeCTHpoBa X Baa dapM3 Be6TecTMpoBa

O C A avn.kgma.kg/webtest/testing
KyMap llWawneu Ko Ru
Bonpoc: N°43
Definition of Kerley lines on chest radiography?

OTBeTbi(oAMH OTBeT)

dilated pulmonary arteries

interiobar pleural thickening

pleuro-phrenic fibrosis

4 intralobular septal thicken ing

5 calcification of valves

39 40 41
4243 4 45 46 47

50

12 3 456 7 89 10
11 12 13 14 15 16 17 18 19 20
21 22 23 24 25 26 27 28 29 30
31 32 33 34 35 36 37 38 39 40
41 42 43 44 45 46 47 48 49 50

OTBeYeHHblM
BOnpoc
nponyuyeHHbIM BOnpoc
TeKyun Bonpoc

Lo 3aBepuweHna TeCTa oCTanocb

1:12:00
3aBepuwTb TeCT

Scanned with CamScannher


SAMSUNG

94%d94%09.03 am
Be6TectpoBa X Baa hapM3 Be6TectvpoBa +

A avn.kgma.kg/webtest/testingg

KyMap lllawneu Kg Ru

Bonpoc: N°44
What may be relative contraindication in performing transthoracic echocardiography?

OTBeTbi(ogMH OTBeT)

pregnancy

enlarged fat pad

intercostal space narrowing

calcification of valves

pericardial effusion

40 4142 43 44 45 46 47 48

50

1 2 3 4 567 8 9 10
11 12 13 14 15 16 17 18 19 20
21 22 23 24 25 26 27 28 29 30
31 32 33 34 35 36 37 38 39 40
4142 43 44 45 46 47 48 49 50

OTBeveHHbiM BOnpoc
nponyueHHblM BOmpoc
Tekyulwn BOnpoc

Ao 3aBepuweHWA TecTa oCTanocb

1:11:57
3aBepuWTb TeCT

Scanned with CamScanner


Bonpoe: 4S

OrseTog oTBer)

1:11:54

Scanned with CamScanner


SAMSUNG

94% 09.03 am

Be6TecTHpoBa i Ba3 dhapM3 Be6TecTpoBa X

C A avn.kgma.kg/webtest/testing

KyMap llawneuw Kg Ru

Bonpoc: Ne45
a contrast
tf both the left atrium and aorta are contrasted itin systole of left ventricle, when
medium is administered into the left ventricle, is?

OTBeTbI(ogMH OTBeT

ventricular septum defect

mitral regurgitation

3 leftventricle diverticulum
tricuspid regurgitation

5 aorticaneurism

41 42 43 44 45 46 47 4849 50

1
23 4 5678 9 10
11 12 13 14 15 16 17 18 19 20
21 22 23 24 25 26 27 28 29 30
31 32 33 34 35 36 37 38 39 40
41 42 43 44 45 46 47 48 49 50
OTBeEHHbiM BOnpoc
nponyuyeHHbl BOnpoc
Texyuwn Bonpoc

Ao 3aBepweHMA TecTa oCTanocb

1:11:51
3aBepuTb TeCT

Scanned with CamScanner


Be6 Tectpoea X i Baa dapM3 Be6TecCTWpoBa

CA avn.kgma.kg/webtest/testing

KyMap Wanneu 9 Ru

Bonpoc: NP46
What vessel is projected onto the anterior chest wall in the area of the right sternum edge and
right costal cartilage from the 1st to the 3rd?

OTBETbI(oAMH OTBeT)

superiorvena cava

brachiocephalic trunk

3 aortic arch

4 pulmonarytrunk

5 ascending aorta

42 4344 45 46 47 48
49 50

12 3 4 5678 9 10
11 12 13 14 15 16 17 18 19 20
21 22 23 24 25 26 27 28 29 30
31 32 33 34 35 36 37 38 39 40

41 42 43 44 45 46 47 48 49 50
OTBeyeHHblM BOnpoc

nponyueHHbiM BOnpoc
TeKyuiwn Bonpoc

Ao 3aBepweHMa TeCTa oCTanoCb

1:11:47
3aBepuTb TeCT

Scanned with CamScanner


Be6 TeCTApOBa X baa dhapM3 Be6TecTHpoBa

C A avn.kgma.kg/webtest/testing

KyMap lWawneuu Kg Ru

Bonpoc: Ne47
Which vessel is projected at the level of attachment of the 3rd costal
cartilage to the sternum on
the left?

OTBeTbI(oAMH OTBeT)

pulmonary trunk

ascending aorta

3 descending aorta

brachiocephalic trunk

aortic arch

43 44 45 46 4748 49 50

12 3 45678910
11 12 13 14 15 16 17 18 19 20D

21 22 23 24 25 26 27 28 29 30
31 32 33 34 35 36 37 38 39 40
41 42 43 44 45 46 47 48 49 50

OTBeYeHHbiM BOnpoc

nponyuyeHHblM BOnpoc
Texyuwn Bonpoc

o 3aBepweHMA TeCTa oCTanoCb:

1:11:43
3aBepwWTb TeCT

Scanned with Camscanner


Be6TecTpoBa X hBaa dapM3 Be6TeCTMpOBa X

O C A avn.kgma.kg/webtest/testing

KyMapllawneu Kg Ru

Bonpoc: N°48
Thefemoral artery and vein are projected along a line connecting a point on the border between:

OTBeTbi(oAMH OTBeT)

middle and medial third inguinal folds with the posterior edge of the lateral condyle of
the femur

middle and medial third inguinal folds with the posterior edge of the medial condyleof
the tibia

3 middle and medial third inguinal folds with the posterior edge of the medial condyle of
the femur

middle and medial third inguinal folds with the posterior edge of the medial condyle of
the fibula

5 middle and medial third inguinal folds with the posterior edge of the lateral condyle of
the tibia

43 4445 46 47 48 49 50

1 2 3 4 567 89 10
11 12 13 14 15 16 17 18 19 20
21 22 23 24 25 26 27 28 29 30
31 32 33 34 35 36 37 38 39 40
41 42 43 44 45 46 47 48 49 50

OTBeYeHHblM BOnpoc
nponyuyeHHbI BOnpoc
TeKyuwi Bonpoc

Ao 3aBepuweHnA TeCTa ocTanocb

1:11:39
Scanned with CamScanner
SAMSUNG
94%,ll94%09.03 am
Be6TectpoBa 6a3 dhapM3 Be6TecTpoBa +

C A avn.kgma.kg/webtest/testing

KyMap lWawneu Kg Ru

Bonpoc: N949
A patient was admitted to the emergency department at the age of 60, an electrocardiogram
diagnosed an infarction of the anterior wall of the left ventricle of the heart. Justify the possible
level of impaired blood flow through the coronary vessels?

OTBeTbi(ogMH OTBeT)

flexor artery

2 posterior intergastric artery

right coronary artery

left coronary artery

5 coronal sinuS

43 44 45 46 47 48 49 50

1 2 3 4 5 6 7 89 10
11 12 13 14 15 16 17 18 19 20
21 22 23 24 25 26 27 28 29 30
31 32 33 34 35 36 37 38 39 40
41 42 43 44 45 46 47 48 49 50

OTBeveHHbIM BOnpoc
nponyuyeHHbIM BOnpoc
Texyuni BOnpoc

o 3aBepweHwa TecCTa oCTanoCb

1:11:34
3aBepunTb TeCT

Scanned with CamScanner


Be6 TecTHpoBarX hBaa dapM3 Be6TeCTMpoBaX

C A avn.kgma.kg/webtest/testing

KyMap llWawneu 9 Ru

Bonpoc: N50
A 70-year-old patient with esophageal bleeding was admitted to the admission unit, the abdomen
is enlarged, in the abdomen there is a jellyfish head" (dilated superficial veins of the chest and
abdomen). Explain, if the blood flow of which vessel is disturbed, the above clinical
manifestations are observed?

OTBETbI(oguH OTBeT)

aortas

2 portal vein

3 intrathoracicartery

subclavian artery

5 brachiocephalic veins

43 44 45 46 47 48 49
50

2 3 4567 8 9 10
11 12 13 14 15 16 17 18 19 20
21 22 23 24 25 26 27 28 29 30
31 32 33 34 35 36 37 38 39 40
41 42 43 44 45 46 47 48 49 50

OTBeHeHHbl Bonpoc
nponyuyeHHbIM BOnpoc
T TeKyuwn Bonpoc

Ao 3aBepweHnA TeCTa oCTanocb:

1:11:29
3asepuiWTb TeCT
Scanned with CamScanner

You might also like